You are on page 1of 468

lOMoARcPSD|35346190

Geri - Test bank 8th edition

Geriatric Nursing (Hondros College of Nursing)

Studocu is not sponsored or endorsed by any college or university


Downloaded by Dallen Mae D. Kadir (kadir.dallenmae.d.bcsi@gmail.com)
lOMoARcPSD|35346190

Prof.Exams

TEST BANK

2022

Downloaded by Dallen Mae D. Kadir (kadir.dallenmae.d.bcsi@gmail.com)


lOMoARcPSD|35346190

Prof.Exams

COMPLETE
Elaborated Test Bank
for
Basic Geriatric Nursing
- 8Ed.
by
Patricia A. Williams.
2022

Downloaded by Dallen Mae D. Kadir (kadir.dallenmae.d.bcsi@gmail.com)


lOMoARcPSD|35346190

Prof.Exams

Basic Geriatric Nursing - 8Ed. by Patricia A. Williams.

ISBN-10 0323828906
ISBN-13 978-0323828901

Test bank for Basic Geriatric Nursing - 8Ed. by Patricia A. Williams

ISBN-10 0323828906
ISBN-13 978-0323828901

THIS IS NOT THE ACTUAL BOOK. YOU ARE BUYING it is the Official Test Bank
in e-version INSTANT DOWNLOAD

>The test bank is what most professors use as a template when making exams for their students, which means
there’s a very high chance that you will see the exact questions in the tests!
>The file is either in.pdf, excel, or zipped in the package and can easily be read on PCs and Macs.
>Delivery is INSTANT. You can download the files IMMEDIATELY once payment is done.
If you have any questions, please feel free to contact us. Our response is the fastest. All questions will alwaysbe
answered in 6 hours, most of the time within 30mins.

We also faced similar difficulties when we were students, and we understand how you feel. But now, with the
Official Test Bank, you will be able to

* Anticipate the type of the questions that will appear in your exam.
* Know the correct answers to those questions.
* Reduce the hassle and stress of your student life.
* Improve your studying and get a better grade!
* Get prepared for examination questions.
* Save you time and help you understand the material.

Downloaded by Dallen Mae D. Kadir (kadir.dallenmae.d.bcsi@gmail.com)


lOMoARcPSD|35346190

Prof.Exams
TABLE OF CONTENTS
UNIT I OVERVIEW OF AGING
1 Trends and Issues
2 Theories of Aging
3 Physiologic Changes
UNIT II BASIC SKILLS FOR GERONTOLOGIC NURSING
4 Health Promotion, Health Maintenance, and Home Health Considerations
5 Communicating With Older Adults
6 Maintaining Fluid Balance and Meeting Nutritional Needs
7 Medications and Older Adults
8 Health Assessment for Older Adults
9 Meeting Safety Needs of Older Adults
UNIT III PSYCHOSOCIAL CARE OF THE ELDERLY
10 Cognition and Perception
11 Self-Perception and Self-Concept
12 Roles and Relationships
13 Coping and Stress
14 Values and Beliefs
15 End-of-Life Care
16 Sexuality and Aging
UNIT IV PHYSICAL CARE OF THE ELDERLY
17 Care of Aging Ski
18 Elimination
19 Activity and Exercise
20 Sleep and Rest

Downloaded by Dallen Mae D. Kadir (kadir.dallenmae.d.bcsi@gmail.com)


lOMoARcPSD|35346190

Prof.Exams

To download the full document, Kindly clink on the links below:


Link I:

Link II:
Should you face any challenges, do not hesitate to send an email to: sturdydrone2707@gmail.com
We respond and resolve to all queries within 6 hours.

Downloaded by Dallen Mae D. Kadir (kadir.dallenmae.d.bcsi@gmail.com)


lOMoARcPSD|35346190

Chapter 01: Nursing and the Health Care System


Williams: deWit's Fundamental Concepts and Skills for Nursing, 8th Edition

MULTIPLE CHOICE

1. Florence Nightingale‘s contributions to nursing practice and education:


a. are historically important but have no validity for nursing today.
b. were neither recognized nor appreciated in her own time.
c. were a major factor in reducing the death rate in the Crimean War.
d. were limited only to the care of severe traumatic wounds.
ANS: C
By improving sanitation, nutrition ventilation, and handwashing techniques, Florence
Nightingale‘s nurses dramatically reduced the death rate from injuries in the Crimean War.

DIF: Cognitive Level: Knowledge REF: p. 2 OBJ: Theory #1


TOP: Nursing History KEY: Nursing Process Step: N/A
MSC: NCLEX: N/A

2. Early nursing education and care in the United States:


a. were directed at community health.
b. provided independence for women through education and employment.
c. were an educational model based in institutions of higher learning.
d. have continued to be entirely focused on hospital nursing.
ANS: B
Because of the influence of early nNuUr Rs iSnI gN Gl eTaBd.eCrOs ,Mn u r s i n g education became more formalized
through apprenticeships in Nightingale schools that offered independence to women through
education and employment.

DIF: Cognitive Level: Knowledge REF: p. 2 OBJ: Theory #4


TOP: Nursing History KEY: Nursing Process Step: N/A
MSC: NCLEX: N/A

3. In order to fulfill the common goals defined by nursing theorists (promote wellness, prevent
illness, facilitate coping, and restore health), the LPN must take on the roles of:
a. caregiver, educator, and collaborator.
b. nursing assistant, delegator, and environmental specialist.
c. medication dispenser, collaborator, and transporter.
d. dietitian, manager, and housekeeper.

ANS: A
In order for the LPN to apply the common goals of nursing, he or she must assume the roles
of caregiver, educator, collaborator, manager, and advocate.

DIF: Cognitive Level: Comprehension REF: p. 3 OBJ: Theory #2


TOP: Art and Science of Nursing KEY: Nursing Process Step: N/A
MSC: NCLEX: N/A

4. Although nursing theories differ in their attempts to define nursing, all of them base their
beliefs on common concepts concerning:

NURSINGTB.COM

Downloaded by Dallen Mae D. Kadir (kadir.dallenmae.d.bcsi@gmail.com)


lOMoARcPSD|35346190

a. self-actualization, fundamental needs, and belonging.


b. stress reduction, self-care, and a systems model.
c. curative care, restorative care, and terminal care.
d. human relationships, the environment, and health.
ANS: D
Although nursing theories differ, they all base their beliefs on human relationships, the
environment, and health.

DIF: Cognitive Level: Comprehension REF: p. 4 OBJ: Theory #2


TOP: Nursing Theories KEY: Nursing Process Step: N/A
MSC: NCLEX: N/A

5. Standards of care for the nursing practice of the LPN are established by the:
a. Boards of Nursing Examiners in each state.
b. National Council of States Boards of Nursing (NCSBN).
c. American Nurses Association (ANA).
d. National Federation of Licensed Practical Nurses.

ANS: D
The National Federation of Licensed Practical Nurses modified the standards published by the
ANA in 2015 to better fit the role of the LPN. In 2015 the American Nurses Association
(ANA) revised the Standards of Nursing Practice which contained 17 standards of national
practice of nursing, describing all facets of nursing practice: who, what, when, where, how.

DIF: Cognitive Level: Comprehension REF: p. 6 OBJ: Theory #2


TOP: Standards of Care KEY: Nursing Process Step: N/A
MSC: NCLEX: N/A NURSINGTB.COM

6. The LPN demonstrates an evidence-based practice by:


a. using a drug manual to check compatibility of drugs.
b. using scientific information to guide decision making.
c. using medical history of a patient to direct nursing interventions.
d. basing nursing care on advice from an experienced nurse.
ANS: B
The use of scientific information from high-quality research to guide nursing decisions is
reflective of the application of evidence-based practice.

DIF: Cognitive Level: Knowledge REF: p. 7 OBJ: Theory #3


TOP: Evidence-Based Practice KEY: Nursing Process Step: N/A
MSC: NCLEX: N/A

7. Lillian Wald and Mary Brewster established the Henry Street Settlement Service in New York
in 1893 in order to:
a. offer a shelter to injured war veterans.
b. found a nursing apprenticeship.
c. provide health care to poor persons living in tenements.
d. offer better housing to low-income families.

ANS: C

Downloaded by: SUCCEEDGRADES | abbieclin@gmail.com


Distribution
Downloaded by Dallen Mae D.ofKadir
this document is illegal
(kadir.dallenmae.d.bcsi@gmail.com)
lOMoARcPSD|35346190

Henry Street Settlement Service brought the provision of community health care to the poor
people living in tenements.

DIF: Cognitive Level: Comprehension REF: p. 2 OBJ: Theory #4


TOP: Growth of Nursing KEY: Nursing Process Step: N/A
MSC: NCLEX: N/A

8. An educational pathway for an LPN/LVN refers to an LPN/LVN:


a. learning on the job and being promoted to a higher level of responsibility.
b. moving from a maternity unit to a more complicated surgical unit.
c. obtaining additional education to move from one level of nursing to another.
d. learning that advancement requires consistent work and commitment.

ANS: C
By broadening the educational base, an LPN/LVN may advance and build a nursing career.

DIF: Cognitive Level: Knowledge REF: p. 7 OBJ: Theory #7


TOP: Nursing Education Pathways KEY: Nursing Process Step: N/A
MSC: NCLEX: N/A

9. When diagnosis-related groups (DRGs) were established by Medicare in 1983, the purpose
was to:
a. put patients with the same diagnosis on the same unit.
b. attempt to contain the costs of health care.
c. increase the availability of medical care to older adults.
d. identify a patient‘s condition more quickly.
ANS: B NURSINGTB.COM
The purpose of instituting DRGs was to contain skyrocketing costs of health care.

DIF: Cognitive Level: Knowledge REF: p. 9 OBJ: Theory #10


TOP: Health Care Delivery KEY: Nursing Process Step: N/A
MSC: NCLEX: N/A

10. The advent of diagnosis-related groups (DRGs) required that nurses working in health care
agencies:
a. record supportive documentation to confirm a patient‘s need for care in order to
qualify for reimbursement.
b. use the DRG rather than their own observations for patient assessment.
c. be aware of the specific drugs related to the diagnosis.
d. acquire cross-training to make staffing more flexible.

ANS: A
DRGs required that nurses provide more supportive documentation of their assessments and
identified patient‘s needs to qualify the facility for Medicare reimbursement. Observant
assessment might also indicate another DRG classification and consequently more
reimbursement for the facility.

DIF: Cognitive Level: Comprehension REF: p. 10 OBJ: Theory #10


TOP: Managed Care KEY: Nursing Process Step: N/A
MSC: NCLEX: N/A

Downloaded by Dallen Mae D. Kadir (kadir.dallenmae.d.bcsi@gmail.com)


lOMoARcPSD|35346190

11. If a member of a health maintenance organization (HMO) is having respiratory problems such
as fever, cough, and fatigue for several days and wants to see a specialist, the person is
required to go:
a. directly to an emergency room for treatment.
b. to any general practitioner of choice.
c. directly to a respiratory specialist.
d. to a primary care provider for a referral.

ANS: D
Participants in an HMO must see their primary provider to receive a referral for a specialist in
order for the HMO to pay for the care.

DIF: Cognitive Level: Comprehension REF: p. 10 OBJ: Theory #11


TOP: Managed Care KEY: Nursing Process Step: N/A
MSC: NCLEX: N/A

12. An advantage of preferred provider organizations (PPOs) is that:


a. they make insurance coverage of employees less expensive to employers.
b. there are fewer physicians to choose from than in an HMO.
c. long-term relationships with physicians are more likely.
d. patients may go directly to a specialist for care.

ANS: A
The use of PPOs allows insurance companies to keep their premiums low and in turn makes
insurance coverage less expensive for the employers. There are usually more physicians from
which to choose than from an HMO, but long-term relationships between physician and
patient cannot be established easily. Patients still must see their primary physician before
being referred to other specialties.NURSINGTB.COM

DIF: Cognitive Level: Knowledge REF: p. 11 OBJ: Theory #11


TOP: Preferred Provider Organizations KEY: Nursing Process Step: N/A
MSC: NCLEX: N/A

13. After passing the National Council Licensure Examination for Practical Nurses (NCLEX PN),
the nurse is qualified to take an additional certification in the field of:
a. pharmacology.
b. care of infants and children.
c. operating room technology.
d. community health.

ANS: A
After becoming an LPN, the nurse may apply for additional certification in pharmacology or
long-term care.

DIF: Cognitive Level: Knowledge REF: p. 7 OBJ: Theory #6


TOP: Educational Opportunities KEY: Nursing Process Step: N/A
MSC: NCLEX: N/A

14. Nursing interventions are best defined as activities that:


a. are taken to improve the patient‘s health.
b. involve researching methods to maintain asepsis.
c. include the family in nursing care.

Downloaded by Dallen Mae D. Kadir (kadir.dallenmae.d.bcsi@gmail.com)


lOMoARcPSD|35346190

d. review guidelines for handling infectious wastes.

ANS: A
Interventions are actions taken to improve, maintain, or restore health.

DIF: Cognitive Level: Comprehension REF: p. 3 OBJ: Theory #2


TOP: Art and Science of Nursing KEY: Nursing Process Step: Planning
MSC: NCLEX: Health Promotion and Maintenance: Prevention and Detection of Disease

15. Nurse Practice Acts define the legal scope of an LPN‘s practice, which are written and
enforced by:
a. American Nurses Association.
b. National Council Licensure Examiners.
c. each state.
d. each health care agency.

ANS: C
Each state writes and enforces the Nurse Practice Act, which defines the legal scope of
nursing practice.

DIF: Cognitive Level: Comprehension REF: p. 6 OBJ: Theory #3


TOP: Nurse Practice Act KEY: Nursing Process Step: N/A
MSC: NCLEX: N/A

16. Women volunteers were organized to give nursing care to the wounded soldiers during the
Civil War by:
a. Florence Nightingale.
b. Dorothea Dix. NURSINGTB.COM
c. Clara Barton.
d. Lillian Wald.

ANS: B
The Union government appointed Dorothea Dix, a social worker, to organize women
volunteers to provide nursing care for the soldiers during the Civil War.

DIF: Cognitive Level: Knowledge REF: p. 2 OBJ: Theory #4


TOP: Nursing History KEY: Nursing Process Step: N/A
MSC: NCLEX: N/A

17. The nursing theory presented by Sister Calista Roy is based on:
a. reduction of stress.
b. achievement of maximum level of wellness.
c. relief of self-care deficit.
d. adaptation modes.
ANS: D
Adaptation modes (physiological, psychological, sociological, and independence) are the
basis of the nursing theory of Sister Calista Roy.

DIF: Cognitive Level: Knowledge REF: p. 5|Table 1-1


OBJ: Theory #2 TOP: Nursing Theory KEY: Nursing Process Step: N/A
MSC: NCLEX: N/A

Downloaded by Dallen Mae D. Kadir (kadir.dallenmae.d.bcsi@gmail.com)


lOMoARcPSD|35346190

18. The founding of the Red Cross is attributed to:


a. Lillian Wald.
b. Dorothea Dix.
c. Florence Nightingale.
d. Clara Barton.
ANS: D
Clara Barton founded the Red Cross.

DIF: Cognitive Level: Knowledge REF: p. 2 OBJ: Theory #4


TOP: Nursing History KEY: Nursing Process Step: N/A
MSC: NCLEX: N/A

19. The nursing theorist whose practice framework is based on 14 fundamental needs is:
a. Dorothy Johnson.
b. Jean Watson.
c. Virginia Henderson.
d. Martha Rogers.

ANS: C
Virginia Henderson‘s nursing theory framework is based on 14 fundamental needs.

DIF: Cognitive Level: Knowledge REF: p. 5|Table 1-1


OBJ: Theory #2 TOP: Nursing Theorists KEY: Nursing Process Step: N/A
MSC: NCLEX: N/A

20. The nursing theory that uses seven behavioral subsystems in an adaptation model is:
a. Betty Neumann. NURSINGTB.COM
b. Sister Calista Roy.
c. Dorothy Johnson.
d. Patricia Benner.

ANS: C
Dorothy Johnson‘s practice framework is based on seven behavioral subsystems in an
adaptation model.

DIF: Cognitive Level: Knowledge REF: p. 5|Table 1-1


OBJ: Theory #2 TOP: Nursing Theorists KEY: Nursing Process Step: N/A
MSC: NCLEX: N/A

21. The Standards of Nursing Practice are designed to direct LPNs to:
a. advance their nursing career.
b. seek a scientific basis for their interventions.
c. deliver safe, knowledgeable care.
d. a leadership role.
ANS: C
The Standards of Nursing Practice are designed to guide the LPN to deliver safe,
knowledgeable care.

DIF: Cognitive Level: Knowledge REF: p. 6 OBJ: Theory #2


TOP: Nursing Standards KEY: Nursing Process Step: N/A
MSC: NCLEX: Safe, Effective Care Environment

Downloaded by Dallen Mae D. Kadir (kadir.dallenmae.d.bcsi@gmail.com)


lOMoARcPSD|35346190

22. A state‘s Nurse Practice Act is designed to protect the:


a. physician.
b. nurse.
c. public.
d. hospital.

ANS: C
Nurse Practice Acts are designed to protect the public.

DIF: Cognitive Level: Knowledge REF: p. 6 OBJ: Theory #5


TOP: Nurse Practice Act KEY: Nursing Process Step: N/A
MSC: NCLEX: N/A

23. It is appropriate for practical nurses to provide direct patient care to persons in a hospital
under the supervision of a:
a. medical assistant.
b. registered nurse on the unit.
c. supervising nurse who is responsible for care on several units.
d. more experienced LPN on the unit.
ANS: B
Practical nurses provide direct patient care under the direct supervision of a registered nurse,
physician, or dentist.

DIF: Cognitive Level: Knowledge REF: p. 7 OBJ: Theory #9


TOP: Scope of Practice KEY: Nursing Process Step: N/A
MSC: NCLEX: N/A NURSINGTB.COM

24. An example of tertiary health care is:


a. hospice care.
b. restorative care.
c. emergency care.
d. home health care.
ANS: A
Tertiary health care includes extended care, chronic disease management, medical homes,
in-home personal care, and hospice care.

DIF: Cognitive Level: Comprehension REF: p. 11|Box 1-2


OBJ: Theory #8 TOP: Health Care Services KEY: Nursing Process Step: N/A
MSC: NCLEX: N/A

25. Which nursing care delivery systems have some nursing schools adopted as the foundation of
their education programs?
a. Relationship-based care
b. Team nursing
c. Patient-centered care
d. Total patient care

ANS: A

Downloaded by Dallen Mae D. Kadir (kadir.dallenmae.d.bcsi@gmail.com)


lOMoARcPSD|35346190

Relationship-based care appeared in the early 2000s (Koloroutis, 2004) and emphasizes three
critical relationships: (a) the relationship between caregivers and the patients and families they
serve; (b) the caregiver‘s relationship with him- or herself; (c) the relationship among health
team members (UCLA Department of Nursing, 2015). The motivation behind
relationship-based care was to promote a cultural transformation by improving relationships to
foster care for the patient. Some schools of nursing have adopted relationship-based care as
the foundation of their nursing education curriculum.

DIF: Cognitive Level: Knowledge REF: p. 9 OBJ: Theory #8


TOP: Delivery of Nursing Care KEY: Nursing Process Step: N/A
MSC: NCLEX: N/A

26. Which nursing care delivery system has been fully embraced by the nursing community and is
identified as one of the seven QSEN competencies?
a. Relationship-based care
b. Team nursing
c. Patient-centered care
d. Total patient care
ANS: C
Patient-centered care has been described since the 1950s, but came to the forefront in 2001
when the Institute of Medicine (IOM) targeted six areas for improvement in the US health
care system, including safety, effective, patient-centered, timely, efficient, and equitable
(Cliff, 2012). Patient-centered care has been fully embraced by the nursing community, and is
identified as one of the seven QSEN competencies (QSEN.org, 2015).
DIF: Cognitive Level: Knowledge REF: p. 18 OBJ: Theory #8
N U RS I N G T B . C O M
TOP: Delivery of Nursing Care K E Y : N ursing Process Step: N/A
MSC: NCLEX: N/A

27. Which of the following is considered a positive aspect of the Affordable Care Act?
a. A 38-year-old mother is penalized on her taxes for not purchasing health
insurance.
b. A 42-year-old laborer who has chronic kidney disease is denied insurance
coverage.
c. Jamie, age 24, cannot continue insurance coverage on his parent‘s insurance since
he has graduated from college.
d. Maria, age 60, is able to obtain health insurance at a rate that is manageable on her
income.
ANS: D

Downloaded by Dallen Mae D. Kadir (kadir.dallenmae.d.bcsi@gmail.com)


lOMoARcPSD|35346190

The Patient Protection and Affordable Care Act is being phased in over several years. There
are positive and negative aspects to this act, and many people have strong opinions about it.
Since 2013 there have been insurance exchanges, along with requirements for uninsured
people to purchase health insurance. Starting in 2015 people who have failed to purchase
health insurance are being penalized on their income taxes. Provisions in the bill now prevent
denial of insurance to those with preexisting illnesses who formerly could not buy health
insurance, and young adults have been allowed to remain on their parents‘ insurance through
age 26. Starting in 2013 affluent people began paying an extra 3.8% tax on unearned income;
drug manufacturers and the insurance industry are paying large annual fees to help cover the
overall costs. Costs of the Medicare program will be contained by reducing payments to
hospitals and health care providers. As coverage under the Affordable Care Act has expanded,
the national uninsured rate has fallen from 16% to 11% of people under age 65 (people over
age 65 are generally have universal coverage by Medicare). People who have benefitted the
most from this coverage include people ages 18-34, blacks, Hispanics, and those living in
rural areas (Quealy and Sanger-Katz, 2014). It is expected that the emphasis on prevention
and coordinated care will produce a shift in nursing from the hospital to the community. There
are many controversial parts of the bill, and the country is divided about whether the bill
should be repealed and other health care legislation written. What happens in the Congress in
the coming years will determine if all parts of the legislation will remain.

DIF: Cognitive Level: Analysis REF: p. 11 OBJ: Theory #10


TOP: The Patient Protection and Affordable Care Act KEY: Nursing Process Step: N/A
MSC: NCLEX: N/A

MULTIPLE RESPONSE
NURSINGTB.COM
1. Characteristics of primary nursing include: (Select all that apply.)
a. elimination of fragmentation of care between shifts.
b. evolved in the mid-1950s.
c. planning and direction performed by one nurse.
d. ancillary workers used to increase productivity.
e. the care plan covering the entire day.
f. associate nurses taking over care and planning when the primary nurse is off duty.
ANS: A, C, D, E, F
Primary care reduces fragmentation of care between shifts. Care is planned by one nurse to
cover a 24-hour period using ancillary workers to increase the productivity. An associate
nurse may take on direction of care in the absence of the primary nurse.

DIF: Cognitive Level: Knowledge REF: p. 8 OBJ: Theory #8


TOP: Nursing Care Delivery KEY: Nursing Process Step: N/A
MSC: NCLEX: N/A

2. In 1991, the American Nurses Association (ANA) published the Standards of Nursing
Practice. These standards are designed to: (Select all that apply.)
a. set standards for safe nursing care delivery.
b. define the legal scope of practice.
c. state legal requirements for clinical practice.
d. protect the nurse, patient, and health care agency.
e. regulate the nursing profession.

Downloaded by Dallen Mae D. Kadir (kadir.dallenmae.d.bcsi@gmail.com)


lOMoARcPSD|35346190

f. define activities in which nurses may engage.


ANS: A, D, F
The Standards of Nursing Practice generally define activities in which nurses may engage, set
standards for nursing care and delivery, and thereby protect the nurse, patient, and health care
agency.

DIF: Cognitive Level: Knowledge REF: p. 6|Box 1-1 OBJ: Theory #2


TOP: Nursing Practice KEY: Nursing Process Step: N/A
MSC: NCLEX: N/A

3. An example of the role of an LPN as a delegator is: (Select all that apply.)
a. changing a patient‘s wound dressing.
b. assisting a patient to complete his or her bath.
c. assigning patient care tasks to certified nursing assistants.
d. requesting the housecleaning staff to mop the floor of a patient‘s room.
e. instructing the unit secretary to page a physician to the floor.

ANS: C, D, E
Delegation under the scope of the practice of an LPN is the assignment of a certified nursing
assistant to certain nursing care or other nonmedical staff to aspects of patient care.

DIF: Cognitive Level: Comprehension REF: p. 3 OBJ: Theory #5


TOP: Art and Science of Nursing KEY: Nursing Process Step: N/A
MSC: NCLEX: N/A

4. During the Civil War, nursing schools offered education to women both in England and in the
United States. The schools in the U
N Un iRt eS dI NSGt Ta tBe. sC O
d iMf f e r e d from those in Europe because in US
schools: (Select all that apply.)
a. students worked without pay.
b. the core curriculum was the same.
c. instruction was presented by physicians at the bedside.
d. the educational focus was on nursing care.
e. classes were held separately from the clinical experience.
ANS: A, C
In the United States, the students staffed the hospital and worked without pay. There were no
formal classes; education was achieved through work. There was no set curriculum, and
content varied depending on the type of cases present in the hospital. Instruction was done at
the bedside by the physician and therefore came from a medical viewpoint.

DIF: Cognitive Level: Comprehension REF: p. 4 OBJ: Theory #4


TOP: Early Nursing Education KEY: Nursing Process Step: N/A
MSC: NCLEX: N/A

COMPLETION

1. Preferred provider organizations (PPOs) use to finance their services and pay
the physical cost of the service.

ANS:
capitated cost

Downloaded by Dallen Mae D. Kadir (kadir.dallenmae.d.bcsi@gmail.com)


lOMoARcPSD|35346190

The capitated cost is the set fee that is paid to the network for each patient enrolled to finance
its services.

DIF: Cognitive Level: Knowledge REF: p. 11 OBJ: Theory #8


TOP: Capitated Cost KEY: Nursing Process Step: N/A
MSC: NCLEX: N/A

2. In the United States, the Young Women‘s Christian Association (YMCA) in New York
opened The School, the first practical nursing school.

ANS:
Ballard

In 1892, the YMCA opened The Ballard School, a 3-month course in practical nursing that
was the first school of practical nursing.

DIF: Cognitive Level: Knowledge REF: p. 2 OBJ: Theory #4


TOP: Ballard School KEY: Nursing Process Step: N/A
MSC: NCLEX: N/A

3. Such health services as surgical procedures, restorative care, and home health care would be
classified as care.

ANS:
secondary

Surgical procedures, restorative c aNr Ue ,RaSnI Nd GhTo Bm. Ce OhMe a l t h are part of the many services classified
as secondary care.

DIF: Cognitive Level: Comprehension REF: p. 11|Box 1-2


OBJ: Theory #10 TOP: Health Care Services KEY: Nursing Process Step: N/A
MSC: NCLEX: N/A

Downloaded by Dallen Mae D. Kadir (kadir.dallenmae.d.bcsi@gmail.com)


lOMoARcPSD|35346190

Chapter 02: Concepts of Health, Illness, Stress, and Health Promotion


Williams: deWit's Fundamental Concepts and Skills for Nursing, 8th Edition

MULTIPLE CHOICE

1. The nurse is aware that any description of health would include the concept that:
a. health is the absence of illness, and illness is the presence of chronic disease.
b. culture, education, and socioeconomic status influence one‘s definition of health or
illness.
c. illness is a biological malfunction, and health is biological soundness.
d. lifestyle factors are the major determinants of health or illness.

ANS: B
The concept of health is influenced by culture, education, and socioeconomic factors.

DIF: Cognitive Level: Comprehension REF: p. 15 OBJ: Theory #1


TOP: Views of Health and Illness KEY: Nursing Process Step: Planning
MSC: NCLEX: Health Promotion and Maintenance: Prevention and Detection of Disease

2. The nurse takes into consideration that the patient with an admitting diagnosis of Type 2
diabetes mellitus and influenza is described as having:
a. two chronic illnesses.
b. two acute illnesses.
c. one chronic and one acute illness.
d. one acute and one infectious illness.
ANS: C NURSINGTB.COM
Chronic illnesses can be controlled but not cured, and are long-lasting. Acute illnesses
develop suddenly and resolve in a short time. Type 2 diabetes mellitus would be considered
chronic, whereas influenza would be considered acute.

DIF: Cognitive Level: Application REF: p. 15 OBJ: Theory #1


TOP: Classification of Illnesses KEY: Nursing Process Step: Planning
MSC: NCLEX: Health Promotion and Maintenance: Prevention and Detection of Disease

3. The nurse explains that an idiopathic disease is one that:


a. is caused by inherited characteristics.
b. develops suddenly, related to new viruses.
c. results from injury during labor or delivery.
d. has an unknown cause.
ANS: D
Idiopathic disease is defined as disease whose cause is unknown.

DIF: Cognitive Level: Knowledge REF: p. 15 OBJ: Theory #1


TOP: Classification of Illnesses KEY: Nursing Process Step: Implementation
MSC: NCLEX: Health Promotion and Maintenance: Prevention and Detection of Disease

4. The nurse assesses a terminal illness in:


a. a 76-year-old admitted to a nursing home with Alzheimer disease who is pacing
and asking to go home.

NURSINGTB.COM

Downloaded by Dallen Mae D. Kadir (kadir.dallenmae.d.bcsi@gmail.com)


lOMoARcPSD|35346190

b. a 43-year-old with Lou Gehrig‘s disease who is refusing food and fluid.
c. a 2-year-old child who burned her esophagus by drinking drain cleaner and who is
being fed by a tube.
d. a 52-year-old diagnosed with lung cancer who had part of one lung removed and
has a closed chest drainage device in place.
ANS: B
A terminal illness is defined as one in which a person will live only a few months, weeks, or
days. A person who refuses food and hydration will generally not live more than a few days.

DIF: Cognitive Level: Comprehension REF: p. 15 OBJ: Theory #1


TOP: Stages of Illness KEY: Nursing Process Step: Assessment
MSC: NCLEX: Physiological Integrity: Physiological Adaptation

5. The nurse clarifies to a patient who now has an abscess following a ruptured appendix that the
abscess is considered to be:
a. a secondary illness.
b. a life-threatening complication.
c. an expected event following any surgery.
d. a disorder easily treated with antibiotics.

ANS: A
A secondary illness is an illness that arises from a primary disorder.

DIF: Cognitive Level: Comprehension REF: p. 15 OBJ: Theory #1


TOP: Views of Health and Illness KEY: Nursing Process Step: Intervention
MSC: NCLEX: Physiological Integrity: Physiological Adaptation
NURSINGTB.COM
6. The nurse uses a diagram to demonstrate how Dunn‘s theory of health and illness can be
compared with a:
a. plant that grows from a seed, blossoms, wilts, and dies.
b. continuum, with peak wellness and death at opposite ends; the person moves back
and forth in a dynamic state of change.
c. ladder; from birth to death the individual moves progressively downward a ladder
to eventual death.
d. state of mind dependent on the individual perception of their own health or illness.
ANS: B
Dunn‘s theory of a health continuum shows how an individual moves between peak wellness
and death in a constant process.

DIF: Cognitive Level: Knowledge REF: p. 16 OBJ: Theory #1


TOP: Views of Health and Illness KEY: Nursing Process Step: Intervention
MSC: NCLEX: Physiological Integrity: Physiological Adaptation

7. A patient has been advised by the primary care provider to take medication for high
cholesterol and to change eating habits after discharge home. The home health nurse
discovered that the patient refused to follow the medical and nutritional directions. The
nurse‘s best initial response to this situation is to:
a. emphasize to the patient how important it is to follow the doctor‘s advice.
b. determine whether any cultural, socioeconomic, or religious values conflict, thus
interfering with the patient‘s compliance.

NURSINGTB.COM

Downloaded by Dallen Mae D. Kadir (kadir.dallenmae.d.bcsi@gmail.com)


lOMoARcPSD|35346190

c. explain that without diet and medication the condition will worsen and serious
problems will develop.
d. inform the primary care provider that the patient is unable to understand the
instructions.
ANS: B
The patient may have cultural, socioeconomic, or religious values that cause conflicts that
prevent her from following the doctor‘s instructions.

DIF: Cognitive Level: Application REF: p. 16 OBJ: Theory #5


TOP: Concepts of Health and Illness, Cultural Influences
KEY: Nursing Process Step: Assessment
MSC: NCLEX: Psychological Integrity: Coping and Adaptation

8. A nurse practicing a holistic approach to nursing care must:


a. recognize that a change in one aspect of the person‘s life can alter the whole of that
person‘s life.
b. take responsibility for health care decisions.
c. promote state of the art technology.
d. discourage the use of more natural remedies and alternative methods of health
care.
ANS: A
Holistic nursing requires that the nurse recognizes that a change in one aspect of the patient‘s
life (biological, sociological, psychological, and spiritual) will bring about changes in that
patient‘s whole life.

DIF: Cognitive Level: C o m p r e h e n sNi oUnR S I NRGETFB: . C pO. M1 9 OBJ: Theory #6


TOP: Holistic Approach to Caring KEY: Nursing Process Step: Assessment
MSC: NCLEX: N/A

9. Included in Maslow‘s hierarchy, physiological needs are those that:


a. nurture intimacy.
b. foster independence.
c. encourage social interaction.
d. protect from harm.
ANS: D
Physiological needs are those that are essential to human life, such as oxygenation, nutrition,
and elimination. Protection from physical harm, from a nursing standpoint, is often equivalent
in importance to physical needs.

DIF: Cognitive Level: Application REF: p. 20 OBJ: Theory #7


TOP: Maslow‘s Hierarchy of Needs KEY: Nursing Process Step: N/A
MSC: NCLEX: Physiological Integrity: Physiological Adaptation

10. The factors involved in assessing the importance the patient attaches to the relief of a
particular deficit include:
a. needs that the nurse must assess to prioritize care, because they may be different
from person to person.
b. ordering needs according to Maslow‘s hierarchy, with lower level needs being
least compelling.

NURSINGTB.COM

Downloaded by Dallen Mae D. Kadir (kadir.dallenmae.d.bcsi@gmail.com)


lOMoARcPSD|35346190

c. needs based on a hierarchy in which higher level needs are more prominent and
demand attention before lower level needs.
d. needs that are usually not known to the patient and that must be determined by the
nurse.
ANS: A
A person‘s concern relative to a needs deficit must be assessed by the nurse to meet the needs
of each patient. Needs are viewed differently from one person to the next.

DIF: Cognitive Level: Comprehension REF: p. 20 OBJ: Theory #7


TOP: Maslow‘s Hierarchy of Needs KEY: Nursing Process Step: Assessment
MSC: NCLEX: Physiological Integrity: Physiological Adaptation

11. The nurse believes that patient teaching of how to give insulin and monitor blood glucose
levels will improve the level of the patient‘s:
a. physiological well-being.
b. security, by providing psychological comfort.
c. self-esteem, by promoting independence and learning.
d. self-actualization, by seeking knowledge and truth.
ANS: C
Patient education activities that are to be used after discharge enhance independence and
promote self-esteem.

DIF: Cognitive Level: Application REF: p. 27 OBJ: Theory #7


TOP: Maslow‘s Hierarchy of Needs KEY: Nursing Process Step: Implementation
MSC: NCLEX: Psychosocial Integrity: Psychosocial Adaptation
NURSINGTB.COM
12. Homeostasis can be described as:
a. the unchanging steady condition of humans in a changing external environment.
b. a tendency of biological systems toward stability of the internal environment by
continuously adjusting to survive.
c. biological wellness that comes from the ability of the body to change and respond
to physical changes in the environment.
d. a response to stress that results from a person‘s choice of coping mechanisms to
deal with the stress.
ANS: B
Homeostasis results from the constant adjustment of the internal environment in response to
change; it is mental, emotional, and biological, as well as conscious and unconscious.

DIF: Cognitive Level: Comprehension REF: p. 22 OBJ: Theory #8


TOP: Homeostasis KEY: Nursing Process Step: Assessment MSC: NCLEX: N/A

13. A patient admitted for diagnostic tests is frightened of hospital procedures and is nervous
about the possible outcome of the tests. She states that her mouth is dry and her heart is
pounding. Her blood pressure is 168/78 mm Hg (her usual blood pressure is 140/80 mm Hg),
pulse is 112 beats/min, and respirations are 22 breaths/min. The nurse will recognize that
these signs and symptoms are:
a. indicative of serious, acute health problems and should be reported to the primary
care provider immediately.
b. most likely related to the disease for which the patient is admitted to the hospital.

NURSINGTB.COM

Downloaded by Dallen Mae D. Kadir (kadir.dallenmae.d.bcsi@gmail.com)


lOMoARcPSD|35346190

c. the effects of the parasympathetic nervous system and can be ignored.


d. the effects of the sympathetic nervous system that can negatively affect the
patient‘s health.
ANS: D
Fear stimulates the sympathetic nervous system to produce the symptoms identified in the
question. If prolonged, they negatively affect a person‘s health.

DIF: Cognitive Level: Analysis REF: p. 26|Table 2-2


OBJ: Theory #10 TOP: Stress KEY: Nursing Process Step: Assessment
MSC: NCLEX: Psychosocial Integrity: Coping and Adaptation

14. According to Hans Selye‘s general adaptation syndrome (GAS), a person who has
experienced excessive and prolonged stress is likely to:
a. develop an illness or disease such as allergy, arthritis, or asthma.
b. become resistant to biological methods of treatment.
c. seek treatment for imagined illnesses and nonexistent symptoms.
d. be admitted to the hospital during the alarm stage.

ANS: A
Many diseases are known to be caused or exacerbated by prolonged stress. Selye concluded
that stress-induced illnesses respond to biological methods of treatment.

DIF: Cognitive Level: Comprehension REF: p. 26|Box 2-2


OBJ: Theory #10 TOP: Adaptation KEY: Nursing Process Step: Assessment
MSC: NCLEX: Psychosocial Integrity: Coping and Adaptation

asUeRxSpIN
15. The nurse is aware that a stressor N erGieTnBc.C
edOM
by an individual is usually perceived:
a. as a negative event or stimulus that affects homeostasis in maladaptive ways.
b. in different ways based on previous experience and personality traits.
c. as an opportunity for growth and learning.
d. in similar ways if age and education are similar.
ANS: B
Stressors are not perceived the same way by different people or even by the same person at
different times. The experience of a stressor depends on previous experience and personality,
as well as factors such as physical or emotional conditions, age, and education.

DIF: Cognitive Level: Comprehension REF: p. 26 OBJ: Theory #9


TOP: General Adaptation Syndrome KEY: Nursing Process Step: Planning
MSC: NCLEX: Psychological Integrity: Psychosocial Adaptation

16. In 1946, the World Health Organization redefined health as the:


a. absence of disease or infirmity.
b. state of complete physical, mental, and social well-being.
c. presence of disease or infirmity.
d. state of incomplete physical, mental, and social well-being.
ANS: B
In 1946, the World Health Organization redefined health as ―the state of complete physical,
mental, and social well–being, and not merely the absence of disease or infirmity.‖

NURSINGTB.COM

Downloaded by Dallen Mae D. Kadir (kadir.dallenmae.d.bcsi@gmail.com)


lOMoARcPSD|35346190

DIF: Cognitive Level: Knowledge REF: p. 28 OBJ: Theory #1


TOP: Views of Health and Illness KEY: Nursing Process Step: N/A
MSC: NCLEX: N/A

17. The nurse assesses that a person is in the acceptance stage of illness when the patient:
a. looks to home remedies to become well.
b. reassumes usual responsibilities and roles.
c. assumes the ―sick‖ role.
d. rejects medical treatment.

ANS: C
When a person enters the acceptance stage of illness, he or she assumes the ―sick role‖ and
withdraws from usual responsibilities and will frequently seek medical treatment at this time.

DIF: Cognitive Level: Comprehension REF: p. 15 OBJ: Theory #1


TOP: Acceptance Stage KEY: Nursing Process Step: Assessment
MSC: NCLEX: Physiological Integrity: Physiological Adaptation

18. The nurse instructs a patient that according to Selye‘s GAS theory, when stress is strong
enough and occurs over a long enough period, the patient will enter the stage of:
a. convalescence.
b. alarm.
c. transition.
d. exhaustion.
ANS: D
The exhaustion stage in the GAS occurs when the stressor has been present for such a period
that the patient will deplete the b oNd Uy ‘RsS rI eNsGo Tu Br c. Ce sO M
f o r adaption.

DIF: Cognitive Level: Comprehension REF: p. 24 OBJ: Theory #1


TOP: Exhaustion Stage of GAS KEY: Nursing Process Step: Intervention
MSC: NCLEX: Psychosocial Integrity: Coping and Adaptation

19. The nurse explains defense mechanisms as a patient‘s attempt to:


a. justify the patient‘s assumption of the ―sick‖ role.
b. reduce anxiety.
c. problem solve.
d. increase dependence.

ANS: B
Defense mechanisms are unconscious strategies to reduce anxiety.

DIF: Cognitive Level: Knowledge REF: p. 26 OBJ: Theory #9


TOP: Defense Mechanisms KEY: Nursing Process Step: Implementation
MSC: NCLEX: Psychological Integrity: Coping and Adaptation

20. In giving nursing care to persons of Asian origin, the nurse should:
a. keep the room warm and free of drafts.
b. look the patient directly in the eye.
c. ask permission before touching the patient.
d. warmly clasp the patient‘s hand in greeting.

ANS: C

NURSINGTB.COM

Downloaded by Dallen Mae D. Kadir (kadir.dallenmae.d.bcsi@gmail.com)


lOMoARcPSD|35346190

Seek permission before touching persons of Asian extraction, because they may be sensitive
to physical, personal contact.

DIF: Cognitive Level: Application REF: p. 18|Table 2-1


OBJ: Theory #4 TOP: Cultural Sensitivity
KEY: Nursing Process Step: Implementation
MSC: NCLEX: Psychological Integrity: Coping and Adaptation

21. Sickle cell anemia is an example of a biological trait found primarily in:
a. Asian populations.
b. African populations.
c. American Indian populations.
d. Hispanic populations.

ANS: B
Sickle cell anemia is a biological variation found predominantly in people of African descent.

DIF: Cognitive Level: Knowledge REF: p. 18|Table 2-1


OBJ: Theory #5 TOP: Cultural Influences KEY: Nursing Process Step: N/A
MSC: NCLEX: N/A

22. When a young family man hospitalized after breaking his leg confides to the nurse that he is
concerned about the well-being of his family and financial stress, the nurse can best support
his sense of security by:
a. reassuring him that his leg will heal quickly.
b. actively listening to his concerns.
c. encouraging family to make frequent visits.
d. distracting him from his conceNrnUsRb
SIyNsGoTcB
ia.CliO
zaMtion.
ANS: B
A nurse‘s ability to use active listening will enhance the sense of security when patients feel
that their needs are perceived accurately.

DIF: Cognitive Level: Application REF: p. 21 OBJ: Theory #7


TOP: Maslow‘s Hierarchy of Needs KEY: Nursing Process Step: Intervention
MSC: NCLEX: Psychosocial Integrity: Coping and Adaptation

23. The nurse assesses successful adaptation in a post stroke patient when the patient:
a. learns to walk and maintain balance with the aid of a walker.
b. consistently takes antihypertensive drugs.
c. attempts to get out of bed unassisted.
d. refuses assistance with feeding.
ANS: A
Adaptation is a readjustment in habits to limitations and disabilities. Learning to walk and
maintain balance with the aid of a walker is an example of this.

DIF: Cognitive Level: Application REF: p. 22 OBJ: Theory #1


TOP: Adaptation KEY: Nursing Process Step: Assessment
MSC: NCLEX: Physiological Integrity: Physiological Adaptation

24. The nurse takes into consideration that in the stage of resistance in Selye‘s GAS, the patient:

NURSINGTB.COM

Downloaded by Dallen Mae D. Kadir (kadir.dallenmae.d.bcsi@gmail.com)


lOMoARcPSD|35346190

a. regresses to a dependent state.


b. continues to battle for equilibrium.
c. becomes maladaptive.
d. begins to develop stress-related disorders.
ANS: B
The resistance stage is the second stage in the GAS when a patient is still attempting to find
equilibrium.

DIF: Cognitive Level: Comprehension REF: p. 24 OBJ: Theory #10


TOP: Salye‘s GAS KEY: Nursing Process Step: Planning
MSC: NCLEX: Physiological Integrity: Physiological Adaptation

25. A patient states, ―I am not obese. My entire family is large.‖ The nurse assesses that the
patient is using the defense mechanism of:
a. sublimation.
b. projection.
c. denial.
d. displacement.

ANS: C
Denial is a defense mechanism that allows a person to live as though an unwanted piece of
information or reality does not exist. There is a persistent refusal to be swayed by the
evidence.

DIF: Cognitive Level: Application REF: p. 27|Table 2-3


OBJ: Theory #8 TOP: Denial KEY: Nursing Process Step: Assessment
MSC: NCLEX: Psychosocial IntegriNtyU: RCSoIpNiG
ngTBan. CdOAMdaptation

26. A child who has just been scolded by her mother proceeds to hit her doll with a hairbrush. The
nurse recognizes the child‘s actions are characteristics of:
a. denial.
b. displacement.
c. rationalization.
d. repression.
ANS: B
Displacement is a defense mechanism that characterizes discharging intense feelings for one
person onto an object or another person who is less threatening, thereby satisfying an impulse
with a substitute object.

DIF: Cognitive Level: Application REF: p. 27|Table 2-3


OBJ: Theory #8 TOP: Defense Mechanisms
KEY: Nursing Process Step: Assessment
MSC: NCLEX: Psychosocial Integrity: Coping and Adaptation

27. The nurse encourages a patient to participate in health maintenance by maintaining an ideal
body weight as a method of:
a. primary prevention.
b. secondary prevention.
c. tertiary prevention.
d. simple prevention.

NURSINGTB.COM

Downloaded by Dallen Mae D. Kadir (kadir.dallenmae.d.bcsi@gmail.com)


lOMoARcPSD|35346190

ANS: A
Primary prevention avoids or delays occurrence of a specific disease or disorder.

DIF: Cognitive Level: Comprehension REF: p. 27 OBJ: Theory #1


TOP: Primary Prevention KEY: Nursing Process Step: Implementation
MSC: NCLEX: Health Promotion and Maintenance: Prevention and Early Detection of Disease

28. A nurse clarifies that methods of tertiary prevention are designed for:
a. rehabilitation.
b. delay of the development of a disorder.
c. screening for early detection of disease.
d. using an established protocol of therapy for a specific disease.
ANS: A
Tertiary prevention consists of rehabilitation measures after the disease or disorder has
stabilized. Latent prevention does not exist.

DIF: Cognitive Level: Comprehension REF: p. 27 OBJ: Theory #1


TOP: Tertiary Prevention KEY: Nursing Process Step: Implementation
MSC: NCLEX: Physiological Integrity: Physiological Adaptation

29. When a new admission to an extended care facility wanders about listlessly, eats only a small
amount of each meal, and keeps himself isolated, the nurse can intervene by:
a. assisting with feeding at each meal.
b. reminding him that he is in a safe and secure area.
c. socializing with him in the privacy of his room.
d. supporting him to interact withNU
anRSeIxNeGrcTiBse.CgOrM
oup.
ANS: D
The membership and social interaction in a group may provide a means for a sense of
belonging.

DIF: Cognitive Level: Application REF: p. 27 OBJ: Theory #11


TOP: Love and Belonging KEY: Nursing Process Step: Implementation
MSC: NCLEX: Psychosocial Integrity: Psychosocial Adaptation

MULTIPLE RESPONSE

1. When the brain perceives a situation as threatening, the sympathetic nervous system reacts by
stimulating which of the following physiological functions? (Select all that apply.)
a. Constriction of the pupils
b. Dilation of the bronchial tubes
c. Decreased heart rate
d. Dilation of the pupils
ANS: B, D
Activation of the sympathetic nervous system causes the pupils and bronchial tubes to dilate.
It also causes the heart rate to increase.

DIF: Cognitive Level: Analysis REF: p. 23 OBJ: Theory #11


TOP: Sympathetic Nervous System KEY: Nursing Process Step: N/A

NURSINGTB.COM

Downloaded by Dallen Mae D. Kadir (kadir.dallenmae.d.bcsi@gmail.com)


lOMoARcPSD|35346190

MSC: NCLEX: N/A

2. The nurse describes behaviors of the transition stage of illness, which are: (Select all that
apply.)
a. awareness of vague symptoms.
b. denial of feeling ill.
c. resorts to self-medication.
d. withdrawal from roles and responsibilities.
e. recovery from illness begins.
ANS: A, B, C
The transition stage (onset) of illness is demonstrated by the patient‘s awareness of vague
symptoms, denial of feeling ill, and initiation of self-medication; however, he or she still
fulfills the roles and responsibilities of life.

DIF: Cognitive Level: Comprehension REF: p. 15 OBJ: Theory #1


TOP: Stages of Illness KEY: Nursing Process Step: Assessment
MSC: NCLEX: Physiological Integrity: Physiological Adaptation

3. Which defines the holistic approach to caring for the sick and promoting wellness? (Select all
that apply.)
a. The nurse‘s focus is specific to the disease or injury.
b. The nurse realizes that each person has a responsibility for his or her own health.
c. Health care providers are required to intervene on behalf of all persons to ensure
that health goals are met.
d. Providers combine traditional methods of health care with relaxation techniques
for pain management. NURSINGTB.COM
e. A change in one aspect of a person‘s life may or may not alter the person as a
whole.
ANS: B, C, D, E
The holistic approach to medicine treats the patient as a whole and may use a mix of
traditional medicine and alternative medicine. Any change in one aspect of the whole may
change the entire whole.

DIF: Cognitive Level: Comprehension REF: p. 19 OBJ: Theory #6


TOP: Holistic Approach KEY: Nursing Process Step: N/A
MSC: NCLEX: N/A

4. The responses during the alarm stage of the general adaptation syndrome as defined by Hans
Selye include: (Select all that apply.)
a. slight increase in body temperature.
b. substantial increase in energy.
c. decreased appetite.
d. hormones released for mobilization for defense.
e. the body‘s adaptation abilities temporarily overreacting.
ANS: A, C, D
The responses during the alarm stage according to the general adaptation syndrome include a
slight rise in temperature, a loss of energy, decreased appetite, and a release of hormones that
mobilizes the body‘s defenses.

NURSINGTB.COM

Downloaded by Dallen Mae D. Kadir (kadir.dallenmae.d.bcsi@gmail.com)


lOMoARcPSD|35346190

DIF: Cognitive Level: Comprehension REF: p. 24 OBJ: Theory #10


TOP: GAS KEY: Nursing Process Step: N/A MSC: NCLEX: N/A

5. The nurse clarifies that a person who is self-actualized would have the characteristics of:
(Select all that apply.)
a. having met all other need levels.
b. being certain of their beliefs and values.
c. not being swayed by new ideas.
d. having little need for creative self-expression.
e. depending on significant others.

ANS: A, B
A self-actualized person has been able to meet all other basic need levels and is certain of his
or her beliefs and values. He or she is open to new ideas and finds many ways of creative
self-expression.

DIF: Cognitive Level: Comprehension REF: p. 16 OBJ: Theory #7


TOP: Self Actualization KEY: Nursing Process Step: Implementation
MSC: NCLEX: Psychological Integrity: Coping and Adaptation

COMPLETION

1. Exercise can reduce stress and anxiety by the release of .

ANS:
endorphins
NURSINGTB.COM
The release of endorphins induces a feeling of well-being and tranquility.

DIF: Cognitive Level: Knowledge REF: p. 26 OBJ: Theory #11


TOP: Views of Health and Illness KEY: Nursing Process Step: N/A
MSC: NCLEX: N/A

2. Adequate is necessary in the communication between nurse and patient in


order to meet the higher basic needs of security, love, belonging, and self-esteem.

ANS:
feedback

Adequate feedback and clarification are essential in assisting the patient meet the higher level
needs.

DIF: Cognitive Level: Comprehension REF: p. 21 OBJ: Theory #7


TOP: Communication KEY: Nursing Process Step: N/A
MSC: NCLEX: N/A

NURSINGTB.COM

Downloaded by Dallen Mae D. Kadir (kadir.dallenmae.d.bcsi@gmail.com)


lOMoARcPSD|35346190

Chapter 03: Legal and Ethical Aspects of Nursing


Williams: deWit's Fundamental Concepts and Skills for Nursing, 8th Edition

MULTIPLE CHOICE

1. A student nurse who is not yet licensed:


a. may not perform nursing actions until he or she has passed the licensing
examination.
b. is not responsible for his or her actions as a student under the state licensing law.
c. are held to the same standards as a licensed nurse.
d. must apply for a temporary student nurse permit to practice as a student.

ANS: C
Student nurses are held to the same standards as a licensed nurse. This means that
although a student nurse may not perform a task as quickly or as smoothly as the licensed
nurse would, the student is expected to perform it as effectively. In other words, she must
achieve the same outcome without harm to the patient. The student is legally responsible for
her own actions or inaction, and many schools require the student to carry malpractice
insurance.

DIF: Cognitive Level: Knowledge REF: p. 32 OBJ: Theory #1


TOP: Practice Regulations for the Student Nurse KEY: Nursing Process Step: N/A
MSC: NCLEX: N/A

2. During an employment interview, the interviewer asks the nurse applicant about HIV status.
The nurse applicant can legally reN
spUoRnSdI:NGTB.COM
a. ―No,‖ even though he or she has a positive HIV test.
b. ―I don‘t know, but I would be willing to be tested.‖
c. ―I don‘t know, and I refuse to be tested.‖
d. ―You do not have a right to ask me that question.‖

ANS: D
In employment practice, it is illegal to discriminate against people with certain diseases or
conditions. Asking a question about health status, especially HIV or AIDS infection, is illegal.

DIF: Cognitive Level: Application REF: p. 34 OBJ: Clinical Practice #1


TOP: Discrimination KEY: Nursing Process Step: N/A
MSC: NCLEX: N/A

3. An example of a violation of criminal law by a nurse is:


a. taking a controlled substance from agency supply for personal use.
b. accidentally administering a drug to the wrong patient, who then has a serious
reaction.
c. advising a patient to sue the doctor for a supposed mistake the doctor made.
d. writing a letter to the newspaper outlining questionable or unsafe hospital
practices.
ANS: A
Theft of a controlled substance is a federal crime and consequently a crime against society.

DIF: Cognitive Level: Application REF: p. 32 OBJ: Theory #2

NURSINGTB.COM

Downloaded by Dallen Mae D. Kadir (kadir.dallenmae.d.bcsi@gmail.com)


lOMoARcPSD|35346190

TOP: Criminal Law KEY: Nursing Process Step: N/A


MSC: NCLEX: N/A

4. The LPN (LVN) assigns part of the care for her patients to a nursing assistant. The LPN is
legally required to perform which of the following for the residents assigned to the assistant?
a. Toilet the residents every 2 hours and as needed.
b. Feed breakfast to one of the residents who needs assistance.
c. Give medications to the residents at the prescribed times.
d. Transport the residents to the physical therapy department.

ANS: C
Toileting, feeding, and transporting residents or patients are tasks that can be legally assigned
to a nurse‘s aide. Administering medications is a nursing act that can be performed only by a
licensed nurse or by a student nurse under the supervision of a licensed nurse.

DIF: Cognitive Level: Application REF: p. 33 OBJ: Theory #3


TOP: Delegation KEY: Nursing Process Step: Implementation
MSC: NCLEX: Safe, Effective Care Environment: Coordinated Care

5. If a nurse is reported to a state board of nursing for repeatedly making medication errors, it is
most likely that:
a. the nurse will immediately have his or her license revoked.
b. the nurse will have to take the licensing examination again.
c. a course in legal aspects of nursing care will be required.
d. there will be a hearing to determine whether the charges are true.

ANS: D
The nurse may have his or her l i c eNnUs Re Sr IeNv Go kT eBd. CoOr Mb e required to take a refresher course, but
this would be based on the evidence presented at a hearing. The licensing examination is not
usually required as a correction of the situation as described.

DIF: Cognitive Level: Knowledge REF: p. 33 OBJ: Theory #3


TOP: Professional Discipline KEY: Nursing Process Step: N/A
MSC: NCLEX: N/A

6. A nurse co-worker arrives at work 30 minutes late, smelling strongly of alcohol. The fellow
nurses‘ legal course of action is to:
a. have the nurse lie down in the nurses‘ lounge and sleep while others do the work.
b. state that, if this happens again, it will be reported.
c. report the condition of the nurse to the nursing supervisor.
d. offer a breath mint and instruct the nurse co-worker to work.

ANS: C
Nurses must report the condition. It is a nurse‘s legal and ethical duty to protect patients from
impaired or incompetent workers. Allowing the impaired nurse to sleep enables the impaired
nurse to avoid the consequences of his or her actions and to continue the risky behavior.
Threatening to report ―the next time‖ continues to place patients at risk, as does masking the
signs of impairment with breath mints.

DIF: Cognitive Level: Application REF: p. 33 OBJ: Theory #3


TOP: Professional Discipline KEY: Nursing Process Step: N/A
MSC: NCLEX: N/A

NURSINGTB.COM

Downloaded by Dallen Mae D. Kadir (kadir.dallenmae.d.bcsi@gmail.com)


lOMoARcPSD|35346190

7. When a student nurse performs a nursing skill, it is expected that the student:
a. performs the skill as quickly as the licensed nurse.
b. achieves the same result as the licensed nurse.
c. not be held to the same standard as the licensed nurse.
d. always be directly supervised by an instructor.
ANS: B
Students are not expected to perform skills as quickly or as smoothly as experienced nurses,
but students must achieve the same result in a safe manner.

DIF: Cognitive Level: Comprehension REF: p. 33 OBJ: Theory #1


TOP: Practice Regulations for the Student Nurse KEY: Nursing Process Step: N/A
MSC: NCLEX: N/A

8. If a nurse receives unwelcome sexual advances from a nursing supervisor, the first step the
nurse should take is to:
a. send an anonymous letter to the nursing administration to alert them to the
situation.
b. tell the nursing supervisor that she is uncomfortable with the sexual advances and
ask the supervisor to refrain from this behavior.
c. report the nursing supervisor to the state board for nursing.
d. resign and seek employment in a more comfortable environment.
ANS: B
The first step in dealing with sexual harassment in the workplace is to indicate to the person
that the actions or conversations are offensive and ask the person to stop. If the actions
continue, then reporting the occ ur NreUnRc Se I tNoGtTh Be . sCuOpMe rvi sor or the offender‘s supervisor is
indicated.

DIF: Cognitive Level: Application REF: p. 34 OBJ: Clinical Practice #1


TOP: Sexual Harassment KEY: Nursing Process Step: N/A
MSC: NCLEX: N/A

9. A person who has been brought to the emergency room after being struck by a car insists on
leaving, although the doctor has advised him to be hospitalized overnight. The nurse caring
for this patient should:
a. have him sign a Leave Against Medical Advice (AMA) form.
b. tell him that he cannot leave until the doctor releases him.
c. immediately begin the process of involuntary committal.
d. contact the person‘s health care proxy to assist in the decision-making process.
ANS: A
A person has the right to refuse medical care, and agencies use the Leave AMA to document
the medical advice given and the patient‘s informed choice to leave against that advice.

DIF: Cognitive Level: Application REF: p. 39 OBJ: Clinical Practice #3


TOP: Patient Rights KEY: Nursing Process Step: Implementation
MSC: NCLEX: N/A

10. The information in a patient‘s medical record may legally be:


a. copied by students for use in school reports or case studies.

NURSINGTB.COM

Downloaded by Dallen Mae D. Kadir (kadir.dallenmae.d.bcsi@gmail.com)


lOMoARcPSD|35346190

b. provided to lawyers or insurers without the patient‘s permission.


c. shared with other health care providers at the patient‘s request.
d. withheld from the patient, because it is the property of the doctor or agency.
ANS: C
A release or consent is required to provide information from a patient‘s medical record to
anyone not directly caring for that patient. The patient must provide consent to provide
information to insurers, lawyers, or other health care agencies or providers. The patient has
the right to access the information in his or her medical record (copies), but the agency or
doctor retains ownership of the document.

DIF: Cognitive Level: Application REF: p. 39 OBJ: Theory #5


TOP: Legal Documents KEY: Nursing Process Step: N/A
MSC: NCLEX: N/A

11. If a patient indicates that he is unsure if he needs the surgery he is scheduled for later that
morning, the nurse would best reply:
a. ―Your doctor explained all of that yesterday when you signed the consent.‖
b. ―Your doctor is in the operating room; she can‘t talk to you now.‖
c. ―You should have the surgery; your doctor recommended that you have it.‖
d. ―I will call the doctor to speak with you before you go to the operating room.‖

ANS: D
A consent can be withdrawn at any time before the treatment or procedure has been started.
The primary care provider should be notified by the supervising nursing staff of the unit.

DIF: Cognitive Level: Application REF: p. 38 OBJ: Clinical Practice #4


TOP: Informed Consent N U R S I NKGETY
B :. C N
OMur sin g Process Step: Implementation
MSC: NCLEX: Physiological Integrity: Basic Care and Comfort

12. A 16-year-old boy is admitted to the emergency room after fracturing his arm from falling off
his bike while visiting with his stepfather who is not the custodial parent. The nurse is
preparing him to go to the operating room but must obtain a valid informed consent by:
a. having the patient sign the consent for surgery.
b. obtaining the signature of his stepfather for the surgery.
c. declaring the patient to be an emancipated minor.
d. obtaining permission of the custodial parent for the surgery.
ANS: D
The patient is a minor and cannot legally sign his own consent unless he is an emancipated
minor; the guardian for this patient is the custodial parent. A step parent is not a legal
guardian for a minor unless the child has been adopted by the step parent. The hospital does
not have the authority to declare the patient an emancipated minor.

DIF: Cognitive Level: Application REF: p. 38 OBJ: Clinical Practice #3


TOP: Consent KEY: Nursing Process Step: Intervention
MSC: NCLEX: Safe, Effective Care Environment: Coordinated Care

13. A patient has advance directives spelled out in a durable power of attorney, with the
appointment of his daughter as his health care agent. The daughter will be responsible for:
a. paying all the medical bills associated with the father‘s illness.
b. making all informed consent decisions for her father.

NURSINGTB.COM

Downloaded by Dallen Mae D. Kadir (kadir.dallenmae.d.bcsi@gmail.com)


lOMoARcPSD|35346190

c. making all choices about her father‘s health care if the father is unable.
d. paying only for those health care decisions based on the advance directives.
ANS: C
A health care agent makes decisions for the patient only when a patient is unable, according to
the wishes made known by the patient in advance directives. A health care agent is not
responsible for financial decisions or payments.

DIF: Cognitive Level: Application REF: p. 39 OBJ: Clinical Practice #5


TOP: Advance Directives KEY: Nursing Process Step: N/A
MSC: NCLEX: N/A

14. A patient has signed a do-not-resuscitate (DNR) order. If a nurse performs cardiopulmonary
resuscitation (CPR) when the patient stops breathing and then successfully revives the patient,
the:
a. nurse could be found guilty of battery.
b. patient would have no grounds for legal action.
c. patient could charge the nurse with false imprisonment.
d. nurse could be found guilty of assault.
ANS: A
A nurse who attempts CPR on a patient who had a doctor‘s order for a DNR could be found
guilty of battery.

DIF: Cognitive Level: Comprehension REF: p. 39 OBJ: Clinical Practice #3


TOP: DNR KEY: Nursing Process Step: N/A MSC: NCLEX: N/A

15. A patient refuses to take his m e di cNaUt iRoSnIsNoGrTtBo. Ce O


a tMh is breakfast. He is alert, mentally
competent, and fairly comfortable. The nurse should:
a. give the medications by injection if the patient will not take them orally.
b. respect the patient‘s right to refuse medications or food, because he is competent.
c. tell the patient that he must cooperate with his care.
d. contact the doctor to insert a feeding tube to supply both medicine and food.

ANS: B
The competent patient has the right to refuse medicine, food, treatments, and procedures.
Giving (or threatening to give) medications by injection over the patient‘s objections is
considered battery. Threatening the patient or overriding the patient‘s wishes is a violation of
the patient‘s bill of rights and constitutes assault or battery.

DIF: Cognitive Level: Application REF: p. 40 OBJ: Clinical Practice #3


TOP: Patient‘s Rights KEY: Nursing Process Step: N/A
MSC: NCLEX: N/A

16. A nurse remarks to several people that ―Dr. X must be getting senile because she makes so
many mistakes.‖ If that remark results in some of Dr. X‘s patients changing to another doctor,
Dr. X would have grounds to sue the nurse for:
a. slander.
b. libel.
c. invasion of privacy.
d. negligence.

NURSINGTB.COM

Downloaded by Dallen Mae D. Kadir (kadir.dallenmae.d.bcsi@gmail.com)


lOMoARcPSD|35346190

ANS: A
A person who makes untrue, malicious, or harmful remarks that damage a person‘s reputation
and cause injury (loss of business) is guilty of defamation and slander. Libel is defamation
that is written.

DIF: Cognitive Level: Application REF: p. 40 OBJ: Clinical Practice #5


TOP: Defamation/Slander KEY: Nursing Process Step: N/A
MSC: NCLEX: N/A

17. A licensed nurse is liable for charges of malpractice when she:


a. does not show up for work and fails to call to notify the agency.
b. clocks in for another nurse to prevent that nurse from having pay docked.
c. falsifies data, causing the patient to suffer problems resulting in death.
d. assists in performing CPR that is unsuccessful, and the patient dies.

ANS: C
Malpractice is professional negligence or, in this case, doing (falsifying) something the
reasonable and prudent nurse would not do. It is the proximate cause of the patient injury.
This is a case of causation.

DIF: Cognitive Level: Application REF: p. 40|Box 3-6


OBJ: Theory #5 TOP: Negligence and Malpractice KEY: Nursing Process Step: N/A
MSC: NCLEX: N/A

18. A postoperative patient in the intensive care unit (ICU) is so confused and agitated that staff
have not been able to safely care for him. He has pulled out his central line once, and he slides
to the bottom of the bed, where he attempts to climb out, pulling and disrupting the various
tubes and monitors. The nurse‘s bNe sUt RcSoIuNrGseT Bo .fCaOcMt i on is to:
a. place him in a protective vest device.
b. use a sheet to tie him in a chair at the nurses‘ station.
c. request that the doctor write an order for a protective device and/or medication.
d. call a family member to stay with the patient.

ANS: C
A protective device may not be used (except in an emergency) without a doctor‘s order, and it
is used only when other less restrictive means do not provide safety for the patient.

DIF: Cognitive Level: Application REF: p. 41 OBJ: Clinical Practice #3


TOP: False Imprisonment KEY: Nursing Process Step: Implementation
MSC: NCLEX: Safe, Effective Care Environment: Safety and Infection Control

19. An elderly, slightly confused patient sustains an injury from a heating pad that was wrongly
applied by the nurse. The nurse should:
a. pretend to be unaware of the injury to the patient.
b. report the incident to the risk management team via an incident report.
c. document in the patient‘s medical record that an incident report was filled out.
d. not document anything about the injury in the patient‘s medical record.

ANS: B

NURSINGTB.COM

Downloaded by Dallen Mae D. Kadir (kadir.dallenmae.d.bcsi@gmail.com)


lOMoARcPSD|35346190

When an incident occurs that has potential for a future lawsuit, the risk management team
should be aware of it as soon as possible. An incident report should be filled out, and the
patient medical record should be documented to describe the injury. No mention of the
incident report is usually made in the patient medical record. Honesty and a forthright
explanation to the patient reduce the risk of lawsuits.

DIF: Cognitive Level: Application REF: p. 43 OBJ: Theory #5


TOP: Incident Reports KEY: Nursing Process Step: N/A
MSC: NCLEX: N/A

20. Nursing liability insurance is a policy purchased and put into effect by the nurse for the
purpose of:
a. providing protection against being sued.
b. reducing the chance of litigation.
c. paying attorney fees and any award won by the plaintiff.
d. providing the hospital with added protection.

ANS: C
Nursing liability insurance pays attorney fees and any award won by the plaintiff.

DIF: Cognitive Level: Comprehension REF: p. 43 OBJ: Theory #5


TOP: Nursing Ethics KEY: Nursing Process Step: N/A
MSC: NCLEX: N/A

21. Ethics and law are different from each other in that ethics:
a. bear a penalty if violated.
b. are voluntary.
c. rarely change. NURSINGTB.COM
d. can always direct all decisions.
ANS: B
Ethics are voluntary and are based on values. Ethics may change as parameters of health care
change. There is no penalty for violation.

DIF: Cognitive Level: Analysis REF: p. 43 OBJ: Theory #6


TOP: Nursing Ethics KEY: Nursing Process Step: N/A
MSC: NCLEX: N/A

22. To best protect himself or herself from being sued, the nurse should:
a. continue to do procedures as taught in school.
b. purchase malpractice insurance.
c. maintain competency.
d. use evidence-based practice.
ANS: C
Keeping up with continuing education, maintaining competency, and seeking to improve
one‘s own practice by self-evaluation will best protect the nurse.

DIF: Cognitive Level: Comprehension REF: p. 42|Box 3-7


OBJ: Theory #5 TOP: Avoiding Lawsuits KEY: Nursing Process Step: N/A
MSC: NCLEX: N/A

NURSINGTB.COM

Downloaded by Dallen Mae D. Kadir (kadir.dallenmae.d.bcsi@gmail.com)


lOMoARcPSD|35346190

23. The Health Insurance Portability and Accountability Act‘s (HIPAA) main focus is in keeping:
a. patients safe from harm.
b. patient information in a secure office area.
c. medications in a locked area.
d. hospital infections under control.
ANS: B
HIPAA regulates the way patient information is conveyed and stored.

DIF: Cognitive Level: Comprehension REF: p. 37|Box 3-4


OBJ: Clinical Practice #1 TOP: HIPAA KEY: Nursing Process Step: N/A
MSC: NCLEX: N/A

24. Which of the following could place the nurse in a serious legal situation?
a. A nurse posts a poem about the qualities of a compassionate nurse on his or her
social media page.
b. A nurse‘s mother shares a ―selfie‖ of her daughter (a nurse) and a celebrity patient
she is caring for on her social media page.
c. A nurse posts a request for prayer for strength after a difficult day at work.
d. A nurse posts a video of fellow nurse‘s lip syncing and dancing to a popular song,
―We are Strong.‖
ANS: B
Legal and Ethical Considerations
Social Media and HIPAA
Health care agencies and institutions have had to become more diligent in protecting personal
health information (PHI) as a result. It is imperative that no PHI be disseminated, either
intentionally or unintentionally, ovNeUrRsSoIcNiG
alTm
B.eCdOiaM. Posting of pictures, discussions (even
those that do not use patient or hospital names), and images of x-rays all violate HIPAA and
place the nurse in a serious legal situation. It is generally best to separate one‘s personal and
professional life when dealing with social media. The National Council of State Boards of
Nursing (2011) provides guidelines and suggestions for nurses in dealing with social media
and nursing practice.

DIF: Cognitive Level: Analysis REF: p. 37 OBJ: Clinical Practice #6


TOP: Social Media and HIPAA KEY: Nursing Process Step: N/A
MSC: NCLEX: N/A

25. When a patient asks a nurse to witness the signing of a will, the nurse should refer the request
to the:
a. nurse supervisor.
b. hospital legal department.
c. notary public for the hospital.
d. nurse‘s attorney.

ANS: C
Although witnessing a legal document for a patient is not illegal, most agencies have a policy
regarding the proper course of action by referring the patient to the notary public.

DIF: Cognitive Level: Application REF: p. 39 OBJ: Theory #1


TOP: Witnessing Wills and Other Legal Documents
KEY: Nursing Process Step: Implementation

NURSINGTB.COM

Downloaded by Dallen Mae D. Kadir (kadir.dallenmae.d.bcsi@gmail.com)


lOMoARcPSD|35346190

MSC: NCLEX: Safe, Effective Care Environment: Safety and Infection Control

26. Criteria that justify becoming an emancipated minor and able to sign a medical consent
include all of the following except:
a. independence established through a court order.
b. service in the armed forces.
c. a 14-year-old whose parents are dead.
d. a 17-year-old pregnant female.
ANS: C
Criteria are that the minor be independent by court order, be a member of the military, be
pregnant, or be married.

DIF: Cognitive Level: Application REF: p. 38 OBJ: Clinical Practice #3


TOP: Emancipated Minor KEY: Nursing Process Step: N/A
MSC: NCLEX: N/A

27. A written statement expressing the wishes of a patient regarding future consent for or refusal
of treatment in case the patient is incapable of participating in decision making is an example
of:
a. a privileged relationship.
b. a health care agent.
c. an advance directive.
d. witnessed will.

ANS: C
An advance directive makes the patient‘s wishes known regarding medical decisions and
consent in the event that he or sheNiU
s Ru Sn IaNbGl eT tBo. CpOa M
r t ic i p a te in decision making.

DIF: Cognitive Level: Knowledge REF: p. 39 OBJ: Clinical Practice #5


TOP: Legal Terms KEY: Nursing Process Step: N/A MSC: NCLEX: N/A

28. A nurse is caring for an unmarried 16-year-old patient who has just given birth to a baby boy.
The nurse will get the consent to perform a circumcision on the patient‘s son from the:
a. patient‘s father.
b. patient‘s primary care provider.
c. patient‘s mother.
d. 16-year-old patient.

ANS: D
Pregnancy qualifies as the basis for the 16-year-old to be treated as an emancipated minor.

DIF: Cognitive Level: Application REF: p. 38 OBJ: Clinical Practice #3


TOP: Patient Rights KEY: Nursing Process Step: Implementation
MSC: NCLEX: N/A

29. A 48-year-old man refuses to take a medication ordered for the control of his blood pressure.
The nurse‘s most effective response would be:
a. ―Your doctor expects you to be compliant.‖
b. ―You have the right to refuse. This medication keeps your blood pressure under
control.‖
c. ―Fine. I will document that you are refusing this drug.‖

NURSINGTB.COM

Downloaded by Dallen Mae D. Kadir (kadir.dallenmae.d.bcsi@gmail.com)


lOMoARcPSD|35346190

d. ―Are you aware that you could have a stroke?‖

ANS: B
Patients have the right to refuse medication, but it is the nurse‘s responsibility to explain the
reason for the particular drug.

DIF: Cognitive Level: Application REF: p. 38 OBJ: Theory #1


TOP: Legal Standards KEY: Nursing Process Step: Implementation
MSC: NCLEX: Health Promotion and Maintenance: Prevention and Early Detection of Disease

30. The Occupational Safety and Health Act includes all of the following, except:
a. regulations for handling infectious materials.
b. radiation and electrical equipment safeguards.
c. staffing ratios and delegation criteria.
d. regulations for handling toxic materials.

ANS: C
The Occupational Safety and Health Act was passed in 1970 to improve the work
environment in areas that affect workers‘ health or safety. It includes regulations for handling
infectious or toxic materials, radiation safeguards, and the use of electrical equipment.

DIF: Cognitive Level: Comprehension REF: p. 34 OBJ: N/A


TOP: OSHA KEY: Nursing Process Step: N/A MSC: NCLEX: N/A

31. The most frequently cited cause of a sentinel event by the Joint Commission is a problem in:
a. applying physical restraints.
b. methods of patient transportation.
c. medication errors. NURSINGTB.COM
d. inadequate communication.

ANS: D
The most frequently cited cause of a sentinel event by the Joint Commission is
communication. During ―handoff‖ communication, there is a risk that critical patient care
information might be lost due to lack of communication.

DIF: Cognitive Level: Knowledge REF: p. 35 OBJ: Clinical Practice #2


TOP: Communication KEY: Nursing Process Step: N/A
MSC: NCLEX: N/A

32. The acronym SBAR is a method to communicate with a primary care provider that clarifies a
situation that may result in litigation. The acronym stands for:
a. situation, background, alterations, results.
b. subjective, believable, actual, recommendation.
c. situation, background, assessment, recommendation.
d. situation, basis, assessment, recommendation.

ANS: C
SBAR is an acronym that stands for situation, background, assessment, and recommendation.
This undetailed analysis clarifies the situation in a manner that is concise yet complete.

DIF: Cognitive Level: Knowledge REF: p. 35 OBJ: Theory #5


TOP: SBAR Reporting KEY: Nursing Process Step: N/A
MSC: NCLEX: N/A

NURSINGTB.COM

Downloaded by Dallen Mae D. Kadir (kadir.dallenmae.d.bcsi@gmail.com)


lOMoARcPSD|35346190

33. The patient who cannot legally sign his or her own surgical consent is:
a. a 17-year-old who is serving in the armed forces.
b. a 16-year-old who is legally married.
c. a 17-year-old emancipated minor.
d. an 18-year-old who received a narcotic 30 minutes ago.
ANS: D
The person giving the consent must be able to take part in the decision making. A sedated
person does not have this ability.

DIF: Cognitive Level: Application REF: p. 38 OBJ: Clinical Practice #3


TOP: Patient Rights KEY: Nursing Process Step: N/A
MSC: NCLEX: N/A

34. The nurse who may be liable for invasion of privacy would be the nurse who is:
a. refusing to give patient information to a relative over the phone.
b. firmly closing the door prior to bathing the patient.
c. discussing her patients with a fellow nurse.
d. reporting the patient as a possible victim of elder abuse.
ANS: C
Discussing a patient with anyone, even another health professional, who is not involved in the
patient‘s care can put a nurse at risk for invasion of privacy.

DIF: Cognitive Level: Application REF: p. 38 OBJ: Clinical Practice #3


TOP: Patient Rights KEY: Nursing Process Step: Implementation
MSC: NCLEX: N/A NURSINGTB.COM

35. A characteristic of an advance directive is that:


a. advance directives do not expire.
b. only some states recognize advance directives.
c. advance directives can be nonverbal.
d. advance directives from one state are recognized by another.
ANS: A
An advance directive is a written statement expressing the wishes of the patient regarding
future consent for or refusal of treatment if the patient is incapable of participating in decision
making, and they do not expire. All states recognize advance directives, but each state
regulates advance directives differently, and an advance directive from one state may not be
recognized in another.

DIF: Cognitive Level: Comprehension REF: p. 38 OBJ: Clinical Practice #5


TOP: Advance Directives KEY: Nursing Process Step: N/A
MSC: NCLEX: N/A

36. A patient who is refusing to take his medication is threatened that he will be held down and
forced to take the dose. This is an example of:
a. battery.
b. defamation.
c. assault.
d. invasion of privacy.

NURSINGTB.COM

Downloaded by Dallen Mae D. Kadir (kadir.dallenmae.d.bcsi@gmail.com)


lOMoARcPSD|35346190

ANS: C
Assault is the threat to harm another or even to touch another without that person‘s
permission. The person being threatened must believe that the nurse has the ability to carry
out the threat.

DIF: Cognitive Level: Comprehension REF: p. 40 OBJ: Theory #3


TOP: Legal Terms KEY: Nursing Process Step: N/A MSC: NCLEX: N/A

37. The nurse explains that a sentinel event is a situation in which a patient:
a. refuses care.
b. is accidentally exposed.
c. leaves the hospital against medical advice.
d. comes to harm.

ANS: D
A sentinel event is an unexpected situation in which the patient comes to harm.

DIF: Cognitive Level: Comprehension REF: p. 35 OBJ: Theory #5


TOP: Legal Terms KEY: Nursing Process Step: N/A MSC: NCLEX: N/A

MULTIPLE RESPONSE

1. Professional accountability includes: (Select all that apply.)


a. understanding theory.
b. adhering to the dress code of the facility.
c. asking for assistance when unsure of a procedure or primary care provider order.
d. participating in continuing eduNcUatRiSoInNcGlTasBs.eCsO. M
e. meeting the health care needs of the patient.
f. reporting patient health status changes to all family members.
ANS: A, C, D, E
Professional accountability is a nurse‘s responsibility to meet the health care needs of the
patient in a safe and caring application of nursing skills and understanding of human needs.

DIF: Cognitive Level: Analysis REF: p. 33 OBJ: Theory #3


TOP: Professional Accountability KEY: Nursing Process Step: N/A
MSC: NCLEX: N/A

2. A nurse arrives at the scene of a motor vehicle accident. A person in the vehicle mumbles
incoherently when asked his name. Which actions are not covered by the Good Samaritan
Law? (Select all that apply.)
a. Using two magazines and a bandana to splint a broken arm
b. Applying a tourniquet to a lacerated leg while awaiting emergency personnel
c. Pulling the individual from the surface of the highway
d. Initiating an emergency tracheotomy when the individual goes into respiratory
arrest
e. Compressing a bleeding wound with a soiled shirt
ANS: D
The Good Samaritan Law covers care given in an emergency, but only within the scope of
one‘s practice, and care that does not cause harm resulting from negligence.

NURSINGTB.COM

Downloaded by Dallen Mae D. Kadir (kadir.dallenmae.d.bcsi@gmail.com)


lOMoARcPSD|35346190

DIF: Cognitive Level: Comprehension REF: p. 35 OBJ: Theory #5


TOP: Legal Scope of Practice KEY: Nursing Process Step: N/A
MSC: NCLEX: Safe, Effective Care Environment

3. The Ethics Committee of a facility has the responsibility to: (Select all that apply.)
a. develop policies.
b. address issues in their facility.
c. modify the established codes of ethics as suits the situation.
d. create a master plan for decision making to be followed in ethical dilemmas.
e. help to find a better understanding of ethical dilemmas from different standpoints.

ANS: A, B, E
An Ethics Committee of an institution has representatives from various fields to formulate,
address, and help clarify ethical problems that present themselves in their facility.

DIF: Cognitive Level: Comprehension REF: p. 45 OBJ: Theory #6


TOP: Ethics KEY: Nursing Process Step: N/A MSC: NCLEX: N/A

4. The commonalities of The Codes of Ethics of the National Association for Practical Education
and Service (NAPNES) and The National Federation of Licensed Practical Nurses (NFLPN)
include: (Select all that apply.)
a. commitment to continuing education.
b. respect for human dignity.
c. maintenance of competence.
d. requirement for membership in a national organization.
e. preserving the confidentiality of the nurse-patient relationship.
NURSINGTB.COM
ANS: A, B, C, E
Both Codes of Ethics support maintenance of competency, preservation of confidentiality of
the nurse patient relationship, commitment to continuing education, and respect for human
dignity.

DIF: Cognitive Level: Application REF: p. 43 OBJ: Theory #6


TOP: Ethics KEY: Nursing Process Step: N/A MSC: NCLEX: N/A

COMPLETION

1. In 2003, the Patients‘ Bill of Rights was revised to become the : Understanding
Expectations, Rights, and Responsibilities.

ANS:
Patient Care Partnership

The Patient Care Partnership addresses patient rights and the responsibility of health care
facilities.

DIF: Cognitive Level: Knowledge REF: p. 33 OBJ: Clinical Practice #3


TOP: Patient Rights KEY: Nursing Process Step: N/A
MSC: NCLEX: N/A

NURSINGTB.COM

Downloaded by Dallen Mae D. Kadir (kadir.dallenmae.d.bcsi@gmail.com)


lOMoARcPSD|35346190

2. CAPTA, passed in 1973, is a law regarding the safety of minors. It is the


and .

ANS:
Child Abuse Prevention; Treatment Act

This is a law that requires mandated reporting and defines who is a mandated reporter.

DIF: Cognitive Level: Knowledge REF: p. 34 OBJ: Theory #1


TOP: Professional Accountability KEY: Nursing Process Step: N/A
MSC: NCLEX: N/A

NURSINGTB.COM

NURSINGTB.COM

Downloaded by Dallen Mae D. Kadir (kadir.dallenmae.d.bcsi@gmail.com)


lOMoARcPSD|35346190

DEWITS FUNDAMENTALStC
uvO
iaN
.cC
om
EP- T
ThSe A
MNarD
ketSpK
l a IcL
eLtoSBFuO
y aRndNSUeR
ll S
yoI u
Nr G
Stu
5dTyHME
atD
erIiaTl ION WILLIAMS TEST BANK

Chapter 04: The Nursing Process and Critical Thinking


Williams: deWit's Fundamental Concepts and Skills for Nursing, 8th Edition

MULTIPLE CHOICE

1. The nurse who uses the nursing process will:


a. help reduce the obvious signs of discomfort.
b. help the patient adhere to the primary care provider‘s treatment protocol.
c. approach the patient‘s disorder in a step-by-step method.
d. make all significant nursing care decisions involving patient care.

ANS: C
The nursing process is a collaborative process used throughout the patient‘s stay. It is an
organized method for identifying and meeting patient needs in a step-by-step manner.

DIF: Cognitive Level: Knowledge REF: p. 48 OBJ: Theory #1


TOP: Nursing Process KEY: Nursing Process Step: N/A
MSC: NCLEX: N/A

2. A nurse will arrive at a nursing diagnosis through the nursing process step of:
a. planning.
b. evaluation.
c. research.
d. assessment.

ANS: D
As a result of the nursing assessmN
enUtR, SaI nNuGrTsiBn. Cg OdM
iagnosis is established.

DIF: Cognitive Level: Comprehension REF: p. 50|Table 4-2


OBJ: Theory #2 TOP: Nursing Diagnosis KEY: Nursing Process Step: N/A
MSC: NCLEX: N/A

3. In the collaborative process of delivering care based on the nursing process, the responsibility
of the LPN/LVN is to:
a. collect data of health status.
b. select a nursing diagnosis.
c. organize data to help the RN evaluate patient progress.
d. prioritize nursing diagnoses for more effective care.
ANS: A
The LPN/LVN collects data of the patient‘s health status to assist the RN in selecting a
nursing diagnosis.

DIF: Cognitive Level: Comprehension REF: p. 49|Table 4-1


OBJ: Theory #2 TOP: Critical Thinking KEY: Nursing Process Step: N/A
MSC: NCLEX: N/A

4. The participants of the planning stage of the nursing process during which the health goals are
defined include:
a. the RN.
b. the health team led by the RN.

https://www.coursehero.com/file/62123929/c4pdf/

NURSINGTB.COM

Downloaded by: SUCCEEDGRADES | abbieclin@gmail.com


Distribution
Downloaded by Dallen Mae D.ofKadir
this document is illegal
(kadir.dallenmae.d.bcsi@gmail.com)
lOMoARcPSD|35346190

DEWITS FUNDAMENTALStC
uvO
iaN
.cC
om
EP- T
ThSe A
MNarD
ketSpK
l a IcL
eLtoSBFuO
y aRndNSUeR
ll S
yoI u
Nr G
Stu
5dTyHME
atD
erIiaTl ION WILLIAMS TEST BANK

c. the health team, the patient, and the patient‘s family.


d. the health team as directed by the physician.
ANS: C
The planning stage during which the health goals are defined are best shared by the entire
health team, the patient, and the patient‘s family for the optimum outcome.

DIF: Cognitive Level: Comprehension REF: p. 48 OBJ: Theory #1


TOP: Nursing Process KEY: Nursing Process Step: N/A
MSC: NCLEX: N/A

5. When a resident in the nursing home complains of constipation, the nurse performs a digital
rectal examination and finds a hard fecal mass. This is an example of:
a. implementation.
b. nursing diagnosis.
c. assessment.
d. evaluation.

ANS: C
The examination to confirm and affirm the complaint of constipation is an assessment.

DIF: Cognitive Level: Application REF: p. 48|Table 4-1


OBJ: Theory #1 TOP: Nursing Process
KEY: Nursing Process Step: Assessment
MSC: NCLEX: Physiological Integrity: Basic Care and Comfort

6. The nurse completing morning assessments on a patient who is sitting up in bed is told by the
patient, ―I‘m having trouble breathNiUng—
RS I NI Gc Ta nB ‘. Ct O
seMem to get enough air.‖ The best nursing
response is to:
a. notify the doctor as soon as he or she comes in later in the morning.
b. finish the vital signs for the assigned patients, and then notify the charge nurse.
c. reassure the patient, if his blood pressure and pulse are normal.
d. notify the charge nurse immediately of the patient‘s statement.
ANS: B
The nurse should finish the assessment in order to confirm the complaint and inform the
charge nurse.

DIF: Cognitive Level: Analysis REF: p. 50|Table 4-2


OBJ: Theory #1 TOP: Assessment KEY: Nursing Process Step: Assessment
MSC: NCLEX: Physiological Integrity: Basic Care and Comfort

7. The order in which the nursing process is approached is:


a. planning, assessment, implementation, nursing diagnosis, evaluation.
b. nursing diagnosis, evaluation, assessment, implementation, planning.
c. assessment, nursing diagnosis, planning, implementation, evaluation.
d. evaluation, nursing diagnosis, planning, implementation, assessment.
ANS: C
The order of assessment nursing diagnosis, planning, implementation, and evaluation sets up a
basis for an organized approach to nursing care.

DIF: Cognitive Level: Knowledge REF: p. 49|Box 4-1

https://www.coursehero.com/file/62123929/c4pdf/

NURSINGTB.COM

Downloaded by: SUCCEEDGRADES | abbieclin@gmail.com


Distribution
Downloaded by Dallen Mae D.ofKadir
this document is illegal
(kadir.dallenmae.d.bcsi@gmail.com)
lOMoARcPSD|35346190

DEWITS FUNDAMENTALStC
uvO
iaN
.cC
om
EP- T
ThSe A
MNarD
ketSpK
l a IcL
eLtoSBFuO
y aRndNSUeR
ll S
yoI u
Nr G
Stu
5dTyHME
atD
erIiaTl ION WILLIAMS TEST BANK

OBJ: Theory #1 TOP: Nursing Process KEY: Nursing Process Step: N/A
MSC: NCLEX: N/A

8. Once the nursing plan has been initiated, the nursing care plan will:
a. stay in place until all nursing goals have been met.
b. change as the patient‘s condition changes.
c. remain on the patient record to show progress.
d. be given to the patient for final approval.

ANS: B
The nursing care plan is always a work in progress and will change as the patient condition
changes.

DIF: Cognitive Level: Comprehension REF: p. 50 OBJ: Theory #2


TOP: Nursing Process KEY: Nursing Process Step: N/A
MSC: NCLEX: N/A

9. When a patient states, ―I can‘t walk very well,‖ the first problem-solving step would be to:
a. consider alternatives such as a wheelchair or walker.
b. find out what the problem is, such as weakness or poor balance.
c. choose the alternative with the best chance of success.
d. consider the outcomes of the choices, such as danger of falling with a walker.

ANS: B
Defining the problem clearly assists in the interventions to reduce the problem.

DIF: Cognitive Level: Analysis REF: p. 50 OBJ: Theory #5


TOP: Problem Solving K E Y : N ursing
N U RS I N G T B . C O M Process Step: Assessment
MSC: NCLEX: Physiological Integrity: Basic Care and Comfort

10. A student nurse can begin to develop critical thinking skills by means of:
a. working with a more experienced nurse.
b. questioning every statement made by instructors to be sure of its correctness.
c. memorizing class notes for tests and studying all night for big tests.
d. listening attentively and focusing on the speaker‘s words and meaning.
ANS: D
Critical thinking involves foundation skills such as effective reading and writing and attentive
listening.

DIF: Cognitive Level: Comprehension REF: p. 50 OBJ: Theory #7


TOP: Critical Thinking KEY: Nursing Process Step: N/A
MSC: NCLEX: N/A

11. When a nurse prioritizes the patient care, consideration is given to:
a. completing assessments before mid-shift.
b. considering situations that may result in an alteration of health.
c. assuming all health care activities for a group of patients.
d. identifying who can assist with the aspect of care.

ANS: B
Priority setting includes addressing health endangering situations and physiological needs
first.

https://www.coursehero.com/file/62123929/c4pdf/

NURSINGTB.COM

Downloaded by: SUCCEEDGRADES | abbieclin@gmail.com


Distribution
Downloaded by Dallen Mae D.ofKadir
this document is illegal
(kadir.dallenmae.d.bcsi@gmail.com)
lOMoARcPSD|35346190

DEWITS FUNDAMENTALStC
uvO
iaN
.cC
om
EP- T
ThSe A
MNarD
ketSpK
l a IcL
eLtoSBFuO
y aRndNSUeR
ll S
yoI u
Nr G
Stu
5dTyHME
atD
erIiaTl ION WILLIAMS TEST BANK

DIF: Cognitive Level: Comprehension REF: p. 53 OBJ: Theory #9


TOP: Priority Setting KEY: Nursing Process Step: N/A
MSC: NCLEX: N/A

12. When the nurse checks to see whether a patient has had relief 45 minutes after administering
pain medication, the nurse is performing a(n):
a. nursing diagnosis.
b. implementation.
c. assessment.
d. evaluation.
ANS: D
Evaluation is the step in which the nurse determines whether the plan and interventions are
effective or need to be modified.

DIF: Cognitive Level: Comprehension REF: p. 49|Box 4-1


OBJ: Theory #2 TOP: Nursing Process
KEY: Nursing Process Step: Evaluation
MSC: NCLEX: Physiological Integrity: Basic Care and Comfort

13. The activity that is an implementation in the nursing care is:


a. checking the assigned patient‘s blood pressure, pulse, and respiration.
b. changing the patient‘s surgical dressing.
c. asking the patient to demonstrate how to give himself medication after teaching
him.
d. discussing the patient with other team members to establish a care plan.
NURSINGTB.COM
ANS: B
Changing a dressing that is soiled is a nursing intervention performed to meet a patient‘s need.
Checking vital signs is assessment. Demonstrating medication administration is evaluation.
Discussing the patient with other team members is planning.

DIF: Cognitive Level: Comprehension REF: p. 49|Box 4-1


OBJ: Theory #2 TOP: Implementation KEY: Nursing Process Step: N/A
MSC: NCLEX: N/A

14. Constant nursing assessments and evaluations of the patient will most likely result in:
a. the nursing care plan changing to reflect appropriate priorities.
b. small changes in the patient condition being overlooked.
c. cluttered and confusing documentation.
d. impeded problem solving.

ANS: A
Continued assessment and evaluation are necessary; reprioritizing and reorganizing activities
occur in response to the patient‘s changing condition.

DIF: Cognitive Level: Application REF: p. 50 OBJ: Theory #1


TOP: Nursing Process KEY: Nursing Process Step: N/A
MSC: NCLEX: N/A

https://www.coursehero.com/file/62123929/c4pdf/

NURSINGTB.COM

Downloaded by: SUCCEEDGRADES | abbieclin@gmail.com


Distribution
Downloaded by Dallen Mae D.ofKadir
this document is illegal
(kadir.dallenmae.d.bcsi@gmail.com)
lOMoARcPSD|35346190

DEWITS FUNDAMENTALStC
uvO
iaN
.cC
om
EP- T
ThSe A
MNarD
ketSpK
l a IcL
eLtoSBFuO
y aRndNSUeR
ll S
yoI u
Nr G
Stu
5dTyHME
atD
erIiaTl ION WILLIAMS TEST BANK

15. The effect of using a scientific problem-solving approach in nursing care will cause decision
making to be:
a. slowed down considerably by the multiple steps.
b. rigid and nonpatient oriented.
c. improved nursing care outcomes.
d. unrelated to the nursing process.
ANS: C
A scientific problem-solving approach is most likely to result in positive patient outcomes.

DIF: Cognitive Level: Comprehension REF: p. 50 OBJ: Theory #3


TOP: Problem Solving KEY: Nursing Process Step: Planning
MSC: NCLEX: N/A

16. An emergency room nurse will give first priority to the patient with the most critical need,
which is the patient who:
a. is bleeding from a chin laceration.
b. complains of a productive cough.
c. has a fever of 102F.
d. complains of severe chest pain.
ANS: D
Because the chance of a bad outcome is highest for the patient with chest pain, it is most
appropriate to assess this patient first.

DIF: Cognitive Level: Analysis REF: p. 53 OBJ: Theory #8


TOP: Critical Thinking KEY: Nursing Process Step: Assessment
MSC: NCLEX: N/A NURSINGTB.COM

MULTIPLE RESPONSE

1. Activities considered to be aspects of the implementation step of the nursing process are:
(Select all that apply.)
a. documentation of care given.
b. assembly of supplies.
c. analysis of data gathered.
d. modification of aspects of the plan.
e. evaluation of the patient response.
ANS: A, B
Documentation of care and assembly of supplies are nursing interventions performed during
the implementation step of the nursing process.

DIF: Cognitive Level: Comprehension REF: p. 49|Table 4-1


OBJ: Theory #2 TOP: Nursing Process KEY: Nursing Process Step: N/A
MSC: NCLEX: N/A

2. Descriptions of the activities involved in the nursing diagnosis step of the nursing process are:
(Select all that apply.)
a. determination of potential health problems.
b. clustering of related assessments.

https://www.coursehero.com/file/62123929/c4pdf/

NURSINGTB.COM

Downloaded by: SUCCEEDGRADES | abbieclin@gmail.com


Distribution
Downloaded by Dallen Mae D.ofKadir
this document is illegal
(kadir.dallenmae.d.bcsi@gmail.com)
lOMoARcPSD|35346190

DEWITS FUNDAMENTALStC
uvO
iaN
.cC
om
EP- T
ThSe A
MNarD
ketSpK
l a IcL
eLtoSBFuO
y aRndNSUeR
ll S
yoI u
Nr G
Stu
5dTyHME
atD
erIiaTl ION WILLIAMS TEST BANK

c. sharing of information with the patient and physician.


d. determination of desired outcomes.
e. evaluation of probable outcomes.
ANS: A, B
During the nursing diagnosis step, assessment data are analyzed and clustered to determine
health problems, and appropriate nursing diagnoses are selected.

DIF: Cognitive Level: Comprehension REF: p. 49|Table 4-1


OBJ: Theory #1 TOP: Nursing Process KEY: Nursing Process Step: N/A
MSC: NCLEX: N/A

3. Which of the following items could be the responsibility of the LPN/LVN for a patient‘s plan
of care? (Select all that apply.)
a. Collect data.
b. Perform nursing interventions.
c. Initiate the plan of care.
d. Assist the RN with evaluation of the patient‘s response to nursing interventions.
e. Document nursing care.

ANS: A, B, D
Registered nurses are officially responsible for the initiation of nursing care plans for each
patient, but the LPN/LVN assists with each part of the care plan. The LPN/LVN is often
responsible for data collection to assist the RN with the assessment phase.

DIF: Cognitive Level: Comprehension REF: p. 49|Table 4-1


OBJ: Theory #2 TOP: Nursing Process KEY: Nursing Process Step: N/A
MSC: NCLEX: N/A NURSINGTB.COM

COMPLETION

1. When the nurse constructs a nursing approach after careful judgment and sound reasoning, the
nurse has used a system of .

ANS:
critical thinking

Critical thinking is a concept in which decisions are made using solidly based judgments and
reasoning.

DIF: Cognitive Level: Knowledge REF: p. 50 OBJ: Theory #2


TOP: Critical Thinking KEY: Nursing Process Step: N/A
MSC: NCLEX: N/A

2. Clinical thinking is considered to be the keystone and foundation of the development of


.

ANS:
clinical judgment

Clinical judgment is built on the ability to think critically.

https://www.coursehero.com/file/62123929/c4pdf/

NURSINGTB.COM

Downloaded by: SUCCEEDGRADES | abbieclin@gmail.com


Distribution
Downloaded by Dallen Mae D.ofKadir
this document is illegal
(kadir.dallenmae.d.bcsi@gmail.com)
lOMoARcPSD|35346190

DEWITS FUNDAMENTALStC
uvO
iaN
.cC
om
EP- T
ThSe A
MNarD
ketSpK
l a IcL
eLtoSBFuO
y aRndNSUeR
ll S
yoI u
Nr G
Stu
5dTyHME
atD
erIiaTl ION WILLIAMS TEST BANK

DIF: Cognitive Level: Knowledge REF: p. 50 OBJ: Theory #2


TOP: Critical Thinking KEY: Nursing Process Step: N/A
MSC: NCLEX: N/A

3. The tasks of synthesizing data and linking nursing interventions with patient health problems
are enhanced by the process of .

ANS:
concept mapping

Concept mapping is a method to promote critical thinking by visualizing relationships


between patient health problems and effective intervention.

DIF: Cognitive Level: Knowledge REF: p. 51 OBJ: Theory #3


TOP: Concept Mapping KEY: Nursing Process Step: N/A
MSC: NCLEX: N/A

ORDERING

1. A nurse begins rounds on a medical-surgical nursing unit. Review the following patients on
her assignment. Prioritize the order in which the patients should be assessed, based on their
descriptions.
a. A 22-year-old patient who is awakening from neck surgery.
b. An 82-year-old patient who is blind and needs discharge instructions.
c. A 44-year-old patient with dehydration from vomiting and diarrhea, who was admitted 3
days ago and who has an IV infusNioUnRoSfI NflGuTidBs. .CO M
d. A 35-year-old patient admitted for an injury to his left femoral artery, which required
surgical repair 8 hours ago following an ice skating accident.

ANS:
A, D, C, B

Nursing priorities need to address patients with life-threatening concerns first. A patient just
awakening from neck surgery needs to be assessed first because of the concerns of tracheal
swelling. A patient with a compromised limb is the next priority. The patient on IV fluids for
dehydration is next. The patient for discharge is the last priority.

DIF: Cognitive Level: Analysis REF: p. 53 OBJ: Theory #9


TOP: Prioritizing KEY: Nursing Process Step: Assessment
MSC: NCLEX: Physiological Integrity: Basic Care and Comfort

2. Place the steps of the problem-solving approach in the appropriate order:


a. Predict the likelihood of each outcome occurring.
b. Choose the alternative with the best chance of success.
c. Consider all possible alternatives as the solution to the problem.
d. Identify the problem.
e. Examine possible outcomes of each alternative.

ANS:

https://www.coursehero.com/file/62123929/c4pdf/

NURSINGTB.COM

Downloaded by: SUCCEEDGRADES | abbieclin@gmail.com


Distribution
Downloaded by Dallen Mae D.ofKadir
this document is illegal
(kadir.dallenmae.d.bcsi@gmail.com)
lOMoARcPSD|35346190

DEWITS FUNDAMENTALStC
uvO
iaN
.cC
om
EP- T
ThSe A
MNarD
ketSpK
l a IcL
eLtoSBFuO
y aRndNSUeR
ll S
yoI u
Nr G
Stu
5dTyHME
atD
erIiaTl ION WILLIAMS TEST BANK

D, C, E, A, B

The problem-solving approach requires that a problem be clearly identified, all possible
alternative solutions be examined, outcomes of solutions be considered, probability of
outcome occurring be predicted, and the best alternative be chosen.

DIF: Cognitive Level: Knowledge REF: p. 50 OBJ: Theory #4


TOP: Problem Solving KEY: Nursing Process Step: N/A
MSC: NCLEX: N/A

NURSINGTB.COM

https://www.coursehero.com/file/62123929/c4pdf/

NURSINGTB.COM

Downloaded by: SUCCEEDGRADES | abbieclin@gmail.com


Distribution
Downloaded by Dallen Mae D.ofKadir
this document is illegal
(kadir.dallenmae.d.bcsi@gmail.com)
lOMoARcPSD|35346190

DEWITS FUNDAMENTALStC
uvO
iaN
.cC
om
EP- T
ThSe A
MNarD
ketSpK
l a IcL
eLtoSBFuO
y aRndNSUeR
ll S
yoI u
Nr G
Stu
5dTyHME
atD
erIiaTl ION WILLIAMS TEST BANK

Chapter 05: Assessment, Nursing Diagnosis, and Planning


Williams: deWit's Fundamental Concepts and Skills for Nursing, 8th Edition

MULTIPLE CHOICE

1. When the patient complains of nausea and dizziness, the nurse recognizes these complaints as
data.
a. objective
b. medical
c. subjective
d. adjunct
ANS: C
Subjective data are symptoms that only the patient can identify.

DIF: Cognitive Level: Application REF: p. 58 OBJ: Theory #3


TOP: Assessment Data KEY: Nursing Process Step: Assessment
MSC: NCLEX: Physiological Integrity: Basic Care and Comfort

2. The major goal of the admission interview (usually performed by the RN) is to:
a. establish rapport.
b. help the patient understands the objectives of care.
c. identify the patient‘s major complaints.
d. initiate nursing care plan forms.

ANS: C
The interview is used as part of t hNe Ua Rs sSeI sNsGmTeBn. tC pOrMo c e s s to elicit information about the patient‘s
physical, emotional, and spiritual health.

DIF: Cognitive Level: Comprehension REF: p. 58 OBJ: Theory #1


TOP: Interview KEY: Nursing Process Step: Assessment MSC: NCLEX: N/A

3. An example of a structured format for gathering data that aids in forming a database is:
a. North American Nursing Diagnosis Association–International (NANDA-I).
b. Maslow‘s hierarchy.
c. QSENl
d. Gordon‘s 11 Health Patterns.
ANS: D
Mary Gordon‘s assessment guide is a guided path to cover 11 health points. Although Maslow
may be used, it is not structured.

DIF: Cognitive Level: Knowledge REF: p. 58|Box 5-1


OBJ: Theory # 2 TOP: Gordon‘s 11 Health Patterns KEY: Nursing Process Step: N/A
MSC: NCLEX: N/A

4. During the assessment phase of the nursing process, the nurse:


a. develops a care plan to meet the patient‘s nursing needs.
b. begins to formulate plans for providing nursing intervention.
c. establishes a nursing diagnosis for the nursing care plan.
d. gathers, organizes, and documents data in a logical database.

NURSINGTB.COM

Downloaded by: SUCCEEDGRADES | abbieclin@gmail.com


Distribution
Downloaded by Dallen Mae D.ofKadir
this document is illegal
(kadir.dallenmae.d.bcsi@gmail.com)
lOMoARcPSD|35346190

DEWITS FUNDAMENTALStC
uvO
iaN
.cC
om
EP- T
ThSe A
MNarD
ketSpK
l a IcL
eLtoSBFuO
y aRndNSUeR
ll S
yoI u
Nr G
Stu
5dTyHME
atD
erIiaTl ION WILLIAMS TEST BANK

ANS: D
Gathering and organizing data is the first step in the assessment phase of the nursing process.

DIF: Cognitive Level: Comprehension REF: p. 58 OBJ: Theory #1


TOP: Data Collection KEY: Nursing Process Step: Assessment
MSC: NCLEX: N/A

5. After the admission assessment is completed, on subsequent shifts or days, the nurse:
a. does not assess the patient again unless the condition changes.
b. refers only to the admission assessment during the hospitalization.
c. performs a complete physical examination every day.
d. assesses the patient briefly in the first hour of the shift.
ANS: D
The patient should be briefly assessed at the beginning of each shift and more thoroughly if
his or her condition changes or as per the plan of care.

DIF: Cognitive Level: Comprehension REF: p. 70 OBJ: Theory #1


TOP: Physical Assessment KEY: Nursing Process Step: Assessment
MSC: NCLEX: Physiological Integrity: Basic Care and Comfort

6. The nurse performing an admission interview on an older adult person should:


a. rush through the interview to avoid tiring the patient.
b. direct questions to the family rather than the patient.
c. allow more time for a response to questions.
d. prompt the patient to speed recall.

ANS: C NURSINGTB.COM
When interviewing an older adult person, allow more time because the person will probably
have a more extensive history and may take a little longer to recall the needed information.

DIF: Cognitive Level: Application REF: p. 59 OBJ: Theory #5


TOP: Admission Interview KEY: Nursing Process Step: Intervention
MSC: NCLEX: Physiological Integrity: Basic Care and Comfort

7. A nursing diagnosis consists of:


a. the health care provider‘s medical diagnosis listed as the nursing diagnosis.
b. diagnostic labels formulated by the North American Nursing Diagnosis
Association–International (NANDA-I).
c. the patient‘s explanation of his or her ―chief complaint‖ or ―current complaint.‖
d. the results of the nursing assessment without consideration of doctor‘s orders.
ANS: B
NANDA-I has formulated an official list of nursing diagnoses to identify patient problems and
problems that patients are at risk of developing. A nursing diagnosis is independent of a
medical diagnosis.

DIF: Cognitive Level: Comprehension REF: p. 65 OBJ: Theory #5


TOP: Nursing Diagnosis KEY: Nursing Process Step: Planning
MSC: NCLEX: N/A

NURSINGTB.COM

Downloaded by: SUCCEEDGRADES | abbieclin@gmail.com


Distribution
Downloaded by Dallen Mae D.ofKadir
this document is illegal
(kadir.dallenmae.d.bcsi@gmail.com)
lOMoARcPSD|35346190

DEWITS FUNDAMENTALStC
uvO
iaN
.cC
om
EP- T
ThSe A
MNarD
ketSpK
l a IcL
eLtoSBFuO
y aRndNSUeR
ll S
yoI u
Nr G
Stu
5dTyHME
atD
erIiaTl ION WILLIAMS TEST BANK

8. An older adult patient with a medical diagnosis of chronic lung disease has developed
pneumonia. She is coughing frequently and expectorating thick, sticky secretions. She is very
short of breath, even with oxygen running, and she is exhausted and says she ―can‘t breathe.‖
Based on this information, an appropriately worded nursing diagnosis for this patient is:
a. Airway clearance, ineffective, related to lung secretions as evidenced by cough and
shortness of breath.
b. Pneumonia, cough, and shortness of breath related to chronic lung disease.
c. Difficulty breathing not relieved by oxygen and evidenced by shortness of breath.
d. Cough and shortness of breath caused by pneumonia, chronic lung disease,
advanced age, and exhaustion.
ANS: A
The nursing diagnosis from the NANDA list is complete with a cause and signs and
symptoms.

DIF: Cognitive Level: Analysis REF: p. 66|Box 5-4


OBJ: Theory #5 TOP: Nursing Diagnosis
KEY: Nursing Process Step: Diagnosis
MSC: NCLEX: Physiological Integrity: Basic Care and Comfort

9. If a patient has several nursing diagnoses, the nurse will first:


a. consult with the doctor regarding which diagnosis is most important.
b. devise nursing interventions for the most quickly solved problems.
c. prioritize the nursing problems according to Maslow‘s hierarchy of needs.
d. review the patient‘s medical prescriptions and other drugs being taken.

ANS: C
Nursing diagnoses (and thus t h e i r Ni nUtRe rSvI NenGtTi oBn. Cs )Om
M u st be prioritized to identify the order of
importance based on Maslow‘s hierarchy.

DIF: Cognitive Level: Analysis REF: p. 65 OBJ: Clinical Practice #4


TOP: Prioritizing KEY: Nursing Process Step: Planning MSC: NCLEX: N/A

10. A patient has a nursing diagnosis of imbalanced nutrition: less than body requirements, related
to mental impairment and decreased intake, as evidenced by increasing confusion and weight
loss of more than 30 pounds over the last 6 months. An appropriate short-term goal for this
patient is to:
a. eat 50% of six small meals every day by the end of 1 week.
b. demonstrate progressive weight gain over 6 months.
c. eat all of the meals prepared during admission.
d. verbalize understanding of caloric needs and intention to eat.

ANS: A
Short-term goals should be realistic and attainable and should have a timeline of 7 to 10 days
before discharge.

DIF: Cognitive Level: Application REF: p. 66 OBJ: Clinical Practice #6


TOP: Expected Outcomes KEY: Nursing Process Step: Planning
MSC: NCLEX: Physiological Integrity: Basic Care and Comfort

11. The nursing diagnoses that has the highest priority is:
a. Mobility, impaired physical, related to muscular weakness as evidenced by the

NURSINGTB.COM

Downloaded by: SUCCEEDGRADES | abbieclin@gmail.com


Distribution
Downloaded by Dallen Mae D.ofKadir
this document is illegal
(kadir.dallenmae.d.bcsi@gmail.com)
lOMoARcPSD|35346190

DEWITS FUNDAMENTALStC
uvO
iaN
.cC
om
EP- T
ThSe A
MNarD
ketSpK
l a IcL
eLtoSBFuO
y aRndNSUeR
ll S
yoI u
Nr G
Stu
5dTyHME
atD
erIiaTl ION WILLIAMS TEST BANK

inability to walk without assistance.


b. Communication, impaired verbal, related to neuromuscular weakness as evidenced
by facial weakness and inability to speak.
c. Imbalanced nutrition: less than body requirements, related to difficulty
swallowing and inadequate food intake as evidenced by weight loss of 10 pounds.
d. Airway clearance, ineffective, related to neuromuscular disorder as evidenced by
choking and coughing while eating.
ANS: D
Choking and aspiration are life-threatening events and take priority over problems such as
weakness, inability to speak, or weight loss.

DIF: Cognitive Level: Analysis REF: p. 65 OBJ: Clinical Practice #4


TOP: Prioritizing KEY: Nursing Process Step: Planning
MSC: NCLEX: Physiological Integrity: Basic Care and Comfort

12. A patient with visual impairment is identified as at-risk for falls related to blindness. An
appropriate intervention would be:
a. assist the patient with feeding herself at the end of the meal.
b. arrange furnishings in room to provide clear pathways and orient the patient to
these.
c. take the patient‘s blood pressure before she gets up in the morning.
d. report any falls immediately to the charge nurse and the doctor.

ANS: B
Providing clear pathways directly reduces the risk of patient falls.

DIF: Cognitive Level: Analysis N U R S I NRGETFB: . C pO. M6 2 OBJ: Clinical Practice #6


TOP: Clinical Planning KEY: Nursing Process Step: Planning
MSC: NCLEX: Safe, Effective Care Environment: Safety and Infection Control

13. The North American Nursing Diagnosis Association–I (NANDA-I) list is revised and updated
every:
a. year.
b. 2 years.
c. 3 years.
d. 5 years.

ANS: B
NANDA-I meets every 2 years to revise and update the list.

DIF: Cognitive Level: Knowledge REF: p. 65 OBJ: Theory #5


TOP: NANDA I KEY: Nursing Process Step: N/A MSC: NCLEX: N/A

14. A nursing care plan consists of:


a. nursing orders for individualized interventions to assist the patient to meet
expected outcomes.
b. orders for diagnostic and therapeutic procedures such as laboratory tests or
radiographs.
c. the health care provider‘s history and physical examination, as well as medical
diagnoses.
d. laboratory and radiograph reports, pathology reports, and the medication record.

NURSINGTB.COM

Downloaded by: SUCCEEDGRADES | abbieclin@gmail.com


Distribution
Downloaded by Dallen Mae D.ofKadir
this document is illegal
(kadir.dallenmae.d.bcsi@gmail.com)
lOMoARcPSD|35346190

DEWITS FUNDAMENTALStC
uvO
iaN
.cC
om
EP- T
ThSe A
MNarD
ketSpK
l a IcL
eLtoSBFuO
y aRndNSUeR
ll S
yoI u
Nr G
Stu
5dTyHME
atD
erIiaTl ION WILLIAMS TEST BANK

ANS: A
The nursing care plan consists of the nursing orders for interventions to address problems and
establish outcomes by which the plan can be evaluated.

DIF: Cognitive Level: Comprehension REF: p. 69 OBJ: Clinical Practice #5


TOP: Nursing Care Plan KEY: Nursing Process Step: Planning
MSC: NCLEX: N/A

15. In an acute care facility, a nursing care plan is usually reviewed and updated:
a. every shift.
b. every 24 hours.
c. once every 3 days.
d. on admission and discharge.

ANS: B
Ongoing assessment, intervention, and evaluation lead to attainment or modification of the
original plan for the patient who is acutely ill. The nursing care plan must be updated every
day to reflect these changes.

DIF: Cognitive Level: Knowledge REF: p. 69 OBJ: Clinical Practice #6


TOP: Nursing Care Plan KEY: Nursing Process Step: Planning
MSC: NCLEX: N/A

16. The nurse takes into consideration that the difference between a sign and a symptom is that a
sign is:
a. subjective data.
b. unreliable because it depends N
onURtrSaInNsGlaTtiBo.C
n.OM
c. can be verified by examination.
d. something a patient reports that is verified by a relative.
ANS: C
Signs are objective data that can be confirmed by examination, assessment, or observation.
Signs are reliable research-based data.

DIF: Cognitive Level: Comprehension REF: p. 58 OBJ: Theory #2


TOP: Assessment (Data Collection) KEY: Nursing Process Step: Assessment
MSC: NCLEX: Physiological Integrity: Basic Care and Comfort

17. The nurse clarifies that nursing orders are also called:
a. goals.
b. qualifiers.
c. interventions.
d. measurement criteria.

ANS: C
Nursing orders are also called nursing interventions and follow the same requirements when
placed in a nursing care plan.

DIF: Cognitive Level: Knowledge REF: p. 64 OBJ: Theory #2


TOP: Nursing Orders KEY: Nursing Process Step: N/A
MSC: NCLEX: N/A

NURSINGTB.COM

Downloaded by: SUCCEEDGRADES | abbieclin@gmail.com


Distribution
Downloaded by Dallen Mae D.ofKadir
this document is illegal
(kadir.dallenmae.d.bcsi@gmail.com)
lOMoARcPSD|35346190

DEWITS FUNDAMENTALStC
uvO
iaN
.cC
om
EP- T
ThSe A
MNarD
ketSpK
l a IcL
eLtoSBFuO
y aRndNSUeR
ll S
yoI u
Nr G
Stu
5dTyHME
atD
erIiaTl ION WILLIAMS TEST BANK

18. The nurse designs the goals for patients in long-term facilities to be:
a. conditional.
b. open ended.
c. based on behavioral norms.
d. long term.
ANS: D
Long-term goals are more appropriate for patients in long-term facilities because they will be
there for an extended period and many of their health problems are chronic.

DIF: Cognitive Level: Comprehension REF: p. 69 OBJ: Theory #7


TOP: Long Term Goals KEY: Nursing Process Step: Planning
MSC: NCLEX: Physiological Integrity: Basic Care and Comfort

19. Standardized Nursing Care Plans can:


a. be documented without alteration.
b. have items altered or deleted.
c. become part of the record without documentation.
d. help the family understand the concept of Nursing Care Plans.
ANS: B
Standardized Nursing Care Plans are generic and need to be altered to become individualized.
They must be documented.

DIF: Cognitive Level: Comprehension REF: p. 69 OBJ: Theory #7


TOP: Assessment (Data Collection) KEY: Nursing Process Step: Assessment
MSC: NCLEX: N/A
NURSINGTB.COM
20. A nurse is caring for a patient with a medical diagnosis of right lower lobe pneumonia. The
patient is expectorating thick green mucus, has an oxygen saturation level of 90%, and has
audible crackles in the base of the right lung. An appropriate nursing diagnosis for this patient
is:
a. Airway clearance, ineffective, related to retained secretions as evidenced by
expectoration of thick green mucus, oxygen saturation level of 90%, and audible
crackles in the base of the right lung.
b. Airway clearance, ineffective, related to right lower lobe pneumonia as evidenced
by expectoration of thick green mucus, oxygen saturation level of 90%, and
audible crackles in the base of the right lung.
c. Right lower lobe pneumonia, related to airway clearance, ineffective, as evidenced
by expectoration of thick green mucus, oxygen saturation level of 90%, and
audible crackles in the base of the right lung.
d. Expectoration of thick green mucus, oxygen saturation level of 90%, and audible
crackles in the base of the right lung related to right lower lobe pneumonia as
evidenced by airway clearance.
ANS: A
The nursing diagnosis is from the NANDA-I list and is complete with a cause and signs and
symptoms. The other answers contain a medical diagnosis of pneumonia, which is
inappropriate.

DIF: Cognitive Level: Analysis REF: p. 65 OBJ: Theory #7


TOP: Nursing Diagnosis KEY: Nursing Process Step: Diagnosis

NURSINGTB.COM

Downloaded by: SUCCEEDGRADES | abbieclin@gmail.com


Distribution
Downloaded by Dallen Mae D.ofKadir
this document is illegal
(kadir.dallenmae.d.bcsi@gmail.com)
lOMoARcPSD|35346190

DEWITS FUNDAMENTALStC
uvO
iaN
.cC
om
EP- T
ThSe A
MNarD
ketSpK
l a IcL
eLtoSBFuO
y aRndNSUeR
ll S
yoI u
Nr G
Stu
5dTyHME
atD
erIiaTl ION WILLIAMS TEST BANK

MSC: NCLEX: Physiological Integrity: Basic Care and Comfort

21. Reginald is a nurse caring for a 56-year-old man who is admitted with an acute MI. As he
completes the initial assessment, he knows that concerning the practice of nursing, the
purpose of the assessment on admission is to:
a. gather data so that the patient‘s response to the treatment can be evaluated.
b. gather data for the health care provider, to make decisions based on the condition
of the patient.
c. establish rapport with the patient so that he/she can feel safe and secure in the
acute health care setting.
d. begin the care plan and set the patient on the road to recovery.
ANS: A

The practice of nursing is concerned with how a patient responds, physiologically and
psychologically, to their disease or disorder, to their treatment(s), their life situation and
environment, etc. In order to determine this, a database containing information about the
patient must be established. It is in this capacity that LPN/LVNs contribute, via data
collection, to the assessment stage of the nursing process.

DIF: Cognitive Level: Comprehension REF: p. 58 OBJ: Theory #1


TOP: Assigning Admission Tasks KEY: Nursing Process Step: Assessment
MSC: NCLEX: N/A

22. Theresa is a nurse caring for a 14-year-old girl who is admitted with an asthma attack. When
she writes the nursing diagnosis statement she includes?
a. Two statements; the problem and the signs and/or the symptoms.
NURSINGTB.COM
b. The medical diagnosis.
c. Her clinical judgment regarding the patient‘s response to the problem.
d. Uses the NANDA-I as the stem and the medical diagnosis as the conclusion.
ANS: C
Most care facilities use a problem statement in care planning that may (or may not) conform
to the NANDA-I terminology. Whatever the terminology used, the nursing diagnosis reflects
the nurse‘s clinical judgment regarding the patient‘s response to an actual or potential health
problem, and is the basis for the nurse‘s plan of care for the patient.

DIF: Cognitive Level: Comprehension REF: p. 65 OBJ: Theory #5


TOP: Nursing Diagnosis KEY: Nursing Process Step: Nursing Diagnosis
MSC: NCLEX: N/A

MULTIPLE RESPONSE

1. The nurse understands that an expected outcome should be: (Select all that apply.)
a. realistic.
b. approved by the health care provider.
c. attainable.
d. within a defined time.
e. included after patient collaboration.

ANS: A, C, D, E

NURSINGTB.COM

Downloaded by: SUCCEEDGRADES | abbieclin@gmail.com


Distribution
Downloaded by Dallen Mae D.ofKadir
this document is illegal
(kadir.dallenmae.d.bcsi@gmail.com)
lOMoARcPSD|35346190

DEWITS FUNDAMENTALStC
uvO
iaN
.cC
om
EP- T
ThSe A
MNarD
ketSpK
l a IcL
eLtoSBFuO
y aRndNSUeR
ll S
yoI u
Nr G
Stu
5dTyHME
atD
erIiaTl ION WILLIAMS TEST BANK

An expected outcome should be realistic and attainable and should have a defined time line
after collaboration with the patient.

DIF: Cognitive Level: Knowledge REF: p. 67 OBJ: Theory #6


TOP: Nursing Process KEY: Nursing Process Step: Planning
MSC: NCLEX: Physiological Integrity: Basic Care and Comfort

2. A nurse is caring for a patient with a nursing diagnosis of impaired physical mobility related
to neurological impairment and muscular weakness. Appropriate interventions for this patient
would include which of the following? (Select all that apply.)
a. Assist with range of motion exercises every 4 hours and as needed.
b. Instruct patient to call for assistance when needing to get out of bed.
c. Apply wrist and ankle restraints to promote safety and prevent falls.
d. Teach about exercises that will strengthen muscles while lying in bed.
e. Ambulate with physical therapy assistance at least three times a day.

ANS: A, B, D, E
The nurse selects appropriate nursing interventions to alleviate the problems and assist the
patient in achieving the expected outcomes. Consider all possible interventions for relief of
the problems and then select those most likely to be effective.

DIF: Cognitive Level: Application REF: p. 68 OBJ: Clinical Practice #5


TOP: Assessment KEY: Nursing Process Step: Implementation
MSC: NCLEX: Physiological Integrity: Basic Care and Comfort

3. Appropriate nursing roles in the initial assessment would include: (Select all that apply.)
a. LPN obtains the vital signs of a new patient.
b. RN performs a complete physN icUalRaSsIN
seGsTsm
B.eCnOtM
.
c. LPN organizes data into a database.
d. RN reviews the patient‘s medical record for past medical/surgical history.
e. LVN contributes ongoing assessments.

ANS: A, B, D, E
The LPN/LVN, under the NFLPN standard, contributes assessments; the RN performs the
physical assessment and medical records review and organizes the database.

DIF: Cognitive Level: Comprehension REF: p. 58 OBJ: Theory #2


TOP: Planning KEY: Nursing Process Step: Planning MSC: NCLEX: N/A

4. Aside from the information obtained from the patient (primary source) in the admission
interview, the nurse will also access: (Select all that apply.)
a. the patient‘s family.
b. a reliable and up-to-date reference book.
c. the admission note.
d. the health care provider‘s history and physical.
e. an observation of the patient for nonverbal clues.

ANS: A, C, D, E
The nurse conducting the interview uses information from the patient‘s family, from the
health care provider‘s s admission note and history and physical, and from personal
observation of the patient.

NURSINGTB.COM

Downloaded by: SUCCEEDGRADES | abbieclin@gmail.com


Distribution
Downloaded by Dallen Mae D.ofKadir
this document is illegal
(kadir.dallenmae.d.bcsi@gmail.com)
lOMoARcPSD|35346190

DEWITS FUNDAMENTALStC
uvO
iaN
.cC
om
EP- T
ThSe A
MNarD
ketSpK
l a IcL
eLtoSBFuO
y aRndNSUeR
ll S
yoI u
Nr G
Stu
5dTyHME
atD
erIiaTl ION WILLIAMS TEST BANK

DIF: Cognitive Level: Comprehension REF: p. 58 OBJ: Theory #2


TOP: Assessment KEY: Nursing Process Step: Assessment
MSC: NCLEX: Physiological Integrity: Basic Care and Comfort

5. A nursing diagnosis identifies: (Select all that apply.)


a. patient‘s response to illness.
b. related signs and symptoms.
c. underlying medical diagnosis.
d. causative factors.
e. potential risk for health problems.

ANS: A, B, D, E
Defining characteristics of nursing diagnosis includes the patient‘s response to illness and the
causative factors. Signs and symptoms must also be identified for a nurse to select an
appropriate nursing diagnosis. Medical diagnoses label an illness; nursing diagnoses are
independent of medical diagnoses.

DIF: Cognitive Level: Comprehension REF: p. 65 OBJ: Theory #5


TOP: Defining Characteristics KEY: Nursing Process Step: Planning
MSC: NCLEX: N/A

6. The statements that are correctly stated as expected outcomes are: (Select all that apply.)
a. patient will be able to void in the bathroom independently.
b. patient will be able to ambulate using a walker independently within 3 days.
c. the nurse will assist the patient to the bathroom three times a day.
d. patient will perform active range of motion (ROM) of her upper extremities
independently every 4 hours.
N UhRo SmI N
e. the family will bring food fr om e Gt oT Bi m
. CpOr M
ove patient appetite.
ANS: B, D
Expected outcomes need to have a time frame and be measurable. Ambulating with a walker
within 3 days and performing ROM independently for 4 hours are both measurable outcomes
with clear time frames. The outcome of voiding independently does not have a time frame.
Assisting the patient to the bathroom is a nursing intervention.

DIF: Cognitive Level: Comprehension REF: p. 66|p. 67 OBJ: Theory #6


TOP: Expected Outcomes KEY: Nursing Process Step: Planning
MSC: NCLEX: N/A

7. The nurse should make a point when closing the initial interview to: (Select all that apply.)
a. develop rapport.
b. summarize the problems discussed.
c. thank the patient for his or her time.
d. discuss the nursing goals associated with nursing diagnoses.
e. give a copy of the nursing care plan to the patient.

ANS: B, C
The nurse should summarize the problems discussed, thank the patient for his or her time, and
explain what happens next and when the nurse will return.

DIF: Cognitive Level: Comprehension REF: p. 59 OBJ: Theory #1


TOP: Nursing Process KEY: Nursing Process Step: Assessment

NURSINGTB.COM

Downloaded by: SUCCEEDGRADES | abbieclin@gmail.com


Distribution
Downloaded by Dallen Mae D.ofKadir
this document is illegal
(kadir.dallenmae.d.bcsi@gmail.com)
lOMoARcPSD|35346190

DEWITS FUNDAMENTALStC
uvO
iaN
.cC
om
EP- T
ThSe A
MNarD
ketSpK
l a IcL
eLtoSBFuO
y aRndNSUeR
ll S
yoI u
Nr G
Stu
5dTyHME
atD
erIiaTl ION WILLIAMS TEST BANK

MSC: NCLEX: Physiological Integrity: Basic Care and Comfort

8. The seven domains of the Nursing Interventions Classification (NIC) taxonomy include:
(Select all that apply.)
a. community.
b. health system.
c. socioeconomic level.
d. safety.
e. behavioral.
ANS: A, B, D, E
The seven domains of the NIC taxonomy are physiological: basic; physiological: complex;
behavioral; safety; family; health system; and community.

DIF: Cognitive Level: Knowledge REF: p. 68 OBJ: Theory #5


TOP: NIC KEY: Nursing Process Step: N/A MSC: NCLEX: N/A

9. The purpose of the Nursing Outcomes Classification (NOC) is to: (Select all that apply.)
a. validate classification by field test.
b. identify labels.
c. provide language labels for desired outcomes.
d. generate a readymade nursing care plan for a patient.
e. identify patient outcomes and indicators.
ANS: A, B, C, E
The purpose of NOC is to provide language labels to help identify and classify patient
outcomes and validate classifications by field testing.
NURSINGTB.COM
DIF: Cognitive Level: Knowledge REF: p. 68 OBJ: Theory #6
TOP: NOC KEY: Nursing Process Step: N/A MSC: NCLEX: N/A

COMPLETION

1. Conclusions that have been made based on observed data are .

ANS:
inferences

Inferences are conclusions made based on observed data.

DIF: Cognitive Level: Knowledge REF: p. 65 OBJ: Theory #6


TOP: Inferences KEY: Nursing Process Step: N/A MSC: NCLEX: N/A

NURSINGTB.COM

Downloaded by: SUCCEEDGRADES | abbieclin@gmail.com


Distribution
Downloaded by Dallen Mae D.ofKadir
this document is illegal
(kadir.dallenmae.d.bcsi@gmail.com)
lOMoARcPSD|35346190

DEWITS FUNDAMENTALStC
uvO
iaN
.cC
om
EP- T
ThSe A
MNarD
ketSpK
l a IcL
eLtoSBFuO
y aRndNSUeR
ll S
yoIu
Nr GStu5dTyHME
atD
erIiaTl ION WILLIAMS TEST BANK

Chapter 06: Implementation and Evaluation


Williams: deWit's Fundamental Concepts and Skills for Nursing, 8th Edition

MULTIPLE CHOICE

1. The nurse is aware that one of the time flexible tasks to be accomplished would be:
a. administering daily insulin 30 minutes before breakfast.
b. taking the patient‘s vital signs once a day.
c. weighing the patient before breakfast.
d. monitoring a critical patient‘s vital signs every 15 minutes.

ANS: B
Daily vital signs can be taken at any time during the day, whereas the other tasks mentioned
have a time constraint.

DIF: Cognitive Level: Application REF: p. 73 OBJ: Theory #2


TOP: Care Delivery KEY: Nursing Process Step: Planning
MSC: NCLEX: Physiological Integrity: Basic Care and Comfort

2. Prior to the nurse implementing a nursing procedure for a patient, the nurse should initially:
a. question the rationale for the procedure.
b. perform a physical assessment of the patient.
c. check the agency manual for the procedure.
d. mentally review the procedure.

ANS: D
Reviewing the procedure, c h e c k i nNg Ut Rh eS I m
NGa nT uBa. Cl O
ifMuncertain, confirming the order for the
procedure, assessing that there is no interference with the completion of the procedure, and
identifying the patient are standard steps in deliberative nursing action.

DIF: Cognitive Level: Application REF: p. 76|Box 6-2


OBJ: Theory #1 TOP: Care Delivery
KEY: Nursing Process Step: Planning
MSC: NCLEX: Physiological Integrity: Basic Care and Comfort

3. At the 7:00 AM handoff report, the nurse receives the report that patient A had a sleepless
night related to pain and just fell asleep after an increased pain medication administration 1/2
hour ago. Patient B, who is scheduled for surgery at 8:30 AM, is also sleeping. How would an
organized nurse plan the early morning activities?
a. Wake patient A for breakfast.
b. Perform time flexible tasks that can be done while both patients sleep.
c. Prepare patient B now; allow patient A to sleep.
d. Assign a nursing assistant to wake and help feed patient A.

ANS: C
Setting priorities and identifying time fixed tasks would indicate that patient B needs to be
prepared for surgery. Patient A needs to sleep.

DIF: Cognitive Level: Analysis REF: p. 73 OBJ: Theory #1


TOP: Care Delivery KEY: Nursing Process Step: Implementation
MSC: NCLEX: Physiological Integrity: Basic Care and Comfort

https://www.coursehero.com/file/53537151/c6pdf/

NURSINGTB.COM

Downloaded by: SUCCEEDGRADES | abbieclin@gmail.com


Distribution
Downloaded by Dallen Mae D.ofKadir
this document is illegal
(kadir.dallenmae.d.bcsi@gmail.com)
lOMoARcPSD|35346190

DEWITS FUNDAMENTALStC
uvO
iaN
.cC
om
EP- T
ThSe A
MNarD
ketSpK
l a IcL
eLtoSBFuO
y aRndNSUeR
ll S
yoIu
Nr GStu5dTyHME
atD
erIiaTl ION WILLIAMS TEST BANK

4. Preparing a patient for a diagnostic test, and telling the patient what to expect during and after
the test, is considered:
a. an independent nursing action.
b. the doctor‘s responsibility.
c. a dependent nursing action that requires the doctor‘s authorization.
d. an interdependent nursing action.

ANS: A
Patient education is an independent nursing action.

DIF: Cognitive Level: Comprehension REF: p. 74 OBJ: Theory #2


TOP: Patient Education KEY: Nursing Process Step: Implementation
MSC: NCLEX: Physiological Integrity: Basic Care and Comfort

5. The nurse explains that a multidisciplinary step-by-step approach to patient care is:
a. documented in the nursing care plan in the patient‘s medical record.
b. not used often since managed care became part of health care.
c. referred to as a clinical pathway and is used instead of a nursing care plan.
d. more expensive than the traditional separation of health care services.
ANS: C
An outgrowth of managed care has been collaborative models of care called clinical
pathways.

DIF: Cognitive Level: Knowledge REF: p. 74 OBJ: Theory #1


TOP: Clinical Pathways KEY: Nursing Process Step: Implementation
MSC: NCLEX: Safe, Effective CareNEUnRvSirIoNnGmTeBn.tC: O
CMoordinated Care

6. The nurse documents interventions periodically during the shift in nurses‘ notes primarily to:
a. validate the number of nonlicensed personnel who interact with the patient.
b. indicate that the nursing care plan has been implemented.
c. briefly summarize activities during the shift.
d. confirm that the nursing diagnoses in the care plan are appropriate.
ANS: B
The nursing care must be documented in the nurses‘ notes to prove that interventions were
implemented. In some facilities documentation is required at least every 2 hours.

DIF: Cognitive Level: Comprehension REF: p. 76 OBJ: Theory #3


TOP: Documentation KEY: Nursing Process Step: Implementation
MSC: NCLEX: Safe, Effective Care Environment: Coordinated Care

7. The nurse compares actual nursing outcomes to the expected nursing outcomes in order to:
a. prepare the patient to be discharged from the facility.
b. determine if the patient‘s health problems have been treated.
c. calculate charges for nursing services during the patient‘s hospital stay.
d. determine if progress is made or to determine if revisions are needed.

ANS: D
Evaluation of patient responses to treatment and progress toward goals is performed
continuously so that the nursing care plan may be modified if needed.

https://www.coursehero.com/file/53537151/c6pdf/

NURSINGTB.COM

Downloaded by: SUCCEEDGRADES | abbieclin@gmail.com


Distribution
Downloaded by Dallen Mae D.ofKadir
this document is illegal
(kadir.dallenmae.d.bcsi@gmail.com)
lOMoARcPSD|35346190

DEWITS FUNDAMENTALStC
uvO
iaN
.cC
om
EP- T
ThSe A
MNarD
ketSpK
l a IcL
eLtoSBFuO
y aRndNSUeR
ll S
yoIu
Nr GStu5dTyHME
atD
erIiaTl ION WILLIAMS TEST BANK

DIF: Cognitive Level: Comprehension REF: p. 77 OBJ: Theory #5


TOP: Outcomes KEY: Nursing Process Step: Evaluation
MSC: NCLEX: Physiological Integrity: Basic Care and Comfort

8. The general rule is that the initial care plan for a patient is:
a. developed by an RN in an acute care setting.
b. used as the basis of care throughout a hospital stay without alteration.
c. completed on the day of admission.
d. developed by the primary care provider and incorporated into the nursing care.

ANS: A
An RN is responsible for developing the plan of care for patients in acute care settings. An
LPN may begin the care plan in a skilled nursing facility and will collaborate with the RN for
revision. The nursing care plan will be revised frequently as the patient‘s condition changes.

DIF: Cognitive Level: Comprehension REF: p. 79 OBJ: Theory #2


TOP: Care Planning KEY: Nursing Process Step: Planning
MSC: NCLEX: Safe, Effective Care Environment: Coordinated Care

9. The nurse is aware that the nursing audit is a valuable process used to:
a. determine whether a particular patient received the care indicated in the nursing
care plan.
b. evaluate whether nursing care for a group of patients meets the standards of care in
that facility.
c. determine the cost of nursing care in the hospital in order to set rates for daily care.
d. identify careless or negligent nursing care to protect the facility from lawsuits.
NURSINGTB.COM
ANS: B
Nursing audits are performed to improve nursing practice by checking a group of patient
records for how well particular standards were met and standards of care were being used.

DIF: Cognitive Level: Knowledge REF: p. 79 OBJ: Theory #6


TOP: Nursing Audits KEY: Nursing Process Step: Planning
MSC: NCLEX: Safe, Effective Care Environment: Coordinated Care

10. The nurse evaluates that the patient has met the outcome of feeding himself independently.
The nurse should:
a. inactivate the nursing diagnosis from the care plan.
b. notify the primary care provider that the patient can now feed himself.
c. document the ability to self-feed and mark the nursing diagnosis as resolved.
d. inform the RN to document the self-feeding and to cancel the nursing diagnosis.
ANS: C
The LPN should document the meeting of the outcome and mark the nursing diagnosis as
―resolved.‖

DIF: Cognitive Level: Application REF: p. 77 OBJ: Theory #6


TOP: Nursing Care Plan Revision KEY: Nursing Process Step: Intervention
MSC: NCLEX: Safe, Effective Care Environment: Coordinated Care

https://www.coursehero.com/file/53537151/c6pdf/

NURSINGTB.COM

Downloaded by: SUCCEEDGRADES | abbieclin@gmail.com


Distribution
Downloaded by Dallen Mae D.ofKadir
this document is illegal
(kadir.dallenmae.d.bcsi@gmail.com)
lOMoARcPSD|35346190

DEWITS FUNDAMENTALStC
uvO
iaN
.cC
om
EP- T
ThSe A
MNarD
ketSpK
l a IcL
eLtoSBFuO
y aRndNSUeR
ll S
yoIu
Nr GStu5dTyHME
atD
erIiaTl ION WILLIAMS TEST BANK

11. An example of an appropriately worded nursing goal or outcome for the nursing diagnosis of
―Risk for falls related to weakness‖ would be:
a. nurse will assist the patient to the bathroom every 2 hours.
b. patient will be free of injury from falls.
c. patient will call for assistance when ambulating for the next week.
d. nurse will keep room well lit 24 hours a day.

ANS: C
An appropriately worded outcome is a patient centered, measurable, and time defined goal
based on a nursing diagnosis.

DIF: Cognitive Level: Application REF: p. 79|Box 6-3


OBJ: Theory #5 TOP: Expected Outcomes
KEY: Nursing Process Step: Evaluation
MSC: NCLEX: Physiological Integrity: Basic Care and Comfort

12. Nurses design interventions that are appropriate for a patient that are:
a. based on the primary care provider‘s orders and the medical diagnosis.
b. expected to help the patient meets the goals most quickly.
c. used to evaluate whether the nursing care plan should be revised.
d. based on cost effectiveness and staff availability.
ANS: B
Nursing interventions are based on nursing diagnoses and are those most likely to assist the
patient in meeting outcomes related to those diagnoses.

DIF: Cognitive Level: Comprehension REF: p. 79|Box 6-3


OBJ: Theory #2 TOP: Care DeNl U
i vRerSyI NGT B.C OM
KEY: Nursing Process Step: Planning
MSC: NCLEX: Safe, Effective Care Environment: Coordinated Care

13. Before performing a catheterization, the inexperienced nurse should:


a. close the door or curtains to provide the patient with privacy.
b. provide necessary education and explanation of the procedure to the patient.
c. observe rules of Standard Precautions to protect herself from exposure to blood or
body fluids.
d. review the agency‘s procedure manual for the accepted way of performing the
procedure.
ANS: D
Reviewing the procedure manual should occur before the inexperienced nurse explains to the
patient, provides privacy, or observes Standard Precautions.

DIF: Cognitive Level: Application REF: p. 75 OBJ: Clinical Practice #2


TOP: Standards for All Nursing Procedures
KEY: Nursing Process Step: Implementation
MSC: NCLEX: Physiological Integrity: Basic Care and Comfort

14. During morning care in a skilled nursing facility, the student nurse notices that the patient
who is at risk for impaired skin integrity has developed a small open area on his sacrum. To
best address this situation, the student would first:
a. position the patient to lie on his side, document it, and inform the head nurse.

https://www.coursehero.com/file/53537151/c6pdf/

NURSINGTB.COM

Downloaded by: SUCCEEDGRADES | abbieclin@gmail.com


Distribution
Downloaded by Dallen Mae D.ofKadir
this document is illegal
(kadir.dallenmae.d.bcsi@gmail.com)
lOMoARcPSD|35346190

DEWITS FUNDAMENTALStC
uvO
iaN
.cC
om
EP- T
ThSe A
MNarD
ketSpK
l a IcL
eLtoSBFuO
y aRndNSUeR
ll S
yoIu
Nr GStu5dTyHME
atD
erIiaTl ION WILLIAMS TEST BANK

b. position the patient on his side and encourage him to massage around the area.
c. report to the primary care provider so that the nursing care plan can be revised.
d. tell the nursing assistant to change the patient‘s position every 2 hours.
ANS: A
This change in the patient‘s position with documentation is the initial intervention. The
discovery of an open lesion requires a change in the nursing plan.

DIF: Cognitive Level: Analysis REF: p. 77 OBJ: Theory #2


TOP: Care Delivery KEY: Nursing Process Step: Implementation
MSC: NCLEX: Physiological Integrity: Basic Care and Comfort

15. A review of a patient‘s nursing care plan before beginning care allows the nurse to:
a. make revisions in the plan as indicated by the shift report.
b. use critical thinking skills to organize care for the patient.
c. begin nursing interventions without needing an initial assessment.
d. skip the shift report and begin with the initial assessment.

ANS: B
Reviewing the patient‘s care plan gives the nurse a starting point for organizing care.

DIF: Cognitive Level: Comprehension REF: p. 75 OBJ: Theory #2


TOP: Planning KEY: Nursing Process Step: Planning
MSC: NCLEX: Physiological Integrity: Basic Care and Comfort

16. The nurse giving a patient a back massage is performing an intervention considered to be:
a. a dependent nursing action.
b. an independent nursing actionN . URSINGTB.COM
c. an interdependent nursing action.
d. a semi-dependent nursing action.
ANS: B
An independent nursing action does not require a primary care provider‘s order, but it does
require critical thinking and nursing judgment. Giving a back massage would be an
independent nursing action.

DIF: Cognitive Level: Comprehension REF: p. 74 OBJ: Theory #2


TOP: Care Delivery KEY: Nursing Process Step: Implementation
MSC: NCLEX: Physiological Integrity: Basic Care and Comfort

17. The nurse administering a medication to a patient is performing an intervention that is:
a. an independent nursing action.
b. an interdependent nursing action.
c. a semi-dependent nursing action.
d. a dependent nursing action.

ANS: D
The administration of a medication is a dependent nursing action because giving medication
requires a primary care provider‘s order.

DIF: Cognitive Level: Comprehension REF: p. 74 OBJ: Theory #2


TOP: Care Delivery KEY: Nursing Process Step: Implementation
MSC: NCLEX: Physiological Integrity: Basic Care and Comfort

https://www.coursehero.com/file/53537151/c6pdf/

NURSINGTB.COM

Downloaded by: SUCCEEDGRADES | abbieclin@gmail.com


Distribution
Downloaded by Dallen Mae D.ofKadir
this document is illegal
(kadir.dallenmae.d.bcsi@gmail.com)
lOMoARcPSD|35346190

DEWITS FUNDAMENTALStC
uvO
iaN
.cC
om
EP- T
ThSe A
MNarD
ketSpK
l a IcL
eLtoSBFuO
y aRndNSUeR
ll S
yoIu
Nr GStu5dTyHME
atD
erIiaTl ION WILLIAMS TEST BANK

18. The nurse caring for a group of patients would show cultural sensitivity to assign an older
male nursing assistant to the care of:
a. a 45-year-old white male patient with uncontrolled diabetes.
b. a 50-year-old Hispanic man with a broken leg.
c. a 55-year-old Japanese man with irritable bowel syndrome.
d. a 60-year-old Muslim woman with pneumonia.

ANS: C
Older Japanese men may resist care given by a younger person or a female.

DIF: Cognitive Level: Analysis REF: p. 75 OBJ: Theory #1


TOP: Cultural Considerations KEY: Nursing Process Step: Planning
MSC: NCLEX: Psychosocial Integrity: Coping and Adaptation

19. In assigning tasks to the nursing assistant, the nurse could appropriately select:
a. range of motion exercises to lower limbs.
b. sterile dressing change on a leg wound.
c. postoperative education to a post-hysterectomy patient.
d. witnessing of the signature on an operative permit.
ANS: A
Range of motion exercises may be provided by nursing assistants, physical therapy aides, or
restorative aides. The nurse performs any invasive procedure, legal document witnessing, and
any sterile procedure.

DIF: Cognitive Level: Application REF: p. 75 OBJ: Theory #3


TOP: Delegation KEY: NursingNPUrRoSc eI NssGSTtBep. C: OPM
lanning
MSC: NCLEX: Physiological Integrity: Basic Care and Comfort

20. The nurse is assessing a patient who just returned from a bowel resection 1 hour ago. The
nurse notes a dressing over the suture line that is wet with sero sanguineous drainage. The
nurse should initially:
a. perform a sterile dressing change.
b. document and report the wet dressing to the charge nurse.
c. reinforce the wet dressing and document.
d. place a towel on the bed and turn the patient to the operated side.
ANS: C
The general rule is that the initial dressing change is performed by the surgeon who will give
further orders pertinent to future dressing changes. The dressing should be reinforced with
sterile materials; findings should be documented and reported to the charge nurse.

DIF: Cognitive Level: Analysis REF: p. 74 OBJ: Theory #3


TOP: Initial Dressing Change KEY: Nursing Process Step: Implementation
MSC: NCLEX: Physiological Integrity: Basic Care and Comfort

MULTIPLE RESPONSE

https://www.coursehero.com/file/53537151/c6pdf/

NURSINGTB.COM

Downloaded by: SUCCEEDGRADES | abbieclin@gmail.com


Distribution
Downloaded by Dallen Mae D.ofKadir
this document is illegal
(kadir.dallenmae.d.bcsi@gmail.com)
lOMoARcPSD|35346190

DEWITS FUNDAMENTALStC
uvO
iaN
.cC
om
EP- T
ThSe A
MNarD
ketSpK
l a IcL
eLtoSBFuO
y aRndNSUeR
ll S
yoIu
Nr GStu5dTyHME
atD
erIiaTl ION WILLIAMS TEST BANK

1. The nurse coming on duty has received a report that an IV of 1000 mL of 5% glucose in 0.9%
normal saline is running at a rate of 50 mL an hour to be followed by another 1000 mL to be
run at the same rate. The reporting nurse states that the second IV should be hung at 9:00 AM.
The prudent nurse should: (Select all that apply.)
a. hang the next 1000 mL when the first is finished.
b. check to label on the present IV.
c. confirm the flow rate.
d. check the order for the IVs.
e. speed up the flow so that the IV will be completed by 9:00 AM.

ANS: B, C, D
The nurse should check the order and the flow rate, and the amount and type of fluid to follow
for accuracy, and not depend on the handoff report.

DIF: Cognitive Level: Analysis REF: p. 73 OBJ: Theory #2


TOP: Care Delivery KEY: Nursing Process Step: Implementation
MSC: NCLEX: Physiological Integrity: Basic Care and Comfort

2. The purpose of the evaluation step of the nursing process is to: (Select all that apply.)
a. determine if outcomes have been reached and the goals are met.
b. compare actual outcomes with expected outcomes.
c. identify inefficient care given by assigned staff.
d. confirm that nursing interventions are effective.
e. ensure that the facility has not put itself at risk for litigation.

ANS: A, B, D
Evaluation attempts to determine if the outcomes have been reached and that the interventions
being used are effective. Eval ua ti oNnU aRlSs IoNdGeTmB o. CnOstMr a t e s if the actual outcomes agree with the
expected outcomes in the nursing care plan.

DIF: Cognitive Level: Comprehension REF: p. 77 OBJ: Theory #5


TOP: Nursing Process KEY: Nursing Process Step: Evaluation
MSC: NCLEX: Physiological Integrity: Basic Care and Comfort

3. Standards of care are set by: (Select all that apply.)


a. the state‘s nurse practice act.
b. professional medical association standards.
c. the facility‘s policies and procedures.
d. the primary care provider in charge of the patient‘s treatment.
e. the director of nurses and the agency administrator.
ANS: A, B, C
Standards of care are set by the state‘s nursing practice act, professional association standards,
and the facility‘s policies and procedures.

DIF: Cognitive Level: Comprehension REF: p. 75 OBJ: Theory #3


TOP: Standards of Care KEY: Nursing Process Step: Planning
MSC: NCLEX: N/A

COMPLETION

https://www.coursehero.com/file/53537151/c6pdf/

NURSINGTB.COM

Downloaded by: SUCCEEDGRADES | abbieclin@gmail.com


Distribution
Downloaded by Dallen Mae D.ofKadir
this document is illegal
(kadir.dallenmae.d.bcsi@gmail.com)
lOMoARcPSD|35346190

DEWITS FUNDAMENTALStC
uvO
iaN
.cC
om
EP- T
ThSe A
MNarD
ketSpK
l a IcL
eLtoSBFuO
y aRndNSUeR
ll S
yoIu
Nr GStu5dTyHME
atD
erIiaTl ION WILLIAMS TEST BANK

1. The agency-wide process that takes into consideration nursing audits and compliance to
standards of every department is the .

ANS:
outcome-based quality improvement

The outcome-based quality improvement is the agency-wide evaluation to determine if patient


needs are being met according to standards in every department of the facility.

DIF: Cognitive Level: Knowledge REF: p. 77 OBJ: Theory # 6


TOP: Outcome-Based Quality Improvement KEY: Nursing Process Step: N/A
MSC: NCLEX: N/A

2. When an agency is using a clinical pathway/care map protocol of health care provision, there
is no need for a .

ANS:
nursing care plan

The nursing care plan is not part of the patient‘s medical record when an interdisciplinary care
approach is used.

DIF: Cognitive Level: Comprehension REF: p. 74 OBJ: Theory #2


TOP: Clinical Pathway KEY: Nursing Process Step: Planning
MSC: NCLEX: N/A

NURSINGTB.COM

https://www.coursehero.com/file/53537151/c6pdf/

NURSINGTB.COM

Downloaded by: SUCCEEDGRADES | abbieclin@gmail.com


Distribution
Downloaded by Dallen Mae D.ofKadir
this document is illegal
(kadir.dallenmae.d.bcsi@gmail.com)
lOMoARcPSD|35346190

DEWITS FUNDAMENTALStC
uvO
iaN
.cC
om
EP- T
ThSe A
MNarD
ketSpK
l a IcL
eLtoSBFuO
y aRndNSUeR
ll S
yoI u
Nr G
Stu
5dTyHME
atD
erIiaTl ION WILLIAMS TEST BANK

Chapter 07: Documentation of Nursing Care


Williams: deWit's Fundamental Concepts and Skills for Nursing, 8th Edition

MULTIPLE CHOICE

1. The nurse is with a patient who complains of severe pain, documents every 15 minutes about
the steps taken to try to relieve the pain (without success). The nurse also documents the time
and content of two calls made to the patient‘s primary care provider requesting that the
primary care provider examines the patient for unexpected complications. This documentation
by the nurse is likely to:
a. cause the primary care provider to come to the attention of the hospital
administration.
b. be questioned by the nurse‘s supervisor for time inefficiency.
c. be used against the nurse if a lawsuit results, because it proves the nurse was not
able to relieve the pain.
d. justify insurance reimbursement for an extended duration of hospitalization for the
patient.
ANS: D
Documentation of complications or a patient‘s changing condition is used by insurance
companies to justify payments for hospitalization. Documentation also serves as evidence of
standards of care in a court of law.

DIF: Cognitive Level: Application REF: p. 84 OBJ: Theory #4


TOP: Purposes of Documentation KEY: Nursing Process Step: Assessment
MSC: NCLEX: Physiological In t e g rNi t U
y :RBS aI Ns iGc TCBa.rCeOaM
nd Comfort

2. A patient who is very angry and is leaving the hospital against medical advice (AMA)
demands to have the medical record to take, because it is her personal property. An
appropriate response would be:
a. ―Certainly. This hospital doesn‘t need to keep it if you are leaving and will not be
returning here.‖
b. ―You are entitled to the information in your medical record, but the medical record
is the property of the hospital. I will see about having a copy made for you.‖
c. ―The information in your medical record is confidential, and you cannot leave this
facility with it.‖
d. ―Because you are leaving against the medical advice of your primary care
provider, you may not have the medical record.‖
ANS: B
The medical record is the property of the facility, but the patient has a legal right to the
information in it even if she is leaving AMA.

DIF: Cognitive Level: Application REF: p. 86 OBJ: Theory #3


TOP: The Medical Record KEY: Nursing Process Step: N/A
MSC: NCLEX: N/A

3. A student nurse is assigned to a clinical unit on which one of the patients is a nationally
known celebrity. The student reads the chart to find out why the celebrity is being treated. The
student who is not the assigned caregiver is:

NURSINGTB.COM

Downloaded by: SUCCEEDGRADES | abbieclin@gmail.com


Distribution
Downloaded by Dallen Mae D.ofKadir
this document is illegal
(kadir.dallenmae.d.bcsi@gmail.com)
lOMoARcPSD|35346190

DEWITS FUNDAMENTALStC
uvO
iaN
.cC
om
EP- T
ThSe A
MNarD
ketSpK
l a IcL
eLtoSBFuO
y aRndNSUeR
ll S
yoI u
Nr G
Stu
5dTyHME
atD
erIiaTl ION WILLIAMS TEST BANK

a. motivated to learn about the health problem of this patient and is appropriately
seeking knowledge during his clinical experience.
b. doing appropriate research about nursing care as long as information is not
divulged.
c. violating the confidentiality of the patient‘s record.
d. neglecting the assigned patient load and should read the unassigned patient‘s
medical record only after his assigned work is completed.
ANS: C
A person reading a patient‘s chart who is not involved in the patient‘s care is in violation of
confidentiality. Protecting the patient‘s privacy is of prime importance.

DIF: Cognitive Level: Comprehension REF: p. 92 OBJ: Theory #3


TOP: The Medical Record KEY: Nursing Process Step: N/A
MSC: NCLEX: N/A

4. A patient with a nursing diagnosis of Skin integrity, impaired, related to surgery as evidenced
by disruption of skin surface has the following nursing documentation: ―Incision clean, dry,
intact. No pain or tenderness. Instructed to keep area dry, may wear light dressing to protect
from clothing. Verbalizes understanding of wound care and ability to manage at home.
Wound healing without complication.‖ This documentation is:
a. an example of charting by exception.
b. evidence of the use of the nursing process.
c. using the problem-oriented medical record (POMR) format.
d. usually entered on a flow sheet for treatments and vital signs.
ANS: B
The nursing process is evident in tNhUisRdSoINcG
umTBe.nCtO
atM
ion. Assessment, interventions, and
evaluation are all noted.

DIF: Cognitive Level: Analysis REF: p. 92 OBJ: Theory #2


TOP: Methods of Charting KEY: Nursing Process Step: Implementation
MSC: NCLEX: Physiological Integrity: Basic Care and Comfort

5. Which nursing assessment is an example of brevity and clarity while meeting legal
guidelines?
a. ―4 cm reddened area over sacrum. Skin intact, warm, and dry.‖
b. ―Taking fluids poorly, but more than yesterday.‖
c. ―Apparently comfortable all night. Offers no complaints of pain.‖
d. ―Patient says she is still slightly nauseated, would like to try some toast and tea.‖
ANS: A
Provision of specific objective data—size, location, and characteristics of the patient‘s
skin—is clear and brief and informative.

DIF: Cognitive Level: Comprehension REF: p. 95 OBJ: Clinical Practice #2


TOP: The Charting Process KEY: Nursing Process Step: Implementation
MSC: NCLEX: Physiological Integrity: Basic Care and Comfort

6. A nurse enters a notation in a patient‘s medical record but then discovers that the notation was
made in the wrong chart. The nurse correctly:
a. draws a single line through the notation so that it is still readable and writes

NURSINGTB.COM

Downloaded by: SUCCEEDGRADES | abbieclin@gmail.com


Distribution
Downloaded by Dallen Mae D.ofKadir
this document is illegal
(kadir.dallenmae.d.bcsi@gmail.com)
lOMoARcPSD|35346190

DEWITS FUNDAMENTALStC
uvO
iaN
.cC
om
EP- T
ThSe A
MNarD
ketSpK
l a IcL
eLtoSBFuO
y aRndNSUeR
ll S
yoI u
Nr G
Stu
5dTyHME
atD
erIiaTl ION WILLIAMS TEST BANK

―mistaken entry,‖ his signature, and the date and time.


b. removes the page on which the error is located and documents the other correct
notes.
c. blacks out the note to protect the confidentiality of the patient about whom it was
written and writes in the margin ―wrong patient,‖ his signature, and the date and
time.
d. whites out the wrong entry and writes the note in the chart of the correct patient.
ANS: A
When an error is made, no attempt to hide or obliterate the error should be made, because this
may be questioned in a court of law.

DIF: Cognitive Level: Application REF: p. 94|Box 7-4


OBJ: Theory #6 TOP: Charting Error Corrections
KEY: Nursing Process Step: Implementation
MSC: NCLEX: Physiological Integrity: Basic Care and Comfort

7. A resident in a skilled nursing facility for a short term rehabilitation following a hip
replacement says to the nurse, ―I don‘t want to have you draw any more blood for those
useless tests.‖ When the nurse fails to convince the patient to have the blood drawn, the most
appropriate documentation would be:
a. ―Refuses to have blood drawn. Doctor notified.‖
b. ―Refuses to have blood drawn; says tests are ‗useless.‘ Doctor notified.‖
c. ―Doctor notified of failure to draw ordered blood work.‖
d. ―Blood not drawn because tests are no longer desired by patient.‖
ANS: B
When a patient refuses a t reatm enNt , Ut hReS InNuGr TseB .sChOo M
u l d document the exact words of the patient
regarding why the patient is refusing care.

DIF: Cognitive Level: Application REF: p. 94|Box 7-4


OBJ: Clinical Practice #2 TOP: What to Document
KEY: Nursing Process Step: Implementation
MSC: NCLEX: Physiological Integrity: Basic Care and Comfort

8. A clinic nurse is documenting in a patient medical record about the pain that brought the
patient to seek medical attention. The best description is:
a. ―Abdominal pain, unrelieved by antacids. Had spaghetti, salad, coffee, and ice
cream cake for lunch.‖
b. ―Severe pain around umbilicus, unable to sleep because of pain. Started
approximately 2 hours after lunch.‖
c. ―Pain at level of 7 to 8. Nothing has relieved or lessened pain, it just keeps getting
worse.‖
d. ―Periumbilical sharp pain at pain level of 7 to 8 for last 3 hours, started 2 hours
after lunch. No relief from antacids.‖
ANS: D
When documenting a sign or symptom, the nurse should include the quality (levels 7 to 8),
chronology (after lunch, last 3 hours), and aggravating or alleviating factors, as well as
associated symptoms.

DIF: Cognitive Level: Application REF: p. 94|Box 7-2

NURSINGTB.COM

Downloaded by: SUCCEEDGRADES | abbieclin@gmail.com


Distribution
Downloaded by Dallen Mae D.ofKadir
this document is illegal
(kadir.dallenmae.d.bcsi@gmail.com)
lOMoARcPSD|35346190

DEWITS FUNDAMENTALStC
uvO
iaN
.cC
om
EP- T
ThSe A
MNarD
ketSpK
l a IcL
eLtoSBFuO
y aRndNSUeR
ll S
yoI u
Nr G
Stu
5dTyHME
atD
erIiaTl ION WILLIAMS TEST BANK

OBJ: Clinical Practice #2 TOP: The Charting Process


KEY: Nursing Process Step: Implementation
MSC: NCLEX: Physiological Integrity: Basic Care and Comfort

9. In a medical record for a patient who has had an allergic reaction to a drug and an associated
nursing diagnosis of Skin integrity, impaired, related to allergic reaction as evidenced by rash
and hives, the nurse documents ―Subjective: denies itching. Happy with improvement in skin.
Objective: rash fading on face, chest, and back; no hives visible on skin. Skin warm, dry, and
intact. Assessment: skin integrity improving. Plan: check rash daily until discharge.‖ This type
of documentation is an example of:
a. charting by exception.
b. narrative style.
c. a problem-oriented medical record (POMR).
d. the case management system.
ANS: C
The POMR focuses on a patient problem or nursing diagnosis and typically uses the SOAP
(subjective, objective, assessment, plan) format as shown here.

DIF: Cognitive Level: Application REF: p. 88 OBJ: Theory #4


TOP: Methods of Charting KEY: Nursing Process Step: N/A
MSC: NCLEX: N/A

10. In an agency that uses specific protocols (Standard Procedures) and charting by exception, an
advantage compared with using traditional (narrative or problem-oriented) documentation is
that charting by exception:
a. is well suited to defending nursing actions in court.
b. contains important data certainN U
toRbS eI NnGoTteBd. CiOn Mthe narrative sections.
c. allows staff to learn the system quickly and easily.
d. highlights abnormal data and patient trends.
ANS: D
Charting by exception enables staff to see notation of changes in a patient‘s condition at a
glance.

DIF: Cognitive Level: Comprehension REF: p. 90 OBJ: Theory #4


TOP: Methods of Charting KEY: Nursing Process Step: N/A
MSC: NCLEX: N/A

11. If an agency is using computer-assisted charting, the nurse is responsible for:


a. learning the passwords of the staff nurses and primary care providers so that they
can communicate with one another.
b. guarding the confidentiality of the patient record by not leaving the patient screen
―on‖ if he leaves the terminal.
c. patient education to input information about herself, such as intake and output or
symptoms the patient may experience.
d. choosing whether he will use the computer to help in documentation or continue to
use traditional paper documentation.
ANS: B
Confidentiality of computer records is as important as that of the paper medical record. Nurses
must also be protective of their user passwords.

NURSINGTB.COM

Downloaded by: SUCCEEDGRADES | abbieclin@gmail.com


Distribution
Downloaded by Dallen Mae D.ofKadir
this document is illegal
(kadir.dallenmae.d.bcsi@gmail.com)
lOMoARcPSD|35346190

DEWITS FUNDAMENTALStC
uvO
iaN
.cC
om
EP- T
ThSe A
MNarD
ketSpK
l a IcL
eLtoSBFuO
y aRndNSUeR
ll S
yoI u
Nr G
Stu
5dTyHME
atD
erIiaTl ION WILLIAMS TEST BANK

DIF: Cognitive Level: Comprehension REF: p. 92|Box 7-1


OBJ: Theory #4 TOP: Methods of Charting
KEY: Nursing Process Step: Implementation
MSC: NCLEX: Safe, Effective Care Environment: Coordinated Care

12. A nurse begins the shift caring for a patient who has just returned from the recovery room
after surgery. It is most important to document:
a. at the end of the shift so that the nurse can give his full attention and time to the
patient‘s needs during the shift.
b. a nursing care plan in the medical record before assessing the patient so that the
nurse can identify priorities.
c. at least three times during the shift: at the beginning, in the middle, at the end, and
as needed.
d. an initial assessment of the patient and a plan based on the needs of the patient as
assessed at the beginning of the shift.
ANS: D
An initial assessment should be performed at the beginning of the shift and promptly
documented. It will determine the plan and priorities. Documentation should be done as close
to the time of occurrence as possible.

DIF: Cognitive Level: Application REF: p. 94|Box 7-4


OBJ: Theory #1 TOP: The Charting Process
KEY: Nursing Process Step: Implementation
MSC: NCLEX: Physiological Integrity: Basic Care and Comfort

13. The nurse uses the flow sheet in pNatUieRnStI NcaGrTe Bd. Co cOuMmentation primarily:
a. to track routine assessments, treatments, and frequently given care.
b. to eliminate written narratives and to save time.
c. in computer-assisted charting to create visual graphs showing change.
d. to improve continuity of care and exchange of information among disciplines.
ANS: A
Flow sheets are a time saver but do not eliminate narrative charting. They are used to
document information that is routine and that would be ―lost‖ in a narrative note.

DIF: Cognitive Level: Comprehension REF: p. 86 OBJ: Theory #4


TOP: Flow Sheets KEY: Nursing Process Step: Implementation
MSC: NCLEX: Physiological Integrity: Basic Care and Comfort

14. When the nurse documents in narrative or source-oriented format about the patient‘s condition
and the nursing care provided, it is appropriate for him to record:
a. ―Patient will go to physical therapy after lunch.‖
b. ―Diabetes in excellent control. Continue with current insulin schedule.‖
c. ―I gave the patient a thorough bath and cut her fingernails.‖
d. ―To x-ray by wheelchair at 10:30 AM IV infusing in left arm.‖

ANS: D
Documentation that includes specific information regarding time, method of travel,
destination, and current status (that an IV medication is infusing) is a clear example of
source-oriented charting.

NURSINGTB.COM

Downloaded by: SUCCEEDGRADES | abbieclin@gmail.com


Distribution
Downloaded by Dallen Mae D.ofKadir
this document is illegal
(kadir.dallenmae.d.bcsi@gmail.com)
lOMoARcPSD|35346190

DEWITS FUNDAMENTALStC
uvO
iaN
.cC
om
EP- T
ThSe A
MNarD
ketSpK
l a IcL
eLtoSBFuO
y aRndNSUeR
ll S
yoI u
Nr G
Stu
5dTyHME
atD
erIiaTl ION WILLIAMS TEST BANK

DIF: Cognitive Level: Application REF: p. 86 OBJ: Theory #4


TOP: Source-Oriented Charting KEY: Nursing Process Step: Implementation
MSC: NCLEX: Safe, Effective Care Environment: Coordinated Care

15. The nurse understands that a face sheet contains information pertaining to:
a. serial measurements and observations, such as temperature, pulse, respiration,
blood pressure, and weight.
b. plan of care for the patient, including nursing diagnoses, goals/expected outcomes,
and nursing interventions.
c. written report of the nursing process, record of interventions implemented, and the
patient‘s response to them.
d. patient data, including patient‘s name, address, phone number, insurance company,
and admitting diagnosis.
ANS: D
The type of information contained on a face sheet includes patient data, including the patient‘s
name, address, phone number, next of kin, hospital identification number, religious
preference, place of employment, insurance company, occupation, name of admitting
physician, and admitting diagnosis.

DIF: Cognitive Level: Comprehension REF: p. 85|Table 7-1


OBJ: Theory #4 TOP: Documentation Forms
KEY: Nursing Process Step: Planning
MSC: NCLEX: Safe, Effective Care Environment: Coordinated Care

16. A nurse understands that the primary care provider‘s directives for patient care are also
referred to as the: NURSINGTB.COM
a. history and physical.
b. primary care provider‘s orders.
c. progress notes.
d. face sheet.

ANS: B
The primary care provider‘s directives for patient care are the same as the physician‘s orders.

DIF: Cognitive Level: Knowledge REF: p. 85|Table 7-1


OBJ: Clinical Practice #4 TOP: The Medical Record
KEY: Nursing Process Step: N/A MSC: NCLEX: N/A

17. A nurse tells her neighbor personal information about a hospitalized patient. Telling her
neighbor about this indicates that the:
a. nurse is actively promoting nursing as a profession, and it is important to share
information that might encourage others to pursue a nursing career.
b. actions of the nurse are appropriate since his neighbor is his confidante, and the
neighbor has assured him the information provided will not be shared.
c. nurse has violated the confidentiality of the patient by discussing personal
information about the patient with his neighbor.
d. nurse has not violated the confidentiality of the patient because the patient is
terminal; sharing this information will not harm the patient.
ANS: C

NURSINGTB.COM

Downloaded by: SUCCEEDGRADES | abbieclin@gmail.com


Distribution
Downloaded by Dallen Mae D.ofKadir
this document is illegal
(kadir.dallenmae.d.bcsi@gmail.com)
lOMoARcPSD|35346190

DEWITS FUNDAMENTALStC
uvO
iaN
.cC
om
EP- T
ThSe A
MNarD
ketSpK
l a IcL
eLtoSBFuO
y aRndNSUeR
ll S
yoI u
Nr G
Stu
5dTyHME
atD
erIiaTl ION WILLIAMS TEST BANK

As a legal record, the contents of the medical record must be kept confidential and can be
given out only with the patient‘s written consent because it contains personal information
regarding the patient. Only those health professionals caring directly for the patient, or those
involved in research or education, should have access to the chart. Protecting the privacy of
the patient is of prime importance. Patient information is not discussed with others who are
not directly involved in the patient‘s care.

DIF: Cognitive Level: Application REF: p. 92 OBJ: Theory #3


TOP: Patient Confidentiality KEY: Nursing Process Step: N/A
MSC: NCLEX: N/A

18. The Quality and Safety Education for Nurses (QSEN) project has identified the most
important pre-licensing skills for nurses as:
a. effective communication.
b. informatics.
c. familiarity with medical terms.
d. writing nursing care plans.
ANS: B
The Quality and Safety Education for Nurses (QSEN) project has identified informatics as an
important pre-licensing skill.

DIF: Cognitive Level: Knowledge REF: p. 92 OBJ: Theory #4


TOP: Informatics KEY: Nursing Process Step: N/A MSC: NCLEX: N/A

19. Advantages of source-oriented or narrative charting include all of the following except that it:
a. encourages documentation of normal and abnormal findings.
ntU‘ sR Sc IoNnGd Ti t Bi o. Cn OaM
b. gives information on the pa tie N nd care in chronological order.
c. indicates the patient‘s baseline condition for each shift.
d. includes aspects of all steps of the nursing process.

ANS: A
A disadvantage of source-oriented, or narrative, charting is that it encourages documentation
of both normal and abnormal findings, making it difficult to separate pertinent from irrelevant
information.

DIF: Cognitive Level: Comprehension REF: p. 88 OBJ: Theory #4


TOP: The Charting Process KEY: Nursing Process Step: N/A
MSC: NCLEX: N/A

20. Which examples of documentation would be most informative to transcribe to the patient‘s
medical record?
a. ―Patient consumed two slices of bread and a cup of coffee at breakfast.‖
b. ―Patient does not appear to be hungry after consuming breakfast.‖
c. ―Patient ate a small amount of bread and drank a little coffee for breakfast.‖
d. ―Patient ate well for breakfast, lunch, and dinner and seems content.‖

ANS: A

NURSINGTB.COM

Downloaded by: SUCCEEDGRADES | abbieclin@gmail.com


Distribution
Downloaded by Dallen Mae D.ofKadir
this document is illegal
(kadir.dallenmae.d.bcsi@gmail.com)
lOMoARcPSD|35346190

DEWITS FUNDAMENTALStC
uvO
iaN
.cC
om
EP- T
ThSe A
MNarD
ketSpK
l a IcL
eLtoSBFuO
y aRndNSUeR
ll S
yoI u
Nr G
Stu
5dTyHME
atD
erIiaTl ION WILLIAMS TEST BANK

Use of the words ―appears to‖ or ―seems‖ in phrases such as ―appears to be resting‖ should be
avoided. Document the behavior; the patient either is or is not resting. Words that have
ambiguous meanings and slang should not be used in documentation. For example, how much
is ―a little,‖ ―a small amount,‖ or a ―large amount‖? What do phrases such as ―ate well‖ and
―taking fluids poorly‖ mean? Although such words give a general idea of what is meant, they
are not specific.

DIF: Cognitive Level: Application REF: p. 95 OBJ: Theory #4


TOP: Source-Oriented Charting KEY: Nursing Process Step: Implementation
MSC: NCLEX: Safe, Effective Care Environment: Coordinated Care

21. The electronic medical record was set up as a goal of the Stimulus Law that President Obama
signed in 2009, for the purpose of providing a:
a. comprehensive plan of care for all patients.
b. comprehensive record of a patient‘s history and care across all facilities and
admissions.
c. comprehensive document of health care costs.
d. comprehensive plan to allow patient access to medical records.
ANS: B
An electronic health record (EHR) is a computerized comprehensive record of a patient‘s
history and care across all facilities and admissions. This type of record has been set up as a
goal of the Stimulus Law that President Obama signed in 2009. Health care agencies are
mandated to use electronic documentation of patient care (Centers for Medicare and Medicaid
Services, 2015).

DIF: Cognitive Level: Comprehension REF: p. 90 OBJ: Theory #1


NURSIN G T B .C OM
TOP: Computer-Assisted Charting K E Y : N u r s i n g Process Step: N/A
MSC: NCLEX: N/A

MULTIPLE RESPONSE

1. Advantages of the problem-oriented medical record (POMR) are that this method of
documentation: (Select all that apply.)
a. promotes the problem-solving approach.
b. formats documentation into chronological order.
c. makes tracking trends in patient care easy.
d. allows for easy auditing of patient records to evaluate staff performance.
e. reinforces application of the nursing process.
ANS: A, D, E
POMR promotes problem solving with the reinforcement of the nursing process. This method
allows for easy auditing of patient records.

DIF: Cognitive Level: Comprehension REF: p. 88 OBJ: Theory #4


TOP: Problem-Oriented Charting KEY: Nursing Process Step: N/A
MSC: NCLEX: N/A

2. The method of computer-assisted charting: (Select all that apply.)


a. improves communication between departments.
b. is less costly to educate personnel to the method.

NURSINGTB.COM

Downloaded by: SUCCEEDGRADES | abbieclin@gmail.com


Distribution
Downloaded by Dallen Mae D.ofKadir
this document is illegal
(kadir.dallenmae.d.bcsi@gmail.com)
lOMoARcPSD|35346190

DEWITS FUNDAMENTALStC
uvO
iaN
.cC
om
EP- T
ThSe A
MNarD
ketSpK
l a IcL
eLtoSBFuO
y aRndNSUeR
ll S
yoI u
Nr G
Stu
5dTyHME
atD
erIiaTl ION WILLIAMS TEST BANK

c. speeds reimbursement for services.


d. allows electronic records to be retrieved more quickly.
e. allows entries to be made at point of care.
ANS: A, C, D, E
Computerized charting improves communication between departments, speeds reimbursement
for services and retrieval of records, and allows entries to be made quickly at the point of care.
It is more expensive to educate personnel in the use of computers than in other forms of
documentation.

DIF: Cognitive Level: Knowledge REF: p. 90 OBJ: Theory #4


TOP: Documentation KEY: Nursing Process Step: Implementation
MSC: NCLEX: N/A

3. Helpful cultural information the nurse should include on the admission note is: (Select all that
apply.)
a. primary language spoken.
b. number of children in the immediate household.
c. beliefs about causality of illness.
d. level of English literacy.
e. dietary concerns.
ANS: A, C, D, E
Information relative to primary language, beliefs about cause of illness, level of English
literacy, and dietary concerns are helpful items to include on the admission note.

DIF: Cognitive Level: Comprehension REF: p. 93 OBJ: Theory #1


TOP: Cultural Information N U RS I NKGETY
B :. C N
OMu rsing Process Step: Assessment
MSC: NCLEX: Safe, Effective Care Environment: Coordinated Care

COMPLETION

1. Documentation that follows the nursing process and uses nursing diagnoses while placing the
plan of care within the nurses‘ progress notes is charting.

ANS: PIE(problem identification, intervention, and evaluation)

DIF: Cognitive Level: Knowledge REF: p. 89 OBJ: Theory #2


TOP: Documentation KEY: Nursing Process Step: Implementation
MSC: NCLEX: Safe, Effective Care Environment: Coordinated Care

2. Health care professionals assigned to a patient require access to the medical record to review
information and to document care given. All contents of the medical record must be kept
. The contents of the medical record should not be discussed with persons who
are not involved in the care of the patient.

ANS:
confidential

The nurse needs to be able to identify what confidentiality entails.

NURSINGTB.COM

Downloaded by: SUCCEEDGRADES | abbieclin@gmail.com


Distribution
Downloaded by Dallen Mae D.ofKadir
this document is illegal
(kadir.dallenmae.d.bcsi@gmail.com)
lOMoARcPSD|35346190

DEWITS FUNDAMENTALStC
uvO
iaN
.cC
om
EP- T
ThSe A
MNarD
ketSpK
l a IcL
eLtoSBFuO
y aRndNSUeR
ll S
yoI u
Nr G
Stu
5dTyHME
atD
erIiaTl ION WILLIAMS TEST BANK

DIF: Cognitive Level: Knowledge REF: p. 92 OBJ: Theory #3


TOP: Confidentiality KEY: Nursing Process Step: N/A
MSC: NCLEX: N/A

3. The nurse explains that should a patient return to the hospital for treatment within
years, the medical record can be retrieved from medical records for review.

ANS:
10

Medical records are kept in the health information department of a hospital for a period of 10
years.

DIF: Cognitive Level: Knowledge REF: p. 86 OBJ: Theory #3


TOP: Storage of Medical Records KEY: Nursing Process Step: Implementation
MSC: NCLEX: Safe, Effective Care Environment: Coordinated Care

4. When using a case management system of charting a , an unexpected event in the


patient‘s condition is documented on the back of the pathway sheets.

ANS:
variance

A variance is an unexpected event in the patient‘s course of care. An example would be a


healing wound that was complicated by an infection.
DIF: Cognitive Level: Knowledge REF: p. 95|Figure 7-9
OBJ: Theory #4 N
TOP: VariancesU RS I NKGETY
B :. C N
OMursing Process Step: N/A
MSC: NCLEX: N/A

NURSINGTB.COM

Downloaded by: SUCCEEDGRADES | abbieclin@gmail.com


Distribution
Downloaded by Dallen Mae D.ofKadir
this document is illegal
(kadir.dallenmae.d.bcsi@gmail.com)
lOMoARcPSD|35346190

Stuvia.com - The Marketplace to Buy and Sell your Study Material

Chapter 08: Communication and the Nurse-Patient Relationship


Test Bank

MULTIPLE CHOICE

1. The nurse can best ensure that communication is understood by:


a. speaking slowly and clearly in the patient‘s native language.
b. asking the family members whether the patient understands.
c. obtaining feedback from the patient that indicates accurate comprehension.
d. checking for signs of hearing loss or aphasia before communicating.
ANS: C
The best way to determine understanding is to ask the patient. Factors such as anxiety, hearing
acuity, language, aphasia, or lack of familiarity with medical jargon or routines can all
contribute to misunderstanding.

DIF: Cognitive Level: Comprehension REF: pp. 100-101 OBJ: Theory #1


TOP: Feedback KEY: Nursing Process Step: Evaluation
MSC: NCLEX: Physiological Integrity: basic care and comfort

2. The nurse recognizes a verbal response when the patient:


a. nods her head when asked whether she wants juice.
b. writes the answer to a question asked by the nurse.
c. begins sobbing uncontrollably when asked about her daughter.
d. is moaning and restless and appears to be in pain.
ANS: B
Verbal communication involves words, either written or spoken. Nodding, sobbing, and
moaning are nonverbal communication.

DIF: Cognitive Level: Comprehension REF: p. 99 OBJ: Theory #1


TOP: Verbal Communication Feedback KEY: Nursing Process Step: Assessment
MSC: NCLEX: Physiological Integrity: basic care and comfort

3. The nurse recognizes the patient who demonstrates communication congruency when the
patient:
a. smiles and laughs while speaking of feeling lonely and depressed.
b. wrings her hands and paces around the room while denying that she is upset.
c. is tearful and slow in speech when talking about her husband‘s death.
d. states she is comfortable while she frowns and her teeth are clenched.
ANS: C
Congruent communication is the agreement of verbal and nonverbal messages.

DIF: Cognitive Level: Comprehension REF: p. 99 OBJ: Theory #1


TOP: Congruence KEY: Nursing Process Step: Assessment
MSC: NCLEX: Physiological Integrity: basic care and comfort

4. A Hispanic patient approaches the Asian nurse and, standing very close, touches the nurse‘s
shoulder during their conversation. The nurse begins to step back to 18 to 24 inches, while
smiling and nodding to the patient. This situation is most likely an example of:

Downloaded by: SUCCEEDGRADES | abbieclin@gmail.com


Distribution
Downloaded by Dallen Mae D.ofKadir
this document is illegal
(kadir.dallenmae.d.bcsi@gmail.com)
lOMoARcPSD|35346190

Stuvia.com - The Marketplace to Buy and Sell your Study Material

a. the nurse‘s need to maintain a professional role rather than a social role.
b. a patient‘s attempt to keep the nurse‘s attention.
c. a nurse‘s need to establish a more appropriate location for conversation.
d. a difference in culturally learned personal space of the nurse and the patient.
ANS: D
Personal space between people is a culturally learned behavior; Asians, North American
natives, and Northern European people generally prefer more personal space than people of
Hispanic, Southern European, or Middle Eastern cultures.

DIF: Cognitive Level: Comprehension REF: p. 100 OBJ: Theory #2


TOP: Cultural Differences KEY: Nursing Process Step: Implementation
MSC: NCLEX: Psychosocial Integrity: coping and adaptation

5. A nurse says to a patient, ―I am going to take your TPR, and then I‘ll check to see whether you
can have a PRN analgesic.‖ In considering factors that affect communication, the nurse has:
a. used terminology to clearly inform the patient of what she is doing.
b. given information that is unnecessary for the patient to know.
c. used medical jargon, which might not be understood by the patient.
d. taken into consideration the patient‘s need to know what is happening.
ANS: C
Medical jargon such as abbreviations or medical terminology is often misunderstood, even by
well-educated people.

DIF: Cognitive Level: Comprehension REF: p. 101 OBJ: Theory #3


TOP: Blocks to Communication KEY: Nursing Process Step: Implementation
MSC: NCLEX: Psychosocial Integrity: coping and adaptation

6. A nurse using active listening techniques would:


a. use nonverbal cues such as leaning forward, focusing on the speaker‘s face, and
slightly nodding to indicate that the message has been heard.
b. avoid the use of eye contact to allow the patient to express herself without feeling
stared at or demeaned.
c. anticipate what the speaker is trying to say and help the patient express herself
when she has difficulty with finishing a sentence.
d. ask probing questions to direct the conversation and obtain the information needed
as efficiently as possible.
ANS: A
Eye contact is a culturally learned behavior and in some cases may not be appropriate.
Probing questions or finishing the patient‘s sentence is not part of active listening and is
detrimental to an interview.

DIF: Cognitive Level: Comprehension REF: p. 101 OBJ: Theory #3


TOP: Active Listening KEY: Nursing Process Step: Implementation
MSC: NCLEX: Psychosocial Integrity: coping and adaptation

7. When the patient says, ―I don‘t want to go home,‖ the nurse‘s best therapeutic verbal response
would be:
a. ―I‘m sure everything will be fine once you get home.‖
b. ―You don‘t want to go home?‖

Downloaded by: SUCCEEDGRADES | abbieclin@gmail.com


Distribution
Downloaded by Dallen Mae D.ofKadir
this document is illegal
(kadir.dallenmae.d.bcsi@gmail.com)
lOMoARcPSD|35346190

Stuvia.com - The Marketplace to Buy and Sell your Study Material

c. ―Doesn‘t your family want you to come home?‖


d. ―I felt like that when I had surgery last year.‖

ANS: B
The use of reflecting encourages the patient to expand on his or her feelings or thoughts.

DIF: Cognitive Level: Application REF: p. 103, Table 8-1


OBJ: Theory #3 TOP: Communication Techniques
KEY: Nursing Process Step: Implementation
MSC: NCLEX: Psychosocial Integrity: coping and adaptation

8. To begin talking with a newly admitted patient about pain management, the nurse would most
appropriately state:
a. ―You look pretty comfortable. Are you having any pain?‖
b. ―Tell me about the pain you‘ve been having.‖
c. ―Is this pain the same as the pain you had yesterday?‖
d. ―Don‘t worry; this pain won‘t last forever.‖
ANS: B
An open-ended question allows the patient to express his or her feelings or needs.

DIF: Cognitive Level: Application REF: p. 103, Table 8-1


OBJ: Theory #3 TOP: Communication Techniques
KEY: Nursing Process Step: Implementation
MSC: NCLEX: Psychosocial Integrity: coping and adaptation

9. When a patient begins crying during a conversation with the nurse about the patient‘s
upcoming surgery for possible malignancy, the nurse‘s most therapeutic response would be:
a. ―Your surgeon is excellent, and I know he‘ll do a great job.‖
b. ―Oh, dear, your gown is way too big, let me get you another one.‖
c. ―Don‘t cry; think about something else and you‘ll feel better.‖
d. ―Here is a tissue. I‘d like to sit here for a while if you want to talk.‖

ANS: D
Offering self, or presence, and accepting a patient‘s need to cry is supportive.

DIF: Cognitive Level: Application REF: p. 103, Table 8-1


OBJ: Theory #3 TOP: Therapeutic Techniques
KEY: Nursing Process Step: Implementation
MSC: NCLEX: Psychosocial Integrity: coping and adaptation

10. To enhance the establishment of rapport with a patient, the nurse should:
a. identify himself by name and title each time he introduces himself.
b. share his own personal experiences so that the patient gets to know him as a friend.
c. act in a trustworthy and reliable manner; respect the individuality of the patient.
d. share information with the patient about other patients and why they are
hospitalized.
ANS: C
Trust and reliability, as well as conveying respect for the individual, all promote rapport.
Identifying oneself is important but in itself does not promote rapport. Sharing personal
experiences or divulging the confidential nature of other patients‘ conditions is not appropriate
in the nurse–patient relationship.

Downloaded by: SUCCEEDGRADES | abbieclin@gmail.com


Distribution
Downloaded by Dallen Mae D.ofKadir
this document is illegal
(kadir.dallenmae.d.bcsi@gmail.com)
lOMoARcPSD|35346190

Stuvia.com - The Marketplace to Buy and Sell your Study Material

DIF: Cognitive Level: Comprehension REF: p. 107 OBJ: Clinical Practice #2


TOP: Rapport KEY: Nursing Process Step: Implementation
MSC: NCLEX: Psychosocial Integrity: coping and adaptation

11. The nurse explains that the therapeutic nurse–patient relationship differs from the social
relationship because:
a. a social relationship does not have goals or needs to be met.
b. the nurse–patient relationship ends when the patient is discharged.
c. the focus is mainly on the nurse in the nurse–patient relationship.
d. a social relationship does not require trust or sharing of life experiences.

ANS: B
The nurse–patient relationship is limited to the patient‘s stay in the facility and is focused on
the patient. A social relationship may have goals or needs and does require trust and sharing of
life experiences.

DIF: Cognitive Level: Comprehension REF: pp. 106-107 OBJ: Theory #4


TOP: Relationships KEY: Nursing Process Step: Implementation
MSC: NCLEX: Psychosocial Integrity: coping and adaptation

12. The nurse has selected an outcome for the patient to eat all of the food on the breakfast tray
each day. Assessing that the patient has eaten all of the breakfast, the nurse would give
positive feedback by saying:
a. ―Wow! That breakfast must have been pretty good.‖
b. ―I like pancakes too. Everyone on the hall seemed to enjoy them.‖
c. ―I hope you can keep all that breakfast down.‖
d. ―Hurray! You finished your whole meal! What would you like for tomorrow?‖
ANS: D
Giving positive feedback increases the likelihood of the desired behavior to be repeated.
Commenting on the tastiness of the food or the fact that others liked it is not responding
directly to the patient‘s having eaten the whole meal.

DIF: Cognitive Level: Application REF: p. 100 OBJ: Theory #9


TOP: Positive Feedback KEY: Nursing Process Step: Implementation
MSC: NCLEX: Physiological Integrity: basic care and comfort

13. A 67-year-old woman had major abdominal surgery yesterday. She has IV lines, a urinary
catheter, and an abdominal wound dressing, and she is receiving PRN pain medication. The
end-of-shift report that best conveys the patient status is:
a. ―Doing great, was up in the chair most of the day. No complaints of pain or
discomfort. Voiding adequately.‖
b. ―Abdominal surgery yesterday, dressing is dry and intact, her IVs are on time and
she‘s had pain meds twice. Vital signs stable.‖
c. ―Abdominal dressing dry, IVs—800 mL left in #6; NS running at 125 mL/hr; urine
output 800 mL this shift; had morphine 15 mg for pain at 8:00 AM and at 1:30 PM.
She‘s comfortable now. Vital signs are stable, no fever.‖
d. ―Unchanged since this morning. She wanted to know how soon she can have
something to eat, so maybe you could check with her doctor this evening. Her
husband has been visiting all day and will let you know if she needs anything.‖

Downloaded by: SUCCEEDGRADES | abbieclin@gmail.com


Distribution
Downloaded by Dallen Mae D.ofKadir
this document is illegal
(kadir.dallenmae.d.bcsi@gmail.com)
lOMoARcPSD|35346190

Stuvia.com - The Marketplace to Buy and Sell your Study Material

ANS: C
This brief clear report addresses the major concerns of the abdominal dressing, the status of
the IV fluids, vital signs, and analgesia needs.

DIF: Cognitive Level: Application REF: pp. 109-110 OBJ: Clinical Practice #4
TOP: Shift Report KEY: Nursing Process Step: Implementation
MSC: NCLEX: Safe Effective Care Environment: coordinated care

14. An aspect of computer use in patient care in which the LPN may need to be proficient
includes:
a. input of data such as requests for radiographs or laboratory services.
b. programming the computer to record data from physicians and other health care
workers.
c. teaching patients how to use hospital computers to access information such as
discharge instructions or information relative to specific medications.
d. scheduling admissions, discharges, and nurse staffing to keep the unit at the best
occupancy and utilization.
ANS: A
Many facilities use computers for data entry relative to requesting radiograph or lab services
and physical assessment and medication administration. Programming such computers is not a
nursing task, and patients need to have individualized information about discharge and
medications.

DIF: Cognitive Level: Knowledge REF: pp. 110-111 OBJ: Theory #8


TOP: Computer Use KEY: Nursing Process Step: N/A
MSC: NCLEX: N/A

15. A patient with a nursing diagnosis of Sensory perception, disturbed auditory, would most
appropriately require the nurse to:
a. obtain a sign language interpreter when a family member is unavailable.
b. speak slowly and distinctly, but not shout.
c. provide bright lighting without glare and orient frequently.
d. reorient frequently to time, place, staff, and events.
ANS: B
A patient with disturbed auditory perception cannot hear well (or at all); therefore, speaking
slowly and distinctly without shouting increases patient comprehension.

DIF: Cognitive Level: Application REF: pp. 107-108 OBJ: Clinical Practice #3
TOP: Hearing-Impaired Patient Communication
KEY: Nursing Process Step: Implementation
MSC: NCLEX: Psychosocial Integrity: coping and adaptation

16. When an office nurse asks the patient to repeat information that he has just given to the patient
over the telephone, the nurse is:
a. testing the patient‘s intelligence and memory.
b. acting in a cautious way to avoid charges of negligence.
c. verifying that the patient understands the information.
d. saving the extra time it would take to mail the information.
ANS: C

Downloaded by: SUCCEEDGRADES | abbieclin@gmail.com


Distribution
Downloaded by Dallen Mae D.ofKadir
this document is illegal
(kadir.dallenmae.d.bcsi@gmail.com)
lOMoARcPSD|35346190

Stuvia.com - The Marketplace to Buy and Sell your Study Material

Obtaining feedback from a patient to ascertain that the patient understands instructions is an
important part of the communication process, especially over the phone, when the nurse does
not have nonverbal cues.

DIF: Cognitive Level: Comprehension REF: pp. 100-101 OBJ: Theory #3


TOP: Telephonic Communication KEY: Nursing Process Step: Implementation
MSC: NCLEX: Psychosocial Integrity: coping and adaptation

17. A 36-year-old woman who is in traction for a fractured femur that she received in an auto
accident is found crying quietly. The nurse can best address this situation by saying:
a. ―What‘s the matter? Why are you crying? Are you in pain?‖
b. ―Stop crying and tell me what your problem is.‖
c. ―This could have been much worse. You‘re lucky no one was killed.‖
d. ―You are upset. Can you tell me what‘s wrong?‖
ANS: D
The nurse offers a general lead as to what is causing the distress. The other options are
judgmental or clichés or offer no opportunity for the patient to express feelings.

DIF: Cognitive Level: Application REF: p. 103, Table 8-1


OBJ: Theory #3 TOP: Reflecting Observations
KEY: Nursing Process Step: Implementation
MSC: NCLEX: Psychosocial Integrity: coping and adaptation

18. When the nurse is giving direction to a nursing assistant who is being delegated part of the
patient care, the nurse‘s most effective direction would be:
a. ―Do the morning care first on the patients in 205 and 206 who can‘t get out of bed.‖
b. ―You take care of all the patients in 205 and 206. Let me know how you‘re doing
and whether you need any help.‖
c. ―Give the patient in 204A a shower after breakfast, and call me to check her feet
before you get her dressed.‖
d. ―Take the vital signs on all the patients in the lounge and tell me whether there are
problems.‖
ANS: C
The clarity and brevity of the direction makes the delegated task clear and leaves the
responsibility of assessment to the nurse.

DIF: Cognitive Level: Application REF: p. 110 OBJ: Theory #7


TOP: Delegation KEY: Nursing Process Step: Planning
MSC: NCLEX: Safe Effective Care Environment: coordinated care

19. When the patient says, ―I get so anxious just lying here in this hospital bed. I have a million
things I should be doing at home,‖ the most empathetic response would be:
a. ―I‘d feel the same way you do. I know just what you‘re going through.‖
b. ―It sounds like you‘re having a tough time dealing with this situation.‖
c. ―It‘s always darkest before the dawn. Hang in there; it will get better.‖
d. ―You sound pretty sorry for yourself. Why don‘t you look at the positives?‖
ANS: B
Empathy recognizes a patient‘s situation and encourages expression of feelings.

Downloaded by: SUCCEEDGRADES | abbieclin@gmail.com


Distribution
Downloaded by Dallen Mae D.ofKadir
this document is illegal
(kadir.dallenmae.d.bcsi@gmail.com)
lOMoARcPSD|35346190

Stuvia.com - The Marketplace to Buy and Sell your Study Material

DIF: Cognitive Level: Application REF: p. 107 OBJ: Theory #3


TOP: Empathy KEY: Nursing Process Step: Implementation
MSC: NCLEX: Psychosocial Integrity: coping and adaptation

20. A patient asks the nurse, ―What would you do if you had cancer and had to choose between
surgery and chemotherapy?‖ The reply that can best help the patient is:
a. ―If I were you, I would choose surgery and then consider chemo afterward.‖
b. ―What solutions have you considered?‖
c. ―I would talk it over with my friends first.‖
d. ―I don‘t know. I‘m glad it isn‘t my decision.‖
ANS: B
Nurses can help by reminding patients of alternatives open to them and should refrain from
giving advice but can encourage the patient to consider options. The nurse may be glad not to
face a decision a patient must, but it is not helpful to the patient to say this.

DIF: Cognitive Level: Application REF: p. 103, Table 8-1


OBJ: Theory #3 TOP: Offering Alternatives
KEY: Nursing Process Step: Implementation
MSC: NCLEX: Psychosocial Integrity: coping and adaptation

21. The nurse chooses to use touch in the nurse–patient relationship because touch:
a. can convey caring and support when words are difficult.
b. should be avoided because of problems of cultural misinterpretation.
c. is appropriate only in special circumstances, such as with young children.
d. is a nursing intervention of choice in almost all situations.
ANS: A
Touch is a powerful and supportive nonverbal communication in many situations. It is
appropriate for all ages, but not in some situations. Careful assessment of the patient‘s
situation and cultural values should determine its use, but it should not be avoided because of
stereotypes.

DIF: Cognitive Level: Comprehension REF: p. 103 OBJ: Theory #4


TOP: Caring Touch KEY: Nursing Process Step: Implementation
MSC: NCLEX: Psychosocial Integrity: coping and adaptation

22. When the nurse makes the statement, ―We can come back to that later—right now I need to
know about when your symptoms started,‖ the nurse is:
a. letting the patient know that topic of conversation was inappropriate.
b. setting limits on the expression of feelings.
c. refocusing the patient to the issue at hand when the conversation has wandered.
d. closing off the conversation by quickly getting to the point of the interview.
ANS: C
Refocusing is often necessary to accomplish data collection. It does not block communication
and is not used to close a conversation or stop an inappropriate topic.

DIF: Cognitive Level: Comprehension REF: p. 102 OBJ: Theory #3


TOP: Refocusing Communication KEY: Nursing Process Step: Implementation
MSC: NCLEX: Psychosocial Integrity: coping and adaptation

Downloaded by: SUCCEEDGRADES | abbieclin@gmail.com


Distribution
Downloaded by Dallen Mae D.ofKadir
this document is illegal
(kadir.dallenmae.d.bcsi@gmail.com)
lOMoARcPSD|35346190

Stuvia.com - The Marketplace to Buy and Sell your Study Material

23. A patient who has had a stroke is unable to speak clearly and has right-sided hemiplegia. The
nurse will design the approach to the assessment interview by:
a. asking questions and explaining procedures to the patient‘s daughter.
b. speaking slowly and giving the patient time to respond.
c. telling the patient he will get all necessary information from the daughter.
d. prompting the answers and finishing the sentences for the patient.
ANS: B
Speaking slowly recognizes that the patient may process (if able) information more slowly.

DIF: Cognitive Level: Application REF: p. 103, Table 8-1


OBJ: Clinical Practice #3 TOP: Impaired Communication
KEY: Nursing Process Step: Implementation
MSC: NCLEX: Psychosocial Integrity: coping and adaptation

24. When a nurse is conducting an assessment interview, the most efficient technique would be:
a. explaining the purpose of the interview.
b. excluding relatives and friends from the interaction.
c. telling the patient what data are already available.
d. asking closed questions to obtain essential information.

ANS: D
Closed questions have a definite place when the nurse wants to obtain specific essential data.
Closed questions force the patient to stick to the topic.

DIF: Cognitive Level: Comprehension REF: p. 102 OBJ: Clinical Practice #1


TOP: Interview KEY: Nursing Process Step: Implementation
MSC: NCLEX: Psychosocial Integrity: coping and adaptation

25. While interviewing a Native American man for the admission history, the nurse should expect
to:
a. wait patiently through long pauses in the conversation.
b. maintain eye contact with the patient.
c. give the patient permission to speak.
d. have another family member speak for the patient.
ANS: A
Native Americans use long pauses in their conversation to better consider their answer and
consider the question. The culturally sensitive nurse would wait quietly through the pauses.

DIF: Cognitive Level: Comprehension REF: p. 102 OBJ: Clinical Practice #1


TOP: Cultural Considerations KEY: Nursing Process Step: Planning
MSC: NCLEX: Psychosocial Integrity: coping and adaptation

26. The nurse is aware that the purpose of therapeutic communication is to:
a. gather as much information as possible about the patient‘s problem.
b. direct the patient to communicate about his deepest concerns.
c. focus on the patient and the patient needs to facilitate interaction.
d. gain specific medical information and history of illness.

ANS: C

Downloaded by: SUCCEEDGRADES | abbieclin@gmail.com


Distribution
Downloaded by Dallen Mae D.ofKadir
this document is illegal
(kadir.dallenmae.d.bcsi@gmail.com)
lOMoARcPSD|35346190

Stuvia.com - The Marketplace to Buy and Sell your Study Material

Therapeutic communication is a conversation that is focused on the patient and promotes


understanding between the sender and the receiver.

DIF: Cognitive Level: Comprehension REF: p. 102 OBJ: Theory #4


TOP: Therapeutic Communication KEY: Nursing Process Step: Planning
MSC: NCLEX: Psychosocial Integrity: coping and adaptation

27. The practical nursing student who is engaged in a therapeutic communication with a patient
will have the most difficulty with the technique of:
a. closed questions.
b. restating.
c. using general leads.
d. silence.
ANS: D
The use of silence is the hardest for most students to develop because it makes them
uncomfortable, so they tend to end it prematurely.

DIF: Cognitive Level: Comprehension REF: p. 102 OBJ: Theory #31


TOP: Silence KEY: Nursing Process Step: Implementation
MSC: NCLEX: Psychosocial Integrity: coping and adaptation

28. To convey the intervention of active listening, the nurse would:


a. maintain eye contact by staring at the patient.
b. prompt the patient when the patient stops talking for a moment.
c. make a conscious effort to block out other sounds in the immediate environment.
d. write down remarks on a clipboard to facilitate later topics of conversation.
ANS: C
An active listener maintains eye contact without staring, gives the patient full attention, and
makes a conscious effort to block out other sounds and distractions.

DIF: Cognitive Level: Comprehension REF: p. 101 OBJ: Theory #3


TOP: Active Listening KEY: Nursing Process Step: Implementation
MSC: NCLEX: Psychosocial Integrity: coping and adaptation

29. When the nurse enters the room, the patient is laughing out loud at something on TV. The
patient stops and apologizes for the laughter, saying, ―I guess I ought not be laughing at all
since I am stuck here with two broken legs.‖ The nurse can use evidence-based information
when she responds:
a. ―Laughter is nearly always a cover-up for anxiety when facing a long
rehabilitation.‖
b. ―Long periods of laughter decrease the amount of oxygen available to your body
for healing.‖
c. ―Laughter in a hospital is often distracting and depressing to other patients nearby.‖
d. ―Laughter truly is the best medicine as it has a positive effect on the immune
system.‖
ANS: D
Hasen and Hasen (2009) found that laughter and appropriate use of humor decreased stress
and anxiety and had a positive effect on the immune system.

Downloaded by: SUCCEEDGRADES | abbieclin@gmail.com


Distribution
Downloaded by Dallen Mae D.ofKadir
this document is illegal
(kadir.dallenmae.d.bcsi@gmail.com)
lOMoARcPSD|35346190

Stuvia.com - The Marketplace to Buy and Sell your Study Material

DIF: Cognitive Level: Application REF: p. 101 OBJ: Clinical Practice #2


TOP: Use of Laughter KEY: Nursing Process Step: Implementation
MSC: NCLEX: Psychosocial Integrity: coping and adaptation

30. When interacting with an elderly patient, the nurse would enhance communication by:
a. speaking slowly in order to allow the patient to process the message.
b. addressing him by his first name to encourage a therapeutic relationship.
c. standing in the doorway rather than entering the room to give the elderly patient
more privacy.
d. speaking in simple sentences, as if to a child.
ANS: A
When interacting with an elderly person, the nurse should try not to speak too quickly or
expect an immediate answer because the elderly take more time to process the message. Do
not use baby talk or speak to them as if they were children.

DIF: Cognitive Level: Comprehension REF: p. 101 OBJ: Theory #2


TOP: Communication KEY: Nursing Process Step: Implementation
MSC: NCLEX: Psychosocial Integrity: coping and adaptation

31. When the nurse observes a resident in a long-term facility pounding his fists on his legs and
grinding his teeth, the nurse will validate her perception of the patient‘s non-verbal expression
of anger by:
a. documenting that the patient was agitated and appeared angry.
b. asking the male nursing assistant if it is his perception that the patient appears
angry.
c. accessing the nursing care plan to ascertain if there is a nursing diagnosis relative to
anger.
d. sitting down near the patient and saying, ―You seem upset…can I help?‖
ANS: D
All perceptions based on the observation of non-verbal behavior should be validated by
consulting the patient.

DIF: Cognitive Level: Application REF: p. 100 OBJ: Theory #9


TOP: Validating Perceptions
KEY: Nursing Process Step: Assessment | Nursing Process Step: Implementation
MSC: NCLEX: Psychosocial Integrity: coping and adaptation

32. When a patient states, ―I don‘t feel like walking today,‖ the nurse‘s most therapeutic verbal
response would be:
a. ―You have to walk today.‖
b. ―You don‘t want to walk today?‖
c. ―I don‘t feel like walking today either.‖
d. ―Why don‘t you want to walk today?‖
ANS: B
Reflection is a way to restate the message. The idea is simply reflected back to the speaker in
a statement to encourage continued dialogue on the topic.

DIF: Cognitive Level: Application REF: p. 103, Table 8-1


OBJ: Theory #3 TOP: Communication Techniques

Downloaded by: SUCCEEDGRADES | abbieclin@gmail.com


Distribution
Downloaded by Dallen Mae D.ofKadir
this document is illegal
(kadir.dallenmae.d.bcsi@gmail.com)
lOMoARcPSD|35346190

Stuvia.com - The Marketplace to Buy and Sell your Study Material

KEY: Nursing Process Step: Implementation


MSC: NCLEX: Psychosocial Integrity: coping and adaptation

33. When a patient states, ―My son hasn‘t been to see me in months,‖ the nurse‘s best verbal
response is:
a. ―Don‘t worry; I‘m sure your son will visit.‖
b. ―Your son hasn‘t been around much lately?‖
c. ―My son doesn‘t come to visit me either.‖
d. ―How terrible that he doesn‘t visit you.‖
ANS: B
Restating in different words what the patient said encourages further communication on that
topic.

DIF: Cognitive Level: Application REF: p. 103, Table 8-1


OBJ: Theory #3 TOP: Communication Techniques
KEY: Nursing Process Step: Implementation
MSC: NCLEX: Psychosocial Integrity: coping and adaptation

34. An example of a nurse communicating with a patient using open-ended questions would be:
a. ―Is your pain less today than it was yesterday?‖
b. ―Did you sleep all night without waking?‖
c. ―How many bowel movements have you had today?‖
d. ―What was your daughter‘s reaction to your desire for hospice?‖
ANS: D
An open-ended question is broad, indicating only the topic, and it requires an answer of more
than a word or two. Use of an open-ended question or statement allows the patient to elaborate
on a subject or to choose aspects of the subject to be discussed. Open-ended questions or
statements are helpful to open up the conversation or to proceed to a new topic. They usually
cannot be answered with one word or just ―yes‖ or ―no.‖

DIF: Cognitive Level: Comprehension REF: p. 103, Table 8-1


OBJ: Theory #3 TOP: Communication Techniques
KEY: Nursing Process Step: Implementation
MSC: NCLEX: Psychosocial Integrity: coping and adaptation

35. The nurse tells a patient, ―For the last 2 days we have talked about whether to notify your
daughter of your upcoming surgery in 2 days. You have indicated you do not want to be a
burden to her, but you also would like to have her here. You may have to decide rather quickly
because of the time constraint.‖ The nurse is using the technique of:
a. focusing.
b. reflection.
c. restatement.
d. summarizing.
ANS: D
Summarizing presents the problem and possible solutions with the attendant difficulties. This
technique ―unclutters‖ the problem and presents it back to the patient for his or her choice of a
solution.

DIF: Cognitive Level: Comprehension REF: p. 104 OBJ: Theory #3

Downloaded by: SUCCEEDGRADES | abbieclin@gmail.com


Distribution
Downloaded by Dallen Mae D.ofKadir
this document is illegal
(kadir.dallenmae.d.bcsi@gmail.com)
lOMoARcPSD|35346190

Stuvia.com - The Marketplace to Buy and Sell your Study Material

TOP: Communication Techniques KEY: Nursing Process Step: Implementation


MSC: NCLEX: Psychosocial Integrity: coping and adaptation

36. The nurse is caring for a patient who states, ―I tossed and turned last night.‖ The nurse
responds to the patient, ―You feel like you were awake all night?‖ This is an example of:
a. an open-ended question.
b. restatement.
c. reflection.
d. offering self.
ANS: B
Restatement is a therapeutic communication technique in which the nurse restates in different
words what the patient said. This encourages further communication on that topic.

DIF: Cognitive Level: Comprehension REF: pp. 102-103 OBJ: Theory #3


TOP: Restatement KEY: Nursing Process Step: Implementation
MSC: NCLEX: Psychosocial Integrity: coping and adaptation

37. The nurse is caring for a patient who has just had a mastectomy (breast removal). The patient
expresses concern that her husband will no longer find her attractive because of her
mastectomy. The nurse appropriately responds:
a. ―You‘re concerned your husband will find you unattractive because of your
mastectomy?‖
b. ―You‘re a beautiful woman; of course your husband will find you attractive after
your mastectomy.‖
c. ―Don‘t worry; when I had my mastectomy, my husband still found me very
attractive.‖
d. ―You should leave your husband immediately if he thinks you‘re unattractive after
a mastectomy.‖
ANS: A
This is an example of restatement, which allows the patient to know her message was
understood and encourages the patient to continue about her concerns on the topic.

DIF: Cognitive Level: Application REF: pp. 102-103 OBJ: Theory #3


TOP: Restatement/Reflection KEY: Nursing Process Step: Implementation
MSC: NCLEX: Psychosocial Integrity: coping and adaptation

38. A patient states, ―I‘m so worried that I might have cancer.‖ The nurse responds, ―It is time for
you to eat breakfast.‖ The nurse‘s response is an example of:
a. using clichés.
b. judgmental response.
c. changing the subject.
d. giving false reassurance.
ANS: C
Changing the subject is a block to effective communication in which the patient is deprived of
the chance to verbalize concerns.

DIF: Cognitive Level: Comprehension REF: p. 104 OBJ: Theory #3


TOP: Changing the Subject KEY: Nursing Process Step: Implementation
MSC: NCLEX: Psychosocial Integrity: coping and adaptation

Downloaded by: SUCCEEDGRADES | abbieclin@gmail.com


Distribution
Downloaded by Dallen Mae D.ofKadir
this document is illegal
(kadir.dallenmae.d.bcsi@gmail.com)
lOMoARcPSD|35346190

Stuvia.com - The Marketplace to Buy and Sell your Study Material

39. The nurse is aware that the use of false reassurance is harmful to the nurse–patient
relationship, because this communication block:
a. discounts the patient‘s stated concerns.
b. shows a judgmental attitude on the part of the nurse.
c. summarizes the patient‘s concerns and closes communication.
d. confuses the patient by giving information.
ANS: A
Giving false reassurance is a block to effective communication in which the patient‘s feelings
are negated and in which the patient may be given false hope, which, if things turn out
differently, can destroy trust in the nurse.

DIF: Cognitive Level: Comprehension REF: p. 104 OBJ: Theory #3


TOP: False Reassurance KEY: Nursing Process Step: Implementation
MSC: NCLEX: Psychosocial Integrity: coping and adaptation

40. A home health patient with a bleeding ulcer informs the nurse that she ate a bowl of chili with
jalapenos. An inappropriate communication block with a judgmental tone by the nurse would
be:
a. ―Well, you have had this problem long enough to know what will happen—you
certainly can‘t blame me!‖
b. ―I don‘t think that was a smart thing for you to do considering your ulcer.‖
c. ―Well, you better watch your stool for evidence of blood so you can notify your
physician.‖
d. ―Oh, poo! A bowl of chili every now and then won‘t make a lot of difference to
your ulcer.‖
ANS: B
Judgmental response is a block to effective communication in which the nurse is judging the
patient‘s action. It implies that the patient must take on the nurse‘s values and is demeaning to
the patient.

DIF: Cognitive Level: Analysis REF: p. 105, Table 8-2


OBJ: Theory #3 TOP: Judgmental Response
KEY: Nursing Process Step: Implementation
MSC: NCLEX: Psychosocial Integrity: coping and adaptation

41. A patient tells the nurse that she dislikes the food that is served in the hospital. The nurse
responds, ―Our cooks work very hard; the food that is served is very good.‖ The nurse‘s
response is an example of the communication block of:
a. judgmental response.
b. giving advice.
c. defensive response.
d. using clichés.

ANS: C
Defensive response is a block to effective communication in which the nurse responds by
defending the hospital food. This prevents the patient from feeling that she is free to express
her feelings.

DIF: Cognitive Level: Comprehension REF: p. 105 OBJ: Theory #3

Downloaded by: SUCCEEDGRADES | abbieclin@gmail.com


Distribution
Downloaded by Dallen Mae D.ofKadir
this document is illegal
(kadir.dallenmae.d.bcsi@gmail.com)
lOMoARcPSD|35346190

Stuvia.com - The Marketplace to Buy and Sell your Study Material

TOP: Defensive Response KEY: Nursing Process Step: Implementation


MSC: NCLEX: Psychosocial Integrity: coping and adaptation

42. A nurse caring for a patient who fell off the roof while he was intoxicated asks the patient,
―Why in the world were you on the roof when you had been drinking?‖ The nurse‘s statement
is an example of which type of communication?
a. Changing the subject
b. Defensive response
c. Inattentive listening
d. Asking probing questions
ANS: D
Asking probing questions is a block to effective communication in which the nurse pries into
the patient‘s motives and therefore invades privacy.

DIF: Cognitive Level: Comprehension REF: p. 105 OBJ: Theory #3


TOP: Probing KEY: Nursing Process Step: Implementation
MSC: NCLEX: Psychosocial Integrity: coping and adaptation

43. The nurse caring for a patient who is concerned about her 10-pound weight loss relative to her
chemotherapy tells the patient, ―Lucky you! Every cloud has a silver lining.‖ The nurse‘s
statement is an example of which type of communication block?
a. Defensive response
b. Asking probing questions
c. Using clichés
d. Changing the subject

ANS: C
Using clichés is a block to effective communication in which the patient‘s individual situation
is negated, and the patient is stereotyped. This type of response sounds flippant and prevents
the building of trust between the patient and the nurse.

DIF: Cognitive Level: Comprehension REF: pp. 105-106 OBJ: Theory #3


TOP: Clichés KEY: Nursing Process Step: Implementation
MSC: NCLEX: Psychosocial Integrity: coping and adaptation

44. The nurse is caring for a patient with a diagnosis of lung cancer. The nurse states, ―If I were
you, I would have radiation therapy.‖ The nurse‘s statement is an example of which type of
communication block?
a. Inattentive listening
b. Giving advice
c. Using clichés
d. Defensive response
ANS: B
Giving advice is a block to effective communication and tends to be controlling and
diminishes patients‘ responsibility for taking charge of their own health.

DIF: Cognitive Level: Comprehension REF: p. 105, Table 8-2


OBJ: Theory #3 TOP: Giving Advice
KEY: Nursing Process Step: Implementation
MSC: NCLEX: Psychosocial Integrity: coping and adaptation

Downloaded by: SUCCEEDGRADES | abbieclin@gmail.com


Distribution
Downloaded by Dallen Mae D.ofKadir
this document is illegal
(kadir.dallenmae.d.bcsi@gmail.com)
lOMoARcPSD|35346190

Stuvia.com - The Marketplace to Buy and Sell your Study Material

45. The nurse is caring for a patient who is concerned about living alone. The best response by the
nurse is:
a. ―Where have you considered living?‖
b. ―Why don‘t you live with your family?‖
c. ―I think you should live with your family.‖
d. ―If you were my mom, I‘d have you live with me.‖
ANS: A
Rephrasing will help the patient explore various alternatives. The nurse should not use phrases
such as ―Why don‘t you...,‖ ―When that happened to me, I did...,‖ or ―I think you should. ‖
Rephrasing, for example, ―Have you thought of your options?‖ or ―You might want to think
about...,‖ or ―Have you considered. ?‖ will help the patient explore various alternatives.

DIF: Cognitive Level: Application REF: p. 104 OBJ: Theory #3


TOP: Offering Alternatives KEY: Nursing Process Step: Implementation
MSC: NCLEX: Psychosocial Integrity: coping and adaptation

46. The characteristic that is representative of the nurse–patient relationship is that this
relationship:
a. focuses on the nurse‘s ability to build rapport.
b. continues after discharge.
c. does not include humor.
d. focuses on the assessed patient health problems.
ANS: D
The nurse–patient relationship focuses on the patient, has goals, and is defined by specific
boundaries. The relationship takes place in the health care setting, and boundaries are defined
by the patient‘s problems, the help needed, and the nurse‘s professional role. When the patient
is discharged, the relationship ends.

DIF: Cognitive Level: Knowledge REF: pp. 106-107 OBJ: Theory #4


TOP: Nurse–Patient Relationship KEY: Nursing Process Step: N/A
MSC: NCLEX: N/A

47. When communicating with an aphasic patient, the nurse appropriately:


a. speaks quickly and shouts so the patient can hear.
b. assumes the patient can understand what is heard.
c. speaks to the patient‘s caregiver about the patient.
d. assumes the patient cannot understand what is heard.
ANS: B
When communicating with an aphasic patient, the nurse assumes the patient can understand
what is heard even though speech is jargon or the person is mute, unless deafness has been
diagnosed. The nurse should talk to the patient, and not talk to someone else in the room about
the patient. The nurse should speak slowly and distinctly and should not shout.

DIF: Cognitive Level: Comprehension REF: p. 108, Box 8-1


OBJ: Clinical Practice #3 TOP: Impaired Communication
KEY: Nursing Process Step: Implementation
MSC: NCLEX: Psychosocial Integrity: coping and adaptation

Downloaded by: SUCCEEDGRADES | abbieclin@gmail.com


Distribution
Downloaded by Dallen Mae D.ofKadir
this document is illegal
(kadir.dallenmae.d.bcsi@gmail.com)
lOMoARcPSD|35346190

Stuvia.com - The Marketplace to Buy and Sell your Study Material

48. When communicating with a hearing-impaired patient, the nurse appropriately:


a. shouts repeatedly at the patient.
b. speaks directly into the patient‘s ear.
c. uses long, complex sentences.
d. uses short, simple sentences.

ANS: D
When communicating with a hearing-impaired patient, the nurse appropriately uses short,
simple sentences. The nurse should not shout because this can distort speech and does not
make the message any clearer. The nurse should never speak directly into the person‘s ear.
This can distort the message and hide all visual cues.

DIF: Cognitive Level: Comprehension REF: p. 108, Box 8-1


OBJ: Clinical Practice #3 TOP: Impaired Communication
KEY: Nursing Process Step: Implementation
MSC: NCLEX: Psychosocial Integrity: coping and adaptation

49. When communicating with a preschooler, the nurse should:


a. use abstract explanations.
b. use unfamiliar language.
c. use long, complex sentences.
d. consider the developmental level, using familiar words.
ANS: D
When interacting with a toddler or a preschooler, the nurse should focus on the child‘s needs
and concerns. The nurse should also use simple, short sentences and concrete explanations
with familiar words.

DIF: Cognitive Level: Comprehension REF: p. 109 OBJ: Clinical Practice #3


TOP: Communication with Children KEY: Nursing Process Step: Implementation
MSC: NCLEX: Psychosocial Integrity: coping and adaptation

50. When communicating with an adolescent, the nurse should be very sensitive to avoid:
a. asking embarrassing questions.
b. offering advice.
c. interrupting frequently.
d. using active listening.
ANS: C
An adolescent needs time to talk. The nurse should use active listening, avoid interrupting,
and show acceptance. The nurse should try not to give advice.

DIF: Cognitive Level: Comprehension REF: p. 109 OBJ: Clinical Practice #3


TOP: Communication KEY: Nursing Process Step: Implementation
MSC: NCLEX: Psychosocial Integrity: coping and adaptation

51. The physician informs the student nurse that he would like to give a telephone order. The best
response by the student is:
a. document the telephone order on the physician‘s orders.
b. ask another student to listen as a witness to the telephone order.
c. tape-record the physician giving the order to the student nurse.
d. ask the registered nurse to take the telephone order.

Downloaded by: SUCCEEDGRADES | abbieclin@gmail.com


Distribution
Downloaded by Dallen Mae D.ofKadir
this document is illegal
(kadir.dallenmae.d.bcsi@gmail.com)
lOMoARcPSD|35346190

Stuvia.com - The Marketplace to Buy and Sell your Study Material

ANS: D
The student nurse should have an instructor or another registered nurse standing by to take the
new orders from the physician because students cannot legally take telephone orders.

DIF: Cognitive Level: Application REF: p. 110 OBJ: Theory #6


TOP: Telephone Orders KEY: Nursing Process Step: N/A
MSC: NCLEX: N/A

52. A nurse is delegating to a nursing assistant. The most appropriate form of this type of
communication would be:
a.―Let me know if Mr. Jones‘ temperature is high.‖
b.―I need to know if Mr. Jones‘ blood pressure is elevated.‖
c.―Come and get me if Mr. Jones has a high heart rate.‖
d.―If Mr. Jones‘ heart rate is greater than 100, let me know.‖
ANS: D
It is important to communicate well in order to assign tasks and delegate to others effectively.
The nurse should give clear, concise messages that include the desired results.

DIF: Cognitive Level: Comprehension REF: p. 110 OBJ: Theory #7


TOP: Delegation KEY: Nursing Process Step: Implementation
MSC: NCLEX: Safe Effective Care Environment: coordinated care

53. In order to safeguard patient information when using a computer, the nurse should:
a. only use the computer located in the nurse‘s station.
b. wait until the end of the shift and chart all information at one time.
c. use personal code words and abbreviations to disguise information.
d. change the computer password frequently.

ANS: D
Computerized patient information requires extra vigilance by the nurse to safeguard
confidentiality. Changing personal passwords frequently helps safeguard information. When
using the computer at a health care facility, the nurse must remember not to leave a computer
screen open when he or she is finished. The nurse should always log out so that someone else
cannot access information using his or her password and must not share his or her password
with others. Computers in the nurse‘s station are not as convenient as those at the bedside or
in the hall. Personal codes and abbreviations are not useful.

DIF: Cognitive Level: Comprehension REF: p. 111 OBJ: Theory #8


TOP: Patient Information Safety KEY: Nursing Process Step: N/A
MSC: NCLEX: N/A

COMPLETION

54. Pain is often conveyed through non-verbal communication. Two other common, non-verbally
expressed emotions are and .

ANS:
anxiety; fear
fear; anxiety

Downloaded by: SUCCEEDGRADES | abbieclin@gmail.com


Distribution
Downloaded by Dallen Mae D.ofKadir
this document is illegal
(kadir.dallenmae.d.bcsi@gmail.com)
lOMoARcPSD|35346190

Stuvia.com - The Marketplace to Buy and Sell your Study Material

Anxiety and fear can be expressed non-verbally by such behaviors as restlessness and picking
at the bed covers.

DIF: Cognitive Level: Knowledge REF: p. 100 OBJ: Theory #1


TOP: Nonverbal Communication KEY: Nursing Process Step: N/A
MSC: NCLEX: Physiological Integrity: basic care and comfort

55. To elicit more information from a patient, the nurse should ask questions that require more
than a one-word answer. This type of question is called .

ANS:
open-ended
Open-ended questions provide more information than can be gathered from closed questions.

DIF: Cognitive Level: Knowledge REF: p. 102 OBJ: Theory #1


TOP: Open-Ended Questions KEY: Nursing Process Step: Implementation
MSC: NCLEX: Psychosocial Integrity: coping and adaptation

56. The communication technique of gives the caregiver the opportunity to ask and
respond to questions.

ANS:
ISBAR-R
ISBAR-R format allows the opportunity to ask and respond to questions concerning patient
care during the end of shift report. The initials stand for introduction, situation, background,
assessment, recommendation, and readback.

DIF: Cognitive Level: Comprehension REF: p. 110 OBJ: Theory #9


TOP: ISBAR-R KEY: Nursing Process Step: Implementation
MSC: NCLEX: Safe Effective Care Environment: coordinated care

MULTIPLE RESPONSE

57. The nurse is alert to avoid using blocks to effective communication that include: (Select all
that apply.)
a. changing the subject.
b. using non-judgmental remarks.
c. giving advice.
d. asking probing questions.
e. offering hope.
f. using clichés.
ANS: A, C, D, F
Such behavior as changing the subject, giving advice, asking probing questions that probe into
a patient‘s motive, and using clichés all block communication. Offering hope and giving
remarks that are non-judgmental are appropriate forms of communication.

DIF: Cognitive Level: Comprehension REF: pp. 104-106 OBJ: Theory #3


TOP: Effective Communication KEY: Nursing Process Step: Implementation
MSC: NCLEX: Psychosocial Integrity: coping and adaptation

Downloaded by: SUCCEEDGRADES | abbieclin@gmail.com


Distribution
Downloaded by Dallen Mae D.ofKadir
this document is illegal
(kadir.dallenmae.d.bcsi@gmail.com)
lOMoARcPSD|35346190

Stuvia.com - The Marketplace to Buy and Sell your Study Material

58. During the initial interview of a patient, the nurse should: (Select all that apply.)
a. assess the language capabilities of the patient.
b. use open-ended questions.
c. limit the interview to approximately 30 minutes.
d. assess comprehension abilities of the patient.
e. make the patient as comfortable as possible.
f. obtain the patient‘s medical history from the physician.

ANS: A, C, D, E
During the initial assessment, the patient should be comfortable and the nurse should ask
closed questions to elicit specific information. The interview should last approximately 30
minutes, and the nurse needs to evaluate the language and comprehension skills of the patient
to ensure effective communication.

DIF: Cognitive Level: Comprehension REF: pp. 107-108 OBJ: Clinical Practice #1
TOP: Interview Skills KEY: Nursing Process Step: Implementation
MSC: NCLEX: Psychosocial Integrity: coping and adaptation

59. When using the telephone to communicate with a physician about a patient, the student nurse
should have ready: (Select all that apply.)
a. current information relative to patient‘s condition change.
b. assessment of vital signs.
c. information on urinary output.
d. patient‘s social security number or hospital identification number.
e. medications received.
ANS: A, B, C, E
As a rule the physician does not need to have the social security number or the hospital
identification number, but does need information on the patient‘s condition, vital signs,
urinary output, and medications received.

DIF: Cognitive Level: Comprehension REF: p. 110 OBJ: Theory #6


TOP: Telephone Communication with a Physician
KEY: Nursing Process Step: Implementation
MSC: NCLEX: Safe Effective Care Environment: coordinated care

60. The nurse will appropriately and deliberately use the closed question technique when the
patient is: (Select all that apply.)
a. being asked for specific information.
b. extremely anxious and unfocused.
c. having difficulty expressing feelings.
d. confused.
e. angry and ranting about his lack of medical care.
ANS: A, B, D
Closed questions are useful for gaining specific information such as age, address, and listing
of allergies. Closed questions help the anxious, confused, and unfocused patient to respond.
Patients who are having difficulty expressing feelings are not aided by closed questions.
Angry patients need to be helped by silence or general leads.

DIF: Cognitive Level: Application REF: p. 102 OBJ: Theory #3


TOP: Closed Questions KEY: Nursing Process Step: Intervention

Downloaded by: SUCCEEDGRADES | abbieclin@gmail.com


Distribution
Downloaded by Dallen Mae D.ofKadir
this document is illegal
(kadir.dallenmae.d.bcsi@gmail.com)
lOMoARcPSD|35346190

Stuvia.com - The Marketplace to Buy and Sell your Study Material

MSC: NCLEX: Psychosocial Integrity: coping and adaptation

61. Behaviors that indicate to the patient that the nurse is inattentive to the patient‘s concerns are
such activities as: (Select all that apply.)
a. turning back to straighten the bedside table while the patient is talking.
b. tapping feet or fingers.
c. sitting down in a chair near the bed with arms crossed.
d. leaving a hand on the door to go out.
e. nodding and asking for elaboration.
ANS: A, B, C, D
Turning from the patient, tapping the feet or fingers, sitting with arms crossed, and leaving the
patient all indicate to the patient that his or her concerns are not important and the information
is boring to the nurse. Nodding and asking for elaboration indicate that the nurse is attentive
and focused on his or her concerns.

DIF: Cognitive Level: Analysis REF: p. 105, Table 8-2


OBJ: Theory #3 TOP: Inattentive Listening
KEY: Nursing Process Step: Implementation
MSC: NCLEX: Psychosocial Integrity: coping and adaptation

Downloaded by: SUCCEEDGRADES | abbieclin@gmail.com


Distribution
Downloaded by Dallen Mae D.ofKadir
this document is illegal
(kadir.dallenmae.d.bcsi@gmail.com)
lOMoARcPSD|35346190

Stuvia.com - The Marketplace to Buy and Sell your Study Material

Chapter 09: Patient Teaching for Health Promotion


Test Bank

MULTIPLE CHOICE

1. Before beginning to teach a patient to give himself insulin, the nurse asks, ―Have you ever
known anyone who gave himself insulin injections?‖ This question is primarily designed to:
a. assess the patient‘s learning needs.
b. stimulate the patient to focus on the teaching goal.
c. reduce the patient‘s anxiety relative to insulin injection.
d. reduce the amount of information the nurse has to provide.

ANS: A
Assessing a patient‘s previous experience (as well as education, learning mode, and
motivation) gives the nurse valuable information in developing a teaching plan tailored to the
individual. It may reduce the amount of information needed, or it may increase it if some of
what the patient ―knows‖ is erroneous.

DIF: Cognitive Level: Analysis REF: p. 119 OBJ: Theory #3


TOP: Assessing Learning Needs KEY: Nursing Process Step: Assessment
MSC: NCLEX: Physiological Integrity: basic care and comfort

2. The nurse uses a syringe and vial of insulin to show how to draw up the correct dose while she
explains the procedure to the patient. To best promote learning, her next step should be to:
a. give the patient written materials to study and learn the procedure.
b. have the patient explain the procedure to the nurse to assess understanding.
c. give the patient a day to allow him to process and absorb the information.
d. have the patient practice the procedure with the nurse helping.

ANS: D
Kinesthetic, or hands-on, learning reinforces the visual demonstration. Immediate handling of
the materials reduces anxiety. Giving the patient reading materials or asking the patient to
explain verbally will not be as effective as the kinesthetic application.

DIF: Cognitive Level: Application REF: p. 116 OBJ: Theory #3


TOP: Modes of Learning KEY: Nursing Process Step: Implementation
MSC: NCLEX: Physiological Integrity: basic care and comfort

3. In teaching an 82-year-old patient to perform a dressing change to be done at home after


discharge, the nurse would adjust the teaching session to:
a. include another person in the instruction because an 82-year-old person will be
unable to master the technique.
b. slow the pace and frequently ask questions to assess comprehension.
c. speed through the details because age and experience will shorten learning time.
d. provide written material and diagrams alone.

Downloaded by: SUCCEEDGRADES | abbieclin@gmail.com


Distribution
Downloaded by Dallen Mae D.ofKadir
this document is illegal
(kadir.dallenmae.d.bcsi@gmail.com)
lOMoARcPSD|35346190

Stuvia.com - The Marketplace to Buy and Sell your Study Material

ANS: B
The older patient needs to have the pace slowed and have time to ask questions to confirm
comprehension. The inclusion of written materials to reinforce teaching is also good, but
should not be the only method of instruction.

DIF: Cognitive Level: Application REF: p. 118 OBJ: Theory #5


TOP: Factors Affecting Learning KEY: Nursing Process Step: Planning
MSC: NCLEX: Physiological Integrity: basic care and comfort

4. An 80-year-old patient is to be taught the process of colostomy irrigation and reattachment of


the colostomy bag. The nurse‘s initial assessment prior to instruction should address the
patient‘s:
a. understanding of the process of irrigation.
b. familiarity with the irrigation materials.
c. manual dexterity.
d. motivation to learn.

ANS: D
The patient‘s motivation to learn a new skill is essential to the success of the instruction. Some
patients need to see the advantage of independence to motivate them to learn. Manual
dexterity and basic understanding of materials and process are important, but initially the
motivation needs to be assessed.

DIF: Cognitive Level: Analysis REF: p. 119 OBJ: Clinical Practice #1


TOP: Motivation KEY: Nursing Process Step: Assessment
MSC: NCLEX: Physiological Integrity: basic care and comfort

5. The nurse can assess her patient‘s ability to read and comprehend written instructions by
doing which of the following?
a. Asking the patient, ―Did you graduate from high school?‖
b. Giving the patient a printed instruction sheet and saying, ―Some people have
difficulty with written instructions. Others find them helpful. Would these be
helpful to you?‖
c. Asking the patient, ―Are you able to read?‖
d. Giving the patient some printed materials and saying, ―After you have read this, I‘ll
ask you some questions about what‘s in them, to see if you‘ve learned it.‖
ANS: B
Graduation from high school does not guarantee reading comprehension. Actually reading
allows the nurse to know if the patient can read as well as comprehend.

DIF: Cognitive Level: Application REF: p. 119 OBJ: Theory #3


TOP: Assessing Literacy KEY: Nursing Process Step: Assessment
MSC: NCLEX: Physiological Integrity: basic care and comfort

Downloaded by: SUCCEEDGRADES | abbieclin@gmail.com


Distribution
Downloaded by Dallen Mae D.ofKadir
this document is illegal
(kadir.dallenmae.d.bcsi@gmail.com)
lOMoARcPSD|35346190

Stuvia.com - The Marketplace to Buy and Sell your Study Material

6. A patient being assessed for pre-operative learning needs says his mother had the same
surgery by the same surgeon 3 years ago. The nurse should design the teaching plan to:
a. do a brief review of the preoperative teaching, because the patient is already
familiar with the procedure.
b. teach thoroughly as the procedure may have changed.
c. simply give the patient a written list of preoperative instructions.
d. explore with the patient what he knows about the proposed surgery and add or
correct where necessary.
ANS: D
Assessing a patient‘s experience and knowledge allows the nurse to tailor the teaching to the
individual. The nurse should never assume that a patient ―knows‖ what he is supposed to
know and that teaching again what the patient already knows is a waste of time or insults the
patient‘s intelligence and experience. Giving a list of preoperative instructions is simply
impossible.

DIF: Cognitive Level: Analysis REF: p. 119 OBJ: Theory #4


TOP: Assessing Learning Needs KEY: Nursing Process Step: Assessment
MSC: NCLEX: Physiological Integrity: basic care and comfort

7. The nurse is aware that the knowledge deficit of a postpartum patient with her first child that
can be safely addressed by the community nurse after discharge is:
a. weaning the child from breast-feeding.
b. care of the patient‘s surgical incision.
c. feeding the baby by breast or bottle.
d. recognizing signs or symptoms of infection.
ANS: A
Priority teaching needs prior to discharge are those that have to do with physiologic or safety
needs. Thus feeding the baby, care of the incision (prevent infection), and recognition of signs
that affect safety must be addressed before discharge. Weaning will not occur until much later
and can be addressed safely by the home health nurse.

DIF: Cognitive Level: Comprehension REF: p. 116 OBJ: Theory #8


TOP: Prioritizing Learning Needs KEY: Nursing Process Step: Planning
MSC: NCLEX: Health Promotion and Maintenance: prevention and early detection of disease

8. The nurse evaluates the effectiveness of teaching relative to how to use an eye shield after eye
surgery is to:
a. have the patient tell the nurse what he is going to do.
b. have the patient demonstrate that he can secure the eye shield.
c. ask the patient if he has any questions related to the use of the shield.
d. call the patient at home in 3 days and ask if he has been wearing the shield.

ANS: B

Downloaded by: SUCCEEDGRADES | abbieclin@gmail.com


Distribution
Downloaded by Dallen Mae D.ofKadir
this document is illegal
(kadir.dallenmae.d.bcsi@gmail.com)
lOMoARcPSD|35346190

Stuvia.com - The Marketplace to Buy and Sell your Study Material

A return demonstration and explanation by the patient will evaluate whether the patient‘s
learning needs are met. Having the patient describe the process and ask questions might be
helpful but does not show that the patient can place the shield correctly (a psychomotor skill).
Evaluation of teaching should be done to allow time to revise the teaching plan if the patient is
unable to meet the behavioral objectives. Calling after discharge is too late to correct
problems.

DIF: Cognitive Level: Application REF: p. 120 OBJ: Theory #2


TOP: Evaluation of Learning KEY: Nursing Process Step: Evaluation
MSC: NCLEX: Physiological Integrity: basic care and comfort

9. The nurse will choose the best time to continue postoperative teaching regarding wound care
and dressings, which would be:
a. immediately after the patient has been medicated for pain.
b. just before the patient is discharged, so the information is current.
c. when the patient is comfortable and receptive to the teaching.
d. the last thing in the evening, after visitors have left, before bedtime.

ANS: C
A patient who is in pain, sedated from pain medication, or fatigued at the end of the day after
visitors leave will not be receptive to teaching. Teaching should begin before discharge to
improve learning.

DIF: Cognitive Level: Comprehension REF: p. 119 OBJ: Theory #3


TOP: Readiness to Learn KEY: Nursing Process Step: Planning
MSC: NCLEX: Physiological Integrity: basic care and comfort

10. A nurse plans to teach a 4-year-old about what to expect after his broken arm has been casted
by:
a. bringing a doll and casting materials to the room, showing the casting materials and
actually casting the doll‘s arm, and explaining the purpose of the cast.
b. telling the child that while he is asleep, the doctor will take off his arm and wrap it
up.
c. breaking up the teaching sessions into two separate 5-minute sessions.
d. being treated as an adult because this approach helps the child to feel ―grown up.‖

ANS: C
Children benefit from teaching that is geared toward their age and level of understanding.
Teaching in short sessions, allowing for the child‘s brief attention span, will enhance teaching.
Children are very literal and improbable stories will be believed.

DIF: Cognitive Level: Comprehension REF: p. 118 OBJ: Theory #5


TOP: Teaching a Child KEY: Nursing Process Step: Planning
MSC: NCLEX: Physiological Integrity: basic care and comfort

Downloaded by: SUCCEEDGRADES | abbieclin@gmail.com


Distribution
Downloaded by Dallen Mae D.ofKadir
this document is illegal
(kadir.dallenmae.d.bcsi@gmail.com)
lOMoARcPSD|35346190

Stuvia.com - The Marketplace to Buy and Sell your Study Material

11. The best way for a nurse to reinforce learning during a return demonstration by the patient is
for the nurse to:
a. give recognition and praise for the parts the patient does well and to assist or teach
when the patient becomes confused or forgetful.
b. watch quietly until the return demonstration is finished and then list the errors.
c. instruct the patient to read the written material again when an error is made.
d. stop the patient each time he makes a mistake and have him start again after the
nurse reviews the procedure with him.
ANS: A
Praise and ―walking through‖ the procedure reinforces learning.

DIF: Cognitive Level: Application REF: p. 117 OBJ: Theory #3


TOP: Teaching Methods KEY: Nursing Process Step: Implementation
MSC: NCLEX: Physiological Integrity: basic care and comfort

12. A patient states, ―I don‘t think I‘ll ever be able to give myself an injection.‖ The best reply by
the nurse is:
a. ―Everyone feels like that at first. You‘ll get over it.‖
b. ―Don‘t be afraid. It‘s an easy skill for anyone to learn.‖
c. ―What bothers you most about the idea of giving yourself an injection?‖
d. ―I know just how you feel. I would have trouble giving myself an injection.‖
ANS: C
When a patient lacks self-confidence, the nurse needs to explore the patient‘s feelings.

DIF: Cognitive Level: Application REF: p. 117 OBJ: Theory #4


TOP: Confidence and Abilities KEY: Nursing Process Step: Assessment
MSC: NCLEX: Physiological Integrity: basic care and comfort

13. The nurse takes into consideration that when using printed material for teaching a 65-year-old
Middle-Eastern patient who speaks perfect English, the nurse should:
a.use teaching material printed in English.
b.determine if the patient can read English.
c.engage a translator to read the English material to the patient.
d.use English material that is printed in bold type on white paper.
ANS: B
Determine if the patient is literate in English. If not, a translator may be able to rewrite the
instructions in the preferred language. Simply reading the English version is not helpful if the
patient is to refer to the material after discharge. Bold print will not help a person who does
not read English.

DIF: Cognitive Level: Application REF: p. 119 OBJ: Theory #3


TOP: Using Printed Materials KEY: Nursing Process Step: Assessment
MSC: NCLEX: Physiological Integrity: basic care and comfort

Downloaded by: SUCCEEDGRADES | abbieclin@gmail.com


Distribution
Downloaded by Dallen Mae D.ofKadir
this document is illegal
(kadir.dallenmae.d.bcsi@gmail.com)
lOMoARcPSD|35346190

Stuvia.com - The Marketplace to Buy and Sell your Study Material

14. Because a person may learn best in a particular manner, to improve patient teaching, the nurse
should:
a. ask the patient whether he learns best visually, aurally, or kinesthetically.
b. use a hands-on approach, because it works best for most people.
c. test the patient‘s reading comprehension before using visual handouts.
d. use a combination of the three modes of learning to enhance learning.

ANS: D
Many people do not know which mode of learning is their dominant one, and most people
learn best with a combination of teaching/learning techniques.

DIF: Cognitive Level: Knowledge REF: p. 116 OBJ: Theory #3


TOP: Learning Modalities KEY: Nursing Process Step: Implementation
MSC: NCLEX: Physiological Integrity: basic care and comfort

15. Once a teaching plan is formulated and placed in the nursing care plan for a hospitalized
patient:
a. one nurse will be designated to teach the plan on a priority basis.
b. behavioral objectives are used to identify expected outcomes.
c. it is printed and given to the patient as a guide for learning.
d. it outlines all that will be taught before the patient is discharged.
ANS: B
Behavioral objectives identify actions that can be measured; thus they serve as evaluation
tools of expected outcomes. Many people are involved in a teaching plan, with responsibility
designated in the plan. Not all of the teaching plan may be accomplished during the hospital
stay. Priorities identify which learning needs are most important to teach before discharge and
which can be taught by the community nurse after discharge.

DIF: Cognitive Level: Comprehension REF: p. 119 OBJ: Clinical Practice #2


TOP: Behavioral Objectives KEY: Nursing Process Step: N/A
MSC: NCLEX: Health Promotion and Maintenance: prevention and early detection of disease

16. In a skilled nursing home, a newly admitted resident becomes terminally ill following a
cerebrovascular accident (CVA). To diminish the family‘s anxiety, the nurse teaches the family
members about activities that are being performed to provide care and comfort to their loved
one. This teaching is provided in order to:
a. reduce the likelihood of a lawsuit over the anticipated death.
b. decrease the family‘s needs in the expression of their grief.
c. increase the family‘s comfort in their affective domain.
d. enable the family to be better prepared for the approaching death.

ANS: C
Teaching that addresses a person‘s feelings, beliefs, or values addresses the affective domain.

DIF: Cognitive Level: Comprehension REF: p. 116 OBJ: Theory #3


TOP: Affective Domain KEY: Nursing Process Step: Implementation

Downloaded by: SUCCEEDGRADES | abbieclin@gmail.com


Distribution
Downloaded by Dallen Mae D.ofKadir
this document is illegal
(kadir.dallenmae.d.bcsi@gmail.com)
lOMoARcPSD|35346190

Stuvia.com - The Marketplace to Buy and Sell your Study Material

MSC: NCLEX: Psychosocial Integrity: coping and adaptation

17. The nurse designing a teaching plan for a patient admitted to the hospital for treatment of a
heart problem after years of treating the ailment at home with herbal remedies and practices
common in his cultural group should:
a. help the patient to see that using herbal remedies has not worked in the past.
b. explain that cultural remedies may conflict with conventional medicine.
c. help the patient to identify optimum outcomes that can be achieved through
education and compromise.
d. ask family members to intervene for the cessation of the use of cultural remedies.

ANS: C
A person‘s cultural values must be considered in formulating a nursing care plan. Working
with the patient to identify what is of value to the patient can assist the nurse to plan care that
meets the patient‘s needs for education.

DIF: Cognitive Level: Application REF: p. 117 OBJ: Theory #4


TOP: Cultural Values and Expectations KEY: Nursing Process Step: Planning
MSC: NCLEX: Psychosocial Integrity: coping and adaptation

18. The nurse recognizes the American Indian‘s need for the intervention of a shaman in dealing
with illness because the shaman helps the patient in seeking:
a. a sense of peace and harmony with nature.
b. a spiritual route to healing.
c. healing through the domination over evil.
d. support from deceased ancestors.

ANS: B
The American Indian has a strong belief that spiritual healing is essential to physical health.

DIF: Cognitive Level: Knowledge REF: p. 117 OBJ: Clinical Practice #2


TOP: Cultural Values and Expectations KEY: Nursing Process Step: Planning
MSC: NCLEX: Psychosocial Integrity: coping and adaptation

19. The nurse appropriately begins discharge planning when:


a. the physician writes orders to discharge the patient.
b. the patient feels ready to be discharged home.
c. it is anticipated the patient will be discharged in 8 hours.
d. the patient is admitted to the health care facility.

ANS: D
Discharge planning requires looking ahead in order to meet the patient‘s ongoing needs at
home. It is a process that begins at the time of admission.

DIF: Cognitive Level: Comprehension REF: p. 119 OBJ: Theory #1


TOP: Discharge Planning KEY: Nursing Process Step: Implementation
MSC: NCLEX: Health Promotion and Maintenance: prevention and early detection of disease

Downloaded by: SUCCEEDGRADES | abbieclin@gmail.com


Distribution
Downloaded by Dallen Mae D.ofKadir
this document is illegal
(kadir.dallenmae.d.bcsi@gmail.com)
lOMoARcPSD|35346190

Stuvia.com - The Marketplace to Buy and Sell your Study Material

20. A nurse is showing a diabetic patient how to draw insulin out of a syringe. The mode of
learning that the nurse is using is learning.
a. auditory
b. visual
c. kinesthetic
d. oral
ANS: B
Visual learning is based on learning through what the learner sees.

DIF: Cognitive Level: Comprehension REF: p. 116 OBJ: Theory #3


TOP: Modes of Learning KEY: Nursing Process Step: Implementation
MSC: NCLEX: Physiological Integrity: basic care and comfort

21. The nurse will plan to offer the teaching session in a quiet area in order to:
a. ensure that the patient can hear what the nurse says.
b. reduce distractions.
c. provide absolute privacy.
d. make the environment more like a classroom.

ANS: B
Teaching sessions are best done in a quiet environment to reduce distractions.

DIF: Cognitive Level: Comprehension REF: p. 119 OBJ: Theory #4


TOP: Enhancing Learning KEY: Nursing Process Step: Planning
MSC: NCLEX: Physiological Integrity: basic care and comfort

22. When a nurse is ―talking through‖ a procedure or assisting the patient to learn, the nurse
encourages the patient to:
a. close her eyes and envision the process.
b. read the listed steps written on a poster board on the wall.
c. write down the steps as she performs them.
d. verbalize each step until the steps are memorized.

ANS: C
Writing down the steps as they are performed provides a guide in the patient‘s own words that
can be followed independently.

DIF: Cognitive Level: Application REF: p. 120 OBJ: Theory #3


TOP: Modes of Learning KEY: Nursing Process Step: Implementation
MSC: NCLEX: Physiological Integrity: basic care and comfort

23. A nurse who is communicating with a school-age child about receiving anesthesia for surgery
later this afternoon would best describe the process by saying:
a. ―The doctor who will be wearing a mask will put a needle in your arm and then you
go to sleep for a long time.‖

Downloaded by: SUCCEEDGRADES | abbieclin@gmail.com


Distribution
Downloaded by Dallen Mae D.ofKadir
this document is illegal
(kadir.dallenmae.d.bcsi@gmail.com)
lOMoARcPSD|35346190

Stuvia.com - The Marketplace to Buy and Sell your Study Material

b. ―You will just float off to dreamland and after you come back your tonsils will have
been cut out.‖
c. ―After the doctor puts medicine in your arm, you will ride on a pony to where
fairies will take out your tonsils. Then you will ride right back here.‖
d. ―You will be given a ride on a special bed to a big room where the doctor will give
you some medicine that will make you very sleepy.‖
ANS: D
Children interpret language literally, so avoid idioms or stories that might be frightening
because they can be easily misunderstood. Language should be tailored to the child‘s
understanding.

DIF: Cognitive Level: Application REF: p. 118 OBJ: Theory #5


TOP: Communication with School-Age Child
KEY: Nursing Process Step: Implementation
MSC: NCLEX: Physiological Integrity: basic care and comfort

24. When teaching an elderly patient about changing his dressing, the nurse would most
appropriately:
a. be certain the patient is wearing his glasses and/or hearing aid.
b. talk through the process rapidly to keep the patient from becoming tired.
c. wait for the patient to ask any questions about the procedure.
d. point out each mistake during the return demonstration.

ANS: A
Special considerations when teaching the elderly include being certain the patient is wearing
glasses and/or a hearing aid that is turned on and adjusted, if needed. Short sentences should
be used, and the nurse should speak slowly. Pointing out mistakes without any praise can
diminish the confidence of the patient.

DIF: Cognitive Level: Application REF: p. 118 OBJ: Theory #5


TOP: Communication with the Elderly KEY: Nursing Process Step: Implementation
MSC: NCLEX: Physiological Integrity: basic care and comfort

25. The nurse would identify an opportunity for a ―teachable moment‖ in the situation of a patient
who:
a.has just been told of the malignancy of his tumor.
b.says, ―How will I remember all the things about my new diet?‖
c.has just returned from surgery for a deviated septum.
d.is packing belongings in preparation for discharge.
ANS: B
The ―teachable moment‖ occurs when the patient is at an optimal level of readiness to learn
and shows a willingness to apply that information.

DIF: Cognitive Level: Application REF: p. 115 OBJ: Theory #4


TOP: Teachable Moment KEY: Nursing Process Step: Assessment

Downloaded by: SUCCEEDGRADES | abbieclin@gmail.com


Distribution
Downloaded by Dallen Mae D.ofKadir
this document is illegal
(kadir.dallenmae.d.bcsi@gmail.com)
lOMoARcPSD|35346190

Stuvia.com - The Marketplace to Buy and Sell your Study Material

MSC: NCLEX: Physiological Integrity: basic care and comfort

MULTIPLE RESPONSE

26. Continuous learning needs for the patient upon discharge should be communicated to the:
(Select all that apply.)
a. visiting nurse.
b. family.
c. primary care physician.
d. pharmacy or medical supply facility.
e. home health aide.

ANS: A, B, C
Health care entities that need to be aware of post-discharge continuous learning needs include
the visiting home health nurse, the family, and the primary care physician. Pharmacies and
medical supply facilities are not notified. The home health aide will not be instructed by the
home health nurse.

DIF: Cognitive Level: Comprehension REF: p. 115 OBJ: Theory #8


TOP: Continued Teaching After Discharge
KEY: Nursing Process Step: Planning
MSC: NCLEX: Health Promotion and Maintenance: prevention and early detection of disease

27. The nurse reminds the patient that health instruction supports the goals of Healthy People
2020, which include: (Select all that apply.)
a. promoting healthy behavior.
b. increasing the life span.
c. providing equipment for self-care.
d. ensuring access to adequate health care.
e. strengthening community relationships.
ANS: A, D, E
Health instruction supports the goals of Healthy People 2020, which include promoting
healthy behaviors, protecting health, ensuring access to quality health care, and strengthening
community health promotion programs.

DIF: Cognitive Level: Knowledge REF: p. 115 OBJ: Theory #2


TOP: Healthy People 2020 KEY: Nursing Process Step: Implementation
MSC: NCLEX: Health Promotion and Maintenance: prevention and early detection of disease

28. The nurse is aware that the major modes of learning are: (Select all that apply.)
a. oral.
b. tactile.
c. auditory.
d. kinesthetic.
e. gustatory.

Downloaded by: SUCCEEDGRADES | abbieclin@gmail.com


Distribution
Downloaded by Dallen Mae D.ofKadir
this document is illegal
(kadir.dallenmae.d.bcsi@gmail.com)
lOMoARcPSD|35346190

Stuvia.com - The Marketplace to Buy and Sell your Study Material

f. visual.
ANS: C, D, F
Kinesthetic, auditory, and visual are the major modes of learning.

DIF: Cognitive Level: Knowledge REF: p. 116 OBJ: Theory #3


TOP: Modes of Learning KEY: Nursing Process Step: Planning
MSC: NCLEX: Physiological Integrity: basic care and comfort

29. The LPN/LVN is qualified to give patient teaching on information relative to: (Select all that
apply.)
a. disease process.
b. postoperative care.
c. prognosis.
d. rehabilitation.
e. disaster preparedness.
ANS: A, B, D, E
LPNs and LVNs are qualified to give teaching on topics relative to disease process,
postoperative care, rehabilitation, and disaster preparedness. Information on prognosis is not
appropriate.

DIF: Cognitive Level: Comprehension REF: p. 116 OBJ: Theory #1


TOP: Areas of Patient Teaching KEY: Nursing Process Step: N/A
MSC: NCLEX: N/A

Downloaded by: SUCCEEDGRADES | abbieclin@gmail.com


Distribution
Downloaded by Dallen Mae D.ofKadir
this document is illegal
(kadir.dallenmae.d.bcsi@gmail.com)
lOMoARcPSD|35346190

Stuvia.com - The Marketplace to Buy and Sell your Study Material

Chapter 10: Delegation, Leadership, and Management


Test Bank

MULTIPLE CHOICE

1. Leadership is best defined as a process that:


a. motivates people to accomplish set goals.
b. provides a framework for health care delivery systems.
c. guides staff to use resources to meet patient needs.
d. uses advanced management training.

ANS: C
A comprehensive process that guides staff to use resources to meet patient needs best defines
leadership.

DIF: Cognitive Level: Knowledge REF: p. 123 OBJ: Theory #1


TOP: Leadership KEY: Nursing Process Step: N/A MSC: NCLEX: N/A

2. The best description of an autocratic leader is a leader who:


a. is permissive.
b. has confidence in the staff.
c. tightly controls team members.
d. accepts all responsibility for the team.

ANS: C
An autocratic leader tightly controls team members and closely monitors the work of each
staff member.

DIF: Cognitive Level: Knowledge REF: p. 124 OBJ: Theory #1


TOP: Leadership KEY: Nursing Process Step: N/A MSC: NCLEX: N/A

3. A laissez-faire leader would be most likely to:


a. consult staff members.
b. tightly control team members.
c. allow team members to function independently.
d. set goals that are task oriented.

ANS: C
A laissez-faire leader does not attempt to control the team and offers little direction. The
leader allows the team members to function independently.

DIF: Cognitive Level: Knowledge REF: p. 124 OBJ: Theory #1


TOP: Leadership KEY: Nursing Process Step: N/A MSC: NCLEX: N/A

4. A team leader with effective communication skills would:


a. make precise authoritarian assignments to team members.

Downloaded by: SUCCEEDGRADES | abbieclin@gmail.com


Distribution
Downloaded by Dallen Mae D.ofKadir
this document is illegal
(kadir.dallenmae.d.bcsi@gmail.com)
lOMoARcPSD|35346190

Stuvia.com - The Marketplace to Buy and Sell your Study Material

b. give specific information in a tactful, friendly manner.


c. maintain eye contact when giving directions.
d. limit time for feedback and complaints.

ANS: B
Effective communication needs to be concise and delivered in a tactful and friendly manner
and allows for feedback and two-way communication. Limitation of flow of feedback and
complaints and precise authoritarian behavior limit two-way communication.

DIF: Cognitive Level: Comprehension REF: p. 124 OBJ: Theory #2


TOP: Leadership KEY: Nursing Process Step: N/A MSC: NCLEX: N/A

5. The most effective communication from a nurse leader to a team member that is most likely to
have a positive outcome would be:
a. ―Jane, be sure to get those vital signs recorded on time today.‖
b. ―Jane, I need those vital signs before breakfast.‖
c. ―Jane, please give me a list of those vital signs before breakfast.‖
d. ―Jane, breakfast trays are being served. You need to get those vital signs.‖
ANS: C
Communication should be clear and concise, but also accompanied by a polite ―please‖ and
―thank you.‖ Effective communication demonstrates a sense of trust and respect toward
others.

DIF: Cognitive Level: Application REF: p. 125 OBJ: Theory #3


TOP: Leadership KEY: Nursing Process Step: N/A MSC: NCLEX: N/A

6. Prior to addressing a situation, the nurse is aware that an effective leader must the
problem.
a. define
b. identify persons to address
c. know the legal implications of
d. look to alternatives to address

ANS: A
The nurse leader must first be clear about what a problem involves before attempting to
address the situation.

DIF: Cognitive Level: Application REF: p. 125 OBJ: Theory #3


TOP: Problem Solving KEY: Nursing Process Step: Assessment
MSC: NCLEX: Physiological Integrity: basic care and comfort

7. The nurse is aware that the best way to evaluate an unlicensed assistive personnel‘s (UAP‘s)
ability to perform a skill or task is to:
a. obtain verbal confirmation from another nurse that the nursing assistant is
proficient.
b. review documentation that the nursing assistant is competent in skills.

Downloaded by: SUCCEEDGRADES | abbieclin@gmail.com


Distribution
Downloaded by Dallen Mae D.ofKadir
this document is illegal
(kadir.dallenmae.d.bcsi@gmail.com)
lOMoARcPSD|35346190

Stuvia.com - The Marketplace to Buy and Sell your Study Material

c. observe the nursing assistant performing the skill or task.


d. demonstrate the skill to the nursing assistant before his demonstration.

ANS: C
The best way to evaluate a person‘s ability to perform a skill is to observe him.
Documentation and information from another nurse support a nurse‘s decision to delegate the
task to the nursing assistant but are not the best means for evaluation of competency.

DIF: Cognitive Level: Application REF: p. 126 OBJ: Theory #4


TOP: Delegation KEY: Nursing Process Step: Assessment
MSC: NCLEX: Safe Effective Care Environment: safety and infection control

8. The nurse‘s most appropriate selection of a task to be delegated to an unlicensed assistive


personnel (UAP) would be:
a. assessing circulation in the toes of a patient in a cast.
b. changing a patient‘s wound dressing.
c. taking the blood pressure of a patient who has just returned from surgery.
d. toileting a patient on a bladder-training regimen.

ANS: D
Assessing tissue perfusion, changing a patient‘s wound dressing, and assessing a patient
require interventions by licensed personnel. Toileting a patient on a routine basis is
appropriate to delegate to a nursing assistant.

DIF: Cognitive Level: Application REF: p. 126, Box 10-2


OBJ: Theory #4 TOP: Delegation KEY: Nursing Process Step: Planning
MSC: NCLEX: Safe Effective Care Environment: safety and infection control

9. The nurse is aware that when a task is delegated to an unlicensed assistive personnel (UAP),
the nurse is:
a. no longer responsible to that patient.
b. responsible to communicate outcome to appropriate senior staff.
c. responsible for overall patient care.
d. liable for all adverse outcomes.
ANS: C
Nurses are responsible for all patients‘ care, regardless of tasks they have delegated to other
staff members.

DIF: Cognitive Level: Comprehension REF: p. 126 OBJ: Theory #4


TOP: Delegation KEY: Nursing Process Step: Planning
MSC: NCLEX: Safe Effective Care Environment: safety and infection control

10. The nurse recognizes that one of the responsibilities of a charge nurse as opposed to the team
leader is that the charge nurse is responsible for:
a. evaluating members of the health care team.
b. evaluating unlicensed assistive personnel.

Downloaded by: SUCCEEDGRADES | abbieclin@gmail.com


Distribution
Downloaded by Dallen Mae D.ofKadir
this document is illegal
(kadir.dallenmae.d.bcsi@gmail.com)
lOMoARcPSD|35346190

Stuvia.com - The Marketplace to Buy and Sell your Study Material

c. making rounds and assessing all patients on the unit.


d. collaborating with physicians and other health team members.

ANS: C
The charge nurse is an advanced leadership role, which includes the assessments of all
patients on the unit, and is responsible for the total nursing care during the shift.

DIF: Cognitive Level: Comprehension REF: p. 127 OBJ: Theory #6


TOP: Advanced Leadership Roles KEY: Nursing Process Step: N/A
MSC: NCLEX: N/A

11. When transcribing a physician‘s orders, the nurse must:


a. transcribe only the generic name of the drug.
b. give a copy of the order to the nurse responsible for that patient‘s care.
c. review unclear orders with the charge nurse.
d. transfer orders to the Medication Administration Record (MAR).

ANS: D
Transcribing orders means to transfer them to the Kardex and/or MAR. Preferably both the
generic and trade name of the drug should be recorded. The nurse giving the actual care
should be informed of the order change but does not need a copy of the order. Questions
relative to unclear orders are clarified with the prescribing physician.

DIF: Cognitive Level: Application REF: p. 129 OBJ: Clinical Practice #5


TOP: Written Orders KEY: Nursing Process Step: Implementation
MSC: NCLEX: Safe Effective Care Environment: coordinated care

12. The example of a physician‘s order that was received by telephone that has all the essential
documentation is:
a. Increase Coumadin to 3 mg daily: order from A. Doctor received by A. Nurse
11/11/11 @ 0930.
b. 11/11/11 @ 0930 TO. From A. Doctor: Increase Coumadin (Warfarin) to 3 mg
daily. A. Nurse, LVN.
c. TO: from A. Doctor: Increase Coumadin to 3 mg every day. A. Nurse.
d. 11/11/11 received TO to increase Coumadin (Warfarin) to 3 mg daily A. Nurse.

ANS: B
Documentation for a telephone order should be recorded as a TO with the date and time of the
receipt of the order, the order as requested by the physician, and signed with a first initial and
last name with professional designation.

DIF: Cognitive Level: Analysis REF: p. 130 OBJ: Clinical Practice #5


TOP: Transcribing Telephone Orders KEY: Nursing Process Step: N/A
MSC: NCLEX: N/A

13. The nurse in a long-term care facility is aware that the accuracy and comprehensiveness of the
Minimum Data Set (MDS) facilitate the:

Downloaded by: SUCCEEDGRADES | abbieclin@gmail.com


Distribution
Downloaded by Dallen Mae D.ofKadir
this document is illegal
(kadir.dallenmae.d.bcsi@gmail.com)
lOMoARcPSD|35346190

Stuvia.com - The Marketplace to Buy and Sell your Study Material

a. patient receiving the appropriate treatments.


b. effectiveness of the planning for quality improvement.
c. reimbursement to the facility from Medicare and Medicaid payments.
d. enhancement of the database for procedures related to risk management.
ANS: C
MDS sheets must be filled out correctly for facilities to receive the maximal Medicare or
Medicaid payment for services rendered.

DIF: Cognitive Level: Comprehension REF: p. 131 OBJ: Clinical Practice #6


TOP: Documentation for Reimbursement KEY: Nursing Process Step: N/A
MSC: NCLEX: N/A

14. A nurse can best minimize the liability of a lawsuit against the facility by:
a. strictly adhering to the policies and procedures of the facility.
b. demonstrating concern and attending to patient complaints.
c. advising the supervisor of significant events or problems with patient relations.
d. giving a copy of the policy and procedures to each patient.

ANS: B
A nurse can best minimize liability for a lawsuit by demonstrating concern and attending to
patient complaints and concerns. Adherence to procedures and policies may be a defense in a
lawsuit but does not necessarily diminish the risk. Advising the supervisor of a significant
event does not diminish the risk.

DIF: Cognitive Level: Comprehension REF: p. 131 OBJ: Clinical Practice #7


TOP: Risk Reduction KEY: Nursing Process Step: N/A
MSC: NCLEX: N/A

15. Nursing documentation that would best assist with reimbursement to a facility would be:
a. Patient was up in chair for 2 hours after breakfast.
b. Patient was ambulated in the hall three times this shift.
c. Patient was assisted to a recliner chair to use the oxygen concentrator.
d. Patient received bath in shower room.

ANS: C
Nurses should document care delivered and equipment used in order to get appropriate
reimbursement.

DIF: Cognitive Level: Application REF: p. 131 OBJ: Clinical Practice #6


TOP: Leadership KEY: Nursing Process Step: N/A MSC: NCLEX: N/A

16. The behavior least likely seen in an autocratic leader would be a person who:
a. provides close supervision of work by staff members.
b. often consults staff when making decisions.
c. quickly points out mistakes made by staff members.
d. frequently gives out new directives.

Downloaded by: SUCCEEDGRADES | abbieclin@gmail.com


Distribution
Downloaded by Dallen Mae D.ofKadir
this document is illegal
(kadir.dallenmae.d.bcsi@gmail.com)
lOMoARcPSD|35346190

Stuvia.com - The Marketplace to Buy and Sell your Study Material

ANS: B
The autocratic leader never consults staff relative to decision making. The autocratic leader
tightly controls team members. This type of leadership style has been described as ―my way or
the highway.‖ The leader closely supervises the work of each staff member. When mistakes
are made, they are quickly pointed out.

DIF: Cognitive Level: Comprehension REF: p. 124 OBJ: Theory #1


TOP: Leadership KEY: Nursing Process Step: N/A MSC: NCLEX: N/A

17. The nurse recognizes that the use of the democratic approach in leadership will:
a. cause team members to feel that their feelings are secondary to the accomplishment
of the goal.
b. cause mistakes to be discovered and pointed out.
c. take more time to accomplish goals.
d. use the leader‘s skills and knowledge.

ANS: C
The democratic process will take more time to accomplish the goal because the leader
frequently consults with staff members and seeks staff participation in decision making. The
skills and knowledge of the team members are readily used to ensure that the team functions
efficiently. Team members are respected as individuals, and there is an open and trusting
attitude overall. The democratic leader is part of the team, not sitting above it, and accepts
responsibility for the actions of the team.

DIF: Cognitive Level: Comprehension REF: p. 124 OBJ: Theory #1


TOP: Leadership KEY: Nursing Process Step: N/A MSC: NCLEX: N/A

18. An inappropriate delegation to an unlicensed assistive personnel (UAP) would be:


a. applying a condom catheter.
b. assessing a patient‘s pain.
c. giving a sitz bath.
d. giving an enema.

ANS: B
Assessment or aspects of the analysis, planning, or evaluation phases of the nursing process
must be performed by the registered nurse. These functions cannot be delegated to UAPs.

DIF: Cognitive Level: Application REF: p. 124 OBJ: Theory #4


TOP: Leadership KEY: Nursing Process Step: N/A MSC: NCLEX: N/A

19. The nurse is aware that assignment of an unlicensed assistive personnel


(UAP) differs from delegation in that in delegation, the:
a. nurse has transferred the authority to perform the task.
b. UAPs are covered by their own certification.
c. UAPs are performing activities within the scope of their job description.
d. licensed person must accompany the UAP to supervise the activity.

Downloaded by: SUCCEEDGRADES | abbieclin@gmail.com


Distribution
Downloaded by Dallen Mae D.ofKadir
this document is illegal
(kadir.dallenmae.d.bcsi@gmail.com)
lOMoARcPSD|35346190

Stuvia.com - The Marketplace to Buy and Sell your Study Material

ANS: A
Delegation occurs when a licensed nurse transfers the authority to perform a specific task.

DIF: Cognitive Level: Comprehension REF: p. 125 OBJ: Theory #4


TOP: Delegation KEY: Nursing Process Step: N/A MSC: NCLEX: N/A

20. The nurse is initially guided in the process of delegation to an unlicensed assistive personnel
(UAP) by the:
a. state‘s nurse practice act.
b. competencies of the UAP.
c. policies of the facility.
d. needs of the patient.

ANS: A
The initial delegation guidelines are described by the state‘s nurse practice act. The policies of
the facility may further limit what delegation may occur regardless of the competencies of the
UAP.

DIF: Cognitive Level: Knowledge REF: p. 126 OBJ: Theory #4


TOP: Delegation Guidelines KEY: Nursing Process Step: N/A
MSC: NCLEX: N/A

21. When the team leader becomes concerned about the frequent tardiness of the unlicensed
assistive personnel (UAP), the most appropriate documentation would be:
a. Has been late to work 6 times in this pay period (January 2, 4, 6, 8, 10, and 14).
States she overslept. Counseled on January 9. Unit management made aware.
b. Frequently late to work. States she overslept. Co-workers state she is working
elsewhere at night. Will follow up.
c. Tardiness is causing inability to finish assignments without assistance from others.
d. Oversleeping and active social life have caused tardiness six times in this pay
period.
ANS: A
Documentation of the specific facts (not opinion) should occur. The unit manager should also
be made aware of the problem.

DIF: Cognitive Level: Application REF: p. 127 OBJ: Theory #6


TOP: Leadership KEY: Nursing Process Step: N/A MSC: NCLEX: N/A

22. The LPN/LVN who is transcribing orders is unclear about the intent of an order. The
LPN/LVN should:
a.consult the charge nurse.
b.call the physician.
c.transcribe the order as written.
d.mark the order in red as UNTRANSCRIBED.
ANS: B

Downloaded by: SUCCEEDGRADES | abbieclin@gmail.com


Distribution
Downloaded by Dallen Mae D.ofKadir
this document is illegal
(kadir.dallenmae.d.bcsi@gmail.com)
lOMoARcPSD|35346190

Stuvia.com - The Marketplace to Buy and Sell your Study Material

An order that is unclear should be clarified with the physician.

DIF: Cognitive Level: Application REF: p. 129 OBJ: Clinical Practice #5


TOP: Order Transcription KEY: Nursing Process Step: N/A
MSC: NCLEX: N/A

23. The best communication given by the nurse to the unlicensed assistive personnel (UAP)
would be:
a. ―Let me know if my patient‘s temperature is high.‖
b. ―Please measure each urine output and report it to me.‖
c. ―Tell me if the patient seems more uncomfortable.‖
d. ―Notify me when the dressing needs to be changed.‖

ANS: B
When delegating, the nurse should make it clear as to what the unlicensed assistive personnel
(UAP) is to do. Nurses should not ask UAPs to perform assessments where professional
judgment is required about patient condition.

DIF: Cognitive Level: Application REF: pp. 125-126 OBJ: Theory #4


TOP: Communication KEY: Nursing Process Step: Implementation
MSC: NCLEX: Safe Effective Care Environment: safety and infection control

24. The team leader who is reviewing the list of the assigned patients would give priority to the
patient who:
a. has a scheduled medication due.
b. requires dressing changes three times a day.
c. is experiencing acute chest pain.
d. is confused and disoriented.

ANS: C
Unstable patients take precedence over stable patients. Scheduled medications and treatments
must be done before tasks that are ordered ―three times a day.‖

DIF: Cognitive Level: Analysis REF: p. 128 OBJ: Theory #6


TOP: Prioritization KEY: Nursing Process Step: Planning
MSC: NCLEX: Physiological Integrity: basic care and comfort

25. A nurse is aware that the medication orders on the MAR/Kardex should be verified with the
chart orders every:
a. shift.
b. 12 hours.
c. 24 hours.
d. 48 hours.
ANS: C
Medications recorded on the MAR/Kardex should be verified with the chart orders every 24
hours.

Downloaded by: SUCCEEDGRADES | abbieclin@gmail.com


Distribution
Downloaded by Dallen Mae D.ofKadir
this document is illegal
(kadir.dallenmae.d.bcsi@gmail.com)
lOMoARcPSD|35346190

Stuvia.com - The Marketplace to Buy and Sell your Study Material

DIF: Cognitive Level: Comprehension REF: p. 129 OBJ: Clinical Practice #5


TOP: Verification of Orders KEY: Nursing Process Step: N/A
MSC: NCLEX: N/A

26. The considerations for an LPN/LVN to be eligible for an advanced leadership role as a charge
nurse include a minimum of staff nursing experience of months.
a. 12
b. 18
c. 24
d. 36

ANS: A
A minimum of 12 months is required for an LPN/LVN to be considered for the role of charge
nurse.

DIF: Cognitive Level: Knowledge REF: p. 127 OBJ: Theory #6


TOP: Prioritization KEY: Nursing Process Step: Implementation
MSC: NCLEX: N/A

27. The nurse is aware that orders for a patient going to surgery are:
a. pertinent only for the immediate preoperative period.
b. canceled when the physician writes, ―resume previous orders.‖
c. can be continued when the patient returns to the unit.
d. canceled by the nurse in the operating room when the surgery is complete.

ANS: A
The preoperative orders are pertinent only for the preoperative period. All orders written
preoperatively are considered canceled at the time the patient enters surgery. Brand new
orders must be written in their entirety for the postsurgical patient. ―Resume previous orders‖
is not acceptable by most institutional policies.

DIF: Cognitive Level: Application REF: p. 130 OBJ: Clinical Practice #5


TOP: Physician Orders KEY: Nursing Process Step: Implementation
MSC: NCLEX: N/A

28. The nurse receiving a telephone order from a physician who is following the Joint
Commission International Center for Patient Safety should:
a. receive the order from the physician, transcribe the order to the chart, and ask the
physician if he will confirm the order.
b. receive the order from the physician, repeat the order to the physician, and then
transcribe the order to the chart.
c. receive the order from the physician, transcribe the order to the chart, and then read
back the order to the physician.
d. write the telephone order verbatim as the physician gives it and then read it back to
the charge nurse.

Downloaded by: SUCCEEDGRADES | abbieclin@gmail.com


Distribution
Downloaded by Dallen Mae D.ofKadir
this document is illegal
(kadir.dallenmae.d.bcsi@gmail.com)
lOMoARcPSD|35346190

Stuvia.com - The Marketplace to Buy and Sell your Study Material

ANS: C
The Joint Commission requires institutions to verify verbal or telephone orders by having the
person receiving the order ―read back‖ the order to the person initiating the order, usually the
physician. This ―read back‖ requires that the person accepting the order actually write the
order down in the chart in order to be reading it back.

DIF: Cognitive Level: Application REF: p. 130 OBJ: Theory #9


TOP: Read Back KEY: Nursing Process Step: N/A MSC: NCLEX: N/A

COMPLETION

29. Staff nurses have many responsibilities to their patients throughout their shift. However, the
is responsible for the total nursing care of patients during a shift and is also
responsible for giving reports to the oncoming shift and evaluating members of the health care
team.

ANS:
charge nurse
All nurses need to know the responsibilities of each person on the health care team, and the
charge nurse is responsible for the total nursing care of patients during a shift.

DIF: Cognitive Level: Knowledge REF: p. 127 OBJ: Theory #6


TOP: Charge Nurse Role KEY: Nursing Process Step: N/A
MSC: NCLEX: N/A

MULTIPLE RESPONSE

30. Characteristics of effective communication would include: (Select all that apply.)
a. using eye contact.
b. using concise statements when giving information.
c. addressing conflicts before delegation of duties.
d. obtaining feedback about directions given.
e. assigning responsibility for creation of any conflict.

ANS: A, B, D
Effective communication includes using direct eye contact between the persons involved,
using concise statements for clarity, and obtaining feedback to ensure that information has
been understood. Conflict resolution may or may not pertain to communication and may be
addressed following instructions.

DIF: Cognitive Level: Comprehension REF: p. 124 OBJ: Theory #2


TOP: Effective Communication KEY: Nursing Process Step: N/A
MSC: NCLEX: N/A

Downloaded by: SUCCEEDGRADES | abbieclin@gmail.com


Distribution
Downloaded by Dallen Mae D.ofKadir
this document is illegal
(kadir.dallenmae.d.bcsi@gmail.com)
lOMoARcPSD|35346190

Stuvia.com - The Marketplace to Buy and Sell your Study Material

31. Which statements are true regarding delegation of duties to unlicensed assistive personnel
(UAPs) by a licensed nurse? (Select all that apply.)
a. The LPN/LVN in charge must be familiar with the competency of staff.
b. The LPN/LVN must be familiar with the job descriptions of UAPs.
c. An LPN/LVN may delegate any skill or task to a UAP once the nursing assistant
has demonstrated proficiency.
d. A nurse must be familiar with the nurse practice act.
e. The certification of the UAP makes nursing assistants liable for their actions.

ANS: A, B, D
Delegation requires that the LPN/LVN be familiar with the competency of the staff as well as
their job description. Even if the nursing assistant is proficient in a skill, no delegation
requiring professional judgment is allowed. The nursing assistant is not liable; the supervising
nurse is.

DIF: Cognitive Level: Application REF: pp. 125-126 OBJ: Theory #4


TOP: Delegation KEY: Nursing Process Step: N/A MSC: NCLEX: N/A

32. Important characteristics of delegation to an unlicensed assistive personnel (UAP) include:


(Select all that apply.)
a. use of effective communication.
b. provision of constructive criticism immediately.
c. direction for the desired result and time for completion.
d. provision of tactful feedback.
e. informing the patient that the delegated task will be performed by a UAP.
ANS: A, C, D, E
When delegating to UAPs, it is important to use effective communication, as well as direction
for results. Patients should be informed that the task will be performed by a UAP.
Constructive criticism should be done privately after the event and should always be done
tactfully.

DIF: Cognitive Level: Comprehension REF: pp. 126-127 OBJ: Theory #4


TOP: Delegation KEY: Nursing Process Step: Evaluation MSC: NCLEX: N/A

33. Which responsibilities are common to an LPN/LVN team leader in both health care facilities
and medical clinics? (Select all that apply.)
a. Organizes staff meetings
b. Assists in writing policies and procedures
c. Assists in resolving staff conflicts
d. Prepares the schedule for patient activities
e. Evaluates unlicensed assistive personnel (UAPs)

ANS: B, C, E
Writing policies and procedures, assisting to resolve staff conflicts, and evaluating UAPs are
duties common to LPN/LVN team leaders in health care facilities. Only in medical clinics do
LPNs/LVNs generally attend staff meetings and oversee scheduling of patients.

Downloaded by: SUCCEEDGRADES | abbieclin@gmail.com


Distribution
Downloaded by Dallen Mae D.ofKadir
this document is illegal
(kadir.dallenmae.d.bcsi@gmail.com)
lOMoARcPSD|35346190

Stuvia.com - The Marketplace to Buy and Sell your Study Material

DIF: Cognitive Level: Comprehension REF: p. 127 OBJ: Theory #7


TOP: Team Leader Responsibilities KEY: Nursing Process Step: N/A
MSC: NCLEX: N/A

OTHER

34. Place the steps of transcription of physician‘s orders in proper sequence. (Separate letters with
a comma and space as follows: A, B, C, D, E.)
A. Transmit orders to pharmacy, dietary, or ancillary services.
B. Add medication changes to MAR.
C. Process the ―stat‖ orders first.
D. Read through all of the order.
E. Sign off order with a red line across page.

ANS:
D, C, B, A, E
The nurse should read through all of the orders before transmission; process the ―stat‖ orders
first; add medications to the MAR; transmit orders to pharmacy, dietary, or ancillary services;
and then sign off the order with name and title.

DIF: Cognitive Level: Analysis REF: pp. 129-130 OBJ: Clinical Practice #5
TOP: Transcribing Orders KEY: Nursing Process Step: N/A
MSC: NCLEX: N/A

Downloaded by: SUCCEEDGRADES | abbieclin@gmail.com


Distribution
Downloaded by Dallen Mae D.ofKadir
this document is illegal
(kadir.dallenmae.d.bcsi@gmail.com)
lOMoARcPSD|35346190

DEWITS FUNDAMENTALStC
uvO
iaN
.cC
om
EP- T
ThSe A
MNarD
ketSpK
l a IcL
eLtoSBFuO
y aRndNSUeR
ll S
yoI u
Nr G
Stu
5dTyHME
atD
erIiaTl ION WILLIAMS TEST BANK

Chapter 11: Growth and Development: Infancy Through Adolescence


Williams: deWit's Fundamental Concepts and Skills for Nursing, 8th Edition

MULTIPLE CHOICE

1. A mother of an 11-month-old is concerned about her baby not walking and states that his
older brother was walking at 10 months. The nurse‘s response, based on knowledge of normal
development, should be:
a. ―Your son may be somewhat slow developmentally. You might have him
evaluated by a neurologist.‖
b. ―The rate of development can be very different from one child to another, even
among brothers and sisters.‖
c. ―Don‘t worry. Children can be perfectly normal and not walk until they are 2 years
of age.‖
d. ―It‘s hard to predict when a child will walk. Some walk before they are able to
crawl or sit alone.‖
ANS: B
Development follows a pattern (sit before creep, creep before walk) but varies in normal
children. Eleven months is not developmentally slow, but children normally do walk before 2
years of age.

DIF: Cognitive Level: Application REF: p. 148|Box 11-1


OBJ: Theory #3 TOP: Motor Development, Infants
KEY: Nursing Process Step: Implementation
MSC: NCLEX: Health Promotion anNdUM
RSaIint
NGenTaBn.cCeO: M
Gro wth and Development

2. The nurse is aware that according to Piaget‘s theory, the child‘s cognitive development is:
a. present at birth as a genetic imprint that will begin to unfold in the first year,
independent of the child‘s environment.
b. a series of psychosocial tasks to be accomplished at various stages throughout life,
such as developing a sense of identity in adolescence.
c. the physical increase in size that occurs in the brain of the child, with most of the
growth completed by the age of adolescence.
d. occurring in stages, which allows a child to organize knowledge and adapt to the
environment in increasingly complex ways.
ANS: D
Cognitive refers to intellectual aspects. It is strongly influenced by environment and is more
than just physical development of the brain. Piaget theorized stages a child goes through in
organizing knowledge and adapting to the environment.

DIF: Cognitive Level: Comprehension REF: p. 148 OBJ: Theory #7


TOP: Cognitive Development KEY: Nursing Process Step: Planning
MSC: NCLEX: Health Promotion and Maintenance: Growth and Development

3. A new parent with her first child tells the nurse that her mother has advised her to feed the
baby on a schedule and let the baby cry between feedings so that he does not get spoiled.
Based on Erikson‘s growth and development theory, the nurse‘s most informative response
would be:

NURSINGTB.COM

Downloaded by: SUCCEEDGRADES | abbieclin@gmail.com


Distribution
Downloaded by Dallen Mae D.ofKadir
this document is illegal
(kadir.dallenmae.d.bcsi@gmail.com)
lOMoARcPSD|35346190

DEWITS FUNDAMENTALStC
uvO
iaN
.cC
om
EP- T
ThSe A
MNarD
ketSpK
l a IcL
eLtoSBFuO
y aRndNSUeR
ll S
yoI u
Nr G
Stu
5dTyHME
atD
erIiaTl ION WILLIAMS TEST BANK

a. a sense of trust is developed in infancy when a child‘s needs are met by warm,
loving people.
b. an infant who learns to adapt to uncomfortable situations and to comfort himself
will develop autonomy and independence.
c. the infant learns cause and effect at this stage and will learn he can‘t cry and get
his own way.
d. in the first year, crying and learning to wait have positive outcomes for both the
mother and the baby.
ANS: A
Meeting the crying baby‘s needs fosters trust; it does not ―spoil‖ a child. Crying inconsolably
is not a positive outcome for a baby when he fails to have his needs met.

DIF: Cognitive Level: Application REF: p. 148 OBJ: Theory #6


TOP: Psychosocial Development, Infants
KEY: Nursing Process Step: Implementation
MSC: NCLEX: Health Promotion and Maintenance: Growth and Development

4. A 4-year-old attends a preschool group three mornings a week. He has been treated kindly and
lovingly at home. At school, according to Kohlberg‘s theory of moral development, he is
likely to:
a. treat his peers and siblings in a loving and kind manner, because he has
internalized the value of treating others as he has been treated.
b. conform to social standards and rules to avoid feeling guilty or risking social
disapproval.
c. follow the rules regarding peer interactions to avoid punishment.
d. be developmentally advanced a nd h a v e a c o ns cience that is based on an innate
N U RS I N G T B . C O M
sense of justice and duty.
ANS: C
According to Kohlberg, the child‘s behavior at preschool will be preconventional reasoning,
governed by fear of punishment. This stage lasts until about the age of 9 years.

DIF: Cognitive Level: Comprehension REF: p. 144 OBJ: Theory #5


TOP: Developmental Theories KEY: Nursing Process Step: Assessment
MSC: NCLEX: Health Promotion and Maintenance: Growth and Development

5. During the first trimester of pregnancy, the most important teaching a nurse can provide the
expectant mother is:
a. avoiding very strenuous exercises or activities.
b. eating large high caloric meals to provide for the growing fetus.
c. preparing self-physically and mentally for the work of labor.
d. acquiring adequate prenatal care.

ANS: D
Early prenatal care is the most important factor in a healthy pregnancy outcome.

DIF: Cognitive Level: Application REF: p. 146 OBJ: Clinical Practice #1


TOP: Prenatal Development KEY: Nursing Process Step: Implementation
MSC: NCLEX: Health Promotion and Maintenance: Prevention and Early Detection of Disease

NURSINGTB.COM

Downloaded by: SUCCEEDGRADES | abbieclin@gmail.com


Distribution
Downloaded by Dallen Mae D.ofKadir
this document is illegal
(kadir.dallenmae.d.bcsi@gmail.com)
lOMoARcPSD|35346190

DEWITS FUNDAMENTALStC
uvO
iaN
.cC
om
EP- T
ThSe A
MNarD
ketSpK
l a IcL
eLtoSBFuO
y aRndNSUeR
ll S
yoI u
Nr G
Stu
5dTyHME
atD
erIiaTl ION WILLIAMS TEST BANK

6. A nurse encourages the father of a newborn infant to hold the baby close, talk softly, stroke
the baby‘s fingers, and make eye contact in order to:
a. assist the father‘s confidence in holding the baby.
b. promote bonding, which in turn promotes a sense of trust in infancy.
c. generate a sense of parenting.
d. accustom the baby to being handled by different caregivers.
ANS: B
Bonding is an important feature of the parent-child relationship that must be supported and
facilitated in the first weeks of life (a teachable moment).

DIF: Cognitive Level: Comprehension REF: p. 148 OBJ: Theory #6


TOP: Psychosocial Development, Infant Bondings
KEY: Nursing Process Step: Implementation
MSC: NCLEX: Health Promotion and Maintenance: Growth and Development

7. The nurse confirms that in the embryonic stage of prenatal development (third to eighth
week), the embryo‘s:
a. growth slows.
b. organs begin to function.
c. limbs move.
d. heart begins to beat.
ANS: D
The embryo‘s heart begins to beat at 3.5 weeks.

DIF: Cognitive Level: Comprehension REF: p. 142 OBJ: Clinical Practice #4


TOP: Health Promotion, Infants N U R S I NKGETYB :. C N
OMur sin g Process Step: Implementation
MSC: NCLEX: Health Promotion and Maintenance: Growth and Development

8. A parent asks the nurse about how to discipline her 2-year-old who has become uncooperative
and negative and is having temper tantrums. The most helpful suggestion made by the nurse
would be for the parent to deal with this normal behavior by:
a. scolding the child sternly to discourage this behavior.
b. taking away a favorite toy until behavior improves.
c. using a ―time-out‖ (a quiet time without toys).
d. asking the child to ―help Mama‖ by not being naughty.
ANS: C
This is a normal 2-year-old behavior; the mother needs to learn the use of ―time-out‖ or
nonpunitive, less authoritarian ways of dealing with the 2-year-old that are not so likely to
provoke ―no.‖

DIF: Cognitive Level: Application REF: p. 151 OBJ: Clinical Practice #9


TOP: Discipline, Young Child KEY: Nursing Process Step: Implementation
MSC: NCLEX: Health Promotion and Maintenance: Growth and Development

9. A woman in her first trimester of pregnancy asks the nurse how much weight she can expect
to gain by the time of EDD (estimated delivery date). The nurse correctly responds
approximately by informing her:
a. 5 to 10 pounds.
b. 15 to 20 pounds.

NURSINGTB.COM

Downloaded by: SUCCEEDGRADES | abbieclin@gmail.com


Distribution
Downloaded by Dallen Mae D.ofKadir
this document is illegal
(kadir.dallenmae.d.bcsi@gmail.com)
lOMoARcPSD|35346190

DEWITS FUNDAMENTALStC
uvO
iaN
.cC
om
EP- T
ThSe A
MNarD
ketSpK
l a IcL
eLtoSBFuO
y aRndNSUeR
ll S
yoI u
Nr G
Stu
5dTyHME
atD
erIiaTl ION WILLIAMS TEST BANK

c. 25 to 30 pounds.
d. 35 to 40 pounds.
ANS: D
A woman beginning pregnancy at a healthy weight should expect to gain approximately 30
pounds.

DIF: Cognitive Level: Knowledge REF: p. 146 OBJ: Clinical Practice #1


TOP: Nutrition KEY: Nursing Process Step: N/A MSC: NCLEX: N/A

10. The nurse reminds a parent that a major milestone for a 2- to 3-year-old child is to learn to:
a. play team games with others his age.
b. speak clearly and fluently.
c. use the toilet for bladder and bowel functions.
d. tie shoelaces.

ANS: C
Toilet training is usually accomplished in the second or third year. Team games are a young
school age phenomenon, as is tying shoelaces. Speaking clearly and fluently does not occur
until after the toddler stage.

DIF: Cognitive Level: Comprehension REF: p. 150 OBJ: Theory #8


TOP: Motor Development, Young Child KEY: Nursing Process Step: Implementation
MSC: NCLEX: Health Promotion and Maintenance: Growth and Development

11. When the clinic nurse speaks to the 3-year-old child and says, ―You can get dressed now and
put your shirt and pants back on,‖ she is promoting his sense of:
a. autonomy. NURSINGTB.COM
b. industry.
c. initiative.
d. trust.

ANS: A
Autonomy is the sense of independence—―I can do it myself.‖ It is accomplished in the
toddler stage as evidenced by toilet training, self-feeding, and self-dressing. Industry is a
school age task, initiative is a 4- to 6-year-old task (preschool), and trust is a task of infancy.

DIF: Cognitive Level: Comprehension REF: p. 144|Tables 11-2 and 11-27


OBJ: Theory #6 TOP: Psychosocial Development, Young Child
KEY: Nursing Process Step: Implementation
MSC: NCLEX: Health Promotion and Maintenance: Growth and Development

12. A 4-year-old child tells other children that last night he flew to the moon and walked around
with his friend Sam. He says that he is the only one who can see Sam. The nurse should:
a. warn the child‘s mother that he may be developing emotional problems.
b. interrupt the conversation with a comment that children can‘t fly to the moon.
c. take the child aside and tell him gently that he should stop telling such stories or
people will not trust him.
d. recognize that imaginary friends and fantasy are normal at this age.
ANS: D
Fantasy and imaginary friends are normal in this age group.

NURSINGTB.COM

Downloaded by: SUCCEEDGRADES | abbieclin@gmail.com


Distribution
Downloaded by Dallen Mae D.ofKadir
this document is illegal
(kadir.dallenmae.d.bcsi@gmail.com)
lOMoARcPSD|35346190

DEWITS FUNDAMENTALStC
uvO
iaN
.cC
om
EP- T
ThSe A
MNarD
ketSpK
l a IcL
eLtoSBFuO
y aRndNSUeR
ll S
yoI u
Nr G
Stu
5dTyHME
atD
erIiaTl ION WILLIAMS TEST BANK

DIF: Cognitive Level: Analysis REF: p. 150 OBJ: Theory #7


TOP: Cognitive Development, Young Child
KEY: Nursing Process Step: Implementation
MSC: NCLEX: Health Promotion and Maintenance: Growth and Development

13. The risk for poisoning is most likely in a situation in which:


a. a caretaker with an infant with a high fever has been instructed by the clinic nurse
to give a liquid fever reducer (antipyretic).
b. a toddler is staying with his grandparents for a week while his mother is in the
hospital.
c. a 7-year-old has asthma and uses daily medications for it.
d. a 12-year-old is home alone after school until his parents return from work.
ANS: B
A toddler staying with grandparents for an extended emergency visit is at risk for poisoning
related to the grandparents‘ home not being ―toddler proof.‖

DIF: Cognitive Level: Analysis REF: p. 150 OBJ: Clinical Practice #7


TOP: Cognitive Development, Young Child
KEY: Nursing Process Step: Diagnosis
MSC: NCLEX: Safe, Effective Care Environment: Growth and Development

14. A 9-year-old has returned to the pediatric unit from the recovery room after having his
appendix removed. He has an intravenous solution running into a vein and a dressing covering
the operative site. His parents went home briefly. When he becomes upset and starts crying,
the nurse should:
a. help the child recall the eventsNoUfRt ShIeN pGrTeBv .iCo O
u sMda y, explaining that his mother will
be back later in the morning.
b. recognize his need for family by calling his mother on the telephone and telling her
to come back and stay with her son.
c. appeal to his gender identity by telling him he is a big boy, and big boys don‘t cry.
d. be aware that the child cannot reliably tell the nurse about pain at this age and call
the care provider for an order for a pain medication or sedative.
ANS: A
A 9-year-old can use cognitive abilities to understand the nurse‘s explanation. He can also tell
about pain.

DIF: Cognitive Level: Application REF: p. 153|Table 11-3


OBJ: Clinical Practice #7 TOP: Cognition, Middle and Older Child
KEY: Nursing Process Step: Implementation
MSC: NCLEX: Health Promotion and Maintenance: Growth and Development

15. In planning anticipatory guidance for the parents of an 18-month-old child at a well child
clinic, the nurse should include information about the:
a. child‘s need for and ability to participate in quiet activities such as reading stories
and watching television and movies on a phone or computer for long periods of
time.
b. advantages of having the child begin to attend day care to develop his social
interactions and group play skills with other children.
c. need for close supervision and making the environment safe related to poisons,

NURSINGTB.COM

Downloaded by: SUCCEEDGRADES | abbieclin@gmail.com


Distribution
Downloaded by Dallen Mae D.ofKadir
this document is illegal
(kadir.dallenmae.d.bcsi@gmail.com)
lOMoARcPSD|35346190

DEWITS FUNDAMENTALStC
uvO
iaN
.cC
om
EP- T
ThSe A
MNarD
ketSpK
l a IcL
eLtoSBFuO
y aRndNSUeR
ll S
yoI u
Nr G
Stu
5dTyHME
atD
erIiaTl ION WILLIAMS TEST BANK

motor vehicles, stairs, and electrical hazards.


d. need for increased nutritional intake to maintain the continued rapid growth of
infancy into this stage.
ANS: C
Parents with a child at 18 months of age need to be guided in how to protect their child from
his own curiosity and dangers. An 18-month-old is not capable of sitting still for stories for
long periods, and at this age, children engage in parallel play rather than group play. His
nutrition needs to be balanced but will probably decline in amount because growth is not as
rapid as in infancy. The American Academy of Pediatrics guidelines recommend no
television, smart phones, or computers for children under 2 years, and less than 2 hours of
screen time per day for children and teens. These activities have been linked to an increase in
attention problems, development of unhealthy eating habits, higher body mass index, and
obesity.

DIF: Cognitive Level: Application REF: p. 150 OBJ: Clinical Practice #3


TOP: Cognitive Development, Young Child
KEY: Nursing Process Step: Planning
MSC: NCLEX: Safe, Effective Care Environment: Growth and Development

16. A parent asks how long she should enforce a ―time-out‖ for her 4-year-old, who frequently
hits her younger brother and takes his toys. The nurse recommends:
a. 2 minutes.
b. 3 minutes.
c. 4 minutes.
d. 5 minutes.

ANS: C NURSINGTB.COM
Time-outs are effective in this stage as a consequence for undesired behavior. A general
guideline is that the length of time of the time-out should match the age (number of minutes =
age).

DIF: Cognitive Level: Application REF: p. 151 OBJ: Theory #9


TOP: Discipline, Young Child KEY: Nursing Process Step: Implementation
MSC: NCLEX: Health Promotion and Maintenance: Growth and Development

17. A dejected 8-year-old has brought home an art project that was finished at school that didn‘t
get put up on the bulletin board with some better projects. The best way to support the child‘s
sense of industry would be for the parents to:
a. remind the child that not all kids are artistic.
b. point out the areas in the project that could have been improved.
c. request that the teacher allows the project to be posted on the bulletin board.
d. post the project on the front of the refrigerator.

ANS: D
Posting the project in a prominent place and remarking on the positive aspects of it like color
selection or composition will boost the child‘s sense of industry.

DIF: Cognitive Level: Application REF: p. 153 OBJ: Clinical Practice #6


TOP: Psychosocial Development, Middle and Older Child
KEY: Nursing Process Step: Implementation
MSC: NCLEX: Health Promotion and Maintenance: Growth and Development

NURSINGTB.COM

Downloaded by: SUCCEEDGRADES | abbieclin@gmail.com


Distribution
Downloaded by Dallen Mae D.ofKadir
this document is illegal
(kadir.dallenmae.d.bcsi@gmail.com)
lOMoARcPSD|35346190

DEWITS FUNDAMENTALStC
uvO
iaN
.cC
om
EP- T
ThSe A
MNarD
ketSpK
l a IcL
eLtoSBFuO
y aRndNSUeR
ll S
yoI u
Nr G
Stu
5dTyHME
atD
erIiaTl ION WILLIAMS TEST BANK

18. The school nurse is teaching parents of fifth graders (10 year olds) about the physical changes
of puberty that they can expect in their children, such as:
a. boys begin to change physically before girls.
b. there will be children in different stages of puberty throughout the next several
years.
c. the physical changes of puberty occur gradually, and children adjust without
difficulty.
d. most boys and girls will begin the physical changes of puberty by seventh grade
(12 years old). If this has not happened, the child should be evaluated by a care
provider.
ANS: B
In general, girls begin to develop 2 to 3 years ahead of boys. A child may have great difficulty
with body changes, especially if he is very much ahead of or behind his peers. Many boys do
not begin to develop until the high school years and continue their growth into their 20s.

DIF: Cognitive Level: Application REF: p. 154 OBJ: Theory #11


TOP: Physical Development, Middle and Older Child
KEY: Nursing Process Step: Implementation
MSC: NCLEX: Health Promotion and Maintenance: Growth and Development

19. A 10-year-old girl has recently grown 3 inches and gained 20 pounds. She began having
menstrual periods and is embarrassed about her breast development. The parents are
concerned about her complaints of fatigue. The most helpful response from the nurse would
be to suggest:
a. having her evaluated for a physical problem.
N U R S IN GT B . COM
b. minimizing her feelings of emb ar ra s sm en t.
c. enrolling in a health club for exercise.
d. helping the child recognize bodily self-consciousness and fatigue as a normal part
of maturation.
ANS: D
Fatigue is a common teen complaint associated with rapid growth and change.
Self-consciousness about body changes is also common.

DIF: Cognitive Level: Application REF: p. 154 OBJ: Clinical Practice #3


TOP: Physical Development, Adolescent KEY: Nursing Process Step: Implementation
MSC: NCLEX: Health Promotion and Maintenance: Growth and Development

20. The school nurse reminds the eighth grade boys‘ physical education classes to make time for
warm-up exercises and stretching before strenuous physical activity because:
a. warm-up exercises allow for a more proficient performance.
b. bones are growing faster than muscles and tendons and may result in injury.
c. the body‘s circulating blood needs warm-up time to perfuse larger frames.
d. warm-ups give an opportunity for perspiration to cool overheated muscles.

ANS: B
Bones are growing faster than muscles or tendons. Heavy physical activity without warm up
exercises could cause injury.

DIF: Cognitive Level: Comprehension REF: p. 154|Safety Alert

NURSINGTB.COM

Downloaded by: SUCCEEDGRADES | abbieclin@gmail.com


Distribution
Downloaded by Dallen Mae D.ofKadir
this document is illegal
(kadir.dallenmae.d.bcsi@gmail.com)
lOMoARcPSD|35346190

DEWITS FUNDAMENTALStC
uvO
iaN
.cC
om
EP- T
ThSe A
MNarD
ketSpK
l a IcL
eLtoSBFuO
y aRndNSUeR
ll S
yoI u
Nr G
Stu
5dTyHME
atD
erIiaTl ION WILLIAMS TEST BANK

OBJ: Theory #3 TOP: Psychosocial Development, Adolescent


KEY: Nursing Process Step: Implementation
MSC: NCLEX: Health Promotion and Maintenance: Growth and Development

21. The school nurse recognizes that the adolescent most at risk for a problem relative to body
image would be the 15-year-old:
a. girl with severe acne.
b. boy with a broken leg from a motorcycle accident.
c. boy with thinning hair.
d. girl who is 2 inches taller than classmates.
ANS: A
Severe acne will have the most impact on body image because it alters the appearance of the
face. Adolescents are very concerned about their appearance.

DIF: Cognitive Level: Application REF: p. 157 OBJ: Theory #11


TOP: Adolescent Development, Concerns
KEY: Nursing Process Step: Assessment
MSC: NCLEX: Health Promotion and Maintenance: Growth and Development

22. The home health nurse reminds parents that dental care for a child should begin:
a. when the child has all his permanent teeth.
b. when the child has all his deciduous teeth.
c. at approximately the age of 4 or 5 years, when the child can cooperate with a
dentist.
d. with tooth brushing and flossing of the first teeth by a parent or caregiver.
ANS: D NURSINGTB.COM
Dental care properly starts when the child first cuts teeth and the caregiver cleans them.
Toddlers quickly learn to imitate adult behaviors and can brush their own teeth with
supervision before all teeth have erupted.

DIF: Cognitive Level: Comprehension REF: p. 152 OBJ: Theory #8


TOP: Physical Development, Young Child
KEY: Nursing Process Step: Implementation
MSC: NCLEX: Health Promotion and Maintenance: Prevention and Early Detection of Disease

23. The well baby clinic nurse suggests that the most important aspect parents should consider
when selecting a day care facility for their preschooler would be the:
a. cost and convenience of the facility.
b. educational qualifications of the teachers and the aides.
c. cleanliness and provision of healthy snacks.
d. facility‘s approach to discipline and a nurturing atmosphere.
ANS: D
Although all aspects listed may be important, the atmosphere of nurturing and the level of
discipline should be the major consideration in selecting a day care facility.

DIF: Cognitive Level: Application REF: p. 152 OBJ: Theory #10


TOP: Day Care KEY: Nursing Process Step: Implementation
MSC: NCLEX: Health Promotion and Maintenance: Growth and Development

NURSINGTB.COM

Downloaded by: SUCCEEDGRADES | abbieclin@gmail.com


Distribution
Downloaded by Dallen Mae D.ofKadir
this document is illegal
(kadir.dallenmae.d.bcsi@gmail.com)
lOMoARcPSD|35346190

DEWITS FUNDAMENTALStC
uvO
iaN
.cC
om
EP- T
ThSe A
MNarD
ketSpK
l a IcL
eLtoSBFuO
y aRndNSUeR
ll S
yoI u
Nr G
Stu
5dTyHME
atD
erIiaTl ION WILLIAMS TEST BANK

24. The pediatric nurse identifies egocentrism in:


a. a 2-year-old who hits playmates and takes all the toys.
b. a 5-year-old who prefers to read or be read to instead of playing kickball.
c. a 12-year-old who is failing school and in frequent fights with his peers.
d. a 15-year-old who is a volunteer at the local hospital.
ANS: A
Egocentric behavior is generally seen in the toddler and teenage groups. It is characterized by
an intense focus on self, with little awareness of others or their needs.

DIF: Cognitive Level: Comprehension REF: p. 155 OBJ: Theory #7


TOP: Egocentrism KEY: Nursing Process Step: Assessment
MSC: NCLEX: Health Promotion and Maintenance: Growth and Development

25. The nurse explains that the Denver Developmental Screening Test (DDST) evaluated gross
motor skills by observing the child‘s ability to:
a. stack a pile of blocks.
b. hop on one foot.
c. draw a detailed figure of a person.
d. use different colors when drawing a house.
ANS: B
Gross motor skills evaluated by the DDST are walking, jumping, using a tricycle, throwing
and catching a ball, and hopping or standing on one foot.

DIF: Cognitive Level: Comprehension REF: p. 150 OBJ: Theory #8


TOP: Denver Developmental Screening Test
mUenRtSINGTB.COM
KEY: Nursing Process Step: Assess N
MSC: NCLEX: Health Promotion and Maintenance: Growth and Development

26. The anxious father to be asks the nurse when the sex of the fetus can be determined. The
nurse‘s answer is based on the knowledge that the external genitalia appear in the:
a. second month.
b. third month.
c. fourth month.
d. fifth month.
ANS: B
External genitalia can be visualized on ultrasound in the third month.

DIF: Cognitive Level: Knowledge REF: p. 142 OBJ: Theory #1


TOP: Prenatal Development KEY: Nursing Process Step: Implementation
MSC: NCLEX: Health Promotion and Maintenance: Growth and Development

27. A nurse assessing a 13-year-old girl observes that she has begun her menstruation cycle. The
nurse is aware the hormone responsible for this change is:
a. parathyroid hormone (PTH).
b. thyroid-stimulating hormone (TSH).
c. adrenocorticotropic hormone (ACTH).
d. follicle-stimulating hormone (FSH).

ANS: D

NURSINGTB.COM

Downloaded by: SUCCEEDGRADES | abbieclin@gmail.com


Distribution
Downloaded by Dallen Mae D.ofKadir
this document is illegal
(kadir.dallenmae.d.bcsi@gmail.com)
lOMoARcPSD|35346190

DEWITS FUNDAMENTALStC
uvO
iaN
.cC
om
EP- T
ThSe A
MNarD
ketSpK
l a IcL
eLtoSBFuO
y aRndNSUeR
ll S
yoI u
Nr G
Stu
5dTyHME
atD
erIiaTl ION WILLIAMS TEST BANK

The pituitary hormone follicle-stimulating hormone (FSH) stimulates the ovaries to begin
producing estrogen hormones that initiate menses.

DIF: Cognitive Level: Knowledge REF: p. 142 OBJ: Theory #11


TOP: Teaching KEY: Nursing Process Step: Assessment
MSC: NCLEX: Health Promotion and Maintenance: Growth and Development

28. A nurse instructing a group of adolescents about development includes in her remarks that the
presence of follicle-stimulating hormone (FSH) will cause boys to:
a. have a significant growth spurt.
b. grow a beard.
c. maintain an erection.
d. produce sperm.

ANS: D
Follicle-stimulating hormone (FSH) stimulates the testes to begin producing sperm.

DIF: Cognitive Level: Knowledge REF: p. 143 OBJ: Theory #11


TOP: Teaching KEY: Nursing Process Step: Implementation
MSC: NCLEX: Health Promotion and Maintenance: Growth and Development

29. A nurse assessing a 13-year-old female is aware that the presence of axillary and pubic hair
indicates the presence of:
a. progesterone.
b. insulin.
c. estradiol.
d. cortisol.
NURSINGTB.COM
ANS: C
Estradiol is an estrogen secreted by the ovaries that is responsible for the appearance of
secondary sex characteristics.

DIF: Cognitive Level: Comprehension REF: p. 142 OBJ: Theory #11


TOP: Teaching KEY: Nursing Process Step: Assessment
MSC: NCLEX: Health Promotion and Maintenance: Growth and Development

30. The nurse encourages the parent to allow the toddler to dress himself and feed himself and
make simple choices about clothing or food in order to follow the theory of:
a. Erik Erikson.
b. Jean Piaget.
c. Lawrence Kohlberg.
d. Sigmund Freud.
ANS: A
Erik Erikson‘s theory of psychosocial stages encourages the toddler toward autonomy by
allowing the child to begin to become independent.

DIF: Cognitive Level: Comprehension REF: p. 144 OBJ: Theory #6


TOP: Theorists KEY: Nursing Process Step: Implementation
MSC: NCLEX: Health Promotion and Maintenance: Growth and Development

NURSINGTB.COM

Downloaded by: SUCCEEDGRADES | abbieclin@gmail.com


Distribution
Downloaded by Dallen Mae D.ofKadir
this document is illegal
(kadir.dallenmae.d.bcsi@gmail.com)
lOMoARcPSD|35346190

DEWITS FUNDAMENTALStC
uvO
iaN
.cC
om
EP- T
ThSe A
MNarD
ketSpK
l a IcL
eLtoSBFuO
y aRndNSUeR
ll S
yoI u
Nr G
Stu
5dTyHME
atD
erIiaTl ION WILLIAMS TEST BANK

31. When asked, the nurse explains that the superego described in Sigmund Freud‘s theory
influences human behavior by playing the part of the:
a. reality tester.
b. seeker of pleasure.
c. moral dictator.
d. primitive urges.
ANS: C
Freud‘s theory describes three parts of the personality: identity, ego, and superego. The
superego represents the moral component of the personality.

DIF: Cognitive Level: Knowledge REF: p. 143 OBJ: Theory #4


TOP: Freud KEY: Nursing Process Step: Implementation
MSC: NCLEX: Health Promotion and Maintenance: Growth and Development

32. When a 3-year-old toddler repeatedly tries to reach some cookies on top of a counter and then,
after several failures to do so, drags a chair over to the counter and climbs on the chair to get
the cookies, she is exhibiting:
a. adaptation as described by Piaget.
b. egocentricity as described by Freud.
c. sensorimotor motivation as described by Piaget.
d. preconventional reasoning as described by Kohlberg.

ANS: A
Adaptation as described by Piaget outlines that a child can adjust thinking patterns as the child
discovers new information.

DIF: Cognitive Level: Analysis N U R S I NRGETFB: . C pO. M1 4 4 OBJ: Theory #7


TOP: Adaptation KEY: Nursing Process Step: Assessment
MSC: NCLEX: Health Promotion and Maintenance: Growth and Development

33. When a 4-year-old boy announces that he is going to marry his mother, the nurse recognizes
that this statement is an indicator that he is in the Freudian stage of:
a. oral.
b. anal.
c. phallic.
d. latency.
ANS: C
In the Freud‘s phallic or Oedipal stage, children identify with the parent of the same sex. The
little boy is imitating his father by declaring his desire to marry his mother.

DIF: Cognitive Level: Comprehension REF: p. 144|Table 11-1


OBJ: Theory #4 TOP: Theorists KEY: Nursing Process Step: Assessment
MSC: NCLEX: Health Promotion and Maintenance: Growth and Development

34. An 11-year-old who obeys all the rules set down by the teacher to avoid guilt or disapproval is
acting out Kohlberg‘s theory of:
a. conventional reasoning.
b. preconventional reasoning.
c. postconventional reasoning.
d. concrete operations.

NURSINGTB.COM

Downloaded by: SUCCEEDGRADES | abbieclin@gmail.com


Distribution
Downloaded by Dallen Mae D.ofKadir
this document is illegal
(kadir.dallenmae.d.bcsi@gmail.com)
lOMoARcPSD|35346190

DEWITS FUNDAMENTALStC
uvO
iaN
.cC
om
EP- T
ThSe A
MNarD
ketSpK
l a IcL
eLtoSBFuO
y aRndNSUeR
ll S
yoI u
Nr G
Stu
5dTyHME
atD
erIiaTl ION WILLIAMS TEST BANK

ANS: A
During the conventional reasoning level of moral development, children obey rules to avoid
guilt and disapproval. Moral values are not internalized. Most children are at this stage at
about age 9 or 10 years of age.

DIF: Cognitive Level: Comprehension REF: p. 144 OBJ: Theory #5


TOP: Theorists KEY: Nursing Process Step: Assessment
MSC: NCLEX: Health Promotion and Maintenance: Growth and Development

35. The nurse clarifies that growth differs from development in that growth:
a. is the same for all siblings in a family.
b. is solely dependent on heredity.
c. progresses in an orderly, predictable way.
d. continues throughout life.

ANS: C
Growth progresses in an orderly and predictable way. Its patterns and stages can be
anticipated. Growth potential is hereditary, but many things such as nutrition and illness can
affect growth.

DIF: Cognitive Level: Knowledge REF: p. 145 OBJ: Theory #3


TOP: Growth KEY: Nursing Process Step: Implementation
MSC: NCLEX: Health Promotion and Maintenance: Growth and Development

36. A distressed mother confides in the school nurse that she witnessed her 15-year-old son and a
15-year-old friend masturbating in her son‘s bedroom. The nurse‘s best response would be:
a. ―Perhaps you should ask your Ns U
onRSifI NhGe ThBa.sC hOoMmosexual feelings toward his
friend.‖
b. ―The fact that they were doing this secretly indicates that they feel guilty about the
experience.‖
c. ―Many kids have a homosexual encounter, but it does not mean they are
homosexual.‖
d. ―Mutual masturbation is frequently the initial experience of a person who is
actively homosexual.‖
ANS: C
Approximately 8% of adolescents experience a homosexual encounter. This does not mean
they are homosexual.

DIF: Cognitive Level: Application REF: p. 156 OBJ: Theory #13


TOP: Adolescents Sexual KEY: Nursing Process Step: Implementation
MSC: NCLEX: Health Promotion and Maintenance: Growth and Development

37. The school nurse suspects a 14-year-old of having anorexia nervosa because the nurse has
observed that the adolescent:
a. is binging on rich foods.
b. has early onset of sexual development.
c. uses frequent enemas and laxatives.
d. is doing excessive rigorous exercises.

ANS: D

NURSINGTB.COM

Downloaded by: SUCCEEDGRADES | abbieclin@gmail.com


Distribution
Downloaded by Dallen Mae D.ofKadir
this document is illegal
(kadir.dallenmae.d.bcsi@gmail.com)
lOMoARcPSD|35346190

DEWITS FUNDAMENTALStC
uvO
iaN
.cC
om
EP- T
ThSe A
MNarD
ketSpK
l a IcL
eLtoSBFuO
y aRndNSUeR
ll S
yoI u
Nr G
Stu
5dTyHME
atD
erIiaTl ION WILLIAMS TEST BANK

Anorexics control their weight be reducing nutritional intake and engaging in rigorous
exercise. The body becomes emaciated and sexual development is delayed.

DIF: Cognitive Level: Comprehension REF: p. 157 OBJ: Theory #13


TOP: Anorexia Nervosa KEY: Nursing Process Step: Assessment
MSC: NCLEX: Health Promotion and Maintenance: Prevention and Early Detection of Disease

38. A 15-year-old girl reports to the school nurse that she feels she does not fit in and feels near
tears all the time. She states her grades are declining. The nurse‘s best approach to this
situation would be:
a. ―Everyone has ‗downtimes.‘ Let‘s get you signed up with a tutor.‖
b. ―I think I better refer you to the school psychologist to help you.‖
c. ―Don‘t be silly…of course you fit in. You have many friends.‖
d. ―Come back again next week and we can talk some more about this problem.‖
ANS: B
The feelings of not fitting in and being close to tears all of the time in addition to declining
grades are indicators of depression. Any adolescent who exhibits signs of depression should
be evaluated by a mental health professional rather than being treated by the school nurse. The
other options belittle the girl‘s feelings.

DIF: Cognitive Level: Application REF: p. 157 OBJ: Theory #13


TOP: Theorists KEY: Nursing Process Step: N/A
MSC: NCLEX: Health Promotion and Maintenance: Growth and Development

39. The mother of a 13-year-old girl tells the school nurse that her daughter will appear one day
neatly dressed and groomed and the next will have on tattered jeans and a T-shirt and untidy
hair. The nurse realizes that these NarUeR―StIrNiaGl sT‖B .tCo Of M
ind:
a. initiative.
b. industry.
c. identity.
d. social approval.
ANS: C
The major developmental task of Erik Erikson‘s theory is that of discovery of one‘s identity.
The changing of clothing and behaviors indicate trying for a ―fit.‖

DIF: Cognitive Level: Comprehension REF: p. 144|Table 11-2


OBJ: Theory #4 TOP: Erikson KEY: Nursing Process Step: Assessment
MSC: NCLEX: Health Promotion and Maintenance: Growth and Development

40. A group of fifth grade boys play together at school and after school are seen in each other‘s
company. The school nurse is aware that this sort of group is representative of Freud‘s:
a. anal stage.
b. oral stage.
c. latency stage.
d. phallic stage.
ANS: C
The major developmental tasks of Sigmund Freud‘s latency stage of psychosexual
development include a focus on relationships with same sex peers.

NURSINGTB.COM

Downloaded by: SUCCEEDGRADES | abbieclin@gmail.com


Distribution
Downloaded by Dallen Mae D.ofKadir
this document is illegal
(kadir.dallenmae.d.bcsi@gmail.com)
lOMoARcPSD|35346190

DEWITS FUNDAMENTALStC
uvO
iaN
.cC
om
EP- T
ThSe A
MNarD
ketSpK
l a IcL
eLtoSBFuO
y aRndNSUeR
ll S
yoI u
Nr G
Stu
5dTyHME
atD
erIiaTl ION WILLIAMS TEST BANK

DIF: Cognitive Level: Comprehension REF: p. 144|Table 11-1


OBJ: Theory #4 TOP: Theorists KEY: Nursing Process Step: Assessment
MSC: NCLEX: Health Promotion and Maintenance: Growth and Development

41. The nurse recognizes that the parenting style in which a parent is firm, in control, and has a
warm encouraging relationship with the child is:
a. authoritarian.
b. authoritative.
c. autocratic.
d. absolute.
ANS: B
In the authoritative parenting style, a parent has a firm approach, is in control, and has a
warm, loving, and encouraging relationship with their children.

DIF: Cognitive Level: Comprehension REF: p. 154|Table 11-4


OBJ: Theory #4 TOP: Parenting KEY: Nursing Process Step: N/A
MSC: NCLEX: Health Promotion and Maintenance: Growth and Development

42. The nurse tells a parent of adolescents that teenage pregnancies, although on the decline, often
result in the baby having:
a. learning difficulties at a later age.
b. a low birth weight.
c. a congenital deformity.
d. retarded development.

ANS: B
Because the teenage mother is stilNl Ug rRoSwI Ni nGgT, Bt .hCeObMa b i e s from a teen pregnancy are frequently
of low birth weight. None of the other options are a direct result of teenage pregnancy.

DIF: Cognitive Level: Comprehension REF: p. 157 OBJ: Theory #13


TOP: Teen Pregnancy KEY: Nursing Process Step: Implementation
MSC: NCLEX: Health Promotion and Maintenance: Prevention and Early Detection of Disease

43. An 8-year-old‘s mother asked the child to set the table, but then remarked that it was not very
neat and then reset the table herself. According to Erikson, this sort of behavior on the part of
the mother will cause the child to feel:
a. mistrustful.
b. ashamed.
c. guilty
d. inferior.

ANS: D
Erik Erikson‘s psychosocial development theory includes the stage industry vs. inferiority,
which is characterized by a child not being recognized for his or her industry.

DIF: Cognitive Level: Comprehension REF: p. 144|Table 11-2


OBJ: Theory #6 TOP: Erikson KEY: Nursing Process Step: N/A
MSC: NCLEX: N/A

44. The school nurse is alert to the behavior of a child who has extremely permissive parents,
because this sort of parenting style produces children who are:

NURSINGTB.COM

Downloaded by: SUCCEEDGRADES | abbieclin@gmail.com


Distribution
Downloaded by Dallen Mae D.ofKadir
this document is illegal
(kadir.dallenmae.d.bcsi@gmail.com)
lOMoARcPSD|35346190

DEWITS FUNDAMENTALStC
uvO
iaN
.cC
om
EP- T
ThSe A
MNarD
ketSpK
l a IcL
eLtoSBFuO
y aRndNSUeR
ll S
yoI u
Nr G
Stu
5dTyHME
atD
erIiaTl ION WILLIAMS TEST BANK

a. mistrusting.
b. depressed.
c. anxious.
d. impulsive.
ANS: D
Permissive parenting produces children who are impulsive, aggressive, and lacking in
self-control.

DIF: Cognitive Level: Knowledge REF: p. 154|Table 11-4


OBJ: Theory #9 TOP: Permissive Parenting KEY: Nursing Process Step: N/A
MSC: NCLEX: Health Promotion and Maintenance: Growth and Development

45. Based on the theory of Erikson, the nurse advises that the best discipline for an adolescent
would be to:
a. make him get a job.
b. increase his household chores.
c. implement a long period of ―time-out.‖
d. ―ground‖ him and deprive him of contact with friends.
ANS: D
Erik Erikson‘s psychosocial development theory includes the stage of identity vs. identity
confusion. This age values association with peers above everything else, so depriving the
child from contacting his friends would have the most impact.

DIF: Cognitive Level: Comprehension REF: p. 144|Table 11-2


OBJ: Theory #6 TOP: Theorists KEY: Nursing Process Step: N/A
MSC: NCLEX: Health Promotion anNdUM
RSaIin
NtG
enTaBn.cCeO: M
Gro wth and Development

46. A school nurse suspects that a 7-year-old child at the school is being physically abused. The
nurse should:
a. interview the child about possible abuse.
b. notify the principal.
c. call the parent about her concern.
d. report the suspected abuse to the authorities.

ANS: D
Most states require health workers to report child abuse to the authorities.

DIF: Cognitive Level: Comprehension REF: p. 154 OBJ: Theory #9


TOP: Child Abuse KEY: Nursing Process Step: N/A
MSC: NCLEX: Health Promotion and Maintenance: Growth and Development

47. The nurse reminds parents that social competence can be recognized in their child if the child:
a. learns to cope with minor stressors.
b. develops a sense of humor.
c. finishes jobs and chores they start.
d. develops a talent.
ANS: A
Persons with social competence can handle minor stressors and are aware of how their
behavior influences others and how they may appear to others.

NURSINGTB.COM

Downloaded by: SUCCEEDGRADES | abbieclin@gmail.com


Distribution
Downloaded by Dallen Mae D.ofKadir
this document is illegal
(kadir.dallenmae.d.bcsi@gmail.com)
lOMoARcPSD|35346190

DEWITS FUNDAMENTALStC
uvO
iaN
.cC
om
EP- T
ThSe A
MNarD
ketSpK
l a IcL
eLtoSBFuO
y aRndNSUeR
ll S
yoI u
Nr G
Stu
5dTyHME
atD
erIiaTl ION WILLIAMS TEST BANK

DIF: Cognitive Level: Comprehension REF: p. 153 OBJ: Theory #8


TOP: Social Competence KEY: Nursing Process Step: Implementation
MSC: NCLEX: Health Promotion and Maintenance: Growth and Development

48. The nurse instructs the parents that emotional intelligence can be supported in a child by the
parents:
a. insisting on IQ testing.
b. talking to the child about his or her feelings.
c. allowing the child to bring friends into their home.
d. encouraging the child to read.
ANS: B
Parents can assist their children in developing emotional intelligence by talking to their
children about their feelings, taking their children‘s feelings seriously, and helping them find
ways to cope with their feelings.

DIF: Cognitive Level: Comprehension REF: p. 153 OBJ: Theory #8


TOP: Emotional Intelligence KEY: Nursing Process Step: Implementation
MSC: NCLEX: Health Promotion and Maintenance: Growth and Development

49. A child who can draw pictures of objects not in evidence in the room is exhibiting Piaget‘s:
a. preoperational stage.
b. sensorimotor stage.
c. concrete operations.
d. formal operations.
ANS: A NURSINGTB.COM
Jean Piaget‘s cognitive development theory includes the preoperational stage, which occurs
from 2 to 7 years of age. During this stage, a child can draw and think about objects that are
not in evidence in the immediate environment.

DIF: Cognitive Level: Comprehension REF: p. 144|Table 11-3


OBJ: Theory #7 TOP: Theorists KEY: Nursing Process Step: Assessment
MSC: NCLEX: Health Promotion and Maintenance: Growth and Development

50. A person who demonstrates the ability to logically manipulate abstract and unobservable
concepts is in which stage of Jean Piaget‘s cognitive development theory?
a. Preoperational stage
b. Sensorimotor stage
c. Concrete operations
d. Formal operations
ANS: D
Jean Piaget‘s cognitive development theory includes the formal operations stage, which
occurs from 11 years of age to death. During this stage, a person demonstrates the ability to
logically manipulate abstract and unobservable concepts.

DIF: Cognitive Level: Knowledge REF: p. 144|Table 11-3


OBJ: Theory #6 TOP: Theorists KEY: Nursing Process Step: N/A
MSC: NCLEX: Health Promotion and Maintenance: Growth and Development

NURSINGTB.COM

Downloaded by: SUCCEEDGRADES | abbieclin@gmail.com


Distribution
Downloaded by Dallen Mae D.ofKadir
this document is illegal
(kadir.dallenmae.d.bcsi@gmail.com)
lOMoARcPSD|35346190

DEWITS FUNDAMENTALStC
uvO
iaN
.cC
om
EP- T
ThSe A
MNarD
ketSpK
l a IcL
eLtoSBFuO
y aRndNSUeR
ll S
yoI u
Nr G
Stu
5dTyHME
atD
erIiaTl ION WILLIAMS TEST BANK

51. An anxious father to be tells the nurse that his wife‘s due date was 5 days ago and there are
still no signs of labor. The nurse‘s most helpful response would be:
a. ―I think you should contact your wife‘s doctor about this concern.‖
b. ―Just be patient…babies come when they are ready.‖
c. ―Any time 2 weeks on either side of the due date is considered normal.‖
d. ―Sometimes estimated due dates have been off by as much as 6 weeks.‖
ANS: C
Births occurring 2 weeks on either side of the estimated due date are considered normal.

DIF: Cognitive Level: Comprehension REF: p. 145 OBJ: Clinical Practice #1


TOP: Prenatal Care KEY: Nursing Process Step: Implementation
MSC: NCLEX: Health Promotion and Maintenance: Growth and Development

52. A nurse teaches a woman who is considering pregnancy that her intake of folic acid to help
prevent congenital abnormalities should be:
a. 100 mcg/day.
b. 200 mcg/day.
c. 300 mcg/day.
d. 400 mcg/day.
ANS: D
If planning to become pregnant, a woman should be certain her intake of folic acid is at least
400 mcg/day to assist in preventing neural tubal defects.

DIF: Cognitive Level: Comprehension REF: p. 146 OBJ: Clinical Practice #1


TOP: Prenatal Care KEY: Nursing Process Step: Implementation
MSC: NCLEX: Health Promotion anNdUM
RSaIiN
ntG
enTaBn.cCeO: M
Gr o wt h and Development

53. A woman who has just given birth expresses concern about the cheesy, waxy substance that is
covering her neonate. The nurse accurately responds:
a. ―This material develops in the absence of adequate subcutaneous fat.‖
b. ―This material is often seen on infants who have experienced intrauterine stress.‖
c. ―This substance will be absorbed and used as a nutrient by your infant.‖
d. ―This is a normal finding and is called vernix caseosa.‖
ANS: D
At birth, the skin and scalp are often covered with vernix caseosa (a cheesy, waxy substance
that protects the skin in fetal life). This wears off in a day or two.

DIF: Cognitive Level: Knowledge REF: p. 147 OBJ: Theory #1


TOP: Appearance of Newborn KEY: Nursing Process Step: Implementation
MSC: NCLEX: Health Promotion and Maintenance: Growth and Development

54. When the nurse makes a loud noise near the infant, the infant reacts by arching his back and
assuming a typical posture with flexion and adduction of the extremities, and his fingers fan
initially. This is called the:
a. stepping reflex.
b. grasping reflex.
c. Babinski reflex.
d. Moro reflex.

NURSINGTB.COM

Downloaded by: SUCCEEDGRADES | abbieclin@gmail.com


Distribution
Downloaded by Dallen Mae D.ofKadir
this document is illegal
(kadir.dallenmae.d.bcsi@gmail.com)
lOMoARcPSD|35346190

DEWITS FUNDAMENTALStC
uvO
iaN
.cC
om
EP- T
ThSe A
MNarD
ketSpK
l a IcL
eLtoSBFuO
y aRndNSUeR
ll S
yoI u
Nr G
Stu
5dTyHME
atD
erIiaTl ION WILLIAMS TEST BANK

ANS: D
The Moro reflex (startle response) can be elicited by making a loud noise near the baby, who
will react by arching his back and assuming a typical posture with flexion and adduction of
the extremities and fingers fanned initially.

DIF: Cognitive Level: Knowledge REF: p. 147 OBJ: Theory #3


TOP: Moro Reflex KEY: Nursing Process Step: Assessment
MSC: NCLEX: Health Promotion and Maintenance: Growth and Development

55. The nurse instructs the caregiver of a newborn that newborns should double their birth weight
by:
a. 2 to 3 months.
b. 3 to 4 months.
c. 4 to 5 months.
d. 5 to 6 months.
ANS: D
All babies should double their birth weight by 5 to 6 months, and triple it by 1 year.

DIF: Cognitive Level: Knowledge REF: p. 147 OBJ: Theory #3


TOP: Principles of Growth and Development
KEY: Nursing Process Step: Implementation
MSC: NCLEX: Health Promotion and Maintenance: Growth and Development

56. The nurse instructs the mother of an infant about infant safety factors. The nurse would assess
the need for further instruction if the caregiver states:
a. ―I will place my infant on her back for sleep.‖
b. ―I will place my infant in a c a rN sUeRa St Iw
N hGeTnB d. Cr O
i vMing.‖
c. ―I will not leave my infant alone in a house or a car.‖
d. ―I will place my infant on her stomach for sleep.‖

ANS: D
The infant should be restrained in a car seat in the back seat of the automobile whenever the
vehicle is in motion. The infant should be placed on his back for sleep, with propping slightly
on one side or the other for variation. No infant should be placed on the stomach to sleep. An
infant should never be left alone in a house or car.

DIF: Cognitive Level: Comprehension REF: p. 148 OBJ: Clinical Practice #8


TOP: Infant Safety Factors KEY: Nursing Process Step: Implementation
MSC: NCLEX: Safe, Effective Care Environment: Safety and Infection Control

57. The nurse points out that an infant who pulls at a drawer handle to open a drawer that holds a
toy that is out of view is exhibiting:
a. industry.
b. initiative.
c. concrete operations.
d. object permanence.

ANS: D
By the time babies are about 8 months old, they realize that an object still exists even when
they cannot see it. This is called object permanence. It is demonstrated when an infant is
shown an object that is then hidden, and the baby looks for it.

NURSINGTB.COM

Downloaded by: SUCCEEDGRADES | abbieclin@gmail.com


Distribution
Downloaded by Dallen Mae D.ofKadir
this document is illegal
(kadir.dallenmae.d.bcsi@gmail.com)
lOMoARcPSD|35346190

DEWITS FUNDAMENTALStC
uvO
iaN
.cC
om
EP- T
ThSe A
MNarD
ketSpK
l a IcL
eLtoSBFuO
y aRndNSUeR
ll S
yoI u
Nr G
Stu
5dTyHME
atD
erIiaTl ION WILLIAMS TEST BANK

DIF: Cognitive Level: Comprehension REF: p. 148 OBJ: Theory #7


TOP: Principles of Growth and Development
KEY: Nursing Process Step: Implementation
MSC: NCLEX: Health Promotion and Maintenance: Growth and Development

58. The nurse would anticipate that the child who prefers onlooker play would be:
a. 1 year old.
b. 3 years old.
c. 4 years old.
d. 5 years old.
ANS: A
Onlooker play is a type of play in which a child watches others playing but does not interact.
Children younger than the age of 2 engage normally in this type of play.

DIF: Cognitive Level: Knowledge REF: p. 152|Box 11-2


OBJ: Theory #8 TOP: Principles of Growth and Development
KEY: Nursing Process Step: N/A
MSC: NCLEX: Health Promotion and Maintenance: Growth and Development

59. A nurse would expect a child of 2 years to communicate in:


a. squeals and pointing.
b. two word sentences.
c. three word sentences.
d. four word sentences.
ANS: B NURSINGTB.COM
Children should be using two word sentences by the age of 2 years. Precocious children may
be able to communicate with sentences of greater length.

DIF: Cognitive Level: Comprehension REF: p. 148 OBJ: Theory #8


TOP: Communication of Toddlers KEY: Nursing Process Step: Planning
MSC: NCLEX: Health Promotion and Maintenance: Growth and Development

60. The nurse reminds parents that infants who have developed trust by the age of 6 months will
be able to:
a. tolerate short separations from parents.
b. learn to speak at an earlier age.
c. attempt walking around 8 months of age.
d. seldom cry.

ANS: A
Trustful infants can tolerate parental separations for short periods of time and will have
confidence to explore new situations.

DIF: Cognitive Level: Comprehension REF: p. 149 OBJ: Theory #8


TOP: Principles of Growth and Development
KEY: Nursing Process Step: Implementation
MSC: NCLEX: Health Promotion and Maintenance: Growth and Development

NURSINGTB.COM

Downloaded by: SUCCEEDGRADES | abbieclin@gmail.com


Distribution
Downloaded by Dallen Mae D.ofKadir
this document is illegal
(kadir.dallenmae.d.bcsi@gmail.com)
lOMoARcPSD|35346190

DEWITS FUNDAMENTALStC
uvO
iaN
.cC
om
EP- T
ThSe A
MNarD
ketSpK
l a IcL
eLtoSBFuO
y aRndNSUeR
ll S
yoI u
Nr G
Stu
5dTyHME
atD
erIiaTl ION WILLIAMS TEST BANK

61. The well child clinic nurse reminds the parents of a 2-year-old that the primary environmental
influence for physical development is:
a. play.
b. nurturing parents.
c. nutrition.
d. frequent checkups.
ANS: C
Nutrition is the primary environmental influence on physical development.

DIF: Cognitive Level: Comprehension REF: p. 149 OBJ: Theory #8


TOP: Principles of Growth and Development
KEY: Nursing Process Step: Implementation
MSC: NCLEX: Health Promotion and Maintenance: Growth and Development

62. A nurse arranges for cooperative play in a pediatric unit by:


a. coloring a picture in a coloring book with a 6-year-old patient.
b. arranging for two 3 year olds to play with their dolls at the same table.
c. showing an 11-year-old how to do a crossword puzzle.
d. organizing a checkers game between two 10 year olds.
ANS: D
Cooperative play is a type of play in which children play together. Games have rules and
goals that are accepted and followed.

DIF: Cognitive Level: Application REF: p. 152|Box 11-2


OBJ: Theory #8 TOP: Principles of Growth and Development
KEY: Nursing Process Step: N/A NURSINGTB.COM
MSC: NCLEX: Health Promotion and Maintenance: Growth and Development

63. The nurse is assessing 6-month-old infant. The infant was born at 38 weeks gestation,
weighed 4 pounds at birth, and was 18 inches long. Which of the following statements by the
mother indicates that further parental education is needed?
a. My baby should weigh at least, 12 pounds today.
b. My baby is considered a preterm baby.
c. My baby is considered a small for date baby.
d. My baby should be at least 28 inches long.

ANS: C
The average newborn weighs 7 to 7 1/2 lb and is 20 to 21 inches long. Less than 5.5 pounds is
considered low birth weight and more than 8.8 pounds is considered high birth weight. There
are two categories of low-birth-weight babies. The first is preterm babies, who are born before
37 weeks of gestation and weigh less than 5 pounds. The second category is small-for-dates
babies. These infants are born at term but weigh less than 90% of what they should weigh. All
babies should double their birth weight at 5 to 6 months and triple it by 1 year. They should
also grow 1/2 to 1 inch per month the first 6 months and 3/8 inch per month from 6 to 12
months. At well baby visits the baby‘s growth is measured and documented on standard
growth charts.

DIF: Cognitive Level: Analysis REF: p. 148 OBJ: Theory #3


TOP: Physical Development KEY: Nursing Process Step: N/A
MSC: NCLEX: N/A

NURSINGTB.COM

Downloaded by: SUCCEEDGRADES | abbieclin@gmail.com


Distribution
Downloaded by Dallen Mae D.ofKadir
this document is illegal
(kadir.dallenmae.d.bcsi@gmail.com)
lOMoARcPSD|35346190

DEWITS FUNDAMENTALStC
uvO
iaN
.cC
om
EP- T
ThSe A
MNarD
ketSpK
l a IcL
eLtoSBFuO
y aRndNSUeR
ll S
yoI u
Nr G
Stu
5dTyHME
atD
erIiaTl ION WILLIAMS TEST BANK

64. A school nurse is planning an education course for high school students on the effects of early
sexual activity. Which of the following topical points, from the CDC would she include?
a. 35% of US high school students had experienced intercourse.
b. 89% of US high school students, who had experienced intercourse, used a condom.
c. Adolescent students may have difficulty with the emotional aspects of sexual
activity.
d. Poor risk of sexually transmitted infections.
ANS: C
Centers for Disease Control and Prevention (CDC) reported in 2013 that 47% of US high
school students had experienced intercourse, and 41% had not used a condom the last time
they had sex (CDC, 2015). Although the body is capable of intercourse in adolescence, many
young people are not prepared for the emotional aspects of sexual activity. Early sexual
activity is associated with high risk of sexually transmitted infections, pregnancy, and
emotional pain (Wisnieski and Matzo, 2013).

DIF: Cognitive Level: Comprehension REF: p. 156 OBJ: Theory #13


TOP: Sexuality KEY: Nursing Process Step: N/A MSC: NCLEX: N/A

65. As a nurse working in an adolescent mental health clinic, sees many young people struggle
with sexual orientation issues. Which of the following is true according to the 2013 National
Health Interview Survey?
a. 3.6% of the population is homosexual.
b. 0.7% of the population is bisexual.
c. 30% of the population is gay, lesbian, bisexual, or transgender.
d. 0.8% of the population i d e nt i f Ni eUd Rt ShIeNmGsTeBl v. Ce Os M
as gay, lesbian, bisexual, or
transgender.
ANS: B
According to the 2013 National Health Interview Survey, studies estimate that 1.6% of the
population is homosexual and 0.7% is bisexual. However, according to 2015 Gallup polls,
Americans who were surveyed estimated that 23% of the population is gay, lesbian, bisexual,
or transgender, which is substantially higher than the 3.8% of the population who identified
themselves in the early results of the survey.

DIF: Cognitive Level: Comprehension REF: p. 156 OBJ: Theory #12


TOP: Tasks of Adolescence KEY: Nursing Process Step: N/A
MSC: NCLEX: N/A

66. A middle school female student is evaluated for depression. During the assessment, it was
evident that she was a victim of cyberbullying. According to The Cyberbullying Research
Center, which of the following will promote a positive outcome for this patient?
a. She is not alone, 26% of middle school at high school students have been the
victim of cyberbullying.
b. She is not alone, 16% of middle school at high school students have been involved
in bullying.
c. She is at risk for substance abuse.
d. She has a group of loving, supportive adults in her family.

ANS: D

NURSINGTB.COM

Downloaded by: SUCCEEDGRADES | abbieclin@gmail.com


Distribution
Downloaded by Dallen Mae D.ofKadir
this document is illegal
(kadir.dallenmae.d.bcsi@gmail.com)
lOMoARcPSD|35346190

DEWITS FUNDAMENTALStC
uvO
iaN
.cC
om
EP- T
ThSe A
MNarD
ketSpK
l a IcL
eLtoSBFuO
y aRndNSUeR
ll S
yoI u
Nr G
Stu
5dTyHME
atD
erIiaTl ION WILLIAMS TEST BANK

According to The Cyberbullying Research Center, 26% of middle school and high school
students have been victims of cyberbullying, and 16% have been involved in bullying. Studies
demonstrate decreased substance use and mental health problems when youth view the family
as a resource for support and advice (Ackley et al., 2014).

DIF: Cognitive Level: Analysis REF: p. 157 OBJ: Theory #13


TOP: Concerns in Adolescent Development KEY: Nursing Process Step: N/A
MSC: NCLEX: N/A

MULTIPLE RESPONSE

1. The nurse emphasizes the influences that help maintain a healthy pregnancy, which are:
(Select all that apply.)
a. early prenatal care.
b. health of the mother.
c. socioeconomic status.
d. mother‘s age.
e. age when menses began.
f. emotional stress.

ANS: A, B, D, F
Adequate prenatal care, health and age of the mother, good nutrition, and emotional status all
influence the health of a pregnancy. Socioeconomics and the age of menarche are not direct
influences.

DIF: Cognitive Level: Comprehension REF: p. 146 OBJ: Clinical Practice #1


TOP: Healthy Pregnancy N U R S I NKGETY
B :. C N
OMur sin g Process Step: Implementation
MSC: NCLEX: Health Promotion and Maintenance: Prevention and Early Detection of Disease

2. Parents of a 5-month-old ask the nurse what they can feed their infant besides breast milk. The
nurse would appropriately suggest: (Select all that apply.)
a. baby cereals.
b. strained fruits.
c. any food eaten by the parents.
d. small pieces of sausage.
e. grapes.
f. strained vegetables.

ANS: A, B, F
Baby cereals and strained fruits and vegetables are appropriate foods to introduce at this age.
Pieces of sausage, grapes, and some foods eaten by parents may be too small and may be
aspirated.

DIF: Cognitive Level: Comprehension REF: p. 147 OBJ: Clinical Practice #2


TOP: Infant Nutrition KEY: Nursing Process Step: Implementation
MSC: NCLEX: Health Promotion and Maintenance: Growth and Development

COMPLETION

1. The sperm and ovum each contain unpaired chromosomes.

NURSINGTB.COM

Downloaded by: SUCCEEDGRADES | abbieclin@gmail.com


Distribution
Downloaded by Dallen Mae D.ofKadir
this document is illegal
(kadir.dallenmae.d.bcsi@gmail.com)
lOMoARcPSD|35346190

DEWITS FUNDAMENTALStC
uvO
iaN
.cC
om
EP- T
ThSe A
MNarD
ketSpK
l a IcL
eLtoSBFuO
y aRndNSUeR
ll S
yoI u
Nr G
Stu
5dTyHME
atD
erIiaTl ION WILLIAMS TEST BANK

ANS:
23

Each sperm and ovum contains 23 unpaired chromosomes, which become 46 chromosomes at
fertilization.

DIF: Cognitive Level: Knowledge REF: p. 145 OBJ: Theory #1


TOP: Prenatal Development KEY: Nursing Process Step: N/A
MSC: NCLEX: Health Promotion and Maintenance: Growth and Development

2. The nurse advises the young mother that most pediatricians recommend feeding newborns
using .

ANS:
breast milk

Breast milk is designed for the newborn‘s digestive system and contains protective antibodies.

DIF: Cognitive Level: Knowledge REF: p. 147 OBJ: Clinical Practice #2


TOP: Infant Nutrition KEY: Nursing Process Step: Implementation
MSC: NCLEX: Health Promotion and Maintenance: Growth and Development

3. The nurse explains that babies can lift their heads before they can lift their chests, control their
shoulders before they control their arms and fingers, sit before they stand, and crawl before
they walk. This is a result of development.

ANS:
cephalocaudal NURSINGTB.COM

Cephalocaudal development means development occurs from head to tail. Motor development
follows from head to tail, which means that babies can use the upper body earlier than the
lower body.

DIF: Cognitive Level: Knowledge REF: p. 148 OBJ: Theory #8


TOP: Principles of Growth and Development
KEY: Nursing Process Step: Implementation
MSC: NCLEX: Health Promotion and Maintenance: Growth and Development

4. The nurse uses a diagram to show that the fetal prenatal period begins after the
week.

ANS:
eighth

The fetal period begins after the eighth week of intrauterine life and continues until birth.

DIF: Cognitive Level: Knowledge REF: p. 142 OBJ: Theory #1


TOP: Fetal Period KEY: Nursing Process Step: Implementation
MSC: NCLEX: Health Promotion and Maintenance: Growth and Development

NURSINGTB.COM

Downloaded by: SUCCEEDGRADES | abbieclin@gmail.com


Distribution
Downloaded by Dallen Mae D.ofKadir
this document is illegal
(kadir.dallenmae.d.bcsi@gmail.com)
lOMoARcPSD|35346190

Stuvia.com - The Marketplace to Buy and Sell your Study Material

Chapter 12: Adulthood and the Family


Williams: deWit's Fundamental Concepts and Skills for Nursing, 8th Edition

MULTIPLE CHOICE

1. The nurse is aware that according to Schaie‘s theory of development, the most significant
event in the life of a 26-year-old single woman who is graduating from graduate school
would be:
a. marriage and starting a family.
b. academic recognition for her high grades.
c. employment in a prestigious business.
d. a long-term relationship with a significant other.
ANS: C
According to Schaie, the achievement stage is satisfied by young adults being able to apply
what they have learned to increase their competence and increase their career choices.

DIF: Cognitive Level: Comprehension REF: p. 161 OBJ: Theory #2


TOP: Schaie‘s Theory, Young Adulthood
KEY: Nursing Process Step: Assessment
MSC: NCLEX: Health Promotion and Maintenance: Prevention of Disease and Early
Detection of Disease

2. The nurse assesses a 36-year-old clinic patient as having slightly elevated blood pressure
and being overweight by approximately 20 pounds. The most motivating suggestion the
nurse could make would be:
a. ―If you could lose that extra weight in the next month, your blood pressure would
probably be back to normal.‖
b. ―The chances of you having a stroke or heart attack will increase if you don‘t lose
some weight and bring your blood pressure down.‖
c. ―You‘ll need to take lifelong medications if diet and exercise don‘t reduce your
blood pressure.‖
d. ―Exercise and diet modification started now, even in small amounts, can improve
your overall health and delay the onset of aging.‖
ANS: D
Research has established that healthy exercise and eating patterns delay the onset of aging
and help control blood pressure.

DIF: Cognitive Level: Application REF: p. 166


OBJ: Clinical Practice #1 TOP: Exercise for Weight Loss
KEY: Nursing Process Step: Implementation
MSC: NCLEX: Health Promotion and Maintenance: Prevention of Disease and Early
Detection of Disease

3. A 48-year-old married father of two teens is involved with his children‘s sports teams as a
coach and referee. He manages a large fast-food franchise, which employs a number of
teenagers whom he trains. The nurse recognizes that this lifestyle is conducive to:
a. generativity from contact with young persons.

Downloaded by: SUCCEEDGRADES | abbieclin@gmail.com


Distribution
Downloaded by Dallen Mae D.ofKadir
this document is illegal
(kadir.dallenmae.d.bcsi@gmail.com)
lOMoARcPSD|35346190

Stuvia.com - The Marketplace to Buy and Sell your Study Material

b. self-absorption because he focuses on his recreational and business activities.


c. isolation because of limited contact with people close to his age.
d. stagnation based on repetitiveness of his work and recreation.
ANS: A
Generativity is the positive psychosocial outcome in adulthood that comes from educating
and guiding the younger generation. Isolation occurs when people fail to develop intimate
relationships; stagnation results from not being involved in the younger generation‘s life.

DIF: Cognitive Level: Application REF: p. 169 OBJ: Theory #3


TOP: Psychosocial Development, Middle Adulthood
KEY: Nursing Process Step: Assessment
MSC: NCLEX: Health Promotion and Maintenance: Growth and Development

4. When caring for a child from another culture, the nurse takes into consideration the child‘s
family because families:
a. are disruptive to the child‘s therapy.
b. shape how the child reacts to other people.
c. may create a break in confidentiality.
d. bring inappropriate foods to the child.
ANS: B
Families from all cultures shape cultural values and how the child interacts with others.

DIF: Cognitive Level: Comprehension REF: p. 161 OBJ: Theory #4


TOP: Families KEY: Nursing Process Step: Planning
MSC: NCLEX: Health Promotion and Maintenance: Growth and Development

5. A recently divorced young mother tells the nurse in the well child clinic that her 5-year-old
has become very withdrawn since the divorce. The nurse‘s best response to this concern
would be:
a. ―He probably misses his dad. Young children adjust quickly though.‖
b. ―He may be acting out his resentment toward you for the breaking up of the family
unit.‖
c. ―Young children frequently feel they are to blame for the family breakup.‖
d. ―You should explain to him why the marriage was no longer sustainable.‖
ANS: C
Young children frequently feel that they are to blame for the family breakup and experience
guilt feelings. The other three items are subjective.

DIF: Cognitive Level: Application REF: p. 162


OBJ: Clinical Practice #5
TOP: Psychosocial Development, Young Adulthood
KEY: Nursing Process Step: Implementation
MSC: NCLEX: Psychosocial Integrity: Coping and Adaptation

6. The nurse takes into consideration that divorce affects more than just the parents;
adolescents who come from a divorced family may experience:
a. lack of trust in their own heterosexual relationships.

Downloaded by: SUCCEEDGRADES | abbieclin@gmail.com


Distribution
Downloaded by Dallen Mae D.ofKadir
this document is illegal
(kadir.dallenmae.d.bcsi@gmail.com)
lOMoARcPSD|35346190

Stuvia.com - The Marketplace to Buy and Sell your Study Material

b. a distancing from the family unit.


c. hostile feeling that is acted out in activities such as vandalism and drug use.
d. being rushed into adulthood.
ANS: A
Divorce frequently leads adolescents to lack trust in their own heterosexual relationships.

DIF: Cognitive Level: Knowledge REF: p. 162 OBJ: Theory #5


TOP: Divorce and Families KEY: Nursing Process Step: Planning
MSC: NCLEX: Psychosocial Integrity: Coping and Adaptation

7. During a routine health examination, to obtain effective information relating to common


health problems, the question the nurse could ask a 24-year-old patient that can introduce
further assessments is:
a. ―Are you sexually active?‖
b. ―Have you noticed a change in your vision or hearing?‖
c. ―Do you experience chest pain when exercising?‖
d. ―Have you ever had the flu or pneumonia immunizations?‖
ANS: A
A common health problem of young adults is risk for unhealthy behaviors, among which is
irresponsible sexual behavior that can result in unwanted pregnancy, sexually transmitted
infections, and, in particular, HIV infection.

DIF: Cognitive Level: Application REF: p. 163


OBJ: Clinical Practice #2 TOP: Health Concerns, Young Adulthood
KEY: Nursing Process Step: Assessment
MSC: NCLEX: Physiological Integrity: Basic Care and Comfort

8. The nurse appropriately advises a 28-year-old woman that to detect breast cancer in early
stages she should:
a. schedule annual breast mammograms for early detection.
b. perform monthly breast self-examinations and begin mammograms at age 40.
c. arrange for annual examinations by her primary care provider and mammograms as
indicated from the physical examination.
d. schedule a complete physical examination every 5 years until age 50, then annual
examinations that include a mammogram.
ANS: B
Breast self-examination should be taught and emphasized as a monthly early detection
technique to all women. Mammograms are usually begun after age 40 unless there is a
family history of breast cancer.

DIF: Cognitive Level: Application REF: p. 163


OBJ: Clinical Practice #2 TOP: Health Concerns, Young Adulthood
KEY: Nursing Process Step: Implementation
MSC: NCLEX: Health Promotion and Maintenance: Prevention of Disease and Early
Detection of Disease

Downloaded by: SUCCEEDGRADES | abbieclin@gmail.com


Distribution
Downloaded by Dallen Mae D.ofKadir
this document is illegal
(kadir.dallenmae.d.bcsi@gmail.com)
lOMoARcPSD|35346190

Stuvia.com - The Marketplace to Buy and Sell your Study Material

9. A nurse cautions that stress-related illnesses such as headaches and hypertension in young
adults most likely will:
a. diminish as the person ages.
b. require no long-term treatment.
c. increase in intensity over the years.
d. post no threat to health.
ANS: C
Headaches and hypertension are often stress related and should be treated because these
disorders may intensify as the person ages and pose a threat to health.

DIF: Cognitive Level: Knowledge REF: p. 163


OBJ: Clinical Practice #2 TOP: Health Concerns, Young Adulthood
KEY: Nursing Process Step: Implementation
MSC: NCLEX: Health Promotion and Maintenance: Prevention of Disease and Early
Detection of Disease

10. The school nurse speaking to a group of college sophomores reminds them that in the
modern world of work, it is normal to have as many as different jobs throughout their
work life.
a. four
b. five
c. six
d. seven
ANS: D
The myth of the ideal job has given way to a person‘s being employed in as many as seven
jobs in their work life.

DIF: Cognitive Level: Knowledge REF: p. 164 OBJ: Theory #6


TOP: Employment for Young Adults KEY: Nursing Process Step: Implementation
MSC: NCLEX: Health Promotion and Maintenance: Prevention of Disease and Early
Detection of Disease

11. The nurse suggests that one option for people in middle adulthood to reduce stress is to:
a. begin planning for retirement by developing leisure time activities or hobbies that
they will be able to continue into their later years.
b. focus their time and energy on succeeding in their work environment with the
knowledge that hard work will guarantee a secure position until retirement.
c. change employment frequently to increase their skills and employability in a
rapidly changing job market.
d. find employment that pays very well, even if the work is not enjoyable, because
that is the way to accumulate enough funds for eventual retirement.
ANS: A
Leisure activities and early planning for retirement are excellent stress reducers for the
middle adult.

DIF: Cognitive Level: Comprehension REF: p. 168


OBJ: Clinical Practice #3 TOP: Work Life, Middle Adulthood

Downloaded by: SUCCEEDGRADES | abbieclin@gmail.com


Distribution
Downloaded by Dallen Mae D.ofKadir
this document is illegal
(kadir.dallenmae.d.bcsi@gmail.com)
lOMoARcPSD|35346190

Stuvia.com - The Marketplace to Buy and Sell your Study Material

KEY: Nursing Process Step: Implementation


MSC: NCLEX: Health Promotion and Maintenance: Prevention of Disease and Early
Detection of Disease

12. The nurse is aware that when the children of middle adults have grown and left home, the
middle adults may:
a. enjoy being part of a ―sandwich generation‖ that has no responsibilities to either
children or older parents.
b. suddenly show marked changes in their personalities and habits that have
characterized them since young adulthood.
c. experience the ―empty nest syndrome‖ characterized by depression and a sense of
loss or sadness.
d. look forward to their children becoming ―boomerang children‖ who visit briefly
with their expanding families and quickly return to their own homes.
ANS: C
The empty nest syndrome, with its sense of depression and loss, occurs for some adults,
especially those whose lives had revolved around their children, when their children leave
home.

DIF: Cognitive Level: Comprehension REF: p. 168 OBJ: Theory #6


TOP: Empty Nest Syndrome KEY: Nursing Process Step: Assessment
MSC: NCLEX: Health Promotion and Maintenance: Growth and Development

13. A 48-year-old woman asks the clinic nurse about symptoms she has experienced, such as
trouble sleeping, because she wakes up feeling hot and sweaty. She has also felt tired and
depressed about aging. The most helpful nursing response would be:
a. ―You may have an illness that the doctor should evaluate.‖
b. ―It sounds to me as though you might have symptoms of menopause.‖
c. ―You‘re too young to be going through menopause. Are you worried about
anything?‖
d. ―Those symptoms are all related to aging, and there‘s not much to do about it.‖
ANS: B
Symptoms of menopause often occur before the cessation of menses and are treatable. They
are not a disease or illness but part of normal development.

DIF: Cognitive Level: Application REF: p. 167


OBJ: Clinical Practice #3 TOP: Menopause
KEY: Nursing Process Step: Implementation
MSC: NCLEX: Physiological Integrity: Basic Care and Comfort

14. The nurse reviews the medications of a 52-year-old woman in the outpatient clinic to
confirm that her calcium dose has been:
a. increased to 1200 mg/day.
b. reduced to 1000 mg/day.
c. replaced by a daily dose of vitamin D.
d. augmented with glucosamine.
ANS: A

Downloaded by: SUCCEEDGRADES | abbieclin@gmail.com


Distribution
Downloaded by Dallen Mae D.ofKadir
this document is illegal
(kadir.dallenmae.d.bcsi@gmail.com)
lOMoARcPSD|35346190

Stuvia.com - The Marketplace to Buy and Sell your Study Material

Women over the age of 51 should increase their calcium to 1200 mg a day. Vitamin D is also
important to help utilize the calcium.

DIF: Cognitive Level: Application REF: p. 167


OBJ: Clinical Practice #2 TOP: Calcium Replacement
KEY: Nursing Process Step: Implementation
MSC: NCLEX: Physiological Integrity: Pharmacological Therapies

15. A nurse is instructing a patient about the prevention of automobile accidents and reminds the
65-year-old patient that he may be prone to an accident because of:
a. increasing speed of traffic.
b. his tendency to be distracted.
c. a slower reaction time.
d. reduced strength.
ANS: C
Among the changes associated with age is the slowed reaction time.

DIF: Cognitive Level: Completions REF: p. 167 OBJ: Theory #6


TOP: Changes Associated with Age KEY: Nursing Process Step: Implementation
MSC: NCLEX: Physiological Integrity: Physiological Adaptation

16. A primary care provider has ordered a Mantoux test. The patient asks the purpose of this
test. The nurse tells the patient this test is done to screen for:
a. Lyme disease.
b. lung cancer.
c. valley fever.
d. tuberculosis.
ANS: D
Periodic Mantoux tests are advised to screen for tuberculosis.

DIF: Cognitive Level: Knowledge REF: p. 164


OBJ: Clinical Practice #1 TOP: Screening Tests
KEY: Nursing Process Step: Implementation
MSC: NCLEX: Health Promotion and Maintenance: Prevention of Disease and Early
Detection of Disease

17. When the nurse in a long-term facility observes a resident bringing his reading material very
close to his face, the nurse assesses that the patient is experiencing:
a. presbyopia.
b. myopia.
c. presbycusis.
d. cataracts.
ANS: A
Presbyopia is caused by decreased flexibility of the eye lens. This makes near vision more
difficult and leads to the need for reading glasses.

DIF: Cognitive Level: Knowledge REF: p. 166

Downloaded by: SUCCEEDGRADES | abbieclin@gmail.com


Distribution
Downloaded by Dallen Mae D.ofKadir
this document is illegal
(kadir.dallenmae.d.bcsi@gmail.com)
lOMoARcPSD|35346190

Stuvia.com - The Marketplace to Buy and Sell your Study Material

OBJ: Clinical Practice #3 TOP: Middle Adults


KEY: Nursing Process Step: Assessment
MSC: NCLEX: Physiological Integrity: Physiological Adaptation

18. A patient states that she uses the supplement black cohosh. The nurse recognizes black
cohosh is used:
a. for reduction of blood pressure and cholesterol.
b. to lessen signs and symptoms of menopause.
c. to help maintain cartilage and decrease arthritis.
d. to improve blood flow to the brain and decrease forgetfulness.
ANS: B
Black cohosh is used to lessen signs and symptoms of menopause.

DIF: Cognitive Level: Comprehension REF: p. 166


OBJ: Clinical Practice #3 TOP: Middle Adults
KEY: Nursing Process Step: Assessment
MSC: NCLEX: Physiological Integrity: Pharmacological Therapies

19. A nurse is educating a female patient about bone health in middle adults. The nurse would
recognize the need for further education if the patient states:
a. ―Since I have a family history of osteoporosis, I will begin regular bone density
screening at age 45.‖
b. ―Since I have experienced a loss of 2 inches in height since I was 20, I will seek
screening for osteoporosis.‖
c. ―I will obtain sufficient vitamin D from sunlight or take a vitamin D supplement as
recommended by my primary care provider.‖
d. ―I will take 1500 mg/day of calcium until menopause and then decrease my
calcium intake to 1200 mg/day after menopause.‖
ANS: D
Calcium intake should be 1000 mg/day prior to menopause and should be increased to 1200
mg/day after menopause. Obtaining sufficient vitamin D from sunlight or supplementation
is also important.

DIF: Cognitive Level: Comprehension REF: p. 167


OBJ: Clinical Practice #3 TOP: Middle Adults
KEY: Nursing Process Step: Assessment
MSC: NCLEX: Health Promotion and Maintenance: Prevention of Disease and Early
Detection of Disease

20. A nurse talking to a group of women about the signs and symptoms occurring with
menopause would explain that expected symptom include all of the following except:
a. frequent insomnia.
b. increased vaginal lubrication.
c. increased emotional lability.
d. increased fatigue.
ANS: B

Downloaded by: SUCCEEDGRADES | abbieclin@gmail.com


Distribution
Downloaded by Dallen Mae D.ofKadir
this document is illegal
(kadir.dallenmae.d.bcsi@gmail.com)
lOMoARcPSD|35346190

Stuvia.com - The Marketplace to Buy and Sell your Study Material

Signs and symptoms occurring with menopause include decreased vaginal lubrication,
emotional lability, fatigue, flushing and hot flashes with heavy sweating, headache, heart
palpitations, and insomnia.

DIF: Cognitive Level: Knowledge REF: p. 167|Box 12-5


OBJ: Clinical Practice #6 TOP: Menopause
KEY: Nursing Process Step: Assessment
MSC: NCLEX: Physiological Integrity: Basic Care and Comfort

21. A nurse is concerned about a 21-year-old who does not seem to have any intimate
relationships. The nurse is aware that according to the Eriksonian theory, a lack of intimacy
in this person‘s life can result in:
a. mistrust.
b. guilt.
c. role confusion.
d. isolation.
ANS: D
Erik Erikson‘s psychosocial development theory includes the stage of intimacy vs. isolation.
This stage is where love relationships are developed, and fear of intimate relationships will
lead to a sense of isolation.

DIF: Cognitive Level: Knowledge REF: p. 161 OBJ: Theory #3


TOP: Theorists KEY: Nursing Process Step: Implementation
MSC: NCLEX: Health Promotion and Maintenance: Growth and Development

22. The nurse caring for a 42-year-old patient is satisfied that Erik Erikson‘s psychosocial
development phase of generativity has been satisfied by the fact that this patient is:
a. reflecting on one‘s life and one‘s achievements.
b. investigating and identifying alternatives regarding vocation.
c. attaining new intellectual skills and developing his career.
d. fulfilling parenting and civic responsibilities.
ANS: D
Erik Erikson‘s psychosocial development theory includes the stage of generativity vs.
stagnation in which parenting, nurturing others, and fulfilling civic responsibilities are the
tasks. The adult finds ways to be productive and of help to others in order to grow
personally.

DIF: Cognitive Level: Knowledge REF: p. 161 OBJ: Theory #3


TOP: Theorists KEY: Nursing Process Step: Assessment
MSC: NCLEX: Health Promotion and Maintenance: Growth and Development

23. Marriages may improve after the children are grown and gone if the partners:
a. are of the same age.
b. have developed mutual interests.
c. have adequate income.
d. are busy with community activities.
ANS: B

Downloaded by: SUCCEEDGRADES | abbieclin@gmail.com


Distribution
Downloaded by Dallen Mae D.ofKadir
this document is illegal
(kadir.dallenmae.d.bcsi@gmail.com)
lOMoARcPSD|35346190

Stuvia.com - The Marketplace to Buy and Sell your Study Material

The development of mutual interests and having more time for each other often improve
marriages after the children are grown.

DIF: Cognitive Level: Comprehension REF: p. 168 OBJ: Theory #1


TOP: Theorists KEY: Nursing Process Step: N/A
MSC: NCLEX: Health Promotion and Maintenance: Growth and Development

24. A young adult patient is admitted to the hospital with asthma. He is considered a millennial.
Millennials:
a. were born between 1978 and 2004.
b. make up over one half of the population.
c. are less ethnically diverse than previous generations.
d. outnumber the baby boomers.
ANS: A
Millennials (people born between 1982 and 2000) now outnumber the baby boomers. There
are now over 83 million millennials, making up over one quarter of the US population (US
Census Bureau, 2015). Millennials are also more ethnically diverse than previous
generations.

DIF: Cognitive Level: Comprehension REF: p. 162 OBJ: Theory #6


TOP: Young Adults KEY: Nursing Process Step: N/A
MSC: NCLEX: N/A

25. A nurse is preparing a patient education brochure for a class she is teaching at a local
community outreach center on pregnancy. Which of the following facts would she include in
the brochure?
a. Age of first-time mothers is 30.
b. Birth rates are increasing for teenagers aged 15 to 19.
c. Birth rates are declining for women in their 20s.
d. Birth rates are decreasing for women in their 30s and late 40s.
ANS: C
The National Center for Health Statistics reports that the average age of first-time mothers is
26 (Family Facts, 2011). The birth rates are declining for teenagers aged 15 to 19, as well as
for women in their 20s. In contrast, rates are increasing for women in their 30s and late 40s
(National Vital Statistics Report, 2015).

DIF: Cognitive Level: Comprehension REF: p. 165 OBJ: Theory #6


TOP: Parenting KEY: Nursing Process Step: N/A MSC: NCLEX: N/A

26. A middle aged adult has been diagnosed with stage 1 hypertension. The nurse prepares
patient teaching that would include which of the following?
a. Diet with increased savory, rich foods.
b. Regular social drinking with friends and family.
c. Regular aerobic exercise.
d. Keep blood pressure below 120/80 mm Hg.
ANS: C
Pressure control in middle adults.

Downloaded by: SUCCEEDGRADES | abbieclin@gmail.com


Distribution
Downloaded by Dallen Mae D.ofKadir
this document is illegal
(kadir.dallenmae.d.bcsi@gmail.com)
lOMoARcPSD|35346190

Stuvia.com - The Marketplace to Buy and Sell your Study Material

Adults should keep their blood pressure—below 120/80 mm Hg; adults with
prehypertension 120-139/80-89; adults for stage 1 hypertension 140-159/90-99—by
• Decreasing sodium intake
• Drinking minimal alcohol
• Participating in regular aerobic exercise
• Promoting an overall healthy lifestyle
Source: Mayo Clinic (2015)

DIF: Cognitive Level: Analysis REF: p. 168


OBJ: Clinical Practice #7 TOP: Health Concerns
KEY: Nursing Process Step: N/A MSC: NCLEX: N/A

MULTIPLE RESPONSE

1. When planning a community health fair, which of the following topics are most appropriate
for the nurse to include to target the young adult? (Select all that apply.)
a. Sexually transmitted infections, including HIV
b. Alcohol and drug abuse
c. Development of proper diet and exercise regimen
d. Foot care
e. Screening for presbyopia
f. Osteoporosis
ANS: A, B, C
Young adults typically are in good health but may develop unhealthy behaviors including
chemical abuse; sedentary lifestyle; overeating; and sexual promiscuity, which can lead to
sexually transmitted infections, including HIV.

DIF: Cognitive Level: Application REF: p. 163


OBJ: Clinical Practice #2 TOP: Health Promotion, Young Adulthood
KEY: Nursing Process Step: Planning
MSC: NCLEX: Health Promotion and Maintenance: Prevention of Disease and Early
Detection of Disease

2. When planning a community health program that is directed at young adults, the nurse will
prepare the material to: (Select all that apply.)
a. build on previous knowledge.
b. design clear goals for the program.
c. indicate how the new knowledge can be of benefit.
d. use straight lecture method to get points across quickly.
e. relate material to common concerns of young people.
ANS: A, B, C, E
Building on previous knowledge, designing clear goals based on common concerns, and
stressing how the material can be of benefit should be part of the plan. The straight lecture
method is not the most effective way of communicating with young adults.

DIF: Cognitive Level: Application REF: p. 164


OBJ: Clinical Practice #3 TOP: Health Promotion, Young Adulthood

Downloaded by: SUCCEEDGRADES | abbieclin@gmail.com


Distribution
Downloaded by Dallen Mae D.ofKadir
this document is illegal
(kadir.dallenmae.d.bcsi@gmail.com)
lOMoARcPSD|35346190

Stuvia.com - The Marketplace to Buy and Sell your Study Material

KEY: Nursing Process Step: Planning


MSC: NCLEX: Health Promotion and Maintenance: Prevention of Disease and Early
Detection of Disease

3. Susan, a 21-year-old, has just graduated from college and will begin teaching at the local
elementary school in the fall. Her parents have been supportive and have assisted with her
tuition throughout college and consoled her when she became overwhelmed at school. They
are also helping Susan find an apartment close to the school where she will teach. At this
point in her life, Susan will be faced with which of the following developmental tasks?
(Select all that apply.)
a. Marriage
b. Parenting
c. Planning for retirement
d. Caring for parents
e. Developing a social group
ANS: A, B, E
The decision to marry and become a parent, as well as developing a network of friends, are
developmental tasks for the young adult.

DIF: Cognitive Level: Application REF: p. 165 OBJ: Theory #3


TOP: Developmental Tasks, Young Adulthood
KEY: Nursing Process Step: Assessment
MSC: NCLEX: Health Promotion and Maintenance: Growth and Development

COMPLETION

1. A 35-year-old mother of three children, ages 8, 10, and 13, has decided to return to school to
become a nurse. Her husband and parents have been very supportive of her decision and will
help with child care while she is in school. According to Schaie‘s theory for cognitive
development, she is in the stage, which occurs in the stage
of adulthood.

ANS:
achievement; young

Schaie identifies the young adult stage with the achievement stage of cognitive
development. The achievement stage features the person‘s need to improve, demonstrate
competence, and advance in a career. Young adulthood is generally considered to be
between the ages of 19 and 45.

DIF: Cognitive Level: Application REF: p. 161 OBJ: Theory #2


TOP: Cognitive Theory, Young Adulthood
KEY: Nursing Process Step: N/A MSC: NCLEX: N/A

2. A 55-year-old is married with two children, ages 15 and 17. He is a supervisor for the local
utility company, where he has been employed for 25 years. He is active in his community
and serves on the town council. According to Schaie‘s theory of cognitive development, he
is in the stage, which occurs in the stage of adulthood.

Downloaded by: SUCCEEDGRADES | abbieclin@gmail.com


Distribution
Downloaded by Dallen Mae D.ofKadir
this document is illegal
(kadir.dallenmae.d.bcsi@gmail.com)
lOMoARcPSD|35346190

Stuvia.com - The Marketplace to Buy and Sell your Study Material

ANS:
responsibility; middle

Schaie identifies the middle stage of adulthood with the responsibility stage of cognitive
development. The responsibility stage features a time of great responsibility for self, family,
job, and community. Middle adulthood is generally considered to be between the ages of 46
and 64.

DIF: Cognitive Level: Application REF: p. 161 OBJ: Theory #2


TOP: Cognitive Theory, Middle Adulthood
KEY: Nursing Process Step: N/A MSC: NCLEX: N/A

3. A 24-year-old mother of two has just become divorced from her husband of 4 years. She is
living with her parents again until she is able to get her finances in order to find her own
apartment. This is an example of children.

ANS:
boomerang

Young adults who return to the parental home for a period of time are referred to as
boomerang children.

DIF: Cognitive Level: Comprehension REF: p. 165 OBJ: Theory #6


TOP: Young Adult Psychosocial Development
KEY: Nursing Process Step: Assessment
MSC: NCLEX: Psychosocial Integrity: Coping and Adaptation

Downloaded by: SUCCEEDGRADES | abbieclin@gmail.com


Distribution
Downloaded by Dallen Mae D.ofKadir
this document is illegal
(kadir.dallenmae.d.bcsi@gmail.com)
lOMoARcPSD|35346190

DEWITS FUNDAMENTALStC
uvO
iaN
.cC
om
EP- T
ThSe A
MNarD
ketSpK
l a IcL
eLtoSBFuO
y aRndNSUeR
ll S
yoI u
Nr G
Stu
5dTyHME
atD
erIiaTl ION WILLIAMS TEST BANK

Chapter 13: Promoting Healthy Adaptation to Aging


Williams: deWit's Fundamental Concepts and Skills for Nursing, 8th Edition

MULTIPLE CHOICE

1. An 82-year-old man is alert and oriented and in good physical health except for some
arthritic pain that ―slows me down, but I just keep moving.‖ He lives alone in an apartment
in a senior citizen complex but enjoys the company of other residents and takes part in the
social activities there. His lifestyle is an example of:
a. an exception to the expected norm.
b. the disengagement theory.
c. the activity theory.
d. the biological theory.
ANS: C
The activity theory states that persons who remain active and interested in outside activities
live longer.

DIF: Cognitive Level: Analysis REF: p. 173 OBJ: Theory #3


TOP: Theories of Aging KEY: Nursing Process Step: Assessment
MSC: NCLEX: Psychosocial Integrity: Psychosocial Adaptation

2. The nurse understands that the best explanation of why a particular person lives into his 90s
in relatively good health is that the person:
a. had healthy parents who passed on to him ―good‖ genes and no hereditary
diseases. NURSINGTB.COM
b. has lived a healthful lifestyle, which has included preventative care, good nutrition,
exercise, and a positive outlook.
c. has treated illness with ―old-fashioned‖ home remedies while avoiding the use of
many prescription drugs.
d. reads a lot about the newest advances in health care and tries these approaches as
soon as they are publicized.
ANS: B
Lifestyle and personality together probably contribute to longevity more than genetics, new
medical approaches, or old-fashioned home remedies, although those may contribute in
important ways. A healthy lifestyle decreases the risk of disease and its sequelae.

DIF: Cognitive Level: Comprehension REF: p. 174 OBJ: Theory #3


TOP: Longevity KEY: Nursing Process Step: Assessment
MSC: NCLEX: Physiological Integrity: Physiological Adaptation

3. An 85-year-old widow who lives alone has fallen several times in the last month and has
been noted by her children to be confused about her medications and to frequently ―forget‖
what is cooking on the stove until it is burned. These observations indicate that:
a. she needs to be hospitalized to determine the cause of her confusion and falls.
b. home delivery of a hot meal every day would solve her cooking risk.
c. nursing home placement would be the best solution to her problems of aging.
d. further assessment is needed to evaluate an increased level of assistance.

NURSINGTB.COM

Downloaded by: SUCCEEDGRADES | abbieclin@gmail.com


Distribution
Downloaded by Dallen Mae D.ofKadir
this document is illegal
(kadir.dallenmae.d.bcsi@gmail.com)
lOMoARcPSD|35346190

DEWITS FUNDAMENTALStC
uvO
iaN
.cC
om
EP- T
ThSe A
MNarD
ketSpK
l a IcL
eLtoSBFuO
y aRndNSUeR
ll S
yoI u
Nr G
Stu
5dTyHME
atD
erIiaTl ION WILLIAMS TEST BANK

ANS: D
The widow is showing signs of needing additional assistance. It might include home
delivered meals or nursing home placement, but further assessment needs to be performed.
Acute care hospitalization is not indicated in this situation.

DIF: Cognitive Level: Analysis REF: p. 180


OBJ: Clinical Practice #5 TOP: Aging Parents
KEY: Nursing Process Step: Planning
MSC: NCLEX: Psychosocial Integrity: Coping and Adaptation

4. The nurse clarifies that the immune system failure theory states that:
a. older adults lose their ability to effectively respond to infections and are more
likely to die from them.
b. the body no longer recognizes itself and begins to attack itself, causing illness.
c. toxins and harmful chemicals (free radicals) in the environment cause damage to
body cells.
d. the diminished activity of older adults make them susceptible to illness.
ANS: A
With advancing age, the immune system of older adults has less ability to fight infections.

DIF: Cognitive Level: Knowledge REF: p. 173 OBJ: Theory #1


TOP: Theories of Aging KEY: Nursing Process Step: Assessment
MSC: NCLEX: Physiological Integrity: Physiological Adaptation

5. The nurse seeks out residents wiNt hU RwShI Nom


G TtBh .eCyOrM
em i n i sc e in an interview regarding the
resident‘s life story. The value of this activity is that:
a. it gives the staff more time to care for other residents and perform routine care.
b. it can affirm the positives of the resident‘s life and lead to ego integrity for the
resident.
c. nursing home residents have long days to pass, and any activity helps the time pass
more quickly.
d. students learn about the ―good old days‖ and better appreciate the modern
conveniences available today.
ANS: B
Erikson‘s psychosocial task for older adults is ego integrity, which can be assisted by life
review and reminiscence.

DIF: Cognitive Level: Comprehension REF: p. 178 OBJ: Theory #6


TOP: Psychological Aspects of Aging KEY: Nursing Process Step: Implementation
MSC: NCLEX: Psychosocial Integrity: Psychosocial Adaptation

6. The home health aide who visits an older adult reports to the nurse that the patient has many
new bruises on her arms and face. The patient states that her daughter ―has a temper and got
mad‖ when she visited the day before. The nurse should:
a. call the daughter for clarification.
b. notify her nursing supervisor to report the incident to the authorities.
c. tell the aide to report it to the nursing supervisor.

NURSINGTB.COM

Downloaded by: SUCCEEDGRADES | abbieclin@gmail.com


Distribution
Downloaded by Dallen Mae D.ofKadir
this document is illegal
(kadir.dallenmae.d.bcsi@gmail.com)
lOMoARcPSD|35346190

DEWITS FUNDAMENTALStC
uvO
iaN
.cC
om
EP- T
ThSe A
MNarD
ketSpK
l a IcL
eLtoSBFuO
y aRndNSUeR
ll S
yoI u
Nr G
Stu
5dTyHME
atD
erIiaTl ION WILLIAMS TEST BANK

d. plan to visit the patient when the daughter will be present.


ANS: B
It is a legal requirement for licensed health care personnel to report suspected elder abuse to
protect the public (patient). An LPN should report to the RN supervisor first. The aide is
supervised by the nurse and thus the nurse is responsible. It is not the nurse‘s responsibility
to determine whether the daughter is abusing the patient but only to report it so that it can be
investigated by the proper agency.

DIF: Cognitive Level: Analysis REF: p. 179 OBJ: Theory #4


TOP: Family Relationships KEY: Nursing Process Step: Assessment
MSC: NCLEX: Safe, Effective Care Environment: Safety and Infection Control

7. A 76-year-old neighbor confides to the nurse that she is having to get up in the middle of the
night to void and often wets herself before she can get to the bathroom during the day. The
nurse‘s best explanation would be:
a. ―As you get older, your bladder muscle loses tone and you can hold less urine.‖
b. ―You are probably drinking too much during the day and especially before
bedtime.‖
c. ―You probably have a bladder infection that should be treated with antibiotics.‖
d. ―With age, the urine becomes very concentrated and causes the bladder to empty
spontaneously.‖
ANS: A
Physiological changes of aging include decreased bladder and sphincter tone, resulting in
stress incontinence and more frequent voiding.
NURSINGTB.COM
DIF: Cognitive Level: Application REF: p. 175|Table 13-1
OBJ: Clinical Practice #1 TOP: Physical Changes
KEY: Nursing Process Step: Assessment
MSC: NCLEX: Physiological Integrity: Physiological Adaptation

8. A nurse making a home visit to a 75-year-old woman discovers that the patient stays in bed
or on the couch most of the time because she is afraid of falling. The nurse should arrange
for:
a. a sitter to stay with the woman during the day.
b. a wheelchair to increase mobility.
c. a physical therapist to teach resistance training.
d. a special cushion for the chair and bed to reduce the risk of decubiti.
ANS: C
A physical therapist can teach resistance and balance training to help prevent falls.

DIF: Cognitive Level: Analysis REF: p. 171 OBJ: Theory #4


TOP: Physical Health Concerns KEY: Nursing Process Step: Implementation
MSC: NCLEX: Physiological Integrity: Physiological Adaptation

NURSINGTB.COM

Downloaded by: SUCCEEDGRADES | abbieclin@gmail.com


Distribution
Downloaded by Dallen Mae D.ofKadir
this document is illegal
(kadir.dallenmae.d.bcsi@gmail.com)
lOMoARcPSD|35346190

DEWITS FUNDAMENTALStC
uvO
iaN
.cC
om
EP- T
ThSe A
MNarD
ketSpK
l a IcL
eLtoSBFuO
y aRndNSUeR
ll S
yoI u
Nr G
Stu
5dTyHME
atD
erIiaTl ION WILLIAMS TEST BANK

9. The home health nurse assesses an 80-year-old who has fallen twice in the last 10 days. The
nurse discovers that the patient uses a cane for ambulation stability, drinks up to 6 cups of
coffee a day, has altered depth perception because of cataracts, and has recently carpeted her
home with a smooth tight weave carpet. The nurse suspects the cause of her recent falls to
be:
a. the use of the cane.
b. the new carpet.
c. the large intake of caffeine.
d. the altered depth perception.
ANS: D
Altered depth perception, changes in gait, and slow reaction times may cause falls in older
adults. The cane helps with stability, and the smooth tight weave carpet should not cause
falls.

DIF: Cognitive Level: Analysis REF: p. 175|Table 13-1


OBJ: Theory #4 TOP: Fall Prevention
KEY: Nursing Process Step: Assessment
MSC: NCLEX: Physiological Integrity: Physiological Adaptation

10. A 70-year-old widower continues to work part time in a demanding business, while
traveling in his free time with a 68-year-old widow to visit longtime friends and relatives.
He remains an active member of several community organizations. His life is an example of
a person who:
a. is acting like a younger person to prove to himself that he is not ―old.‖
b. is adjusting well to the changes of benign senescence.
c. should slow down and en jo yNrUe Rti SreI NmGeTnBt .w
CO
hiM
le he is still healthy.
d. is avoiding the psychosocial tasks of aging such as retirement and slowing down.
ANS: B
Benign senescence is the normal changes of aging, and many healthy people at 70 years of
age do not fit the stereotype of ―old‖ but are more like older adults of 45 to 70 years.

DIF: Cognitive Level: Analysis REF: p. 170 OBJ: Theory #3


TOP: Cognitive Aspects of Aging KEY: Nursing Process Step: Assessment
MSC: NCLEX: Health Promotion and Maintenance

11. A 78-year-old man is admitted to the hospital after a fall from his bed at home that resulted
in a fractured hip and several fractured ribs. He states, ―I don‘t know how I broke so many
bones. I only fell out of bed.‖ The nurse can explain that:
a. loss of calcium from bone occurs in older adults and can result in fractures from
minor trauma.
b. it is likely that an underlying disease made him more susceptible to fractures.
c. the bedside table near the bed added to his injuries.
d. the height of the bed will need to be lowered when he goes home to prevent further
injuries.
ANS: A
A physiological change of aging is loss of calcium from bone, resulting in osteoporosis and
greater likelihood of fractures.

NURSINGTB.COM

Downloaded by: SUCCEEDGRADES | abbieclin@gmail.com


Distribution
Downloaded by Dallen Mae D.ofKadir
this document is illegal
(kadir.dallenmae.d.bcsi@gmail.com)
lOMoARcPSD|35346190

DEWITS FUNDAMENTALStC
uvO
iaN
.cC
om
EP- T
ThSe A
MNarD
ketSpK
l a IcL
eLtoSBFuO
y aRndNSUeR
ll S
yoI u
Nr G
Stu
5dTyHME
atD
erIiaTl ION WILLIAMS TEST BANK

DIF: Cognitive Level: Application REF: p. 175|Table 13-1


OBJ: Theory #4 TOP: Physical Changes of Aging
KEY: Nursing Process Step: Implementation
MSC: NCLEX: Physiological Integrity: Physiological Adaptation

12. An 84-year-old patient who is hospitalized for pneumonia says to the nurse, ―I don‘t know
why I‘ve lived so long. All my friends are gone, my children and grandchildren are all
independent and successful—nobody needs me anymore.‖ An appropriate response by the
nurse is:
a. ―You have a lot to live for. Your children and grandchildren really love you.‖
b. ―You must have inherited good genes. You should be thankful you‘re still alive.‖
c. ―Tell me about your children and what it was like when you were raising them.‖
d. ―The pneumonia has made you feel morbid. Things will look up!‖
ANS: C
Encouraging reminiscing helps the older adult put her life in perspective and acquire a sense
of ego.

DIF: Cognitive Level: Application REF: p. 178


OBJ: Clinical Practice #4 TOP: Psychosocial Aspects of Aging
KEY: Nursing Process Step: Implementation
MSC: NCLEX: Psychosocial Integrity: Coping and Adaptation

13. An 80-year-old man has no chronic diseases and is alert, oriented, and physically active.
Which of the following nursing diagnoses is likely to be present as a result of the normal
aging process? NURSINGTB.COM
a. Poisoning, risk for, related to the many medications an older person must take.
b. Suffocation, risk for, related to declining respiratory function.
c. Hopelessness, related to recognition of the end of life drawing near.
d. Constipation, related to slowed peristalsis.
ANS: D
Constipation is a common nursing diagnosis for older adults in otherwise good health. In
addition to peristalsis slowing, most older adults experience decreased taste and smell
resulting in decreased appetite and food intake, as well as decreased physical mobility.

DIF: Cognitive Level: Analysis REF: p. 175|Table 13-1


OBJ: Theory #4 TOP: Physical Changes
KEY: Nursing Process Step: Nursing Diagnosis
MSC: NCLEX: Health Promotion and Maintenance

14. The nurse warns that infection after exposure to respiratory illness in the older adult:
a. is more likely to result in serious lower respiratory infection related to weakened
respiratory muscles and fewer cilia.
b. is likely to be fatal due to the older person‘s low resistance.
c. results in rates of infection similar to those in the younger adult.
d. can be easily prevented with the use of antibiotics when the adult is exposed.
ANS: A

NURSINGTB.COM

Downloaded by: SUCCEEDGRADES | abbieclin@gmail.com


Distribution
Downloaded by Dallen Mae D.ofKadir
this document is illegal
(kadir.dallenmae.d.bcsi@gmail.com)
lOMoARcPSD|35346190

DEWITS FUNDAMENTALStC
uvO
iaN
.cC
om
EP- T
ThSe A
MNarD
ketSpK
l a IcL
eLtoSBFuO
y aRndNSUeR
ll S
yoI u
Nr G
Stu
5dTyHME
atD
erIiaTl ION WILLIAMS TEST BANK

Changes in the older adult respiratory system make older adults more susceptible to
infections that can be very serious. Although some infections are life threatening, most are
not.

DIF: Cognitive Level: Comprehension REF: p. 175|Table 13-1


OBJ: Theory #4 TOP: Physical Changes
KEY: Nursing Process Step: Implementation
MSC: NCLEX: Health Promotion and Maintenance: Prevention and Early Detection of
Disease

15. When the nurse is conducting a class for senior citizens at a local assisted living facility, to
enhance physical health, he encourages the older residents to engage in some form of
exercise for at least:
a. 1 hour every other day.
b. 10 minutes at a time several times a day.
c. 30 minutes a day, five times a week.
d. 1 hour every morning.
ANS: C
Exercise for as little as 30 minutes a day 5 days a week is beneficial.

DIF: Cognitive Level: Application REF: p. 177


OBJ: Clinical Practice #2 TOP: Physical Health Promotion Behaviors
KEY: Nursing Process Step: Implementation
MSC: NCLEX: Physiological Integrity: Physiological Adaptation

16. The nurse points out that there aNr eU m


R SaI nNyG m
T By. tChOs Ma b o u t older adults that are not true. The
statement that reflects the most accuracy about older adults is:
a. most old people live in nursing homes.
b. genetics is the main factor in longevity.
c. the optimistic, happy person generally lives longer.
d. most old people are isolated from their families.
ANS: C
Myths about old people include: most old people live in nursing homes, genetics is the main
factor in longevity, and most old people are isolated from their families. It is true that a
major contributor to longer life is that the optimistic, happy person generally lives longer.

DIF: Cognitive Level: Comprehension REF: p. 174 OBJ: Theory #3


TOP: Theories of Aging KEY: Nursing Process Step: Implementation
MSC: NCLEX: Psychosocial Integrity: Psychosocial Adaptation

17. When the 75-year-old patient tells the nurse that he has had a good and rewarding life and
has enjoyed every minute of it, the nurse is aware that the patient has attained the Eriksonian
stage of:
a. generativity.
b. autonomy.
c. integrity.
d. intimacy.

NURSINGTB.COM

Downloaded by: SUCCEEDGRADES | abbieclin@gmail.com


Distribution
Downloaded by Dallen Mae D.ofKadir
this document is illegal
(kadir.dallenmae.d.bcsi@gmail.com)
lOMoARcPSD|35346190

DEWITS FUNDAMENTALStC
uvO
iaN
.cC
om
EP- T
ThSe A
MNarD
ketSpK
l a IcL
eLtoSBFuO
y aRndNSUeR
ll S
yoI u
Nr G
Stu
5dTyHME
atD
erIiaTl ION WILLIAMS TEST BANK

ANS: C
Erik Erikson‘s psychosocial development theory includes the stage of ego integrity vs.
despair, which is characterized by reflection on one‘s life and one‘s achievements. A sense
of pride or despair is developed regarding the accomplishments in life that have been made
or were lost.

DIF: Cognitive Level: Comprehension REF: p. 178 OBJ: Theory #3


TOP: Theorists KEY: Nursing Process Step: Assessment
MSC: NCLEX: Health Promotion and Maintenance: Growth and Development

18. When a 68-year-old recent retiree confides in the health clinic nurse that he has felt
depressed and withdrawn since retirement, the nurse suggests that he:
a. talk with the primary care provider about antidepression medication.
b. arrange his day so that he is able to take a nap in the afternoon.
c. set a small goal for himself to be met every day.
d. eat three regular meals as he did when he was employed.
ANS: C
Setting a small goal to be met allows for direction in the day. Many recent retirees miss the
regulation of employment.

DIF: Cognitive Level: Application REF: p. 178 OBJ: Theory #3


TOP: Theorists KEY: Nursing Process Step: Assessment
MSC: NCLEX: Psychosocial Integrity: Psychosocial Adaptation

19. A nurse caring for an older adult suspects that he is experiencing depression. Which of the
following are true for the older aNdUuRl tS aI NndG TdBe .pCr Oe sMsi o n?
a. More than three million older adults experience depression.
b. If experiencing suicidal thoughts, the older adult more likely to act on them
successfully.
c. Depression is a risk factor for diabetes.
d. Difficult to treat with medications and counseling.
ANS: B
It is estimated that more than one million older adults experience depression, and older
adults who have suicidal thoughts are more likely to act on them successfully than younger
people with similar thoughts (Aziz and Steffans, 2013). Signs and symptoms of depression
in the older adults are listed in Box 13-2. Depression can often be treated successfully with
medications and counseling. Depression is a risk factor and possibly an early sign of
dementia (degeneration of brain tissue).

DIF: Cognitive Level: Comprehension REF: p. 173


OBJ: Clinical Practice #7 TOP: Health Concerns
KEY: Nursing Process Step: N/A MSC: NCLEX: N/A

20. A nurse working for an assisted living facility is reviewing the Healthy People Objectives
for Older Adults. Which of the following would she incorporate into the environment to
promote health for the older adult population?
a. Encourage staff management of chronic health conditions.
b. Discourage participation in sessions for ―Welcome to Medicare‖ benefit.

NURSINGTB.COM

Downloaded by: SUCCEEDGRADES | abbieclin@gmail.com


Distribution
Downloaded by Dallen Mae D.ofKadir
this document is illegal
(kadir.dallenmae.d.bcsi@gmail.com)
lOMoARcPSD|35346190

DEWITS FUNDAMENTALStC
uvO
iaN
.cC
om
EP- T
ThSe A
MNarD
ketSpK
l a IcL
eLtoSBFuO
y aRndNSUeR
ll S
yoI u
Nr G
Stu
5dTyHME
atD
erIiaTl ION WILLIAMS TEST BANK

c. Decrease opportunities for light, moderate, or vigorous physical activities.


d. Develop and implement strategies to prevent pressure ulcer formation.
ANS: D
Healthy People Objectives: Older Adult
• Increase the proportion of older adults who use the ―Welcome to Medicare‖ benefit.
• Increase the proportion of older adults who are up to date on a core set of clinical
preventive services.
• Increase the proportion of older adults with one or more chronic health conditions who
report confidence in managing their conditions.
• Increase the proportion of older adults who receive Diabetes Self-Management Benefits.
• Reduce the proportion of older adults who have moderate to severe functional limitations.
• Increase the proportion of older adults with reduced physical or cognitive function who
engage in light, moderate, or vigorous physical activities.
• Increase the proportion of the health care workforce with geriatric certification.
• Reduce the proportion of noninstitutionalized older adults with disabilities who have an
unmet need for long-term services and supports.
• Reduce the proportion of unpaid caregivers of older adults who report an unmet need for
caregiver support services.
• Reduce the rate of pressure ulcer-related hospitalizations among older adults.
• Reduce the rate of emergency department (ED) visits due to falls among older adults.
• Increase the number of states and the District of Columbia that collect and make publicly
available information on the characteristics of victims, perpetrators, and cases of elder
abuse, neglect, and exploitation. (Source:
https://www.healthypeople.gov/2020/topics-objectives/topic/older-adults)
NURSINGTB.COM
DIF: Cognitive Level: Analysis REF: p. 173
OBJ: Clinical Practice #8 TOP: Health Promotion
KEY: Nursing Process Step: N/A MSC: NCLEX: N/A

MULTIPLE RESPONSE

1. To promote cognitive health for a 79-year-old, the nursing student should encourage the
patient to do which of the following? (Select all that apply.)
a. Read the newspaper.
b. Put together puzzles.
c. Take naps when tired.
d. Write letters.
e. Toilet independently.
ANS: A, B, D
Mental stimulation such as these activities maintains active and healthy neural connections,
which promotes cognitive health.

DIF: Cognitive Level: Application REF: p. 178


OBJ: Clinical Practice #3 TOP: Cognitive Health Promotion Behaviors
KEY: Nursing Process Step: Implementation
MSC: NCLEX: Health Promotion and Maintenance: Prevention and Early Detection of
Disease

NURSINGTB.COM

Downloaded by: SUCCEEDGRADES | abbieclin@gmail.com


Distribution
Downloaded by Dallen Mae D.ofKadir
this document is illegal
(kadir.dallenmae.d.bcsi@gmail.com)
lOMoARcPSD|35346190

DEWITS FUNDAMENTALStC
uvO
iaN
.cC
om
EP- T
ThSe A
MNarD
ketSpK
l a IcL
eLtoSBFuO
y aRndNSUeR
ll S
yoI u
Nr G
Stu
5dTyHME
atD
erIiaTl ION WILLIAMS TEST BANK

2. A 76-year-old recently lost her spouse. They lived in a large house with a yard that they
managed well until her spouse‘s sudden death. She is in good health and stays active but is
becoming concerned about her ability to maintain the large house by herself. The nurse‘s
most helpful suggestion would be to: (Select all that apply.)
a. recommend living with her daughter who lives 2 hours away.
b. encourage her to investigate apartment rentals.
c. encourage her to check herself into a skilled nursing facility.
d. hire a gardener and a housekeeper to maintain the property.
e. encourage her to involve family in any decisions related to her living
arrangements.
ANS: B, E
Many older adults choose to rent an apartment when they cannot maintain a house and yard.
Including family members in important decisions keeps communication lines open. Family
members can offer support and recommendations to their parent(s), easing the
decision-making process.

DIF: Cognitive Level: Analysis REF: p. 178


OBJ: Clinical Practice #5
TOP: Psychosocial Health Promotion Behaviors
KEY: Nursing Process Step: Planning MSC: NCLEX: Psychosocial Integrity

3. An 82-year-old patient expresses concern of always being cold. Based on knowledge of the
physical changes associated with aging, the nurse could appropriately respond: (Select all
that apply.)
a. ―Wrap up warmly, because tNhU e RcSoIlNdGfeTeBl.iCnOgMis a result of decreased bone calcium
that occurs with aging.‖
b. ―Chronic constipation can lead to a sense of cold because of the reduced
peristalsis.‖
c. ―Subcutaneous fat is reduced as part of aging. This fat helps to keep you warm
decreases.‖
d. ―A low level of thyroid hormone causes you to feel cold.‖
e. ―Eat more protein foods to help your body generate heat.‖
ANS: C, D
Decreased subcutaneous fat and slowed thyroid production contribute to older people
feeling cold.

DIF: Cognitive Level: Application REF: p. 175|Table 13-1


OBJ: Theory #4 TOP: Physical Changes
KEY: Nursing Process Step: N/A MSC: NCLEX: Physiological Integrity

4. The nurse points out elements in our culture that support longevity in older people, which
include: (Select all that apply.)
a. reducing stress from regular exercise on the body in younger years.
b. being actively involved in their own health care.
c. taking of antibiotics through their lifetime.
d. increasing education about health matters.
e. avoiding the eating of red meat.

NURSINGTB.COM

Downloaded by: SUCCEEDGRADES | abbieclin@gmail.com


Distribution
Downloaded by Dallen Mae D.ofKadir
this document is illegal
(kadir.dallenmae.d.bcsi@gmail.com)
lOMoARcPSD|35346190

DEWITS FUNDAMENTALStC
uvO
iaN
.cC
om
EP- T
ThSe A
MNarD
ketSpK
l a IcL
eLtoSBFuO
y aRndNSUeR
ll S
yoI u
Nr G
Stu
5dTyHME
atD
erIiaTl ION WILLIAMS TEST BANK

ANS: B, D
Active participation in one‘s own health care, being better educated about illness, and illness
prevention are elements in attaining a longer life span.

DIF: Cognitive Level: Application REF: p. 175|Table 13-1


OBJ: Theory #3 TOP: Longevity KEY: Nursing Process Step: Implementation
MSC: NCLEX: Health Promotion and Maintenance: Prevention and Early Detection of
Disease

COMPLETION

1. An 85-year-old has been increasingly confused and disoriented to place and time over the
last several months. He also has difficulty remembering what he ate, who visited, and where
the recreation room is. This behavior is indicative of .

ANS:
dementia

Confusion, memory loss, and disordered thinking are early signs of dementia. Causes of
dementia include malnutrition, medications, mini strokes, and Alzheimer disease.

DIF: Cognitive Level: Analysis REF: p. 176


OBJ: Clinical Practice #4 TOP: Physical Health Concerns
KEY: Nursing Process Step: Assessment
MSC: NCLEX: Physiological INnU
teRgSrIiN
tyG: TPBh.yCsOi oMlogical Adaptation

2. A 67-year-old submitted a job application at a local gas station. The station manager said,
―Aren‘t you retired? I can‘t hire you. You‘re too old to be working!‖ This is an example of
.

ANS:
ageism

Ageism is defined as any discrimination related to age, including job discrimination.

DIF: Cognitive Level: Analysis REF: p. 178 OBJ: Theory #2


TOP: Employment and Retirement KEY: Nursing Process Step: N/A
MSC: NCLEX: N/A

3. The home health nurse assesses the older patients in her case load for signs of abuse. The
nurse is aware that the primary incidence of abuse is .

ANS:
neglect

In patients over the age of 80 who suffer abuse, the primary type of abuse is neglect.

DIF: Cognitive Level: Knowledge REF: p. 179 OBJ: Theory #4

NURSINGTB.COM

Downloaded by: SUCCEEDGRADES | abbieclin@gmail.com


Distribution
Downloaded by Dallen Mae D.ofKadir
this document is illegal
(kadir.dallenmae.d.bcsi@gmail.com)
lOMoARcPSD|35346190

DEWITS FUNDAMENTALStC
uvO
iaN
.cC
om
EP- T
ThSe A
MNarD
ketSpK
l a IcL
eLtoSBFuO
y aRndNSUeR
ll S
yoI u
Nr G
Stu
5dTyHME
atD
erIiaTl ION WILLIAMS TEST BANK

TOP: Elder Abuse KEY: Nursing Process Step: Assessment


MSC: NCLEX: Physiological Integrity: Basic Care and Comfort

NURSINGTB.COM

NURSINGTB.COM

Downloaded by: SUCCEEDGRADES | abbieclin@gmail.com


Distribution
Downloaded by Dallen Mae D.ofKadir
this document is illegal
(kadir.dallenmae.d.bcsi@gmail.com)
lOMoARcPSD|35346190

DEWITS FUNDAMENTALStC
uvO
iaN
.cC
om
EP- T
ThSe A
MNarD
ketSpK
l a IcL
eLtoSBFuO
y aRndNSUeR
ll S
yoI u
Nr G
Stu
5dTyHME
atD
erIiaTl ION WILLIAMS TEST BANK

Chapter 14: Cultural and Spiritual Aspects of Patient Care


Williams: deWit's Fundamental Concepts and Skills for Nursing, 8th Edition

MULTIPLE CHOICE

1. A nurse‘s neighbor confides that she has been treating a health problem through a faith healer
in her religion but is worried because the condition seems to be worsening. She asks the nurse
what she should do. The nurse‘s best response would be:
a. ―Faith healing, although frequently effective, makes a disease more difficult or
even impossible to treat with modern medicine.‖
b. ―Many people use medical treatment along with faith healing. Is there anything in
your religion that forbids medical treatment?‖
c. ―Faith healing may take longer than medical treatment to work. Are you doing
exactly as your healer is directing you?‖
d. ―You should see a health care provider or your faith healer, but not both.‖
ANS: B
A person‘s health care choices are influenced by religious beliefs and culture. Many people
use religious or folk medicine in addition to modern medicine.

DIF: Cognitive Level: Application REF: p. 192|Table 14-3


OBJ: Theory #1 TOP: Religious Influence in Health Care
KEY: Nursing Process Step: Implementation
MSC: NCLEX: Psychosocial Integrity: Coping and Adaptation

2. The most culturally sensitive and useful question the admitting nurse should ask the Jewish
NURSINGTB.COM
patient would be:
a. ―Do you follow any dietary restrictions or will you eat hospital food as it is
prepared here?‖
b. ―Are there any medications or treatments your religion does not allow in your
medical treatment?‖
c. ―Would you like me to notify your rabbi so that he can make visits to you during
your stay?‖
d. ―Tell me about any religious practices you observe that we need to incorporate into
your care.‖
ANS: D
The response to the general question requesting any special concerns would include
information relative to food, spiritual practices, and rabbinical visitation. The other questions
do have importance, but the general question will allow the nurse to hear what is most
significant to the patient.

DIF: Cognitive Level: Application REF: p. 192|Table 14-3


OBJ: Theory #7 TOP: Religion and Health Care
KEY: Nursing Process Step: Assessment
MSC: NCLEX: Physiological Integrity: Basic Care and Comfort

3. A nurse referring to an older Asian man says, ―He probably uses acupuncture and believes in
yin and yang.‖ This remark is an example of:
a. stereotyping of the patient.

NURSINGTB.COM

Downloaded by: SUCCEEDGRADES | abbieclin@gmail.com


Distribution
Downloaded by Dallen Mae D.ofKadir
this document is illegal
(kadir.dallenmae.d.bcsi@gmail.com)
lOMoARcPSD|35346190

DEWITS FUNDAMENTALStC
uvO
iaN
.cC
om
EP- T
ThSe A
MNarD
ketSpK
l a IcL
eLtoSBFuO
y aRndNSUeR
ll S
yoI u
Nr G
Stu
5dTyHME
atD
erIiaTl ION WILLIAMS TEST BANK

b. ethnocentrism of the patient.


c. cultural sensitivity on the part of the nurse.
d. a worldview of the nurse.
ANS: A
Stereotyping occurs when common characteristics of a cultural group are applied to an
individual with no attention to the individual‘s characteristics.

DIF: Cognitive Level: Comprehension REF: p. 190 OBJ: Clinical Practice #5


TOP: Stereotyping KEY: Nursing Process Step: Assessment
MSC: NCLEX: Physiological Integrity: Basic Care and Comfort

4. A Roman Catholic patient going to surgery for an emergency cesarean delivery is afraid the
baby may not survive, and because she is Roman Catholic, she asks you to be sure that the
baby is baptized when it is born. Your response to her should be:
a. ―There won‘t be time to baptize the baby in the operating room.‖
b. ―I will call your priest to come in, but if he is unable to be there, I‘ll be sure the
baby is baptized.‖
c. ―Baptisms are not usually done in the hospital, and certainly not in the operating
room, but I will tell the charge nurse your concerns.‖
d. ―You need to think positively. Both you and the baby are going to be just fine.‖
ANS: B
Baptism is very important to practicing Roman Catholic patients, and even stillborn is
baptized. Baptism should be performed by the priest, but any believer can baptize in an
emergency.

DIF: Cognitive Level: Application NURSINRGETFB:.CpO.M186|Box 14-1


OBJ: Theory #6 TOP: Religious Characteristics
KEY: Nursing Process Step: Implementation
MSC: NCLEX: Psychosocial Integrity: Coping and Adaptation

5. A 76-year-old Hispanic woman is in the skilled nursing facility where she is to participate in a
rehabilitation program following a hip replacement. She is alert, oriented, and cooperative but
speaks only Spanish; her adult children interpret for her when they are present. The nurse
plans the most effective way to communicate with this patient is to:
a. communicate with the patient when the children are present and can translate.
b. arrange to have one of the children present at all times.
c. create a translation guide with commonly used Spanish and English words and
phrases.
d. call on the facility translator for every interaction with the resident.
ANS: C
A simple translation guide can be used with a cooperative patient in this nonacute setting. The
other solutions are unworkable and unrealistic.

DIF: Cognitive Level: Application REF: p. 191 OBJ: Clinical Practice #3


TOP: Communication KEY: Nursing Process Step: Implementation
MSC: NCLEX: Physiological Integrity: Basic Care and Comfort

6. A person who believes and practices the Christian Science religion is most likely to:
a. seek the latest technological medical and surgical care available.

NURSINGTB.COM

Downloaded by: SUCCEEDGRADES | abbieclin@gmail.com


Distribution
Downloaded by Dallen Mae D.ofKadir
this document is illegal
(kadir.dallenmae.d.bcsi@gmail.com)
lOMoARcPSD|35346190

DEWITS FUNDAMENTALStC
uvO
iaN
.cC
om
EP- T
ThSe A
MNarD
ketSpK
l a IcL
eLtoSBFuO
y aRndNSUeR
ll S
yoI u
Nr G
Stu
5dTyHME
atD
erIiaTl ION WILLIAMS TEST BANK

b. use medicinal herbs and drugs rather than surgery to treat illness and disease.
c. use spiritual practices and altered thinking to correct the causes of illness and
disease.
d. require the dead to be buried within 24 hours of the death.
ANS: C
A person of the Christian Science belief does not normally seek traditional health care and
uses prayer for healing.

DIF: Cognitive Level: Comprehension REF: p. 186|Table 14-1


OBJ: Theory #6 TOP: Religious Characteristics KEY: Nursing Process Step: N/A
MSC: NCLEX: N/A

7. A woman who is part of a traditional Muslim family is hospitalized after an accident. It is


most important for the nurse to include in the care to:
a. provide privacy when the priest comes to administer the sacrament of the sick.
b. respect her need for modesty in keeping her body covered.
c. keep her hair covered at all times.
d. refrain from touching the soles of her feet.

ANS: B
Traditional Muslim women are extremely modest and keep their bodies covered.

DIF: Cognitive Level: Comprehension REF: p. 187|Box 14-2


OBJ: Theory #6 TOP: Religious Characteristics
KEY: Nursing Process Step: Implementation
MSC: NCLEX: Psychosocial Integrity: Coping and Adaptation
NURSINGTB.COM
8. The nurse is aware that when an Orthodox Jewish family has a baby boy, it is expected that
the:
a. baby will be circumcised by the health care provider as soon as possible after the
birth.
b. husband of the woman will provide the physical care she needs.
c. naming of the baby will be done by the mother before she leaves the hospital.
d. baby will be circumcised on the eighth day of life when he is named.
ANS: D
Male Orthodox Jewish children are named and circumcised by the mohel on the eighth day of
life.

DIF: Cognitive Level: Comprehension REF: p. 192|Table 14-3


OBJ: Theory #6 TOP: Religious Practices
KEY: Nursing Process Step: Planning
MSC: NCLEX: Psychosocial Integrity: Psychosocial Adaptation

9. A single African American working mother of three children who is in a state of poverty often
misses appointments for the children‘s immunizations or well child checkups. The nurse
understands that the likely reason for this could be that she:
a. is more concerned with surviving day-to-day problems than with taking a child to a
clinic when there is no obvious illness.
b. would rather enjoy her time off in leisure activities than taking the children to a
clinic.

NURSINGTB.COM

Downloaded by: SUCCEEDGRADES | abbieclin@gmail.com


Distribution
Downloaded by Dallen Mae D.ofKadir
this document is illegal
(kadir.dallenmae.d.bcsi@gmail.com)
lOMoARcPSD|35346190

DEWITS FUNDAMENTALStC
uvO
iaN
.cC
om
EP- T
ThSe A
MNarD
ketSpK
l a IcL
eLtoSBFuO
y aRndNSUeR
ll S
yoI u
Nr G
Stu
5dTyHME
atD
erIiaTl ION WILLIAMS TEST BANK

c. is from a culture that values nontraditional medical care from curanderos and faith
healers.
d. belongs to a religious group that is opposed to immunizations and medical
treatment.
ANS: A
People who live in poverty are often focused on day-to-day living. Preventative medicine is
future oriented.

DIF: Cognitive Level: Application REF: p. 184 OBJ: Theory #2


TOP: Low Income Families KEY: Nursing Process Step: Assessment
MSC: NCLEX: Psychosocial Integrity: Psychosocial Adaptation

10. The nurse is aware that Muslims and Orthodox Jews have similar dietary restrictions in that
both groups require that their followers:
a. remain mainly vegetarians.
b. avoid milk and meat at the same meal.
c. eat only meat that has been slaughtered according to their religious law.
d. avoid shellfish.

ANS: C
Muslim and Jewish followers require food to be prepared according to religious law. In the
Jewish faith, it is called kosher, which also requires the separation of milk and meat.

DIF: Cognitive Level: Comprehension REF: p. 189|p. 192|Table 14-2|Table 14-3


OBJ: Theory #4 TOP: Dietary Restrictions
KEY: Nursing Process Step: Planning
MSC: NCLEX: Psychosocial IntegriNtyU: RPSsIyNcho
G TsBo.cCiaOlM
Adaptation

11. A resident in a skilled nursing facility indicates that she is an agnostic and is afraid of what
will happen to her when she dies. An appropriate nursing intervention for this patient is to:
a. request the health care provider to order an anxiety reducing medication.
b. ask the patient whether she would like to talk to the facility social worker or
chaplain to address her spiritual distress.
c. offer reassurance to the patient that God will understand her confusion.
d. share your religious belief with the patient.
ANS: B
Exploring spiritual (meaning of life and death) issues with a social worker or spiritual care
person is appropriate to address spiritual distress as demonstrated by the patient‘s anxiety over
her impending death.

DIF: Cognitive Level: Application REF: p. 195 OBJ: Clinical Practice #4


TOP: Spiritual Distress KEY: Nursing Process Step: Implementation
MSC: NCLEX: Psychosocial Integrity: Coping and Adaptation

12. The nurse is sensitive to the fact that Hispanics believe that some foods and fluids have ―hot
and cold‖ properties that affect their health by:
a. destroying the pathogen causing the disease.
b. restoring equilibrium to the body.
c. strengthening the mind to dispel an evil spirit.
d. ensuring healing.

NURSINGTB.COM

Downloaded by: SUCCEEDGRADES | abbieclin@gmail.com


Distribution
Downloaded by Dallen Mae D.ofKadir
this document is illegal
(kadir.dallenmae.d.bcsi@gmail.com)
lOMoARcPSD|35346190

DEWITS FUNDAMENTALStC
uvO
iaN
.cC
om
EP- T
ThSe A
MNarD
ketSpK
l a IcL
eLtoSBFuO
y aRndNSUeR
ll S
yoI u
Nr G
Stu
5dTyHME
atD
erIiaTl ION WILLIAMS TEST BANK

ANS: B
Hispanics believe that certain foods through their hot or cold properties can restore the body‘s
equilibrium.

DIF: Cognitive Level: Comprehension REF: p. 192|Table 14-3


OBJ: Theory #6 TOP: Nutrition and Religion
KEY: Nursing Process Step: Planning
MSC: NCLEX: Psychosocial Integrity: Coping and Adaptation

13. A patient who is Native American and visiting a health care clinic for the first time keeps his
eyes on the floor and seldom makes eye contact with the nurse. The nurse would be correct in
thinking that the patient:
a. is depressed and concerned about his health status.
b. would be more comfortable with an elder from his tribe present.
c. is uncomfortable talking about health issues with a woman.
d. finds direct or sustained eye contact rude or disrespectful.

ANS: D
Native Americans do not usually sustain eye contact in the same way as European Americans
do because they consider it rude.

DIF: Cognitive Level: Comprehension REF: p. 190 OBJ: Theory #5


TOP: Cultural Behaviors KEY: Nursing Process Step: Assessment
MSC: NCLEX: Psychosocial Integrity: Coping and Adaptation

14. When a devout Muslim dies, the nurse makes arrangements to:
a. notify a family member to come b at he t he patient.
N U R S I N G T B .C O M
b. extinguish all light and leave the room in darkness.
c. cover the body with the patient‘s prayer rug.
d. place an open Koran over the patient‘s heart.

ANS: A
A family member must bathe the deceased person before anybody else touches the body.

DIF: Cognitive Level: Application REF: p. 187|Table 14-2


OBJ: Theory #6 TOP: Muslim Death
KEY: Nursing Process Step: Implementation
MSC: NCLEX: Psychosocial Integrity: Coping and Adaptation

15. A nurse who practices in a culturally competent manner is one who is able to:
a. work in a large facility where there are a variety of diverse cultures.
b. assist a patient from another culture to change and adapt to the medical culture.
c. look at the world through the eyes of the culturally diverse patient.
d. recognize which cultural traits are harmful in the health care culture.

ANS: C
Cultural competence is attained when the nurse makes a conscious attempt to learn about
people of diverse cultures and look at the world through their perspective.

DIF: Cognitive Level: Comprehension REF: p. 189 OBJ: Clinical Practice #2


TOP: Transcultural Competence KEY: Nursing Process Step: N/A
MSC: NCLEX: N/A

NURSINGTB.COM

Downloaded by: SUCCEEDGRADES | abbieclin@gmail.com


Distribution
Downloaded by Dallen Mae D.ofKadir
this document is illegal
(kadir.dallenmae.d.bcsi@gmail.com)
lOMoARcPSD|35346190

DEWITS FUNDAMENTALStC
uvO
iaN
.cC
om
EP- T
ThSe A
MNarD
ketSpK
l a IcL
eLtoSBFuO
y aRndNSUeR
ll S
yoI u
Nr G
Stu
5dTyHME
atD
erIiaTl ION WILLIAMS TEST BANK

16. A young African American man is admitted to the hospital in sickle cell crisis. He is
unmarried and lives with an extended family headed by his grandmother. The nurse takes into
consideration that this type of family arrangement is:
a. egalitarian.
b. patriarchal.
c. matriarchal.
d. transcultural.

ANS: C
Matriarchal families are headed by women (mother) and are commonly seen in African
American families.

DIF: Cognitive Level: Knowledge REF: p. 191 OBJ: Clinical Practice #2


TOP: Family Types KEY: Nursing Process Step: Planning
MSC: NCLEX: Psychosocial Integrity: Coping and Adaptation

17. The nurse must be alert to nonverbal expressions to assess pain in patients from a(n):
a. Hispanic background.
b. Pacific Islander background.
c. European background.
d. African American background.
ANS: B
Pacific Islanders are reluctant to express emotion to others and are stoic about pain.

DIF: Cognitive Level: Comprehension REF: p. 192|Table 14-3


OBJ: Theory #4 TOP: CulturaN
l SUeRnSsIiN
tivGiTtyB. C O M
KEY: Nursing Process Step: Assessment
MSC: NCLEX: Physiological Integrity: Basic Care and Comfort

18. A desired outcome for a patient with a nursing diagnosis of ―spiritual distress‖ is that the
patient expresses:
a. acceptance that she is not being punished by God with illness.
b. concern about the meaning and importance of her life.
c. continued anger at God for afflicting her with the illness.
d. refusal to participate in religious rituals that have no meaning for her now.

ANS: A
Spiritual distress is often caused by a belief that illness is a punishment for wrongdoing or sin.

DIF: Cognitive Level: Application REF: p. 195 OBJ: Clinical Practice #4


TOP: Spiritual Distress KEY: Nursing Process Step: Planning
MSC: NCLEX: Psychosocial Integrity: Coping and Adaptation

19. The nurse understands that when an Asian patient refers to yin, the patient is referring to a
balancing force that is represented by:
a. light.
b. a masculine force.
c. cold and feminine elements.
d. harmony.

NURSINGTB.COM

Downloaded by: SUCCEEDGRADES | abbieclin@gmail.com


Distribution
Downloaded by Dallen Mae D.ofKadir
this document is illegal
(kadir.dallenmae.d.bcsi@gmail.com)
lOMoARcPSD|35346190

DEWITS FUNDAMENTALStC
uvO
iaN
.cC
om
EP- T
ThSe A
MNarD
ketSpK
l a IcL
eLtoSBFuO
y aRndNSUeR
ll S
yoI u
Nr G
Stu
5dTyHME
atD
erIiaTl ION WILLIAMS TEST BANK

ANS: C
Yin is the balancing force to yang that is represented by elements that are dark, cold, and
feminine.

DIF: Cognitive Level: Comprehension REF: p. 192|Table 14-3


OBJ: Theory #7 TOP: Nutritional Practices
KEY: Nursing Process Step: Planning
MSC: NCLEX: Physiological Integrity: Basic Care and Comfort

20. The nurse understands that acupuncture and acupressure are alternative therapies based on:
a. healing through spiritual practices.
b. concepts of hot and cold and balancing those against the illness.
c. the power of suggestion and autohypnosis.
d. interrupting or changing the flow of energy fields in the body.
ANS: D
Acupressure, massage, and acupuncture all are based on theories of energy flow in the body
that can be manipulated to affect health.

DIF: Cognitive Level: Comprehension REF: p. 192|Table 14-3


OBJ: Theory #5 TOP: Alternative Treatments
KEY: Nursing Process Step: Planning
MSC: NCLEX: Psychosocial Integrity: Coping and Adaptation

21. A nurse is caring for a Catholic patient who has just given birth to a nonviable fetus. When
considering the religion of this patient, the nurse should:
a. discourage baptism since the fetus is nonviable.
b. baptize the fetus if a priest is nNoUtRim
SIm
NGedTiBa.tCelOyMavailable.
c. encourage the mother to consider tubal ligation.
d. discuss hormonal contraception with the mother.
ANS: B
Within the Catholic religion, infants must be baptized soon after birth because of the belief
that babies not baptized will not go to Heaven. Even aborted fetuses must be baptized. If a
priest is not immediately available, the nurse may baptize by pouring holy water on the head
and saying, ―I baptize you in the name of the Father, of the Son, and of the Holy Spirit.‖

DIF: Cognitive Level: Application REF: p. 186|Box 14-1


OBJ: Clinical Practice #6 TOP: Religious Influence in Health Care
KEY: Nursing Process Step: Implementation
MSC: NCLEX: Psychosocial Integrity: Coping and Adaptation

22. A nurse caring for a patient who is a practicing Jehovah‘s Witness and who has had surgery
confirms that is on hand to be infused.
a. frozen packed cells.
b. Dextran.
c. 5% glucose in water.
d. normal saline.
ANS: B
Jehovah‘s Witnesses are not permitted to take blood into their bodies, but blood expanders
such as Dextran can be infused.

NURSINGTB.COM

Downloaded by: SUCCEEDGRADES | abbieclin@gmail.com


Distribution
Downloaded by Dallen Mae D.ofKadir
this document is illegal
(kadir.dallenmae.d.bcsi@gmail.com)
lOMoARcPSD|35346190

DEWITS FUNDAMENTALStC
uvO
iaN
.cC
om
EP- T
ThSe A
MNarD
ketSpK
l a IcL
eLtoSBFuO
y aRndNSUeR
ll S
yoI u
Nr G
Stu
5dTyHME
atD
erIiaTl ION WILLIAMS TEST BANK

DIF: Cognitive Level: Application REF: p. 186|Table 14-1


OBJ: Clinical Practice #1 TOP: Religious Influence in Health Care
KEY: Nursing Process Step: Planning
MSC: NCLEX: Physiological Integrity: Basic Care and Comfort

23. A practicing member of the Church of Jesus Christ of Latter Day Saints is being prepared for
surgery on his left knee. The nurse should remove all the personal possessions with the
exception of:
a. the wedding ring.
b. the religious medallion around the neck.
c. undergarments.
d. linen socks.

ANS: C
Members of the Church of Jesus Christ of Latter Day Saints may wear sacred undergarments
that should only be removed in an emergency. If removed, the undergarments should be put
back on as soon as possible.

DIF: Cognitive Level: Application REF: p. 186|Table 14-1


OBJ: Clinical Practice #1 TOP: Religious Influence in Health Care
KEY: Nursing Process Step: Implementation
MSC: NCLEX: Psychosocial Integrity: Coping and Adaptation

24. A nurse is caring for a patient of the Muslim faith. The nurse would send the food tray back if
it contains:
a. pork chops and sweet potatoes.
b. beef patty with mushroom sauNceU.RSINGTB.COM
c. liver and mashed potatoes.
d. crab cakes and rice with almonds.

ANS: A
Practicing Muslims do not consume pork or alcoholic beverages. They also forbid cremation.

DIF: Cognitive Level: Application REF: p. 187|Box 14-2


OBJ: Clinical Practice #1 TOP: Religious Influence in Health Care
KEY: Nursing Process Step: Implementation
MSC: NCLEX: Psychosocial Integrity: Coping and Adaptation

25. A nurse is caring for a postpartum patient who adheres to the beliefs of Orthodox Judaism.
The question that would be the most culturally sensitive would be:
a. ―Would you like crab cakes for your lunch?‖
b. ―Would you like your husband to give you a back rub?‖
c. ―Has your husband decided on a name for your new baby?‖
d. ―Will your doctor circumcise your baby tomorrow?‖
ANS: C
Practicing members of Orthodox Judaism name their male children 8 days after being born,
when circumcision is performed. Babies are named by the father. The Orthodox Jewish man
cannot touch the woman until after the ritual bath (mikvah). Jewish persons are discouraged
from eating shellfish.

NURSINGTB.COM

Downloaded by: SUCCEEDGRADES | abbieclin@gmail.com


Distribution
Downloaded by Dallen Mae D.ofKadir
this document is illegal
(kadir.dallenmae.d.bcsi@gmail.com)
lOMoARcPSD|35346190

DEWITS FUNDAMENTALStC
uvO
iaN
.cC
om
EP- T
ThSe A
MNarD
ketSpK
l a IcL
eLtoSBFuO
y aRndNSUeR
ll S
yoI u
Nr G
Stu
5dTyHME
atD
erIiaTl ION WILLIAMS TEST BANK

DIF: Cognitive Level: Application REF: p. 192|Box 14-3


OBJ: Clinical Practice #1 TOP: Religious Influence in Health Care
KEY: Nursing Process Step: Implementation
MSC: NCLEX: Psychosocial Integrity: Coping and Adaptation

26. The nurse supervisor becomes concerned when observing the nurse caring for an Orthodox
Jewish patient preparing to trim the patient‘s beard with a(n):
a. pair of scissors.
b. electric razor.
c. razor blade.
d. barber‘s straight razor.
ANS: C
Practicing members of Orthodox Judaism view the beard as a sign of holiness, and no blade
must touch the skin. Scissors or an electric razor may be used.

DIF: Cognitive Level: Application REF: p. 192|Box 14-3


OBJ: Clinical Practice #1 TOP: Religious Influence in Health Care
KEY: Nursing Process Step: Implementation
MSC: NCLEX: Psychosocial Integrity

27. A young Hindu woman who is part of a traditional Hindu family is in a coma and is going to
be transferred to the ICU. The family member whom the nurse should consult about this
health matter would be the:
a. eldest woman in the family.
b. family council.
c. patient‘s husband.
d. the older siblings. NURSINGTB.COM

ANS: A
Traditional Hindu families consider the eldest woman in the family to be the authority on
health and healing matters. She should be consulted and included in any patient education.

DIF: Cognitive Level: Application REF: p. 188 OBJ: Clinical Practice #1


TOP: Religious Influence in Health Care KEY: Nursing Process Step: Implementation
MSC: NCLEX: Psychosocial Integrity: Coping and Adaptation

28. An example of an ethnocentric remark would be:


a. ―I think Catholics should have as many children as they want.‖
b. ―I don‘t see why Muslims can‘t pray in church like we Protestants do.‖
c. ―You have to admire the Asian immigrants for keeping all their eastern traditions.‖
d. ―It seems that Hispanics have taken over this country.‖
ANS: B
Ethnocentrism is the tendency of human beings to think that their way of thinking, believing,
and doing things is the only way or the only right way. Beliefs that greatly differ from one‘s
own are seen as strange and are therefore considered wrong.

DIF: Cognitive Level: Comprehension REF: p. 190 OBJ: Clinical Practice #5


TOP: Ethnocentrism KEY: Nursing Process Step: N/A
MSC: NCLEX: N/A

NURSINGTB.COM

Downloaded by: SUCCEEDGRADES | abbieclin@gmail.com


Distribution
Downloaded by Dallen Mae D.ofKadir
this document is illegal
(kadir.dallenmae.d.bcsi@gmail.com)
lOMoARcPSD|35346190

DEWITS FUNDAMENTALStC
uvO
iaN
.cC
om
EP- T
ThSe A
MNarD
ketSpK
l a IcL
eLtoSBFuO
y aRndNSUeR
ll S
yoI u
Nr G
Stu
5dTyHME
atD
erIiaTl ION WILLIAMS TEST BANK

29. A nursing instructor is educating a group of nursing students about cultural values practiced
by Arab Americans. The nursing instructor would recognize the need for further patient
education if a nursing student states, ―I will:
a. offer Arab American patient‘s items using my left hand.‖
b. make sure all females are cared for by females.‖
c. refrain from sitting with the sole of my shoe visible.‖
d. refrain from offering Arab American patient‘s pork.‖

ANS: A
Arab Americans do not eat pork or drink alcohol. It is considered rude to pass things with the
left hand because it is considered ―unclean.‖ It is considered rude to sit with the sole of the
shoe within view of someone. Arab women are uncomfortable with care performed by males.

DIF: Cognitive Level: Comprehension REF: p. 192|Table 14-3


OBJ: Clinical Practice #1 TOP: Religious Influence in Health Care
KEY: Nursing Process Step: Implementation
MSC: NCLEX: Psychosocial Integrity: Psychosocial Adaptation

30. A nurse caring for a newborn of Eastern European Jewish ancestry is aware that this patient is
predisposed to:
a. Tay Sachs disease.
b. sickle cell anemia.
c. diabetes mellitus.
d. sickle cell trait.
ANS: A
People of Eastern European Jewish ancestry may carry the trait for a fatal neurological
disorder of infancy called Tay S a cNhUs RdSi sI Ne aGsTe B. .COM

DIF: Cognitive Level: Comprehension REF: p. 194 OBJ: Clinical Practice #1


TOP: Religious Influence in Health Care KEY: Nursing Process Step: Implementation
MSC: NCLEX: Health Promotion and Maintenance: Prevention and Early Detection of Disease

31. A home health nurse caring for a patient of American Indian ancestry will be particularly alert
to symptoms of because of a genetic predisposition to that illness.
a. diabetes mellitus
b. Tay Sachs disease
c. sickle cell anemia
d. sickle cell trait
ANS: A
Diabetes is more common among the Hispanic and American Indian populations because of a
genetic susceptibility.

DIF: Cognitive Level: Knowledge REF: p. 194 OBJ: Clinical Practice #1


TOP: Religious Influence in Health Care KEY: Nursing Process Step: Implementation
MSC: NCLEX: Physiological Integrity: Basic Care and Comfort

32. A nurse caring for an African who is visiting the United States knows the following about
transcultural nursing to be true:
a. Nursing care based on a patient‘s cultural and spiritual needs, improves outcomes.
b. Nursing care based on scientific principles and evidence-based care improves

NURSINGTB.COM

Downloaded by: SUCCEEDGRADES | abbieclin@gmail.com


Distribution
Downloaded by Dallen Mae D.ofKadir
this document is illegal
(kadir.dallenmae.d.bcsi@gmail.com)
lOMoARcPSD|35346190

DEWITS FUNDAMENTALStC
uvO
iaN
.cC
om
EP- T
ThSe A
MNarD
ketSpK
l a IcL
eLtoSBFuO
y aRndNSUeR
ll S
yoI u
Nr G
Stu
5dTyHME
atD
erIiaTl ION WILLIAMS TEST BANK

patient outcomes.
c. Nursing care based on the patients‘ needs and wishes improves therapeutic
response.
d. Nursing care based on the patient family‘s needs and wishes improves outcomes.
ANS: A
Research has repeatedly demonstrated that nursing care based on patients‘ unique cultural and
spiritual needs significantly improves patients‘ outcomes and overall quality of care. This
understanding has given a rise to the discipline of transcultural nursing.

DIF: Cognitive Level: Comprehension REF: p. 186 OBJ: Theory #1


TOP: Transcultural Care KEY: Nursing Process Step: N/A
MSC: NCLEX: Patient-Centered Care

33. A nurse is caring for a prisoner who is hospitalized for injuries received during a terrorist
attack at a nightclub in which several people were killed. While the hospital staff‘s safety is
well managed by the police, to remain grounded in providing care to this patient, an important
principle for the nurse to remember about transcultural nursing is:
a. human caring is not a universal aspect of every culture.
b. all cultures provide what every human needs in order to grow, remain well, avoid
illness, and survive or face death.
c. nurses are expected to provide care that is congruent with their own cultural needs.
d. based on choices this patient made according to his spiritual beliefs, he doesn‘t
deserve quality health care.
ANS: B
Leininger‘s theory postulates that human caring is a universal aspect of every culture. As
such, it is present in all cultures alNthUoRuSgIhNG
itTm
B.aCyObM
e expressed in culturally unique ways. It
provides what every human needs in order to grow, remain well, avoid illness, and survive or
face death.

DIF: Cognitive Level: Comprehension REF: p. 184 OBJ: Theory #1


TOP: Transcultural Care KEY: Nursing Process Step: N/A
MSC: NCLEX: Patient-Centered Care

34. Karen is a recent graduate. She is planning to go to India with her church group as a nurse on
a medical mission trip. She will be working in a church funded clinic. Which of the following
shows willingness for the development of cultural competence?
a. She is planning to attend an Indian Food cooking class.
b. She is learning simple phrases in the Indian language so she will be able to
communicate with her patients.
c. She is reviewing her materials from school on public health.
d. She is obtaining all of the necessary travel documents and immunizations.

ANS: C

NURSINGTB.COM

Downloaded by: SUCCEEDGRADES | abbieclin@gmail.com


Distribution
Downloaded by Dallen Mae D.ofKadir
this document is illegal
(kadir.dallenmae.d.bcsi@gmail.com)
lOMoARcPSD|35346190

DEWITS FUNDAMENTALStC
uvO
iaN
.cC
om
EP- T
ThSe A
MNarD
ketSpK
l a IcL
eLtoSBFuO
y aRndNSUeR
ll S
yoI u
Nr G
Stu
5dTyHME
atD
erIiaTl ION WILLIAMS TEST BANK

Nurses must develop cultural competence in order to deliver care that meets standards of
culturally congruent care. Developing cultural competence is a lifelong process. It begins with
the development of cultural awareness and cultural sensitivity. Cultural awareness involves
developing understanding that health is expressed differently across cultures, and that culture
influences an individual‘s response to health, illness, disease, and death. Cultural sensitivity is
the ability to engage and communicate with an individual from another culture in a manner
that demonstrates respect for their cultural norms and beliefs.

DIF: Cognitive Level: Analysis REF: p. 189 OBJ: Theory #1


TOP: Cultural Competence KEY: Nursing Process Step: N/A
MSC: NCLEX: Patient-Centered Care

35. Laura is a nurse who is not answering the requests of a patient based on the fact that she is
scheduled for an abortion. This is an example of:
a. cultural relativism.
b. cultural imposition.
c. cultural blindness.
d. racism.
ANS: D
Prejudice can be defined as an emotional manifestation of negative stereotypes and deeply
held beliefs about a group. Racism is a form of prejudice that takes place when individuals,
groups, and/or institution exercise power against groups that are judged to be inferior.

DIF: Cognitive Level: Analysis REF: p. 190 OBJ: Theory #7


TOP: Cultural Competence KEY: Nursing Process Step: N/A
MSC: NCLEX: Patient-Centered Care
NURSINGTB.COM
36. David is a nurse who is caring for a patient who is in the process of gender reassignment. He
has strong beliefs against this and discusses with the patient his strong beliefs. This is an
example of which of the following?
a. Cultural relativism
b. Cultural imposition
c. Cultural blindness
d. Racism
ANS: B
Ethnocentrism is the belief that one‘s own cultural group determines the standards by which
other groups‘ behavior should be judged. Cultural blindness is an inability to recognize the
differences between one‘s own cultural beliefs, values, and practices and those of another
culture. Cultural imposition is the act of imposing own cultural beliefs, values, and practices
on individuals and groups from another culture. It is different from cultural relativism, which
is an ability to recognize that each cultural group has its own set of beliefs and that each
culture should be evaluated on its own merit. Nurses often impose their values on patients by
forcefully promoting western biomedical traditions while ignoring the patients‘ value of
treatment forms.

DIF: Cognitive Level: Analysis REF: p. 190 OBJ: Theory #7


TOP: Cultural Competence KEY: Nursing Process Step: N/A
MSC: NCLEX: Patient-Centered Care

NURSINGTB.COM

Downloaded by: SUCCEEDGRADES | abbieclin@gmail.com


Distribution
Downloaded by Dallen Mae D.ofKadir
this document is illegal
(kadir.dallenmae.d.bcsi@gmail.com)
lOMoARcPSD|35346190

DEWITS FUNDAMENTALStC
uvO
iaN
.cC
om
EP- T
ThSe A
MNarD
ketSpK
l a IcL
eLtoSBFuO
y aRndNSUeR
ll S
yoI u
Nr G
Stu
5dTyHME
atD
erIiaTl ION WILLIAMS TEST BANK

MULTIPLE RESPONSE

1. Although the words spirituality and religion are often used interchangeably, spirituality
actually refers to which of the following? (Select all that apply.)
a. Concerns of the spirit
b. Formalized system of beliefs
c. Understanding one‘s place in the world
d. An element of religion
e. Belief in a higher power or creative force
ANS: A, C, D, E
Spirituality and religion are similar, but both must be addressed for the patient‘s psychosocial
integrity to improve.

DIF: Cognitive Level: Comprehension REF: p. 184 OBJ: Theory #1


TOP: Spirituality and Religion KEY: Nursing Process Step: N/A
MSC: NCLEX: N/A

2. The nurse takes into consideration that culture and religion influence life choices that include:
(Select all that apply.)
a. smoking.
b. drug use.
c. frequency of intercourse.
d. nutrition.
e. reading material.
f. use of caffeine.

ANS: A, B, D, E, F NURSINGTB.COM
The use of drugs (including caffeine, tobacco, and alcohol), nutritional choices, and food
preparation are influenced by culture.

DIF: Cognitive Level: Comprehension REF: p. 186 OBJ: Theory #1


TOP: Cultural Influences on Life Choices
KEY: Nursing Process Step: Planning
MSC: NCLEX: Psychosocial Integrity: Coping and Adaptation

3. The nurse is aware that cultural influences shape the worldview of all people, which define for
each person the concepts of the: (Select all that apply.)
a. meaning of life‘s events.
b. basis of a value system.
c. rational explanation of life‘s mysteries.
d. meaningful employment.
e. difference between good and evil.

ANS: A, B, C, E
The ―worldview‖ of an individual shapes his or her value system, defines right and wrong,
and helps to explain life‘s events and mysteries. Significance of employment is not included
in a worldview.

DIF: Cognitive Level: Comprehension REF: p. 184 OBJ: Theory #1


TOP: Worldview KEY: Nursing Process Step: Assessment
MSC: NCLEX: Psychosocial Integrity: Coping and Adaptation

NURSINGTB.COM

Downloaded by: SUCCEEDGRADES | abbieclin@gmail.com


Distribution
Downloaded by Dallen Mae D.ofKadir
this document is illegal
(kadir.dallenmae.d.bcsi@gmail.com)
lOMoARcPSD|35346190

DEWITS FUNDAMENTALStC
uvO
iaN
.cC
om
EP- T
ThSe A
MNarD
ketSpK
l a IcL
eLtoSBFuO
y aRndNSUeR
ll S
yoI u
Nr G
Stu
5dTyHME
atD
erIiaTl ION WILLIAMS TEST BANK

4. Protestant denominations that use their own holy books in addition to the Bible include:
(Select all that apply.)
a. Baptists.
b. Christian Scientists.
c. Mormons.
d. Lutherans.
e. Episcopalians.

ANS: B, C
Protestant denominations that use another holy book in addition to the Bible are Christian
Scientists, Mormons, Jehovah‘s Witnesses, and the Unification Church.

DIF: Cognitive Level: Knowledge REF: p. 186 OBJ: Clinical Practice #2


TOP: Holy Books KEY: Nursing Process Step: N/A MSC: NCLEX: N/A

COMPLETION

1. Persons of African extraction are predisposed to the three disorders of ,


, and .

ANS:
sickle cell anemia, sarcoidosis, keloids
sickle cell anemia, keloids, sarcoidosis
keloids, sickle cell anemia, sarcoidosis
keloids, sarcoidosis, sickle cell anemia
sarcoidosis, sickle cell anemia, keNloUidRsSI NG T B. COM
sarcoidosis, keloids, sickle cell anemia

People of African or Mediterranean heritage are prone to sickle cell anemia and trait,
sarcoidosis, and keloids.

DIF: Cognitive Level: Knowledge REF: p. 194 OBJ: Theory #1


TOP: Racial Predisposition KEY: Nursing Process Step: N/A
MSC: NCLEX: N/A

2. The term developed to describe care that recognizes cultural diversity and that is sensitive to
the cultural needs of the patient is .

ANS:
transcultural nursing

Recognizing the need for culturally diverse capabilities in nursing is a growing aspect of
health care, especially in our global community.

DIF: Cognitive Level: Knowledge REF: p. XX OBJ: Theory #1


TOP: Transcultural Nursing KEY: Nursing Process Step: N/A
MSC: NCLEX: N/A

3. The concept of described by Dr. Madeline Leininger is the force that allows
people to grow, remain well, and avoid or face death.

NURSINGTB.COM

Downloaded by: SUCCEEDGRADES | abbieclin@gmail.com


Distribution
Downloaded by Dallen Mae D.ofKadir
this document is illegal
(kadir.dallenmae.d.bcsi@gmail.com)
lOMoARcPSD|35346190

DEWITS FUNDAMENTALStC
uvO
iaN
.cC
om
EP- T
ThSe A
MNarD
ketSpK
l a IcL
eLtoSBFuO
y aRndNSUeR
ll S
yoI u
Nr G
Stu
5dTyHME
atD
erIiaTl ION WILLIAMS TEST BANK

ANS:
human caring

Dr. Leininger described human caring as the force that allows people to grow, remain well,
avoid illness, and face or survive death.

DIF: Cognitive Level: Knowledge REF: p. 184 OBJ: Theory #2


TOP: Human Caring KEY: Nursing Process Step: N/A
MSC: NCLEX: N/A

NURSINGTB.COM

NURSINGTB.COM

Downloaded by: SUCCEEDGRADES | abbieclin@gmail.com


Distribution
Downloaded by Dallen Mae D.ofKadir
this document is illegal
(kadir.dallenmae.d.bcsi@gmail.com)
lOMoARcPSD|35346190

DEWITS FUNDAMENTALStC
uvO
iaN
.cC
om
EP- T
ThSe A
MNarD
ketSpK
l a IcL
eLtoSBFuO
y aRndNSUeR
ll S
yoI u
Nr G
Stu
5dTyHME
atD
erIiaTl ION WILLIAMS TEST BANK

Chapter 15: Loss, Grief, and End-of-Life Care


Williams: deWit's Fundamental Concepts and Skills for Nursing, 8th Edition

MULTIPLE CHOICE

1. A 33-year-old patient has been told that her disease is terminal. The patient is crying and
states, ―I will just be a dependent drain on my family. I hate being a burden to them.‖ The
nurse can best respond:
a. ―Is there any reason to tell your family about this now? You still have some time in
which you can enjoy life.‖
b. ―You have heard some very bad news today. Can you describe what you mean by
burden?‖
c. ―I doubt that your loving family will view your terminal care as a burden.‖
d. ―I know. I would feel terrible if I had been told I was dying.‖
ANS: B
A person‘s reaction to a loss is influenced by the importance of what that person perceives the
loss entails. To help the patient validate what the loss means is therapeutic.

DIF: Cognitive Level: Application REF: p. 200 OBJ: Clinical Practice #2


TOP: Value and Loss KEY: Nursing Process Step: Implementation
MSC: NCLEX: Psychosocial Integrity: Coping and Adaptation

2. A patient tells the nurse during an admission interview that his wife ―went on to her reward.‖
The nurse assesses that this statement is an indication that the patient:
a. has a strong religious belief sy s te m .
N U R S INGTB.COM
b. has dysfunctional grieving and is unable to address his wife‘s death.
c. is uncomfortable with the term ―death‖ and is using a euphemism.
d. is no longer grieving the death of his wife.
ANS: C
In many societies there is avoidance of using words such as ―died‖ or ―death,‖ and
euphemisms are commonly used.

DIF: Cognitive Level: Analysis REF: p. 199 OBJ: Theory #1


TOP: Societal Views of Death KEY: Nursing Process Step: Assessment
MSC: NCLEX: Psychosocial Integrity: Coping and Adaptation

3. A neighbor who was widowed 2 months ago gives the nurse his wife Helen‘s gardening books
―because the two of you loved flowers.‖ He tries to hold back tears, but begins to cry. To
decrease his discomfort, the nurse should say:
a. ―We certainly did, and I‘m going to miss her help—she was a real expert.‖
b. ―Helen wouldn‘t want to see you crying—she would rather you smile and
remember the good times,‖ and give him a hug.
c. ―Things will get easier with time. Time has a way of healing even the most painful
losses.‖
d. ―Helen is in a better place; you should be glad she isn‘t suffering anymore.‖
ANS: A

NURSINGTB.COM

Downloaded by: SUCCEEDGRADES | abbieclin@gmail.com


Distribution
Downloaded by Dallen Mae D.ofKadir
this document is illegal
(kadir.dallenmae.d.bcsi@gmail.com)
lOMoARcPSD|35346190

DEWITS FUNDAMENTALStC
uvO
iaN
.cC
om
EP- T
ThSe A
MNarD
ketSpK
l a IcL
eLtoSBFuO
y aRndNSUeR
ll S
yoI u
Nr G
Stu
5dTyHME
atD
erIiaTl ION WILLIAMS TEST BANK

Sharing the feeling of loss is appropriate and supportive and recognizes the loss of the
grieving husband.

DIF: Cognitive Level: Application REF: p. 200 OBJ: Clinical Practice #2


TOP: Therapeutic Communication KEY: Nursing Process Step: Implementation
MSC: NCLEX: Psychosocial Integrity: Coping and Adaptation

4. A terminally ill patient and his family have requested hospice care. The nurse clarifies hospice
care philosophy as:
a. helping the patient dies easily and quickly at home.
b. focusing on symptom management and comfort care.
c. excluding hospital care.
d. supporting assisted suicide if that is the patient‘s wish.

ANS: B
The primary goal of hospice and palliative care is to improve quality of life, alleviate
suffering, and improve end-of-life experience for the patients and their loved ones.

DIF: Cognitive Level: Comprehension REF: p. 201 OBJ: Theory #2


TOP: Hospice KEY: Nursing Process Step: Implementation
MSC: NCLEX: Physiological Integrity: Basic Care and Comfort

5. A terminally ill patient is angry and belligerent. He yells at the nurses and his family for not
coming quickly enough when he calls, and he states that he isn‘t receiving the care he needs.
The nurse‘s most informative response would be:
a. ―Don‘t let him get away with this abuse. He doesn‘t need to yell at you.‖
b. ―This may be how he feels best in dealing with this illness. You‘ll just have to
ignore it.‖ NURSINGTB.COM
c. ―I‘m afraid he is having a bad day and feels neglected.‖
d. ―Anger is a reaction that dying people experience. You may be feeling some anger
yourselves.‖
ANS: D
Anger is one of the stages terminally ill patients and their caregivers may experience. It may
be prolonged, or it may be a passing stage or one that is returned to in the course of the final
illness. Reacting with anger or just passively ignoring it does not assist the patient in dealing
with his feelings.

DIF: Cognitive Level: Analysis REF: p. 201|Table 15-1


OBJ: Theory #1 TOP: Anger Phase KEY: Nursing Process Step: Implementation
MSC: NCLEX: Psychosocial Integrity: Coping and Adaptation

6. A patient who was admitted to hospice for comfort care of end-stage respiratory disease is
asked what his hope is for his remaining life. He says, ―I‘m hoping for a miracle—that I will
wake up one of these mornings and be able to breathe like when I was 20.‖ The nurse would
best respond:
a. ―That‘s not likely to happen. Is there something more realistic you are hoping
for?‖
b. ―Your lungs are too damaged for you ever to breathe like you did when you were
20.‖
c. ―It must be very tiring to have to struggle for every breath you take.‖

NURSINGTB.COM

Downloaded by: SUCCEEDGRADES | abbieclin@gmail.com


Distribution
Downloaded by Dallen Mae D.ofKadir
this document is illegal
(kadir.dallenmae.d.bcsi@gmail.com)
lOMoARcPSD|35346190

DEWITS FUNDAMENTALStC
uvO
iaN
.cC
om
EP- T
ThSe A
MNarD
ketSpK
l a IcL
eLtoSBFuO
y aRndNSUeR
ll S
yoI u
Nr G
Stu
5dTyHME
atD
erIiaTl ION WILLIAMS TEST BANK

d. ―Your miracle may be in the form of medication that will improve your comfort.‖

ANS: D
Instilling hope that the struggle for respiratory relief will be available is a comfort measure.

DIF: Cognitive Level: Application REF: p. 202 OBJ: Clinical Practice #1


TOP: Hope KEY: Nursing Process Step: Implementation
MSC: NCLEX: Psychosocial Integrity: Basic Care and Comfort

7. An oncologist has recommended that a hospice patient receives palliative radiation to reduce
the pain of bony metastases. The patient reacts with anger, saying, ―I don‘t want any more
treatments that don‘t cure my cancer!‖ The hospice nurse tells the patient that the use of
radiation:
a. may disqualify him from receiving hospice care.
b. may halt the metastatic process.
c. of this type is not curative and is part of hospice care.
d. will be used as a single treatment measure. Previous medications will be
discontinued.
ANS: C
The goal of palliative care is to reduce or relieve the symptoms of a disease without
attempting to provide a cure (WHO, 2015a).

DIF: Cognitive Level: Analysis REF: p. 202 OBJ: Theory #2


TOP: Palliative Radiation KEY: Nursing Process Step: Implementation
MSC: NCLEX: Physiological Integrity: Basic Care and Comfort

NoUm
8. A patient admitted to the nursing h eIfNoGr TteBr.m
RS inMal care is determined that he should not
CO
receive extraordinary measures such as resuscitation or the use of a ventilator if he stops
breathing, or intravenous (IV) fluids or a feeding tube if he is unable to eat or drink. The nurse
explains that these desires can best be honored by:
a. signing a do-not-resuscitate/do-not-intubate (DNR/DNI) request.
b. including these desires in his will.
c. filing an advance directive with the health care facility.
d. informing his primary care provider of his desires.
ANS: C
An advance directive outlines the patient‘s desires relative to end-of-life issues. A DNR/DNI
only prevents the initiation of cardiopulmonary resuscitation (CPR). The naming of a durable
power of attorney for health care or a health care proxy appoints a person to carry out the
advance directive.

DIF: Cognitive Level: Application REF: p. 209 OBJ: Clinical Practice #4


TOP: Advance Directive KEY: Nursing Process Step: Implementation
MSC: NCLEX: Physiological Integrity: Basic Care and Comfort

9. The nurse is aware that in order to provide effective support to grieving patients and families,
the nurse must:
a. keep a professional distance from the situation.
b. understand all the theories of grief.
c. solidify his or her own view of death.
d. stay positive and optimistic at all times.

NURSINGTB.COM

Downloaded by: SUCCEEDGRADES | abbieclin@gmail.com


Distribution
Downloaded by Dallen Mae D.ofKadir
this document is illegal
(kadir.dallenmae.d.bcsi@gmail.com)
lOMoARcPSD|35346190

DEWITS FUNDAMENTALStC
uvO
iaN
.cC
om
EP- T
ThSe A
MNarD
ketSpK
l a IcL
eLtoSBFuO
y aRndNSUeR
ll S
yoI u
Nr G
Stu
5dTyHME
atD
erIiaTl ION WILLIAMS TEST BANK

ANS: C
Nurses must understand and solidify their own view of death before they can offer effective
support to dying patients and their significant others. Constant optimism and positive attitudes
are not always appropriate.

DIF: Cognitive Level: Comprehension REF: p. 211 OBJ: Clinical Practice #1


TOP: Attitude Toward Death KEY: Nursing Process Step: Planning
MSC: NCLEX: Psychosocial Integrity: Psychosocial Adaptation

10. The patient in hospice care says to the hospice nurse, ―I want you to read my obituary that I
just wrote.‖ The nurse assesses that this patient is in the Satir Blevins (2008) stage of:
a. practice.
b. chaos.
c. integration.
d. acceptance.

ANS: A
According to the Satir Blevins theory of loss (2008), this patient is in the phase of practice.
The patient is practicing with the writing of the obituary the fact that life is coming to an end.

DIF: Cognitive Level: Analysis REF: p. 199 OBJ: Theory #1


TOP: Satir/Blevins theory of Change and Loss
KEY: Nursing Process Step: Assessment
MSC: NCLEX: Psychosocial Integrity: Coping and Adaptation

11. The nurse can assess Cheyne-Stokes respiration by its characteristics of respirations that are:
a. harsh and rattling.
NURSINGTB.COM
b. wheezing and labored.
c. shallow followed by periods of apnea.
d. long periods of apnea followed by a hiccoughing breath.
ANS: C
Cheyne-Stokes respirations become more and more shallow followed by periods of apnea, and
then respirations resume.

DIF: Cognitive Level: Comprehension REF: p. 208 OBJ: Theory #5


TOP: The Dying Process KEY: Nursing Process Step: Assessment
MSC: NCLEX: Physiological Integrity: Basic Care and Comfort

12. When a patient admitted for terminal care is asked by the admitting nurse, ―What has your
doctor told you about your illness?‖ the patient states, ―She says that I have a terminal illness,
but I know she‘s wrong.‖ The nurse recognizes that:
a. the patient has a doctor who does not tell patients when they have a terminal
illness.
b. the patient is in denial and is unable to consciously admit what he has been told by
the doctor.
c. the patient needs to be told that he doesn‘t have long to live so that he can make
his final arrangements.
d. she will have to avoid all conversation that refers to his disease or symptoms so as
not to increase his anxiety.

NURSINGTB.COM

Downloaded by: SUCCEEDGRADES | abbieclin@gmail.com


Distribution
Downloaded by Dallen Mae D.ofKadir
this document is illegal
(kadir.dallenmae.d.bcsi@gmail.com)
lOMoARcPSD|35346190

DEWITS FUNDAMENTALStC
uvO
iaN
.cC
om
EP- T
ThSe A
MNarD
ketSpK
l a IcL
eLtoSBFuO
y aRndNSUeR
ll S
yoI u
Nr G
Stu
5dTyHME
atD
erIiaTl ION WILLIAMS TEST BANK

ANS: B
The patient‘s statement indicates Kübler Ross‘s stage of denial in which patients will not
accept information and buffer themselves from information that may be painful.

DIF: Cognitive Level: Analysis REF: p. 201|Table 15-1


OBJ: Theory #1 TOP: Denial Phase
KEY: Nursing Process Step: Assessment
MSC: NCLEX: Psychosocial Integrity: Coping and Adaptation

13. A terminal patient asks the nurse if he could be given a deliberate overdose of medication that
would ―end this.‖ According to the Code for Nurses, the nurse‘s most ethical response would
be:
a. ―Bringing on your death in this manner would be a subject you should discuss with
your family and primary care provider.‖
b. ―I can contact the primary care provider and request an order for a drug that could
be used as you request.‖
c. ―I cannot ethically give you a deliberate overdose.‖
d. ―Let me have a while to consider your request.‖
ANS: C
The position of the American Nurses Association (ANA) expressly states that active
euthanasia and assisted suicide are considered a violation of the ANA‘s Code for Nurses.

DIF: Cognitive Level: Application REF: p. 210 OBJ: Theory #7


TOP: Assisted Suicide KEY: Nursing Process Step: Implementation
MSC: NCLEX: Physiological Integrity: Basic Care and Comfort

14. A patient who is terminally ill h a sNbUeRe Sn I NunGaTbBl .eCtOo Mm a i nt a i n good nutrition because of nausea
and anorexia and has lost a great deal of weight. He is now unable to change his position in
bed and needs frequent perineal care because of urinary incontinence. The nurse planning his
care would include in the plan of care to:
a. coax him to eat high-calorie, high-fat food.
b. provide laxatives and stool softeners to prevent constipation.
c. contact the primary care provider for an order for tube feeding.
d. provide him with an air pressure mattress.
ANS: D
Provision of an air pressure mattress will help preserve the patient‘s skin integrity that the
weight loss, inactivity, poor nutritional intake, and incontinence have caused.

DIF: Cognitive Level: Application REF: p. 207 OBJ: Clinical Practice #3


TOP: Skin Integrity KEY: Nursing Process Step: Implementation
MSC: NCLEX: Physiological Integrity: Reduction of Risk

15. A terminal patient in a skilled nursing home has stated that he does not want to get out of bed,
because he is too tired and weak to sit in a chair. He sleeps on and off all day and night, his
position is changed every 2 hours, and he is comfortable on his pain regimen. The next day
the nurse will:
a. assess his strength and desire to get out of bed, but permit him to remain in bed if
he chooses, because that is his position of comfort.
b. encourage him to sit in the chair for an hour, because it is important to change his

NURSINGTB.COM

Downloaded by: SUCCEEDGRADES | abbieclin@gmail.com


Distribution
Downloaded by Dallen Mae D.ofKadir
this document is illegal
(kadir.dallenmae.d.bcsi@gmail.com)
lOMoARcPSD|35346190

DEWITS FUNDAMENTALStC
uvO
iaN
.cC
om
EP- T
ThSe A
MNarD
ketSpK
l a IcL
eLtoSBFuO
y aRndNSUeR
ll S
yoI u
Nr G
Stu
5dTyHME
atD
erIiaTl ION WILLIAMS TEST BANK

activity or he will get weaker from inactivity.


c. leave him in bed if he wishes, but have him do active exercises of his legs and
arms to prevent further muscle weakness.
d. get him out of bed for an hour in the morning and afternoon and for a short time
(15 minutes) before bedtime so that he does not sleep all day and stay awake at
night.
ANS: A
Comfort care recognizes that the patient is the best judge of his energy; if he does not wish to
get out of bed, then frequent turning and positioning and use of appropriate pressure and pain
relief measures are a better choice.

DIF: Cognitive Level: Application REF: p. 204 OBJ: Clinical Practice #3


TOP: Comfort KEY: Nursing Process Step: Assessment
MSC: NCLEX: Physiological Integrity: Basic Care and Comfort

16. A hospice patient is unable to tolerate food or fluids and has advance directives that indicate
that he does not want IV fluids or tube feedings if he is unable to take oral feedings. His
family is concerned that he will be very uncomfortable without food or fluids. The nurse
should tell them that:
a. once the patient is unconscious, his health care proxy can order IV fluids or tube
feedings to prevent dehydration.
b. dehydration is painful, and medication can be given to overcome any pain.
c. IV fluids or a tube feeding would make the patient much more comfortable, and
they should try to talk him into accepting them.
d. IV fluids or tube feeding would likely make the patient more uncomfortable, and
dehydration has been shown t oN Ud eRcSrIeNaGs eT Bp .aCi On Ma n d discomfort.
ANS: D
Dehydration in the terminally ill patient has been shown in research to decrease pain and to be
a more comfortable condition than when tube feeding is introduced into the intestine or when
IV fluids are used to reverse dehydration.

DIF: Cognitive Level: Application REF: p. 207 OBJ: Clinical Practice #5


TOP: Comfort KEY: Nursing Process Step: Planning
MSC: NCLEX: Physiological Integrity: Basic Care and Comfort

17. To help the family deal with the delirium of their dying relative, the nurse can suggest that
they should:
a. stimulate the patient with music and visits from friends.
b. talk to the patient in quiet tones.
c. sit quietly in the room with the patient.
d. speak firmly to the patient to bring him back to reality.

ANS: B
Talking with the patient is comforting to the patient. Even when unresponsive, patients can
hear. Stimulation is not helpful and may confuse the patient further.

DIF: Cognitive Level: Application REF: p. 207 OBJ: Theory #5


TOP: Delirium KEY: Nursing Process Step: Implementation
MSC: NCLEX: Physiological Integrity: Basic Care and Comfort

NURSINGTB.COM

Downloaded by: SUCCEEDGRADES | abbieclin@gmail.com


Distribution
Downloaded by Dallen Mae D.ofKadir
this document is illegal
(kadir.dallenmae.d.bcsi@gmail.com)
lOMoARcPSD|35346190

DEWITS FUNDAMENTALStC
uvO
iaN
.cC
om
EP- T
ThSe A
MNarD
ketSpK
l a IcL
eLtoSBFuO
y aRndNSUeR
ll S
yoI u
Nr G
Stu
5dTyHME
atD
erIiaTl ION WILLIAMS TEST BANK

18. After a patient death, the nurse is preparing to perform postmortem care. The body is placed
supine with the head raised so that:
a. the deceased will appear to be sleeping.
b. blood will not pool in the face and cause discoloration.
c. movement of the deceased will be more convenient.
d. feces and urine will drain onto the bed pads.
ANS: B
After death, the head of the deceased is raised to prevent discoloration by the pooling of blood
in the face.

DIF: Cognitive Level: Comprehension REF: p. 212 OBJ: Clinical Practice #6


TOP: Postmortem Care KEY: Nursing Process Step: Implementation
MSC: NCLEX: N/A

19. The loved ones of a young man, who is in the intensive care unit on life support after suffering
irreversible brain damage resulting from a motorcycle accident, have been approached by the
organ transplant team to consider organ donation. When they ask the nurse about this process,
the nurse‘s best response would be:
a. ―There is a small cost to the family for the donation, but it is for a good cause.‖
b. ―Often families are comforted by the knowledge that some good came from this
tragedy.‖
c. ―Because your son has been on life support, the only organs they can use would be
his kidneys.‖
d. ―Organ donation will probably delay the funeral.‖
ANS: B
There are many more persons neeN diUnRgSoIN
rgGaTnBs.C
thOaM
n there are donors. Nurses are in a position to
educate the public about organ donation.

DIF: Cognitive Level: Application REF: p. 214 OBJ: Clinical Practice #2


TOP: Organ Donation KEY: Nursing Process Step: Implementation
MSC: NCLEX: Physiological Integrity: Basic Care and Comfort

20. A terminally ill patient is experiencing a great deal of dyspnea and noisy, ―rattling,‖ rapid
respirations. The nurse administers morphine in a very small liquid or intramuscular dose. The
purpose of this is to:
a. decrease pain caused by dyspnea.
b. hasten death by stopping respirations.
c. decrease respiratory rate and relieve dyspnea.
d. dry up secretions that are causing rattling.
ANS: C
Morphine is used to ease terminal dyspnea by reducing the rate and increasing the depth of
respirations.

DIF: Cognitive Level: Comprehension REF: p. 207 OBJ: Clinical Practice #3


TOP: Use of Morphine KEY: Nursing Process Step: Implementation
MSC: NCLEX: Physiological Integrity: Basic Care and Comfort

NURSINGTB.COM

Downloaded by: SUCCEEDGRADES | abbieclin@gmail.com


Distribution
Downloaded by Dallen Mae D.ofKadir
this document is illegal
(kadir.dallenmae.d.bcsi@gmail.com)
lOMoARcPSD|35346190

DEWITS FUNDAMENTALStC
uvO
iaN
.cC
om
EP- T
ThSe A
MNarD
ketSpK
l a IcL
eLtoSBFuO
y aRndNSUeR
ll S
yoI u
Nr G
Stu
5dTyHME
atD
erIiaTl ION WILLIAMS TEST BANK

21. When the patient says, ―I can die happily if I can live long enough to see my first grandchild
that will be born next month.‖ The nurse assesses that this patient is experiencing Kübler
Ross‘s stage of:
a. denial.
b. bargaining.
c. anger.
d. depression.

ANS: B
Kübler Ross‘s stages of coping with death include the bargaining stage in which the person
seeks the reward of extended life.

DIF: Cognitive Level: Comprehension REF: p. 204|Table 15-1


OBJ: Theory #1 TOP: Stages of Coping with Death
KEY: Nursing Process Step: Assessment
MSC: NCLEX: Psychosocial Integrity: Coping and Adaptation

22. When the nurse notes an increase in the level of daily function in the terminal patient, the
nurse assesses that this patient has reached Kübler Ross‘s level of:
a. yearning.
b. bargaining.
c. depression.
d. acceptance.
ANS: D
Kübler Ross‘s stages of coping with death include the acceptance stage in which the struggle
is over, the pain is gone, and the patient has found peace. The patient frequently demonstrates
a heightened level of activity in h iNs UoRr ShI eNrGdTaBi l. yC Of uMnc t i on during this stage.

DIF: Cognitive Level: Comprehension REF: p. 201 OBJ: Theory #1


TOP: Acceptance Stage KEY: Nursing Process Step: Assessment
MSC: NCLEX: Psychosocial Integrity: Coping and Adaptation

23. When the parents of a child who is on life support ask what indicates brain death, the nurse
responds that brain death occurs when:
a. there have not been signals on the EKG.
b. no deep tendon reflexes can be stimulated.
c. there is an absence of EEG waves.
d. there have been no voluntary respirations for 5 minutes.

ANS: C
Brain death, which is defined as the absence of brain activity as evidenced by the absence of
electroencephalogram (ECG) waves.

DIF: Cognitive Level: Knowledge REF: p. 201 OBJ: Theory #1


TOP: Brain Death KEY: Nursing Process Step: Implementation
MSC: NCLEX: Psychosocial Integrity: Coping and Adaptation

24. The culturally sensitive nurse would delay a discussion relative to a patient‘s terminal illness
if that patient were a:
a. 28-year-old Caucasian auto mechanic.
b. 35-year-old Mexican American farm laborer.

NURSINGTB.COM

Downloaded by: SUCCEEDGRADES | abbieclin@gmail.com


Distribution
Downloaded by Dallen Mae D.ofKadir
this document is illegal
(kadir.dallenmae.d.bcsi@gmail.com)
lOMoARcPSD|35346190

DEWITS FUNDAMENTALStC
uvO
iaN
.cC
om
EP- T
ThSe A
MNarD
ketSpK
l a IcL
eLtoSBFuO
y aRndNSUeR
ll S
yoI u
Nr G
Stu
5dTyHME
atD
erIiaTl ION WILLIAMS TEST BANK

c. 40-year-old Chinese businessman.


d. 50-year-old American Indian policeman.
ANS: B
Persons with Hispanic and Korean backgrounds are less likely to want to know about their
terminal disease, feeling that the family should make the decisions about life-sustaining
treatments.

DIF: Cognitive Level: Comprehension REF: p. 201 OBJ: Theory #3


TOP: Cultural Considerations KEY: Nursing Process Step: Planning
MSC: NCLEX: Psychosocial Integrity: Coping and Adaptation

25. The nurse caring for a terminally ill patient with renal failure would question an order for pain
control that prescribed:
a. methadone.
b. oxycodone.
c. meperidine.
d. morphine.

ANS: C
Patients in renal failure cannot adequately clear meperidine (Demerol) from their system and
will become oversedated.

DIF: Cognitive Level: Application REF: p. 207|Table 15-2


OBJ: Theory #4 TOP: Meperidine KEY: Nursing Process Step: Implementation
MSC: NCLEX: Physiological Integrity: Pharmacological Therapies

MULTIPLE RESPONSE NURSINGTB.COM

1. During the dying process, patients may experience a bad taste in their mouth. Which of the
following are appropriate nursing actions? (Select all that apply.)
a. Offer the patient hard candy.
b. Assist the patient with frequent oral care.
c. Offer the patient large amounts of water or juice.
d. Explain to the patient that this is a normal part of dying and nothing can be done.
e. Offer the patient mouthwash.
f. Offer the patient frequent meals.

ANS: A, B, E
Hard candy, mouthwash, and oral care help stimulate saliva production and relieve the
dryness and any bad taste in the patient‘s mouth related to the dryness.

DIF: Cognitive Level: Application REF: p. 207 OBJ: Clinical Practice #3


TOP: Nursing Process and the Dying Patient
KEY: Nursing Process Step: Implementation
MSC: NCLEX: Physiological Integrity: Basic Care and Comfort

2. A coroner is required to investigate death in which of the following situations? (Select all that
apply.)
a. Deaths from injury
b. All deaths that occur at home

NURSINGTB.COM

Downloaded by: SUCCEEDGRADES | abbieclin@gmail.com


Distribution
Downloaded by Dallen Mae D.ofKadir
this document is illegal
(kadir.dallenmae.d.bcsi@gmail.com)
lOMoARcPSD|35346190

DEWITS FUNDAMENTALStC
uvO
iaN
.cC
om
EP- T
ThSe A
MNarD
ketSpK
l a IcL
eLtoSBFuO
y aRndNSUeR
ll S
yoI u
Nr G
Stu
5dTyHME
atD
erIiaTl ION WILLIAMS TEST BANK

c. Suspected suicides
d. Any death in the hospital
e. Deaths of persons not under the care of a primary care provider
ANS: A, C, E
The coroner investigates death when there may be a question of cause.

DIF: Cognitive Level: Analysis REF: p. 211 OBJ: Clinical Practice #6


TOP: Postmortem Issues KEY: Nursing Process Step: N/A
MSC: NCLEX: N/A

3. The nurse discusses the grief theory of Maciejewski, which outlines the stages of grief as
including: (Select all that apply.)
a. yearning.
b. bargaining.
c. anger.
d. denial.
e. depression.

ANS: A, C, D, E
The grief theory of Maciejewski states that the loss is accepted, but followed by denial,
yearning, anger, depression, and acceptance.

DIF: Cognitive Level: Comprehension REF: p. 200 OBJ: Theory #1


TOP: Maciejewski Theory KEY: Nursing Process Step: Implementation
MSC: NCLEX: Psychosocial Integrity: Coping and Adaptation

COMPLETION NURSINGTB.COM

1. When a patient chooses to die by refusing treatment that would prolong life, it is called
.

ANS:
passive euthanasia

Passive euthanasia is when the patient is choosing to refuse treatment for himself or herself,
which will result in death.

DIF: Cognitive Level: Comprehension REF: p. 209 OBJ: Theory #6


TOP: End-of-Life Issues KEY: Nursing Process Step: N/A
MSC: NCLEX: N/A

2. The five stages identified by Dr. Elisabeth Kübler Ross are , ,


, , and .

ANS:
denial; anger; bargaining; depression; acceptance
denial; acceptance; anger; bargaining; depression
denial; depression; acceptance; anger; bargaining
denial; bargaining; depression; acceptance; anger
acceptance; anger; bargaining; depression; denial

NURSINGTB.COM

Downloaded by: SUCCEEDGRADES | abbieclin@gmail.com


Distribution
Downloaded by Dallen Mae D.ofKadir
this document is illegal
(kadir.dallenmae.d.bcsi@gmail.com)
lOMoARcPSD|35346190

DEWITS FUNDAMENTALStC
uvO
iaN
.cC
om
EP- T
ThSe A
MNarD
ketSpK
l a IcL
eLtoSBFuO
y aRndNSUeR
ll S
yoI u
Nr G
Stu
5dTyHME
atD
erIiaTl ION WILLIAMS TEST BANK

acceptance; denial; anger; bargaining; depression


acceptance; depression; denial; anger; bargaining
acceptance; bargaining; depression; denial; anger
anger; bargaining; depression; acceptance; denial
anger; denial; bargaining; depression; acceptance
anger; acceptance; denial; bargaining; depression
anger; depression; acceptance; denial; bargaining
bargaining; anger; depression; acceptance; denial
bargaining; denial; anger; depression; acceptance
bargaining; acceptance; denial; anger; depression
bargaining; depression; acceptance; denial; anger
depression; acceptance; denial; bargaining; anger
depression; anger; acceptance; denial; bargaining
depression; bargaining; anger; acceptance; denial
depression; denial; bargaining; anger; acceptance

Each dying patient and loved ones may experience any of these stages during the dying
process. Understanding the stages assists in early identification and appropriate interventions
to help the patient and family cope.

DIF: Cognitive Level: Knowledge REF: p. 211 OBJ: Theory #1


TOP: The Dying Process KEY: Nursing Process Step: N/A
MSC: NCLEX: N/A

3. Individuals move through the stages of grief at their own pace. When a person is ―stuck‖ in a
stage and cannot move forward, this is called .
NURSINGTB.COM
ANS:
dysfunctional grieving

Dysfunctional grieving is visible grief over a prolonged period of time. Some theorists say
grief after 6 months is dysfunctional; others say up to 2 years.

DIF: Cognitive Level: Analysis REF: p. 201 OBJ: Theory #1


TOP: Grieving KEY: Nursing Process Step: N/A MSC: NCLEX: N/A

4. The nurse who adopts the Maciejewski theory of grief assesses for the reduction of grief after
a period of months.

ANS:
6
six

According to the theory of Maciejewski, the indicators of grief should peak and begin to
diminish after a period of 6 months.

DIF: Cognitive Level: Application REF: p. 200 OBJ: Theory #1


TOP: Maciejewski Theory KEY: Nursing Process Step: Assessment
MSC: NCLEX: Psychosocial Integrity: Coping and Adaptation

NURSINGTB.COM

Downloaded by: SUCCEEDGRADES | abbieclin@gmail.com


Distribution
Downloaded by Dallen Mae D.ofKadir
this document is illegal
(kadir.dallenmae.d.bcsi@gmail.com)
lOMoARcPSD|35346190

DEWITS FUNDAMENTALStC
uvO
iaN
.cC
om
EP- T
ThSe A
MNarD
ketSpK
l a IcL
eLtoSBFuO
y aRndNSUeR
ll S
yoI u
Nr G
Stu
5dTyHME
atD
erIiaTl ION WILLIAMS TEST BANK

ORDERING

1. Place the steps of postmortem care in their proper sequence. (Separate letters by a comma and
space as follows: A, B, C, D, E.)
a. Remove all tubing and equipment from the room.
b. Cleanse the body of all body fluids.
c. Position the body with the head of the bed raised 15 degrees.
d. Dress the deceased in a hospital gown and cover the deceased up to the chest.
e. Replace dentures.

ANS:
C, E, B, A, D

Postmortem care should be performed with dignity and reference for the deceased. The body
should be positioned with the head raised, dentures should be replaced, the body should be
cleansed of all body fluids and soiled dressings, all tubing and equipment should be removed
from the room, and the deceased should be dressed in a gown and covered up to the chest.

DIF: Cognitive Level: Analysis REF: p. 212 OBJ: Clinical Practice #6


TOP: Postmortem Care KEY: Nursing Process Step: N/A
MSC: NCLEX: N/A

NURSINGTB.COM

NURSINGTB.COM

Downloaded by: SUCCEEDGRADES | abbieclin@gmail.com


Distribution
Downloaded by Dallen Mae D.ofKadir
this document is illegal
(kadir.dallenmae.d.bcsi@gmail.com)
lOMoARcPSD|35346190

Stuvia.com - The Marketplace to Buy and Sell your Study Material

Chapter 16: Infection Prevention and Control: Protective Mechanisms and Asepsis
Test Bank

MULTIPLE CHOICE

1. When a patient in the ambulatory clinic is diagnosed as having pneumococcal pneumonia, the
nurse is aware that this infection is:
a. viral and will not respond to antibiotics.
b. bacterial and should respond to treatment with antibiotics.
c. fungal and is caused by the alteration of the normal flora of the lung.
d. resultant from a resistant organism and extreme caution must be taken.

ANS: B
The -coccal suffix indicates a bacterial infection with round cocci, which are bacteria that
usually respond to antibiotic therapy.

DIF: Cognitive Level: Comprehension REF: p. 209 OBJ: Theory #1


TOP: Infectious Agents KEY: Nursing Process Step: Planning
MSC: NCLEX: Physiological Integrity: physiological adaptation

2. The nurse explains to the patient who has pneumococcal pneumonia that the lungs serve as
the:
a. mode of transfer.
b. transmission of the disease.
c. reservoir.
d. organisms that cause the infection.

ANS: C
The reservoir is the place where the organism is found, such as a wound or, in this case, the
infected lungs. Droplets are modes of transmission from the reservoir.

DIF: Cognitive Level: Comprehension REF: p. 212 OBJ: Theory #2


TOP: Process of Infection KEY: Nursing Process Step: N/A
MSC: NCLEX: Physiological Integrity: physiological adaptation

3. The patient inquires about how his body will kill pathogens unassisted by antibiotics. The
nurse responds that a process called phagocytosis will:
a. stimulate the body to make more white blood cells.
b. create antibodies against the pathogen.
c. engulf and destroy the pathogen.
d. stimulate the production of interferons.
ANS: C
Phagocytes that are stored in the GI tract, liver, and spleen kill pathogens by engulfing and
destroying the invaders and cleaning up the debris.

Downloaded by: SUCCEEDGRADES | abbieclin@gmail.com


Distribution
Downloaded by Dallen Mae D.ofKadir
this document is illegal
(kadir.dallenmae.d.bcsi@gmail.com)
lOMoARcPSD|35346190

Stuvia.com - The Marketplace to Buy and Sell your Study Material

DIF: Cognitive Level: Comprehension REF: p. 208 OBJ: Theory #5


TOP: Body Defenses KEY: Nursing Process Step: Implementation
MSC: NCLEX: Physiological Integrity: physiological adaptation

4. The nurse explains that the immunizations against hepatitis B will:


a. stimulate the body to make antibodies the hepatitis B antigen.
b. offer immediate protection from hepatitis B by the injection of ready-made
antibodies.
c. introduce live antigens into the body that will stimulate the production of
antibodies.
d. offer protection against hepatitis A, C, and D, in addition to hepatitis B.

ANS: A
Artificially acquired immunization is achieved by introducing killed or attenuated pathogens
that are recognized as antigens by the immune system but that are not able to cause disease.
These antigens stimulate the formation of antibodies specific to hepatitis B only.

DIF: Cognitive Level: Comprehension REF: p. 218 OBJ: Theory #5


TOP: Immune Response KEY: Nursing Process Step: Implementation
MSC: NCLEX: Physiological Integrity: physiological adaptation

5. A mother and her 2-week-old infant, who is breast-fed, have been exposed to chickenpox.
Although the mother had chickenpox as a child, she is concerned about her baby. The nurse
explains:
a. the infant is at risk because the baby has not been immunized against the disease.
b. both infant and mother are at risk because the mother‘s immunity was acquired too
long ago to be effective.
c. the baby should receive immune globulin to protect him from the infection.
d. neither are at risk, because the mother has naturally acquired immunity, and she
passes antibodies to the baby through the breast milk.
ANS: D
Having had a disease such as chickenpox provides lifelong naturally acquired immunity,
which allows the mother to pass on antibodies through the placenta and breast milk to protect
the infant until his immune system is more mature.

DIF: Cognitive Level: Analysis REF: p. 218 OBJ: Theory #4


TOP: Immune Response KEY: Nursing Process Step: Implementation
MSC: NCLEX: Physiological Integrity: physiological adaptation

6. Health personnel should wash their hands with soap and water at the beginning of the shift
for:
a. 10 seconds.
b. 15 seconds.
c. 1 minute.
d. 2 minutes.

Downloaded by: SUCCEEDGRADES | abbieclin@gmail.com


Distribution
Downloaded by Dallen Mae D.ofKadir
this document is illegal
(kadir.dallenmae.d.bcsi@gmail.com)
lOMoARcPSD|35346190

Stuvia.com - The Marketplace to Buy and Sell your Study Material

ANS: B
Hand hygiene as suggested by the CDC should be done at the beginning of the shift for 15
seconds (30 seconds or longer in specialty areas).

DIF: Cognitive Level: Knowledge REF: p. 221 OBJ: Clinical Practice #2


TOP: Asepsis KEY: Nursing Process Step: Implementation
MSC: NCLEX: Safe Effective Care Environment: safety and infection control

7. An organism that is included in the extended-spectrum beta-lactamase producing pneumonia


(ESBL) group is:
a. Staphylococcus aureus.
b. Clostridium difficile.
c. Enterococcus.
d. Escherichia coli.
ANS: D
E. coli and Klebsiella pneumoniae are the organisms in the ESBL group.

DIF: Cognitive Level: Knowledge REF: p. 209 OBJ: Theory #1


TOP: Resistant Organisms KEY: Nursing Process Step: N/A
MSC: NCLEX: N/A

8. The nurse explains that medical asepsis differs from surgical asepsis in that medical asepsis:
a. kills all organisms.
b. is confined to the patient‘s room.
c. uses sterile attire to protect the patient.
d. uses sterile equipment before contact with the patient.

ANS: B
Medical asepsis confines the microorganisms to the patient‘s room. Medical asepsis does not
kill all organisms; it uses clean attire and equipment.

DIF: Cognitive Level: Comprehension REF: p. 218 OBJ: Theory #7


TOP: Medical Asepsis KEY: Nursing Process Step: Implementation
MSC: NCLEX: Safe Effective Care Environment: safety and infection control

9. When the nurse performs a procedure using sterile technique in the patient‘s unit, it means
that:
a. the equipment and supplies used are disposable and clean.
b. all organisms have been killed or removed from materials that come in contact with
the patient.
c. the nurse will do a 10-minute surgical scrub before beginning the procedure.
d. the nurse will be required to don a sterile gown, mask, and eye shields.

ANS: B

Downloaded by: SUCCEEDGRADES | abbieclin@gmail.com


Distribution
Downloaded by Dallen Mae D.ofKadir
this document is illegal
(kadir.dallenmae.d.bcsi@gmail.com)
lOMoARcPSD|35346190

Stuvia.com - The Marketplace to Buy and Sell your Study Material

Surgical asepsis or sterile technique uses sterilization to remove all organisms, not just
pathogens. The purpose is to protect the patient from the introduction of organisms that could
endanger health. Although sterile gloves are used, there is no need for a sterile gown.

DIF: Cognitive Level: Comprehension REF: p. 218 OBJ: Theory #7


TOP: Asepsis KEY: Nursing Process Step: Implementation
MSC: NCLEX: Safe Effective Care Environment: safety and infection control

10. A nurse teaching family members about hand hygiene in the home would emphasize:
a. keeping fingernails short and avoiding wearing rings.
b. washing hands up to the elbows for 2 minutes the first time in the day, and for 1
minute after a diaper change.
c. using disposable gloves after hand hygiene when feeding the infant.
d. that home care requires less attention to medical asepsis, so hand hygiene is
necessary only after toileting or handling soiled diapers.
ANS: A
Fingernails and jewelry provide hiding places for organisms and can scratch the baby. A 10- to
20-second hand hygiene routine is appropriate before handling the baby and after changing
diapers (as well as after toileting, before eating, and anytime they are soiled).

DIF: Cognitive Level: Application REF: p. 219 OBJ: Clinical Practice #2


TOP: Medical Asepsis KEY: Nursing Process Step: Implementation
MSC: NCLEX: Safe Effective Care Environment: safety and infection control

11. A nurse is using personal protective equipment (PPE) before entering the room of a patient
with diarrhea and vomiting who is being treated for an intestinal infection. The nurse most
likely needs to use which combination of PPE?
a. Gown, gloves, and mask
b. Gown, gloves, and goggles (or glasses)
c. Shoe covers, gown, and gloves
d. Reusable gown and mask

ANS: B
The gown is necessary to protect the nurse‘s uniform from becoming contaminated with
organisms from stool or vomitus, gloves protect hands from contact with these body
secretions, and goggles prevent splashing of contaminated material into the eyes. Protective
gowns are not reused.

DIF: Cognitive Level: Application REF: p. 228, Skill 16-2


OBJ: Clinical Practice #3 TOP: Standard Precautions
KEY: Nursing Process Step: Implementation
MSC: NCLEX: Safe Effective Care Environment: safety and infection control

12. When removing a used face mask, the nurse correctly:


a. lowers it below his chin to use the next time he enters that patient‘s room.

Downloaded by: SUCCEEDGRADES | abbieclin@gmail.com


Distribution
Downloaded by Dallen Mae D.ofKadir
this document is illegal
(kadir.dallenmae.d.bcsi@gmail.com)
lOMoARcPSD|35346190

Stuvia.com - The Marketplace to Buy and Sell your Study Material

b. removes the mask first before removing any other PPE.


c. unties the bottom ties first, then the top, and disposes of the mask without touching
it.
d. discards the mask only if it is wet; otherwise, he folds and stores it to reuse the next
time.
ANS: C
Face masks are not reused, and they should not be worn dangling around the neck. Gloves
should be removed first (and eyewear, if worn) before removing the mask so as not to
contaminate the nurse‘s hair or face.

DIF: Cognitive Level: Comprehension REF: p. 228, Skill 16-2


OBJ: Clinical Practice #3 TOP: Standard Precautions
KEY: Nursing Process Step: Implementation
MSC: NCLEX: Safe Effective Care Environment: safety and infection control

13. The nurse uses the Standard Precautions, as outlined by the Centers for Disease Control and
Prevention (CDC), when:
a. there is a suspicion of or risk of infection.
b. preventing transmission of respiratory and wound infections.
c. caring for patients who have wounds draining body fluids.
d. caring for all patients.

ANS: D
Standard Precautions include hand hygiene and use of PPE when there is exposure to blood,
body fluids, mucous membranes, nonintact skin, secretions, excretions, and contaminated
articles. Hand hygiene is part of patient care for all patients all the time, regardless of
diagnosis, and the selection of PPE is appropriate for all patients at some time or another.

DIF: Cognitive Level: Application REF: p. 224, Box 16-3


OBJ: Clinical Practice #3 TOP: Standard Precautions
KEY: Nursing Process Step: Implementation
MSC: NCLEX: Safe Effective Care Environment: safety and infection control

14. When the nurse is using a syringe and needle to give a patient an injection, he or she should:
a. never recap the needle afterwards to avoid risk of needle stick.
b. carefully break the needle from the syringe, using the needle cover to prevent reuse
of a used syringe and needle.
c. throw the needle and syringe immediately in a covered garbage can with a red
plastic liner to indicate the materials are biohazards.
d. recap the needle and place it carefully on the patient‘s table until leaving the room,
then discard it in a garbage container in the nurses‘ medication room.
ANS: A

Downloaded by: SUCCEEDGRADES | abbieclin@gmail.com


Distribution
Downloaded by Dallen Mae D.ofKadir
this document is illegal
(kadir.dallenmae.d.bcsi@gmail.com)
lOMoARcPSD|35346190

Stuvia.com - The Marketplace to Buy and Sell your Study Material

Used needles and sharps need to be handled very carefully to prevent sticks with contaminated
body fluids (blood). Never recapping the needle is an approved CDC Standard Precaution.
The other choices are contraindicated. Sharps should be discarded in a specially marked
container that does not endanger the person who empties it. The needle should not be broken
or recapped unless it is done with a one hand scoop method. Throwing needles or sharps in a
red plastic bag endangers others.

DIF: Cognitive Level: Application REF: p. 229 OBJ: Clinical Practice #3


TOP: Standard Precautions KEY: Nursing Process Step: Implementation
MSC: NCLEX: Safe Effective Care Environment: safety and infection control

15. The nurse using protective non-sterile gloves in the provision of patient care will wash his or
her hands after removal of the gloves in order to:
a. avoid transfer of organisms.
b. diminish possibility of latex allergy.
c. keep skin of hands from cracking and drying.
d. enhance the ease of donning a fresh pair of gloves.

ANS: A
Washing the hands immediately after removing barrier gloves decreases the likelihood that
organisms will gain access to the skin through small holes or imperfections in the gloves and
reduces the transfer of microorganisms to the environment and other persons.

DIF: Cognitive Level: Comprehension REF: p. 218, Box 16-3


OBJ: Clinical Practice #2 TOP: Standard Precautions
KEY: Nursing Process Step: Implementation
MSC: NCLEX: Safe Effective Care Environment: safety and infection control

16. The nurse encourages the 84-year-old patient who is recovering from a hip replacement to:
a. cough frequently to make up for the loss of cilia.
b. restrict fluid to prevent pulmonary congestion.
c. keep the bed flat to aid in lung expansion.
d. encourage bed rest.

ANS: A
Because of the loss of an adequate cough mechanism and loss of cilia, the mature adult should
be encouraged to cough frequently.

DIF: Cognitive Level: Application REF: p. 21, Table 16-6


OBJ: Theory #3 TOP: Prevention of Infection in the Elderly
KEY: Nursing Process Step: Implementation
MSC: NCLEX: Physiological Integrity: basic care and comfort

17. A patient is sent home with an open wound that is still infected and being treated with wet-to-
dry dressing changes four times a day. Before discharge, in order to prevent infecting other
family members, the nurse would teach the patient to:

Downloaded by: SUCCEEDGRADES | abbieclin@gmail.com


Distribution
Downloaded by Dallen Mae D.ofKadir
this document is illegal
(kadir.dallenmae.d.bcsi@gmail.com)
lOMoARcPSD|35346190

Stuvia.com - The Marketplace to Buy and Sell your Study Material

a. be the only person to perform the dressing changes, thus eliminating the risk of
infection to other family members.
b. wash hands thoroughly before the dressing change.
c. use gowns, gloves, and masks for any family contact with him.
d. maintain medical asepsis and proper handling of the contaminated dressings.

ANS: D
Medical asepsis is the most important way of preventing the spread of infection in the home
or in the hospital or nursing home.

DIF: Cognitive Level: Application REF: p. 224, Box 16-3


OBJ: Clinical Practice #4 TOP: Asepsis in the Home Environment
KEY: Nursing Process Step: Implementation
MSC: NCLEX: Safe Effective Care Environment: safety and infection control

18. The nurse instructing a patient in the home use of disinfectant would include the information
that the disinfectant can be used to:
a. decrease organisms on the patient‘s body but take care not to use it around the
patient‘s eyes or in the mouth.
b. sterilize instruments with a bacteriostatic disinfectant.
c. thoroughly clean and rinse all soap off the equipment before disinfecting it.
d. first remove all organic matter prior to disinfecting.

ANS: C
Disinfectants are irritating to the skin. Bacteriostatic disinfectants only weaken or slow the
growth of organisms; they do not kill them. Hot water hardens organic matter; therefore
equipment should be rinsed with cold water before hot soapy water is used.

DIF: Cognitive Level: Comprehension REF: p. 230 OBJ: Theory #6


TOP: Disinfection KEY: Nursing Process Step: Implementation
MSC: NCLEX: Safe Effective Care Environment: safety and infection control

19. The nurse is aware that the use of ethylene oxide gas is reserved for the sterilization of:
a. dressings.
b. surgical instruments.
c. heat-sensitive items.
d. floors and walls.

ANS: C
Ethylene oxide is used to sterilize heat-sensitive items and offers good penetration.

DIF: Cognitive Level: Knowledge REF: p. 230 OBJ: Theory #8


TOP: Cleaning and Disinfection KEY: Nursing Process Step: Planning
MSC: NCLEX: Safe Effective Care Environment: safety and infection control

20. The nurse recommends a good agent for disinfecting contaminated areas in the home is:
a. to cover the area with boiling water and let air dry.

Downloaded by: SUCCEEDGRADES | abbieclin@gmail.com


Distribution
Downloaded by Dallen Mae D.ofKadir
this document is illegal
(kadir.dallenmae.d.bcsi@gmail.com)
lOMoARcPSD|35346190

Stuvia.com - The Marketplace to Buy and Sell your Study Material

b. a 1:10 solution of chlorine bleach.


c. a 1:2 solution of alcohol.
d. to soak in a solution of povidone-iodine for 30 minutes and rinse with hot water.

ANS: B
A 1:10 solution of chlorine bleach is a good home disinfectant.

DIF: Cognitive Level: Comprehension REF: p. 230 OBJ: Theory #8


TOP: Asepsis in the Home Environment KEY: Nursing Process Step: Implementation
MSC: NCLEX: Safe Effective Care Environment: safety and infection control

21. The situation in which protective eyewear is required is:


a. suctioning a tracheotomy.
b. applying a dressing on the leg.
c. changing a baby‘s diaper.
d. gathering the linens off a contaminated bed.

ANS: A
Suctioning a tracheotomy causes the patient to cough and spray mucus. Protective eyewear is
used when there is danger of splashing blood or body fluids, such as in the operating room. It
is not necessary in the other situations.

DIF: Cognitive Level: Application REF: p. 226, Skill 16-2


OBJ: Clinical Practice #3 TOP: Standard Precautions
KEY: Nursing Process Step: Implementation
MSC: NCLEX: Safe Effective Care Environment: safety and infection control

22. To prevent a urinary infection in an elderly patient who is in traction for a broken femur, the
nurse would:
a. request a Foley catheter to be inserted.
b. encourage fluid intake to keep urine dilute.
c. encourage intake of apple juice to keep urine acidic.
d. offer a urinal every 2 hours.

ANS: B
Encouraging fluid intake to keep urine dilute would be the best way to prevent a urinary
infection because concentrated urine is a good medium for pathogens.

DIF: Cognitive Level: Application REF: p. 215, Table 16-6


OBJ: Theory #3 TOP: Urine Infections in the Elderly
KEY: Nursing Process Step: Implementation
MSC: NCLEX: Physiological Integrity: basic care and comfort

23. Fecal matter has contaminated the patient‘s bed sheet. The nurse should:
a. place a folded clean, dry sheet or plastic-backed protector over the soiled sheet
until it dries and then change the sheet.
b. don non-sterile gloves and gown, remove the soiled sheet, replace it with a clean

Downloaded by: SUCCEEDGRADES | abbieclin@gmail.com


Distribution
Downloaded by Dallen Mae D.ofKadir
this document is illegal
(kadir.dallenmae.d.bcsi@gmail.com)
lOMoARcPSD|35346190

Stuvia.com - The Marketplace to Buy and Sell your Study Material

one, and then dispose of the sheet in a plastic bag to prevent skin or clothing
contact.
c. remove the soiled sheet without exposure of skin or clothing to the sheet and rinse
it in the patient‘s bathroom sink to dilute or remove as much feces as possible.
d. use PPE to remove the sheet and place it in a pillowcase on the floor; then replace it
with a clean sheet.
ANS: B
When handling, transporting, or processing linens contaminated with blood or body fluids,
prevention of contact with skin, mucous membranes, and clothing or transfer of organisms to
the environment is most important. The sheets should not be rinsed in the patient‘s room, nor
should they ever be placed on the floor (clean or soiled).

DIF: Cognitive Level: Application REF: p. 224, Box 16-3


OBJ: Clinical Practice #3 TOP: Standard Precautions
KEY: Nursing Process Step: Implementation
MSC: NCLEX: Safe Effective Care Environment: safety and infection control

24. The nurse explains that the body‘s normal flora serve as:
a. aids to digestion and blood production.
b. prevention to the colonizing of pathogens.
c. managers of fluid balance of the body.
d. cell rebuilders.

ANS: B
Normal flora of the body prevent harmful microorganisms from colonizing and multiplying.

DIF: Cognitive Level: Comprehension REF: p. 208 OBJ: Clinical Practice #3


TOP: Prevention of Infection KEY: Nursing Process Step: Implementation
MSC: NCLEX: Physiological Integrity: physiological adaptation

25. A patient has been diagnosed with Creutzfeldt-Jakob disease (mad cow disease). The nurse
recognizes this disease is caused by a:
a. prion.
b. virus.
c. protozoa.
d. fungus.
ANS: A
Prions are protein particles that lack nucleic acids and are not inactivated by usual procedures
for destroying viruses. They do not trigger an immune response but instead cause degenerative
neurologic disease such as variant Creutzfeldt-Jakob disease (mad cow disease).

DIF: Cognitive Level: Knowledge REF: p. 209 OBJ: Theory #1


TOP: Disease KEY: Nursing Process Step: Planning
MSC: NCLEX: Physiological Integrity: physiological adaptation

Downloaded by: SUCCEEDGRADES | abbieclin@gmail.com


Distribution
Downloaded by Dallen Mae D.ofKadir
this document is illegal
(kadir.dallenmae.d.bcsi@gmail.com)
lOMoARcPSD|35346190

Stuvia.com - The Marketplace to Buy and Sell your Study Material

26. A patient has been diagnosed with Rocky Mountain spotted fever. The nurse recognizes this
disease is caused by a tick bite that infected the patient with:
a. Rickettsia rickettsii.
b. Rickettsia prowazekii.
c. Coxiella burnetii.
d. Aspergillus.

ANS: A
Rickettsia rickettsii is the rickettsia responsible for Rocky Mountain spotted fever.

DIF: Cognitive Level: Knowledge REF: p. 211 OBJ: Theory #1


TOP: Disease KEY: Nursing Process Step: Planning
MSC: NCLEX: Physiological Integrity: physiological adaptation

27. The nurse is aware that family education is necessary for the control of the most common
helminth infection, which is:
a. hook worms.
b. tape worms.
c. pinworms.
d. round worms.

ANS: C
Pinworms are the most common helminth infection, usually seen in children. Families must be
educated about the control of these parasites.

DIF: Cognitive Level: Knowledge REF: p. 210 OBJ: Theory #1


TOP: Helminth Infection KEY: Nursing Process Step: Planning
MSC: NCLEX: Safe Effective Care Environment: safety and infection control

28. A patient has been diagnosed with vaginal candidiasis. The nurse recognizes that this
condition is usually the result of:
a. unprotected sex.
b. poor personal hygiene.
c. long-term antimicrobial therapy.
d. using bath oils.
ANS: C
Long-term microbial therapy can alter the vaginal flora and cause fungal infections such as
vaginal candidiasis.

DIF: Cognitive Level: Knowledge REF: p. 210 OBJ: Theory #1


TOP: Candidiasis KEY: Nursing Process Step: Assessment
MSC: NCLEX: Physiological Integrity: physiological adaptation

29. A young patient became ill with mononucleosis that she contracted from drinking out of the
same glass as her boyfriend who also had the disease. The glass, an inanimate object, has
caused the indirect transmission. The inanimate transmitter is called:

Downloaded by: SUCCEEDGRADES | abbieclin@gmail.com


Distribution
Downloaded by Dallen Mae D.ofKadir
this document is illegal
(kadir.dallenmae.d.bcsi@gmail.com)
lOMoARcPSD|35346190

Stuvia.com - The Marketplace to Buy and Sell your Study Material

a. fomite.
b. prions.
c. vector.
d. interferon.
ANS: A
A fomite is an inanimate object that can transmit pathogens indirectly.

DIF: Cognitive Level: Comprehension REF: p. 212 OBJ: Theory #2


TOP: Disease KEY: Nursing Process Step: N/A MSC: NCLEX: N/A

30. The nurse instructs a patient that in order to reduce diseases that are transmitted via droplet,
the nose and mouth should be covered by:
a. moistened towelette.
b. handkerchief.
c. clean paper tissue.
d. bent elbow.

ANS: D
Covering the mouth with a bent elbow prevents the respirator droplets from contaminating the
hands.

DIF: Cognitive Level: Comprehension REF: p. 212 OBJ: Theory #2


TOP: Prevention of Disease KEY: Nursing Process Step: Implementation
MSC: NCLEX: Safe Effective Care Environment: safety and infection control

31. The nurse is aware that the first barrier to pathogen invasion is the:
a. skin.
b. immunizations.
c. good hygiene.
d. immune response.

ANS: A
Skin is the first barrier to pathogen invasion.

DIF: Cognitive Level: Knowledge REF: p. 213 OBJ: Theory #4


TOP: Prevention of Disease KEY: Nursing Process Step: Planning
MSC: NCLEX: Physiological Integrity: physiological adaptation

32. An enzyme found in the mucous membranes that is bactericidal is:


a. lysozyme.
b. ptyalin.
c. serotonin.
d. histamine.

ANS: A
Lysozyme is an enzyme found in the mucous membranes, tears, and saliva, which is
bactericidal and helps with disease prevention.

Downloaded by: SUCCEEDGRADES | abbieclin@gmail.com


Distribution
Downloaded by Dallen Mae D.ofKadir
this document is illegal
(kadir.dallenmae.d.bcsi@gmail.com)
lOMoARcPSD|35346190

Stuvia.com - The Marketplace to Buy and Sell your Study Material

DIF: Cognitive Level: Knowledge REF: pp. 213-214 OBJ: Theory #2


TOP: Prevention of Disease KEY: Nursing Process Step: N/A
MSC: NCLEX: N/A

33. A nurse is caring for a patient who was exposed to Bacillus anthracis. The nurse should wash
her hands with:
a. soap and water.
b. alcohol wipes.
c. chlorhexidine.
d. an antiseptic.

ANS: A
If exposure to Bacillus anthracis is suspected or proven, hands must be washed with soap and
water. The physical action of washing and rinsing hands is recommended because alcohols,
chlorhexidine products, iodophors, and other antiseptic agents have poor activity against
spores.

DIF: Cognitive Level: Application REF: p. 226, Box 16-2


OBJ: Theory #6 TOP: Hand Washing
KEY: Nursing Process Step: Implementation
MSC: NCLEX: Safe Effective Care Environment: safety and infection control

34. After reading a differential blood count on a patient, the nurse assesses that the patient‘s
infection is viral because the:
a. monocyte count is increased.
b. number of basophils is greatly elevated.
c. monocyte and neutrophil counts are decreased.
d. neutrophil count is decreased and the monocyte and the lymphocyte counts are both
elevated.
ANS: D
A viral infection will show a decreased neutrophil count with an elevation of monocytes and
lymphocytes.

DIF: Cognitive Level: Analysis REF: p. 216 OBJ: Theory #4


TOP: Viral Infection KEY: Nursing Process Step: Assessment
MSC: NCLEX: Physiological Integrity: physiological adaptation

COMPLETION

35. Fleas, ticks, mosquitoes, and other insects that harbor infection are called .

ANS:
vectors

Downloaded by: SUCCEEDGRADES | abbieclin@gmail.com


Distribution
Downloaded by Dallen Mae D.ofKadir
this document is illegal
(kadir.dallenmae.d.bcsi@gmail.com)
lOMoARcPSD|35346190

Stuvia.com - The Marketplace to Buy and Sell your Study Material

Fleas, ticks, mosquitoes, and other insects that harbor infection and transmit disease through
bites are called vectors.

DIF: Cognitive Level: Knowledge REF: p. 212 OBJ: Theory #2


TOP: Infection Control KEY: Nursing Process Step: N/A
MSC: NCLEX: N/A

36. The elderly should receive influenza immunization every .

ANS:
year
Influenza immunizations are recommended to be taken every year by the elderly, health care
workers, infants over the age of 6 months, and persons with chronic illnesses.

DIF: Cognitive Level: Comprehension REF: p. 215, Table 16-6


OBJ: Theory #3 | Theory #8 TOP: Influenza Immunization
KEY: Nursing Process Step: N/A MSC: NCLEX: N/A

37. The nurse is aware that gram-negative bacteria are capable of causing hemorrhagic shock by
the production of a(n) .

ANS:
endotoxin
Gram-negative bacteria are more dangerous than gram-positive bacteria because they produce
an endotoxin that can cause hemorrhagic shock.

DIF: Cognitive Level: Comprehension REF: p. 209 OBJ: Theory #1


TOP: Gram-Negative Bacteria KEY: Nursing Process Step: Planning
MSC: NCLEX: Physiological Integrity: physiological adaptation

MULTIPLE RESPONSE

38. Portal of exit transmission can be controlled by: (Select all that apply.)
a. treating infected patients.
b. isolation techniques.
c. effective inoculations.
d. improved hygiene.
e. barrier precautions.

ANS: A, B, E
Portal of exit transmission can be controlled by identifying and treating infected patients,
isolation techniques, barrier precautions, and proper handling of secretions, urine, feces, and
exudates.

DIF: Cognitive Level: Comprehension REF: p. 212 OBJ: Theory #2

Downloaded by: SUCCEEDGRADES | abbieclin@gmail.com


Distribution
Downloaded by Dallen Mae D.ofKadir
this document is illegal
(kadir.dallenmae.d.bcsi@gmail.com)
lOMoARcPSD|35346190

Stuvia.com - The Marketplace to Buy and Sell your Study Material

TOP: Body Defenses KEY: Nursing Process Step: N/A


MSC: NCLEX: N/A

39. The elderly are more susceptible to infection for a variety of reasons. The nurse should be
aware in planning care of elderly patients that the elderly are at risk due to: (Select all that
apply.)
a. increased gastric secretions.
b. increased macrophage activity in the lungs.
c. delayed immune response.
d. impaired thorax expansion.
e. urine stasis.

ANS: C, D, E
Because the elderly have decreased gastric secretion, decreased macrophage activity in the
lungs, immediate and delayed immune response, impaired thorax expansion, and urine stasis,
they are prone to infection.

DIF: Cognitive Level: Comprehension REF: p. 215, Table 16-6


OBJ: Theory #3 TOP: Susceptibility of the Elderly
KEY: Nursing Process Step: Planning
MSC: NCLEX: Physiological Integrity: physiological adaptation

40. The nurse outlines characteristics that affect the virulence of microorganisms, which include:
(Select all that apply.)
a. sensitivity to heat.
b. adherence to mucosal surfaces.
c. secretion of enzymes.
d. secretion of toxins.
e. penetration of mucous membranes.
ANS: B, D, E
The virulence of a microorganism is affected by the ability to adhere to and penetrate mucosal
membranes, multiply in the body, secrete harmful toxins, resist phagocytosis, and bind with
iron.

DIF: Cognitive Level: Comprehension REF: p. 210 OBJ: Theory #1


TOP: Virulence KEY: Nursing Process Step: Implementation
MSC: NCLEX: Physiological Integrity: physiological adaptation

OTHER

41. Place the process of an inflammatory response in the appropriate sequence. (Separate letters
with a comma and space as follows: A, B, C, D, E.)
A. Release of histamine
B. Edema or swelling
C. Redness

Downloaded by: SUCCEEDGRADES | abbieclin@gmail.com


Distribution
Downloaded by Dallen Mae D.ofKadir
this document is illegal
(kadir.dallenmae.d.bcsi@gmail.com)
lOMoARcPSD|35346190

Stuvia.com - The Marketplace to Buy and Sell your Study Material

D. Cell injury
E. Vasodilation

ANS:
D, E, C, B, A
After a cell injury the inflammatory response stimulates vasodilation, which brings blood and
fluid to the injury site causing redness and then edema. Histamine and serotonin are released,
which make the capillaries more permeable to allow defensive cells to surround the damaged
cells.

DIF: Cognitive Level: Analysis REF: p. 216 OBJ: Theory #5


TOP: Inflammatory Response KEY: Nursing Process Step: Assessment
MSC: NCLEX: Physiological Integrity: physiological adaptation

Downloaded by: SUCCEEDGRADES | abbieclin@gmail.com


Distribution
Downloaded by Dallen Mae D.ofKadir
this document is illegal
(kadir.dallenmae.d.bcsi@gmail.com)
lOMoARcPSD|35346190

Stuvia.com - The Marketplace to Buy and Sell your Study Material

Chapter 17: Infection Prevention and Control in the Hospital and Home
Williams: deWit's Fundamental Concepts and Skills for Nursing, 8th Edition

MULTIPLE CHOICE

1. When the patient complains of vague symptoms of malaise and fatigue and has a low-grade
fever, but has no other specific signs of illness, the nurse suspects that this patient is in the
prodromal phase of infection (the time immediately before the illness is diagnosed). The
nurse should include in the plan of care to:
a. increase assessment for specific signs of illness.
b. increase fluid intake.
c. place the patient in isolation.
d. report findings to the Infection Preventionist Officer.
ANS: A
The nurse should increase observation and assessments for specific signs of illness during
the prodromal phase of infection.

DIF: Cognitive Level: Application REF: p. 243 OBJ: Theory #1


TOP: Stages of Infection KEY: Nursing Process Step: Assessment
MSC: NCLEX: Safe, Effective Care Environment: Physiological Integrity: Basic Care and
Comfort

2. The nurse is aware that the patient most at risk for a health care–associated infection (HAI)
would be the:
a. 45-year-old in traction for a fractured femur.
b. 56-year-old with pneumonia who is receiving oxygen by mask.
c. 65-year-old with a Foley catheter.
d. 70-year-old with congestive heart failure attached to a monitor.
ANS: C
Patients with incisions, IV lines, urinary catheters, and other invasive procedures are more
likely to develop a health care–associated infection (HAI).

DIF: Cognitive Level: Analysis REF: p. 243 OBJ: Theory #2


TOP: Health Care–Associated Infections KEY: Nursing Process Step: Assessment
MSC: NCLEX: Safe, Effective Care Environment: Safety and Infection Control

3. The most effective part of infection control to reduce the incidence of health care–associated
infections (HAIs) is to:
a. use surgical asepsis for care of patients outside the operating room who are most at
risk for an HAI.
b. put all patients with wounds or invasive procedures on Transmission-Based
Precautions before they become infected.
c. place an alcohol-based hand sanitizer solution in every patient room.
d. use proper hand hygiene before and after caring for any patient, before donning
gloves and after their removal.
ANS: D

Downloaded by: SUCCEEDGRADES | abbieclin@gmail.com


Distribution
Downloaded by Dallen Mae D.ofKadir
this document is illegal
(kadir.dallenmae.d.bcsi@gmail.com)
lOMoARcPSD|35346190

Stuvia.com - The Marketplace to Buy and Sell your Study Material

Proper hand hygiene is the single most important aspect of medical asepsis and infection
control.

DIF: Cognitive Level: Application REF: p. 244 OBJ: Theory #2


TOP: Infection Control KEY: Nursing Process Step: Implementation
MSC: NCLEX: Safe, Effective Care Environment: Safety and Infection Control

4. The nurse cautions that a person in the incubation period of an infection:


a. has identifiable signs of a specific illness.
b. can transmit the disease although he or she does not feel ill.
c. will seek medical attention for the relief of symptoms.
d. will always exhibit symptoms within 48 hours.
ANS: B
During the incubation period, patients can transmit the disease, although they do not have
identifiable symptoms. The incubation period varies depending on the microorganisms.

DIF: Cognitive Level: Comprehension REF: p. 243 OBJ: Theory #1


TOP: Infection Control KEY: Nursing Process Step: Implementation
MSC: NCLEX: Physiological Integrity: Physiological Adaptation

5. The nurse clarifies that the difference between the use of earlier types of isolation
procedures and the use of current Standard Procedures plus Transmission-Based Precautions
as outlined by the CDC:
a. is that new diseases have continued to appear for which the older isolation
techniques were ineffective.
b. is based on the premise in the new procedures that all body substances except sweat
may be infectious, even when the person is not known to have a specific disease.
c. is complicated and hard to follow.
d. is based on newer knowledge of how HIV is spread, to better protect health care
workers from blood-borne pathogens.
ANS: B
Earlier Universal Precautions and Isolation Precautions focused on preventing the spread of
blood-borne pathogens and secretions or excretions of a person specifically known to be
infected. They have been replaced with Standard Precautions for all patients, and
Transmission-Based Precautions for those specifically infected, because of the
understanding that infection may be present in body secretions or excretions even when the
person is apparently healthy.

DIF: Cognitive Level: Comprehension REF: p. 245|Box 17-1


OBJ: Theory #3 TOP: History of Infection Control
KEY: Nursing Process Step: Planning
MSC: NCLEX: Safe, Effective Care Environment: Safety and Infection Control

6. A patient who has active primary tuberculosis is placed on Airborne Precautions. In addition
to observing Standard Precautions for this patient, the nurse expects that:
a. the patient can be in a room with a roommate, if both persons wear masks.
b. a special particulate filter mask (respirator) will be worn by anyone entering the
room.

Downloaded by: SUCCEEDGRADES | abbieclin@gmail.com


Distribution
Downloaded by Dallen Mae D.ofKadir
this document is illegal
(kadir.dallenmae.d.bcsi@gmail.com)
lOMoARcPSD|35346190

Stuvia.com - The Marketplace to Buy and Sell your Study Material

c. the patient may leave the room freely as long as the patient wears a mask at all
times.
d. no mask is needed unless performing close contact nursing care.
ANS: B
An N95 particulate filter respirator is worn by anyone entering the room of a patient with
primary tuberculosis. A private room is preferred. The patient should leave the room only
for essential activities (with a mask).

DIF: Cognitive Level: Application REF: p. 247 OBJ: Theory #5


TOP: Infection Control KEY: Nursing Process Step: Planning
MSC: NCLEX: Safe, Effective Care Environment: Safety and Infection Control

7. The nurse performing a surgical scrub is aware that the average time for the scrub is:
a. 3 minutes.
b. 5 minutes.
c. 6 minutes.
d. 7 minutes.
ANS: A
The average length of time required for a surgical scrub will differ with the antimicrobial
solution used and the agency policy, but the average is 2 to 4 minutes.

DIF: Cognitive Level: Knowledge REF: p. 254 OBJ: Theory #3


TOP: Infection Control KEY: Nursing Process Step: Implementation
MSC: NCLEX: Safe, Effective Care Environment: Safety and Infection Control

8. A patient has a nursing diagnosis of Infection, related to inadequate primary defenses, as


evidenced by surgical incision and intravenous (IV) line access. An appropriate nursing
intervention for this patient is to:
a. assess and document skin condition around the incision and IV site at each shift.
b. limit visitors to immediate family to decrease exposure to infection.
c. require the use of a facemask by nursing staff when they are providing care.
d. maintain ―clean‖ technique in the change of wound dressing and IV site.
ANS: A
Early identification of infection leads to prompt treatment and decreased transmission to
others. Effective documentation provides a comparison from shift to shift about appearance
of incision and IV site.

DIF: Cognitive Level: Application REF: p. 243 OBJ: Theory #2


TOP: Infection Control KEY: Nursing Process Step: Assessment
MSC: NCLEX: Safe, Effective Care Environment: Safety and Infection Control

9. The correct way to handle soiled linens in the room of a patient who is on Contact
Precautions is for the nurse to:
a. shake out the linens before placing them in a designated laundry bag to ensure that
there are no plastics or valuables caught in the sheets.
b. wear a gown to protect the uniform and wear barrier gloves to roll the soiled sheets
together and place them in the designated container.

Downloaded by: SUCCEEDGRADES | abbieclin@gmail.com


Distribution
Downloaded by Dallen Mae D.ofKadir
this document is illegal
(kadir.dallenmae.d.bcsi@gmail.com)
lOMoARcPSD|35346190

Stuvia.com - The Marketplace to Buy and Sell your Study Material

c. remove the soiled sheets using barrier gloves and to keep the linens away from the
uniform and then place them in a laundry bag held by another nurse outside the
room.
d. carry the soiled sheets directly to the unit laundry area before removing the
personal protective equipment (PPE).
ANS: B
The nurse should wear a gown to protect the uniform and should wear barrier gloves to roll
the soiled sheets together and place them in the designated container inside the room.

DIF: Cognitive Level: Application REF: p. 152|Box 17-3


OBJ: Theory #7 TOP: Infection Control
KEY: Nursing Process Step: Implementation
MSC: NCLEX: Safe, Effective Care Environment: Safety and Infection Control

10. A patient on Airborne Precautions says to the nurse, ―I feel like I‘m going crazy cooped up
in here. I feel like just sneaking out and finding someone to talk to.‖ The best response by
the nurse is:
a. ―You would be jeopardizing everyone you come into contact with. You could give a
lot of innocent people your disease.‖
b. ―It won‘t be long before you can safely get out of here without being a danger to
others.‖
c. ―You must be feeling bored being shut up in here. Have you been following the
wonderful season our football team has been having?‖
d. ―I know just how you feel. Sometimes I can‘t get outdoors because of the rain, and
it‘s so hard being cooped up.‖
ANS: C
Talking about a shared interest will divert the patient.

DIF: Cognitive Level: Application REF: p. 254 OBJ: Theory #8


TOP: Isolation KEY: Nursing Process Step: Implementation
MSC: NCLEX: Psychosocial Integrity: Coping and Adaptation

11. A patient is discharged home with a draining wound that was infected and for which he was
on Contact Precautions while in the hospital. He lives at home with his 48-year-old wife and
their 17-year-old daughter. It is most important to emphasize to this patient that:
a. he should maintain a safe distance from his family.
b. he should use paper plates and disposable utensils.
c. soiled dressings should be disposed of in plastic bags that are tied securely.
d. his family members should wear gloves when handling his plate and eating
utensils.
ANS: C
In the home, Contact Precautions require the use of barrier gloves to prevent contact with
wound secretions during dressing changes (or if linens were soiled) and during the handling
of soiled dressings. Soiled dressings should be disposed of in a securely tied plastic bag.
Frequent hand hygiene and attention to medical asepsis principles are sufficient.

DIF: Cognitive Level: Application REF: p. 252|Box 17-4

Downloaded by: SUCCEEDGRADES | abbieclin@gmail.com


Distribution
Downloaded by Dallen Mae D.ofKadir
this document is illegal
(kadir.dallenmae.d.bcsi@gmail.com)
lOMoARcPSD|35346190

Stuvia.com - The Marketplace to Buy and Sell your Study Material

OBJ: Theory #6 TOP: Infection Control in the Home


KEY: Nursing Process Step: Implementation
MSC: NCLEX: Safe, Effective Care Environment: Safety and Infection Control

12. A family member has been instructed in the administration of subcutaneous medication at
home. The nurse instructs her to:
a. break the needle off from the syringe so that it can‘t be reused and wrap the broken
needle and syringe in newspaper and throw them in the garbage.
b. recap the needle and dispose of it in the garbage, because it can‘t accidentally stick
anyone with the cap replaced.
c. save the used needles and syringes for the visiting nurse, who can collect them and
arrange for proper disposal.
d. place the used syringe and needle, without recapping it, in a large plastic bottle
with a secure lid.
ANS: D
The use of a heavy plastic container with a secure lid, such as a bleach or detergent bottle, is
a safe way to dispose of the uncapped syringe.

DIF: Cognitive Level: Application REF: p. 252|Box 17-4


OBJ: Theory #6 TOP: Infection Control in the Home
KEY: Nursing Process Step: Implementation
MSC: NCLEX: Safe, Effective Care Environment: Safety and Infection Control

13. The nurse recognizes a break in aseptic technique when:


a. the hand towel is kept above the waist level.
b. the hands are kept lower than the elbows during the surgical scrub.
c. faucets are turned off with a paper towel elbow or foot control in a scrub for a
surgical operation.
d. the sterile glove is donned by sliding the ungloved hand under the cuff of the sterile
glove.
ANS: B
Allowing the hands to become lower than the elbow is a break in aseptic technique.

DIF: Cognitive Level: Application REF: p. 255|Box 17-5


OBJ: Theory #9 TOP: Surgical Asepsis
KEY: Nursing Process Step: Implementation
MSC: NCLEX: Safe, Effective Care Environment: Safety and Infection Control

14. The nursing intervention most likely to decrease the chance of health care–associated
infections (HAIs) for a 76-year-old patient following bowel resection surgery would be to
have the patient:
a. turn, cough, and deep breathe every 2 hours.
b. limit ambulation.
c. get blood pressure, pulse, and respirations assessed every 4 hours.
d. keep the room door closed.
ANS: A

Downloaded by: SUCCEEDGRADES | abbieclin@gmail.com


Distribution
Downloaded by Dallen Mae D.ofKadir
this document is illegal
(kadir.dallenmae.d.bcsi@gmail.com)
lOMoARcPSD|35346190

Stuvia.com - The Marketplace to Buy and Sell your Study Material

Having the patient turn, cough, and deep breathe every 2 hours can help prevent the
complication of pneumonia in the postoperative patient. Turning, coughing, and deep
breathing can enhance the prevention of pneumonia in the surgical patient.

DIF: Cognitive Level: Application REF: p. 244 OBJ: Theory #2


TOP: Health Care–Associated Infections KEY: Nursing Process Step: Implementation
MSC: NCLEX: Physiological Integrity: Basic Care and Comfort

15. When caring for a patient on Droplet Precautions, it is most important for the nurse to:
a. wear the appropriate respiratory device for any entry into the room.
b. cover the patient with a clean sheet when transporting the patient to x-ray.
c. wear a gown and gloves for any contact with the patient.
d. wear a mask if working within 3 feet of the patient.
ANS: D
The wearing of a mask when working within 3 feet is all that is necessary for a patient on
Droplet Precautions.

DIF: Cognitive Level: Application REF: p. 245|Box 17-1


OBJ: Theory #4 TOP: Infection Control
KEY: Nursing Process Step: Implementation
MSC: NCLEX: Safe, Effective Care Environment: Safety and Infection Control

16. The nurse adding sterile liquids to a sterile field should:


a. hold the liquid container high over the sterile field.
b. remove the cap and place the container with the inside facing down.
c. prepare a new sterile field if it becomes wet during the procedure.
d. carefully reach over the sterile field to pour the liquid.
ANS: C
Once a sterile field is wet, it is contaminated and a new field needs to be prepared.

DIF: Cognitive Level: Application REF: p. 262|Steps 17 1


OBJ: Clinical Practice #2 TOP: Surgical Asepsis
KEY: Nursing Process Step: Implementation
MSC: NCLEX: Safe, Effective Care Environment: Safety and Infection Control

17. A patient with primary tuberculosis is on Airborne Precautions, and he spends much of the
day sleeping and is confused and awake at night. An appropriate nursing intervention for
this patient is to:
a. provide diversionary activities or visitors during the day to decrease his sense of
isolation and sensory deprivation.
b. arrange for the patient to be transported to the lounge while wearing a surgical
mask to provide more stimulation.
c. consult with the health care provider for an order for an antidepressant and sleeping
medication to treat the depression.
d. encourage staff to ―visit‖ with him from the doorway so that they do not have to
wear a mask and they can increase his wakeful times during the day.
ANS: A

Downloaded by: SUCCEEDGRADES | abbieclin@gmail.com


Distribution
Downloaded by Dallen Mae D.ofKadir
this document is illegal
(kadir.dallenmae.d.bcsi@gmail.com)
lOMoARcPSD|35346190

Stuvia.com - The Marketplace to Buy and Sell your Study Material

A patient on Airborne Precautions (or any isolation) may become confused and have
difficulty sleeping related to sensory deprivation, and the daytime sleeping may reflect
boredom and further contribute to sleeplessness at night. Providing diversionary activities
addresses these problems.

DIF: Cognitive Level: Analysis REF: p. 254 OBJ: Theory #8


TOP: Isolation KEY: Nursing Process Step: Planning
MSC: NCLEX: Psychosocial Integrity: Coping and Adaptation

18. An 84-year-old patient is hospitalized for an infected stasis ulcer on his ankle. The nurse is
aware that this patient is at risk for a hospital-acquired infection (HAI) because the:
a. patient already has a blood-borne infection.
b. patient‘s defenses are already engaged with the initial infection.
c. ulcer will make this patient bedfast.
d. stasis ulcers predispose the older adult to pneumonia and urinary infections.
ANS: B
Older adults have an impaired immune system. This patient‘s immune system is already
engaged with the stasis ulcer, which may put him at risk for an HAI.

DIF: Cognitive Level: Application REF: p. 246 OBJ: Theory #2


TOP: Infection Control KEY: Nursing Process Step: Diagnosis
MSC: NCLEX: Safe, Effective Care Environment: Safety and Infection Control

19. The nurse collecting a sputum specimen for a patient with staphylococcal pneumonia will:
a. wipe the specimen container with antimicrobial solution and hand carry it to the
laboratory.
b. double bag the specimen container and send the specimen to the laboratory.
c. send the specimen to the laboratory in a Biohazard bag.
d. notify the laboratory to collect the contaminated specimen.
ANS: C
Contaminated specimens are collected using appropriate PPEs and sent to the laboratory in a
clearly marked Biohazard bag as required by OSHA.

DIF: Cognitive Level: Application REF: p. 249


OBJ: Clinical Practice #2 TOP: Infection Control
KEY: Nursing Process Step: Implementation
MSC: NCLEX: Safe, Effective Care Environment: Safety and Infection Control

20. The nurse is helping the health care provider perform a sterile procedure at the bedside.
Halfway through the procedure, the nurse believes the health care provider has contaminated
the sterile field. The nurse should:
a. report the health care provider for violating surgical asepsis and endangering the
patient.
b. ask the health care provider whether she contaminated her glove and the sterile
field.
c. point out the possible break in surgical asepsis and provide another set of sterile
gloves and a fresh sterile field.
d. not say anything, because it is near the end of the procedure.

Downloaded by: SUCCEEDGRADES | abbieclin@gmail.com


Distribution
Downloaded by Dallen Mae D.ofKadir
this document is illegal
(kadir.dallenmae.d.bcsi@gmail.com)
lOMoARcPSD|35346190

Stuvia.com - The Marketplace to Buy and Sell your Study Material

ANS: C
It is the responsibility of the nurse to point out any possible break in surgical asepsis. Saying
nothing does not protect the patient and is negligence on the part of the nurse. A fresh sterile
field should be provided.

DIF: Cognitive Level: Application REF: p. 255|Box 17-5


OBJ: Theory #9 | Clinical Practice #2 TOP: Surgical Asepsis
KEY: Nursing Process Step: Implementation
MSC: NCLEX: Safe, Effective Care Environment: Safety and Infection Control

21. A nurse is instructing one of the facility‘s unlicensed assistive personnel (UAP) in ways to
prevent health care–associated infections. The nurse recognizes that further instruction is
warranted when the UAP states, ―I will:
a. wash my hands before and after caring for patients.‖
b. cleanse patients from the rectum to the urinary meatus.‖
c. clean residual urine off the catheter bag when emptying it.‖
d. put all the soiled linen in the hamper in the room.‖
ANS: B
A person should never cleanse from the rectal area to the urinary meatus because of the risk
of introducing infection into the urinary tract.

DIF: Cognitive Level: Application REF: p. 243 OBJ: Theory #2


TOP: Prevention of Infection
KEY: Nursing Process Step: Implementation Evaluation
MSC: NCLEX: Safe, Effective Care Environment: Safety and Infection Control

22. A patient is hospitalized with suspected disseminated zoster (varicella). The nurse should
anticipate that this patient will be placed on:
a. Standard Precautions.
b. Droplet Precautions.
c. Airborne Precautions.
d. Contact Precautions.
ANS: C
Airborne Precautions are used in addition to Standard Precautions for patients with known
or suspected serious illnesses transmitted by airborne droplet nuclei. Examples of such
diseases are measles, varicella (including disseminated zoster), and pulmonary tuberculosis.

DIF: Cognitive Level: Application REF: p. 245|Box 17-1


OBJ: Theory #4 TOP: Airborne Precautions
KEY: Nursing Process Step: Planning
MSC: NCLEX: Safe, Effective Care Environment: Safety and Infection Control

23. A patient is hospitalized with pertussis. The nurse should place the patient on what type of
precautions?
a. Contact Precautions
b. Airborne Precautions
c. Droplet Precautions

Downloaded by: SUCCEEDGRADES | abbieclin@gmail.com


Distribution
Downloaded by Dallen Mae D.ofKadir
this document is illegal
(kadir.dallenmae.d.bcsi@gmail.com)
lOMoARcPSD|35346190

Stuvia.com - The Marketplace to Buy and Sell your Study Material

d. Standard Precautions
ANS: C
Droplet Precautions are used in addition to Standard Precautions for patients with known or
suspected serious illnesses transmitted by large particle droplets. An example of such an
illness is pertussis.

DIF: Cognitive Level: Application REF: p. 245|Box 17-1


OBJ: Theory #24 TOP: Prevention of Infection
KEY: Nursing Process Step: Implementation MSC: NCLEX: N/A

24. The patient for whom the nurse should observe Contact Precautions in addition to Standard
Precautions would be diagnosed with:
a. pulmonary tuberculosis.
b. Haemophilus influenzae meningitis.
c. pertussis.
d. respiratory syncytial virus.
ANS: D
Contact Precautions are used in addition to Standard Precautions for patients with known or
suspected serious illnesses easily transmitted by direct patient contact or by contact with
items in the patient‘s environment. An example of such an illness is respiratory syncytial
virus.

DIF: Cognitive Level: Application REF: p. 245|Box 17-1


OBJ: Theory #24 TOP: Contact Precautions
KEY: Nursing Process Step: Implementation
MSC: NCLEX: Safe, Effective Care Environment: Safety and Infection Control

25. When the nurse is explaining Tier 2 as developed by the Hospital Infection Control Practices
Advisory Committees (HICPAC), the nurse will emphasize that the purpose of Tier 2 is to:
a. interrupt the mode of transmission.
b. monitor the efficiency of the treatment.
c. be put in place of Standard Precautions.
d. update all information relative to infections.
ANS: C
Tier 2, or Transmission-Based Precautions, is focused on interrupting the mode of
transmission and is always used in combination with Standard Precautions.

DIF: Cognitive Level: Comprehension REF: p. 252|Box 17-3


OBJ: Theory #23 TOP: Prevention of Infection
KEY: Nursing Process Step: Implementation
MSC: NCLEX: Safe, Effective Care Environment: Safety and Infection Control

26. In caring for a patient with active tuberculosis, the nurse should anticipate:
a. wearing an N95 mask.
b. wearing two masks to better filter microorganisms.
c. donning a mask only in the case of close contact.
d. placing a mask on the patient while care is being performed.

Downloaded by: SUCCEEDGRADES | abbieclin@gmail.com


Distribution
Downloaded by Dallen Mae D.ofKadir
this document is illegal
(kadir.dallenmae.d.bcsi@gmail.com)
lOMoARcPSD|35346190

Stuvia.com - The Marketplace to Buy and Sell your Study Material

ANS: A
When the patient has known or suspected pulmonary tuberculosis, a special particulate filter
mask called an N95 mask must be worn.

DIF: Cognitive Level: Application REF: p. 252|Box 17-3


OBJ: Theory #5 TOP: Prevention of Infection
KEY: Nursing Process Step: Planning
MSC: NCLEX: Safe, Effective Care Environment: Safety and Infection Control

27. The nurse is instructing one of the facility‘s unlicensed assistive personnel (UAP) about how
to correctly use a sharps container. The nurse recognizes that further instruction is warranted
when the UAP states, ―I will:
a. drop sharp items, including needles, into a sharps container.‖
b. shake the sharps container gently to settle the contents.‖
c. put my fingers inside the opening to push the item well inside the container.‖
d. replace the sharps container when it is two thirds full.‖
ANS: C
No one should ever put fingers inside the opening of the sharps container. The container
may be shaken gently to settle the contents and make more room if necessary. Sharps
containers should be replaced when they are two thirds full.

DIF: Cognitive Level: Comprehension REF: p. 250 OBJ: Theory #7


TOP: Safety KEY: Nursing Process Step: Implementation
MSC: NCLEX: Safe, Effective Care Environment: Safety and Infection Control

28. A nurse is caring for a patient who is in isolation. The nurse would correctly do which of the
following?
a. Freely take items in and out of the isolation room.
b. Shake linen when removing it from the bed.
c. Turn faucets on and off using a paper towel.
d. Consider items dropped on the floor useable.
ANS: C
General principles regarding isolation include that anything that is brought into the isolation
area must not be removed except in proper containers, which are then placed in an outside
container labeled ―Hazardous Material—Biohazard.‖ They also include never shaking linen
when removing it or placing it on the bed, turning faucets on and off using a paper towel to
protect the hands, and that floors are considered contaminated; anything dropped on the
floor is contaminated and must be discarded or cleaned carefully before reuse.

DIF: Cognitive Level: Application REF: p. 251|Box 17-2


OBJ: Theory #7 TOP: Isolation KEY: Nursing Process Step: Implementation
MSC: NCLEX: Safe, Effective Care Environment: Safety and Infection Control

29. When picking up the first sterile glove, the nurse will:
a. grasp the cuff with the thumb and fingers.
b. insert fingers into the opening and pull the glove on while holding the cuff.
c. slip a thumb in the opening and grasp the glove between the thumb and fingers.
d. leave the glove on a flat surface and work the fingers into the opening.

Downloaded by: SUCCEEDGRADES | abbieclin@gmail.com


Distribution
Downloaded by Dallen Mae D.ofKadir
this document is illegal
(kadir.dallenmae.d.bcsi@gmail.com)
lOMoARcPSD|35346190

Stuvia.com - The Marketplace to Buy and Sell your Study Material

ANS: C
The first glove is grasped by slipping the thumb in the opening and grasping the cuff with
the fingers.

DIF: Cognitive Level: Application REF: p. 263|Skill 17-4


OBJ: Theory #5 TOP: Prevention of Infection
KEY: Nursing Process Step: Implementation MSC: NCLEX: N/A

30. A nurse is instructing a nursing student about principles of aseptic technique. The nurse
would recognize the need for further instruction if the nursing student states, ―I must:
a. avoid coughing, sneezing, or unnecessary talking near or over a sterile field.‖
b. avoid reaching across or above a sterile field with my bare hands or arms.‖
c. open the wrapper of a sterile pack toward my body, the proximal flap first.‖
d. keep my sterile gloved hands in sight, away from all unsterile objects.‖
ANS: C
Principles of aseptic technique include avoiding coughing, sneezing, or unnecessary talking
near or over a sterile field. They also include avoiding reaching across or above a sterile
field with bare hands or arms or with other nonsterile items. The wrapper of a sterile pack
must be opened away from the body, the distal flap first, the lateral flaps next, and the
proximal flap toward the body last, thus making it unnecessary to reach over the sterile
field. When wearing sterile gloves, hands must be kept in sight away from all unsterile
objects, and above the waist level.

DIF: Cognitive Level: Application REF: p. 255|Box 17-5


OBJ: Theory #92 TOP: Prevention of Infection
KEY: Nursing Process Step: Evaluation
MSC: NCLEX: Safe, Effective Care Environment: Safety and Infection Control

31. A nurse caring for a ventilator-dependent patient will incorporate which of the following
best practices recommended by the IHI into the care plan, to reduce the incidence of health
acquired infections (HAIs)?
a. Continue to keep patient sedated to reduce anxiety.
b. Instill eye drops to reduce dryness.
c. Administer medication to reduce the likelihood of peptic ulcer disease.
d. Elevate the head of the bed 15 degrees to prevent pneumonia.
ANS: C
The Institute for Healthcare Improvement (IHI) developed a VAP bundle based on research
and Best Practices. According to the IHI (2012), health care providers can decrease the
incidence of VAP by elevating the head of the patient‘s bed between 30 and 45 degrees,
unless contraindicated, daily ―sedation vacation‖ (reducing sedation) and assessing the
patient‘s ability to breathe independently and allow mechanical ventilation to be
discontinued as soon as possible, prophylaxis (medication) for preventing peptic ulcer
disease (PUD), prophylaxis (medication) for preventing deep vein thrombosis (DVT), daily
oral care using chlorhexidine, an antimicrobial oral rinse.

DIF: Cognitive Level: Analysis REF: p. 243 OBJ: Theory #2


TOP: Health Care–Associated Infections (HAIs)

Downloaded by: SUCCEEDGRADES | abbieclin@gmail.com


Distribution
Downloaded by Dallen Mae D.ofKadir
this document is illegal
(kadir.dallenmae.d.bcsi@gmail.com)
lOMoARcPSD|35346190

Stuvia.com - The Marketplace to Buy and Sell your Study Material

KEY: Nursing Process Step: N/A MSC: NCLEX: N/A

MULTIPLE RESPONSE

1. Differences between Airborne and Droplet Precautions include which of the following?
(Select all that apply.)
a. A surgical mask must be worn with Airborne Precautions.
b. Patients on Droplet Precautions do not need to wear a surgical mask when outside
their rooms.
c. A surgical mask must be worn if working within 3 feet of a patient on Droplet
Precautions.
d. Airborne Precautions include placing the patient in a negative pressure room.
e. A specialized respirator mask must be worn with patients on Droplet Precautions.
f. Standard Precautions need to be used only with patients on Airborne Precautions.
ANS: C, D, E
The Centers for Disease Control and Prevention (CDC) recommends the use of specialized
respirator masks when working with patients on Airborne Precautions and a surgical mask
when working within 3 feet of a patient on Droplet Precautions. Both Airborne and Droplet
Precautions include the use of a private room; however, a negative pressure room is required
for Airborne Precautions. Standard Precautions are to be used with all patients.

DIF: Cognitive Level: Comprehension REF: p. 252|Box 17-3


OBJ: Theory #4 TOP: Infection Control
KEY: Nursing Process Step: N/A
MSC: NCLEX: Safe, Effective Care Environment

2. A nurse is caring for a patient in protective isolation for extreme immunosuppression.


Before entering the room, which of the following actions should the nurse take? (Select all
that apply.)
a. Don a gown.
b. Don a mask.
c. Put on gloves.
d. Apply a head covering.
e. Apply shoe coverings.
f. Bring in the blood pressure cuff and stethoscope.
ANS: A, B, C
A gown, mask, and gloves are all that is required; however, the CDC or hospital policies
may require full PPE protection.

DIF: Cognitive Level: Application REF: p. 245|Box 17-1


OBJ: Clinical Practice #2 TOP: Infection Control
KEY: Nursing Process Step: Planning
MSC: NCLEX: Safe, Effective Care Environment: Safety and Infection Control

3. The nurse clarifies that the duties of the facility‘s infection preventionist include: (Select all
that apply.)
a. viewing every culture that is performed in the facility that is positive for pathogens.

Downloaded by: SUCCEEDGRADES | abbieclin@gmail.com


Distribution
Downloaded by Dallen Mae D.ofKadir
this document is illegal
(kadir.dallenmae.d.bcsi@gmail.com)
lOMoARcPSD|35346190

Stuvia.com - The Marketplace to Buy and Sell your Study Material

b. investigating possible causes for the occurrence of health care–associated infections


(HAIs).
c. sanitizing isolation rooms after patients have been discharged.
d. counseling persons who have been found to be careless about infection control
protocols.
e. providing education to health care staff relative to infection control.
ANS: A, B, D, E
The infection preventionist (IP) follows up on all positive cultures, investigates possible
causes of HAIs, and provides ongoing education to staff. Counseling and sanitizing are not
directly the responsibility of the IP, although he or she might suggest counseling and
supervise sanitizing procedures.

DIF: Cognitive Level: Application REF: p. 243 OBJ: Theory #2


TOP: Health Care–Associated Infections KEY: Nursing Process Step: Assessment
MSC: NCLEX: Safe, Effective Care Environment: Safety and Infection Control

COMPLETION

1. A patient had abdominal surgery 3 days ago and now has a temperature of 101.2° F and
reports feelings of malaise. The nurse assesses the abdominal incision and observes edema
around the incision and some purulent drainage. This patient is in the stage
of infection.

ANS:
illness

The illness period of infection is defined by the presence of systemic signs and symptoms
such as fever, headache, malaise, rash, swelling, leukocytosis, purulent wound exudate,
diarrhea, and vomiting.

DIF: Cognitive Level: Analysis REF: p. 243 OBJ: Theory #1


TOP: Stages of Infection KEY: Nursing Process Step: Assessment
MSC: NCLEX: Physiological Integrity

2. The most contagious stage of infection is the period.

ANS:
prodromal

Although the prodromal period is short, it is the most contagious. People often do not realize
they are ill, and precautions may not be taken, which increases the risk of transmission.

DIF: Cognitive Level: Comprehension REF: p. 243 OBJ: Theory #1


TOP: Infection Control KEY: Nursing Process Step: N/A
MSC: NCLEX: Physiological Integrity

Downloaded by: SUCCEEDGRADES | abbieclin@gmail.com


Distribution
Downloaded by Dallen Mae D.ofKadir
this document is illegal
(kadir.dallenmae.d.bcsi@gmail.com)
lOMoARcPSD|35346190

Stuvia.com - The Marketplace to Buy and Sell your Study Material

Chapter 18: Lifting, Moving, and Positioning Patients


Test Bank

MULTIPLE CHOICE

1. The nurse uses professional knowledge about body mechanics to prevent the most common
occupational disorder in nurses, which is:
a. carpal tunnel syndrome from use of computer keyboards in nursing documentation.
b. shoulder and elbow injuries from moving patients.
c. knee injuries from standing for long periods.
d. back injuries from lifting and twisting.
ANS: D
Back injuries are the most common injury in health care workers, and in many cases, they are
preventable through use of proper body mechanics.

DIF: Cognitive Level: Knowledge REF: p. 259 OBJ: Theory #2


TOP: Body Mechanics KEY: Nursing Process Step: Planning
MSC: NCLEX: Safe Effective Care Environment: safety and infection control

2. The nurse assisting a weak patient from a bed to the wheelchair to go to physical therapy
would:
a. seat the patient on the side of the bed with feet touching the floor.
b. place hands under the patient‘s elbows to assist in rising.
c. lock knees as the patient is lowered to the chair.
d. assist the patient to don a robe after being seated in the wheelchair.
ANS: A
After locking the wheels of the wheelchair, seat the patient on the side of the bed with the feet
touching the floor.

DIF: Cognitive Level: Application REF: p. 276, Skill 18-4


OBJ: Clinical Practice #1 TOP: Patient Transfers
KEY: Nursing Process Step: Implementation
MSC: NCLEX: Safe Effective Care Environment

3. A frail older patient is able to stand but not to ambulate. She has an order to be up in a
wheelchair as desired during the day. A safe and appropriate way to assist her up to a chair is
to:
a. use a mechanical lift to transfer her from the bed to a chair.
b. assist her to stand and pivot to a chair at right angles to the bed, using a transfer
belt.
c. have another staff member help lift her out of bed to the chair on the count of three.
d. place a chair close to the bed and use a roller board to slide her into it.
ANS: B
A patient who can stand can safely be assisted to pivot and transfer with the use of a transfer
belt. This benefits the patient (active exercise) and is safe for both the nurse and the patient.

DIF: Cognitive Level: Application REF: p. 276, Skill 18-4


OBJ: Theory #5 TOP: Patient Transfers

https://www.coursehero.com/file/19280177/c18/

Downloaded by: SUCCEEDGRADES | abbieclin@gmail.com


Distribution
Downloaded by Dallen Mae D.ofKadir
this document is illegal
(kadir.dallenmae.d.bcsi@gmail.com)
lOMoARcPSD|35346190

Stuvia.com - The Marketplace to Buy and Sell your Study Material

KEY: Nursing Process Step: Implementation


MSC: NCLEX: Safe Effective Care Environment: safety and infection control

4. The charge nurse on the night shift of a skilled nursing facility is orienting a new aide to the
unit. The LPN‘s most accurate information relative to moving patients is:
a. ―Most of your assigned patients are able to move about a little. Don‘t wake them to
change their positions in bed if they are sleeping.‖
b. ―When you get Mrs. S up to the toilet, be sure to keep your feet together and your
knees locked, or she will pull you over.‖
c. ―Get one other aide to help and use the mechanical lift when you get Mr. A out of
bed in the morning. He is heavy and doesn‘t assist at all.‖
d. ―Use your back muscles to lift—that will strengthen them and make it easier for
you to lift or move heavy patients.‖
ANS: C
Getting adequate assistance and using mechanical assistance are important to reduce injury to
staff and patients. It also increases the comfort of the move for the patient.

DIF: Cognitive Level: Application REF: p. 260, Box 18-4


OBJ: Theory #3 TOP: Patient Transfers
KEY: Nursing Process Step: Implementation
MSC: NCLEX: Safe Effective Care Environment: safety and infection control

5. The patient for whom passive range-of-motion exercises would be most beneficial would be
the:
a. 66-year-old patient with loss of mobility related to a recent cerebrovascular
accident (CVA).
b. 72-year-old patient with chronic dementia who alternately sits in his wheelchair
and wanders around the unit.
c. 80-year-old patient with chronic lung disease who can breathe only when he is
sitting in a tripod position.
d. 94-year-old patient with increasing fatigue and weight loss who needs assistance to
ambulate.
ANS: A
A patient with a recent CVA is unable to independently change position or move the affected
side. The patient may regain use of motor functions lost, so it is very important to prevent loss
of muscle strength, contractures, and pressure ulcers.

DIF: Cognitive Level: Analysis REF: p. 272, Skill 18-3


OBJ: Theory #2 TOP: Patient Positioning
KEY: Nursing Process Step: Assessment
MSC: NCLEX: Physiological Integrity: basic care and comfort

6. An emaciated semiconscious bed-bound patient does not remain in a side-lying position and
repeatedly turns onto her back, where she is developing a pressure area over her sacrum. The
nurse should add to the nursing care plan to:
a. raise the knees to keep the patient from sliding down.
b. position the patient on her side and use protective wrist and vest devices to keep her
from turning onto her back.
c. assist the patient to sit in a wheelchair for short periods before returning her to bed.

https://www.coursehero.com/file/19280177/c18/

Downloaded by: SUCCEEDGRADES | abbieclin@gmail.com


Distribution
Downloaded by Dallen Mae D.ofKadir
this document is illegal
(kadir.dallenmae.d.bcsi@gmail.com)
lOMoARcPSD|35346190

Stuvia.com - The Marketplace to Buy and Sell your Study Material

d. place the patient on her stomach (prone position) using a small pillow below her
diaphragm.
ANS: D
The prone position is an excellent (but underused) position to take pressure off the sacral area.
Raising the head and the knees of the patient interferes with venous return from the legs and
puts a great deal of pressure on the sacrum. Use of a wheelchair for a semiconscious patient is
not effective.

DIF: Cognitive Level: Application REF: p. 264 OBJ: Theory #2


TOP: Patient Positioning KEY: Nursing Process Step: Implementation
MSC: NCLEX: Physiological Integrity: basic care and comfort

7. To place a patient in the Sims‘ or lateral-lying position, the nurse would initially:
a. raise the head of the bed to a 45- or 60-degree angle.
b. raise the bed to a waist-high working level.
c. bring the patient to the edge of the bed so that she will be centered when turned on
her side.
d. place a pillow behind the patient‘s back to support her and prevent her from rolling
onto her back.
ANS: B
A waist-high bed height is a comfortable and safe working height for the nurse and also
prevents staff back injuries. The head is not raised in a side-lying position; it is in a Fowler‘s
or semi-Fowler‘s position.

DIF: Cognitive Level: Application REF: p. 265, Skill 18-1


OBJ: Clinical Practice #1 TOP: Body Mechanics
KEY: Nursing Process Step: Implementation
MSC: NCLEX: Safe Effective Care Environment: safety and infection control

8. To provide correct body alignment for a physically immobile patient in bed in the supine
position, the nurse:
a. uses trochanter rolls between the patient‘s legs to prevent inward rotation.
b. places a large pillow behind the patient‘s head and neck to hyperflex the neck.
c. raises the head and knees to maintain as much flexion of the hips and knees as
possible.
d. uses a footboard or places high-top sneakers on the patient‘s feet to maintain
dorsiflexion.
ANS: D
The use of high-top sneakers (or a footboard) prevents footdrop and maintains dorsiflexion.

DIF: Cognitive Level: Application REF: p. 264 OBJ: Theory #2


TOP: Patient Positioning KEY: Nursing Process Step: Implementation
MSC: NCLEX: Physiological Integrity: basic care and comfort

9. A nurse and an assistant are preparing to get a patient out of bed for the first time after a week
of bed rest. They begin by having the patient dangle on the edge of the bed. The nurse should:
a. allow the patient to dangle for 10 to 15 minutes and then transfer her to a nearby
chair.
b. perform passive range-of-motion exercises on the patient‘s arms and legs while she

https://www.coursehero.com/file/19280177/c18/

Downloaded by: SUCCEEDGRADES | abbieclin@gmail.com


Distribution
Downloaded by Dallen Mae D.ofKadir
this document is illegal
(kadir.dallenmae.d.bcsi@gmail.com)
lOMoARcPSD|35346190

Stuvia.com - The Marketplace to Buy and Sell your Study Material

is dangling to improve circulation.


c. assess the patient‘s response to the changed position, looking for orthostatic
hypotension, nausea, or dizziness before proceeding.
d. dangle the patient only momentarily and then assist her to ambulate as far as she is
able.
ANS: C
A patient who has been immobilized for any length of time may feel dizzy or experience a
drop in blood pressure when sitting or standing for the first time. Therefore the nurse must
assess the patient carefully to determine whether transfer to a chair, ambulation, or return to
bed is indicated.

DIF: Cognitive Level: Analysis REF: p. 276 OBJ: Clinical Practice #2


TOP: Sitting up on Side of Bed KEY: Nursing Process Step: Assessment
MSC: NCLEX: Physiological Integrity: basic care and comfort

10. The nurse caring for a patient with a nursing diagnosis of Injury, risk for, related to right-sided
weakness as evidenced by unsteady gait, would accommodate the patient by:
a. keeping the right arm in a sling to prevent injury.
b. keeping bed rails up to prevent the patient from attempting to get up unassisted.
c. placing the wheelchair on the left side of the patient before transfer.
d. allowing unassisted ambulation with the support of a walker.
ANS: C
Placing the wheelchair on the patient‘s stronger side aids in transfer.

DIF: Cognitive Level: Application REF: p. 276, Skill 18-4


OBJ: Theory #5 TOP: Patient Transfers
KEY: Nursing Process Step: Implementation
MSC: NCLEX: Physiological Integrity: basic care and comfort

11. The nurse explains to the unlicensed assistive personnel (UAP) that a shearing force is applied
to the patient when:
a. a lifting sheet is used to move the patient to a stretcher.
b. the patient is pulled up in bed without being lifted.
c. the patient is seated in a wheelchair without a pressure cushion.
d. the patient is left in the supine position.
ANS: B
When a patient is pulled up in bed without being lifted up first, shearing force is applied on
the bony prominences and tissues of the back, which predisposes the patient to a pressure
ulcer.

DIF: Cognitive Level: Comprehension REF: p. 262 OBJ: Theory #3


TOP: Positioning KEY: Nursing Process Step: Implementation
MSC: NCLEX: Physiological Integrity: basic care and comfort

12. A patient who has had spinal surgery is not permitted to bend at the waist or to sit in a chair.
To position the patient correctly in bed, the nurse:
a. places her in low- or semi-Fowler‘s position only.
b. uses logrolling to accomplish position changes from side to side.
c. moves the top half of her body first, then the middle, and finally her legs.

https://www.coursehero.com/file/19280177/c18/

Downloaded by: SUCCEEDGRADES | abbieclin@gmail.com


Distribution
Downloaded by Dallen Mae D.ofKadir
this document is illegal
(kadir.dallenmae.d.bcsi@gmail.com)
lOMoARcPSD|35346190

Stuvia.com - The Marketplace to Buy and Sell your Study Material

d. keeps her in a prone position to keep pressure off her back.


ANS: B
Logrolling, or moving the patient‘s body as one unit, is used after back surgery or trauma or
when twisting or flexion must be avoided. Logrolling is accomplished using a sheet and at
least two persons.

DIF: Cognitive Level: Application REF: p. 268 OBJ: Theory #3


TOP: Positioning KEY: Nursing Process Step: Implementation
MSC: NCLEX: Physiological Integrity: basic care and comfort

13. A patient in the skilled nursing facility has left-sided paralysis from a stroke several years
before, as well as generalized weakness. The nurse should ensure that which of the following
devices is in place to prevent flexion contractures?
a. A trochanter roll to keep her legs from turning outward
b. A rolled washcloth in the palm of her left hand or a hand splint
c. A protective vest to keep her sitting upright in the chair
d. A trapeze to permit her to change her position in bed more easily
ANS: B
A hand splint or rolled cloth in the palm of her hand (along with range-of-motion exercises)
will help prevent flexion contractures of her hand. A trochanter roll prevents outward rotation,
not flexion.

DIF: Cognitive Level: Application REF: p. 265, Skill 18-1


OBJ: Theory #3 TOP: Positioning KEY: Nursing Process Step: Implementation
MSC: NCLEX: Physiological Integrity: basic care and comfort

14. When the post-stroke patient complains to the nurse, ―I don‘t see why you are wasting your
time doing the passive range-of-motion exercises on my legs,‖ the nurse‘s most informative
response would be based on the knowledge that the exercises:
a. guarantee the prevention of pressure ulcers.
b. are part of the basic care given to all patients.
c. prevent contractures of the hips.
d. maintain the muscle mass of the limb prior to the stroke.

ANS: C
Passive range-of-motion (ROM) exercises, although not part of care given to all patients, does
prevent contractures in persons who are bedfast. ROM does not guarantee the prevention of
pressure ulcers but helps in the improved circulation of the limbs.

DIF: Cognitive Level: Application REF: p. 272, Skill 18-3


OBJ: Clinical Practice #3 TOP: Effects of ROM
KEY: Nursing Process Step: Implementation
MSC: NCLEX: Physiological Integrity: physiological adaptation

15. While the nurse is assisting a patient to ambulate, the patient suddenly says, ―I‘m dizzy. I can‘t
stand up.‖ As the patient begins to fall, the nurse should:
a. tell the patient, ―Look up, take some deep breaths, and stand up straight. You can
do it.‖
b. call for another nurse or aide to get a wheelchair to return the patient to her room
via wheelchair.

https://www.coursehero.com/file/19280177/c18/

Downloaded by: SUCCEEDGRADES | abbieclin@gmail.com


Distribution
Downloaded by Dallen Mae D.ofKadir
this document is illegal
(kadir.dallenmae.d.bcsi@gmail.com)
lOMoARcPSD|35346190

Stuvia.com - The Marketplace to Buy and Sell your Study Material

c. step behind the patient, grasp her around the waist or chest, and slide her down his
leg gently to the floor.
d. look for the nearest chair and assist the patient to it.
ANS: C
A patient who is threatening to fall needs to be lowered to the floor to avoid injury from a fall
by allowing the patient to gently slide down the nurse‘s leg to the floor.

DIF: Cognitive Level: Analysis REF: p. 282, Skill 18-6


OBJ: Clinical Practice #6 TOP: Patient Transfers
KEY: Nursing Process Step: Implementation
MSC: NCLEX: Physiological Integrity: reduction of risk

16. A patient who is weak from inactivity following a car accident benefits most if the nurse
provides for:
a. passive range-of-motion (ROM) exercises to all joints four times a day.
b. active ROM exercises to arms and legs several times a day.
c. active ROM exercises with weights twice a day with 20 repetitions each.
d. passive ROM exercises to the point of resistance or pain and then slightly beyond.
ANS: B
Active ROM is best to restore strength in a weak patient who can independently perform
activities of daily living but is immobilized because of injury.

DIF: Cognitive Level: Application REF: p. 271 OBJ: Clinical Practice #3


TOP: Active Range of Motion KEY: Nursing Process Step: Implementation
MSC: NCLEX: Physiological Integrity: physiological adaptation

17. A nurse is ambulating an unsteady patient from the bed to a chair in the patient‘s home. To do
so safely, the nurse applies a gait belt and:
a. slides his hand from the bottom under the gait belt at the middle of the patient‘s
back.
b. grasps the gait belt from the top at the middle of the patient‘s back, pulling it tight
against the patient‘s abdomen.
c. has one person on each side grasp the belt from the top.
d. secures a regular man‘s belt snugly around the patient‘s waist to use if the patient
starts to fall.
ANS: A
The nurse puts his hand from the bottom at the rear, so he can pull up if the patient starts to
fall and not lose the grip on the gait belt. The gait belt should be tight enough to secure the
patient, but loose enough for the passage of the nurse‘s hand.

DIF: Cognitive Level: Application REF: p. 279


OBJ: Theory #5 | Clinical Practice #6 TOP: Assisted Ambulation
KEY: Nursing Process Step: Implementation
MSC: NCLEX: Physiological Integrity: reduction of risk

18. An example of the principles of good body mechanics applied to patient care occurs when the
nurse:
a. keeps his feet fixed, spread one in front of the other, and turns his upper body to
move the patient up in bed with a rocking movement.

https://www.coursehero.com/file/19280177/c18/

Downloaded by: SUCCEEDGRADES | abbieclin@gmail.com


Distribution
Downloaded by Dallen Mae D.ofKadir
this document is illegal
(kadir.dallenmae.d.bcsi@gmail.com)
lOMoARcPSD|35346190

Stuvia.com - The Marketplace to Buy and Sell your Study Material

b. assists another nurse in pushing a patient from one side of the bed to the other.
c. bends at the waist to pick up and empty or move the urinary drainage bag attached
to the lower end of the side rail.
d. works at arm‘s distance from the patient when lifting or transferring the patient.
ANS: A
Fixing feet and placing one foot in front of the other and facing the direction of the movement
will ease the work of moving a patient up in bed. Pulling requires less effort than pushing in
this scenario. Twisting should be avoided; nurses should use leg muscles rather than back
muscles to pick up objects from the floor. Work should be close to the body to reduce effort
and strain.

DIF: Cognitive Level: Application REF: p. 265, Skill 18-1


OBJ: Theory #2 TOP: Body Mechanics
KEY: Nursing Process Step: Implementation
MSC: NCLEX: Safe Effective Care Environment: safety and infection control

19. A nurse is instructing one of the facility‘s unlicensed assistive personnel (UAPs) regarding
body mechanics for moving and lifting. The nurse recognizes that further instruction is
warranted when the UAP states, ―I will:
a. lift using my back muscles.‖
b. obtain help whenever possible.‖
c. ask the patient to help if able.‖
d. use a wide base of support.‖
ANS: A
Guidelines for moving and lifting include obtaining help whenever possible; asking the
patient to help if able; using thigh, arm, or leg muscles rather than back muscles; and using a
wide base of support.

DIF: Cognitive Level: Comprehension REF: p. 265, Skill 18-1


OBJ: Theory #2 TOP: Body Mechanics
KEY: Nursing Process Step: Evaluation
MSC: NCLEX: Safe Effective Care Environment: safety and infection control

20. A physician orders the nurse to place a patient in Fowler‘s position. The nurse should elevate
the head of the patient‘s bed degrees.
a. 60 to 90
b. 30 to 60
c. 15 to 30
d. 10 to 15

ANS: A
Fowler‘s position is arranged by elevating the head of the bed 60 to 90 degrees. Semi-
Fowler‘s position is an elevation of 30 to 60 degrees, and low-Fowler‘s is an elevation of 15
to 30 degrees. Unless contraindicated, the knees can be raised 10 to 15 degrees in these
positions.

DIF: Cognitive Level: Comprehension REF: p. 265, Skill 18-1


OBJ: Clinical Practice #1 TOP: Patient Positioning
KEY: Nursing Process Step: Implementation
MSC: NCLEX: Physiological Integrity: basic care and comfort

https://www.coursehero.com/file/19280177/c18/

Downloaded by: SUCCEEDGRADES | abbieclin@gmail.com


Distribution
Downloaded by Dallen Mae D.ofKadir
this document is illegal
(kadir.dallenmae.d.bcsi@gmail.com)
lOMoARcPSD|35346190

Stuvia.com - The Marketplace to Buy and Sell your Study Material

21. As the nurse is helping an 85-year-old man to stand and ambulate, he complains that he feels
that he has lost all of his strength in the last several years and cannot do the things he could do
when he was 80. The nurse‘s most informative response would be:
a. ―An increase in testosterone will build your muscle bulk back to where it was when
you were younger.‖
b. ―As we age our muscle cells are lost and replaced by fat, which leads to loss of
strength.‖
c. ―Inactivity makes our muscles ‗lazy‘ and they just won‘t do the work they used to
do.‖
d. ―Additional vitamins will build your strength back up in a few months.‖
ANS: B
Fat replaces muscle cells, which leads to loss of strength and stamina.

DIF: Cognitive Level: Application REF: p. 259 OBJ: Clinical Practice #1


TOP: Patient Positioning KEY: Nursing Process Step: Implementation
MSC: NCLEX: Physiological Integrity: physiological adaptation

COMPLETION

22. The primary function of a joint is to provide to the skeleton.

ANS:
movement
Ligaments and tendons attach to bones at the joints, which allows movement.

DIF: Cognitive Level: Comprehension REF: p. 259 OBJ: Theory #1


TOP: Structure and Function of Musculoskeletal System KEY: Nursing Process Step: N/A
MSC: NCLEX: Physiological Integrity

23. There are two main factors in the development of pressure ulcers. One is pressure and the
other is .

ANS:
shearing force
Shearing is the applied force that causes a downward and forward pressure on the tissues
beneath the skin. Examples include pulling sheets or clothing from underneath the patient and
the force applied when a patient pushes down on the bed with her heels while trying to move
up in bed.

DIF: Cognitive Level: Knowledge REF: p. 262 OBJ: Theory #2


TOP: Complications KEY: Nursing Process Step: N/A
MSC: NCLEX: Physiological Integrity

24. The nurse reminds a patient that one of the anatomic parts of a joint that allows the joint to
move freely is the fluid-filled .

ANS:
bursa

https://www.coursehero.com/file/19280177/c18/

Downloaded by: SUCCEEDGRADES | abbieclin@gmail.com


Distribution
Downloaded by Dallen Mae D.ofKadir
this document is illegal
(kadir.dallenmae.d.bcsi@gmail.com)
lOMoARcPSD|35346190

Stuvia.com - The Marketplace to Buy and Sell your Study Material

Bursa are small fluid-filled sacs that provide a cushion at friction points and provide freely
movable joints.

DIF: Cognitive Level: Knowledge REF: p. 259 OBJ: Theory #1


TOP: Bursa KEY: Nursing Process Step: Implementation
MSC: NCLEX: Physiological Integrity: physiological adaptation

MULTIPLE RESPONSE

25. Complications from incorrect alignment and positioning include which of the following?
(Select all that apply.)
a. Pressure ulcers
b. Osteoporosis
c. Contractures
d. Increased blood pressure
e. Fluid in the lungs
f. Elevated temperature
ANS: A, C, E
Constant pressure on the skin, especially on bony prominences, interferes with circulation,
causing pressure ulcers. Contractures occur when joints are not positioned frequently, and
fluid can accumulate in the lungs with infrequent positioning. Osteoporosis, increased blood
pressure, and elevated temperature are not results of improper alignment or positioning.

DIF: Cognitive Level: Comprehension REF: p. 262 OBJ: Theory #2


TOP: Patient Positioning KEY: Nursing Process Step: Assessment
MSC: NCLEX: Physiological Integrity: reduction of risk

26. A 70-year-old immobile patient, who has right-sided weakness caused by a recent stroke,
weighs approximately 250 pounds and needs to be moved up in bed. Which of the following
actions should the nurse take? (Select all that apply.)
a. Summon at least one other person to assist.
b. Obtain a mechanical lift.
c. Perform the move by himself, because it should not be too difficult.
d. Obtain a lift sheet.
e. Put the bed in semi-Fowler‘s position.
f. Place the patient flat on her back.

ANS: A, D, F
The patient‘s increased weight and inability to assist requires at least two people to move her
up in bed. A lift sheet enables the patient to be moved. Placing the patient on her back
decreases gravitational pull, making the move easier. A mechanical lift is used to transfer a
patient, not to move her up in bed.

DIF: Cognitive Level: Application REF: p. 269, Skill 18-2


OBJ: Clinical Practice #3 TOP: Patient Moving
KEY: Nursing Process Step: Implementation
MSC: NCLEX: Safe Effective Care Environment: safety and infection control

https://www.coursehero.com/file/19280177/c18/

Downloaded by: SUCCEEDGRADES | abbieclin@gmail.com


Distribution
Downloaded by Dallen Mae D.ofKadir
this document is illegal
(kadir.dallenmae.d.bcsi@gmail.com)
lOMoARcPSD|35346190

Stuvia.com - The Marketplace to Buy and Sell your Study Material

Chapter 19: Assisting with Hygiene, Personal Care, Skin Care, and the Prevention of
Pressure Ulcers
Test Bank

MULTIPLE CHOICE

1. The nurse instructs the patient that any injury to the skin initially puts the patient at risk for:
a. scar formation at the injury site resulting from the healing process.
b. infection with bacteria or viruses that may affect the person systemically.
c. loss of sensation caused by damage to the nerves in the area.
d. loss of body fluids and an upset in the fluid and electrolyte balance.
ANS: B
The skin (and intact mucous membrane) is the first line of defense against invasion by
pathogens, and any cut or abrasion can be an entry site. Scar formation, nerve damage, and
fluid/electrolyte disturbance are likely only when there is a large or deep wound.

DIF: Cognitive Level: Comprehension REF: p. 288 OBJ: Theory #1


TOP: Skin Integrity KEY: Nursing Process Step: Assessment
MSC: NCLEX: Physiological Integrity: physiological adaptation

2. When the patient returns from the physical therapy department, he is diaphoretic and his skin
is flushed but cool. Nursing intervention in this situation should be for the nurse to:
a. call his physician about the amount of exertion in physical therapy.
b. suggest the patient walk slowly in the hall to ―cool down.‖
c. offer additional fluids to replace those lost through normal cooling.
d. place a light cover over the patient to prevent his chilling.
ANS: C
Diaphoresis (sweating) is the body‘s normal response to rid itself of heat. Drinking fluids to
replace those lost prevents dehydration.

DIF: Cognitive Level: Application REF: p. 288 OBJ: Theory #1


TOP: Fluid Replacement KEY: Nursing Process Step: Assessment
MSC: NCLEX: Physiological Integrity: basic care and comfort

3. During an admission assessment to a skilled care facility, the nurse notes that a 76-year-old
man is thin and unsteady on his feet and has dry flaky skin on his arms and legs. An
appropriate hygiene goal for this patient is that the:
a. patient will shower daily on an independent basis by the end of 1 month.
b. nurse will give a tub bath or full bed bath daily.
c. patient will shower or tub bathe with assistance twice a week.
d. patient will tub bathe or shower with assistance daily.
ANS: C
Elderly people have decreased sweat and sebaceous gland activity and do not need a full bath
or shower daily. Their skin is thinner and it becomes drier and itchy with overly frequent
bathing. Because of the patient‘s unsteadiness, it is not safe to have him shower alone.

DIF: Cognitive Level: Application REF: p. 293, Elder Care


OBJ: Theory #6 TOP: Hygiene and Safety

Downloaded by: SUCCEEDGRADES | abbieclin@gmail.com


Distribution
Downloaded by Dallen Mae D.ofKadir
this document is illegal
(kadir.dallenmae.d.bcsi@gmail.com)
lOMoARcPSD|35346190

Stuvia.com - The Marketplace to Buy and Sell your Study Material

KEY: Nursing Process Step: Planning


MSC: NCLEX: Physiological Integrity: basic care and comfort

4. In assessing the skin condition of an elderly patient, the nurse notes that, over the sacral area,
there is a 2-cm 3-cm area that is reddened, does not blanch around the perimeter, and is
open at the center. The most effective documentation would be:
a. ―Patient has stage II ulcer on sacrum. No blanching of perimeter.‖
b. ―Reddened area over sacrum, skin open in center.‖
c. ―Pressure ulcer on sacrum. Massaged with no improvement in color.‖
d. ―2-cm 3-cm reddened area on sacrum with open center. Does not blanch.‖
ANS: D
Description of a pressure ulcer should be specific and give a visual picture of the area. Such
documentation will be useful in calculating the Medicare reimbursement for the facility.

DIF: Cognitive Level: Application REF: p. 292, Elder Care


OBJ: Clinical Practice #2 TOP: Pressure Ulcers
KEY: Nursing Process Step: Assessment
MSC: NCLEX: Physiological Integrity: basic care and comfort

5. When instructing a nursing assistant about hygiene needs of a frail elderly patient, the nurse
correctly educates the nursing assistant to:
a. ―Use warm, not hot, water and be sure the room is warm beforehand to avoid
chilling.‖
b. ―Put bath oil in the tub and use plenty of soap to really clean the patient‘s skin
while she is in the tub.‖
c. ―Use brisk drying and an alcohol rub to close the patient‘s pores and prevent heat
loss after the bath.‖
d. ―Completely dry the patient‘s skin and apply a mild moisturizer.‖
ANS: A
Elderly people have drier, thinner skin and less subcutaneous fat. Therefore warm, not hot,
water is needed, and chilling should be avoided. The elderly should use less soap (to decrease
dryness), and the use of oils in the water can be hazardous. Elderly people should be patted,
not rubbed, dry and moisturizer should be applied to skin that is still damp.

DIF: Cognitive Level: Knowledge REF: p. 293, Elder Care


OBJ: Theory #6 TOP: Skin Care KEY: Nursing Process Step: Planning
MSC: NCLEX: Physiological Integrity: basic care and comfort

6. An important factor to consider when assessing the hygiene needs of a patient is that:
a. the patient knows best what is needed in his hygiene routine.
b. the routine of the agency will determine when the patient is able to bathe.
c. hygiene is not as important as other needs of the patient.
d. the patient may not have the same hygiene practices as the nurse.

ANS: D
Different cultures have different views of hygiene practices, such as use of deodorant,
shaving, or daily bathing. These needs are an important part of health and recovery from
illness.

DIF: Cognitive Level: Application REF: p. 289 OBJ: Theory #2

Downloaded by: SUCCEEDGRADES | abbieclin@gmail.com


Distribution
Downloaded by Dallen Mae D.ofKadir
this document is illegal
(kadir.dallenmae.d.bcsi@gmail.com)
lOMoARcPSD|35346190

Stuvia.com - The Marketplace to Buy and Sell your Study Material

TOP: Culture and Hygiene KEY: Nursing Process Step: Assessment


MSC: NCLEX: Psychosocial Integrity: psychosocial adaptation

7. What nursing interventions related to hygiene are appropriate for a patient who has had a
recent stroke that caused right-sided (dominant) paralysis and inability to speak?
a. Perform a full bed bath, brush and floss his teeth, and give him a good back
massage.
b. Encourage the patient to use his nondominant hand to wash his face, brush his
teeth, and perform other hygiene activities with assistance as necessary.
c. Set up a washbasin and supplies, tell the patient to wash what he can, and provide
privacy for the patient to do what he can.
d. Teach a family member to give a full bath so that the family member will be able to
care for the patient at home.
ANS: B
Patients should be encouraged to do as much of their hygiene as possible (and allowed) in
order to increase their sense of independence.

DIF: Cognitive Level: Application REF: p. 287 OBJ: Theory #2


TOP: Promoting Independence KEY: Nursing Process Step: Implementation
MSC: NCLEX: Physiological Integrity: basic care and comfort

8. The patient most at risk for a pressure ulcer would be:


a. a 46-year-old man in traction for a fractured femur, who exercised regularly before
his accident and is alert and oriented.
b. a 54-year-old overweight man who is unconscious from a stroke, has a urinary
catheter in place, and has been incontinent of liquid stool since a feeding tube was
placed.
c. a 72-year-old man admitted for elective surgery to replace his hip joint, who was an
avid bowler and gardener before his hip disease slowed him down.
d. an 84-year-old man with Alzheimer‘s disease who is pacing in the halls and who is
incontinent of urine if not toileted every 2 hours.
ANS: B
With risk factors of obesity, immobility, lowered mental awareness, and incontinence of stool,
this patient clearly is at greatest risk of developing a pressure ulcer.

DIF: Cognitive Level: Analysis REF: p. 289, Box 18-1


OBJ: Theory #3 TOP: Risks for Skin Breakdown
KEY: Nursing Process Step: Assessment MSC: NCLEX: Physiological Integrity: reduction of risk

9. Because the elderly patient lies curled up in a side-lying position most of the time, the nurse,
seeking to avoid a pressure ulcer, makes frequent assessments of the:
a. sacrum.
b. heels.
c. ilium.
d. scapula.
ANS: C
A patient who lies in a constant side-lying position puts pressure on the bony prominence of
the ilium. The sacrum, heels, and scapula are at risk in a patient who lies on his or her back.

Downloaded by: SUCCEEDGRADES | abbieclin@gmail.com


Distribution
Downloaded by Dallen Mae D.ofKadir
this document is illegal
(kadir.dallenmae.d.bcsi@gmail.com)
lOMoARcPSD|35346190

Stuvia.com - The Marketplace to Buy and Sell your Study Material

DIF: Cognitive Level: Application REF: p. 291 OBJ: Theory #3


TOP: Pressure Ulcers KEY: Nursing Process Step: Assessment
MSC: NCLEX: Physiological Integrity: reduction of risk

10. A patient has a quarter-sized blackened eschar on both heels surrounded by a 1- to 2-cm
indurated reddened area. The nurse is aware that these lesions are:
a. pressure ulcers that cannot be accurately staged because of the eschar.
b. stage I pressure ulcers because of the induration and redness.
c. stage II pressure ulcers because the skin has been broken.
d. stage III or IV pressure ulcers because of the eschar.
ANS: A
Eschar must be removed to accurately stage an ulcer, because the nurse cannot know how
deep the ulcer is.

DIF: Cognitive Level: Analysis REF: p. 291 OBJ: Clinical Practice #1


TOP: Pressure Ulcer Staging KEY: Nursing Process Step: Assessment
MSC: NCLEX: Physiological Integrity: basic care and comfort

11. A patient with a nursing diagnosis of Skin integrity, risk for impaired, is noted to have
reddened areas on his right shoulder and hip when he is repositioned on a 2-hour turning
schedule. The nurse should:
a. massage the areas vigorously to restore circulation to the pressured areas.
b. document that the patient has a stage I pressure ulcer of the right shoulder and hip.
c. not position the patient on the right side for at least 8 hours.
d. reassess the area after 30 to 45 minutes for reactive hyperemia.

ANS: D
Redness and nonblanching that remain after relief of pressure for 30 to 45 minutes are an
indication of a stage I pressure ulcer. Therefore the area needs to be reassessed before it is
labeled a stage I pressure ulcer.

DIF: Cognitive Level: Application REF: p. 291 OBJ: Clinical Practice #1


TOP: Pressure Ulcers KEY: Nursing Process Step: Evaluation
MSC: NCLEX: Physiological Integrity: basic care and comfort

12. To perform oral care for an unconscious patient, the nurse takes which action first?
a. Position the patient in an upright sitting position with the bed at a comfortable
working height for the nurse.
b. Raise the bed to a comfortable working height and position the patient in a flat
side-lying position.
c. Move the patient to the far edge of the bed with the head slightly elevated.
d. Lower the bed, lower both side rails, and turn the patient‘s head to one side.
ANS: B
The bed should be at a comfortable working level for the nurse. The patient should be in a flat
side-lying position to promote fluids draining from the mouth rather than running down the
back of the throat and possibly resulting in aspiration.

DIF: Cognitive Level: Application REF: p. 303, Skill 19-2


OBJ: Clinical Practice #3 TOP: Oral Care
KEY: Nursing Process Step: Implementation

Downloaded by: SUCCEEDGRADES | abbieclin@gmail.com


Distribution
Downloaded by Dallen Mae D.ofKadir
this document is illegal
(kadir.dallenmae.d.bcsi@gmail.com)
lOMoARcPSD|35346190

Stuvia.com - The Marketplace to Buy and Sell your Study Material

MSC: NCLEX: Physiological Integrity: basic care and comfort

13. The nurse assessing for a pressure ulcer in a patient with darkly pigmented skin should:
a. examine the area under full florescent light.
b. look for a purple hue under natural light.
c. reassess areas that appear lighter under a halogen light.
d. identify areas of a green hue under a halogen light.

ANS: B
Patients with darkly pigmented skin will show a purple coloration under natural light in the
beginning stages of a pressure ulcer.

DIF: Cognitive Level: Application REF: p. 289 OBJ: Theory #3


TOP: Assessing for Pressure Ulcers KEY: Nursing Process Step: Assessment
MSC: NCLEX: Physiological Integrity: basic care and comfort

14. A patient who has a dry, itchy dermatitis will most likely benefit from:
a. an oatmeal or starch therapeutic bath with tepid water.
b. having his skin patted with alcohol to decrease the itching.
c. a very warm whirlpool bath for 20 to 30 minutes.
d. avoiding any skin contact with water in the affected areas.
ANS: A
Oatmeal or starch baths are used to soothe dermatitis. Very hot water, soaps, perfumes, and
alcohol rubs are contraindicated. The skin must be kept clean, even if there is dermatitis
present, so although bathing may be decreased or modified, it is not eliminated.

DIF: Cognitive Level: Comprehension REF: p. 301 OBJ: Theory #5


TOP: Skin Care KEY: Nursing Process Step: Planning
MSC: NCLEX: Physiological Integrity: pharmacological therapies

15. A nurse is preparing to give a complete bath to an unconscious patient. After performing the
standard steps done before any procedure, the nurse:
a. washes each eye with a fresh area of the washcloth before washing the rest of the
patient‘s face.
b. wears protective gloves throughout the entire procedure.
c. begins with a back wash and rub to assess for pressure areas over the sacrum.
d. changes the water after washing the patient‘s face, and again after washing his
back.
ANS: A
The eyes should be washed without soap and before the water is soiled by face washing.
Separate areas of the washcloth prevent the transfer of organisms from one eye to the other.

DIF: Cognitive Level: Application REF: p. 294, Skill 19-1


OBJ: Clinical Practice #2 TOP: Bed Bath
KEY: Nursing Process Step: Implementation
MSC: NCLEX: Physiological Integrity: basic care and comfort

16. When providing perineal care for an uncircumcised male patient, the nurse:
a. provides perineal care the same as for a circumcised male.
b. ensures that the foreskin is retracted and the glans is exposed at the end of the

Downloaded by: SUCCEEDGRADES | abbieclin@gmail.com


Distribution
Downloaded by Dallen Mae D.ofKadir
this document is illegal
(kadir.dallenmae.d.bcsi@gmail.com)
lOMoARcPSD|35346190

Stuvia.com - The Marketplace to Buy and Sell your Study Material

procedure.
c. does not touch the glans during the procedure because it is very sensitive.
d. retracts the foreskin and then cleans the glans, being sure to replace it at the end of
the procedure.
ANS: D
In the uncircumcised male, the foreskin covers the glans and must be retracted to adequately
cleanse the secretions that accumulate under the foreskin and can lead to infection. The
foreskin must be pulled down to cover the glans after cleaning or it can swell and cause pain
and constriction of the glans.

DIF: Cognitive Level: Application REF: p. 294, Skill 19-1


OBJ: Clinical Practice #4 TOP: Bathing
KEY: Nursing Process Step: Implementation
MSC: NCLEX: Physiological Integrity: basic care and comfort

17. Providing oral care to a patient who has dentures includes:


a. asking the patient to place his teeth directly in a covered, labeled container for
overnight storage.
b. removing, cleaning, and storing the dentures in a labeled container at bedtime.
c. cleaning the dentures in hot water after each meal to remove debris and bacteria.
d. using a toothbrush and toothpaste to clean the dentures in the patient‘s mouth.
ANS: B
Dentures should be removed and cleaned before they are stored. Hot water should never be
used. Dentures may be cleaned in the patient‘s mouth, but they need to be removed to clean
the patient‘s palate and gums, as well as the undersides of the dentures.

DIF: Cognitive Level: Application REF: p. 304, Skill 19-3


OBJ: Clinical Practice #4 TOP: Oral Care
KEY: Nursing Process Step: Implementation
MSC: NCLEX: Physiological Integrity: basic care and comfort

18. The nurse caring for a patient who is not taking any food or fluids by mouth because he is
unconscious is aware that the patient:
a. does not need mouth care as frequently as the patient who is eating and drinking.
b. should have complete mouth care once a day when the nurse assesses the condition
of his skin and mucous membranes.
c. needs to have his mouth swabbed to moisten and remove secretions every 4 hours.
d. should have his lips lubricated and his teeth brushed with mouthwash once a shift.

ANS: C
An unconscious patient needs mouth care about every 4 hours to prevent drying of secretions,
halitosis, and possible blocking of the respiratory passage with accumulated dried secretions.

DIF: Cognitive Level: Application REF: p. 302 OBJ: Clinical Practice #3


TOP: Oral Care KEY: Nursing Process Step: Implementation
MSC: NCLEX: Physiological Integrity: basic care and comfort

19. It is most important for the nurse to write specific personal care plan modifications for the
patient who:
a. is 76 years old, alert, oriented, and able to provide his own care.

Downloaded by: SUCCEEDGRADES | abbieclin@gmail.com


Distribution
Downloaded by Dallen Mae D.ofKadir
this document is illegal
(kadir.dallenmae.d.bcsi@gmail.com)
lOMoARcPSD|35346190

Stuvia.com - The Marketplace to Buy and Sell your Study Material

b. had a hip replacement 2 years ago and uses a cane to ambulate.


c. has an artificial eye and poor vision in the other.
d. prefers a tub bath to a shower, preferably before bedtime.

ANS: C
Special care is necessary for the artificial eye, especially because the patient has poor vision in
the remaining eye.

DIF: Cognitive Level: Analysis REF: p. 310 OBJ: Theory #6


TOP: Hygiene KEY: Nursing Process Step: Planning
MSC: NCLEX: Physiological Integrity: basic care and comfort

20. A 20-year-old male patient is admitted after an auto accident. He has blood and dirt matted in
his hair. The nurse should:
a. blot the tangled, bloodied hair and then provide a bed shampoo to remove the
remaining dirt and debris.
b. comb the tangles out with a fine-toothed comb, starting at the scalp and working
down to the ends of the strands.
c. remove tangles by using alcohol or water on small sections of hair, holding the hair
between the scalp and the area the nurse is brushing or combing.
d. shampoo the hair as well as possible and leave the tangles alone.
ANS: C
Removing tangles in small sections is more comfortable for the patient. Trying to shampoo
before removing some of the tangles makes the situation worse.

DIF: Cognitive Level: Application REF: p. 306, Skill 19-4


OBJ: Clinical Practice #4 TOP: Hair Care
KEY: Nursing Process Step: Implementation
MSC: NCLEX: Physiological Integrity: basic care and comfort

21. When the nurse is assisting a male patient to shave his face, it is most important for her to:
a. practice on a male friend or relative before trying it on a patient.
b. have the patient shave first before any other hygiene measures are performed.
c. be sure the patient knows to draw the razor in the direction the hair grows.
d. check whether a safety razor can be used or whether it is contraindicated.
ANS: D
A patient who is on anticoagulants or who has a bleeding tendency should use an electric
razor.

DIF: Cognitive Level: Application REF: p. 308 OBJ: Clinical Practice #4


TOP: Shaving KEY: Nursing Process Step: Planning
MSC: NCLEX: Physiological Integrity

22. A usual routine for providing nail care to a patient includes:


a. soaking the nails in warm soapy water to soften before cleaning under the nail edge
with an orangewood stick.
b. gently cleaning under the nails with a metal file to remove dirt and dead skin and
then soaking hands or feet afterward.
c. cutting toenails with rounded edges to prevent scratching or ingrown nails.
d. cutting toenails and fingernails every 2 or 3 days to keep them short and clean.

Downloaded by: SUCCEEDGRADES | abbieclin@gmail.com


Distribution
Downloaded by Dallen Mae D.ofKadir
this document is illegal
(kadir.dallenmae.d.bcsi@gmail.com)
lOMoARcPSD|35346190

Stuvia.com - The Marketplace to Buy and Sell your Study Material

ANS: A
Soaking nails softens them and makes it easier to remove dirt or to cut them. A metal file
should not be used under the nails.

DIF: Cognitive Level: Knowledge REF: p. 308 OBJ: Clinical Practice #4


TOP: Nail Care KEY: Nursing Process Step: Implementation
MSC: NCLEX: Physiological Integrity: basic care and comfort

23. A nurse is caring for a patient who is wearing contact lenses. If the patient cannot care for the
lenses himself, and the nurse has difficulty removing a hard lens by hand, it is correct for the
nurse to:
a. leave the contacts in place for up to a month.
b. use a lens suction cup to remove the lens.
c. request an ophthalmologist (eye specialist) to come in to remove the lenses.
d. irrigate the eye with saline until the lens floats out.
ANS: B
A lens suction cup is usually available in health care facilities to remove contact lenses.

DIF: Cognitive Level: Knowledge REF: p. 310 OBJ: Clinical Practice #5


TOP: Eye Care KEY: Nursing Process Step: Implementation
MSC: NCLEX: Physiological Integrity: basic care and comfort

24. During the provision of oral care to an unconscious patient, the nurse uses suction primarily
to:
a. remove secretions that might block respiratory passages.
b. remove emesis if the patient should vomit.
c. prevent fluids from collecting in the patient‘s mouth and being aspirated.
d. stimulate the patient‘s gums and mucous membrane.
ANS: C
An unconscious patient may not have a gag or swallowing reflex, and thus fluids introduced
during mouth care need to be suctioned out (and the patient is positioned to facilitate drainage
with the head lowered and turned to the side).

DIF: Cognitive Level: Comprehension REF: p. 303, Skill 19-2


OBJ: Clinical Practice #3 TOP: Oral Care
KEY: Nursing Process Step: Implementation
MSC: NCLEX: Physiological Integrity: reduction of risk

25. A patient with insulin-dependent diabetes has a below-the-knee amputation on the right leg.
What modification of his personal care is noted as most important?
a. Perineal care should be performed at least twice a day to prevent urinary tract
infections.
b. A safety razor should not be used for shaving; an electric razor should be used.
c. The patient should be assisted to the shower, where he can use a shower chair.
d. The patient‘s left foot should be soaked and gently dried, but his toenails should not
be cut.
ANS: D

Downloaded by: SUCCEEDGRADES | abbieclin@gmail.com


Distribution
Downloaded by Dallen Mae D.ofKadir
this document is illegal
(kadir.dallenmae.d.bcsi@gmail.com)
lOMoARcPSD|35346190

Stuvia.com - The Marketplace to Buy and Sell your Study Material

A diabetic with a below-the-knee amputation is likely to have circulatory problems in the


remaining foot. Therefore, good foot care is essential, but toenail cutting should be performed
by a podiatrist.

DIF: Cognitive Level: Comprehension REF: p. 308 OBJ: Theory #4


TOP: Diabetic Foot Care KEY: Nursing Process Step: Planning
MSC: NCLEX: Physiological Integrity: basic care and comfort

26. A nurse notes that her patient has an area of red skin that does not blanch with fingertip
pressure. The nurse documents this finding as a stage pressure ulcer.
a. I
b. III
c. IV
d. II
ANS: A
A stage I pressure ulcer is characterized by an area of red, deep pink, or mottled skin that does
not blanch with fingertip pressure.

DIF: Cognitive Level: Comprehension REF: p. 291 OBJ: Clinical Practice #1


TOP: Skin Integrity KEY: Nursing Process Step: Assessment
MSC: NCLEX: Physiological Integrity: basic care and comfort

27. The nurse stages a pressure ulcer as a stage II based on the knowledge that such lesions have:
a. mottled skin and induration.
b. full-thickness skin loss and a deep crater.
c. partial thickness skin loss with the appearance of a blister.
d. a deep pink area of unblanchable skin.
ANS: D
A stage II pressure ulcer is characterized by an area of partial-thickness skin loss involving the
epidermis and/or dermis. It may look like an abrasion, blister, or shallow crater. The area
surrounding the damaged skin may feel warmer.

DIF: Cognitive Level: Comprehension REF: p. 291 OBJ: Clinical Practice #1


TOP: Skin Integrity KEY: Nursing Process Step: Assessment
MSC: NCLEX: Physiological Integrity: basic care and comfort

28. The culturally sensitive nurse caring for a Muslim woman who has noticeable body odor as
well as abundant underarm hair should:
a.use soap and water under the arms.
b. apply a cream type deodorant.
c.shave the underarms.
d. cut hair close to the armpit with scissors.
ANS: A
Washing the area with soap and water will reduce odor. Several cultures do not consider the
use of deodorant or shaving underarms essential. These personal preferences should be
respected.

DIF: Cognitive Level: Application REF: p. 289 OBJ: Theory #2


TOP: Cultural Considerations KEY: Nursing Process Step: Assessment

Downloaded by: SUCCEEDGRADES | abbieclin@gmail.com


Distribution
Downloaded by Dallen Mae D.ofKadir
this document is illegal
(kadir.dallenmae.d.bcsi@gmail.com)
lOMoARcPSD|35346190

Stuvia.com - The Marketplace to Buy and Sell your Study Material

MSC: NCLEX: Physiological Integrity: basic care and comfort

29. A nurse admitting a 76-year-old patient to the unit carefully documents the appearance of a
stage III pressure ulcer and informs the charge nurse because:
a. the presence of an ulcer suggests previous lack of care.
b. the charge nurse will need to report the presence of the ulcer.
c. Medicare will reimburse the facility if the ulcer advances.
d. documentation of a stage III ulcer on admission is part of good assessment.
ANS: C
Medicare will reimburse the facility at a higher rate if stages III and IV ulcers are documented
within 2 days of admission.

DIF: Cognitive Level: Comprehension REF: p. 289, Clinical Cues


OBJ: Clinical Practice #1 TOP: Documentation of Pressure Ulcer
KEY: Nursing Process Step: Assessment
MSC: NCLEX: Physiological Integrity: basic care and comfort

30. One of the facility‘s unlicensed assistive personnel (UAPs) is being instructed on foot care for
a 74-year-old patient with severely overgrown ragged toenails. The UAP should be reminded
to:
a. use an emery board to smooth the nail edges.
b. use scissors to round off the nail near the end of the toe.
c. apply lotion to the feet and apply bed socks.
d. cut the nail straight across with a nail clipper.
ANS: D
Cutting the nails straight will prevent ingrown toenails.

DIF: Cognitive Level: Application REF: p. 309 OBJ: Clinical Practice #4


TOP: Skin Care KEY: Nursing Process Step: Implementation
MSC: NCLEX: Physiological Integrity: basic care and comfort

31. A nurse is instructing a nursing student regarding prevention of pressure ulcers. The nurse
would recognize further instruction is warranted when the nursing student states, ―I will:
a. position the patient directly on the trochanter.‖
b. use a written schedule for turning and repositioning.‖
c. gently rub around a reddened area to restore circulation.‖
d. wash and dry the incontinent patient promptly.‖
ANS: A
Prevention of pressure ulcers includes not positioning the patient directly on the trochanter.

DIF: Cognitive Level: Application REF: p. 292, Safety Alert


OBJ: Clinical Practice #1 TOP: Prevention of Pressure Ulcers
KEY: Nursing Process Step: Evaluation
MSC: NCLEX: Physiological Integrity: basic care and comfort

32. The nurse shampooing the hair of an African American takes into consideration that the hair:
a. is oilier than the hair of whites.
b. should only be washed every 7 to 10 days.
c. should be dried with a hair dryer.

Downloaded by: SUCCEEDGRADES | abbieclin@gmail.com


Distribution
Downloaded by Dallen Mae D.ofKadir
this document is illegal
(kadir.dallenmae.d.bcsi@gmail.com)
lOMoARcPSD|35346190

Stuvia.com - The Marketplace to Buy and Sell your Study Material

d. should be combed with a fine-toothed comb.


ANS: B
The hair of African Americans should be washed only every 7 to 10 days because the hair is
much drier and less oily. Heat and the use of a fine-toothed comb cause the hair to break.

DIF: Cognitive Level: Comprehension REF: p. 308, Cultural


OBJ: Theory #2 TOP: Hair of African Americans
KEY: Nursing Process Step: Implementation
MSC: NCLEX: Physiological Integrity: basic care and comfort

COMPLETION

33. Skin that is frequently wet leads to , the softening of tissue that increases the
chance of trauma or infection.

ANS:
maceration
Knowing the risk factors for impaired skin integrity allows for prevention.

DIF: Cognitive Level: Knowledge REF: p. 289 OBJ: Theory #4


TOP: Skin Integrity KEY: Nursing Process Step: N/A
MSC: NCLEX: N/A

34. The buildup of tough necrotic tissue found with a pressure ulcer is called .

ANS:
eschar
Eschar is necrotic tissue that needs to be removed before a pressure ulcer can be accurately
staged.

DIF: Cognitive Level: Knowledge REF: p. 292 OBJ: Clinical Practice #1


TOP: Pressure Ulcer Staging KEY: Nursing Process Step: Assessment
MSC: NCLEX: N/A

MULTIPLE RESPONSE

35. Which of the following are main functions of the skin? (Select all that apply.)
a. Protection
b. Warmth
c. Excretion
d. Sensation
e. Secretion
f. Cleansing

ANS: A, C, D, E
Understanding the functions of the skin is imperative for providing proper care for patients
and for identifying risk factors for skin integrity.

DIF: Cognitive Level: Knowledge REF: p. 288 OBJ: Theory #1


TOP: Skin Functions KEY: Nursing Process Step: N/A

Downloaded by: SUCCEEDGRADES | abbieclin@gmail.com


Distribution
Downloaded by Dallen Mae D.ofKadir
this document is illegal
(kadir.dallenmae.d.bcsi@gmail.com)
lOMoARcPSD|35346190

Stuvia.com - The Marketplace to Buy and Sell your Study Material

MSC: NCLEX: N/A

36. The changes in the integumentary system that are part of the normal aging process are: (Select
all that apply.)
a. hair becomes thin and grows more slowly.
b. temperature control is altered because of the increased sebaceous gland activity.
c. skin is more fragile because of loss of collagen fibers.
d. skin wrinkles and sags.
e. nail growth increases.
ANS: A, C, D
Noting normal aging processes helps the nurse detect abnormal changes earlier. Sebaceous
gland activity is decreased and nail growth decreases.

DIF: Cognitive Level: Comprehension REF: p. 288 OBJ: Clinical Practice #1


TOP: Aging KEY: Nursing Process Step: N/A MSC: NCLEX: N/A

37. A stage III pressure ulcer is indicated by: (Select all that apply.)
a. full-thickness skin loss.
b. widespread infection.
c. drainage from the ulcer.
d. damaged subcutaneous tissue.
e. induration.
f. warmth of surrounding tissue.
ANS: A, C, D
Stage III includes full-thickness skin loss, drainage, and damage to the subcutaneous tissue.
Stage I may have an induration. Stage II may be indicated by warmth of the surrounding
tissue. Stage IV may be indicated by widespread infection.

DIF: Cognitive Level: Comprehension REF: p. 291 OBJ: Clinical Practice #2


TOP: Pressure Ulcer Staging KEY: Nursing Process Step: N/A
MSC: NCLEX: N/A

Downloaded by: SUCCEEDGRADES | abbieclin@gmail.com


Distribution
Downloaded by Dallen Mae D.ofKadir
this document is illegal
(kadir.dallenmae.d.bcsi@gmail.com)
lOMoARcPSD|35346190

Stuvia.com - The Marketplace to Buy and Sell your Study Material

Chapter 20: Patient Environment and Safety


Williams: deWit's Fundamental Concepts and Skills for Nursing, 8th Edition

MULTIPLE CHOICE

1. An older adult patient who is unable to get out of bed complains that the room is too cold
because of the air-conditioning and asks the nurse to open the window. The nurse‘s best
reply is:
a. ―Certainly, that will let in warm air from outside and should make you warmer.‖
b. ―The air conditioner is set to keep the most comfortable temperature in the room.‖
c. ―I‘ll adjust the thermostat in your room and get a blanket for you.‖
d. ―Agency policy prevents me from opening the window.‖
ANS: C
Older inactive people need a warmer environment because of their poor thermoregulation.
Rooms should be kept at a comfortable 68° to 74° F. Most health care facilities prohibit the
opening of windows for safety reasons.

DIF: Cognitive Level: Application REF: p. 321 OBJ: Theory #1


TOP: Environment Management KEY: Nursing Process Step: Implementation
MSC: NCLEX: Physiological Integrity: Basic Care and Comfort

2. An appropriate environmental nursing intervention for a patient with respiratory congestion


is to:
a. maintain the room temperature slightly cooler to decrease congestion.
b. moisten the respiratory passages with the use of an air humidifier.
c. order a large floor fan to make it easier to breathe.
d. open the windows to encourage air circulation.
ANS: B
A very low humidity will dry respiratory passages. Vaporizers or humidifiers may be
ordered for a patient with a respiratory condition. Small table fans may help some persons to
breathe more easily.

DIF: Cognitive Level: Application REF: p. 322


OBJ: Clinical Practice #1 TOP: Environment
KEY: Nursing Process Step: Implementation
MSC: NCLEX: Physiological Integrity: Basic Care and Comfort

3. The patient complains of an odor in his room that smells like something is rotting. The nurse
makes an assessment of the room and:
a. changes the linens, which are wrinkled and rumpled from 24-hour use.
b. rinses out the emesis basin of used dry tissues.
c. removes an old flower arrangement.
d. heavily sprays room deodorant around the patient‘s bed.
ANS: C

Downloaded by: SUCCEEDGRADES | abbieclin@gmail.com


Distribution
Downloaded by Dallen Mae D.ofKadir
this document is illegal
(kadir.dallenmae.d.bcsi@gmail.com)
lOMoARcPSD|35346190

Stuvia.com - The Marketplace to Buy and Sell your Study Material

Odors in health care facilities are frequently unpleasant. The odor from a deteriorating
flower arrangement is offensive. The arrangement should be discarded in a container outside
the patient‘s room.

DIF: Cognitive Level: Analysis REF: p. 322|Box 20-1


OBJ: Theory #1 TOP: Environment
KEY: Nursing Process Step: Implementation
MSC: NCLEX: Physiological Integrity: Basic Care and Comfort

4. A patient complains of not being able to sleep because of the noise in the hall at night. The
nurse should:
a. move the patient to the far end of the hall.
b. ask the doctor for a sleeping medication for the patient.
c. tell the patient to close the door.
d. request that co-workers limit hallway conversations.
ANS: D
The main cause of noise is people. Encourage the staff to limit conversations in the hallway
and speak in lowered voices.

DIF: Cognitive Level: Application REF: p. 322 OBJ: Theory #2


TOP: Environment KEY: Nursing Process Step: Implementation
MSC: NCLEX: Physiological Integrity: Basic Care and Comfort

5. The best way to maintain safety measures relative to helping a patient get into bed is to:
a. set the bed height at the nurse‘s waist level.
b. make sure that the bed wheels are locked.
c. place the bed against the wall.
d. insist that the patient stays in bed.
ANS: B
The goal is to provide safety when getting a patient into or out of bed. Locking the wheels to
the bed is one way to ensure this safety measure.

DIF: Cognitive Level: Application REF: p. 323 OBJ: Theory #4


TOP: Safety KEY: Nursing Process Step: Planning
MSC: NCLEX: Safe, Effective Care Environment: Safety and Infection Control

6. When the nurse is making an occupied bed, back safety indicates that the nurse should
initially:
a. raise the bed to the proper working height before starting.
b. encourage the patient to use the side rail to help turn side to side.
c. keep one side rail up at all times to keep the patient from falling.
d. complete the linen change on one side before moving to the other side.
ANS: A
Bringing the bed to height appropriate working level can prevent a back injury. Although
other options are part of the occupied bed skill, they are not directed at preventing back
strain.

Downloaded by: SUCCEEDGRADES | abbieclin@gmail.com


Distribution
Downloaded by Dallen Mae D.ofKadir
this document is illegal
(kadir.dallenmae.d.bcsi@gmail.com)
lOMoARcPSD|35346190

Stuvia.com - The Marketplace to Buy and Sell your Study Material

DIF: Cognitive Level: Application REF: p. 327|Box 20-2


OBJ: Clinical Practice #2 TOP: Safety
KEY: Nursing Process Step: Implementation
MSC: NCLEX: Safe, Effective Care Environment: Safety and Infection Control

7. A patient has left sided paralysis following a right-sided cerebrovascular accident (CVA).
After completing a bed bath, the nurse should begin to change the sheets by:
a. lowering both side rails and rolling the patient to the side of the bed.
b. asking the patient to roll to his right and hold on to the side rail for support.
c. positioning the patient in a supine position with both side rails raised.
d. positioning the patient in a side lying position on his left side with the near side
rails raised.
ANS: D
Moving the patient to the left side lying position provides safety for the patient and allows
the patient to use his good (right) hand to hold the rail.

DIF: Cognitive Level: Analysis REF: p. 327|Skill 20-2


OBJ: Clinical Practice #2 TOP: Safety
KEY: Nursing Process Step: Implementation
MSC: NCLEX: Safe, Effective Care Environment: Safety and Infection Control

8. An older adult patient is discharged home after hip surgery. The statement that indicates a
family member understands discharge safety instructions given by the nurse is:
a. ―I will install grab bars in the bathroom for both the toilet and bathtub.‖
b. ―I will put all personal items away to prevent my mother from dropping things.‖
c. ―I will dim the lights at night to prevent wakefulness.‖
d. ―I will ensure that my mother takes naps during the day to prevent tiredness.‖
ANS: A
Grab bars in the tub and at the toilet help the person with joint impairment to bathe and
toilet safely. Using well-lit areas during the day and night lights at night is helpful to avoid
falls. Daytime napping may cause restlessness at night.

DIF: Cognitive Level: Analysis REF: p. 330|Box 20-3


OBJ: Theory #4 TOP: Safety KEY: Nursing Process Step: Assessment
MSC: NCLEX: Safe, Effective Care Environment: Safety and Infection Control

9. The nurse in a long-term facility who is making a fall assessment would identify the person
most at risk for a fall to be a resident who:
a. paces all day in the halls and sleeps well at night.
b. had knee replacement surgery 2 days ago and wears a knee brace.
c. had a stroke with right-sided weakness 2 weeks ago and is confused.
d. uses a walker to ambulate both indoors and outdoors.
ANS: C
The most common factors predisposing a person to falls are impaired physical mobility,
altered mental status, and unavailability of assistance.

DIF: Cognitive Level: Analysis REF: p. 329 OBJ: Theory #4

Downloaded by: SUCCEEDGRADES | abbieclin@gmail.com


Distribution
Downloaded by Dallen Mae D.ofKadir
this document is illegal
(kadir.dallenmae.d.bcsi@gmail.com)
lOMoARcPSD|35346190

Stuvia.com - The Marketplace to Buy and Sell your Study Material

TOP: Safety KEY: Nursing Process Step: Assessment


MSC: NCLEX: Physiological Integrity: Safety and Infection Control

10. A patient who has right-sided weakness following a stroke is admitted to a long-term care
facility and exhibits increasing wandering and inability for self-care. To protect the patient
from the most frequent cause of injury among the older adult, the nurse‘s most efficient
intervention would be:
a. provide a night light in the bathroom.
b. keep pathways clear of paper, shoes, and equipment.
c. apply a personal alarm.
d. provide hip protectors.
ANS: C
Because falls are the most common accidents among residents, the provision of a personal
alarm to sound when the person attempts to get out of bed is the most efficient intervention.
Keeping the pathways clear, provision of adequate light, and provision of hip protectors are
all safety oriented but do not prevent falls.

DIF: Cognitive Level: Analysis REF: p. 330|Box 20-3


OBJ: Theory #3 TOP: Safety KEY: Nursing Process Step: Implementation
MSC: NCLEX: Safe, Effective Care Environment: Safety and Infection Control

11. A diabetic patient has chronic peripheral vascular disease, which results in edema and poor
circulation to her feet. She constantly complains of cold legs. The best nursing action is to
provide:
a. a heating pad and place it under the patient‘s feet.
b. an electric blanket to increase warmth to legs at night.
c. a hot shower to increase circulation to legs.
d. additional blankets and encourage the use of warm bed socks.
ANS: D
Extra blankets and bed socks will reduce the sense of cold. A person with diabetes or
impaired circulation is more easily burned than a person in good health.

DIF: Cognitive Level: Application REF: p. 331 OBJ: Theory #1


TOP: Safety KEY: Nursing Process Step: Implementation
MSC: NCLEX: Physiological Integrity: Basic Care and Comfort

12. An agitated resident who is seated in his wheelchair calls the nurse because the bed linens
are smoldering. After moving the patient to the hall, the nurse should:
a. close the door to the room to confine the fire.
b. assess the patient for burns.
c. extinguish the flames with an appropriate extinguisher.
d. activate the fire alarm system immediately.
ANS: D
RACE is used as an acronym to respond to fire. ―RACE‖ represents Rescuing the patient
from immediate danger, Activating the fire alarm system, Containing the fire by closing
doors and windows, and Extinguishing the flames with an appropriate extinguisher.

Downloaded by: SUCCEEDGRADES | abbieclin@gmail.com


Distribution
Downloaded by Dallen Mae D.ofKadir
this document is illegal
(kadir.dallenmae.d.bcsi@gmail.com)
lOMoARcPSD|35346190

Stuvia.com - The Marketplace to Buy and Sell your Study Material

DIF: Cognitive Level: Application REF: p. 322 OBJ: Theory #4


TOP: Safety KEY: Nursing Process Step: Implementation
MSC: NCLEX: Safe, Effective Care Environment: Safety and Infection Control

13. When caring for a patient with acute radiation sickness (ARS) after an accident at an atomic
power plant, the nurse should:
a. wear a paper gown and boots, gloves, and a mask.
b. stay in the room and talk to the patient to alleviate anxiety.
c. decrease the amount of time spent in the room.
d. wear a chemical mask with a filtered respirator.
ANS: A
For prolonged periods in caring for a patient with ARS, the nurse should use the barrier
protection of gown, boots, a mask, and gloves.

DIF: Cognitive Level: Application REF: p. 333 OBJ: Theory #1


TOP: Safety KEY: Nursing Process Step: Implementation
MSC: NCLEX: Safe, Effective Care Environment: Safety and Infection Control

14. There is evidence that a resident in a home care environment might have accidentally
ingested gasoline left by the gardeners. The nurse should first:
a. call the family members to notify them of the incident.
b. call the poison control center and describe the situation.
c. induce the patient to vomit.
d. place the gasoline can in a safe place.
ANS: B
If a nurse suspects gasoline poisoning, it is important to call the poison control center to
obtain further instructions. It is also important to prevent vomiting, because this may cause
respiratory problems.

DIF: Cognitive Level: Application REF: p. 335


OBJ: Clinical Practice #1 TOP: Safety
KEY: Nursing Process Step: Implementation
MSC: NCLEX: Health Promotion and Maintenance: Prevention and Early Detection of
Disease

15. A nursing assistant on the day shift reports that he has raised the bed rails to keep an agitated
patient from climbing out of bed. The nurse‘s best response to this information is:
a. ―Good idea. Be sure to check on the patient every hour to assess the patient‘s
comfort.‖
b. ―A vest protective device will work better; put one on the patient, please.‖
c. ―The rails won‘t prevent falling; bring the patient out to sit by the nurses‘ station
where we can watch her.‖
d. ―You‘ll need to check the patient every 15 minutes and reorient the patient as to
why the rails are up.‖
ANS: C
Seating the patient close to the nurses‘ station will allow the nurse to check on the patient
frequently. The nurse needs to get an order for a vest protective device.

Downloaded by: SUCCEEDGRADES | abbieclin@gmail.com


Distribution
Downloaded by Dallen Mae D.ofKadir
this document is illegal
(kadir.dallenmae.d.bcsi@gmail.com)
lOMoARcPSD|35346190

Stuvia.com - The Marketplace to Buy and Sell your Study Material

DIF: Cognitive Level: Application REF: p. 330|Box 20-3


OBJ: Clinical Practice #6 TOP: Safety
KEY: Nursing Process Step: Implementation
MSC: NCLEX: Safe, Effective Care Environment

16. A patient is agitated and confused and keeps getting out of bed and needs to be observed
constantly. The best initial nursing intervention is to:
a. have a family member or friend sit with the patient.
b. obtain an order for a sedative from the health care provider.
c. instruct the nurse‘s aide to apply a vest protective device.
d. make sure the side rails are up and close the door.
ANS: A
Local and federal laws prohibit the use of physical and chemical restraints except those
authorized by a health care provider. Health care workers are encouraged to find other
alternatives such as asking a family member to supervise the patient before resorting to the
use of protective devices.

DIF: Cognitive Level: Application REF: p. 340 OBJ: Theory #1


TOP: Safety KEY: Nursing Process Step: Implementation
MSC: NCLEX: Physiological Integrity: Basic Care and Comfort

17. The doctor has written an order to place a resident in the nursing home in a vest protective
device. It is the nurse‘s responsibility to:
a. check with the nursing supervisor about the legality of the order.
b. remove the device every 2 hours and change the patient‘s position.
c. remove the device every 4 hours to toilet the patient.
d. apply the device loosely to prevent circulation impairment.
ANS: B
Changing position helps prevent other complications such as skin decubiti.

DIF: Cognitive Level: Comprehension REF: p. 330|Box 20-5


OBJ: Theory #6 TOP: Safety KEY: Nursing Process Step: Implementation
MSC: NCLEX: Physiological Integrity: Basic Care and Comfort

18. The health care provider orders wrist protective devices for an agitated patient. To safely use
this protective device, the nurse:
a. checks that circulation is not impaired by evaluating color, warmth, and pulses
distal to the device.
b. secures the ties of the device to the side rails of the bed to allow for easy access by
the nurse.
c. draws the protective device tightly to prevent the patient‘s hands from slipping out.
d. uses a knot that is not easily undone for patient security.
ANS: A
Checking for signs indicating that circulation has been impaired or skin abraded or for
evidence of nerve impairment is part of the nurse‘s responsibility in upholding the principles
of the use of protective devices.

Downloaded by: SUCCEEDGRADES | abbieclin@gmail.com


Distribution
Downloaded by Dallen Mae D.ofKadir
this document is illegal
(kadir.dallenmae.d.bcsi@gmail.com)
lOMoARcPSD|35346190

Stuvia.com - The Marketplace to Buy and Sell your Study Material

DIF: Cognitive Level: Application REF: p. 338 OBJ: Theory #6


TOP: Safety KEY: Nursing Process Step: Implementation
MSC: NCLEX: Physiological Integrity: Basic Care and Comfort

19. The home health nurse assessing the home for safety hazards notes a hazard that should be
remedied is:
a. an extension cord lying across the floor.
b. nonskid bath mats on the bathroom floor and in the shower.
c. night lights high on the wall in the bathroom.
d. lack of scatter rugs on the wooden floor.
ANS: A
Extension cords pose a hazard for falls. The rest of the items assist in the prevention of falls.

DIF: Cognitive Level: Application REF: p. 330|Box 20-3


OBJ: Theory #1 TOP: Safety KEY: Nursing Process Step: Implementation
MSC: NCLEX: Physiological Integrity

20. A resident is confused and teary. She is threatening to leave the facility to return home. The
nurse should:
a. call her family immediately and notify them of the problem.
b. have the nurse‘s aide place a vest protective device on the patient.
c. call the doctor immediately and get an order for a protective device.
d. stay with the patient and attempt to determine the cause of the problem.
ANS: D
Protective devices may not be used without an order or to punish or discipline a patient.
Talking to the patient is an excellent strategy to determine the cause of the problem.
Medications may also cause mood alterations.
Stay with the patient who is confused or unsteady. Second action would be to determine if
there is family members that might be able to stay with the patient.

DIF: Cognitive Level: Analysis REF: p. 330 OBJ: Theory #6


TOP: Alternative to Protective Devices KEY: Nursing Process Step: Implementation
MSC: NCLEX: Physiological Integrity: Basic Care and Comfort

21. The nurse clarifies to the worried family that the guiding principle for using protective
devices is:
a. to use the least amount of immobilization needed for the situation.
b. to use only immobilization techniques necessary to keep the patient safe.
c. that protective devices are mandated for behavioral use only.
d. that protective devices must be applied by qualified personnel.
ANS: A
The principle is derived from local and federal laws and endorsed by the Joint Commission
on Accreditation of Healthcare Organizations (JCAHO), which supports protective devices
only as a last resort. This is because of previous misuse and abuse of these devices by health
care personnel.

Downloaded by: SUCCEEDGRADES | abbieclin@gmail.com


Distribution
Downloaded by Dallen Mae D.ofKadir
this document is illegal
(kadir.dallenmae.d.bcsi@gmail.com)
lOMoARcPSD|35346190

Stuvia.com - The Marketplace to Buy and Sell your Study Material

DIF: Cognitive Level: Knowledge REF: p. 335 OBJ: Theory #6


TOP: Safety KEY: Nursing Process Step: Implementation
MSC: NCLEX: Safe, Effective Care Environment: Reduction of Risk

22. Material safety data sheets (MSDS) are required by the Occupational Safety and Health
Administration (OSHA). The nurse must:
a. have a copy of all MSDS on the unit to safely handle biohazards.
b. know the location of the MSDS and comply with their guidelines.
c. not handle biohazards identified in the MSDS.
d. keep the MSDS confidential and not discuss them outside the agency.
ANS: B
MSDS are consulted for recommended methods of storage, labeling, handling spills, and
disposal of biohazards.

DIF: Cognitive Level: Knowledge REF: p. 333


OBJ: Clinical Practice #3 TOP: Environment Management
KEY: Nursing Process Step: Planning
MSC: NCLEX: Safe, Effective Care Environment: Safety and Infection Control

23. A nurse is instructing a nursing student about protective device use. The nurse recognizes
the need for further instruction when the nursing student states, ―I will:
a. tie the protective device to the side rails to ensure the protective devices are
secure.‖
b. use a half bow knot to secure the protective devices to the bed frame.‖
c. check the area distal to the protective devices every 15 to 30 minutes.‖
d. observe for signs of adequate circulation, including distal pulses.‖
ANS: A
Protective device ties should be secured to an immovable part of the bed frame. They should
not be tied to the side rails because lowering the rails may cause the device to be pulled too
tightly around the patient or cause strain on a joint of an immobilized extremity. A half bow
knot should be used to secure the device to the bed frame or chair. The area distal to the
protective device should be checked every 15 to 30 minutes and should be observed for
signs of adequate circulation, including pulses distal to the device.

DIF: Cognitive Level: Application REF: p. 330|Skill 20-3


OBJ: Theory #6 TOP: Protective Devices
KEY: Nursing Process Step: Evaluation
MSC: NCLEX: Physiological Integrity: Basic Care and Comfort

24. A nurse caring for a patient with a chair alarm will do which of the following interventions
as recommended by the 2016 National Safety Goals to prevent alarm fatigue.
a. Respond promptly to alarm.
b. Educate patient to turn off alarm when it goes off.
c. Check alarm volume each hour.
d. Monitor functionality of alarm each morning.
ANS: A

Downloaded by: SUCCEEDGRADES | abbieclin@gmail.com


Distribution
Downloaded by Dallen Mae D.ofKadir
this document is illegal
(kadir.dallenmae.d.bcsi@gmail.com)
lOMoARcPSD|35346190

Stuvia.com - The Marketplace to Buy and Sell your Study Material

Bed and chair alarms along with cardiac and oxygen sensor alarms are among the alarms
specified in the 2015 National Patient Safety Goal of improving the safety of medical
alarms. All medical equipment alarms need to be closely monitored and checked for
functionality and volume at the start of the shift and frequently throughout the shift. Steps
need to be taken to prevent the catastrophic outcomes of alarm fatigue (Box 20-4). Alarm
fatigue occurs when nurses become desensitized to patient care alarms and then either miss
or delay response to an alarm. These absent or delayed responses have resulted in adverse
patient outcomes (Sendelback, S. and Funk 2013).

DIF: Cognitive Level: Application REF: p. 330 OBJ: Theory #2


TOP: Falls KEY: Nursing Process Step: N/A MSC: NCLEX: N/A

25. A fire has started in a work area on the unit. Which of the following is the response which
demonstrates correctly using a fire extinguisher?
a. Aim the stream to the top of the flames.
b. Squeeze the pin to activate the extinguisher.
c. Move the extinguisher in a sweeping, side to side motion.
d. Call 911.
ANS: C
The acronym PASS can be used to correctly use a fire extinguisher: Pull the pin, Aim at the
base of the fire, Squeeze the trigger, and Sweep side to side.

DIF: Cognitive Level: Analysis REF: p. 332 OBJ: Theory #4


TOP: Fire KEY: Nursing Process Step: N/A MSC: NCLEX: N/A

MULTIPLE RESPONSE

1. The certified nursing assistant (CNA) places a confused, weak patient in a wheelchair and
applies a vest protective device. The nurse should instruct the CNA to: (Select all that
apply.)
a. secure the ties in the front to prevent the patient from falling.
b. secure the ties in the back to prevent the patient from falling.
c. use a double knot to prevent the patient from undoing the tie.
d. use a half bow knot to secure the device to a chair.
e. provide passive range of motion to the upper extremities as needed.
ANS: B, D, E
Placing the ties under the armrests and securing at the back will keep the patient from
sliding. The half bow knot makes it difficult for the patient but easy for the health care
worker to undo.

DIF: Cognitive Level: Application REF: p. 337|Skill 20-3


OBJ: Clinical Practice #6 TOP: Safety
KEY: Nursing Process Step: Implementation
MSC: NCLEX: Safe, Effective Care Environment: Safety and Infection Control

2. Legal implications for using a protective device require thorough documentation and require
that the nurse include: (Select all that apply.)

Downloaded by: SUCCEEDGRADES | abbieclin@gmail.com


Distribution
Downloaded by Dallen Mae D.ofKadir
this document is illegal
(kadir.dallenmae.d.bcsi@gmail.com)
lOMoARcPSD|35346190

Stuvia.com - The Marketplace to Buy and Sell your Study Material

a. alternative methods and actions used.


b. medications that the patient is taking.
c. education done for patient and family.
d. the patient‘s medical diagnosis.
e. type of device and placement.
ANS: A, C, E
The nurse should document alternative methods and actions taken before placing the device,
the education done for both patient and family, and the type of device used and where it was
placed. Remember, if it is not documented, it was not done.

DIF: Cognitive Level: Knowledge REF: p. 337|Skill 20-3


OBJ: Theory #7 TOP: Safety KEY: Nursing Process Step: Implementation
MSC: NCLEX: Physiological Integrity: Reduction of Risk

3. Each resident admitted must have a fall risk assessment performed so that appropriate
actions to prevent falls can be included in the nursing care plan. The items are considered
when doing a fall risk assessment on a newly admitted resident include: (Select all that
apply.)
a. gender.
b. age.
c. weight.
d. medications.
e. balance.
ANS: B, D, E
The common factors that predispose a person to falls may include age, the type of
medications the resident is taking, and physical mobility.

DIF: Cognitive Level: Knowledge REF: p. 330|Figure 20 3


OBJ: Theory #1 TOP: Safety KEY: Nursing Process Step: Assessment
MSC: NCLEX: Physiological Integrity: Reduction of Risk

Downloaded by: SUCCEEDGRADES | abbieclin@gmail.com


Distribution
Downloaded by Dallen Mae D.ofKadir
this document is illegal
(kadir.dallenmae.d.bcsi@gmail.com)
lOMoARcPSD|35346190

Stuvia.com - The Marketplace to Buy and Sell your Study Material

Chapter 21: Measuring Vital Signs


Williams: deWit's Fundamental Concepts and Skills for Nursing, 8th Edition

MULTIPLE CHOICE

1. The nurse would anticipate a patient diagnosed with damage to the hypothalamus after
suffering a head injury from a fall to exhibit:
a. a blood pressure elevation.
b. a temperature abnormality.
c. a decrease in pulse rate.
d. depressed respirations.
ANS: B
The hypothalamus, which is located between the cerebral hemispheres, controls body
temperature. Any damage to the hypothalamus prevents the body from regulating its
temperature.

DIF: Cognitive Level: Comprehension REF: p. 344 OBJ: Theory #1


TOP: Vital Signs: Temperature KEY: Nursing Process Step: Assessment
MSC: NCLEX: Physiological Integrity: Physiological Adaptation

2. The nurse documents vital signs on a newly admitted patient as: ―blood pressure is 148/94
mm Hg, the pulse is 80 beats/min, and the respirations are 16 breaths/min.‖ The nurse would
record the pulse pressure as:
a. 14 mm Hg.
b. 54 mm Hg.
c. 64 mm Hg.
d. 80 mm Hg.
ANS: B
In calculating pulse pressure, take the difference between the systolic and diastolic pressures
(ie, 148 – 94 = 54).

DIF: Cognitive Level: Analysis REF: p. 364


OBJ: Clinical Practice #4 TOP: Vital Signs: Blood Pressure
KEY: Nursing Process Step: Assessment
MSC: NCLEX: Physiological Integrity: Physiological Adaptation

3. A patient has been admitted with hypothermia after lying unconscious overnight in an
unheated apartment. The most appropriate route to assess the patient‘s core temperature
would be:
a. rectal.
b. tympanic arterial thermometer.
c. axillary.
d. tympanic.
ANS: D

Downloaded by: SUCCEEDGRADES | abbieclin@gmail.com


Distribution
Downloaded by Dallen Mae D.ofKadir
this document is illegal
(kadir.dallenmae.d.bcsi@gmail.com)
lOMoARcPSD|35346190

Stuvia.com - The Marketplace to Buy and Sell your Study Material

The same blood vessels serve the hypothalamus and the tympanic membrane, so the
tympanic temperature is an excellent indicator of core body temperature, although it can be
affected by ear wax.

DIF: Cognitive Level: Application REF: p. 348


OBJ: Theory #3 | Clinical Practice #1 TOP: Vital Signs: Temperature
KEY: Nursing Process Step: Implementation
MSC: NCLEX: Physiological Integrity: Physiological Adaptation

4. The nurse would document a patient as being febrile if the patient‘s temperature was over:
a. 99.5° F
b. 99.8° F
c. 100° F
d. 100.5° F
ANS: D
A patient with a temperature above the normal range (100.2° F) is called febrile.

DIF: Cognitive Level: Knowledge REF: p. 349 OBJ: Theory #3


TOP: Vital Signs: Temperature KEY: Nursing Process Step: Implementation
MSC: NCLEX: Physiological Integrity: Physiological Adaptation

5. To ensure an accurate reading when using a glass oral thermometer, it is necessary to:
a. rinse the thermometer with water.
b. wipe the thermometer with alcohol.
c. shake down the galinstan alloy to below normal.
d. dry the thermometer with a dry cotton ball.
ANS: C
Oral thermometers remain at the last reading until they are shaken down; therefore, for
accuracy, the thermometer must be below normal range before using.

DIF: Cognitive Level: Application REF: p. 351


OBJ: Clinical Practice #1 TOP: Vital Signs: Temperature
KEY: Nursing Process Step: Implementation
MSC: NCLEX: Physiological Integrity: Basic Care and Comfort

6. The nurse taking an apical pulse would place the stethoscope at:
a. the left of the sternum at the third intercostal space.
b. directly below the sternum.
c. slightly above the left nipple.
d. the left midclavicular line at the fifth intercostal space.
ANS: D
The apical pulse is determined by placing a stethoscope on a point midway between the
imaginary line running from the midclavicle through the left nipple in the fifth intercostal
space.

DIF: Cognitive Level: Application REF: p. 359| Skill 21-4


OBJ: Theory #2 | Clinical Practice #2 TOP: Vital Signs: Pulse

Downloaded by: SUCCEEDGRADES | abbieclin@gmail.com


Distribution
Downloaded by Dallen Mae D.ofKadir
this document is illegal
(kadir.dallenmae.d.bcsi@gmail.com)
lOMoARcPSD|35346190

Stuvia.com - The Marketplace to Buy and Sell your Study Material

KEY: Nursing Process Step: Assessment


MSC: NCLEX: Physiological Integrity: Basic Care and Comfort

7. The nurse would record a pulse as bradycardic if the rate were:


a. 64 beats/min.
b. 62 beats/min.
c. 60 beats/min.
d. 59 beats/min.
ANS: D
Bradycardia indicates a slow pulse that is less than 60 beats/min.

DIF: Cognitive Level: Comprehension REF: p. 373 OBJ: Theory #3


TOP: Vital Signs: Pulse KEY: Nursing Process Step: Assessment
MSC: NCLEX: Physiological Integrity: Basic Care and Comfort

8. The nurse is aware that the use of an oral glass thermometer would be contraindicated in a:
a. 5-year-old with a facial laceration.
b. 12-year-old patient with a recent seizure.
c. 15-year-old with an abscessed tooth.
d. 20-year-old with severe dehydration.
ANS: B
The rectal method is best for patients who have seizure activity so as not to put them at risk
for biting and breaking the thermometer.

DIF: Cognitive Level: Application REF: p. 349


OBJ: Clinical Practice #1 TOP: Vital Signs: Temperature
KEY: Nursing Process Step: Planning
MSC: NCLEX: Physiological Integrity: Basic Care and Comfort

9. The nurse anticipates that if the stroke volume of a patient is reduced, the pulse will be:
a. stronger.
b. weaker.
c. bradycardic.
d. irregular.
ANS: B
A weak pulse will result if the stroke volume is reduced, because this decreases circulating
volume.

DIF: Cognitive Level: Comprehension REF: p. 345 OBJ: Theory #2


TOP: Vital Signs: Pulse KEY: Nursing Process Step: Assessment
MSC: NCLEX: Physiological Integrity: Basic Care and Comfort

10. When caring for a victim with a gunshot wound to the abdomen who has lost a significant
amount of blood, the nurse would anticipate the vital signs to reflect:
a. increase in temperature.
b. decrease in blood pressure.
c. decrease in pulse.

Downloaded by: SUCCEEDGRADES | abbieclin@gmail.com


Distribution
Downloaded by Dallen Mae D.ofKadir
this document is illegal
(kadir.dallenmae.d.bcsi@gmail.com)
lOMoARcPSD|35346190

Stuvia.com - The Marketplace to Buy and Sell your Study Material

d. decrease in respirations.
ANS: B
If blood volume decreases, as with bleeding, blood pressure decreases.

DIF: Cognitive Level: Analysis REF: p. 347 OBJ: Theory #2


TOP: Vital Signs: Blood Pressure KEY: Nursing Process Step: Assessment
MSC: NCLEX: Physiological Integrity: Physiological Adaptation

11. When a frail 83-year-old patient whose temperature was 96.8° F at 8:00 AM shows a
temperature of 98.6° F at 4:00 PM, the nurse is:
a. pleased that the temperature has come up to normal.
b. satisfied that the patient is warm enough.
c. concerned about the evidence of fever.
d. relieved that the patient is improving.
ANS: C
In older patients who have a frail frame, the normal temperature is often 97.2° F. An
elevation of 2° F is indicative of fever.

DIF: Cognitive Level: Application REF: p. 349 OBJ: Theory #4


TOP: Vital Signs in the Older Adult KEY: Nursing Process Step: Assessment
MSC: NCLEX: Physiological Integrity: Physiological Adaptation

12. A patient who is terminally ill is described during shift report as having Cheyne-Stokes
breathing. On assessment, the nurse anticipates finding:
a. a breathing pattern of dyspnea followed by a short period of apnea.
b. rapid wheezing respirations for two or three breaths with short periods of apnea.
c. quick shallow respirations with long periods of apnea.
d. respirations gradually decreasing in rate and depth.
ANS: A
Cheyne-Stokes respirations are faster and deeper rather than slower and are followed by a
period of no breathing.

DIF: Cognitive Level: Analysis REF: p. 363 OBJ: Theory #5


TOP: Vital Signs: Respirations KEY: Nursing Process Step: Assessment
MSC: NCLEX: Physiological Integrity: Physiological Adaptation

13. The nurse explains to a patient that the pulse oximeter can measure the arterial oxygen by:
a. assessing the amount of blood passing through the sensor.
b. assessing the relative warmth of the skin on the monitored part.
c. measuring the oxygenated hemoglobin through a capillary bed.
d. measuring the respirations to the blood pressure via infrared rays.
ANS: C
The pulse oximeter measures oxygen saturation by means of a sensor/probe attached to
peripheral digits, an earlobe, the nose, or the forehead as it passes through the capillary bed.
Oxygenated blood absorbs more infrared than red light.

DIF: Cognitive Level: Comprehension REF: p. 364 OBJ: Theory #5

Downloaded by: SUCCEEDGRADES | abbieclin@gmail.com


Distribution
Downloaded by Dallen Mae D.ofKadir
this document is illegal
(kadir.dallenmae.d.bcsi@gmail.com)
lOMoARcPSD|35346190

Stuvia.com - The Marketplace to Buy and Sell your Study Material

TOP: Vital Signs: Pulse KEY: Nursing Process Step: Implementation


MSC: NCLEX: Physiological Integrity: Basic Care and Comfort

14. Because the older adult‘s blood vessels are nonelastic, they are prone to orthostatic
hypotension. A priority intervention for a patient with orthostatic hypotension is to:
a. keep the patient in bed in a high Fowler‘s position.
b. allow the patient to sit on the side of the bed for a minute before standing.
c. instruct the patient to use the wheelchair for all mobility activity.
d. help the patient to rise quickly and support the patient for a minute.
ANS: B
The older adult often experiences orthostatic hypotension and are at risk for falls and should
be encouraged to sit on the side of the bed a minute before standing. These patients also
benefit from the use of elastic stockings.

DIF: Cognitive Level: Application REF: p. 370 OBJ: Theory #2


TOP: Orthostatic Hypotension KEY: Nursing Process Step: Planning
MSC: NCLEX: Safe, Effective Care Environment: Safety and Infection Control

15. An older adult patient has a tympanic temperature of 96.2° F (35.7° C). What nursing
intervention would best meet this patient‘s need?
a. Take the patient‘s vital signs every 4 hours, including temperature.
b. Provide fluids to increase circulation.
c. Increase room temperature to 72° F (22.2° C) and add blankets to the bed.
d. Check the temperature orally to confirm the accuracy of the reading.
ANS: C
Nursing interventions for treating hypothermia should focus on reducing heat loss and
supplying additional warmth, such as increasing the room temperature and adding blankets
to the bed.

DIF: Cognitive Level: Application REF: p. 350 OBJ: Theory #3


TOP: Vital Signs: Temperature KEY: Nursing Process Step: Implementation
MSC: NCLEX: Physiological Integrity: Basic Care and Comfort

16. The nurse using either a regular or an electronic sphygmomanometer would ensure that the
cuff is the correct size by:
a. using a narrow cuff for an obese patient.
b. making sure the width of the bladder is at least 3 inches.
c. confirming that the bladder goes around three fourths of the arm.
d. always using a wide cuff.
ANS: C
For accuracy in a BP reading, the cuff of the sphygmomanometer should have a bladder that
goes around three fourths of the arm.

DIF: Cognitive Level: Comprehension REF: p. 366


OBJ: Clinical Practice #4 TOP: Vital Signs: Blood Pressure
KEY: Nursing Process Step: Assessment
MSC: NCLEX: Physiological Integrity: Basic Care and Comfort

Downloaded by: SUCCEEDGRADES | abbieclin@gmail.com


Distribution
Downloaded by Dallen Mae D.ofKadir
this document is illegal
(kadir.dallenmae.d.bcsi@gmail.com)
lOMoARcPSD|35346190

Stuvia.com - The Marketplace to Buy and Sell your Study Material

17. For the nurse to assess the most accurate respiration count, the nurse should:
a. inform the patient about his respirations and ask him to breathe normally.
b. count each inhalation and expiration for 1 full minute.
c. watch the patient‘s chest rise and fall from a distance.
d. continue to hold the patient‘s radial pulse, and count the respirations for 30 seconds
and multiply them by 2.
ANS: D
The respirations should be counted for 30 seconds and multiplied by 2 if they are regular. If
the patient knows the nurse is assessing the respiration, he or she may alter breathing.

DIF: Cognitive Level: Application REF: p. 361|Skill 21-5


OBJ: Clinical Practice #3 TOP: Vital Signs: Respirations
KEY: Nursing Process Step: Assessment
MSC: NCLEX: Physiological Integrity: Basic Care and Comfort

18. Older adult patients with hypertension may have an auscultatory gap in their Korotkoff
sounds. It is important when taking their blood pressure measurement to:
a. continue to listen until the cuff is deflated.
b. pump up the cuff until no sound is heard and then let the air out.
c. make sure the bell of the stethoscope is placed firmly over the artery.
d. stop midway and begin to inflate again.
ANS: A
Many older adults with hypertension have an auscultatory gap in their Korotkoff sounds,
making it important to listen until the cuff is deflated to avoid mistaking the auscultatory
gap as the Korotkoff sound.

DIF: Cognitive Level: Application REF: p. 368 OBJ: Theory #6


TOP: Vital Signs in the Older Adult KEY: Nursing Process Step: Assessment
MSC: NCLEX: Physiological Integrity: Basic Care and Comfort

19. Regarding the blood pressure in children, the diastolic pressure is assessed by the
auscultation of a:
a. clear tapping that gradually grows louder.
b. murmur or swishing sound that increases with depression of the cuff.
c. sudden change or muffling of the sound.
d. louder knocking sound that occurs with each heartbeat.
ANS: C
A sudden change or muffling sound (Phase IV) indicates the diastolic pressure in children
and in some adults.

DIF: Cognitive Level: Application REF: p. 368


OBJ: Clinical Practice #4 TOP: Vital Signs in Children
KEY: Nursing Process Step: Assessment
MSC: NCLEX: Physiological Integrity: Basic Care and Comfort

20. The nurse covers a newborn baby‘s head with a cap, because the head:

Downloaded by: SUCCEEDGRADES | abbieclin@gmail.com


Distribution
Downloaded by Dallen Mae D.ofKadir
this document is illegal
(kadir.dallenmae.d.bcsi@gmail.com)
lOMoARcPSD|35346190

Stuvia.com - The Marketplace to Buy and Sell your Study Material

a. is wet and needs to be dried.


b. has large fontanels.
c. allows loss of body heat.
d. can be reshaped more quickly.
ANS: C
Infants lose considerable body heat through the scalp; therefore, a cap helps prevent heat
loss.

DIF: Cognitive Level: Application REF: p. 350 OBJ: Theory #3


TOP: Vital Signs: Infant Temperature KEY: Nursing Process Step: Implementation
MSC: NCLEX: Physiological Integrity: Basic Care and Comfort

21. The nurse is caring for a patient who had a cardiac catheterization 2 hours ago and has a
pressure dressing to his left groin. In addition to taking routine vital signs, the nurse should
also check the:
a. strength of the femoral pulse.
b. presence of the pedal pulse.
c. temperature of the right foot.
d. ability to move the left toes.
ANS: B
Pedal pulses are checked to determine whether there is any blockage in the artery following
a cardiac catheterization.

DIF: Cognitive Level: Application REF: p. 360


OBJ: Clinical Practice #7 TOP: Pedal Pulse
KEY: Nursing Process Step: Assessment
MSC: NCLEX: Physiological Integrity: Basic Care and Comfort

22. The accuracy in measuring the apical pulse is enhanced when the nurse:
a. counts the radial pulse at the same time.
b. counts the beats for a minute.
c. keeps the patient warm.
d. uses the bell of the stethoscope.
ANS: B
Using the diaphragm of the stethoscope, the nurse counts the beats for 1 full minute.

DIF: Cognitive Level: Application REF: p. 359|Skill 21-4


OBJ: Clinical Practice #2 TOP: Counting Apical Pulse
KEY: Nursing Process Step: Implementation
MSC: NCLEX: Physiological Integrity: Basic Care and Comfort

23. A 45-year-old patient who is alert and oriented has a blood pressure of 98/66 mm Hg, radial
pulse of 76 beats/min (irregular), and respirations of 18 breaths/min (regular). The best
nursing intervention is to:
a. notify the charge nurse of the hypotension.
b. notify the doctor of the bradycardia.
c. check medications that might be the cause of the irregularity.

Downloaded by: SUCCEEDGRADES | abbieclin@gmail.com


Distribution
Downloaded by Dallen Mae D.ofKadir
this document is illegal
(kadir.dallenmae.d.bcsi@gmail.com)
lOMoARcPSD|35346190

Stuvia.com - The Marketplace to Buy and Sell your Study Material

d. check the patient‘s record to determine his baseline blood pressure.


ANS: D
Check to see what the patient‘s baseline vital signs indicate regarding the cardiac
arrhythmia.

DIF: Cognitive Level: Application REF: p. 359


OBJ: Clinical Practice #6 TOP: Vital Signs
KEY: Nursing Process Step: Implementation
MSC: NCLEX: Physiological Integrity: Basic Care and Comfort

24. A nurse is caring for a patient with a cardiac disease history. When measuring vital signs, the
nurse finds that the radial pulse is 102 beats/min and irregular. The nurse correctly:
a. listens to the apical pulse for 1 full minute.
b. takes the pulse for 30 seconds on the other wrist.
c. records the findings on the graphic sheet.
d. takes the pulse for 1 full minute on the other wrist.
ANS: A
An apical pulse is measured whenever the radial pulse is irregular or when the patient has a
cardiac disease history.

DIF: Cognitive Level: Application REF: p. 359|Skill 21-6


OBJ: Clinical Practice #2 TOP: Vital Signs: Pulse
KEY: Nursing Process Step: Implementation
MSC: NCLEX: Physiological Integrity: Basic Care and Comfort

25. The nurse caring for a 30-year-old postsurgical patient would assess that the patient is in
pain as indicated by:
a. a temperature of 102° F.
b. respirations of 16 breaths/min.
c. a pulse rate of 120 beats/min.
d. blood pressure of 128/86 mm Hg.
ANS: C
Pain increases the pulse rate.

DIF: Cognitive Level: Application REF: p. 360|Table 21-2


OBJ: Theory #2 TOP: Vital Signs KEY: Nursing Process Step: Assessment
MSC: NCLEX: Physiological Integrity: Basic Care and Comfort

26. The nurse explains that one method of environmental heat loss is convection, which is
exemplified by body heat being reduced by:
a. being transferred to ice packs.
b. production of sweat.
c. being removed by fast air currents from a fan.
d. exposure to a cool environment.
ANS: C
Heat loss through convection can be accomplished by the use of a fan, which produces fast
air currents.

Downloaded by: SUCCEEDGRADES | abbieclin@gmail.com


Distribution
Downloaded by Dallen Mae D.ofKadir
this document is illegal
(kadir.dallenmae.d.bcsi@gmail.com)
lOMoARcPSD|35346190

Stuvia.com - The Marketplace to Buy and Sell your Study Material

DIF: Cognitive Level: Comprehension REF: p. 345 OBJ: Theory #1


TOP: Heat Loss by Convection KEY: Nursing Process Step: Implementation
MSC: NCLEX: Physiological Integrity: Physiological Adaptation

27. The home health nurse is instructing a caregiver about caring for a patient with
hypothermia. The nurse recognizes that further instruction is warranted when the caregiver
states, ―I will:
a. offer warm fluids to the patient, if permitted.‖
b. instruct the patient to remain on strict bed rest.‖
c. provide the patient with additional blankets.‖
d. encourage the patient to increase his muscle activity.‖
ANS: B
Nursing activities for treating the patient with a below normal body temperature should
focus on reducing heat loss and supplying additional warmth. These activities may include
(1) providing additional clothing or blankets for warmth (an electric blanket is most
effective for raising temperature); (2) giving warm fluids, if permitted; (3) adjusting the
temperature of the room to 72° F or higher; (4) eliminating drafts; and (5) increasing the
patient‘s muscle activity.

DIF: Cognitive Level: Analysis REF: p. 350 OBJ: Theory #3


TOP: Vital Signs: Hypothermia KEY: Nursing Process Step: Implementation
MSC: NCLEX: Physiological Integrity: Basic Care and Comfort

28. A nurse educates patients with prehypertension to implement lifestyle changes that would
decrease their systolic pressure from 140 to 120 mm Hg. Which of the following is his or
her rationale for this?
a. Reduced deaths by 50% in people over age 40.
b. Reduced rates of strokes by 10%.
c. Reduced rates of COPD by 25%.
d. Reduced rates of heart attacks by 30%.
ANS: D
A large study was recently stopped early because it demonstrated that lowering the systolic
blood pressure to 120 instead of the recommended 140 reduced the rates of heart attacks and
strokes by 30% and deaths by 25% in people over age 50 (AJN, 2015).

DIF: Cognitive Level: Application REF: p. 370 OBJ: Clinical #14


TOP: Life Span Considerations KEY: Nursing Process Step: N/A
MSC: NCLEX: N/A

MULTIPLE RESPONSE

1. Standards of the Joint Commission state that pain is the fifth vital sign and should be
documented by assessments of: (Select all that apply.)
a. location.
b. duration.
c. usual methods of relief.

Downloaded by: SUCCEEDGRADES | abbieclin@gmail.com


Distribution
Downloaded by Dallen Mae D.ofKadir
this document is illegal
(kadir.dallenmae.d.bcsi@gmail.com)
lOMoARcPSD|35346190

Stuvia.com - The Marketplace to Buy and Sell your Study Material

d. character.
e. intensity.
ANS: A, B, D, E
Pain should be monitored when vital signs are monitored, to closely assess for any cardiac
changes. Pain is documented by assessments relative to location, intensity, character,
frequency, and duration.

DIF: Cognitive Level: Application REF: p. 371 OBJ: Theory #7


TOP: Pain Assessment KEY: Nursing Process Step: Implementation
MSC: NCLEX: Physiological Integrity: Basic Care and Comfort

2. The nurse would refrain from applying a blood pressure cuff on the affected arm of a patient
who has a(n): (Select all that apply.)
a. previous mastectomy.
b. patent IV line.
c. injured hand.
d. 2-year-old hand amputation.
e. dialysis shunt.
ANS: A, B, E
Arms affected by previous mastectomies, patent IVs, or dialysis shunts should not be used
to assess the blood pressure using an inflatable cuff.

DIF: Cognitive Level: Application REF: p. 366|Skill 21-6


OBJ: Clinical Practice #4
TOP: Contraindications for Blood Pressure Cuff Application
KEY: Nursing Process Step: Implementation
MSC: NCLEX: Physiological Integrity: Basic Care and Comfort

3. The nurse assesses that the 86-year-old patient is experiencing orthostatic hypotension when
assessments indicate: (Select all that apply.)
a. dizziness upon rising to a standing position.
b. a drop of 15 to 20 mm Hg from baseline when changing position.
c. nausea.
d. syncope.
e. blurred vision.
ANS: A, B, D, E
Assessment of dizziness, drop in up to 20 mm Hg from baseline BP, syncope, and blurred
vision are all indicative of orthostatic hypotension.

DIF: Cognitive Level: Application REF: p. 371|Box 21-5


OBJ: Clinical Practice #6 TOP: Orthostatic Hypotension
KEY: Nursing Process Step: Assessment
MSC: NCLEX: Physiological Integrity: Physiological Adaptation

COMPLETION

1. The nurse clarifies the average cardiac output in the adult is about L/min.

Downloaded by: SUCCEEDGRADES | abbieclin@gmail.com


Distribution
Downloaded by Dallen Mae D.ofKadir
this document is illegal
(kadir.dallenmae.d.bcsi@gmail.com)
lOMoARcPSD|35346190

Stuvia.com - The Marketplace to Buy and Sell your Study Material

ANS:
5

The average cardiac output of the normal adult is about 5 L/min.

DIF: Cognitive Level: Knowledge REF: p. 345 OBJ: Theory #2


TOP: Cardiac Output KEY: Nursing Process Step: Implementation
MSC: NCLEX: Physiological Integrity: Physiological Adaptation

2. The nurse converts the Fahrenheit temperature of 99.2 to a Celsius reading of


.

ANS:
37.3

To convert Fahrenheit to Celsius: subtract 32 from the Fahrenheit reading and multiply by
5/9: 99.2 – 32 = 67.2 5 = 336/9 = 37.3.

DIF: Cognitive Level: Analysis REF: p. 347|Tables 21-1 and 21-21


OBJ: Clinical Practice #1 TOP: Conversion of Fahrenheit to Celsius
KEY: Nursing Process Step: Implementation
MSC: NCLEX: Physiological Integrity: Basic Care and Comfort

Downloaded by: SUCCEEDGRADES | abbieclin@gmail.com


Distribution
Downloaded by Dallen Mae D.ofKadir
this document is illegal
(kadir.dallenmae.d.bcsi@gmail.com)
lOMoARcPSD|35346190

Stuvia.com - The Marketplace to Buy and Sell your Study Material

Chapter 22: Assessing Health Status


Williams: deWit's Fundamental Concepts and Skills for Nursing, 8th Edition

MULTIPLE CHOICE

1. During a health interview, an older adult patient has difficulty remembering information
about the health history. In order to get the information more reliably, the nurse should:
a. repeat the questions at the end of the visit to cross check for accuracy of data.
b. reassure the patient that forgetfulness is a normal part of the aging process.
c. gather information from a family member accompanying the patient.
d. omit the interview and proceed to a physical examination.
ANS: C
If an older adult has difficulty with memory, data may be gathered from a family member or
significant other.

DIF: Cognitive Level: Analysis REF: p. 376 OBJ: Theory #2


TOP: Data Collection KEY: Nursing Process Step: Implementation
MSC: NCLEX: Health Promotion and Maintenance: Prevention and Early Detection of
Disease

2. It is the responsibility of the nurse to perform a quick focused assessment of the patient
upon:
a. admission to the unit.
b. discharge.
c. the beginning of each shift.
d. the patient‘s wakening in the morning.
ANS: C
A quick focused assessment should be performed on each patient at the beginning of each
shift to monitor for subtle changes in condition. This assessment is not the full assessment
done on admission.

DIF: Cognitive Level: Comprehension REF: p. 375


OBJ: Clinical Practice #2 TOP: Quick Focused Assessment
KEY: Nursing Process Step: Assessment
MSC: NCLEX: Health Promotion and Maintenance: Prevention and Early Detection of
Disease

3. The nurse lightly palpates the abdomen of a patient during a physical examination. On
palpation to the right side of the abdomen, the patient cries out and draws the knees to the
chest. The nurse should:
a. discontinue the examination and report findings to the primary care provider.
b. palpate the abdominal skin 1.5 to 2 inches to determine the cause of pain.
c. continue the examination and have the patient take deep breaths.
d. proceed to percuss the abdomen with a quick snap of the wrist.
ANS: A

Downloaded by: SUCCEEDGRADES | abbieclin@gmail.com


Distribution
Downloaded by Dallen Mae D.ofKadir
this document is illegal
(kadir.dallenmae.d.bcsi@gmail.com)
lOMoARcPSD|35346190

Stuvia.com - The Marketplace to Buy and Sell your Study Material

When palpating, the nurse should observe the patient‘s face for signs of discomfort and
discontinue palpations if they appear to cause pain.

DIF: Cognitive Level: Analysis REF: p. 380 OBJ: Theory #3


TOP: Physical Examination Techniques KEY: Nursing Process Step: Assessment
MSC: NCLEX: Physiological Integrity: Basic Care and Comfort

4. When performing deep palpation, the nurse should:


a. use one hand and exert pressure to depress tissue about one half to three fourths of
an inch.
b. use either one or two hands to depress the tissue about 1 inch.
c. use either one or two hands to depress the tissue about 1.5 to 2 inches.
d. use two hands and exert pressure to depress the tissue about 3 to 4 inches.
ANS: C
Deep palpation uses either hand to depress the tissue 1.5 to 2 inches.

DIF: Cognitive Level: Comprehension REF: p. 389 OBJ: Theory #3


TOP: Deep Palpation KEY: Nursing Process Step: Assessment
MSC: NCLEX: Health Promotion and Maintenance: Prevention and Early Detection of
Disease

5. To the breath sounds of a patient correctly, the nurse should:


a. inspect the chest wall for characteristics of movements and respirations.
b. use a stethoscope and properly position the earpieces and diaphragm.
c. percuss the chest by quickly tapping on the chest wall surface to produce sounds.
d. touch the chest wall and note the texture, temperature, and moisture of the skin.
ANS: B
Auscultation requires properly placing the earpieces in the ears pointing forward toward the
nose and using the diaphragm to auscultate for breath sounds.

DIF: Cognitive Level: Knowledge REF: p. 380


OBJ: Clinical Practice #2 TOP: Physical Assessment
KEY: Nursing Process Step: Planning
MSC: NCLEX: Physiological Integrity: Basic Care and Comfort

6. The nurse weighing an infant in an outpatient clinic should:


a. place a towel on the scale prior to weighing the baby.
b. place the baby in a prone position on the scale to reduce the infant‘s movement.
c. keep one hand hovering over the infant during the weighing procedure.
d. rest a hand lightly on the infant‘s abdomen during weighing to prevent a fall.
ANS: C
The nurse should keep a hand hovering over the infant during the weighing to prevent a fall.
Infants are never left unattended on a scale.

DIF: Cognitive Level: Application REF: p. 381 OBJ: Theory #3


TOP: Basic Physical Assessment: Safety, Weight
KEY: Nursing Process Step: Implementation

Downloaded by: SUCCEEDGRADES | abbieclin@gmail.com


Distribution
Downloaded by Dallen Mae D.ofKadir
this document is illegal
(kadir.dallenmae.d.bcsi@gmail.com)
lOMoARcPSD|35346190

Stuvia.com - The Marketplace to Buy and Sell your Study Material

MSC: NCLEX: Health Promotion and Maintenance: Prevention and Early Detection of
Disease

7. An 8-year-old patient is due for height measurement during a routine examination. For an
accurate measurement to be obtained, the child should be asked to stand:
a. with back toward the rod and the feet 6 inches apart.
b. with back toward the rod and the feet centered together.
c. with front toward the rod and the feet 8 inches apart.
d. with front toward the rod and the feet 4 inches apart.
ANS: B
The patient stands with back toward and the feet together and centered under the height rod
attached to a standing scale.

DIF: Cognitive Level: Comprehension REF: p. 381


OBJ: Clinical Practice #2 TOP: Basic Physical Assessment: Height
KEY: Nursing Process Step: Planning
MSC: NCLEX: Physiological Integrity: Basic Care and Comfort

8. The nurse taking a blood pressure should:


a. place the arm so that the brachial artery is at waist level.
b. position the patient so that the arm is level with the shoulder.
c. request that the patient put feet flat on the floor.
d. chat with the patient to reduce any anxiety in the patient.
ANS: C
For best accuracy, the patient should have both feet flat on the floor and the brachial artery
at the level of the right atrium.

DIF: Cognitive Level: Application REF: p. 382 OBJ: Theory #3


TOP: Vital Signs: Blood Pressure KEY: Nursing Process Step: Implementation
MSC: NCLEX: Physiological Integrity: Physiological Adaptation

9. When the heart is assessed for the point of maximal impulse (PMI), the stethoscope should
be placed on the:
a. fifth intercostal space, left midclavicular line.
b. fifth intercostal space, left anterior axillary line.
c. second intercostal space, right midclavicular line.
d. fourth intercostal space, left lateral sternal border.
ANS: A
The PMI of the heart is located close to the fifth intercostal space at the left midclavicular
line.

DIF: Cognitive Level: Knowledge REF: p. 383 OBJ: Theory #3


TOP: Physical Examination Techniques KEY: Nursing Process Step: Planning
MSC: NCLEX: Physiological Integrity: Basic Care and Comfort

10. A nurse caring for a patient on bed rest with a history of respiratory health problems should:
a. monitor for skin turgor every shift.

Downloaded by: SUCCEEDGRADES | abbieclin@gmail.com


Distribution
Downloaded by Dallen Mae D.ofKadir
this document is illegal
(kadir.dallenmae.d.bcsi@gmail.com)
lOMoARcPSD|35346190

Stuvia.com - The Marketplace to Buy and Sell your Study Material

b. monitor peripheral pulses once a shift.


c. auscultate lung sounds at the beginning of a shift.
d. auscultate for bowel sounds once a shift.
ANS: C
Auscultation of lungs is performed on initial assessment for persons with respiratory
problems and once a shift for patients on bed rest.

DIF: Cognitive Level: Application REF: p. 385 OBJ: Theory #3


TOP: Physical Examination Techniques KEY: Nursing Process Step: Implementation
MSC: NCLEX: Physiological Integrity: Basic Care and Comfort

11. The nurse assessing a patient‘s capillary refill finds that it took 5 seconds for the color to
return. The most appropriate intervention to do following this assessment is to:
a. assess the radial pulse and the blood pressure.
b. document the results as normal.
c. repeat the assessment.
d. notify the charge nurse.
ANS: C
If the color returns slowly, check the capillary refill again, because normal refill time is less
than 3 seconds.

DIF: Cognitive Level: Analysis REF: p. 385 OBJ: Theory #3


TOP: Capillary Refill KEY: Nursing Process Step: Implementation
MSC: NCLEX: Physiological Integrity: Basic Care and Comfort

12. The nurse is aware that the best way to assess dependent pitting edema in a patient with
congestive heart failure is to:
a. measure the circumference of the ankles daily.
b. inquire whether the patient‘s shoes fit tightly.
c. auscultate lung sounds every shift.
d. press fingers into the tissue over the tibia, just above the ankle.
ANS: D
When the nurse presses his fingers into the tissue over the tibia just above the ankle, an
indentation mark appears as evidence of pitting edema.

DIF: Cognitive Level: Comprehension REF: p. 385 OBJ: Theory #3


TOP: Pitting Edema KEY: Nursing Process Step: Implementation
MSC: NCLEX: Physiological Integrity: Basic Care and Comfort

13. A nurse records absence of bowel sounds after assessing the abdomen:
a. in the two lower quadrants for 2 minutes each.
b. in the two upper quadrants for 5 minutes.
c. in all quadrants for 3 minutes each.
d. in each quadrant for 1 minute.
ANS: C

Downloaded by: SUCCEEDGRADES | abbieclin@gmail.com


Distribution
Downloaded by Dallen Mae D.ofKadir
this document is illegal
(kadir.dallenmae.d.bcsi@gmail.com)
lOMoARcPSD|35346190

Stuvia.com - The Marketplace to Buy and Sell your Study Material

Auscultate for bowel sounds with the patient in a supine position. An absence of bowel
sounds should be documented after there are no bowel sounds in each quadrant for 2 to 5
minutes.

DIF: Cognitive Level: Comprehension REF: p. 387 OBJ: Theory #3


TOP: Absence of Bowel Sound KEY: Nursing Process Step: Implementation
MSC: NCLEX: Health Promotion and Maintenance: Prevention and Early Detection of
Disease

14. The nurse who is assessing the patient with the Glasgow Coma Scale finds a patient who
can open his eyes spontaneously, obeys all commands, and is oriented. The nurse documents
a score of:
a. 7.
b. 10.
c. 12.
d. 15.
ANS: D
The Glasgow Coma Scale is used for the evaluation of neurologically impaired patients:
spontaneous eye opening = 4, obeying commands = 6, orientation = 5. This is the highest
possible score of 15.

DIF: Cognitive Level: Analysis REF: p. 396|Table 22-3


OBJ: Clinical Practice #4 TOP: Glasgow Coma Scale
KEY: Nursing Process Step: Implementation
MSC: NCLEX: Health Promotion and Maintenance: Prevention and Early Detection of
Disease

15. Prior to preparing a female patient for a pelvic examination, the nurse should:
a. encourage her to void in the bathroom.
b. provide a pillow for the head and the hips.
c. hand the patient a sheet and allow her to drape herself.
d. cleanse the external genitalia with soap and water.
ANS: A
Before a pelvic examination, the bladder should be emptied for a more effective
examination.

DIF: Cognitive Level: Application REF: p. 392 OBJ: Theory #3


TOP: Special Focus Examinations KEY: Nursing Process Step: Implementation
MSC: NCLEX: Physiological Integrity: Basic Care and Comfort

16. To perform the Weber test, the tuning fork is struck and placed:
a. at the nape of the neck.
b. in the middle of the bridge of the nose.
c. behind the right and then the left ear.
d. in the middle of the forehead or skull.
ANS: D

Downloaded by: SUCCEEDGRADES | abbieclin@gmail.com


Distribution
Downloaded by Dallen Mae D.ofKadir
this document is illegal
(kadir.dallenmae.d.bcsi@gmail.com)
lOMoARcPSD|35346190

Stuvia.com - The Marketplace to Buy and Sell your Study Material

The Weber test is performed by striking the tuning fork and placing it in the middle of the
patient‘s forehead or skull.

DIF: Cognitive Level: Comprehension REF: p. 393 OBJ: Theory #3


TOP: Auditory Examination KEY: Nursing Process Step: Implementation
MSC: NCLEX: Health Promotion and Maintenance: Prevention and Early Detection of
Disease

17. When examining a patient‘s pupils with a light, the nurse notes that both pupils constrict,
regardless of which eye is stimulated by the light. The nurse should document that the pupils
exhibit:
a. consensual reflex
b. brisk reflex.
c. accommodation.
d. dilation reflex.
ANS: A
Consensual reflex is when both pupils constrict when either eye is stimulated by light.

DIF: Cognitive Level: Comprehension REF: p. 394


OBJ: Clinical Practice #4 TOP: Pupillary Response
KEY: Nursing Process Step: Assessment
MSC: NCLEX: Physiological Integrity: Physiological Adaptation

18. To correctly determine the strength of a patient‘s lower extremities during a neurological
examination, the nurse asks the patient to:
a. wiggle the toes of both feet at the same time.
b. push against his hand with the sole of one foot and then the other.
c. pull both feet up at the same time to stretch the Achilles tendons.
d. stand up independently.
ANS: B
To test extremity muscle strength, the nurse should have the patient push against his hand
with the sole of one foot and then with the other.

DIF: Cognitive Level: Comprehension REF: p. 395


OBJ: Clinical Practice #4 TOP: Review of Body Systems: Neurologic
KEY: Nursing Process Step: Assessment
MSC: NCLEX: Physiological Integrity: Physiological Adaptation

19. The nurse tells a patient that he will be performing a visual acuity test using the Snellen eye
chart. The patient asks how the test is done. The nurse‘s best reply is:
a. ―You stand 50 feet away from the chart while I test each of your eyes.‖
b. ―I will be testing your vision with your reading glasses on.‖
c. ―You stand 20 feet away from the chart while I test each of your eyes.‖
d. ―The number beside the largest print read is your visual acuity score.‖
ANS: C
To perform a visual acuity test using a Snellen eye chart, you should position the patient 20
feet from the chart for the test to be accurate.

Downloaded by: SUCCEEDGRADES | abbieclin@gmail.com


Distribution
Downloaded by Dallen Mae D.ofKadir
this document is illegal
(kadir.dallenmae.d.bcsi@gmail.com)
lOMoARcPSD|35346190

Stuvia.com - The Marketplace to Buy and Sell your Study Material

DIF: Cognitive Level: Comprehension REF: p. 383|Step 22-2


OBJ: Clinical Practice #3 TOP: Visual Testing
KEY: Nursing Process Step: Implementation
MSC: NCLEX: Health Promotion and Maintenance: Basic Care and Comfort

20. The nurse is informed that a patient had abnormal heart sounds during the night shift. When
auscultating abnormal heart sounds, the nurse knows to listen to heart sounds with the:
a. bell of the stethoscope directly on the patient‘s skin.
b. bell of the stethoscope on top of the patient‘s gown.
c. diaphragm of the stethoscope directly on the patient‘s skin.
d. diaphragm of the stethoscope on top of the patient‘s gown.
ANS: A
Abnormal heart sounds are best heard when the bell of the stethoscope, which picks up
lower pitched sounds, is placed against the patient‘s skin.

DIF: Cognitive Level: Comprehension REF: p. 380


OBJ: Clinical Practice #4
TOP: Physical Examination Techniques: Heart
KEY: Nursing Process Step: Planning
MSC: NCLEX: Physiological Integrity: Basic Care and Comfort

21. Before starting the initial morning care or the physical assessment of the patient, the first
intervention the nurse would perform would be:
a. putting down the side rails.
b. washing his or her hands.
c. placing the bed at working height.
d. turning on the overhead light.
ANS: B
The nurse should wash his or her hands before starting a procedure to prevent the spread of
microorganisms.

DIF: Cognitive Level: Application REF: p. XX|Steps 22 3


OBJ: Clinical Practice #2 TOP: Physical Assessment
KEY: Nursing Process Step: Implementation
MSC: NCLEX: Safe, Effective Care Environment

22. A patient was admitted with possible head trauma after a motor vehicle accident. The
nursing implementation with the highest priority is to:
a. monitor intake and output.
b. auscultate lung and abdominal sounds.
c. check for verbal and motor response.
d. monitor daily weight.
ANS: C
A neurological examination that includes the Glasgow Coma Scale is performed for any
patient who has sustained a head injury.

Downloaded by: SUCCEEDGRADES | abbieclin@gmail.com


Distribution
Downloaded by Dallen Mae D.ofKadir
this document is illegal
(kadir.dallenmae.d.bcsi@gmail.com)
lOMoARcPSD|35346190

Stuvia.com - The Marketplace to Buy and Sell your Study Material

DIF: Cognitive Level: Application REF: p. 396|Table 22-3


OBJ: Clinical Practice #4 TOP: Physical Examination: Neurologic
KEY: Nursing Process Step: Implementation
MSC: NCLEX: Physiological Integrity: Basic Care and Comfort

23. A nurse is instructing a nursing student on performing pupillary checks on a patient with a
possible head injury. Which statement indicates that the nursing student understands the
concept?
a. ―When I shine a light into the patient‘s eyes, the pupils should constrict.‖
b. ―When I shine a light into the patient‘s eyes, the pupils should dilate.‖
c. ―It is normal for the pupils to react sluggishly to light.‖
d. ―Pupil checks should be performed with the room lights on.‖
ANS: A
The pupils should constrict briskly when a light is shone.

DIF: Cognitive Level: Comprehension REF: p. 394|Skill 22-2


OBJ: Clinical Practice #4 TOP: Health Education
KEY: Nursing Process Step: N/A
MSC: NCLEX: Physiological Integrity: Basic Care and Comfort

24. An older adult American Indian patient has been admitted to the hospital with abdominal
pain. Along with performing a physical assessment, the nurse should also perform a:
a. psychological history.
b. financial history.
c. cultural assessment.
d. literacy assessment.
ANS: C
Cultural assessment should be done to determine the cultural preferences and health beliefs
to better understand how illness is affecting a patient‘s life.

DIF: Cognitive Level: Application REF: p. 376


OBJ: Clinical Practice #1 TOP: Culture
KEY: Nursing Process Step: Assessment
MSC: NCLEX: Psychosocial Integrity: Coping and Adaptation

25. A female patient of Asian descent was admitted to the medical-surgical unit with possible
lung cancer. A male nurse is preparing to perform a physical assessment. It is best for the
male nurse to:
a. ask the family members to leave the room to ensure patient privacy.
b. perform the procedure accurately and quickly to lessen patient anxiety.
c. examine only the affected body systems to decrease patient discomfort.
d. ask the patient for permission to perform the assessment before starting.
ANS: D
Many cultures do not permit the touching of a female by a male outside of the family; the
male nurse should ask for permission before touching her.

DIF: Cognitive Level: Application REF: p. 378|Cultural Considerations

Downloaded by: SUCCEEDGRADES | abbieclin@gmail.com


Distribution
Downloaded by Dallen Mae D.ofKadir
this document is illegal
(kadir.dallenmae.d.bcsi@gmail.com)
lOMoARcPSD|35346190

Stuvia.com - The Marketplace to Buy and Sell your Study Material

OBJ: Clinical Practice #1 TOP: Culture


KEY: Nursing Process Step: Assessment
MSC: NCLEX: Psychosocial Integrity: Psychosocial Adaptation

MULTIPLE RESPONSE

1. When educating a patient about the warning signs of cancer, the nurse includes which of the
following? (Select all that apply.)
a. Difficulty in swallowing
b. Persistent cough
c. Hyperactive bowel sounds
d. Vesicular breath sounds
e. Changes in pulse rate
f. Obvious change in a mole
ANS: A, B, C, F
The warning signs of cancer are changes in bowel or bladder habits, a sore that does not
heal, unusual bleeding or discharge, thickening or a lump in a breast, indigestion or
difficulty swallowing, a nagging cough, and changes in a wart or mole.

DIF: Cognitive Level: Comprehension REF: p. 388


OBJ: Clinical Practice #5 TOP: Health Education
KEY: Nursing Process Step: Implementation
MSC: NCLEX: Health Promotion and Maintenance: Prevention and Early Detection of
Disease

2. The nurse uses the technique of inspection to initially assess: (Select all that apply.)
a. skin tone.
b. skin turgor.
c. body contours.
d. color.
e. characteristics of movement.
ANS: A, C, D, E
Inspection can initially assess skin tone, body contours, color, characteristics of movement,
skin lesions, and obvious weakness.

DIF: Cognitive Level: Comprehension REF: p. 378 OBJ: Theory #1


TOP: Inspection KEY: Nursing Process Step: Assessment
MSC: NCLEX: Health Promotion and Maintenance: Prevention and Early Detection of
Disease

3. Percussion is a technique by which the nurse can assess sounds relative to the underlying
structures that indicate the presence of: (Select all that apply.)
a. air.
b. infection.
c. fluid.
d. the inflammatory process.
e. a solid organ.

Downloaded by: SUCCEEDGRADES | abbieclin@gmail.com


Distribution
Downloaded by Dallen Mae D.ofKadir
this document is illegal
(kadir.dallenmae.d.bcsi@gmail.com)
lOMoARcPSD|35346190

Stuvia.com - The Marketplace to Buy and Sell your Study Material

ANS: A, C, E
Percussion is a technique that causes difference resonance of the underlying structures.
Percussion can detect the presence of air or fluid or the location of a solid organ.

DIF: Cognitive Level: Comprehension REF: p. 380 OBJ: Theory #1


TOP: Percussion KEY: Nursing Process Step: Assessment
MSC: NCLEX: Health Promotion and Maintenance: Prevention and Early Detection of
Disease

COMPLETION

1. Prior to assessing a patient‘s blood pressure in both arms, the nurse will instruct the patient
to lie down for at least minutes.

ANS:
5

When blood pressure is measured in both arms, the nurse should instruct the patient to sit or
lie down for at least 5 minutes.

DIF: Cognitive Level: Application REF: p. 382 OBJ: Theory #1


TOP: Blood Pressure KEY: Nursing Process Step: Assessment
MSC: NCLEX: Physiological Integrity: Physiological Adaptation

2. The nurse who is assessing a patient for heart sounds anticipates that the S2 sound (the ―dub‖
sound‖) can be heard best at the area.

ANS:
aorta

The S2 sound, or the ―dub‖ sound, is heard most clearly over the aortic area.

DIF: Cognitive Level: Application REF: p. 382 OBJ: Theory #3


TOP: Heart Sounds KEY: Nursing Process Step: Assessment
MSC: NCLEX: Physiological Integrity: Basic Care and Comfort

3. The nurse is assessing a patient‘s heart sounds and hears a ―swish‖ that is recorded as a
.

ANS:
murmur

Heart murmurs are heard in addition to the S1 and S2.

DIF: Cognitive Level: Comprehension REF: p. 384 OBJ: Theory #3


TOP: Murmurs KEY: Nursing Process Step: Assessment
MSC: NCLEX: Health Promotion and Maintenance: Prevention and Early Detection of
Disease

Downloaded by: SUCCEEDGRADES | abbieclin@gmail.com


Distribution
Downloaded by Dallen Mae D.ofKadir
this document is illegal
(kadir.dallenmae.d.bcsi@gmail.com)
lOMoARcPSD|35346190

Stuvia.com - The Marketplace to Buy and Sell your Study Material

4. The nurse is assessing a patient‘s lung sounds and hears a wheeze in the lower left lobe. This
wheeze is categorized as a(n) sound.

ANS:
adventitious

Adventitious sounds are abnormal breath sounds that should be documented and reported.

DIF: Cognitive Level: Knowledge REF: p. 384 OBJ: Theory #3


TOP: Adventitious Sounds KEY: Nursing Process Step: Assessment
MSC: NCLEX: Health Promotion and Maintenance: Prevention and Early Detection of
Disease

5. The nurse notes that a patient has an exaggerated lumbar curve. This is indicative of
.

ANS:
lordosis

Lordosis is characterized by an exaggerated lumbar curve.

DIF: Cognitive Level: Comprehension REF: p. 383 OBJ: Theory #2


TOP: Lordosis KEY: Nursing Process Step: Assessment
MSC: NCLEX: Health Promotion and Maintenance: Prevention and Early Detection of
Disease

6. The nurse is aware that the most accurate quick method to check hydration status in the
older adult is to evaluate the moisture of the .

ANS:
mucous membranes

The loss of skin elasticity makes the evaluation of skin turgor ineffective in the elderly. An
assessment of the moisture of the mucous membranes is a more reliable evaluation.

DIF: Cognitive Level: Application REF: p. 385 OBJ: Theory #2


TOP: Hydration Status in the Older Adult
KEY: Nursing Process Step: Assessment
MSC: NCLEX: Health Promotion and Maintenance: Prevention and Early Detection of
Disease

7. The nurse takes special care in the draping of a patient in the lithotomy position in order to
diminish .

ANS:
exposure
embarrassment

A patient who feels exposed and embarrassed will be tense and less able to cooperate.

Downloaded by: SUCCEEDGRADES | abbieclin@gmail.com


Distribution
Downloaded by Dallen Mae D.ofKadir
this document is illegal
(kadir.dallenmae.d.bcsi@gmail.com)
lOMoARcPSD|35346190

Stuvia.com - The Marketplace to Buy and Sell your Study Material

DIF: Cognitive Level: Comprehension REF: p. 392


OBJ: Clinical Practice #7 TOP: Draping
KEY: Nursing Process Step: Implementation
MSC: NCLEX: Physiological Integrity: Basic Care and Comfort

8. When the nurse asks the neurologically impaired patient to follow the motion of the nurse‘s
fingers, the patient‘s eyes track the fingers with jerky movements, which should be
documented as .

ANS:
nystagmus

Nystagmus is the term used to describe jerky movements of the eye as it follows or tracks an
object such as the examiner‘s fingers.

DIF: Cognitive Level: Application REF: p. 395 OBJ: Theory #3


TOP: Nystagmus KEY: Nursing Process Step: Assessment
MSC: NCLEX: Health Promotion and Maintenance: Prevention and Early Detection of
Disease

Downloaded by: SUCCEEDGRADES | abbieclin@gmail.com


Distribution
Downloaded by Dallen Mae D.ofKadir
this document is illegal
(kadir.dallenmae.d.bcsi@gmail.com)
lOMoARcPSD|35346190

Stuvia.com - The Marketplace to Buy and Sell your Study Material

Chapter 23: Admitting, Transferring, and Discharging Patients


Williams: deWit's Fundamental Concepts and Skills for Nursing, 8th Edition

MULTIPLE CHOICE

1. The nurse is aware that patients who are admitted to the acute care facility as a routine
admission under a managed care plan must:
a.have Medicare.
b.be preapproved.
c.be able to pay the deductible.
d.be admitted several days prior to the procedure.
ANS: B
Managed care insurance programs require that all routine admissions be preapproved. Often
the patient is required to come to the acute care facility several days prior to the admission to
complete paperwork or laboratory procedures.

DIF: Cognitive Level: Comprehension REF: p. 398 OBJ: Theory #1


TOP: Managed Care KEY: Nursing Process Step: Planning
MSC: NCLEX: Safe, Effective Care Environment: Coordinated Care

2. A patient is scheduled to have a diagnostic procedure performed on an outpatient basis at 9:00


AM. The nurse will advise the patient to:
a.arrive 2 hours before the scheduled procedure.
b.wear comfortable clothing.
c.read printed materials about the procedure.
d.be prepared to pay at least 10% of the predicted cost of the hospitalization.
ANS: A
Patients are usually required to arrive 1 to 2 hours before a scheduled procedure to complete
the necessary paperwork. The patient may be requested to pay a co-pay or a deductible, but
there is no set amount.

DIF: Cognitive Level: Comprehension REF: p. 399 OBJ: Theory #1


TOP: Outpatient Admission KEY: Nursing Process Step: Implementation
MSC: NCLEX: Safe, Effective Care Environment: Coordinated Care

3. If there is a prior authorization for hospitalization required for a routine admission, the nurse
explains that the notification to the insurance company is the responsibility of the:
a. patient.
b. admissions department of the health facility.
c. patient, primary care provider, and the admissions department.
d. office of the admitting primary care provider.
ANS: D
The office of the admitting primary care provider is the usual agent to get prior approval for a
hospitalization. The admissions department confirms that all preadmission requirements are
met.

DIF: Cognitive Level: Knowledge REF: p. 399 OBJ: Theory #1


TOP: Admission Procedures KEY: Nursing Process Step: Implementation

Downloaded by: SUCCEEDGRADES | abbieclin@gmail.com


Distribution
Downloaded by Dallen Mae D.ofKadir
this document is illegal
(kadir.dallenmae.d.bcsi@gmail.com)
lOMoARcPSD|35346190

Stuvia.com - The Marketplace to Buy and Sell your Study Material

MSC: NCLEX: Safe, Effective Care Environment: Coordinated Care

4. The nurse orienting a new patient to the unit would include:


a. expected cost of the room per day.
b. location of call bell and how to use it.
c. calling the patient by their first name for less formality.
d. times of the shift changes.

ANS: B
Newly admitted patients should be treated with respect without familiarity. The physical
arrangement of the room and bath, how to work all controls, such as the call bell, and the
names of the nurses who will be giving care should be included in the orientation.

DIF: Cognitive Level: Comprehension REF: p. 400 OBJ: Clinical Practice #1


TOP: Admission KEY: Nursing Process Step: Implementation
MSC: NCLEX: Psychosocial Integrity: Coping and Adaptation

5. A patient admitted to the acute care facility through the emergency department has jewelry
and a large amount of money. The most efficient intervention about these valuables would be:
a. send them home with a family member.
b. put them away quickly in the patient‘s closet.
c. lock them in the narcotics cabinet on the nursing unit.
d. place them in a valuables envelope and have them locked in the agency safe.
ANS: A
Valuables such as credit cards, money, or jewelry should be sent home with a family member.

DIF: Cognitive Level: Application REF: p. 400 OBJ: Clinical Practice #1


TOP: Admission KEY: Nursing Process Step: Implementation
MSC: NCLEX: Physiological Integrity: Basic Care and Comfort

6. While admitting a patient from home to the skilled nursing facility, the nurse notes that the
patient has brought medications that are not included on the primary care provider‘s
medication order sheet. The nurse‘s best initial action is to:
a. send the medications home with a family member.
b. seal the medications in an envelope and lock it in the medicine cart.
c. administer the medications with the ordered medications.
d. notify the physician about the medications the patient has been taking.

ANS: D
It is important to notify the primary care provider of any medications the patient has been
taking at home that are not included in the present orders.

DIF: Cognitive Level: Application REF: p. 401 OBJ: Clinical Practice #1


TOP: Admission KEY: Nursing Process Step: Implementation
MSC: NCLEX: Safe, Effective Care Environment: Safety and Infection Control

7. New orders have been written by the primary care provider for a patient admitted to a skilled
nursing facility. After transcription, the orders will be verified by the:
a. unit secretary.
b. administrative RN.
c. LPN/LVN in charge.

Downloaded by: SUCCEEDGRADES | abbieclin@gmail.com


Distribution
Downloaded by Dallen Mae D.ofKadir
this document is illegal
(kadir.dallenmae.d.bcsi@gmail.com)
lOMoARcPSD|35346190

Stuvia.com - The Marketplace to Buy and Sell your Study Material

d. director of nurses.
ANS: C
In most skilled nursing facilities, verification of orders (checking and signing them) is
performed by the LPN/LVN.

DIF: Cognitive Level: Application REF: p. 402 OBJ: Clinical Practice #1


TOP: Orders in Skilled Nursing Facility KEY: Nursing Process Step: Implementation
MSC: NCLEX: Safe, Effective Care Environment: Coordinated Care

8. When the nurse discovers that the patient‘s consent form for an invasive procedure was
transcribed incorrectly, the nurse should:
a. cross out the incorrect information and write error, and then write in the correct
information.
b. destroy the incorrect form and write a new one correctly.
c. cross out the entire form, but leave it in the medical record as a permanent record.
d. notify the primary care provider of the error and clarify what the primary care
provider prefers to be done.
ANS: B
The consent form is considered a legal document and should be transcribed accurately to
prevent errors; the incorrect one should be destroyed.

DIF: Cognitive Level: Application REF: p. 401 OBJ: Clinical Practice #1


TOP: Consent Forms KEY: Nursing Process Step: Implementation
MSC: NCLEX: Safe, Effective Care Environment: Coordinated Care

9. A patient‘s condition warrants a transfer from intensive care to a regular nursing unit in the
same acute care facility. Before assisting with the patient‘s move, the nurse notes that the
transfer has been authorized by the:
a. patient.
b. charge nurse.
c. primary care provider.
d. family.

ANS: C
In general, transfers from one nursing area to another require a specific order by the primary
care provider. The charge nurse of the receiving unit should be notified as well as the patient
and family.

DIF: Cognitive Level: Application REF: p. 402 OBJ: Clinical Practice #3


TOP: Transfer KEY: Nursing Process Step: Implementation
MSC: NCLEX: Safe, Effective Care Environment: Coordinated Care

10. A newly admitted patient has his own walker that he wishes to use during this acute care
facility stay. The responsibility of the nurse to this piece of durable equipment is to:
a. write the patient‘s name on a wide piece of tape and affix it to the walker.
b. list the walker as part of the patient‘s personal belongings and place the list in his
medical record.
c. tell the patient that personal walkers cannot be used in the acute care facility for
safety reasons.
d. document a note in the nursing care plan that the patient has his own walker.

Downloaded by: SUCCEEDGRADES | abbieclin@gmail.com


Distribution
Downloaded by Dallen Mae D.ofKadir
this document is illegal
(kadir.dallenmae.d.bcsi@gmail.com)
lOMoARcPSD|35346190

Stuvia.com - The Marketplace to Buy and Sell your Study Material

ANS: A
All equipment brought to the acute care facility by the patient should be clearly labeled,
usually with a wide piece of tape on which the patient‘s name is written in large letters.

DIF: Cognitive Level: Application REF: p. 402 OBJ: Theory #3


TOP: Admission KEY: Nursing Process Step: Implementation
MSC: NCLEX: Physiological Integrity: Basic Care and Comfort

11. The nurse helping to organize the transfer of an older adult patient from the acute care facility
to an extended care facility will be sure to:
a. check drawers and shelves for personal items.
b. give unused medications to the patient.
c. ask the business office to send stored valuables to the receiving facility.
d. send a small snack with the patient.
ANS: A
Checking drawers and shelves for personal items prior to a transfer is helpful in preventing
loss.

DIF: Cognitive Level: Knowledge REF: p. 402 OBJ: Clinical Practice #3


TOP: Transfer KEY: Nursing Process Step: Planning
MSC: NCLEX: Safe, Effective Care Environment: Coordinated Care

12. As a member of the health care team, the LPN/LVN understands that discharge planning for
the hospitalized patient begins:
a. the day before discharge.
b. at the time of admission.
c. immediately following diagnostic procedures or surgery.
d. as soon as a family meeting is scheduled.
ANS: B
Discharge planning begins at admission, especially if the diagnosis indicates that the patient
will need rehabilitation or long-term assistance.

DIF: Cognitive Level: Comprehension REF: p. 403 OBJ: Theory #5


TOP: Discharge KEY: Nursing Process Step: Planning
MSC: NCLEX: Safe, Effective Care Environment: Coordinated Care

13. A patient who is scheduled for discharge has items that were stored in the hospital safe. After
retrieving them, the nurse should document their return to the patient by:
a. making an entry in the primary care provider progress notes.
b. writing a note to the charge nurse.
c. having the patient sign for them as per policy.
d. asking the unit secretary to place a note in the chart.

ANS: C
Retrieve any valuables stored in the acute care facility safe before discharge and have the
patient sign according to policy and procedure.

DIF: Cognitive Level: Application REF: p. 403 OBJ: Theory #5


TOP: Discharge KEY: Nursing Process Step: Implementation
MSC: NCLEX: Safe, Effective Care Environment: Coordinated Care

Downloaded by: SUCCEEDGRADES | abbieclin@gmail.com


Distribution
Downloaded by Dallen Mae D.ofKadir
this document is illegal
(kadir.dallenmae.d.bcsi@gmail.com)
lOMoARcPSD|35346190

Stuvia.com - The Marketplace to Buy and Sell your Study Material

14. A patient has an acute care facility discharge order for later that day. The LPN/LVN
understands that part of the discharge process to be performed by the registered nurse is:
a. packing the patient‘s personal belongings.
b. writing the discharge instructions.
c. assisting the patient to get dressed.
d. accompanying the patient to the acute care facility entrance.
ANS: B
Written discharge instructions are prepared by the RN. The remaining duties can be performed
by the LPN/LVN.

DIF: Cognitive Level: Application REF: p. 403 OBJ: Theory #5


TOP: Discharge KEY: Nursing Process Step: Implementation
MSC: NCLEX: Safe, Effective Care Environment: Coordinated Care

15. A patient who has questions about the availability of home health services after hospital
discharge should be referred to the:
a. primary care provider.
b. registered nurse.
c. occupational therapist.
d. medical social worker (MSW).
ANS: D
An MSW can provide information about long-term planning, financial assistance, and
community services available after discharge.

DIF: Cognitive Level: Application C REF: p. 404 OBJ: Clinical Practice #5


TOP: Discharge KEY: Nursing Process Step: Planning
MSC: NCLEX: Safe, Effective Care Environment: Coordinated Care

16. A hospitalized patient tells the nurse that he intends to leave the acute care facility, against
medical advice. The nurse‘s initial action(s) should be to:
a. listen to the patient, answer questions, and offer to have the supervisor or physician
speak with the patient.
b. advise the patient that this may mean that insurance would not pay for this
hospitalization.
c. obtain a written explanation of the reasons from the patient and have the patient
sign at the bottom of the sheet.
d. call both the supervisor and a family member to try to get the patient to reconsider.
ANS: A
It is the responsibility of the health team to help patients understand the significance of
leaving against medical advice. Listen to what the patient has to say and offer to help get the
problem resolved without resorting to a discharge. If the ultimate decision is to leave, the
primary care provider is notified and the patient is asked to sign a form indicating that he or
she is leaving against medical advice.

DIF: Cognitive Level: Application REF: p. 404 OBJ: Theory #5


TOP: Discharge Against Medical Advice (AMA)
KEY: Nursing Process Step: Implementation
MSC: NCLEX: Psychosocial Integrity: Coping and Adaptation

Downloaded by: SUCCEEDGRADES | abbieclin@gmail.com


Distribution
Downloaded by Dallen Mae D.ofKadir
this document is illegal
(kadir.dallenmae.d.bcsi@gmail.com)
lOMoARcPSD|35346190

Stuvia.com - The Marketplace to Buy and Sell your Study Material

17. A patient is near death and the family is upset and disorganized. The most helpful intervention
for the patient and the family would be for the nurse to:
a. ask the family the name of their mortician.
b. offer to call the spiritual advisor (eg, priest, minister, or rabbi).
c. encourage the family to perform their rituals.
d. encourage the family to visit the chapel.
ANS: B
If death is anticipated, many people derive significant comfort from spiritual or religious
beliefs or practices.

DIF: Cognitive Level: Application REF: p. 404 OBJ: Clinical Practice #6


TOP: Death of a Patient KEY: Nursing Process Step: Implementation
MSC: NCLEX: Psychosocial Integrity: Coping and Adaptation

18. For the nurse to provide support to families of patients who have died, it is most important to:
a. have an understanding that all people deal with death in due time.
b. read a number of articles about death and dying.
c. have a personal experience of a similar nature.
d. deal with personal feelings about death and dying.
ANS: D
Before someone can be a support person to someone who has lost a loved one, he or she must
have dealt with personal feelings about death.

DIF: Cognitive Level: Comprehension REF: p. 404 OBJ: Clinical Practice #6


TOP: Death of a Patient KEY: Nursing Process Step: Implementation
MSC: NCLEX: Psychosocial Integrity: Psychosocial Adaptation

19. A nurse who was present at the time of the death of a patient should document:
a. time of death.
b. time at which life signs ceased.
c. notification of the mortuary.
d. which family members were notified.

ANS: B
It is still required in most states for a physician to pronounce death. The nurse should
document when all signs of life ceased. The name of the person making the pronouncement
and the time should be documented.

DIF: Cognitive Level: Knowledge REF: p. 404 OBJ: Theory #6


TOP: Pronouncement of Death KEY: Nursing Process Step: Planning
MSC: NCLEX: Safe, Effective Care Environment: Coordinated Care

20. A young patient has died in the emergency department after suffering severe trauma. The
nurse understands that this patient‘s family may take comfort from the opportunity to:
a. allow donation of the patient‘s organs.
b. view all of the injuries to the patient‘s body.
c. plan the funeral before leaving the acute care facility.
d. donate the patient‘s belongings to charity.

Downloaded by: SUCCEEDGRADES | abbieclin@gmail.com


Distribution
Downloaded by Dallen Mae D.ofKadir
this document is illegal
(kadir.dallenmae.d.bcsi@gmail.com)
lOMoARcPSD|35346190

Stuvia.com - The Marketplace to Buy and Sell your Study Material

ANS: A
When handled sensitively, requests for organ donation can be an opportunity for the family to
allow something good to come out of a personal tragedy.

DIF: Cognitive Level: Application REF: p. 405 OBJ: Clinical Practice #6


TOP: Organ Donation KEY: Nursing Process Step: Implementation
MSC: NCLEX: Psychosocial Integrity: Coping and Adaptation

21. A blind, older adult patient is admitted to the acute care facility for dehydration and weakness.
The nurse can make the admission process less stressful by:
a. sending all personal belongings home with family members.
b. performing the initial assessment in a nonhurried manner.
c. providing a printed orientation handout regarding acute care facility policy.
d. performing a quick assessment before orienting the patient to the unit.
ANS: B
Older adult patients need time and support in adjusting to an acute care facility stay. An
unhurried manner will show support and give the patient a little more time to adjust to the
change.

DIF: Cognitive Level: Application REF: p. 400 OBJ: Clinical Practice #3


TOP: Admission KEY: Nursing Process Step: Implementation
MSC: NCLEX: Psychosocial Integrity: Coping and Adaptation

22. It is determined that a patient is brain dead after suffering a massive cerebral bleed. The
primary care provider has just talked to the family about removing the patient from life
support. The nurse would anticipate:
a. calling the coroner‘s office.
b. calling the insurance company.
c. contacting the organ donation team.
d. asking about an autopsy.
ANS: C
Requests for organ donation are usually done by a primary care provider or a nurse trained for
making such requests.

DIF: Cognitive Level: Comprehension REF: p. 405 OBJ: Clinical Practice #6


TOP: Organ Donation KEY: Nursing Process Step: Implementation
MSC: NCLEX: Psychosocial Integrity

23. The nurse recognizes that an autopsy must be performed when the patient:
a. is over 52.
b. died suddenly, unexpectedly, or suspiciously.
c. has requested it on admission.
d. has died of a brain tumor.
ANS: B
Autopsies are performed when a death is sudden or unexpected, the result of injury or
suspicious circumstances (such as suspected homicide). A family may request an autopsy, but
they may have to incur the costs.

DIF: Cognitive Level: Knowledge REF: p. 405 OBJ: Theory #2

Downloaded by: SUCCEEDGRADES | abbieclin@gmail.com


Distribution
Downloaded by Dallen Mae D.ofKadir
this document is illegal
(kadir.dallenmae.d.bcsi@gmail.com)
lOMoARcPSD|35346190

Stuvia.com - The Marketplace to Buy and Sell your Study Material

TOP: Autopsy KEY: Nursing Process Step: Planning


MSC: NCLEX: Safe, Effective Care Environment: Coordinated Care

24. The nurse is performing an initial assessment on a patient with respiratory difficulty. The
nurse would anticipate documenting signs and symptoms such as:
a. alteration in sensation.
b. use of accessory muscles.
c. regular respiratory pattern.
d. excessive dryness.
ANS: B
An example of signs found in a patient with respiratory difficulty is use of accessory muscles
of respiration.

DIF: Cognitive Level: Comprehension REF: p. 401|Table 23-1


OBJ: Clinical Practice #2 TOP: Assessment
KEY: Nursing Process Step: Assessment
MSC: NCLEX: Physiological Integrity: Physiological Adaptation

25. The nurse is assisting with an admission assessment of a patient with hypertension. While the
nurse is preparing to weigh the patient, the patient states, ―It is not necessary to weigh me,
because I weighed 130 pounds last week.‖ What would be the nurse‘s best response?
a. ―Are you sure that your weight has not changed?‖
b. ―I will write down your stated weight.‖
c. ―It is important to get a more recent weight.‖
d. ―Don‘t worry; your weight is confidential.‖
ANS: C
The patient should be weighed and measured rather than the stated height and weight being
accepted.

DIF: Cognitive Level: Application REF: p. 401|Table 23-1


OBJ: Clinical Practice #2 TOP: Assessment
KEY: Nursing Process Step: Implementation
MSC: NCLEX: Physiological Integrity: Basic Care and Comfort

26. The nurse reminds a patient that if enrolled in a managed care program, some procedures will
not be approved for payment, such as:
a. cosmetic surgery to repair a scar from an accident.
b. breast augmentation.
c. emergency admission for shortness of breath.
d. postmastectomy breast implants.
ANS: B
Elective cosmetic surgeries are not covered by managed care companies.

DIF: Cognitive Level: Comprehension REF: p. 399 OBJ: Theory #1


TOP: Nonpayment for Procedures KEY: Nursing Process Step: Implementation
MSC: NCLEX: Safe, Effective Care Environment: Coordinated Care

27. When the orders have been verified, the nurse:


a. draws a line below the orders and signs his or her name and the date.

Downloaded by: SUCCEEDGRADES | abbieclin@gmail.com


Distribution
Downloaded by Dallen Mae D.ofKadir
this document is illegal
(kadir.dallenmae.d.bcsi@gmail.com)
lOMoARcPSD|35346190

Stuvia.com - The Marketplace to Buy and Sell your Study Material

b. signs his or her name in red immediately below the primary care provider‘s
signature.
c. documents, ―Transcribed by A Nurse at 0900.‖
d. draws a line down the left margin; then signs, dates, and times the transcription.
ANS: D
After the verification of the order, the nurse draws a line down the left-hand margin; then the
nurse signs, dates, and times the transcription.

DIF: Cognitive Level: Application REF: p. 402 OBJ: Clinical Practice #2


TOP: Documentation KEY: Nursing Process Step: Implementation
MSC: NCLEX: Safe, Effective Care Environment: Coordinated Care

MULTIPLE RESPONSE

1. The nurse is orienting an older adult patient newly admitted to the nursing unit. Which are
appropriate interventions to apply to alleviate the patient‘s anxiety? (Select all that apply.)
a. Call the patient by his first name.
b. Instruct the patient on the use of the call light.
c. Encourage the patient to ask questions regarding admission.
d. Allow extra time for the patient to process any new information.
e. Lock all patient valuables in the facility‘s safe storage.
ANS: B, C, D
Respectful and proper communication, especially during orientation of the patient to the
facility, alleviates anxiety. It is best to orient the patient to the room, including the use of the
call light, and to allow the patient time to process information and ask questions.

DIF: Cognitive Level: Application REF: p. 400 OBJ: Theory #3


TOP: Health Education KEY: Nursing Process Step: Implementation
MSC: NCLEX: Health Promotion and Maintenance

2. A patient dies after suffering a severe cerebrovascular accident (CVA). The family members
are informed of his demise and are at the bedside. What documentation should be noted in the
patient‘s medical record? (Select all that apply.)
a. Results of the autopsy
b. Who pronounced the patient
c. Official time of death
d. Time vital signs ceased
e. Why the patient died
ANS: B, C, D
Death must be accurately noted in the medical record and should include who pronounced the
patient‘s death, the time vital signs ceased, and the official time of death.

DIF: Cognitive Level: Application REF: p. 404 OBJ: Clinical Practice #6


TOP: Pronouncement of Death KEY: Nursing Process Step: N/A
MSC: NCLEX: Physiological Integrity

3. The nurse appreciates that a routine acute care facility admission differs from an emergency
admission in that a routine admission: (Select all that apply.)

Downloaded by: SUCCEEDGRADES | abbieclin@gmail.com


Distribution
Downloaded by Dallen Mae D.ofKadir
this document is illegal
(kadir.dallenmae.d.bcsi@gmail.com)
lOMoARcPSD|35346190

Stuvia.com - The Marketplace to Buy and Sell your Study Material

a. is scheduled in advance.
b. is not stressful.
c. is completely covered by insurance.
d. has a predictable outcome.
e. allows time to arrange for disruptions in routine.
ANS: A, E
Routine admissions are scheduled in advance with the full knowledge and permission of the
third-party payer. Routine admissions allow for time to arrange for disruptions in a family‘s
routine. All acute care facility admissions can be stressful and potentially have unpredictable
outcomes. Insurance may not completely cover the expense.

DIF: Cognitive Level: Comprehension REF: p. 398 OBJ: Theory #1


TOP: Routine Admissions KEY: Nursing Process Step: Planning
MSC: NCLEX: Safe, Effective Care Environment: Coordinated Care

4. The nurse explains that the Admitting Department of the acute care facility has a number of
significant duties, which include: (Select all that apply.)
a. arranging for preadmission laboratory work and radiographs.
b. notifying the patient‘s spiritual counselor of the admission.
c. confirming that all admission criteria are met.
d. arranging for special diets.
e. making arrangements for co-pays and deductibles.
ANS: C, E
The Admitting Department handles all the paperwork necessary for hospitalization prior to the
actual admission. They confirm that all preadmission studies have been done and the
insurance company is in accordance with the admission. They will also keep track of co-pays
and deductibles of the patient‘s insurance.

DIF: Cognitive Level: Comprehension REF: p. 399 OBJ: Theory #2


TOP: Admitting Department KEY: Nursing Process Step: Implementation
MSC: NCLEX: Safe, Effective Care Environment: Coordinated Care

COMPLETION

1. An examination of the remains of a body by a pathologist to determine the cause of death is


a(n) .

ANS:
autopsy

An autopsy is an examination of the remains by a pathologist to determine the cause of death.


An autopsy is usually performed when the patient has died of unknown causes, has died at the
hands of another, or has not been seen within a specific period of time by a primary care
provider.

DIF: Cognitive Level: Knowledge REF: p. 405 OBJ: Theory #6


TOP: Post Mortem KEY: Nursing Process Step: N/A
MSC: NCLEX: N/A

Downloaded by: SUCCEEDGRADES | abbieclin@gmail.com


Distribution
Downloaded by Dallen Mae D.ofKadir
this document is illegal
(kadir.dallenmae.d.bcsi@gmail.com)
lOMoARcPSD|35346190

Stuvia.com - The Marketplace to Buy and Sell your Study Material

Chapter 24: Diagnostic Tests and Specimen Collection


Williams: deWit's Fundamental Concepts and Skills for Nursing, 8th Edition

MULTIPLE CHOICE

1. A patient who is on an anticoagulant (Coumadin) asks, ―What did the primary care provider
mean when he said I was to have my blood tested every 2 weeks?‖ The nurse explains, ―It is
important to monitor the effects of the drug to see how long it takes your blood to clot. The
blood test the primary care provider was talking about is the:
a. complete blood count (CBC).‖
b. activated partial thromboplastin time (APTT).‖
c. international normalized ratio (INR).‖
d. erythrocyte sedimentation rate (ESR).‖

ANS: C
The INR is a method for reporting the prothrombin time, which is prolonged with warfarin
(Coumadin) therapy.

DIF: Cognitive Level: Knowledge REF: p. 410 OBJ: Theory #2


TOP: Blood Tests KEY: Nursing Process Step: Implementation
MSC: NCLEX: Physiological Integrity: Basic Care and Comfort

2. A patient is scheduled to have a blood chemistry profile drawn at 8 AM tomorrow. The note
should be added on the care plan and report provided to the oncoming shift to withhold food
and drink after:
a. 6 AM.
b. 12 midnight tonight.
c. 4 AM today.
d. noon today.
ANS: B
Food and drink are usually withheld for 8 to 12 hours before blood chemistry tests are
performed.

DIF: Cognitive Level: Application REF: p. 411 OBJ: Theory #3


TOP: Blood Tests KEY: Nursing Process Step: Implementation
MSC: NCLEX: Safe, Effective Care Environment: Coordinated Care

3. A patient wants to know what was meant when the doctor said that his white blood cell
(WBC) count had a shift to the left. The nurse explains that a shift to the left indicates:
a.an improvement in an infectious process.
b.the relative effectiveness of the antibiotic therapy.
c.an increase in the number of immature WBCs.
d.that the infection is viral in nature.
ANS: C
In reporting a differential WBC, the less mature WBCs are reported on the left side of the
page. An increase in immature WBCs causes the left side of the report to show large numbers
and indicates an infection.

DIF: Cognitive Level: Comprehension REF: p. 410 OBJ: Clinical Practice #1

Downloaded by: SUCCEEDGRADES | abbieclin@gmail.com


Distribution
Downloaded by Dallen Mae D.ofKadir
this document is illegal
(kadir.dallenmae.d.bcsi@gmail.com)
lOMoARcPSD|35346190

Stuvia.com - The Marketplace to Buy and Sell your Study Material

TOP: Differential KEY: Nursing Process Step: Implementation


MSC: NCLEX: Physiological Integrity: Physiological Adaptation

4. The nurse instructing in the collection of a midstream urine catch would tell the patient to first
cleanse the external genitalia and then to:
a. begin voiding into the specimen cup.
b. let a few drops of urine dribble into the specimen cup.
c. void until the bladder is almost empty and then collect the end portion of the
voiding in the cup.
d. pass a small amount of urine into the toilet and then collect the specimen.
ANS: D
To collect a midstream specimen, the external genitalia are cleansed, a small amount of urine
is passed, and then a midportion of the voiding is collected in a sterile container and used for a
culture.

DIF: Cognitive Level: Application REF: p. 411 OBJ: Clinical Practice #1


TOP: Urinalysis KEY: Nursing Process Step: Implementation
MSC: NCLEX: Physiological Integrity: Basic Care and Comfort

5. The nurse instructs an outpatient female patient preparing for an abdominal ultrasonography
that prior to the procedure, she should:
a. eat or drink nothing after midnight.
b. drink a liter of water.
c. empty the bladder fully.
d. use enemas at home to clear the bowel fully.
ANS: B
For abdominal ultrasonography, the patient is asked to drink a liter of water before the
procedure. This helps change the echo reflection pattern from the bladder, helping to better
distinguish the bladder from the female reproductive organs that lie nearby.

DIF: Cognitive Level: Application REF: p. 416 OBJ: Clinical Practice #1


TOP: Abdominal Ultrasonography KEY: Nursing Process Step: Implementation
MSC: NCLEX: Physiological Integrity: Basic Care and Comfort

6. The nurse preparing a patient for a magnetic resonance imaging (MRI) should determine if the
patient has:
a. respiratory allergies.
b. claustrophobia.
c. fear of the dark.
d. dizziness.
ANS: B
The patient with claustrophobia can be reassured that there are methods to contact persons
outside the cylinder.

DIF: Cognitive Level: Application REF: p. 422 OBJ: Theory #1


TOP: Fluoroscopy KEY: Nursing Process Step: Implementation
MSC: NCLEX: Psychosocial Integrity: Coping and Adaptation

Downloaded by: SUCCEEDGRADES | abbieclin@gmail.com


Distribution
Downloaded by Dallen Mae D.ofKadir
this document is illegal
(kadir.dallenmae.d.bcsi@gmail.com)
lOMoARcPSD|35346190

Stuvia.com - The Marketplace to Buy and Sell your Study Material

7. Prior to the nurse transporting the patient to have a magnetic resonance imaging (MRI), it is
essential that the nurse confirms that the patient:
a. has eaten a meal.
b. has drunk a liter of fluid.
c. is not wearing anything with metal.
d. has a Foley catheter in place.
ANS: C
Nursing care before an MRI involves obtaining consent and ensuring that all metal is removed
from the patient‘s body, because the machine emits a strong magnetic field.

DIF: Cognitive Level: Application REF: p. 422 OBJ: Clinical Practice #4


TOP: MRI KEY: Nursing Process Step: Implementation
MSC: NCLEX: Safe, Effective Care Environment: Safety and Infection Control

8. A patient who is scheduled for a cardiac catheterization asks what the catheterization will
reveal that an electrocardiogram would not. The nurse explains that the catheterization shows:
a. the entire heart to find evidence of cancer.
b. heart rhythm.
c. electrical activity of the heart action.
d. oxygen concentration at various sites.
ANS: D
Cardiac catheterization is a procedure that determines the function of the heart, valves, and
coronary circulation with its attendant oxygen concentration.

DIF: Cognitive Level: Comprehension REF: p. 422 OBJ: Clinical Practice #1


TOP: Catheterization KEY: Nursing Process Step: Implementation
MSC: NCLEX: Physiological Integrity: Basic Care and Comfort

9. A patient who is to have a treadmill stress test at 11:00 AM today should not consume:
a. toast and jam.
b. coffee and cream.
c. oatmeal and sugar.
d. pancakes and syrup.
ANS: B
The patient should avoid caffeine and smoking for 6 hours before the test, but may have a
light meal 2 or more hours beforehand.

DIF: Cognitive Level: Comprehension REF: p. 425 OBJ: Clinical Practice #1


TOP: Treadmill KEY: Nursing Process Step: Implementation
MSC: NCLEX: Physiological Integrity: Basic Care and Comfort

10. A patient who has undergone endoscopy is fully awake and asks the nurse for something to
drink. After confirming that liquids are allowed on the primary care provider order sheet, the
nurse should:
a. assist the patient to the bathroom to void.
b. listen to lung sounds.
c. take a blood pressure and pulse.
d. check for the return of gag and swallow reflexes.

Downloaded by: SUCCEEDGRADES | abbieclin@gmail.com


Distribution
Downloaded by Dallen Mae D.ofKadir
this document is illegal
(kadir.dallenmae.d.bcsi@gmail.com)
lOMoARcPSD|35346190

Stuvia.com - The Marketplace to Buy and Sell your Study Material

ANS: D
The patient should take nothing by mouth until the effects of local anesthesia have worn off
and airway protective reflexes (such as gag and swallow reflexes) have returned.

DIF: Cognitive Level: Analysis REF: p. 427 OBJ: Clinical Practice #1


TOP: Endoscopic Examinations KEY: Nursing Process Step: Implementation
MSC: NCLEX: Safe, Effective Care Environment: Safety and Infection Control

11. The patient in the skilled nursing facility who is to have a colonoscopy tomorrow complains
about his limited diet prior to the examination. The nurse may offer the patient:
a. lime Jell O.
b. strawberry soda.
c. oatmeal thinned with milk.
d. vanilla ice cream.
ANS: A
The patient is placed on a clear liquid diet for 24 hours before colonoscopy and should avoid
liquids that contain red or purple dye. Jell O is part of a clear liquid diet.

DIF: Cognitive Level: Application REF: p. 426 OBJ: Clinical Practice #1


TOP: Endoscopic Examinations KEY: Nursing Process Step: Planning
MSC: NCLEX: Physiological Integrity: Basic Care and Comfort

12. A patient has undergone cystoscopy and has a Foley catheter in place on return to the nursing
unit. Immediately after the procedure, the nurse expects the urine color to be:
a. clear as water.
b. bright red with clots.
c. pink tinged.
d. cherry colored.
ANS: C
It is common for the urine to be pink tinged after cystoscopy, but red bleeding and clots
should be reported to the primary care provider.

DIF: Cognitive Level: Comprehension REF: p. 427 OBJ: Clinical Practice #1


TOP: Cystoscopy KEY: Nursing Process Step: Assessment
MSC: NCLEX: Physiological Integrity: Basic Care and Comfort

13. A patient will undergo endoscopic retrograde cholangiopancreatography (ERCP) to determine


the cause of jaundice. Before the test, the nurse would assess this patient for an allergy to:
a. eggs.
b. pork.
c. aspirin.
d. shellfish.
ANS: D
Allergy to shellfish is assessed, because an iodine-based contrast medium is used during the
test. Shellfish hold and store iodine.

DIF: Cognitive Level: Application REF: p. 427 OBJ: Clinical Practice #1


TOP: Endoscopic Examinations KEY: Nursing Process Step: Assessment
MSC: NCLEX: Physiological Integrity: Pharmacological and Parenteral Therapies

Downloaded by: SUCCEEDGRADES | abbieclin@gmail.com


Distribution
Downloaded by Dallen Mae D.ofKadir
this document is illegal
(kadir.dallenmae.d.bcsi@gmail.com)
lOMoARcPSD|35346190

Stuvia.com - The Marketplace to Buy and Sell your Study Material

14. For the patient who just had a liver biopsy performed, the nurse should position him:
a. prone for 1 hour.
b. on his right side lying for 2 hours.
c. supine for 3 hours.
d. on his left side lying for 4 hours.

ANS: B
The patient should be turned onto the right side for 2 hours after the procedure to minimize
bleeding from the site.

DIF: Cognitive Level: Comprehension REF: p. 425|Table 24-5


OBJ: Clinical Practice #5 TOP: Liver Biopsy
KEY: Nursing Process Step: Implementation
MSC: NCLEX: Physiological Integrity: Basic Care and Comfort

15. Following a colonoscopy with polyp removal, the wife of the patient is distressed that there is
slight bleeding from her husband‘s rectum. The nurse‘s most helpful response would be:
a. ―This small amount of bleeding is expected after the removal of polyps.‖
b. ―I will notify the primary care provider about this hemorrhage.‖
c. ―I will watch your husband very carefully to assess any further hemorrhage.‖
d. ―Don‘t worry. This small amount of blood happens with these procedures.‖
ANS: A
A small amount of bleeding following a colonoscopy with polyp removal is to be expected.
The family should be prepared for the slight bleeding.

DIF: Cognitive Level: Application REF: p. 426 OBJ: Clinical Practice #1


TOP: Colonoscopy KEY: Nursing Process Step: Implementation
MSC: NCLEX: Psychosocial Integrity: Coping and Adaptation

16. To improve the comfort of an older adult patient who is to be in the radiology department for
several hours, the nurse should send a(n):
a. family member with the patient.
b. extra pillow.
c. blanket.
d. newspaper to read.
ANS: C
Older adults may become chilled by the cooler temperatures commonly experienced in
ancillary departments such as radiology.

DIF: Cognitive Level: Application REF: p. 439 OBJ: Theory #4


TOP: Elder Care KEY: Nursing Process Step: Implementation
MSC: NCLEX: Physiological Integrity: Basic Care and Comfort

17. An older adult patient has had a series of enemas in preparation for a gastrointestinal
diagnostic procedure. Which electrolytes should be monitored following the enemas?
a. Calcium and chloride
b. Sodium and potassium
c. Magnesium and phosphorus
d. Selenium and zinc

Downloaded by: SUCCEEDGRADES | abbieclin@gmail.com


Distribution
Downloaded by Dallen Mae D.ofKadir
this document is illegal
(kadir.dallenmae.d.bcsi@gmail.com)
lOMoARcPSD|35346190

Stuvia.com - The Marketplace to Buy and Sell your Study Material

ANS: B
A series of enemas can upset electrolyte balance, especially potassium and sodium.

DIF: Cognitive Level: Application REF: p. 427 OBJ: Theory #4


TOP: Loss of Electrolytes KEY: Nursing Process Step: Implementation
MSC: NCLEX: Physiological Integrity: Physiological Adaptation

18. The nurse explains to the patient that the significance of the hematocrit is that it:
a. indicates the number of circulating white blood cells.
b. indicates the value of the hemoglobin.
c. refers to the separation of blood cells from plasma.
d. will decrease when the patient is in shock.

ANS: C
The hematocrit refers to the relationship of blood cells to plasma in the circulating volume.

DIF: Cognitive Level: Knowledge REF: p. 409 OBJ: Theory #2


TOP: Hematocrit KEY: Nursing Process Step: Implementation
MSC: NCLEX: Physiological Integrity: Physiological Adaptation

19. When obtaining a capillary blood sample for blood glucose, the nurse will select the puncture
site to cause the least amount of discomfort, which is:
a. the end of the index finger.
b. the ball of the third finger.
c. at right angles to the fingerprint lines.
d. the ball of the thumb.
ANS: C
Using the right angle to the fingerprint lines places the puncture on the side of the finger
rather than on more sensitive areas.

DIF: Cognitive Level: Application REF: p. 414|Skill 24-2


OBJ: Theory #3 TOP: Capillary Blood Test
KEY: Nursing Process Step: Planning
MSC: NCLEX: Physiological Integrity: Physiological Adaptation

20. A patient asks why the blood glucose meter directions state to wipe away the first drop of
blood. The most informative response by the nurse would be:
a. ―This eliminates microorganisms from the sample.‖
b. ―The first drop is usually too small.‖
c. ―The first drop is usually contaminated.‖
d. ―The first drop has serous fluid that can dilute the specimen.‖
ANS: D
Some machines state to wipe away the first drop of blood, which often contains a large
portion of serous fluid that can dilute the specimen, causing a false result.

DIF: Cognitive Level: Comprehension REF: p. 414|Skill 24-2


OBJ: Theory #3 TOP: Blood Glucose Testing
KEY: Nursing Process Step: Implementation
MSC: NCLEX: Physiological Integrity: Physiological Adaptation

Downloaded by: SUCCEEDGRADES | abbieclin@gmail.com


Distribution
Downloaded by Dallen Mae D.ofKadir
this document is illegal
(kadir.dallenmae.d.bcsi@gmail.com)
lOMoARcPSD|35346190

Stuvia.com - The Marketplace to Buy and Sell your Study Material

21. A patient in the outpatient clinic has provided a urine sample. To perform a urine dipstick test
accurately, the nurse wets the dipstick and starts timing:
a. immediately.
b. after 5 seconds.
c. after 10 seconds.
d. after 30 seconds.
ANS: A
The stick is inserted into the urine specimen and removed quickly, and timing is started
immediately. It is tapped gently on the side of the container to remove excess urine.

DIF: Cognitive Level: Knowledge REF: p. 411 OBJ: Theory #3


TOP: Urine Dipstick Test KEY: Nursing Process Step: Implementation
MSC: NCLEX: Physiological Integrity: Physiological Adaptation

22. The nurse obtaining a wound culture would:


a. use clean gloves.
b. rotate the swab vigorously in the wound bed.
c. rinse the exudate on the swab with normal saline.
d. place the swab in the culture tube without touching the sides.
ANS: D
The nurse should use sterile gloves, rotate the swab gently in the wound bed, and place it
directly into the culture tube without touching the sides of the tube.

DIF: Cognitive Level: Knowledge REF: p. 422 OBJ: Theory #3


TOP: Wound Culture KEY: Nursing Process Step: Implementation
MSC: NCLEX: Safe, Effective Care Environment: Safety and Infection Control

23. The patient who has just returned to the unit after an angiography test should be assessed
immediately for:
a. swelling of tongue.
b. pulmonary congestion.
c. bleeding at insertion site.
d. hypotension.
ANS: C
The insertion site is monitored for bleeding or formation of a hematoma.

DIF: Cognitive Level: Application REF: pp. 422-423 OBJ: Clinical Practice #1
TOP: Angiography KEY: Nursing Process Step: Implementation
MSC: NCLEX: Safe, Effective Care Environment: Safety and Infection Control

24. The nurse instructing a patient who is to have a Papanicolaou smear (Pap smear) in 2 days
would tell the patient to avoid:
a.sexual intercourse.
b.douching.
c.eating shellfish.
d.taking a bubble bath.
ANS: B

Downloaded by: SUCCEEDGRADES | abbieclin@gmail.com


Distribution
Downloaded by Dallen Mae D.ofKadir
this document is illegal
(kadir.dallenmae.d.bcsi@gmail.com)
lOMoARcPSD|35346190

Stuvia.com - The Marketplace to Buy and Sell your Study Material

The patient who is to have a Pap smear should avoid douching or using any vaginal
medication that might interfere with the collection of the cells of the cervix.

DIF: Cognitive Level: Application REF: p. 432|Skill 24-6


OBJ: Clinical Practice #7 TOP: Pelvic Examination
KEY: Nursing Process Step: Implementation
MSC: NCLEX: Safe, Effective Care Environment

25. The statement made by a patient that would delay a scheduled CT scan would be:
a. ―I have terrible claustrophobia.‖
b. ―I have just been started on metformin.‖
c. ―I am allergic to penicillin.‖
d. ―I have an implanted pacemaker.‖

ANS: B
Metformin should be discontinued before the test with an iodine-based contrast medium
because metformin significantly alters renal function.

DIF: Cognitive Level: Analysis REF: p. 421 OBJ: Theory #1


TOP: Metformin KEY: Nursing Process Step: Implementation
MSC: NCLEX: Physiological Integrity: Pharmacological and Parenteral Therapy

26. The nurse evaluates a prothrombin time (PT/INR) for a patient who is taking heparin. The
nurse‘s initial action should be to:
a. document the findings in the medical record.
b. notify the laboratory that they have made an error.
c. check the primary care provider‘s order.
d. notify the primary care provider of the laboratory finding.
ANS: C
Check the primary care provider‘s orders to determine what test was ordered. If the PT/INR
was ordered, confer with the charge nurse or primary care provider as to the intent. The
PT/INR measures the effectiveness of the drug warfarin (Coumadin), not heparin. A partial
thromboplastin is the test used to evaluate the effectiveness of heparin.

DIF: Cognitive Level: Analysis REF: p. 410 OBJ: Theory #2


TOP: Partial Thromboplastin KEY: Nursing Process Step: Evaluation
MSC: NCLEX: Safe, Effective Care Environment: Management of Care

27. A major concern for an 86-year-old patient who has been NPO for 8 hours prior to a
diagnostic test would be:
a. fatigue.
b. circulatory status.
c. hydration status.
d. nutritional status.
ANS: C
An older adult who is kept on an NPO status for prolonged periods of time are susceptible to
dehydration and electrolyte imbalances.

DIF: Cognitive Level: Comprehension REF: p. 427 OBJ: Theory #4


TOP: Dehydration in the Older Adult KEY: Nursing Process Step: Planning

Downloaded by: SUCCEEDGRADES | abbieclin@gmail.com


Distribution
Downloaded by Dallen Mae D.ofKadir
this document is illegal
(kadir.dallenmae.d.bcsi@gmail.com)
lOMoARcPSD|35346190

Stuvia.com - The Marketplace to Buy and Sell your Study Material

MSC: NCLEX: Physiological Integrity: Physiological Adaptation

28. A patient is having an MRI for a knee injury. During the test, he complains of burning in his
upper thigh and swelling. Which of the following is the most probable cause of this
complaint?
a. Fluid is trapped in the leg due to the dependent positioning.
b. Discomfort most likely related to injury in the knee.
c. Trace metals in ink from a tattoo on the thigh.
d. Discomfort indicates an emergency and requires discontinuation of the MRI.

ANS: C
Patients with tattoos occasionally report burning or swelling of the tattooed area as some inks
contain traces of metal. The patient must inform the MRI technician of any tattoos, and if they
experience any burning feeling during the procedure.

DIF: Cognitive Level: Analysis REF: p. 422 OBJ: Clinical #8


TOP: MRI KEY: Nursing Process Step: Assessment
MSC: NCLEX: Physiological Integrity: Reduction of Risk Potential

MULTIPLE RESPONSE

1. The nurse is aware that the medical record of a patient going for a cardiac catheterization
should have: (Select all that apply.)
a. a signed consent form.
b. a complete history and physical examination.
c. evidence of the initiation of NPO status at least 2 hours prior.
d. evidence of patient education done before the consent form is signed.
e. report of kidney function tests.
f. administration of ordered preoperative medications.
ANS: A, B, D, F
Patient education must be done before a signed consent form is initiated. A complete history
and physical examination is done and NPO status is initiated at least 6 hours prior to the
procedure. The patient is also given ordered preoperative medications.

DIF: Cognitive Level: Application REF: p. 422 OBJ: Clinical Practice #1


TOP: Informed Consent KEY: Nursing Process Step: Planning
MSC: NCLEX: Physiological Integrity: Basic Care and Comfort

2. The primary care provider has ordered the collection of a 24-hour urine specimen. The nurse‘s
instructions to the patient for proper collection of the urine specimen include: (Select all that
apply.)
a. keep the container refrigerated as needed.
b. empty the bladder into the toilet and begin timing the collection.
c. void a small amount of urine after external genitalia are cleansed.
d. keep the container on ice if instructed to do so.
e. save only the first voiding in the morning.

ANS: A, B, D

Downloaded by: SUCCEEDGRADES | abbieclin@gmail.com


Distribution
Downloaded by Dallen Mae D.ofKadir
this document is illegal
(kadir.dallenmae.d.bcsi@gmail.com)
lOMoARcPSD|35346190

Stuvia.com - The Marketplace to Buy and Sell your Study Material

When a 24-hour urine specimen is collected, the patient should be instructed to empty the
bladder into the toilet and begin timing the collection of the specimen; to add all urine to the
collection container for the next 24 hours; to keep the container on ice or refrigerated; and
when the 24 hours are up, to empty the bladder and add the urine to the collection container
and then seal it and send it to the laboratory.

DIF: Cognitive Level: Application REF: p. 411 OBJ: Clinical Practice #1


TOP: Health Education KEY: Nursing Process Step: Implementation
MSC: NCLEX: Physiological Integrity: Basic Care and Comfort

3. The nurse is aware that patients who are not candidates for magnetic resonance imaging
(MRI) include patients with: (Select all that apply.)
a.a hip prosthesis.
b.bleeding tendencies.
c.allergy to iodine.
d.cardiac pacemakers.
e.previous radiological treatment.
ANS: A, D
Patients with prosthetic hips and knees, implanted pacemakers, or metal clips or staples are
not candidates for an MRI because of the magnetic field the test creates.

DIF: Cognitive Level: Comprehension REF: p. 417 OBJ: Clinical Practice #4


TOP: MRI KEY: Nursing Process Step: Planning
MSC: NCLEX: Safe, Effective Care Environment: Safety and Infection Control

4. The nurse informs the patient who is to have an electroencephalogram (EEG) that the
technician will try to stimulate seizure activity by asking the patient to: (Select all that apply.)
a. close his eyes.
b. hyperventilate.
c. breathe in a rapid shallow fashion.
d. hold a flashing light over his face.
e. submerge his hands in cold water.
ANS: B, C, D
During an EEG, abnormal brain activity can be stimulated by the patient being requested to
hyperventilate, breathe rapidly with shallow breaths, and respond to a flashing light over his
face.

DIF: Cognitive Level: Comprehension REF: p. 430 OBJ: Theory #1


TOP: EEG KEY: Nursing Process Step: Implementation
MSC: NCLEX: Physiological Integrity: Basic Care and Comfort

COMPLETION

1. The nurse is aware that a patient who is to have a colonoscopy is requested to stop taking
drugs that contain iron for days prior to the test.

ANS:
3

Downloaded by: SUCCEEDGRADES | abbieclin@gmail.com


Distribution
Downloaded by Dallen Mae D.ofKadir
this document is illegal
(kadir.dallenmae.d.bcsi@gmail.com)
lOMoARcPSD|35346190

Stuvia.com - The Marketplace to Buy and Sell your Study Material

Drugs containing iron are held 3 days before a colonoscopy because iron salts can obscure the
film. Iron can also cause constipation, which makes the cleansing of the bowel more difficult.

DIF: Cognitive Level: Knowledge REF: p. 427 OBJ: Theory #1


TOP: Colonoscopy KEY: Nursing Process Step: Planning
MSC: NCLEX: Physiological Integrity: Pharmacological and Parenteral Therapies

ORDERING

1. The nurse is to collect a sample of blood for a laboratory test. Arrange the sequence of a
phlebotomy. (Separate letters by a comma and space as follows: A, B, C, D, E, F, G, H.)
a. Place vacutainer tube inside holder.
b. Press tube stopper onto needle.
c. Puncture site.
d. Apply tourniquet and cleanse site.
e. Label tube.
f. Fill tube completely.
g. Loosen tourniquet and apply pressure to site.
h. Perform hand hygiene and apply gloves.

ANS:
E, H, A, D, C, B, F, G

The nurse should label the tube with patient identification, perform hand hygiene and apply
gloves, place the vacutainer tube inside the holder, apply the tourniquet and cleanse the site,
puncture the site, press the tube stopper onto the needle, fill the tube completely, and then
loosen the tourniquet and apply pressure to the site.

DIF: Cognitive Level: Analysis REF: p. 412|Skill 24-1


OBJ: Theory #3 TOP: Phlebotomy KEY: Nursing Process Step: Implementation
MSC: NCLEX: Physiological Integrity: Basic Care and Comfort

Downloaded by: SUCCEEDGRADES | abbieclin@gmail.com


Distribution
Downloaded by Dallen Mae D.ofKadir
this document is illegal
(kadir.dallenmae.d.bcsi@gmail.com)
lOMoARcPSD|35346190

Stuvia.com - The Marketplace to Buy and Sell your Study Material

Chapter 25: Fluid, Electrolyte, and Acid-Base Balance


Williams: deWit's Fundamental Concepts and Skills for Nursing, 8th Edition

MULTIPLE CHOICE

1. A 10-month old infant has had watery green stool for 2 days and refuses the bottle. The nurse
is aware that the primary concern for this baby is:
a. metabolic acidosis.
b. metabolic alkalosis.
c. weight loss.
d. diaper rash.
ANS: A
Loss of bowel contents leads to metabolic acidosis. The child will lose weight and will
probably have diaper rash, but the primary concern is the electrolyte imbalance.

DIF: Cognitive Level: Analysis REF: p. 499 OBJ: Clinical Practice #1


TOP: Dehydration KEY: Nursing Process Step: Assessment
MSC: NCLEX: Physiological Integrity: Physiological Adaptation

2. The patient who was admitted after vomiting for 3 days would show an abnormally low blood
pressure because of a fluid shift from:
a. intracellular to the extracellular.
b. interstitial to intravascular.
c. intravascular to the interstitial.
d. interstitial to the intracellular.
ANS: C
If intravascular fluid, a type of extracellular fluid within the blood vessels, shifts from the
plasma in the vascular space out to the interstitial space, a drop in blood volume occurs.

DIF: Cognitive Level: Comprehension REF: p. 438 OBJ: Theory #3


TOP: Distribution of Body Fluids KEY: Nursing Process Step: Assessment
MSC: NCLEX: Physiological Integrity: Physiological Adaptation

3. An isotonic state exists within a patient‘s body fluids when the solute concentration of:
a. interstitial fluid is less than the transcellular.
b. intracellular and extracellular fluid is equal.
c. intracellular fluid is greater than extracellular fluid.
d. extracellular fluid is lesser than intracellular fluid.

ANS: B
When the intracellular and extracellular fluid has the same concentration of particles, the
solution is called isotonic (equal solute concentration).

DIF: Cognitive Level: Comprehension REF: p. 439 OBJ: Theory #3


TOP: Movement of Fluid KEY: Nursing Process Step: N/A
MSC: NCLEX: Physiological Integrity: Physiological Adaptation

4. The nurse is aware that an infant is more at risk for dehydration because the infant:
a. has kidneys that reabsorb water from the intravascular space.

Downloaded by: SUCCEEDGRADES | abbieclin@gmail.com


Distribution
Downloaded by Dallen Mae D.ofKadir
this document is illegal
(kadir.dallenmae.d.bcsi@gmail.com)
lOMoARcPSD|35346190

Stuvia.com - The Marketplace to Buy and Sell your Study Material

b. has a larger body surface compared with body weight.


c. urinates more frequently.
d. has fat that absorbs water.

ANS: B
Infants are more at risk for dehydration because they have a larger body surface compared
with body weight. Their immature kidneys cannot reabsorb water as well as an adult, and fat
does not absorb water.

DIF: Cognitive Level: Comprehension REF: p. 437 OBJ: Clinical Practice #1


TOP: Dehydration KEY: Nursing Process Step: Assessment
MSC: NCLEX: Physiological Integrity: Physiological Adaptation

5. The nurse points out that nonelectrolyte products of metabolism are as important to health as
electrolytes. Nonelectrolytes include:
a. magnesium.
b. amino acids.
c. calcium.
d. phosphates.
ANS: B
The nonelectrolytes that are products of metabolism and serve to promote health in the body
are amino acids, glucose, and fatty acids.

DIF: Cognitive Level: Knowledge REF: p. 438 OBJ: Theory #2


TOP: Non electrolytes KEY: Nursing Process Step: Implementation
MSC: NCLEX: Physiological Integrity: Physiological Adaptation

6. The nurse assesses that the patient has developed abdominal pain, urinary retention, and
confusion. The nurse concludes these signs are the results of an inadequate supply of:
a. calcium (Ca2+).
b. sodium (NA+).
c. phosphates (PO 43).
d. potassium (K+).
ANS: D
The symptoms of a potassium level below 3.5 mEq/L are abdominal pain, urinary retention,
confusion, decreased reflexes, and ECG changes.

DIF: Cognitive Level: Analysis REF: p. 444|Table 25-4


OBJ: Theory #4 TOP: Hypokalemia
KEY: Nursing Process Step: Assessment
MSC: NCLEX: Physiological Integrity: Basic Care and Comfort

7. A nurse gets a positive Chvostek‘s sign on a young woman with bulimia who has been giving
herself frequent enemas containing phosphate. The nurse anticipates a laboratory finding of:
a. sodium 140 mEq/L.
b. potassium 4.5 mEq/L.
c. magnesium 1.6 mEq/L.
d. calcium 6.5 mEq/L.
ANS: D

Downloaded by: SUCCEEDGRADES | abbieclin@gmail.com


Distribution
Downloaded by Dallen Mae D.ofKadir
this document is illegal
(kadir.dallenmae.d.bcsi@gmail.com)
lOMoARcPSD|35346190

Stuvia.com - The Marketplace to Buy and Sell your Study Material

The low level of calcium is responsible for the sign. The positive Chvostek‘s sign is an
indicator of a reduced calcium level.

DIF: Cognitive Level: Analysis REF: p. 444|Table 25-4


OBJ: Clinical Practice #2 TOP: Hypocalcemia
KEY: Nursing Process Step: Assessment
MSC: NCLEX: Physiological Integrity: Physiological Adaptation

8. A patient has been identified as having a dietary deficiency of vitamin D. The nurse
understands that this patient is also at risk for having a deficiency of:
a. calcium.
b. magnesium.
c. sodium.
d. potassium.
ANS: A
Nutritional deficiency of vitamin D can result in hypocalcemia because of the patient‘s
inability to absorb calcium.

DIF: Cognitive Level: Comprehension REF: p. 444|Table 25-4


OBJ: Clinical Practice #1 TOP: Hypocalcemia
KEY: Nursing Process Step: Assessment
MSC: NCLEX: Physiological Integrity: Physiological Adaptation

9. The nurse explains that the dehydrated patient‘s urine is concentrated because:
a. renal tubules reabsorb more water and reduce urine output.
b. kidneys cease to function.
c. blood pressure drops.
d. the colon retains more fluid from the fecal waste.

ANS: A
When dehydration occurs, the renal tubules of the kidney reabsorb more water to be returned
to the circulating volume, making the urine concentrated.

DIF: Cognitive Level: Comprehension REF: p. 441 OBJ: Theory #3


TOP: Dehydration KEY: Nursing Process Step: Assessment
MSC: NCLEX: Physiological Integrity: Physiological Adaptation

10. The nurse is aware that small ions such as glucose, oxygen, and carbon dioxide redistribute
themselves through semipermeable membranes by a process called:
a. diffusion.
b. osmosis.
c. blood pressure.
d. rehydration.

ANS: A
Glucose, oxygen, carbon dioxide, and other small ions diffuse through membranes until they
are evenly distributed.

DIF: Cognitive Level: Knowledge REF: p. 439 OBJ: Clinical Practice #5


TOP: Diffusion KEY: Nursing Process Step: Planning
MSC: NCLEX: Physiological Integrity: Physiological Adaptation

Downloaded by: SUCCEEDGRADES | abbieclin@gmail.com


Distribution
Downloaded by Dallen Mae D.ofKadir
this document is illegal
(kadir.dallenmae.d.bcsi@gmail.com)
lOMoARcPSD|35346190

Stuvia.com - The Marketplace to Buy and Sell your Study Material

11. A patient with healthy kidneys experiences metabolic alkalosis resulting from episodes of
vomiting. The nurse takes into consideration that the kidneys can clear the alkaline substances
and fully stabilize the patient‘s pH in approximately:
a. 3 to 5 minutes.
b. 12 to 24 hours.
c. 3 days.
d. 1 week.

ANS: C
The compensatory ability of the kidneys takes more time to work than does the compensatory
action of the lungs; 3 days are needed for the kidneys to stabilize pH within normal range.

DIF: Cognitive Level: Knowledge REF: p. 447 OBJ: Clinical Practice #5


TOP: Acid-Base Balance KEY: Nursing Process Step: Evaluation
MSC: NCLEX: Physiological Integrity: Physiological Adaptation

12. A patient with a history of severe chronic obstructive pulmonary disease (COPD) is most
likely to have:
a. respiratory alkalosis.
b. respiratory acidosis.
c. metabolic alkalosis.
d. metabolic acidosis.
ANS: B
People with COPD are prone to chronic respiratory acidosis because of the retained CO2.

DIF: Cognitive Level: Comprehension REF: p. 449 OBJ: Clinical Practice #5


TOP: Acid-Base Balance KEY: Nursing Process Step: Assessment
MSC: NCLEX: Physiological Integrity: Physiological Adaptation

13. A patient who is experiencing severe diarrhea is losing excessive bicarbonate ions. This
patient is at risk for developing:
a. respiratory alkalosis.
b. respiratory acidosis.
c. metabolic alkalosis.
d. metabolic acidosis.
ANS: D
Metabolic acidosis can be caused by either an excessive loss of bicarbonate ions or an
excessive retention of hydrogen ions.

DIF: Cognitive Level: Comprehension REF: p. 449 OBJ: Theory #5


TOP: Acid-Base Balance KEY: Nursing Process Step: Assessment
MSC: NCLEX: Physiological Integrity: Physiological Adaptation

14. An anxious adult patient is experiencing a respiratory rate of 40 breaths/min. The most
appropriate intervention that the nurse could do is to instruct the patient to:
a. sit up.
b. lie down.
c. breathe through a re-breather mask.
d. pant with mouth open.

Downloaded by: SUCCEEDGRADES | abbieclin@gmail.com


Distribution
Downloaded by Dallen Mae D.ofKadir
this document is illegal
(kadir.dallenmae.d.bcsi@gmail.com)
lOMoARcPSD|35346190

Stuvia.com - The Marketplace to Buy and Sell your Study Material

ANS: C
Anxiety can lead to hyperventilation, causing respiratory alkalosis; the treatment is to have the
patient breathe through a re-breather mask. In the home setting, the patient can be asked to
breathe into a paper bag.

DIF: Cognitive Level: Application REF: p. 449 OBJ: Theory #5


TOP: Acid-Base Balance KEY: Nursing Process Step: Implementation
MSC: NCLEX: Physiological Integrity: Basic Care and Comfort

15. The nurse is aware that a more dynamic process that moves molecules into cells regardless of
their electrical charge or concentration in the cell is:
a. filtration.
b. osmosis.
c. active transport.
d. hydrostatic pressures.
ANS: C
Active transport can move molecules into cells regardless of their electrical charge or
concentration already in the cell.

DIF: Cognitive Level: Knowledge REF: p. 440 OBJ: Theory #3


TOP: Active Transport KEY: Nursing Process Step: Planning
MSC: NCLEX: Physiological Integrity: Pharmacological and Parenteral Therapies

16. For the accurate measurement to detect fluid retention, the nurse instructs the nursing
assistants to measure the weight with the same scale:
a. each morning before breakfast after the patient has voided.
b. each day at noon before lunch, dressed in light clothing.
c. in between meals, dressed in light clothing after voiding.
d. just before bedtime, while the patient is in a hospital gown or pajamas.
ANS: A
Weight is measured at the same time every morning on the same scale, after the patient has
voided and before eating.

DIF: Cognitive Level: Application REF: p. 450 OBJ: Clinical Practice #1


TOP: Assessment: Fluid and Electrolytes
KEY: Nursing Process Step: Implementation
MSC: NCLEX: Physiological Integrity: Basic Care and Comfort

17. A patient with heart failure has gained 1.1 pounds over the last 24 hours. The nurse is aware
that this weight gain represents a fluid retention of:
a. 0.25 L.
b. 0.5 L.
c. 1.0 L.
d. 2.0 L.
ANS: B
Each 2.2 pounds of weight equals 1 kg, which in turn equals 1.0 L of fluid. Therefore 1.1
pounds equals 0.5 kg and is equal to 0.5 L of fluid.

DIF: Cognitive Level: Analysis REF: p. 450 OBJ: Theory #4

Downloaded by: SUCCEEDGRADES | abbieclin@gmail.com


Distribution
Downloaded by Dallen Mae D.ofKadir
this document is illegal
(kadir.dallenmae.d.bcsi@gmail.com)
lOMoARcPSD|35346190

Stuvia.com - The Marketplace to Buy and Sell your Study Material

TOP: Calculation KEY: Nursing Process Step: Assessment


MSC: NCLEX: Physiological Integrity: Physiological Adaptation

18. The nurse is comparing sitting and standing vital signs for a patient who has been diagnosed
with dehydration. The pulse rate has increased by 10 beats/min at 1 minute. The nurse then
anticipates the blood pressure to show a(n):
a. increase of 5 mm Hg.
b. drop of 40 mm Hg.
c. drop of 20 mm Hg.
d. increase of 10 mm Hg.
ANS: C
A drop in systolic blood pressure by at least 20 mm Hg accompanied by a pulse rate increase
of at least 10 beats/min at 1 minute following position change is suggestive of fluid volume
deficit.

DIF: Cognitive Level: Analysis REF: p. 450 OBJ: Clinical Practice #1


TOP: Assessment: Fluid and Electrolytes
KEY: Nursing Process Step: Assessment
MSC: NCLEX: Physiological Integrity: Physiological Adaptation

19. A patient drank a cup of coffee, a half glass of orange juice, and half a carton of milk with
breakfast. Using common equivalents of food containers as a guide, the nurse notes on the
intake column of the intake and output sheet that the patient consumed:
a. 360 mL.
b. 400 mL.
c. 420 mL.
d. 600 mL.

ANS: C
A coffee cup is generally equivalent to 240 mL, a half glass of juice is 60 mL, and half a
carton of milk is 120 mL.

DIF: Cognitive Level: Analysis REF: p. 453|Table 25-8


OBJ: Clinical Practice #1 TOP: Intake and Output
KEY: Nursing Process Step: Assessment
MSC: NCLEX: Physiological Integrity: Basic Care and Comfort

20. At the beginning of the shift, a patient‘s IV bag has 960 mL remaining. The IV fluid is running
at 75 mL/hr. In 8 hours, there should be how many milliliters remaining in the IV bag?
a. 150
b. 360
c. 450
d. 600
ANS: B
75 mL/hr 8 hours = 600; 960 600 = 360.

DIF: Cognitive Level: Analysis REF: p. 453 OBJ: Clinical Practice #1


TOP: Intake and Output KEY: Nursing Process Step: Assessment
MSC: NCLEX: Physiological Integrity: Pharmacological and Parenteral Therapies

Downloaded by: SUCCEEDGRADES | abbieclin@gmail.com


Distribution
Downloaded by Dallen Mae D.ofKadir
this document is illegal
(kadir.dallenmae.d.bcsi@gmail.com)
lOMoARcPSD|35346190

Stuvia.com - The Marketplace to Buy and Sell your Study Material

21. The primary care provider orders fluid restriction for a patient with severe fluid volume
excess. When a patient is placed on a fluid restriction, the allowance of fluids should be:
a. greatest during the day shift.
b. greatest during the evening shift.
c. greatest during the night shift.
d. spaced in equal increments for all shifts.
ANS: A
The greatest amount of fluid is given during the day shift, followed by the evening shift. The
least amount of fluid is given at night, when the patient should be sleeping.

DIF: Cognitive Level: Comprehension REF: p. 453 OBJ: Clinical Practice #1


TOP: Intake and Output KEY: Nursing Process Step: Planning
MSC: NCLEX: Physiological Integrity: Basic Care and Comfort

22. The nurse is aware that the patient who suffered a brain injury with cerebral edema will most
likely receive a fluid that is:
a. isotonic.
b. hypertonic.
c. hypotonic.
d. enhanced with vitamin B.
ANS: B
Hypertonic fluids draw fluid from the intracellular space and reduce edema.

DIF: Cognitive Level: Comprehension REF: p. 439 OBJ: Clinical Practice #4


TOP: Hypertonic Solutions KEY: Nursing Process Step: N/A
MSC: NCLEX: Physiological Integrity: Basic Care and Comfort

23. The patient who is prescribed a diuretic for fluid volume excess is discharged home. The
patient verbalizes understanding of his disease process when he says:
a. ―I can put catsup on my scrambled eggs.‖
b. ―I can snack on salted popcorn.‖
c. ―I will snack on raisins.‖
d. ―I will avoid apricots.‖
ANS: C
The patient will lose electrolytes, especially potassium, because he is on a diuretic; snacks
such as raisins and apricots are rich in potassium.

DIF: Cognitive Level: Analysis REF: p. 446 OBJ: Clinical Practice #3


TOP: Teaching Plan for Sodium Restriction
KEY: Nursing Process Step: Evaluation
MSC: NCLEX: Health Promotion and Maintenance: Prevention and Early Detection of Disease

24. The nurse is caring for a patient for whom a dose of IV potassium has been ordered. Prior to
hanging the potassium, the nurse should:
a. check urine output to be above 60 mL/hr.
b. check the dose with another licensed person.
c. confirm the IV fluid running is compatible with potassium.
d. start potassium with another venipuncture.

Downloaded by: SUCCEEDGRADES | abbieclin@gmail.com


Distribution
Downloaded by Dallen Mae D.ofKadir
this document is illegal
(kadir.dallenmae.d.bcsi@gmail.com)
lOMoARcPSD|35346190

Stuvia.com - The Marketplace to Buy and Sell your Study Material

ANS: C
The nurse must confirm that the IV fluid that is running is compatible with potassium. A urine
output of at least 30 mL/hr is essential prior to giving IV potassium.

DIF: Cognitive Level: Application REF: p. 454 OBJ: Clinical Practice #4


TOP: Intake and Output KEY: Nursing Process Step: Implementation
MSC: NCLEX: Physiological Integrity: Basic Care and Comfort

25. The nurse caring for a frail 92-year-old dehydrated patient should add to the plan of care:
a. potential for over-hydration related to excessive thirst.
b. potential for diarrhea related to dehydration.
c. potential for pulmonary congestion related to excessive fluid intake.
d. potential for fall related to confusion.
ANS: D
The dehydrated patient may become confused because of fluid and electrolyte losses.

DIF: Cognitive Level: Analysis REF: p. 441 OBJ: Clinical Practice #4


TOP: Elder Care KEY: Nursing Process Step: Implementation
MSC: NCLEX: Physiological Integrity: Reduction of Risk Potential

26. The nurse is determining if I&O are within normal limits. Which of the following is an
indication of a fluid imbalance?
a. Twenty-four hour I&O totals show 2 L negative output. Compare daily weight to
see if there is a weight gain of 2 kg.
b. Shift I&O totals show a 2 L positive output. Compare the daily weights to see if
there is a weight gain of 2 kg.
c. Shift I&O totals show a 4 L negative output. Compare daily weights to see if there
is a zero weight loss.
d. Twenty-four hour I&O totals are equal. Compare daily weight to see if there is a
negative weight loss.
ANS: A
Determine whether the I&O are within normal limits. Compare the amounts to see if there is
any indication of a fluid imbalance. Compare the total with the totals from the previous 2 days
to see if either intake or output is increasing. Compare I&O 24-hour totals with daily weights
to see that they ―match.‖ (If the output is greater or less than the intake, the patient may have
a fluid imbalance. Comparison of the totals shows whether there is an increase in intake or
output. 24-hour I&O totals should mirror the trends of daily weight; for example, if a patient
is 2 L “fluid positive” you expect to see weight gain of 2 kg.)

DIF: Cognitive Level: Analysis REF: p. 441 OBJ: Clinical #6


TOP: I&O KEY: Nursing Process Step: Evaluation
MSC: NCLEX: Physiological Integrity: Reduction of Risk Potential

MULTIPLE RESPONSE

1. The nurse clarifies that the electrolytes include: (Select all that apply.)
a. sodium.
b. fatty acids.
c. potassium.

Downloaded by: SUCCEEDGRADES | abbieclin@gmail.com


Distribution
Downloaded by Dallen Mae D.ofKadir
this document is illegal
(kadir.dallenmae.d.bcsi@gmail.com)
lOMoARcPSD|35346190

Stuvia.com - The Marketplace to Buy and Sell your Study Material

d. magnesium.
e. amino acids.
f. glucose.

ANS: A, C, D
The intermediate products of metabolism—amino acids (proteins), glucose, and fatty acids—
are nonelectrolytes. Sodium, potassium, and magnesium are all electrolytes.

DIF: Cognitive Level: Comprehension REF: p. 438|Table 25-2


OBJ: Theory #2 TOP: Electrolytes KEY: Nursing Process Step: Implementation
MSC: NCLEX: Physiological Integrity: Physiological Adaptation

2. Based on the information provided, which of these measurements should be recorded on the
output sheet? (Select all that apply.)
a. 250 mL nasogastric secretions
b. 200 mL diarrhea stool
c. 900 mL IV therapy
d. 650 mL urine from Foley catheter
e. 50 mL chest tube drainage
f. 240 mL milk
ANS: A, B, D, E
The nurse should calculate fluid intake, both orally and intravenously, and mark and record
the amount of gastric suction contents, chest tube drainage, Foley catheter drainage, and feces.

DIF: Cognitive Level: Application REF: p. 455|Skill 25-1


OBJ: Clinical Practice #1 TOP: Intake and Output
KEY: Nursing Process Step: Assessment
MSC: NCLEX: Physiological Integrity: Basic Care and Comfort

3. The nurse assessing a newly admitted patient with marked edema from severe heart failure
would anticipate that the patient would exhibit: (Select all that apply.)
a.a thready pulse.
b.concentrated urine.
c.hypertension.
d.weight gain.
e.crackles heard on auscultation.
ANS: C, D, E
Persons with excess fluid volume as with a patient in heart failure would exhibit a bounding
hypertension and weight gain. These persons would also have a bounding pulse and copious
diluted urine as the kidneys try to excrete the excess fluid. Because of fluid accumulation in
the pleural space, crackles can be heard on auscultation when assessing a person with heart
failure.

DIF: Cognitive Level: Comprehension REF: p. 441 OBJ: Theory #4


TOP: Overhydration KEY: Nursing Process Step: Assessment
MSC: NCLEX: Physiological Integrity: Physiological Adaptation

4. The nurse explains that water as a constituent of the body has the functions of: (Select all that
apply.)
a. transportation of nutrients.

Downloaded by: SUCCEEDGRADES | abbieclin@gmail.com


Distribution
Downloaded by Dallen Mae D.ofKadir
this document is illegal
(kadir.dallenmae.d.bcsi@gmail.com)
lOMoARcPSD|35346190

Stuvia.com - The Marketplace to Buy and Sell your Study Material

b. blood pressure regulation.


c. heat regulation.
d. removing waste from the cells.
e. assists with digestion of protein.
ANS: A, C, D
Water has four functions: (1) vehicle of transportation to and from the cells, (2) heat
regulation, (3) assists with hydrogen balance, and (4) acts as medium for enzymatic action of
digestion. Although the amount of water in the circulating volume has an effect on blood
pressure, water does not control blood pressure. Water does not digest protein.

DIF: Cognitive Level: Comprehension REF: p. 436 OBJ: Theory #1


TOP: Function of Water KEY: Nursing Process Step: Implementation
MSC: NCLEX: Physiological Integrity: Physiological Adaptation

COMPLETION

1. A patient with a serum potassium value of less than 3.5 mEq/L is .

ANS:
hypokalemic

The normal range for potassium is 3.5 to 5.0 mEq/L.

DIF: Cognitive Level: Analysis REF: p. 438|Table 25-2


OBJ: Theory #2 TOP: Electrolytes KEY: Nursing Process Step: N/A
MSC: NCLEX: N/A

2. The nurse clarifies that when electrolytes are in solution, they break up and become
.

ANS:
ions

Ions are charged particles of electrolytes in solutions. They become either a cation with a
positive charge or an anion with a negative charge.

DIF: Cognitive Level: Knowledge REF: p. 438 OBJ: Theory #3


TOP: Ions KEY: Nursing Process Step: Implementation
MSC: NCLEX: Physiological Integrity: Physiological Adaptation

Downloaded by: SUCCEEDGRADES | abbieclin@gmail.com


Distribution
Downloaded by Dallen Mae D.ofKadir
this document is illegal
(kadir.dallenmae.d.bcsi@gmail.com)
lOMoARcPSD|35346190

Stuvia.com - The Marketplace to Buy and Sell your Study Material

Chapter 26: Concepts of Basic Nutrition and Cultural Considerations


Williams: deWit's Fundamental Concepts and Skills for Nursing, 8th Edition

MULTIPLE CHOICE

1. The nurse reminds the patient that the salivary glands excrete saliva, which initiates the
digestion of:
a. proteins.
b. starches.
c. fats.
d. fiber.
ANS: B
Saliva initiates the digestion of starches.

DIF: Cognitive Level: Knowledge REF: p. 459 OBJ: Theory #1


TOP: Functions of the Gastrointestinal System
KEY: Nursing Process Step: Implementation
MSC: NCLEX: Physiological Integrity: Physiological Adaptation

2. The nurse is aware that vitamin B12 is absorbed in the:


a. stomach.
b. large intestine.
c. liver.
d. gallbladder.
ANS: A
Vitamin B12, an aid in hemoglobin syntheses, is absorbed in the stomach through the action of
the intrinsic factor, which is secreted from the stomach wall.

DIF: Cognitive Level: Knowledge REF: p. 459 OBJ: Theory #1


TOP: Absorption of Vitamins KEY: Nursing Process Step: Planning
MSC: NCLEX: Physiological Integrity: Physiological Adaptation

3. The nurse emphasizes the dietary recommendations made by the American Heart Association
is to limit cholesterol intake to:
a. 300 mg/day.
b. 400 mg/day.
c. 425 mg/day.
d. 500 mg/day.
ANS: A
The American Heart Association recommends an intake of cholesterol to 300 mg/day or less.

DIF: Cognitive Level: Knowledge REF: p. 461|Box 26-1


OBJ: Theory #4 TOP: Dietary Recommendations
KEY: Nursing Process Step: Planning
MSC: NCLEX: Health Promotion and Maintenance: Prevention and Early Detection of Disease

4. A patient refuses to eat all types of meat, which has led to a protein deficiency. The nurse
recognizes that the only plant source that contains all nine essential amino acids is:

Downloaded by: SUCCEEDGRADES | abbieclin@gmail.com


Distribution
Downloaded by Dallen Mae D.ofKadir
this document is illegal
(kadir.dallenmae.d.bcsi@gmail.com)
lOMoARcPSD|35346190

Stuvia.com - The Marketplace to Buy and Sell your Study Material

a. bean sprouts.
b. lima beans.
c. kidney beans.
d. soybeans.
ANS: D
Soybeans are the only plant source that provides all nine essential amino acids.

DIF: Cognitive Level: Knowledge REF: p. 462 OBJ: Clinical Practice #1


TOP: Sources of Protein KEY: Nursing Process Step: Planning
MSC: NCLEX: Health Promotion and Maintenance: Prevention of Disease and Early Detection of
Disease

5. A patient weighing 132 pounds has been recommended to increase daily protein intake. The
nurse assists the patient to make dietary selections of protein after calculating that the daily
protein requirement for this patient is:
a. 24 g.
b. 36 g.
c. 48 g.
d. 60 g.

ANS: C
The protein requirement for the day is equal to the number of kilograms of weight (convert
pounds to kg) multiplied by 0.8 (ie, 132/2.2 = 48).

DIF: Cognitive Level: Analysis REF: p. 462 OBJ: Clinical Practice #3


TOP: Dietary Protein Recommendation KEY: Nursing Process Step: Planning
MSC: NCLEX: Health Promotion and Maintenance: Prevention and Early Detection of Disease

6. The nurse consults with a patient who is a vegan and stresses that this diet puts the patient at
risk for:
a. diabetes.
b. iron deficiency.
c. osteoporosis
d. scurvy.
ANS: B
In the vegan diet, all animal food sources are excluded, placing a patient who eats this diet
most at risk for deficient intake of protein leading to an iron deficiency.

DIF: Cognitive Level: Comprehension REF: p. 463 OBJ: Clinical Practice #1


TOP: Vegan Diets KEY: Nursing Process Step: Assessment
MSC: NCLEX: Health Promotion and Maintenance: Prevention and Early Detection of Disease

7. The nurse in a long-term care facility understands that the 86-year-old resident‘s frequent
complaints about ―heartburn‖ are most likely due to the age-related decreased:
a.peristalsis.
b.gag reflex.
c.appetite.
d.sphincter tone.
ANS: D

Downloaded by: SUCCEEDGRADES | abbieclin@gmail.com


Distribution
Downloaded by Dallen Mae D.ofKadir
this document is illegal
(kadir.dallenmae.d.bcsi@gmail.com)
lOMoARcPSD|35346190

Stuvia.com - The Marketplace to Buy and Sell your Study Material

The age-related loss of muscle tone in the sphincters increases the incidence of heartburn and
esophageal reflux.

DIF: Cognitive Level: Comprehension REF: p. 460 OBJ: Theory #63


TOP: Planning KEY: Nursing Process Step: Evaluation
MSC: NCLEX: Health Promotion and Maintenance: Prevention and Early Detection of Disease

8. The patient states that he uses a large amount of table sugar (sucrose) in his foods because it
gives him quick energy. The nurse explains that sucrose will:
a. quickly raise the blood sugar, and the level drops slowly.
b. not raise the blood sugar as quickly as a protein source will.
c. cause a hunger and energy lag because of the rapid fall of the blood sugar.
d. also act as a good support to the digestion of fiber.
ANS: C
Table sugar is high in sucrose, which is quickly absorbed into the bloodstream and can cause
rapid rises and falls in blood glucose, which leads to hunger and an energy lag.

DIF: Cognitive Level: Comprehension REF: p. 464 OBJ: Theory #2


TOP: Simple Carbohydrates KEY: Nursing Process Step: Implementation
MSC: NCLEX: Health Promotion and Maintenance: Prevention and Early Detection of Disease

9. The nurse evaluates the patient‘s understanding of the fiber content of grains and cereals when
the patient selects:
a. white bread toast with an orange.
b. wheat bread toast with a peeled apple.
c. shredded wheat and a banana.
d. a biscuit and a grapefruit.
ANS: C
Shredded wheat and a banana contain a total of 6 g of fiber per serving, whereas wheat toast
with an apple contain a total of 4 g, white bread with a banana contain a total of 3 g, and a
biscuit with a grapefruit contain a total of 4 g.

DIF: Cognitive Level: Comprehension REF: p. 466|Table 26-1


OBJ: Theory #3 TOP: Fiber KEY: Nursing Process Step: Evaluation
MSC: NCLEX: Health Promotion and Maintenance: Prevention and Early Detection of Disease

10. A patient has been placed on a reduced cholesterol diet to help control heart disease. The
serving that would be most appropriate for the patient to select for supper is:
a. 3 ounces of tuna canned in water.
b. 1 frankfurter.
c. one 3-ounce pork chop.
d. 1/2 chicken breast with skin.
ANS: A
The tuna canned in water contains only 2 g of fat, whereas the highest fat grams are found in
the pork chop (19 g) followed by the chicken breast with skin (18 g).

DIF: Cognitive Level: Application REF: p. 261|Box 26-1


OBJ: Theory #9 TOP: Cholesterol KEY: Nursing Process Step: Implementation
MSC: NCLEX: Health Promotion and Maintenance: Prevention and Early Detection of Disease

Downloaded by: SUCCEEDGRADES | abbieclin@gmail.com


Distribution
Downloaded by Dallen Mae D.ofKadir
this document is illegal
(kadir.dallenmae.d.bcsi@gmail.com)
lOMoARcPSD|35346190

Stuvia.com - The Marketplace to Buy and Sell your Study Material

11. The nurse points out to the newly diagnosed Type 2 diabetic patient that complex
carbohydrates:
a. do not affect the blood sugar level.
b. keep the blood sugar at an unsatisfactory high level.
c. lack adequate nutritional potential.
d. maintain a more consistent blood sugar level.
ANS: D
Complex carbohydrates (pasta, cereal, rice) provide a more consistent blood sugar level that
simple sugars.

DIF: Cognitive Level: Application REF: p. 464 OBJ: Clinical Practice #4


TOP: Vitamins KEY: Nursing Process Step: Implementation
MSC: NCLEX: Health Promotion and Maintenance: Prevention and Early Detection of Disease

12. When a patient asks how is a good way to increase fiber in the diet, the nurse‘s best response
would be:
a. eating unpeeled apples.
b. increase intake of dark leafy greens.
c. eating broiled salmon.
d. taking daily concentrated fiber supplements.
ANS: A
Eating the skins of fruits is a good source of fiber. Fiber concentrates do not contain needed
vitamins and minerals.

DIF: Cognitive Level: Application REF: p. 465 OBJ: Clinical Practice #4


TOP: Fiber KEY: Nursing Process Step: Planning
MSC: NCLEX: Health Promotion and Maintenance: Prevention and Early Detection of Disease

13. The LPN/LVN filling out the Jewish patient‘s dietary menu for lunch would avoid ordering:
a. meat and fish.
b. milk and vegetables.
c. meat and milk.
d. vegetables and fruit.

ANS: C
Common food practices in Judaism include not eating meat and milk at the same meal.

DIF: Cognitive Level: Application REF: p. 472 OBJ: Theory #7


TOP: Nutrition and Culture/Religion KEY: Nursing Process Step: Implementation
MSC: NCLEX: Psychosocial Integrity: Coping and Adaptation

14. A nurse takes into consideration that the usual diet of Asian Americans has a potential for
altering health because the diet is high in:
a. protein.
b. starch.
c. sodium.
d. vitamin C.
ANS: C

Downloaded by: SUCCEEDGRADES | abbieclin@gmail.com


Distribution
Downloaded by Dallen Mae D.ofKadir
this document is illegal
(kadir.dallenmae.d.bcsi@gmail.com)
lOMoARcPSD|35346190

Stuvia.com - The Marketplace to Buy and Sell your Study Material

The Asian diet is high in sodium and fat.

DIF: Cognitive Level: Comprehension REF: p. 473 OBJ: Theory #7


TOP: Nutrition and Culture/Religion KEY: Nursing Process Step: Planning
MSC: NCLEX: Health Promotion and Maintenance: Prevention and Early Detection of Disease

15. The mother of a 4-month old infant asks what type of cereal is most appropriate to feed the
infant as a first solid food. The best response from the nurse is to suggest
a. wheat.
b. barley.
c. corn.
d. rice.
ANS: D
A cereal such as rice is the best initial choice, because it is easily tolerated, provides additional
calories and iron, and is least likely to be allergenic.

DIF: Cognitive Level: Comprehension REF: p. 474 OBJ: Theory #8


TOP: Nutritional Needs Across the Life Span
KEY: Nursing Process Step: Implementation
MSC: NCLEX: Health Promotion and Maintenance: Growth and Development

16. A mother is concerned that her toddler is not eating enough at mealtimes. The most
informative suggestion by the nurse would be to:
a. provide large portions to stimulate appetite.
b. provide single item foods or finger foods that do not touch each other on the plate.
c. increase the amount of milk at each meal.
d. use plain white dishes to keep attention focused on food.
ANS: B
Toddlers prefer single item foods in small quantities that do not touch each other on a colorful
plate. Milk intake should decrease during the toddler years as solid food takes the place of
milk.

DIF: Cognitive Level: Application REF: p. 475 OBJ: Theory #8


TOP: Nutritional Needs Across the Life Span
KEY: Nursing Process Step: Implementation
MSC: NCLEX: Health Promotion and Maintenance: Growth and Development

17. On assessment, the nurse finds that the female patient has a BMI of 26, a waist of 37 inches,
pale conjunctiva, and a large muscle mass. The indicator of this patient being overweight is:
a. BMI level.
b. waist measurement.
c. conjunctiva.
d. large muscle mass.
ANS: B
A waist measurement in women of over 35 is an indicator of greater risk for overweight and
disease.

DIF: Cognitive Level: Analysis REF: p. 472 OBJ: Clinical Practice #2


TOP: Physical Signs of Obesity KEY: Nursing Process Step: Assessment

Downloaded by: SUCCEEDGRADES | abbieclin@gmail.com


Distribution
Downloaded by Dallen Mae D.ofKadir
this document is illegal
(kadir.dallenmae.d.bcsi@gmail.com)
lOMoARcPSD|35346190

Stuvia.com - The Marketplace to Buy and Sell your Study Material

MSC: NCLEX: Health Promotion and Maintenance: Prevention and Early Detection of Disease

18. The nursing action that is most beneficial toward creating an atmosphere conducive to eating
for a hospitalized patient immobilized in bed is:
a. lower the head of bed as tolerated.
b. remove the urinal from the over the bed table.
c. invite the patient to wash hands and face before eating.
d. use a deodorizer to remove any unpleasant odor in the room.
ANS: B
Remove distracting articles such as the urinal and emesis basin.

DIF: Cognitive Level: Application REF: p. 479 OBJ: Theory #6


TOP: Promoting Appetite KEY: Nursing Process Step: Implementation
MSC: NCLEX: Physiological Integrity: Basic Care and Comfort

19. The nurse is delivering a meal tray to a patient in a skilled nursing facility who is a Muslim.
The nurse should confirm the meal is free of:
a.raw fruits.
b.eggplant.
c.pork.
d.lamb.
ANS: C
People of the Muslim faith are prohibited from eating pork.

DIF: Cognitive Level: Application REF: p. 474|Table 26-7


OBJ: Theory #7 TOP: Culture and Nutrition
KEY: Nursing Process Step: Implementation
MSC: NCLEX: Psychosocial Integrity: Coping and Adaptation

20. The nurse would be sure the diet of a patient in an extended care facility who has a large
pressure ulcer on his sacrum would include foods rich in:
a. vitamin A.
b. vitamin B1 (thiamine).
c. vitamin C.
d. vitamin E.
ANS: C
Vitamin C helps protect the body against infections and promotes wound healing.

DIF: Cognitive Level: Application REF: p. 467 OBJ: Theory #2


TOP: Vitamins KEY: Nursing Process Step: Implementation
MSC: NCLEX: Physiological Integrity: Physiological Adaptation

21. The nurse is visiting an older adult patient who lives alone. The suggestion made by the nurse
that would be most helpful in improving the patient‘s nutrition would be:
a. keep the environment noise free to concentrate on eating.
b. decrease intake of fluids to improve appetite.
c. use salt as needed to spice up the flavor of foods.
d. cook favorite foods in bulk and freeze in individual serving containers.
ANS: D

Downloaded by: SUCCEEDGRADES | abbieclin@gmail.com


Distribution
Downloaded by Dallen Mae D.ofKadir
this document is illegal
(kadir.dallenmae.d.bcsi@gmail.com)
lOMoARcPSD|35346190

Stuvia.com - The Marketplace to Buy and Sell your Study Material

Cooking and freezing favorite foods for easy preparation later is helpful in improving the
overall nutrition of an older adult patient.

DIF: Cognitive Level: Application REF: p. 476 OBJ: Clinical Practice #1


TOP: Nutritional Needs Across the Life Span
KEY: Nursing Process Step: Implementation
MSC: NCLEX: Physiological Integrity: Basic Care and Comfort

22. When assisting with the nutritional assessment of a newly admitted, confused, emaciated
cancer patient, the nurse‘s most beneficial intervention to support the nutritional status of this
patient would be to:
a. obtain the information from the family.
b. ask simple questions of the patient.
c. ask for a dietitian consult.
d. request an order for a full liquid diet.
ANS: C
A dietitian should be consulted if the patient has high nutritional needs.

DIF: Cognitive Level: Application REF: p. 479 OBJ: Theory #9


TOP: Nutritional Assessment KEY: Nursing Process Step: Implementation
MSC: NCLEX: Physiological Integrity: Basic Care and Comfort

23. The nurse caring for a patient with human immunodeficiency syndrome HIV would encourage
the intake of greater amounts of:
a. vitamin D.
b. protein.
c. vitamin C.
d. raw fruits and vegetables.
ANS: B
Patients with HIV lose muscle mass and need to increase their nutritional intake in the form of
extra calories and protein.

DIF: Cognitive Level: Comprehension REF: p. 472 OBJ: Clinical Practice #4


TOP: Nutrition for HIV patient KEY: Nursing Process Step: Implementation
MSC: NCLEX: Physiological Integrity: Basic Care and Comfort

24. The home health nurse is visiting an older adult patient with a history of malnutrition and
memory loss. The nurse addresses the nutritional needs of the patient by:
a.writing down all the information for the patient.
b.giving the patient MyPlate for reference.
c.involving the family.
d.making out a grocery list for the patient.
ANS: C
The patient may not be able to remember what needs to be done, and involving the family will
help in meeting the patient‘s needs.

DIF: Cognitive Level: Application REF: p. 476 OBJ: Clinical Practice #1


TOP: Malnutrition KEY: Nursing Process Step: Implementation
MSC: NCLEX: Physiological Integrity: Reduction of Risk

Downloaded by: SUCCEEDGRADES | abbieclin@gmail.com


Distribution
Downloaded by Dallen Mae D.ofKadir
this document is illegal
(kadir.dallenmae.d.bcsi@gmail.com)
lOMoARcPSD|35346190

Stuvia.com - The Marketplace to Buy and Sell your Study Material

25. A patient of Mexican American descent has a blood pressure of 160/90 mm Hg and is
moderately obese. The nurse can help the patient modify his diet by suggesting:
a.decreasing spices when cooking.
b.avoiding fried foods altogether.
c.limiting corn tortillas to two per day.
d.substituting the use of lard with canola oil.
ANS: D
Canola oil is an unsaturated fat, whereas lard is saturated, and saturated fats should be limited
to 10% of total fat intake.

DIF: Cognitive Level: Application REF: p. 474 OBJ: Clinical Practice #4


TOP: Patient Education KEY: Nursing Process Step: Implementation
MSC: NCLEX: Health Promotion and Maintenance: Prevention and Early Detection of Disease

26. Young women are being educated about the trends in nutrition regarding nutrients that may
decrease the incidence of cancer. Which of the following foods are indicative of these trends?
a. Raspberries, strawberries, blueberries
b. Beef, poultry
c. Eggs, milk, and butter
d. Corn, peas, and green beans
ANS: A
Although studies are inconclusive, evidence suggests that certain food components can
decrease cancer risk including:
• Phytochemicals (eg, carotenoids): found in brightly colored or strongly flavored vegetables
and fruit
• Antioxidants (eg, vitamin C, vitamin E, beta carotene): found in raspberries, strawberries,
blueberries, and many other foods
• Omega-3 fatty acids: found in fish, vegetable oil, flax seed, leafy vegetables

DIF: Cognitive Level: Comprehension REF: p. 463 OBJ: Theory #2


TOP: Health Promotion
KEY: Nursing Process Step: Health Promotion and Maintenance: Prevention and Early Detection of
Disease MSC: NCLEX: N/A

MULTIPLE RESPONSE

1. The nurse stresses to a patient that proteins, one of the biochemical substances used by the
body, can be found in: (Select all that apply.)
a. eggs.
b. glucose.
c. yogurt.
d. fish.
e. soybeans.
f. nuts.
ANS: A, C, D, E, F
Sources of protein are meats, poultry, fish, eggs, dairy products, cereals, grains, legumes, most
vegetables, and soybeans.

Downloaded by: SUCCEEDGRADES | abbieclin@gmail.com


Distribution
Downloaded by Dallen Mae D.ofKadir
this document is illegal
(kadir.dallenmae.d.bcsi@gmail.com)
lOMoARcPSD|35346190

Stuvia.com - The Marketplace to Buy and Sell your Study Material

DIF: Cognitive Level: Comprehension REF: p. 462 OBJ: Theory #3


TOP: Nutrition KEY: Nursing Process Step: Implementation
MSC: NCLEX: Physiological Integrity: Physiological Adaptation

2. When a nurse performs a nutritional assessment on a patient with HIV, what is important to
include? (Select all that apply.)
a. Source of illness
b. Family and social history
c. Patient‘s education
d. Income level
e. Physical assessment

ANS: B, C, D, E
All are factors that can influence the patient‘s nutritional status with the exception of the
source of illness.

DIF: Cognitive Level: Application REF: p. 477 OBJ: Clinical Practice #2


TOP: Nutritional Assessment KEY: Nursing Process Step: Assessment
MSC: NCLEX: Physiological Integrity: Basic Care and Comfort

3. The nurse instructs the patient on a vegetarian diet that protein intake can be supported by
including complementary proteins in the diet with foods such as: (Select all that apply.)
a. bean soup with cornbread.
b. tofu stir fried with vegetables.
c. peanut butter on whole wheat bread.
d. apples and cheese.
e. lean fish with green beans.
ANS: A, B, C
Complementary proteins are plant source foods combined to achieve a complete protein
intake. Beans, tofu, and peanut butter are all examples of complementary proteins.

DIF: Cognitive Level: Application REF: p. 463 OBJ: Theory #3


TOP: Complementary Proteins KEY: Nursing Process Step: Implementation
MSC: NCLEX: Health Promotion and Maintenance: Prevention and Early Detection of Disease

4. The nurse reminds the patient that adequate fiber in the diet has many health benefits, which
include: (Select all that apply.)
a. improves elimination.
b. decreases blood glucose levels in diabetics.
c. delays the absorption of carbohydrates from the intestine.
d. enhances absorption of vitamin D.
e. decreases the absorption of fat.
ANS: A, B, C, E
Dietary fiber adds bulk to the stool for better elimination. Fiber also delays the absorption of
fats and carbohydrate, which lowers the blood sugar levels in diabetics.

DIF: Cognitive Level: Analysis REF: p. 465 OBJ: Theory #3


TOP: Function of Fiber KEY: Nursing Process Step: Implementation
MSC: NCLEX: Physiological Integrity: Physiological Adaptation

Downloaded by: SUCCEEDGRADES | abbieclin@gmail.com


Distribution
Downloaded by Dallen Mae D.ofKadir
this document is illegal
(kadir.dallenmae.d.bcsi@gmail.com)
lOMoARcPSD|35346190

Stuvia.com - The Marketplace to Buy and Sell your Study Material

5. The nurse points out that the advantages of breastfeeding include that breast milk: (Select all
that apply.)
a. provides immunity to most childhood diseases for several months.
b. causes no allergies.
c. should be supplemented with vitamin D.
d. encourages growth of normal bacterial flora in the bowel.
e. contains both fat- and water-soluble vitamins.
ANS: A, B, D, E
Breast milk provides total nutrition, provides immunity to most childhood diseases for the few
several months of life, causes no allergies, encourages normal bacterial flora in the bowel, and
has both fat- and water-soluble vitamins.

DIF: Cognitive Level: Application REF: p. 473 OBJ: Theory #8


TOP: Breastfeeding KEY: Nursing Process Step: Implementation
MSC: NCLEX: Health Promotion and Maintenance: Growth and Development

COMPLETION

1. The nurse takes into consideration that the patient with pernicious anemia who lacks the
intrinsic factor cannot absorb .

ANS:
vitamin B12

The intrinsic factor excreted by the wall of the stomach allows the absorption of vitamin B12.

DIF: Cognitive Level: Knowledge REF: p. 469|Table 26-4


OBJ: Theory #5 TOP: Pernicious Anemia
KEY: Nursing Process Step: Planning
MSC: NCLEX: Physiological Integrity: Physiological Adaptation

2. The nurse uses a chart to show an obese patient who is trying to lose weight by counting
calories that each gram of carbohydrate supplies calories.

ANS:
4

One gram of carbohydrate supplies 4 calories.

DIF: Cognitive Level: Knowledge REF: p. 464 OBJ: Clinical Practice #4


TOP: Calories KEY: Nursing Process Step: Implementation
MSC: NCLEX: Physiological Integrity: Basic Care and Comfort

3. The portion of carbohydrates that cannot be broken down by intestinal enzymes and juices is
.

ANS:
fiber

Downloaded by: SUCCEEDGRADES | abbieclin@gmail.com


Distribution
Downloaded by Dallen Mae D.ofKadir
this document is illegal
(kadir.dallenmae.d.bcsi@gmail.com)
lOMoARcPSD|35346190

Stuvia.com - The Marketplace to Buy and Sell your Study Material

Fiber is the portion of carbohydrates that cannot be broken down by intestinal flora.

DIF: Cognitive Level: Knowledge REF: p. 465 OBJ: Theory #2


TOP: Theory #2 KEY: Nursing Process Step: N/A MSC: NCLEX: N/A

Downloaded by: SUCCEEDGRADES | abbieclin@gmail.com


Distribution
Downloaded by Dallen Mae D.ofKadir
this document is illegal
(kadir.dallenmae.d.bcsi@gmail.com)
lOMoARcPSD|35346190

Stuvia.com - The Marketplace to Buy and Sell your Study Material

Chapter 27: Nutritional Therapy and Assisted Feeding


Williams: deWit's Fundamental Concepts and Skills for Nursing, 8th Edition

MULTIPLE CHOICE

1. Before any nutritional tray is delivered to a patient, the nurse has the responsibility of:
a. determining if the patient needs assistance to eat.
b. confirming the diet on the tray with the diet sheet.
c. evaluating if the food is of the appropriate temperature.
d. adding extra salt and sugar packets.

ANS: B
The nutritional tray should be checked against the nutritional order to be sure that the patient
receives the proper nutritional. No matter who actually delivers the tray, it is the nurse who
confirms the accuracy of the diet.

DIF: Cognitive Level: Comprehension REF: p. 485|Skill 27-1


OBJ: Theory #1 TOP: Nurse Role KEY: Nursing Process Step: Implementation
MSC: NCLEX: Physiological Integrity: Basic Care and Comfort

2. The nutritional documentation that is most informative is:


a. ate all of lunch.
b. ate 50% of lunch without difficulty. Refused all solid food.
c. drank most of liquids without difficulty.
d. assisted feeding liquid diet, choked frequently.
ANS: B
Nutritional documentation should include percentage of intake and how it is tolerated.

DIF: Cognitive Level: Application REF: p. 485|Skill 27-1


OBJ: Theory #1 TOP: Nutrition Documentation
KEY: Nursing Process Step: Implementation
MSC: NCLEX: Physiological Integrity: Basic Care and Comfort

3. When assisting a patient with a severe visual impairment who wishes to feed himself, the
nurse could best facilitate the patient‘s eating by:
a. placing the plate on his lap.
b. seating the patient in a chair and placing over-the-bed table appropriately.
c. orienting the patient to the position of foods on the plate using a clock face
description.
d. placing each food in a separate container or bowl.
ANS: C
It is best to orient a visually impaired patient to the position of the foods on the plate by
describing the plate as if it is a clock face (3 o‘clock, 6 o‘clock, and so on).

DIF: Cognitive Level: Application REF: p. 485|Skill 27-1


OBJ: Theory #1 TOP: Assisting Patient with Eating
KEY: Nursing Process Step: Implementation
MSC: NCLEX: Psychosocial Integrity: Basic Care and Comfort

Downloaded by: SUCCEEDGRADES | abbieclin@gmail.com


Distribution
Downloaded by Dallen Mae D.ofKadir
this document is illegal
(kadir.dallenmae.d.bcsi@gmail.com)
lOMoARcPSD|35346190

Stuvia.com - The Marketplace to Buy and Sell your Study Material

4. A patient who underwent surgery has an order to begin a clear liquid diet and can be offered:
a. tea with milk.
b. Jell O.
c. cream soup.
d. fruit sherbet.

ANS: B
A clear liquid diet consists of foods that are liquid at room temperature and are clear, have a
low residue, and are easily digested. Gelatins are part of a clear liquid diet.

DIF: Cognitive Level: Comprehension REF: p. 488|Box 27-1


OBJ: Theory #2 TOP: Diet for Postoperative Patient
KEY: Nursing Process Step: Implementation
MSC: NCLEX: Physiological Integrity: Basic Care and Comfort

5. A nurse caring for a patient with bulimia nervosa should add to the care plan to assess for:
a. hiding food in napkins or under plate.
b. inducing self to vomit.
c. refusing to eat.
d. flushing food down commode.

ANS: B
With bulimia nervosa, along with binge eating, there is purging, fasting, and the use of
laxatives. These patients may eat everything on their tray then purge by inducing themselves
to vomit.

DIF: Cognitive Level: Comprehension REF: p. 488 OBJ: Theory #3


TOP: Bulimia Nervosa KEY: Nursing Process Step: Assessment
MSC: NCLEX: Physiological Integrity: Basic Care and Comfort

6. An obese clinic patient who is in the latter part of the first trimester of a pregnancy asks how
much weight she should gain. The nurse‘s best response is to say that the total weight gain
should be no more than:
a. 35 pounds.
b. 30 pounds.
c. 20 pounds.
d. 10 pounds.
ANS: C
Total weight gain for an obese patient should be no more than 20 pounds.

DIF: Cognitive Level: Comprehension REF: p. 490|Table 27-1


OBJ: Theory #4 TOP: Pregnancy KEY: Nursing Process Step: Implementation
MSC: NCLEX: Health Promotion and Maintenance: Prevention and Early Detection of Disease

7. The nurse takes into consideration that a patient who uses alcohol is at risk for a vitamin
deficiency in:
a. thiamine.
b. cyanocobalamin.
c. ascorbic acid.
d. iron.

Downloaded by: SUCCEEDGRADES | abbieclin@gmail.com


Distribution
Downloaded by Dallen Mae D.ofKadir
this document is illegal
(kadir.dallenmae.d.bcsi@gmail.com)
lOMoARcPSD|35346190

Stuvia.com - The Marketplace to Buy and Sell your Study Material

ANS: A
Thiamine deficiency is often present in patients who use alcohol.

DIF: Cognitive Level: Knowledge REF: p. 490 OBJ: Theory #4


TOP: Substance Related and Addictive Disorders
KEY: Nursing Process Step: Planning
MSC: NCLEX: Health Promotion and Maintenance: Prevention and Early Detection of Disease

8. A patient who is on a low cholesterol diet verbalizes that he enjoys eating meats and does not
intend to stop. The nurse‘s most helpful response would be, ―You can enjoy your meat if you
will concentrate on such meats as:
a. broiled sirloin steak.‖
b. fried catfish.‖
c. baked turkey breast.‖
d. sausage patties.‖
ANS: C
Red meat, eggs, and high-fat dairy products contain large amounts of saturated fat; poultry
(such as turkey breast) and fish are low-fat items and therefore are desirable when trying to
reduce serum cholesterol. Fried foods also contain extra cholesterol.

DIF: Cognitive Level: Comprehension REF: p. 491 OBJ: Theory #4


TOP: Patient Education KEY: Nursing Process Step: Implementation
MSC: NCLEX: Health Promotion and Maintenance: Prevention and Early Detection of Disease

9. An older male patient is concerned about his cholesterol laboratory report that shows an
elevated high density lipoprotein (HDL) level. The nurse explains that such a report indicates
that:
a. he should go on a strenuous low cholesterol diet.
b. he is at risk for hypertension.
c. is developing atherosclerosis.
d. his vessels are being cleansed of fatty deposits.
ANS: D
High density lipoprotein (HDL) is the ―good‖ cholesterol that tends to cleanse the vessels of
fatty deposits.

DIF: Cognitive Level: Comprehension REF: p. 491 OBJ: Theory #4


TOP: HDL KEY: Nursing Process Step: Implementation
MSC: NCLEX: Health Promotion and Maintenance: Prevention and Early Detection of Disease

10. An outpatient clinic nurse assesses a blood glucose level of 75 mg/dL in a patient who has
been on a low carbohydrate diet for the last 10 days. The nurse should:
a. notify the physician about the ineffectiveness of the diet.
b. document the finding.
c. suggest a moderate increase carbohydrate intake.
d. arrange a dietician consultation to discuss a more effective diet.
ANS: B
Document the finding. Normal blood sugar is between 70 and 120 mg/dL.

DIF: Cognitive Level: Application REF: p. 491 OBJ: Theory #5

Downloaded by: SUCCEEDGRADES | abbieclin@gmail.com


Distribution
Downloaded by Dallen Mae D.ofKadir
this document is illegal
(kadir.dallenmae.d.bcsi@gmail.com)
lOMoARcPSD|35346190

Stuvia.com - The Marketplace to Buy and Sell your Study Material

TOP: Disease Process Benefiting from Nutritional Therapy


KEY: Nursing Process Step: Implementation
MSC: NCLEX: Health Promotion and Maintenance: Prevention and Early Detection of Disease

11. Because of the patient‘s dysphagia, the nurse recommends to the physician that the patient be
placed on a Level II texture level diet, which means that the food is:
a. thickened to prevent aspiration.
b. pureed to a pudding consistency.
c. mechanically altered, moist, minced helpings.
d. minced into bite size pieces.
ANS: C
Level II texture is a diet in which the food has been mechanically altered to moist, 1/4 inch
pieces.

DIF: Cognitive Level: Comprehension REF: p. 495 OBJ: Theory #3


TOP: Nutritional Modifications KEY: Nursing Process Step: Implementation
MSC: NCLEX: Physiological Integrity: Reduction of Risk

12. A nurse positions a patient for the insertion of a nasogastric (NG) tube by:
a. turning the patient to a right side lying position.
b. sitting the patient upright and hyperextending the patient‘s head.
c. lowering the head of the bed to a flat position.
d. raising the head of the bed to 30 degrees.
ANS: B
The head of the bed should be raised and the patient asked to hyperextend his head to
facilitate the passage of the NG tube.

DIF: Cognitive Level: Application REF: p. 497|Skill 27-2


OBJ: Theory #6 TOP: NG Tube Insertion
KEY: Nursing Process Step: Implementation
MSC: NCLEX: Physiological Integrity: Reduction of Risk

13. The nurse who is preparing to give a feeding per a nasogastric (NG) tube tests the placement
of the tube most safely by:
a. checking the lungs for rhonchi.
b. instilling 10 mL of sterile water and checking for cough.
c. aspirating stomach contents.
d. injecting 20 mL of air and listen at the tip of the xiphoid.
ANS: C
The safest and most assured method to test for NG tube placement is to aspirate stomach
contents and check fluid for pH. Using the air method is not as accurate as the stomach
aspiration.

DIF: Cognitive Level: Application REF: p. 497|Skill 27-2


OBJ: Clinical Practice #3 TOP: NG Tube Insertion
KEY: Nursing Process Step: Implementation
MSC: NCLEX: Physiological Integrity: Reduction of Risk

Downloaded by: SUCCEEDGRADES | abbieclin@gmail.com


Distribution
Downloaded by Dallen Mae D.ofKadir
this document is illegal
(kadir.dallenmae.d.bcsi@gmail.com)
lOMoARcPSD|35346190

Stuvia.com - The Marketplace to Buy and Sell your Study Material

14. Stopping the infusion and checking for residual volume, the nurse aspirates 250 mL of gastric
contents. The nurse should next:
a. replace the aspirate and continue with the feeding.
b. throw the aspirate away and flush the tubing.
c. replace the aspirate and delay feeding for 1 to 2 hours.
d. throw the aspirate away and delay feeding for 2 hours.
ANS: C
If the residual volume is greater than 250 mL (or per agency policy), replace the withdrawn
fluids, document the residual, and notify the RN or primary care provider (promotility
medications may be ordered), and delay further feeding for 1 to 2 hours if facility policy states
to do so.

DIF: Cognitive Level: Application REF: p. 500 OBJ: Clinical Practice #4


TOP: Residual Volume KEY: Nursing Process Step: Implementation
MSC: NCLEX: Physiological Integrity: Basic Care and Comfort

15. A patient is scheduled to receive an intermittent tube feeding. This feeding should be allowed
to flow in over how many minutes?
a. 1 minute
b. 2 minutes
c. 5 minutes
d. 10 minutes
ANS: D
An intermittent feeding should take approximately 10 minutes to flow into the tube.

DIF: Cognitive Level: Comprehension REF: p. 500 OBJ: Theory #7


TOP: Tube Feeding KEY: Nursing Process Step: Implementation
MSC: NCLEX: Physiological Integrity: Reduction of Risk

16. When the patient has just finished receiving a tube feeding, the nurse leaves the head of the
patient‘s bed elevated for 30 to 60 minutes after feeding in order to:
a. facilitate stomach emptying and prevent aspiration.
b. maintain skin integrity to the buttocks.
c. facilitate lung drainage and promote ventilation.
d. prevent feeding tube from clogging.
ANS: A
The head of the bed should be left elevated at a 30- to 90-degree angle for 30 to 60 minutes
after the feeding to help reduce the risk of aspiration.

DIF: Cognitive Level: Comprehension REF: p. 501 OBJ: Clinical Practice #4


TOP: Tube Feeding KEY: Nursing Process Step: Implementation
MSC: NCLEX: Physiological Integrity: Reduction of Risk

17. The nurse caring for the patient receiving total parenteral nutrition (TPN) should monitor the
flow rate every:
a. 2 hours.
b. 3 hours.
c. 4 hours.
d. 6 hours.

Downloaded by: SUCCEEDGRADES | abbieclin@gmail.com


Distribution
Downloaded by Dallen Mae D.ofKadir
this document is illegal
(kadir.dallenmae.d.bcsi@gmail.com)
lOMoARcPSD|35346190

Stuvia.com - The Marketplace to Buy and Sell your Study Material

ANS: C
Both the IV site and the flow rate should be monitored every 4 hours and the site assessed for
infection.

DIF: Cognitive Level: Knowledge REF: p. 504|Table 27-4


OBJ: Theory #8 TOP: TPN KEY: Nursing Process Step: Implementation
MSC: NCLEX: Physiological Integrity: Basic Care and Comfort

18. The nurse explains that adequate vitamin D can be acquired by:
a. eating 2 oranges a day every week.
b. eating fish 3 times a week.
c. having 10 to 15 minutes of sunshine exposure per day.
d. eating green leafy vegetables.
ANS: C
10 to 15 minutes of sunshine exposure is sufficient to produce sufficient vitamin D for people
with lighter skin

DIF: Cognitive Level: Comprehension REF: p. 491 OBJ: Theory #5


TOP: Vitamin D KEY: Nursing Process Step: Assessment
MSC: NCLEX: Physiological Integrity: Basic Care and Comfort

19. The nurse is caring for a patient who has total parenteral nutrition (TPN) running finds that
the infusion is behind by 200 mL. The nurse should:
a. increase the flow rate 5% until the infusion has caught up.
b. check the patient‘s stomach residual volume.
c. elevate the head of the bed 30 degrees.
d. document the discrepancy and report to the charge nurse.
ANS: D
The rate of a TPN is never increased because of the danger of causing hyperglycemia or
circulatory overload. The discrepancy is to be documented and reported to the charge nurse or
physician.

DIF: Cognitive Level: Application REF: p. 504 OBJ: Theory #1


TOP: Monitoring TPN KEY: Nursing Process Step: Implementation
MSC: NCLEX: Physiological Integrity: Basic Care and Comfort

20. A patient recently started on enteral tube feedings starts complaining of nausea and having
diarrhea. The best nursing action is to:
a. check the enteral tube for placement.
b. slow down the feedings and monitor.
c. perform a fingerstick blood glucose test.
d. stop the feedings and inform the physician.

ANS: D
Nausea, constipation, and diarrhea are concerns following institution of tube feedings.

DIF: Cognitive Level: Application REF: p. 504|Box 27-3


OBJ: Clinical Practice #4 TOP: Enteral Nutrition
KEY: Nursing Process Step: Implementation
MSC: NCLEX: Physiological Integrity: Basic Care and Comfort

Downloaded by: SUCCEEDGRADES | abbieclin@gmail.com


Distribution
Downloaded by Dallen Mae D.ofKadir
this document is illegal
(kadir.dallenmae.d.bcsi@gmail.com)
lOMoARcPSD|35346190

Stuvia.com - The Marketplace to Buy and Sell your Study Material

21. A patient has a new order to have an NG tube removed. The nurse should initially:
a. wash her hands and apply clean gloves.
b. encourage mouth care as needed.
c. explain the procedure to the patient.
d. pinch the tube while removing it.

ANS: C
Explaining the procedure to the patient before starting helps in gaining the patient‘s
confidence.

DIF: Cognitive Level: Application REF: p. 500|Step 27-2


OBJ: Clinical Practice #3 TOP: NG Tube Removal
KEY: Nursing Process Step: Implementation
MSC: NCLEX: Safe, Effective Care Environment

22. A nurse is instructing a family member who will be caring for a patient receiving enteral
feedings after discharge to home. The nurse would emphasize:
a. taping the gastrostomy tube so that it does not hang lower than the stomach.
b. discarding unused opened refrigerated formula after 3 to 4 days.
c. administering tube feedings while they are still cold from the refrigerator.
d. mixing all medications together for administration at the same time.
ANS: A
The tube should be taped so that it is higher than the entry point into the body.

DIF: Cognitive Level: Application REF: p. 505 OBJ: Clinical Practice #2


TOP: Health Education KEY: Nursing Process Step: Implementation
MSC: NCLEX: Physiological Integrity: Basic Care and Comfort

23. The nurse inserting an NG tube through the nostril into the back of the throat of a patient
would instruct the patient to:
a.hyperextend the head.
b.cough forcefully.
c.drop head forward and begin to swallow.
d.open mouth and extend tongue.
ANS: C
The patient should be instructed to tip the head forward and begin to swallow to help advance
the tube through the esophagus.

DIF: Cognitive Level: Application REF: p. 498|Skill 27-2


OBJ: Clinical Practice #3 TOP: NG Tube Insertion
KEY: Nursing Process Step: Implementation
MSC: NCLEX: Safe, Effective Care Environment

24. The nurse caring for a patient receiving enteral feedings would assess for tolerance of the
feeding by monitoring:
a. for gastric tube patency.
b. for duodenal tube patency.
c. for abdominal distention.
d. the rate of the feeding.

Downloaded by: SUCCEEDGRADES | abbieclin@gmail.com


Distribution
Downloaded by Dallen Mae D.ofKadir
this document is illegal
(kadir.dallenmae.d.bcsi@gmail.com)
lOMoARcPSD|35346190

Stuvia.com - The Marketplace to Buy and Sell your Study Material

ANS: C
Assessing the abdomen for distention helps the nurse identify the intolerance of tube feedings.

DIF: Cognitive Level: Application REF: p. 502|Skill 27-3


OBJ: Clinical Practice #4 TOP: Enteral Nutrition
KEY: Nursing Process Step: Assessment
MSC: NCLEX: Physiological Integrity: Basic Care and Comfort

25. While the nurse is explaining the procedure for inserting a tube for enteral feedings, the
patient interrupts and asks why there is a need for this tube. The nurse‘s best response is:
a. ―Your physician has ordered this to help your condition.‖
b. ―Tell me what your primary care provider told you about this procedure.‖
c. ―Are you telling me you don‘t want this tube inserted?‖
d. ―This tube placement will only be temporary.‖
ANS: B
In assessing the patient‘s understanding, the nurse should assess the level of the patient‘s
understanding and knowledge about the procedure.

DIF: Cognitive Level: Application REF: p. 497|Skill 27-2


OBJ: Clinical Practice #3 TOP: NG Tube Insertion
KEY: Nursing Process Step: Assessment
MSC: NCLEX: Psychosocial Integrity: Basic Care and Comfort

26. When caring for a patient receiving total parenteral nutrition, the nurse knows that it is
essential to:
a. check the flow rate every shift.
b. order electrolytes daily.
c. monitor IV site every shift.
d. monitor the blood glucose.
ANS: D
Total parenteral nutrition contains a high concentration of glucose, and monitoring blood
glucose every 6 to 8 hours will determine patient tolerance.

DIF: Cognitive Level: Comprehension REF: p. 504 OBJ: Theory #10


TOP: TPN KEY: Nursing Process Step: Assessment
MSC: NCLEX: Physiological Integrity: Reduction of Risk

MULTIPLE RESPONSE

1. A nurse instructs a patient who is to receive a soft diet that the diet will include: (Select all
that apply.)
a. eggs.
b. multigrain bread.
c. baked potato.
d. soups.
e. fruit juices.
f. milk products.
ANS: A, D, E, F

Downloaded by: SUCCEEDGRADES | abbieclin@gmail.com


Distribution
Downloaded by Dallen Mae D.ofKadir
this document is illegal
(kadir.dallenmae.d.bcsi@gmail.com)
lOMoARcPSD|35346190

Stuvia.com - The Marketplace to Buy and Sell your Study Material

Soft diets are low in fiber, and foods are softened by cooking, mashing, or chopping. Foods
allowed on a soft diet include eggs, breads without seeds, boiled or mashed potatoes, soups,
fruit juices, tender cooked vegetables, meat that is stewed, boiled or ground, cooked cereals,
mashed bananas, applesauce, and milk products.

DIF: Cognitive Level: Comprehension REF: p. 487 OBJ: Theory #1


TOP: Nutrition KEY: Nursing Process Step: Implementation
MSC: NCLEX: Physiological Integrity: Basic Care and Comfort

2. When the patient complains about the insertion of the total parenteral nutrition (TPN) tube
interfering with his movement, the nurse explains that the insertion in the subclavian vein
allows: (Select all that apply.)
a. adequate dilution of TPN solution.
b. closer proximity to the heart.
c. more effective monitoring from the IV pump.
d. for adequate blood flow.
e. for more ease in dressing insertion site.
ANS: A, D
The placement in the subclavian provides a large vein with large blood flow, which dilutes the
TPN to keep venous irritation at a minimum and ensures better distribution.

DIF: Cognitive Level: Application REF: p. 504 OBJ: Theory #10


TOP: Insertion Site KEY: Nursing Process Step: Implementation
MSC: NCLEX: Physiological Integrity: Basic Care and Comfort

3. A nurse giving a bolus feeding through a nasogastric tube with a syringe would: (Select all
that apply.)
a. pull up 50 mL of formula in the syringe.
b. lower the head of the bed to flat position.
c. allow feeding to flow in by gravity.
d. flush the tube with 50 mL of water.
e. check the position of the tube.
ANS: C, E
The nurse should roll up the head of the bed, check the placement of the tube, allow 30 mL of
formula to flow in by gravity, and flush the tube with 30 mL of sterile water.

DIF: Cognitive Level: Application REF: p. 497 OBJ: Theory #2


TOP: Nutrition KEY: Nursing Process Step: Implementation
MSC: NCLEX: Physiological Integrity: Basic Care and Comfort

4. A nurse caring for a patient diagnosed with AIDS would include in the nutritional plan of
care: (Select all that apply.)
a. asking the patient about sexual history.
b. encouraging the patient to eat solid foods that are high in protein.
c. offering the patient supplements such as Ensure.
d. obtaining an order for a dietitian consult.
e. urging the patient to eat three well-balanced meals per day.
f. offering pureed foods when the patient‘s mouth is painful.
ANS: C, D, F

Downloaded by: SUCCEEDGRADES | abbieclin@gmail.com


Distribution
Downloaded by Dallen Mae D.ofKadir
this document is illegal
(kadir.dallenmae.d.bcsi@gmail.com)
lOMoARcPSD|35346190

Stuvia.com - The Marketplace to Buy and Sell your Study Material

Solid food may be difficult to eat, so consulting with a dietitian and having the patient eat
foods that are high in protein and that are bland or pureed are very appropriate. AIDS patients
should eat small meals several times a day.

DIF: Cognitive Level: Application REF: p. 493 OBJ: Theory #3


TOP: HIV/AIDS and Nutrition KEY: Nursing Process Step: Implementation
MSC: NCLEX: Physiological Integrity: Basic Care and Comfort

5. The nurse counsels the bulimic patient that her eating disorder can lead to serious conditions
such as: (Select all that apply.)
a. esophageal ulcers.
b. diverticulitis.
c. ulcerative colitis.
d. peptic ulcers.
e. heart failure.
ANS: A, D
Long-term bulimic patients may acquire serious conditions such as esophageal and peptic
ulcers.

DIF: Cognitive Level: Comprehension REF: p. 488 OBJ: Theory #4


TOP: Feeding and Eating Disorders KEY: Nursing Process Step: Implementation
MSC: NCLEX: Health Promotion and Maintenance: Prevention and Early Detection of Disease

6. The nurse suggests to a diabetic patient to eat complex carbohydrates, which include: (Select
all that apply.)
a. whole grain foods.
b. brown rice.
c. lima beans.
d. legumes.
e. sweet potatoes.
ANS: A, B, D
Complex carbohydrates usually contain more nutrients and higher fiber content.

DIF: Cognitive Level: Comprehension REF: p. 492 OBJ: Theory #1


TOP: Topic: Complex Carbohydrates KEY: Nursing Process Step: Implementation
MSC: NCLEX: Health Promotion and Maintenance: Prevention and Early Detection of Disease

7. Fluid overload is suspected in an 82-year-old patient who is receiving total parenteral


nutritional (TPN) therapy when the nurse assesses: (Select all that apply.)
a. excessive urine output.
b. increased pulse rate.
c. dyspnea.
d. hyperactive bowel sounds.
e. complaint of headache.

ANS: A, B, C
Indications of fluid overload include output exceeding intake, increased pulse rate, dyspnea,
cough, and crackles.

DIF: Cognitive Level: Comprehension REF: p. 504 OBJ: Theory #9

Downloaded by: SUCCEEDGRADES | abbieclin@gmail.com


Distribution
Downloaded by Dallen Mae D.ofKadir
this document is illegal
(kadir.dallenmae.d.bcsi@gmail.com)
lOMoARcPSD|35346190

Stuvia.com - The Marketplace to Buy and Sell your Study Material

TOP: Fluid Overload KEY: Nursing Process Step: Assessment


MSC: NCLEX: Health Promotion and Maintenance: Prevention and Early Detection of Disease

COMPLETION

1. The correct anatomic landmarks to follow when inserting a nasogastric tube is to measure
from the to the and then to the .

ANS:
tip of the nose; tip of the ear; xiphoid process

Using landmarks individualizes the tube length.

DIF: Cognitive Level: Application REF: p. 497|Skill 27-2


OBJ: Clinical Practice #3 TOP: Nasogastric Tube Insertion
KEY: Nursing Process Step: Implementation
MSC: NCLEX: Physiological Integrity: Basic Care and Comfort

2. The nurse recommends to a patient who desires weight reduction to engage in a minimum of
minutes of exercise most days of the week.

ANS:
30

Physical exercise of moderate intensity for 30 to 60 minutes most days of the week is an
effective support to weight reduction.

DIF: Cognitive Level: Comprehension REF: p. 490 OBJ: Theory #4


TOP: Weight Reduction KEY: Nursing Process Step: Implementation
MSC: NCLEX: Health Promotion and Maintenance: Prevention and Early Detection of Disease

3. The nurse takes into consideration that excessive intake of saturated and trans fat leads to the
development of fatty deposits being laid down in the walls of the blood vessels and causing
.

ANS:
atherosclerosis

Excessive intake of saturated and trans fat leads to the development of fatty deposits being
laid down in the walls of the blood vessels called atherosclerosis.

DIF: Cognitive Level: Knowledge REF: p. 491 OBJ: Theory #4


TOP: Atherosclerosis KEY: Nursing Process Step: Implementation
MSC: NCLEX: Health Promotion and Maintenance: Prevention and Early Detection of Disease

Downloaded by: SUCCEEDGRADES | abbieclin@gmail.com


Distribution
Downloaded by Dallen Mae D.ofKadir
this document is illegal
(kadir.dallenmae.d.bcsi@gmail.com)
lOMoARcPSD|35346190

Stuvia.com - The Marketplace to Buy and Sell your Study Material

Chapter 28: Assisting with Respiration and Oxygen Delivery


Williams: deWit's Fundamental Concepts and Skills for Nursing, 8th Edition

MULTIPLE CHOICE

1. The nurse uses a diagram to show that when the diaphragm moves:
a. up, the increased negative pressure in the thoracic space forces air into the lungs.
b. down, the intercostal muscles retract, forcing air out of the lungs.
c. down, the negative pressure in the thoracic space pulls air into the lungs.
d. up, the decreased negative pressure allows air to enter the lungs.

ANS: C
When the diaphragm moves down, increasing the size of the thoracic space, air is pulled into
the lungs. The respiratory action is controlled by the spinal cord.

DIF: Cognitive Level: Knowledge REF: p. 509 OBJ: Theory #1


TOP: Respiratory Action KEY: Nursing Process Step: Implementation
MSC: NCLEX: Physiological Integrity: Physiological Adaptation

2. The nurse clarifies that the condition in which there is a decreased amount of oxygen in the
blood is:
a. hypoxia.
b. hypercapnia.
c. dyspnea.
d. hypoxemia.
ANS: D
Hypoxemia is a condition in which there is a decreased amount of oxygen in the blood,
hypoxia is inadequate oxygen to meet cellular needs, hypercapnia is increased level of carbon
dioxide in the blood, and dyspnea is difficulty breathing.

DIF: Cognitive Level: Comprehension REF: p. 510 OBJ: Theory #1


TOP: Oxygen Levels KEY: Nursing Process Step: Implementation
MSC: NCLEX: Physiological Integrity: Physiological Adaptation

3. The nurse monitoring patients eating in the dining room of a skilled nursing facility notes that
a patient begins choking. As the nurse prepares to deliver the Heimlich maneuver, the fist
should be positioned:
a. halfway between the xiphoid process and the umbilicus.
b. directly over the sternum.
c. between the umbilicus and the symphysis pubis.
d. directly over the umbilicus.
ANS: A
Proper placement of the fist is halfway between the xiphoid process and the umbilicus.

DIF: Cognitive Level: Application REF: p. 514|Skill 28-2


OBJ: Clinical Practice #1 TOP: Heimlich Maneuver
KEY: Nursing Process Step: Implementation
MSC: NCLEX: Physiological Integrity: Basic Care and Comfort

Downloaded by: SUCCEEDGRADES | abbieclin@gmail.com


Distribution
Downloaded by Dallen Mae D.ofKadir
this document is illegal
(kadir.dallenmae.d.bcsi@gmail.com)
lOMoARcPSD|35346190

Stuvia.com - The Marketplace to Buy and Sell your Study Material

4. A patient has collapsed and cannot be aroused by asking loudly, ―Are you okay?‖ The next
action should be to:
a. position the fingers over the carotid artery to feel for a pulse.
b. tilt the head by placing one hand on the forehead and lift the chin.
c. call for help or, if there is assistance, have that person get help.
d. deliver two quick short breaths into the patient‘s airway.
ANS: C
The sequence for resuscitative interventions is to check for responsiveness; if no response,
activate emergency medical services, check for pulse at carotid, begin compressions, then
open the airway and check for breathing.

DIF: Cognitive Level: Application REF: p. 515|Skill 28-3


OBJ: Theory #3 TOP: Basic Life Support
KEY: Nursing Process Step: Implementation
MSC: NCLEX: Health Promotion and Maintenance: Prevention and Early Detection of Disease

5. The nurse instructing the patient to perform forceful exhalation coughing would instruct the
patient to take in:
a. one deep breath and quickly exhale.
b. two breaths and force the air out quickly.
c. two deep breaths, then inhale deeply again and force out the air quickly.
d. one breath, hold it for 3 seconds, then forcefully exhale three times with mouth
open.
ANS: C
Proper coughing procedure is to take in two deep breaths, inhale deeply again and to forcibly
exhale (cough) at the end of the third breath. This technique is very effective in moving
secretions up the bronchial tree.

DIF: Cognitive Level: Knowledge REF: p. 513 OBJ: Clinical Practice #1


TOP: Effective Coughing KEY: Nursing Process Step: Implementation
MSC: NCLEX: Physiological Integrity: Basic Care and Comfort

6. The nurse is aware that the best time to schedule a postural drainage treatment is:
a. shortly after the patient arises in the morning, before breakfast.
b. in the morning immediately after breakfast.
c. 30 minutes after lunch.
d. 1 hour after supper.
ANS: A
Postural drainage is best accomplished in the morning prior to eating, because more secretions
accumulate while the patient is asleep.

DIF: Cognitive Level: Comprehension REF: p. 518 OBJ: Clinical Practice #1


TOP: Postural Drainage KEY: Nursing Process Step: Planning
MSC: NCLEX: Physiological Integrity: Basic Care and Comfort

7. A patient who will begin oxygen therapy has a history of sinus disorders. This patient would
benefit most from which oxygen setup?
a. High oxygen flow rate
b. A humidifier

Downloaded by: SUCCEEDGRADES | abbieclin@gmail.com


Distribution
Downloaded by Dallen Mae D.ofKadir
this document is illegal
(kadir.dallenmae.d.bcsi@gmail.com)
lOMoARcPSD|35346190

Stuvia.com - The Marketplace to Buy and Sell your Study Material

c. A Venturi mask
d. A nasal cannula

ANS: B
If a patient suffers from sinus problems, it is best to add a humidifier to the oxygen setup.

DIF: Cognitive Level: Application REF: p. 520|Skill 28-4


OBJ: Clinical Practice #2 TOP: Oxygen Therapy
KEY: Nursing Process Step: Implementation
MSC: NCLEX: Physiological Integrity: Basic Care and Comfort

8. A patient has a history of chronic obstructive pulmonary disease. The patient‘s oxygen flow
rate should be set to no more than:
a. 5 to 10 L/min.
b. 4 to 5 L/min.
c. 2 to 3 L/min.
d. 1 to 2 L/min.
ANS: C
Patients with obstructive lung diseases should be given only 2 to 3 L/min because higher
concentrations of oxygen reduce the respiratory rate. This is because their incentive to breathe
comes from lower oxygen levels rather than higher carbon dioxide levels in the blood (they
usually have a continuous high level of carbon dioxide).

DIF: Cognitive Level: Comprehension REF: p. 519 OBJ: Theory #5


TOP: Oxygen Therapy KEY: Nursing Process Step: Implementation
MSC: NCLEX: Physiological Integrity: Reduction of Risk

9. The nurse loosens mucus plugs by using percussion on a patient over the area of the:
a. sternum.
b. thorax.
c. spine between the scapulae.
d. midaxillary line on the rib cage.
ANS: B
Percussion, a rhythmic clapping with cupped hands over the thoracic area, will loosen mucus
plugs. This technique is both useless and painful when applied over bony areas.

DIF: Cognitive Level: Knowledge REF: p. 518 OBJ: Clinical Practice #1


TOP: Oxygen Therapy: Percussion KEY: Nursing Process Step: Implementation
MSC: NCLEX: Physiological Integrity: Physiological Adaptation

10. A patient requires a precise concentration of 40% oxygen. Which of the following devices
would best allow for this?
a.A simple face mask
b.A nonrebreather mask
c.A partial rebreathing mask
d.A Venturi mask
ANS: D
A Venturi mask is useful when accuracy of delivery is essential.

DIF: Cognitive Level: Comprehension REF: p. 523|Table 28-3

Downloaded by: SUCCEEDGRADES | abbieclin@gmail.com


Distribution
Downloaded by Dallen Mae D.ofKadir
this document is illegal
(kadir.dallenmae.d.bcsi@gmail.com)
lOMoARcPSD|35346190

Stuvia.com - The Marketplace to Buy and Sell your Study Material

OBJ: Clinical Practice #2 TOP: Oxygen Therapy: Venturi Mask


KEY: Nursing Process Step: Implementation
MSC: NCLEX: Physiological Integrity: Basic Care and Comfort

11. The nurse recognizes that a postoperative patient who can breathe independently but has
trouble maintaining an airway because of the tongue falling back into the throat would be best
benefitted by a(n):
a. pharyngeal airway.
b. endotracheal tube.
c. tracheostomy.
d. partial rebreather oxygen mask.

ANS: A
A pharyngeal airway such as a nasopharyngeal or an oropharyngeal airway is useful for
patients who can breathe on their own but tend to occlude the airway with the tongue.

DIF: Cognitive Level: Analysis REF: p. 522 OBJ: Theory #4


TOP: Airway KEY: Nursing Process Step: Assessment
MSC: NCLEX: Health Promotion and Maintenance: Prevention and Early Detection of Disease

12. A nurse performing oral suctioning on an adult patient should set the wall suction machine so
that the suction pressure is between:
a. 25 and 50 mm Hg.
b. 50 and 75 mm Hg.
c. 80 and 120 mm Hg.
d. 120 and 180 mm Hg.

ANS: C
The range of suction pressure for an adult patient is between 80 and 120 mm Hg.

DIF: Cognitive Level: Application REF: p. 524 OBJ: Clinical Practice #1


TOP: Suctioning KEY: Nursing Process Step: Implementation
MSC: NCLEX: Physiological Integrity: Basic Care and Comfort

13. A nurse caring for a patient with a tracheostomy should determine whether the patient needs
suctioning by:
a. monitoring the rate of respirations.
b. determining the last time the patient was suctioned.
c. examining the character of the sputum.
d. auscultating the breath sounds.

ANS: D
Auscultating the patient‘s breath sounds helps the nurse assesses for retained secretions and
verifies the need for suctioning. The respiratory rate may rise when suctioning is needed, but
it could also rise for other reasons.

DIF: Cognitive Level: Application REF: p. 527|Skill 28-6


OBJ: Clinical Practice #1 TOP: Suctioning
KEY: Nursing Process Step: Implementation
MSC: NCLEX: Physiological Integrity: Basic Care and Comfort

Downloaded by: SUCCEEDGRADES | abbieclin@gmail.com


Distribution
Downloaded by Dallen Mae D.ofKadir
this document is illegal
(kadir.dallenmae.d.bcsi@gmail.com)
lOMoARcPSD|35346190

Stuvia.com - The Marketplace to Buy and Sell your Study Material

14. A patient requires suctioning via the nasotracheal route. In order to perform this procedure
safely, the nurse should:
a. apply suction while advancing the catheter into the airway.
b. suction the nasotracheal passage after suctioning the mouth.
c. hold the catheter with the dominant hand after donning sterile gloves.
d. insert the nonlubricated catheter into the nasal passage.
ANS: C
The suction catheter should be held with the dominant hand after donning sterile gloves,
because sterile technique must be adhered to when suctioning both the nasopharyngeal and
tracheal areas.

DIF: Cognitive Level: Application REF: p. 525|Skill 28-5


OBJ: Clinical Practice #1 TOP: Suctioning
KEY: Nursing Process Step: Implementation
MSC: NCLEX: Safe, Effective Care Environment: Safety and Infection Control

15. The nurse recognizes that, immediately before a tracheotomy cuff deflation, the patient
should:
a. be administered extra oxygen.
b. have the pharynx suctioned.
c. have the cuff pressure checked.
d. be monitored for respiratory rate.
ANS: B
Immediately before deflating a cuff on a tracheotomy tube, the pharynx should be suctioned to
prevent accumulated oral secretions from entering the bronchial tree once the cuff is deflated.

DIF: Cognitive Level: Application REF: p. 529 OBJ: Clinical Practice #3


TOP: Tracheostomy KEY: Nursing Process Step: Implementation
MSC: NCLEX: Physiological Integrity: Reduction of Risk

16. The nurse takes into consideration that while caring for a patient on oxygen therapy, safety
precautions should be observed, which include:
a. using clothing of synthetic cloth for the patient.
b. removing any adhesive from the patient‘s skin with acetone.
c. assessing equipment in room for frayed cords.
d. reducing humidification on the oxygen delivery device.

ANS: C
All equipment in a room where oxygen is being administered should be in good working order
without frayed or loose connections because of the possibility of fire.

DIF: Cognitive Level: Comprehension REF: p. 518 OBJ: Clinical Practice #5


TOP: Safety Precautions with Oxygen KEY: Nursing Process Step: Planning
MSC: NCLEX: Safe, Effective Care Environment: Safety and Infection Control

17. A nurse caring for a patient with a water seal type chest drainage that is on low suction
assesses that there is constant bubbling in the suction container. The nurse should:
a. immediately turn the patient to the side of the insertion site.
b. check for air leaks in drainage system.
c. document findings.

Downloaded by: SUCCEEDGRADES | abbieclin@gmail.com


Distribution
Downloaded by Dallen Mae D.ofKadir
this document is illegal
(kadir.dallenmae.d.bcsi@gmail.com)
lOMoARcPSD|35346190

Stuvia.com - The Marketplace to Buy and Sell your Study Material

d. clamp the chest tube and place the patient in high Fowler‘s position.
ANS: C
Document findings. Constant bubbling in the suction chamber indicates that suction is on.

DIF: Cognitive Level: Analysis REF: p. 521 OBJ: Clinical Practice #4


TOP: Coughing and Deep Breathing KEY: Nursing Process Step: Implementation
MSC: NCLEX: Physiological Integrity: Basic Care and Comfort

18. A nurse is aware that adequate hydration is necessary to mobilize respiratory secretions. To
thin respiratory secretions for easier expectoration, the patient should consume at least:
a. 500 to 1000 mL/day.
b. 1000 to 1500 mL/day.
c. 1500 to 2000 mL/day.
d. 2500 to 3000 mL/day.
ANS: C
A fluid intake of at least 1500 to 2000 mL/day is needed to thin respiratory secretions for
easier removal by coughing.

DIF: Cognitive Level: Comprehension REF: p. 533 OBJ: Clinical Practice #1


TOP: Mobilizing SecretionsKEY: Nursing Process Step: Implementation
MSC: NCLEX: Physiological Integrity: Basic Care and Comfort

19. The nurse would determine that this patient is aware of how to use the incentive spirometer
device properly when the patient:
a. took 10 slow, deep breaths every hour.
b. took five quick ―huffs‖ and then coughed vigorously.
c. exhaled deeply and then inhaled quickly and forcefully three times.
d. took five deep breaths slowly every 4 hours.
ANS: A
Proper technique for use of an incentive spirometer is to take 10 slow, deep breaths every hour
and to hold each breath for 3 seconds to enhance gas exchange.

DIF: Cognitive Level: Knowledge REF: p. 534 OBJ: Clinical Practice #1


TOP: Incentive Spirometer KEY: Nursing Process Step: Evaluation
MSC: NCLEX: Physiological Integrity: Basic Care and Comfort

20. The nurse assists the patient with emphysema into the most beneficial position to facilitate
respiration, which is:
a. semi-Fowler‘s position with a single pillow behind the head.
b. high Fowler‘s position without a pillow behind the head.
c. right lateral with the head of the bed elevated 45 degrees.
d. sitting upright and forward with arms supported on an over the bed table.
ANS: D
Sitting upright and leaning forward with arms supported on an over the bed table is best for
this patient, because it allows for expansion of the thoracic cage in all four directions (front,
back, and two sides).

DIF: Cognitive Level: Application REF: p. 534 OBJ: Clinical Practice #1


TOP: Positioning KEY: Nursing Process Step: Implementation

Downloaded by: SUCCEEDGRADES | abbieclin@gmail.com


Distribution
Downloaded by Dallen Mae D.ofKadir
this document is illegal
(kadir.dallenmae.d.bcsi@gmail.com)
lOMoARcPSD|35346190

Stuvia.com - The Marketplace to Buy and Sell your Study Material

MSC: NCLEX: Physiological Integrity: Basic Care and Comfort

21. The nurse performing tracheotomy care will:


a. raise the head of the bed to high Fowler‘s position.
b. remove the inner cannula with the ungloved hand.
c. suction tracheotomy before beginning care.
d. clean cannula with gauze and replace and lock.

ANS: C
Proper procedure includes suctioning the tracheostomy before beginning care.

DIF: Cognitive Level: Application REF: p. 535|Skill 28-7


OBJ: Clinical Practice #3 TOP: Tracheotomy Care
KEY: Nursing Process Step: Planning
MSC: NCLEX: Safe, Effective Care Environment: Safety and Infection Control

22. The nurse caring for a patient with a disposable chest drainage system can promote effective
tube function and patient safety by:
a. taping all connections within the system.
b. keeping the system at the level of the patient‘s chest.
c. turning on suction to 35 cm.
d. looping the tubing between the mattress and the bed rail to minimize length.
ANS: A
All connections in the system should be taped. Suction should be set at 20 cm unless ordered
otherwise. Looping the tubing encourages plugs in the tubing.

DIF: Cognitive Level: Application REF: p. 530|Step 28-1


OBJ: Clinical Practice #4 TOP: Chest Tube Care
KEY: Nursing Process Step: Implementation
MSC: NCLEX: Physiological Integrity: Reduction of Risk

23. The nurse takes into consideration that a pulse oximeter may not give an accurate reading if
the patient is:
a. dark skinned.
b. jaundiced.
c. obese.
d. febrile.
ANS: B
An accurate reading is dependent on light passing through the vascular bed. Jaundice may
cause an inaccurate reading.

DIF: Cognitive Level: Knowledge REF: p. 512|Skill 28-1


OBJ: Theory #1 TOP: Pulse Oximetry
KEY: Nursing Process Step: Assessment
MSC: NCLEX: Physiological Integrity: Physiological Adaptation

24. The nurse clarifies that the cough mechanism is stimulated when:
a. foreign substances are propelled by the cilia toward the respiratory tract.
b. dehumidified air enters the upper airway passages.
c. more than 250 mL of air moves in and out of the lungs with each breath.

Downloaded by: SUCCEEDGRADES | abbieclin@gmail.com


Distribution
Downloaded by Dallen Mae D.ofKadir
this document is illegal
(kadir.dallenmae.d.bcsi@gmail.com)
lOMoARcPSD|35346190

Stuvia.com - The Marketplace to Buy and Sell your Study Material

d. the blood transports carbon dioxide to the lungs.


ANS: A
Cilia work to propel foreign substances toward the entrance of the respiratory tract, and the
cough reflex works to expel the secretions.

DIF: Cognitive Level: Knowledge REF: p. 509 OBJ: Theory #1


TOP: Respiratory Structure Function KEY: Nursing Process Step: Implementation
MSC: NCLEX: Physiological Integrity: Basic Care and Comfort

25. When assessing the lungs of a patient, the nurse assesses a wheezing sound on inspiration.
This finding is documented as:
a. apnea.
b. dyspnea.
c. stridor.
d. retractions.
ANS: C
Stridor is a wheezing sound that can be heard on auscultation or even with the naked ear and
indicates respiratory obstruction.

DIF: Cognitive Level: Knowledge REF: p. 510 OBJ: Theory #1


TOP: Stridor KEY: Nursing Process Step: Assessment
MSC: NCLEX: Physiological Integrity: Physiological Adaptation

MULTIPLE RESPONSE

1. When a patient with a tracheostomy tube is taken care of at home by family, tracheostomy
care instructions from the nurse include: (Select all that apply.)
a. use sterile gloves during suctioning.
b. avoid going to crowded theaters and malls.
c. change catheters every 8 hours.
d. keep the home environment free of dust.
e. use bleach to clean suction equipment.
ANS: B, C, D, E
The patient should avoid crowded places to decrease the chance of respiratory infections; use
household bleach, hydrogen peroxide, or soap and water to clean equipment; change catheter
every 8 hours; and maintain the home environment free of air pollutants to decrease irritation
to airway passages.

DIF: Cognitive Level: Comprehension REF: p. 538 OBJ: Clinical Practice #3


TOP: Home Care of Tracheostomy Patient
KEY: Nursing Process Step: Implementation
MSC: NCLEX: Physiological Integrity: Basic Care and Comfort

2. The nurse is aware that changes occur in the respiratory system after the age of 70 that put the
older adult more at risk for respiratory problems. These changes include: (Select all that
apply.)
a. decreased oxygen saturation.
b. increased elasticity in thorax and respiratory tissues.

Downloaded by: SUCCEEDGRADES | abbieclin@gmail.com


Distribution
Downloaded by Dallen Mae D.ofKadir
this document is illegal
(kadir.dallenmae.d.bcsi@gmail.com)
lOMoARcPSD|35346190

Stuvia.com - The Marketplace to Buy and Sell your Study Material

c. incomplete expirations.
d. thinning of alveolar membrane.
e. impaired cilia.

ANS: A, C, E
After the age of 70, changes in the respiratory system that put the older adult at risk for
respiratory disorders are decreased oxygen saturation and elasticity of the thorax and
respiratory tissues, incomplete respirations, thickening of the alveolar membranes, impaired
cilia, and a lessened respiratory reserve.

DIF: Cognitive Level: Comprehension REF: p. 510 OBJ: Theory #1


TOP: Age-Related Changes KEY: Nursing Process Step: Planning
MSC: NCLEX: Physiological Integrity: Physiological Adaptation

3. The multiple causes for hypoxia include: (Select all that apply.)
a. extreme fright.
b. aspirated vomit.
c. pulmonary fibrosis.
d. hiccoughs.
e. high altitude.
ANS: B, C, E
Among the many causes of hypoxia are aspirated vomit, pulmonary fibrosis, and high altitude.

DIF: Cognitive Level: Comprehension REF: p. 510|Box 28-1


OBJ: Theory #2 TOP: Causes for hypoxia KEY: Nursing Process Step: N/A
MSC: NCLEX: N/A

COMPLETION

1. A sputum specimen is best obtained just after the patient or after a


treatment because this is when there is more mucus available or when it is easier to cough up.

ANS:
awakens; nebulizer

A sputum specimen is best obtained just after the patient awakens or after a nebulizer
treatment because this is when there is more mucus available or when it is easier to cough up.

DIF: Cognitive Level: Application REF: p. 518 OBJ: Clinical Practice #1


TOP: Specimen Collection KEY: Nursing Process Step: N/A
MSC: NCLEX: N/A

2. When obtaining a sputum specimen, the nurse should provide the patient with a sterile sputum
cup and instruct the patient to rinse her mouth with .

ANS:
water

When obtaining a sputum specimen, the nurse should provide the patient with a sterile sputum
cup and instruct the patient to rinse her mouth with water.

Downloaded by: SUCCEEDGRADES | abbieclin@gmail.com


Distribution
Downloaded by Dallen Mae D.ofKadir
this document is illegal
(kadir.dallenmae.d.bcsi@gmail.com)
lOMoARcPSD|35346190

Stuvia.com - The Marketplace to Buy and Sell your Study Material

DIF: Cognitive Level: Application REF: p. 518 OBJ: Clinical Practice #1


TOP: Specimen Collection KEY: Nursing Process Step: Implementation
MSC: NCLEX: Physiological Integrity: Basic Care and Comfort

3. The nurse explains that the rate of respiration is triggered when the medulla senses a change in
the level of ions in the blood.

ANS:
hydrogen

When there is an increase in hydrogen ions in the blood (pH), the medulla signals the spinal
nerves to increase and deepen respirations. A drop in the pH reverses the process causing a
slowing of the respirations.

DIF: Cognitive Level: Knowledge REF: p. 509 OBJ: Theory #1


TOP: pH Effect on Respiration KEY: Nursing Process Step: Implementation
MSC: NCLEX: Physiological Integrity: Physiological Adaptation

4. The nurse administering cardiopulmonary resuscitation (CPR) would administer chest


compressions at the rate of compressions/minute.

ANS:
100

The CPR guidelines require that there be 100 chest compressions/minute.

DIF: Cognitive Level: Knowledge REF: p. 515|Skill 28-3


OBJ: Theory #3 TOP: CPR KEY: Nursing Process Step: Implementation
MSC: NCLEX: Physiological Integrity: Reduction of Risk

Downloaded by: SUCCEEDGRADES | abbieclin@gmail.com


Distribution
Downloaded by Dallen Mae D.ofKadir
this document is illegal
(kadir.dallenmae.d.bcsi@gmail.com)
lOMoARcPSD|35346190

Stuvia.com - The Marketplace to Buy and Sell your Study Material

Chapter 29: Promoting Urinary Elimination


Williams: deWit's Fundamental Concepts and Skills for Nursing, 8th Edition

MULTIPLE CHOICE

1. The nurse caring for a severely dehydrated patient who has a Foley catheter in place
assesses the patient to confirm adequate urine perfusion by the urine output of:
a. 15 mL.
b. 30 mL.
c. 45 mL.
d. 60 mL.
ANS: B
There should be an average hourly urine output of 30 mL.

DIF: Cognitive Level: Knowledge REF: p. 543


OBJ: Clinical Practice #1 TOP: Urinary Output
KEY: Nursing Process Step: Assessment
MSC: NCLEX: Physiological Integrity: Basic Care and Comfort

2. To help reduce a patient‘s risk of recurrent cystitis, the nurse teaches the patient to:
a. eat citrus fruits to alkalinize the urine.
b. always wipe the perineal area from back to front.
c. take long, warm bubble baths.
d. wear cotton underwear and avoid nylon or constrictive clothing.
ANS: D
The patient should wear cotton underwear and avoid nylon and constrictive clothing that
worsens perineal moisture.

DIF: Cognitive Level: Comprehension REF: p. 544


OBJ: Clinical Practice #1 TOP: Prevention of Urinary Infections
KEY: Nursing Process Step: Implementation
MSC: NCLEX: Health Promotion and Maintenance: Prevention and Detection of Disease

3. The nurse is aware that in the older adult, a urinary infection may cause the patient to:
a. run an exceptionally high temperature.
b. have foul urine and diarrhea.
c. become disoriented and confused.
d. become irritable.
ANS: C
Urinary infections in the older adult patient may not be manifested by fever. There are subtle
changes in mental status.

DIF: Cognitive Level: Comprehension REF: p. 544


OBJ: Clinical Practice #1 TOP: Urinary Infection in the Older Adult
KEY: Nursing Process Step: Assessment
MSC: NCLEX: Physiological Integrity: Physiological Adaptation

Downloaded by: SUCCEEDGRADES | abbieclin@gmail.com


Distribution
Downloaded by Dallen Mae D.ofKadir
this document is illegal
(kadir.dallenmae.d.bcsi@gmail.com)
lOMoARcPSD|35346190

Stuvia.com - The Marketplace to Buy and Sell your Study Material

4. To prevent changes in the chemical characteristics of urine, a nurse sends a sample of fresh
urine to the laboratory for urinalysis within at least:
a. 1 to 2 minutes.
b. 3 to 5 minutes.
c. 5 to 10 minutes.
d. 20 to 30 minutes.
ANS: C
Urine that stands for 15 minutes or longer changes characteristics, and the urinalysis will no
longer be accurate.

DIF: Cognitive Level: Comprehension REF: p. 547


OBJ: Clinical Practice #1 TOP: Urinalysis
KEY: Nursing Process Step: Implementation
MSC: NCLEX: Physiological Integrity: Basic Care and Comfort

5. A patient has been ordered to have a 24-hour urine collection as part of a diagnostic workup.
The action taken to perform this procedure correctly is to:
a. continue the collection if the patient accidentally voids directly into the toilet.
b. obtain a container and put it in a warm water bath in the bathroom.
c. have the patient void at the beginning of the collection and throw it away.
d. have the patient void for the last time a few hours before the collection ends.
ANS: C
The patient‘s bladder should be empty when the test begins; for this reason, the urine
obtained at the start time is discarded and the urine collected should be stored on ice during
the 24-hour period.

DIF: Cognitive Level: Application REF: p. 546


OBJ: Clinical Practice #1 TOP: 24 Hour Urine Collection
KEY: Nursing Process Step: Implementation
MSC: NCLEX: Physiological Integrity: Basic Care and Comfort

6. A nurse would modify the urine collection technique when a urine sample is needed from an
infant by:
a. placing the infant on a bedpan after removing the diaper.
b. removing the diaper after the infant voids and send the diaper to the laboratory.
c. attaching a bag with adhesive backing to the skin surrounding the genitals.
d. applying a very small condom catheter.
ANS: C
A urine collection bag is attached to the skin by an adhesive backing and is placed so that it
surrounds the genitals; when sufficient urine is collected, the bag is removed and urine is put
into a specimen cup to send to the laboratory.

DIF: Cognitive Level: Application REF: p. 547


OBJ: Clinical Practice #1 TOP: Urinary Collection Bag
KEY: Nursing Process Step: Implementation
MSC: NCLEX: Physiological Integrity: Basic Care and Comfort

Downloaded by: SUCCEEDGRADES | abbieclin@gmail.com


Distribution
Downloaded by Dallen Mae D.ofKadir
this document is illegal
(kadir.dallenmae.d.bcsi@gmail.com)
lOMoARcPSD|35346190

Stuvia.com - The Marketplace to Buy and Sell your Study Material

7. The nurse is admitting a patient with suspected urolithiasis. An appropriate nursing


intervention in the care of such a patient would be to:
a. place a sieve over the commode.
b. obtain an order for indwelling urinary catheter.
c. place a graduated cylinder near the commode.
d. attach a urinary leg bag.
ANS: A
When a patient is suspected of having urolithiasis (a urinary stone), the urine is strained
through a fine sieve.

DIF: Cognitive Level: Application REF: p. 547


OBJ: Clinical Practice #1 TOP: Strained Urine Specimen
KEY: Nursing Process Step: Implementation
MSC: NCLEX: Physiological Integrity: Basic Care and Comfort

8. When the patient who has an order to be out of bed complains of feeling too weak to walk to
the bathroom, the nurse assists the patient with urination elimination by:
a. acquiring a walker so that the patient can go to the bathroom.
b. using a fracture bedpan and keep the patient flat.
c. obtaining a raised toilet seat.
d. placing a commode at the bedside.
ANS: D
A bedside commode allows the patient to get out of bed to void and does not tire the patient
who feels weak. Allowing a weak patient to ambulate unassisted puts the patient at risk for a
fall.

DIF: Cognitive Level: Application REF: p. 548


OBJ: Clinical Practice #4 TOP: Urinary Elimination
KEY: Nursing Process Step: Implementation
MSC: NCLEX: Physiological Integrity: Reduction of Risk

9. A nurse is observing a nursing assistant offering a bedpan to a patient. The nurse will
intervene if the nursing assistant:
a. closes the bedside curtain.
b. dons clean gloves.
c. keeps the head of the bed flat after placing the bedpan.
d. asks the patient to bend his knees and press down with his feet.
ANS: C
The head of the bed should be raised to 30 degrees after the bedpan is placed unless
specifically contraindicated.

DIF: Cognitive Level: Application REF: p. 551|Skill 29-1


OBJ: Clinical Practice #4 TOP: Assisting with Toileting
KEY: Nursing Process Step: Implementation
MSC: NCLEX: Physiological Integrity: Basic Care and Comfort

Downloaded by: SUCCEEDGRADES | abbieclin@gmail.com


Distribution
Downloaded by Dallen Mae D.ofKadir
this document is illegal
(kadir.dallenmae.d.bcsi@gmail.com)
lOMoARcPSD|35346190

Stuvia.com - The Marketplace to Buy and Sell your Study Material

10. The nurse can assist a patient who needs to void but cannot begin the urinary stream by:
a. running water in a nearby sink.
b. pouring cool water over the perineum.
c. inserting an indwelling catheter.
d. distracting the patient with conversation.
ANS: A
The sound of running water helps a patient to start voiding.

DIF: Cognitive Level: Application REF: p. 553 OBJ: Theory #3


TOP: Strategies to Assist with Voiding KEY: Nursing Process Step: Implementation
MSC: NCLEX: Physiological Integrity: Basic Care and Comfort

11. An adult male patient who cannot void has an order to have a urinary catheter inserted.
Which size catheter would be most appropriate to use?
a. 12 French
b. 16 French
c. 18 French
d. 22 French
ANS: C
The average sized urinary catheter used for an adult male is 18 to 20 French.

DIF: Cognitive Level: Knowledge REF: p. 553 OBJ: Theory #4


TOP: Indwelling Catheter KEY: Nursing Process Step: Planning
MSC: NCLEX: Physiological Integrity: Basic Care and Comfort

12. An older adult male patient needs to have a condom catheter applied. An appropriate
technique is to:
a. shave the perineal area before beginning.
b. apply povidone iodine to the penis before catheter application.
c. apply an adhesive strip in a circle around the base of the penis.
d. leave 1 to 2 inches between the tip of the penis and the drainage part of the catheter.
ANS: D
The catheter is placed so that 1 to 2 inches of space are left to allow for urine to drain away
from the penis.

DIF: Cognitive Level: Comprehension REF: p. 555|Skill 29-2


OBJ: Theory #6 TOP: Condom Catheter
KEY: Nursing Process Step: Implementation
MSC: NCLEX: Physiological Integrity: Basic Care and Comfort

13. A patient who underwent prostate surgery is admitted to the surgical unit with a catheter that
is used to provide continuous irrigation. The nurse recognizes this catheter is a(n):
a. Alcock.
b. Malecot.
c. Coudé catheter.
d. de Pezzer catheter.
ANS: A

Downloaded by: SUCCEEDGRADES | abbieclin@gmail.com


Distribution
Downloaded by Dallen Mae D.ofKadir
this document is illegal
(kadir.dallenmae.d.bcsi@gmail.com)
lOMoARcPSD|35346190

Stuvia.com - The Marketplace to Buy and Sell your Study Material

An Alcock catheter is used for bladder irrigation following prostate surgery.

DIF: Cognitive Level: Knowledge REF: p. 554 OBJ: Theory #5


TOP: Continuous Bladder Irrigation KEY: Nursing Process Step: Planning
MSC: NCLEX: Physiological Integrity: Basic Care and Comfort

14. A nurse is cleansing the perineal area of a female patient who is having a urinary catheter
inserted. The nurse should use the last povidone iodine–soaked cotton ball to cleanse
downward over the:
a. urinary meatus.
b. left labia.
c. right labia.
d. perirectal area.
ANS: A
The urinary meatus is cleansed after the labia on each side are cleansed.

DIF: Cognitive Level: Comprehension REF: p. 557|Skill 29-3


OBJ: Clinical Practice #5 TOP: Catheterizing the Female Patient
KEY: Nursing Process Step: Implementation
MSC: NCLEX: Physiological Integrity: Basic Care and Comfort

15. When attempting to catheterize a male patient, there is resistance to catheter insertion. The
nurse‘s initial intervention should be to:
a. withdraw the catheter and start over.
b. ask the patient to take a deep breath.
c. ask the patient to bear down and hold his breath.
d. ask that the patient lie on the right side.
ANS: B
If resistance is met, the catheter should be twisted, and the patient should be asked to take a
deep breath, which relaxes the urinary sphincter.

DIF: Cognitive Level: Application REF: p. 560|Skill 29-4


OBJ: Clinical Practice #5 TOP: Catheterizing the Male Patient
KEY: Nursing Process Step: Implementation
MSC: NCLEX: Physiological Integrity: Basic Care and Comfort

16. A patient has just had a urinary drainage catheter removed. The nurse plans to measure
intake and output for this patient for another:
a. 1 to 2 hours.
b. 4 to 6 hours.
c. 6 to 8 hours.
d. 12 to 24 hours.
ANS: D
Measuring intake and output for 12 to 24 hours after catheter removal allows time to note
whether the bladder is draining adequately.

DIF: Cognitive Level: Application REF: p. 563

Downloaded by: SUCCEEDGRADES | abbieclin@gmail.com


Distribution
Downloaded by Dallen Mae D.ofKadir
this document is illegal
(kadir.dallenmae.d.bcsi@gmail.com)
lOMoARcPSD|35346190

Stuvia.com - The Marketplace to Buy and Sell your Study Material

OBJ: Clinical Practice #1 TOP: Removal of Indwelling Catheter


KEY: Nursing Process Step: Implementation
MSC: NCLEX: Physiological Integrity: Basic Care and Comfort

17. A male patient who suffered a spinal cord injury is learning to perform self-urinary
catheterization before being discharged to home. The statement made by the patient that
indicates more instruction is needed is:
a. ―It is a sterile procedure.‖
b. ―The catheter should be pinched before it is withdrawn.‖
c. ―The penis is lifted to a 60- to 90-degree angle for catheter insertion.‖
d. ―The procedure is done sitting on the toilet.‖
ANS: A
Urinary catheterization in the home after spinal cord injury is usually a clean procedure
rather than a sterile one.

DIF: Cognitive Level: Analysis REF: p. 565 OBJ: Theory #4


TOP: Self Catheterization KEY: Nursing Process Step: Application
MSC: NCLEX: Physiological Integrity: Basic Care and Comfort

18. An adult patient has an order to have his urinary catheter irrigated with normal saline. The
nurse plans to draw up how much solution into the sterile irrigation syringe?
a. 1 to 20 mL
b. 20 to 30 mL
c. 30 to 40 mL
d. 50 to 60 mL
ANS: C
The appropriate amount of solution to draw into the syringe for irrigation is 30 to 40 mL in
an adult patient, which provides effective irrigation without risking over distention of the
bladder.

DIF: Cognitive Level: Application REF: p. 566|Skill 29-5


OBJ: Theory #4 TOP: Bladder Irrigation
KEY: Nursing Process Step: Implementation
MSC: NCLEX: Physiological Integrity: Basic Care and Comfort

19. A nurse irrigating a patient‘s indwelling urinary catheter should instill normal saline as
ordered, and then:
a. unclamp the tubing and lower the collection bag.
b. massage the patient‘s bladder.
c. ask the patient to take a deep breath and hold it.
d. keep the tubing clamped for 30 to 45 minutes.
ANS: A
Immediately after irrigating a urinary catheter, the tubing should be unclamped and the
collection bag lowered below the level of the bladder for proper drainage.

DIF: Cognitive Level: Application REF: p. 566|Skill 29-5


OBJ: Theory #4 TOP: Irrigation KEY: Nursing Process Step: Implementation

Downloaded by: SUCCEEDGRADES | abbieclin@gmail.com


Distribution
Downloaded by Dallen Mae D.ofKadir
this document is illegal
(kadir.dallenmae.d.bcsi@gmail.com)
lOMoARcPSD|35346190

Stuvia.com - The Marketplace to Buy and Sell your Study Material

MSC: NCLEX: Physiological Integrity: Basic Care and Comfort

20. A nurse is reinforcing instructions about Kegel exercises with a female patient. An
appropriate instruction is to:
a. do the exercises 12 times each day.
b. hold each muscle contraction for a count of 3 seconds.
c. tighten the abdominal muscles.
d. tighten the pelvic muscles.
ANS: D
Kegel exercises involve tightening the pelvic muscles to reduce the likelihood of urinary
incontinence.

DIF: Cognitive Level: Application REF: p. 569


OBJ: Clinical Practice #7 TOP: Kegel Exercises
KEY: Nursing Process Step: Implementation
MSC: NCLEX: Physiological Integrity: Basic Care and Comfort

21. A nurse is documenting the removal of a urinary drainage catheter from an assigned patient.
If the catheter is removed at 9:00 AM, the nurse recognizes that the patient is due to void by:
a. 11:00 AM.
b. 12 noon.
c. 5:00 PM.
d. 9:00 PM.
ANS: C
Documentation of removal of a urinary catheter should include the time the patient is due to
void, which is within 8 hours.

DIF: Cognitive Level: Comprehension REF: p. 570


OBJ: Clinical Practice #1
TOP: Voiding After Urinary Drainage Catheter Removal
KEY: Nursing Process Step: Planning
MSC: NCLEX: Physiological Integrity: Basic Care and Comfort

22. A patient with a history of cystitis had surgery 24 hours ago and is now unable to void. A
bladder scan indicates that he has approximately 400 mL of retained urine. The nurse
anticipates that the least invasive intervention the primary care provider will order would be:
a. inserting an indwelling Foley catheter.
b. monitoring intake and output.
c. obtaining a midstream specimen.
d. applying Credé maneuver to the bladder.
ANS: D
Credé maneuver is less invasive and may be used before invasive measures are taken. The
bladder is gently massaged from the top of the bladder and rocking the palm of the hand
steadily downward.

DIF: Cognitive Level: Application REF: p. 553


OBJ: Clinical Practice #4 TOP: Plan of Care

Downloaded by: SUCCEEDGRADES | abbieclin@gmail.com


Distribution
Downloaded by Dallen Mae D.ofKadir
this document is illegal
(kadir.dallenmae.d.bcsi@gmail.com)
lOMoARcPSD|35346190

Stuvia.com - The Marketplace to Buy and Sell your Study Material

KEY: Nursing Process Step: Implementation


MSC: NCLEX: Physiological Integrity: Basic Care and Comfort

23. A nurse is caring for a patient with prostate enlargement who has an indwelling catheter. As
the nurse is attaching a portion of the catheter to the patient‘s abdomen, the patient asks why
this is being done. The correct response is:
a. ―Taping the catheter to your abdomen will prevent pulling on the meatus.‖
b. ―The catheter can‘t be pulled out if it is taped to your abdomen.‖
c. ―Taping it in this way enhances the draining of your bladder.‖
d. ―This will prevent the Foley catheter from kinking.‖
ANS: A
When the catheter is taped to the abdomen, it prevents pulling on the meatus, thus
decreasing irritation.

DIF: Cognitive Level: Comprehension REF: p. 563|Box 29-3


OBJ: Clinical Practice #5 TOP: Catheter Care
KEY: Nursing Process Step: Implementation
MSC: NCLEX: Physiological Integrity: Basic Care and Comfort

24. A nurse instructing a female patient on obtaining a clean catch urine specimen should stress:
a. to spread the labia apart and clean the center area first.
b. to catch the middle portion of urine after voiding a small amount into the toilet.
c. to carefully collect the urine in the container as soon as the urine stream starts.
d. to fill the urine cup to the brim to ensure an adequate sample.
ANS: B
The procedure for a midstream or clean catch urine specimen is to void a small amount of
urine into the toilet and to catch the middle portion of urine by moving the container into the
stream.

DIF: Cognitive Level: Comprehension REF: p. 546


OBJ: Clinical Practice #2 TOP: Clean Catch Urine Specimen
KEY: Nursing Process Step: Implementation
MSC: NCLEX: Physiological Integrity: Basic Care and Comfort

25. A patient is being assessed for a possible urinary tract infection in the outpatient clinic.
Before sending a urinalysis specimen to the laboratory, the nurse collects a small amount of
urine in order to perform a dipstick test in order to detect:
a. protein.
b. glucose.
c. leukocytes.
d. ketones.
ANS: C
A dipstick test is performed to test for different components and, in this case, for white
blood cells or leukocytes, which indicate an infection. The accurate timing of the reading is
essential to the accuracy of the result.

DIF: Cognitive Level: Analysis REF: p. 546

Downloaded by: SUCCEEDGRADES | abbieclin@gmail.com


Distribution
Downloaded by Dallen Mae D.ofKadir
this document is illegal
(kadir.dallenmae.d.bcsi@gmail.com)
lOMoARcPSD|35346190

Stuvia.com - The Marketplace to Buy and Sell your Study Material

OBJ: Clinical Practice #2 TOP: Dipstick Test


KEY: Nursing Process Step: Assessment
MSC: NCLEX: Physiological Integrity: Basic Care and Comfort

26. Nurses in a long-term care facility are developing a prevention program to eliminate catheter
acquired infections (CAUTI). The rationale for this program includes which of the
following?
a. Medicaid will no longer reimburse for this complication.
b. CAUTIs are considered an indicator of adequate care.
c. CAUTIs result in 45% of hospital-acquired infections every year.
d. Nursing interventions have been proven to have little or no effect on the number of
urinary infections.
ANS: A
In performing catheterization and providing daily care, nurses should be aware that catheter
related UTI (CAUTI) results in 30% to 40% of hospital-acquired infections every year. In
addition, Medicare and Medicaid no longer reimburse for this complication, since CAUTI is
viewed as an indicator of poor-care nursing interventions been proven to reduce the number
of urinary tract infections.

DIF: Cognitive Level: Comprehension REF: p. 563 OBJ: Clinical #10


TOP: Preventing Catheter-Related Urinary Tract Infections
KEY: Nursing Process Step: Planning
MSC: NCLEX: QSEN: Evidence-based Practice

MULTIPLE RESPONSE

1. A nurse instructing a patient about how to prevent recurrent cystitis would include the need
to: (Select all that apply.)
a. increasing fluid intake to 2500 to 3000 mL/day.
b. consuming more citrus fruits and juice.
c. wearing cotton underwear.
d. wiping the rectal area from front to back after a bowel movement.
e. avoiding sitting in a wet bathing suit for extended periods.
f. emptying the bladder every 2 to 3 hours.
ANS: A, C, D, E, F
Cystitis and other urinary tract infections may be avoided by increasing fluid intake to 2500
to 3000 mL/day, avoiding citrus fruits and juice because they cause alkaline urine (bacteria
grow more readily in alkaline urine); always wiping the rectal area from front to back after a
bowel movement; wearing cotton underwear; not sitting around in a wet bathing suit for
extended periods; and emptying the bladder every 2 to 3 hours to prevent stasis and
potential for bacteria to multiply if present.

DIF: Cognitive Level: Application REF: p. 541 OBJ: Theory #3


TOP: Infection Prevention KEY: Nursing Process Step: Implementation
MSC: NCLEX: Physiological Integrity: Basic Care and Comfort

Downloaded by: SUCCEEDGRADES | abbieclin@gmail.com


Distribution
Downloaded by Dallen Mae D.ofKadir
this document is illegal
(kadir.dallenmae.d.bcsi@gmail.com)
lOMoARcPSD|35346190

Stuvia.com - The Marketplace to Buy and Sell your Study Material

2. A patient‘s urinalysis results are available. Which of the following are considered normal
characteristics? (Select all that apply.)
a. Straw colored
b. Specific gravity (SpG), 1.015
c. pH, 6.0
d. RBCs, more than 1 per high power field
e. Cloudy appearance
ANS: A, B, C
Urine is normally straw colored; SpG and pH are within normal range; RBCs are abnormal
findings; and cloudy urine may indicate a large amount of protein.

DIF: Cognitive Level: Knowledge REF: p. 543 OBJ: Theory #2


TOP: Urinalysis KEY: Nursing Process Step: Assessment
MSC: NCLEX: Physiological Integrity: Basic Care and Comfort

COMPLETION

1. The nurse should provide enough hydration for the patient so that the patient can void at
least every hours.

ANS:
8

Each patient should void at least every 8 hours unless there is a catheter in place.

DIF: Cognitive Level: Knowledge REF: p. 545


OBJ: Clinical Practice #1 TOP: Frequency of Urination
KEY: Nursing Process Step: Implementation
MSC: NCLEX: Physiological Integrity: Basic Care and Comfort

Downloaded by: SUCCEEDGRADES | abbieclin@gmail.com


Distribution
Downloaded by Dallen Mae D.ofKadir
this document is illegal
(kadir.dallenmae.d.bcsi@gmail.com)
lOMoARcPSD|35346190

Stuvia.com - The Marketplace to Buy and Sell your Study Material

Chapter 30: Promoting Bowel Elimination


Williams: deWit's Fundamental Concepts and Skills for Nursing, 8th Edition

MULTIPLE CHOICE

1. The nurse has assessed that a patient‘s stool has changed from brown to dark black and
sticky. The nurse suspects:
a. blockage of the bile duct.
b. blockage of the pancreatic duct.
c. recent excessive intake of milk products.
d. presence of occult blood.
ANS: D
Occult or old blood is suspected when stool changes from a normal brown appearance to a
dark black color with a sticky appearance.

DIF: Cognitive Level: Analysis REF: p. 574 OBJ: Theory #2


TOP: Stool Characteristics KEY: Nursing Process Step: Assessment
MSC: NCLEX: Physiological Integrity: Basic Care and Comfort

2. The nurse has documented that a patient has had two episodes of steatorrhea, which means
that the character of the stool is:
a. hard and clay colored.
b. frothy and foul smelling.
c. very liquid and streaked with blood.
d. soft and filled with mucus.
ANS: B
Steatorrhea is defined as stools with abnormally high fat content that are usually frothy, foul
smelling, and float on water.

DIF: Cognitive Level: Comprehension REF: p. 574 OBJ: Theory #2


TOP: Terminology KEY: Nursing Process Step: Assessment
MSC: NCLEX: Physiological Integrity: Basic Care and Comfort

3. The nurse should plan interventions to combat constipation in a patient:


a. being treated for diabetes mellitus.
b. who has a routine order for Metamucil.
c. who just completed barium studies of the bowel.
d. with orders to ambulate with assistance.
ANS: C
A patient who is undergoing barium radiograph studies is more prone to constipation than
are the other patients.

DIF: Cognitive Level: Application REF: p. 575 OBJ: Theory #3


TOP: Abnormal Characteristics of Stool KEY: Nursing Process Step: Planning
MSC: NCLEX: Physiological Integrity: Basic Care and Comfort

Downloaded by: SUCCEEDGRADES | abbieclin@gmail.com


Distribution
Downloaded by Dallen Mae D.ofKadir
this document is illegal
(kadir.dallenmae.d.bcsi@gmail.com)
lOMoARcPSD|35346190

Stuvia.com - The Marketplace to Buy and Sell your Study Material

4. An older adult patient who routinely takes the bulk forming laxative psyllium (Metamucil)
is counseled by the home health nurse that in order to prevent constipation and possible
fecal impaction, this patient should be sure to take:
a. extra vitamin C.
b. a fat-soluble vitamin.
c. the medication with a large amount of fluid.
d. an over the counter antacid.
ANS: C
A large amount of fluid should be taken to prevent constipation and fecal impaction when
using a product with psyllium.

DIF: Cognitive Level: Comprehension REF: p. 576


OBJ: Clinical Practice #1 TOP: Abnormal Characteristics of Stool
KEY: Nursing Process Step: Implementation
MSC: NCLEX: Physiological Integrity: Basic Care and Comfort

5. A patient calls the nurse at the health clinic and reports that since his trip to Mexico, he has
been experiencing diarrhea. The nurse suggests he try the antidiarrheal drug:
a. docusate sodium (Colace).
b. loperamide (Imodium).
c. polycarbophil (FiberCon).
d. senna (Senokot).
ANS: B
Loperamide (Imodium) is an antidiarrheal; the rest are used to prevent or treat constipation.

DIF: Cognitive Level: Comprehension REF: p. 576 OBJ: Theory #1


TOP: Medication for Diarrhea KEY: Nursing Process Step: Implementation
MSC: NCLEX: Physiological Integrity: Pharmacological Therapies

6. An older adult resident in a long-term care facility has experienced constant diarrhea for 3
days and is now exhibiting signs and symptoms of dehydration. The nurse initiates an
intervention to offer small amounts of frequently.
a. a cola beverage
b. ginger ale
c. Gatorade
d. Kool-Aid
ANS: C
The patient may be developing electrolyte imbalance because of the diarrhea, so the best
fluid source to offer is Gatorade, which has sodium and potassium. This should be offered 1
to 2 ounces at a time.

DIF: Cognitive Level: Application REF: p. 576


OBJ: Clinical Practice #1 TOP: Abnormal Characteristics of Stool
KEY: Nursing Process Step: Implementation
MSC: NCLEX: Physiological Integrity: Basic Care and Comfort

Downloaded by: SUCCEEDGRADES | abbieclin@gmail.com


Distribution
Downloaded by Dallen Mae D.ofKadir
this document is illegal
(kadir.dallenmae.d.bcsi@gmail.com)
lOMoARcPSD|35346190

Stuvia.com - The Marketplace to Buy and Sell your Study Material

7. A patient who has started antibiotic therapy is having diarrhea as a side effect of the
medication. The nurse should encourage the patient to eat:
a. yogurt.
b. raisins.
c. gelatin fruit flavored dessert (eg, Jell O).
d. poultry.
ANS: A
Diarrhea results from the loss of normal intestinal bacteria that are also killed during
treatment with antibiotics used to treat a different infection; eating yogurt or drinking
buttermilk can help restore normal intestinal flora.

DIF: Cognitive Level: Comprehension REF: p. 579


OBJ: Clinical Practice #1 TOP: Abnormal Characteristics of Stool
KEY: Nursing Process Step: Implementation
MSC: NCLEX: Physiological Integrity: Pharmacological Therapies

8. The nurse caring for a patient with lactose intolerance would anticipate the need to offer
interventions for:
a. diarrhea.
b. steatorrhea.
c. constipation.
d. hemorrhoid discomfort.
ANS: A
Lactose intolerance is the name for the condition in which diarrhea occurs after consuming
milk products.

DIF: Cognitive Level: Knowledge REF: p. 577 OBJ: Theory #2


TOP: Abnormal Characteristics of Stool KEY: Nursing Process Step: Assessment
MSC: NCLEX: Physiological Integrity: Basic Care and Comfort

9. A nurse has performed abdominal assessments on four patients. After reviewing the
findings, the nurse is least concerned about problems with bowel elimination for the patient
with:
a. abdomen nondistended, firm, with hypoactive bowel sounds in all four quadrants.
b. abdomen nondistended, soft, with active bowel sounds in all four quadrants.
c. abdomen distended, firm, with hypoactive bowel sounds in all four quadrants.
d. abdomen distended, soft, with hyperactive bowel sounds in all four quadrants.
ANS: B
Normal abdominal assessment data are an abdomen that is soft and nondistended and that
has active bowel sounds in all four quadrants.

DIF: Cognitive Level: Comprehension REF: p. 577


OBJ: Clinical Practice #3 TOP: Assessment: Bowels
KEY: Nursing Process Step: Assessment
MSC: NCLEX: Physiological Integrity: Basic Care and Comfort

Downloaded by: SUCCEEDGRADES | abbieclin@gmail.com


Distribution
Downloaded by Dallen Mae D.ofKadir
this document is illegal
(kadir.dallenmae.d.bcsi@gmail.com)
lOMoARcPSD|35346190

Stuvia.com - The Marketplace to Buy and Sell your Study Material

10. A nurse is monitoring bowel elimination of a patient who has a history of constipation. The
nurse implements measures to assist with bowel elimination if the patient has not had a
bowel movement within how many days?
a. 5
b. 3
c. 2
d. 1
ANS: B
If bowel evacuation has not occurred within 3 days, measures should be taken to assist the
patient.

DIF: Cognitive Level: Comprehension REF: p. 579 OBJ: Theory #2


TOP: Abnormal Characteristics of Stool KEY: Nursing Process Step: Assessment
MSC: NCLEX: Physiological Integrity: Basic Care and Comfort

11. A patient has just completed a series of upper gastrointestinal tract radiographs that involved
the use of barium as a contrast agent. Which measure will this patient need to help excrete
the barium?
a. Diuretics and fluid restriction to 1.5 L
b. Diuretics and fluid intake increased to 3.5 L
c. Laxatives and fluid restriction to 1.5 L
d. Laxatives and fluid intake increased to 3.5 L
ANS: D
To get rid of the barium, the patient‘s fluid intake should be increased to 3.5 L, or 3500 mL,
and the patient should be given a laxative.

DIF: Cognitive Level: Application REF: p. 579


OBJ: Clinical Practice #1 TOP: Promoting Regular Bowel Elimination
KEY: Nursing Process Step: Implementation: Pharmacological Therapies
MSC: NCLEX: Physiological Integrity

12. An ambulatory clinic patient telephones to report diarrhea and to ask for advice on
medication to manage it. The best response by the nurse is, ―Do not use antidiarrheal
medication for longer than:
a. 24 hours without calling back for an appointment.‖
b. 48 hours without calling back for an appointment.‖
c. 72 hours without calling back for an appointment.‖
d. 96 hours without calling back for an appointment.‖
ANS: B
Antidiarrheal medication should not be continued for more than 48 hours without calling a
primary care provider.

DIF: Cognitive Level: Comprehension REF: p. 579


OBJ: Clinical Practice #1 TOP: Promoting Regular Bowel Elimination
KEY: Nursing Process Step: Implementation
MSC: NCLEX: Physiological Integrity: Pharmacological Therapies

Downloaded by: SUCCEEDGRADES | abbieclin@gmail.com


Distribution
Downloaded by Dallen Mae D.ofKadir
this document is illegal
(kadir.dallenmae.d.bcsi@gmail.com)
lOMoARcPSD|35346190

Stuvia.com - The Marketplace to Buy and Sell your Study Material

13. There is an order to administer a cleansing enema to an adult patient before bowel surgery.
The nurse will fill the enema bag with how many milliliters of fluid for this procedure?
a. 500 to 1000 mL
b. 300 to 500 mL
c. 200 to 300 mL
d. 50 to 150 mL
ANS: A
The volume of the cleansing enema depends on the age of the patient—for adults, it is
between 500 and 1000 mL.

DIF: Cognitive Level: Application REF: p. 580


OBJ: Clinical Practice #5 TOP: Enemas
KEY: Nursing Process Step: Implementation
MSC: NCLEX: Physiological Integrity: Basic Care and Comfort

14. A patient who is badly constipated has just received an oil retention enema. The nurse
encourages this patient to try to hold the enema for at least how long before trying to have a
bowel movement?
a. 10 minutes
b. 15 minutes
c. 20 minutes
d. 40 minutes
ANS: C
The oil retention enema should be retained for 20 minutes.

DIF: Cognitive Level: Comprehension REF: p. 580


OBJ: Clinical Practice #5 TOP: Retention Enemas
KEY: Nursing Process Step: Implementation
MSC: NCLEX: Physiological Integrity: Pharmacological Therapies

15. A nurse is preparing a cleansing enema for an adult patient who is constipated and has not
responded to laxative use. Before giving the enema, the nurse should:
a. cool the solution to 70° F.
b. warm the solution in the microwave.
c. keep the solution at room temperature.
d. warm the solution to 105° F.
ANS: D
Solution that is too cool cannot be retained, and solution that is too hot may cause injury to
rectal tissues; it is best to warm it to 105° F.

DIF: Cognitive Level: Application REF: p. 581


OBJ: Clinical Practice #5 TOP: Cleansing Enemas
KEY: Nursing Process Step: Implementation
MSC: NCLEX: Physiological Integrity: Basic Care and Comfort

16. A patient scheduled for bowel surgery has an order to receive enemas until clear. The nurse
is aware that no more than three enemas should be given because:

Downloaded by: SUCCEEDGRADES | abbieclin@gmail.com


Distribution
Downloaded by Dallen Mae D.ofKadir
this document is illegal
(kadir.dallenmae.d.bcsi@gmail.com)
lOMoARcPSD|35346190

Stuvia.com - The Marketplace to Buy and Sell your Study Material

a. repeated enemas may cause more flatus.


b. the patient may develop an irritated rectum.
c. repeated enemas may cause electrolyte imbalance.
d. the patient may develop severe diarrhea.
ANS: C
No more than three large volume enemas are given because of possible electrolyte
depletion.

DIF: Cognitive Level: Comprehension REF: p. 581


OBJ: Clinical Practice #5 TOP: Enemas
KEY: Nursing Process Step: Implementation
MSC: NCLEX: Physiological Integrity: Reduction of Risk

17. A nurse is digitally removing a fecal impaction from a patient. The nurse should stop the
procedure immediately and take corrective action if the patient‘s:
a. blood pressure increases from 110/84 to 118/88 mm Hg.
b. pulse rate decreases from 78 to 52 beats/min.
c. respiratory rate increases from 16 to 24 breaths/min.
d. temperature increases from 98.8° F to 99.0° F.
ANS: B
Stimulation of the sphincter may cause a vagal response as evidenced by bradycardia.

DIF: Cognitive Level: Application REF: p. 584


OBJ: Clinical Practice #1 TOP: Fecal Impaction
KEY: Nursing Process Step: Assessment
MSC: NCLEX: Physiological Integrity: Reduction of Risk

18. A nurse is reinforcing education with a patient who will begin a bowel training program. An
intervention this program does not include is:
a. regularly scheduled time for toileting.
b. fluid intake of at least 1500 mL daily.
c. use of a suppository.
d. use of an enema.
ANS: D
Enemas and stronger laxatives are not considered a part of the program.

DIF: Cognitive Level: Comprehension REF: p. 584 OBJ: Theory #2


TOP: Bowel Training KEY: Nursing Process Step: Implementation
MSC: NCLEX: Physiological Integrity: Basic Care and Comfort

19. A nurse is assisting a patient with a new continent ileostomy to catheterize the internal
reservoir to drain the ileostomy. When the catheter meets resistance from the internal valve,
the nurse should:
a. have the patient take a deep breath and apply gentle pressure over the area.
b. withdraw the catheter and start again with a new one.
c. ask the patient to bear down and hold her breath.
d. coat the opening with petroleum jelly or a water soluble lubricant.

Downloaded by: SUCCEEDGRADES | abbieclin@gmail.com


Distribution
Downloaded by Dallen Mae D.ofKadir
this document is illegal
(kadir.dallenmae.d.bcsi@gmail.com)
lOMoARcPSD|35346190

Stuvia.com - The Marketplace to Buy and Sell your Study Material

ANS: A
For some patients the taking of a deep breath relaxes muscles and allows passage of the
catheter.

DIF: Cognitive Level: Application REF: p. 586


OBJ: Clinical Practice #7 TOP: Ileostomy
KEY: Nursing Process Step: Implementation
MSC: NCLEX: Physiological Integrity

20. A patient with a new colostomy should have the hole in the faceplate cut to allow
inch around the stoma.
a. 1 1/4
b. 1
c. 1/2
d. 1/4
ANS: D
The faceplate should allow 1/4 inch around the colostomy stoma.

DIF: Cognitive Level: Knowledge REF: p. 589


OBJ: Clinical Practice #8 TOP: Colostomy Faceplate
KEY: Nursing Process Step: Planning
MSC: NCLEX: Physiological Integrity: Basic Care and Comfort

21. A nurse is caring for a patient who had bowel surgery 3 days ago and is now beginning to
have a well-functioning ostomy. The ostomy drainage bag should be emptied whenever it is:
a. one fourth full.
b. one half full.
c. three fourths full.
d. full.
ANS: B
The ostomy bag should be changed when it is one third to one half full so that the weight of
the bag will not detach it.

DIF: Cognitive Level: Knowledge REF: p. 588


OBJ: Clinical Practice #8 TOP: Ostomy Bag
KEY: Nursing Process Step: Planning
MSC: NCLEX: Physiological Integrity: Basic Care and Comfort

22. A patient with a colostomy asks about foods that can be eaten that will reduce odor in the
ostomy drainage bag. The most informative response by the nurse is to say that ostomy odor
can be decreased with the intake of:
a. buttermilk.
b. eggs.
c. cucumbers.
d. beans.
ANS: A
Buttermilk is among the suggested foods that decrease ostomy bag odor.

Downloaded by: SUCCEEDGRADES | abbieclin@gmail.com


Distribution
Downloaded by Dallen Mae D.ofKadir
this document is illegal
(kadir.dallenmae.d.bcsi@gmail.com)
lOMoARcPSD|35346190

Stuvia.com - The Marketplace to Buy and Sell your Study Material

DIF: Cognitive Level: Comprehension REF: p. 587


OBJ: Clinical Practice #8 TOP: Ostomy Bag
KEY: Nursing Process Step: Planning
MSC: NCLEX: Physiological Integrity: Basic Care and Comfort

23. The nurse is caring for an anxious patient who is scheduled for surgery for colostomy
placement. While the nurse is talking to the patient, the patient states, ―I am so scared.‖ The
nurse‘s most supportive response would be:
a. ―Surgeries like yours are very safe.‖
b. ―What about your colostomy scares you?‖
c. ―Why are you scared?‖
d. ―Sounds like someone has been telling you horror stories.‖
ANS: B
The nurse needs to address the patient‘s anxiety and fear first by use of open-ended
questioning, because the patient might be focused on a variety of things, including poor
body image or the prospect of death. Asking a ―Why‖ question is not therapeutic and makes
the patient defensive.

DIF: Cognitive Level: Application REF: p. 585 OBJ: Theory #5


TOP: Preoperative Colostomy KEY: Nursing Process Step: Implementation
MSC: NCLEX: Psychosocial Integrity: Coping and Adaptation

24. The nurse reminds the patient that digestion of food is a complex process with much of the
food breaking down in intestines. The small intestine functions to:
a. reabsorb sodium and chlorides.
b. propel waste material toward the anus.
c. absorb food substances from the bloodstream.
d. return water from the waste material to the bloodstream.
ANS: C
The small intestine processes the chyme into a more liquid state and absorbs food
substances into the bloodstream. All other listed functions are those of the large intestine.

DIF: Cognitive Level: Knowledge REF: p. 573 OBJ: Theory #5


TOP: Intestinal Digestion KEY: Nursing Process Step: Implementation
MSC: NCLEX: Physiological Integrity: Physiological Adaptation

25. The nurse caring for a patient who had a colostomy 2 days ago assesses slight bleeding
around the stoma when the area is cleansed, colostomy bag filled with gas, pale stoma, and a
reddened area under the adhesive of the appliance. The assessment that should be reported
immediately is the assessment pertaining to the:
a. skin irritation.
b. bleeding around the stoma.
c. amount of gas in the bag.
d. pale stoma.
ANS: D

Downloaded by: SUCCEEDGRADES | abbieclin@gmail.com


Distribution
Downloaded by Dallen Mae D.ofKadir
this document is illegal
(kadir.dallenmae.d.bcsi@gmail.com)
lOMoARcPSD|35346190

Stuvia.com - The Marketplace to Buy and Sell your Study Material

The pale stoma indicates a compromised blood supply and should be reported immediately
to the primary care provider.

DIF: Cognitive Level: Analysis (because assessment information is being analyzed to


determine appropriate interventions) REF: p. 587
OBJ: Clinical Practice #8 TOP: Colostomy Stoma
KEY: Nursing Process Step: Implementation
MSC: NCLEX: Safe, Effective Care Environment: Safety and Early Detection of Disease

26. The patient asks the nurse how an ileostomy differs from a colostomy. The most informative
response by the nurse would be that:
a. an ileostomy is performed to remove stool from the colon, whereas a colostomy is
the removal of lower portions of bowel, diverting intestinal contents.
b. an ileostomy has effluent that is more formed, whereas a colostomy has effluent
that is liquid.
c. a colostomy is an opening into the colon, whereas an ileostomy is an opening at the
ileum.
d. an ileostomy requires irrigating, whereas a colostomy requires catheterizing.
ANS: C
The colostomy is an opening into the colon, with formed effluent requiring irrigation,
whereas the ileostomy is an opening in the ileum, with liquid effluent requiring
catheterizing.

DIF: Cognitive Level: Comprehension REF: p. 587 OBJ: Theory #7


TOP: Intestinal Diversions KEY: Nursing Process Step: Implementation
MSC: NCLEX: Physiological Integrity: Basic Care and Comfort

27. The patient with the new colostomy is concerned about how to control diarrhea of the
effluent. The nurse suggests that diarrhea can be controlled by the intake of:
a. cheese.
b. apple juice.
c. raw vegetables.
d. beams.
ANS: A
Cheese can control or decrease the incidence of diarrhea in a colostomy.

DIF: Cognitive Level: Comprehension REF: p. 587


OBJ: Clinical Practice #1 TOP: Control of Diarrhea
KEY: Nursing Process Step: Implementation
MSC: NCLEX: Physiological Integrity: Basic Care and Comfort

MULTIPLE RESPONSE

1. The nurse instructs the patient who has had an ileostomy to modify the diet to include:
(Select all that apply.)
a. increase the protein intake.
b. choose foods that are high in calories.

Downloaded by: SUCCEEDGRADES | abbieclin@gmail.com


Distribution
Downloaded by Dallen Mae D.ofKadir
this document is illegal
(kadir.dallenmae.d.bcsi@gmail.com)
lOMoARcPSD|35346190

Stuvia.com - The Marketplace to Buy and Sell your Study Material

c. select foods that have a milk base.


d. eat raw vegetables and fruits.
e. include whole grain products in diet daily.
ANS: A, B
Patients with ileostomies have lost their lower bowel and need food sources that are high in
calories and vitamins. Water intake should be up to 10 cups a day to make up for the water
reabsorption usually done in the colon. High residue foods such as whole grain products and
raw vegetables and fruits can irritate the intestine.

DIF: Cognitive Level: Comprehension REF: p. 587 OBJ: Theory #7


TOP: Food Appropriate for Ileostomies KEY: Nursing Process Step: Implementation
MSC: NCLEX: Health Promotion and Maintenance: Prevention and Early Detection of
Disease

2. The nurse points out that age-related changes in the intestinal tract are relatively
insignificant. The changes include: (Select all that apply.)
a. atrophy of the villi in the small intestine.
b. increased incidence of hemorrhoids.
c. decreased absorption of fats and vitamin B12.
d. creation of excessive flatus.
e. decreased motility in the large intestine.
ANS: A, C, D
With age there is a decrease in the villi in the small intestine that decreases the absorption of
fats and vitamin B12. Motility frequently decreases in the large intestine.

DIF: Cognitive Level: Comprehension REF: p. 574 OBJ: Theory #1


TOP: Age-Related Changes in the Intestinal Tract
KEY: Nursing Process Step: Implementation
MSC: NCLEX: Physiological Integrity: Basic Care and Comfort

3. The nurse instructs a patient with a new colostomy against eating food that may cause an
obstruction. These foods include: (Select all that apply.)
a. spicy foods.
b. whole kernel corn.
c. cucumbers.
d. tomatoes.
e. shrimp.
ANS: B, D, E
Whole kernel corn, tomatoes, and shrimp are among the food that can cause an obstruction
in a colostomy.

DIF: Cognitive Level: Comprehension REF: p. 587


OBJ: Clinical Practice #4 TOP: Obstructive Foods
KEY: Nursing Process Step: Implementation
MSC: NCLEX: Health Promotion and Maintenance: Prevention and Early Detection of
Disease

Downloaded by: SUCCEEDGRADES | abbieclin@gmail.com


Distribution
Downloaded by Dallen Mae D.ofKadir
this document is illegal
(kadir.dallenmae.d.bcsi@gmail.com)
lOMoARcPSD|35346190

Stuvia.com - The Marketplace to Buy and Sell your Study Material

COMPLETION

1. The gastrocolic reflex initiates .

ANS:
peristalsis

Peristalsis is initiated by the gastrocolic reflex, which creates the urge to defecate.

DIF: Cognitive Level: Knowledge REF: p. 573 OBJ: Theory #1


TOP: Bowel Elimination KEY: Nursing Process Step: N/A
MSC: NCLEX: N/A

2. The nurse assesses a pale, light gray stool and recognizes that the cause of this abnormal
color is due to an obstruction in the duct.

ANS:
bile

An obstruction in the bile ducts prevents bile salts from entering the bile. Bile salts give the
feces its characteristic brown color.

DIF: Cognitive Level: Comprehension REF: p. 574 OBJ: Theory #2


TOP: Bowel Elimination KEY: Nursing Process Step: Assessment
MSC: NCLEX: Physiological Integrity: Physiological Adaptation

3. The nurse reminds a group of older adults that a colonoscopy is recommended every
year(s) after the age of 50.

ANS:
10

Healthy People 2020 recommends a colonoscopy every 10 years in persons over the age of
50.

DIF: Cognitive Level: Knowledge REF: p. 574


OBJ: Clinical Practice #1 TOP: Colonoscopies
KEY: Nursing Process Step: N/A MSC: NCLEX: N/A

Downloaded by: SUCCEEDGRADES | abbieclin@gmail.com


Distribution
Downloaded by Dallen Mae D.ofKadir
this document is illegal
(kadir.dallenmae.d.bcsi@gmail.com)
lOMoARcPSD|35346190

Stuvia.com - The Marketplace to Buy and Sell your Study Material

Chapter 31: Pain, Comfort, and Sleep


Williams: deWit's Fundamental Concepts and Skills for Nursing, 8th Edition

MULTIPLE CHOICE

1. The nurse assesses a patient‘s sleep for NREM (non–rapid eye movement) sleep because
this sleep has the characteristics of:
a. daytime activity.
b. offering the most rest.
c. irregular respirations.
d. increased heart rate.
ANS: B
NREM sleep is the stage of sleep when the body receives the most rest.

DIF: Cognitive Level: Comprehension REF: p. 609 OBJ: Theory #5


TOP: NREM Sleep KEY: Nursing Process Step: Assessment
MSC: NCLEX: Physiological Integrity: Basic Care and Comfort

2. A patient experiencing discomfort because of severe arthritis would be described as having


pain.
a. acute
b. chronic
c. phantom
d. episodic
ANS: B
Chronic pain is pain in which symptoms are controlled, but there is no cure, such as in
arthritis.

DIF: Cognitive Level: Comprehension REF: p. 594


OBJ: Clinical Practice #2 TOP: Types of Pain
KEY: Nursing Process Step: Assessment
MSC: NCLEX: Physiological Integrity: Basic Care and Comfort

3. A patient is beginning treatment for pain with a transcutaneous electrical nerve stimulator
(TENS) unit. The nurse will initially:
a. apply conductive jelly to uncoated electrodes.
b. turn the unit on before attaching it to the patient.
c. place electrodes on all four extremities.
d. adjust the settings below the level at which a tingling sensation is felt.
ANS: A
Conductive jelly is applied to electrodes that are not precoated to assure conductivity.

DIF: Cognitive Level: Application REF: p. 601


OBJ: Clinical Practice #4 TOP: TENS
KEY: Nursing Process Step: Implementation
MSC: NCLEX: Physiological Integrity: Basic Care and Comfort

Downloaded by: SUCCEEDGRADES | abbieclin@gmail.com


Distribution
Downloaded by Dallen Mae D.ofKadir
this document is illegal
(kadir.dallenmae.d.bcsi@gmail.com)
lOMoARcPSD|35346190

Stuvia.com - The Marketplace to Buy and Sell your Study Material

4. The patient with a recent abdominal incision has an abdominal binder applied. The nurse
explains that this appliance helps reduce pain by:
a. increasing warmth to the incision site.
b. keeping sutures and staples in place.
c. supporting surface and internal tissues.
d. adding back support to enhance early ambulation.
ANS: C
Binders are cloths wrapped around a limb or body part that have an incision and are
effective in reducing pain associated with strains, sprains, and surgical incisions by
providing support to the surface of the area as well as the internal tissues.

DIF: Cognitive Level: Comprehension REF: p. 601


OBJ: Clinical Practice #4 TOP: Pain Control
KEY: Nursing Process Step: Assessment
MSC: NCLEX: Physiological Integrity: Basic Care and Comfort

5. A nurse evaluating the effectiveness of an Aquathermia K pad for the leg is aware that the
patient who has the least risk for burn injury from this device would be the patient with:
a. impaired peripheral circulation.
b. severely sprained ankle.
c. decreased level of consciousness from a stroke.
d. neuritis secondary to diabetes.
ANS: B
Patients with poor circulation, impaired movement or feeling, or decreased level of
consciousness are more at risk for injury from burns when applications of heat are used.

DIF: Cognitive Level: Analysis REF: p. 602


OBJ: Clinical Practice #4 TOP: Heat Application
KEY: Nursing Process Step: Planning
MSC: NCLEX: Safe, Effective Care Environment: Safety and Infection Control

6. A nurse caring for a patient who suffered a severe sprain and has an order for a cold pack
application to the injured area would prevent patient injury by:
a. using heavy pressure on the cold pack for greater effectiveness.
b. preparing to apply heat instead if cold is not effective.
c. leaving the pack in place for over 30 minutes at a time.
d. placing a towel between the pack and the skin.
ANS: D
A protective towel or pad should be placed between the pack and the skin to avoid cold
injury.

DIF: Cognitive Level: Application REF: p. 602


OBJ: Clinical Practice #4 TOP: Cold Application
KEY: Nursing Process Step: Implementation
MSC: NCLEX: Safe, Effective Care Environment: Safety and Infection Control

Downloaded by: SUCCEEDGRADES | abbieclin@gmail.com


Distribution
Downloaded by Dallen Mae D.ofKadir
this document is illegal
(kadir.dallenmae.d.bcsi@gmail.com)
lOMoARcPSD|35346190

Stuvia.com - The Marketplace to Buy and Sell your Study Material

7. A nurse is instructing a patient about relaxation techniques for pain management. The
patient should:
a. keep bright lights on in the room.
b. use this technique as a way to wake up in the morning.
c. tense and relax individual muscle groups, starting with the toes and feet.
d. try to tense and relax all of the muscles of the body at the same time.
ANS: C
Relaxation involves alternately tensing and relaxing the toes and feet, then working upward
through the leg, the abdomen, the chest, the arms, and finally, the neck and head.

DIF: Cognitive Level: Comprehension REF: p. 602 OBJ: Theory #4


TOP: Relaxation KEY: Nursing Process Step: Implementation
MSC: NCLEX: Psychosocial Integrity: Coping and Adaptation

8. A patient is receiving an ice massage for relief of muscle spasms in the neck. The patient
complains that the ice is making the pain worse. Most helpful response from the nurse
would be:
a. ―I know it is uncomfortable right now, but it will get better in a few minutes.‖
b. ―I will go get an ice pack to pace on your neck.‖
c. ―The alteration of hot and cold application is very helpful for your spasms.‖
d. ―I will stop these cold applications. Not everyone is helped by them.‖
ANS: D
Cold applications make some patients experience an increase in discomfort.

DIF: Cognitive Level: Analysis REF: p. 602 OBJ: Theory #4


TOP: Ice Application KEY: Nursing Process Step: Implementation
MSC: NCLEX: Physiological Integrity: Basic Care and Comfort

9. A patient experiencing pain states that guided imagery has made the pain more manageable
in the past. To assist this patient, the nurse should:
a. find a focal point in the room.
b. bring a newspaper or deck of cards according to patient choice.
c. obtain skin lotion and a towel to give a back rub.
d. read from a script that helps the patient visualize a restful place.
ANS: D
Persons who have difficulty with imagery can be assisted by someone reading a script to
help a patient mentally travel to a favorite spot that is relaxing, soothing, or peaceful.

DIF: Cognitive Level: Application REF: p. 603 OBJ: Theory #4


TOP: Imagery KEY: Nursing Process Step: Implementation
MSC: NCLEX: Psychosocial Integrity: Basic Care and Comfort

10. The nurse takes into consideration that when the patient has an order for a patient-controlled
analgesia (PCA) the pump will be programmed by the:
a. registered nurse.
b. primary care provider.
c. LPN/LVN.

Downloaded by: SUCCEEDGRADES | abbieclin@gmail.com


Distribution
Downloaded by Dallen Mae D.ofKadir
this document is illegal
(kadir.dallenmae.d.bcsi@gmail.com)
lOMoARcPSD|35346190

Stuvia.com - The Marketplace to Buy and Sell your Study Material

d. pharmaceutical company.
ANS: A
PCA pumps are programmed by the RN or the pharmacist. The primary care provider‘s
order will specify the size of the dose and the minimum time between doses.

DIF: Cognitive Level: Knowledge REF: p. 606 OBJ: Theory #4


TOP: Chiropractic KEY: Nursing Process Step: Planning
MSC: NCLEX: Physiological Integrity: Basic Care and Comfort

11. The nurse is aware that the new order for indomethacin (Indocin) involves the
administration of a(n):
a. COX-2 inhibitor.
b. adjuvant analgesic.
c. narcotic analgesic.
d. nonsteroidal anti-inflammatory medication.
ANS: D
Indomethacin is a nonsteroidal anti-inflammatory (NSAID) drug which requires a
prescription.

DIF: Cognitive Level: Knowledge REF: p. 604


OBJ: Clinical Practice #4 TOP: Indomethacin
KEY: Nursing Process Step: Planning
MSC: NCLEX: Physiological Integrity: Pharmacological Therapies

12. The nurse appreciates the principal advantage in using patient-controlled analgesia (PCA) is
that it:
a. reduces patient anxiety about pain by giving the patient more control in its
management.
b. reduces the workload of the nurse, because it does not have to be checked often.
c. eliminates the risk of adverse drug effects from the medication.
d. completely eliminates any pain the patient is experiencing.
ANS: A
A principal advantage of PCA is that it reduces anxiety about pain, because patients are in
control of their pain medication within machine set limits.

DIF: Cognitive Level: Comprehension REF: p. 605


OBJ: Clinical Practice #5 TOP: PCA
KEY: Nursing Process Step: Planning
MSC: NCLEX: Physiological Integrity: Basic Care and Comfort

13. A nurse caring for a patient with a Fentanyl patch assesses that the patient is abnormally
sleepy, is slurring words and is unsteady when ambulating. The nurse should:
a. put up the side rails on the bed.
b. elevate the head of the bed 45 degrees and offer coffee or cola drink.
c. remove the patch and wipe off the skin.
d. apply ice to skin around the patch.
ANS: C

Downloaded by: SUCCEEDGRADES | abbieclin@gmail.com


Distribution
Downloaded by Dallen Mae D.ofKadir
this document is illegal
(kadir.dallenmae.d.bcsi@gmail.com)
lOMoARcPSD|35346190

Stuvia.com - The Marketplace to Buy and Sell your Study Material

When overdose signs occur with Fentanyl patches, the patch should be removed, the skin
cleansed and the problem documented and reported to the charge nurse or primary care
provider.

DIF: Cognitive Level: Application REF: p. 605


OBJ: Clinical Practice #6 TOP: Fentanyl Patch
KEY: Nursing Process Step: Implementation
MSC: NCLEX: Physiological Integrity: Pharmacological Therapies

14. A nurse is assisting in the care of a patient who is receiving pain medication by the epidural
route. It is most important to monitor this patient for which adverse drug effects?
a. Constipation
b. Headache
c. Nausea
d. Hypoventilation
ANS: D
Respiratory depression is a potentially dangerous side effect of epidural analgesia, and a
patient receiving this type of pain medication requires frequent monitoring of respiratory
rate.

DIF: Cognitive Level: Comprehension REF: p. 607


OBJ: Clinical Practice #6 TOP: Epidural Pain Control
KEY: Nursing Process Step: Implementation
MSC: NCLEX: Physiological Integrity: Pharmacological Therapies

15. A patient with an epidural catheter needs to have the dressing at the insertion site changed.
When cleaning the insertion site with povidone iodine swabs, the nurse should:
a. use a circular motion working from the insertion site outward.
b. use a circular motion working from the outside to the insertion site.
c. start above the insertion site and swab in a downward motion.
d. start below the insertion site and swab in an upward motion.
ANS: A
Proper aseptic technique includes starting at the center (the insertion site) and working
outward using a circular motion.

DIF: Cognitive Level: Application REF: p. 608


OBJ: Clinical Practice #5 TOP: Epidural Catheter
KEY: Nursing Process Step: Implementation
MSC: NCLEX: Safe, Effective Care Environment: Safety and Infection Control

16. A nurse is assessing the status of a patient who is sleeping. Which assessment data indicate
that the patient is most likely in stage 3 of non–rapid eye movement (NREM) sleep?
a. Rapid respirations, rapid heart rate
b. Rapid respirations, slow heart rate
c. Decreased respirations, slow heart rate
d. Decreased respirations, rapid heart rate
ANS: C

Downloaded by: SUCCEEDGRADES | abbieclin@gmail.com


Distribution
Downloaded by Dallen Mae D.ofKadir
this document is illegal
(kadir.dallenmae.d.bcsi@gmail.com)
lOMoARcPSD|35346190

Stuvia.com - The Marketplace to Buy and Sell your Study Material

Stage 3 of NREM sleep, in which respirations and heart rate slow, is similar to a coma; it is
also called delta sleep or slow wave sleep because of the high-voltage slow brain waves that
occur.

DIF: Cognitive Level: Comprehension REF: p. 609 OBJ: Theory #7


TOP: Sleep KEY: Nursing Process Step: Assessment
MSC: NCLEX: Physiological Integrity: Basic Care and Comfort

17. The nurse recommends that normal sleepatchesp patterns can best be acquired by suggesting
to the patient that they:
a. smoke cigarettes.
b. drink wine.
c. take a nap during the day.
d. exercise in the mornings.
ANS: D
Maintaining a regular exercise schedule not too close to bedtime helps a person sleep.

DIF: Cognitive Level: Comprehension REF: p. 610 OBJ: Theory #8


TOP: Sleep KEY: Nursing Process Step: Implementation
MSC: NCLEX: Physiological Integrity: Basic Care and Comfort

18. A sleeping patient has periodic pauses in breathing, then starts to breathe again. The nurse
recognizes this sleep pattern is consistent with:
a. insomnia.
b. sleep apnea.
c. narcolepsy.
d. excessive NREM sleep.
ANS: B
Sleep apnea is a condition in which the person stops breathing for brief periods during sleep.

DIF: Cognitive Level: Analysis REF: p. 610 OBJ: Theory #8


TOP: Sleep KEY: Nursing Process Step: Assessment
MSC: NCLEX: Physiological Integrity: Basic Care and Comfort

19. The suggestion by the nurse which is most helpful to a patient who complains of chronically
not feeling rested after sleep would be:
a. avoid going to bed hungry or overly full.
b. try to exercise just before going to bed.
c. sleep late the next morning if possible if you are tired on awakening.
d. take a nap during the day to help relieve overall fatigue.
ANS: A
Going to bed hungry or with a full stomach can interfere with sleep.

DIF: Cognitive Level: Application REF: p. 611


OBJ: Clinical Practice #8 TOP: Sleep Disturbances
KEY: Nursing Process Step: Implementation
MSC: NCLEX: Physiological Integrity: Basic Care and Comfort

Downloaded by: SUCCEEDGRADES | abbieclin@gmail.com


Distribution
Downloaded by Dallen Mae D.ofKadir
this document is illegal
(kadir.dallenmae.d.bcsi@gmail.com)
lOMoARcPSD|35346190

Stuvia.com - The Marketplace to Buy and Sell your Study Material

20. The most helpful intervention by the nurse for a hospitalized child who is having difficulty
falling asleep would be to:
a. keep a night light on in the room.
b. turn off all the lights in the room.
c. have the parents bring a favorite blanket or pillow from home.
d. give of juice and cookies before being put to bed.
ANS: C
Many patients sleep better when they can have a favorite blanket or pillow from home.

DIF: Cognitive Level: Application REF: p. 611


OBJ: Clinical Practice #8 TOP: Sleep
KEY: Nursing Process Step: Implementation
MSC: NCLEX: Physiological Integrity: Basic Care and Comfort

21. A patient who has difficulty falling asleep at night because of anxiety over family problems
asks if he should start taking sedative pills from the pharmacy to sleep better. The best
advice to give this patient is that these pills can be used for:
a. long periods of time, but it is best to check with the primary care provider first.
b. short periods of time, but it is best to check with the primary care provider first.
c. long periods of time without primary care provider approval, because they are
nonprescription medications.
d. short periods of time without physician approval, because they are nonprescription
medications.
ANS: B
Sedative and hypnotic medications can be used to promote sleep, but they should be used
for short-term relief and only after a primary care provider has been consulted.

DIF: Cognitive Level: Comprehension REF: p. 612


OBJ: Clinical Practice #8 TOP: Sleep
KEY: Nursing Process Step: N/A
MSC: NCLEX: Physiological Integrity: Pharmacological Therapies

22. A natural supplement that can enhance sleep for many people is:
a. melatonin.
b. calcium.
c. magnesium.
d. oil of lavender.
ANS: A
Melatonin, a natural supplement, can enhance sleep hours for many people. Calcium and
magnesium are used to relieve leg cramps.

DIF: Cognitive Level: Knowledge REF: p. 612


OBJ: Clinical Practice #8 TOP: Natural Sleep Aid
KEY: Nursing Process Step: N/A MSC: NCLEX: N/A

Downloaded by: SUCCEEDGRADES | abbieclin@gmail.com


Distribution
Downloaded by Dallen Mae D.ofKadir
this document is illegal
(kadir.dallenmae.d.bcsi@gmail.com)
lOMoARcPSD|35346190

Stuvia.com - The Marketplace to Buy and Sell your Study Material

23. The nurse takes into consideration the Joint Commission on Accreditation of Healthcare
Organizations (JCAHO) standards for pain assessment and treatment include:
a. pain treatment is based on objective data collected by the nurse.
b. pain treatment ends at discharge.
c. pain is assessed only for patients who complain of pain.
d. all patients have the right to appropriate assessment of pain.
ANS: D
The JCAHO standards related to pain state that all patients have a right to appropriate
assessment and management of pain and should be educated in the process prior to
discharge.

DIF: Cognitive Level: Knowledge REF: p. 594 OBJ: Theory #1


TOP: JCAHO KEY: Nursing Process Step: Planning
MSC: NCLEX: Physiological Integrity: Basic Care and Comfort

24. A relative complains that an older adult patient takes frequent naps late in the day and
awakens frequently during the night, and wants to know if this is normal. The nurse explains
that an older adult:
a. will awaken more often during the night, but may nap more often during the day.
b. needs at least 10 hours of sleep a day to prevent fatigue.
c. requires less napping during the day to sleep better at night.
d. should be given hypnotics to induce better sleep.
ANS: A
Sleep habits are very individualized, but normally, as an adult age, sleep may be less at night
with more naps being taken late in the day.

DIF: Cognitive Level: Comprehension REF: p. 609 OBJ: Theory #6


TOP: Sleep Needs KEY: Nursing Process Step: Assessment
MSC: NCLEX: Physiological Integrity: Basic Care and Comfort

25. A nurse explains that endorphins are capable of reducing pain:


a. as a naturally occurring morphine like enzyme.
b. in conditions that are physiological only.
c. by attaching to opioid receptors.
d. when psychological stressors are the cause of pain.
ANS: C
Endorphins are naturally occurring opiate like peptides which attach to the opioid receptors
of nerve endings and bock pain transmission. Endorphins are effective against both
physiologically and psychologically causes of pain.

DIF: Cognitive Level: Comprehension REF: p. 595 OBJ: Theory #4


TOP: Endorphins KEY: Nursing Process Step: Implementation
MSC: NCLEX: Physiological Integrity: Basic Care and Comfort

26. A nurse is preparing a patient for home care following cancer treatment. He is discussing the
use of the fentanyl patch. Which of the following would be the most important instruction
regarding safety?

Downloaded by: SUCCEEDGRADES | abbieclin@gmail.com


Distribution
Downloaded by Dallen Mae D.ofKadir
this document is illegal
(kadir.dallenmae.d.bcsi@gmail.com)
lOMoARcPSD|35346190

Stuvia.com - The Marketplace to Buy and Sell your Study Material

a. Apply as directed by the primary care provider.


b. Store fentanyl patches in a locked cabinet.
c. Prepare the skin by cleaning with an antiseptic scrub solution.
d. Use as needed for break through pain.
ANS: B
Several deaths have been reported from children either fentanyl swallowing the patches or
applying them to their skin. Teach the patient to keep the patches away from children,
locked up is possible.

DIF: Cognitive Level: Analysis REF: p. 605 OBJ: Clinical #1 and #2


TOP: Medical Methods of Pain Control KEY: Nursing Process Step: Implementation
MSC: NCLEX: QSEN: Safety

MULTIPLE RESPONSE

1. The nurse outlines the four phases of nociceptive pain as: (Select all that apply.)
a. translation.
b. transduction.
c. transmission.
d. modulation.
e. perception.
ANS: B, C, D, E
The four phases of nociceptive pain are transduction, transmission, perception, and
modulation.

DIF: Cognitive Level: Comprehension REF: p. 597


OBJ: Clinical Practice #1 TOP: Nociceptive Pain
KEY: Nursing Process Step: Implementation
MSC: NCLEX: Physiological Integrity: Basic Care and Comfort

2. The nurse attempts to help an 86-year-old patient describe his pain because the nurse is
aware the older adult may not express pain because they: (Select all that apply.)
a. are unaware of their discomfort.
b. believe pain is a natural consequence to aging.
c. are reluctant to bother the nursing staff.
d. have been culturally trained not to complain.
e. tear taking pain medication.
ANS: B, C, D
Older adults often do not verbalize their pain out of the mistaken belief that pain is part of
aging. They are reluctant to ―bother‖ the staff and have been culturally trained not to
complain.

DIF: Cognitive Level: Comprehension REF: p. 598


OBJ: Clinical Practice #1 TOP: Pain in the Older Adult
KEY: Nursing Process Step: Assessment
MSC: NCLEX: Physiological Integrity: Basic Care and Comfort

Downloaded by: SUCCEEDGRADES | abbieclin@gmail.com


Distribution
Downloaded by Dallen Mae D.ofKadir
this document is illegal
(kadir.dallenmae.d.bcsi@gmail.com)
lOMoARcPSD|35346190

Stuvia.com - The Marketplace to Buy and Sell your Study Material

3. The nurse performing a focused assessment on pain will assess: (Select all that apply.)
a. history of pain.
b. verbal indicators.
c. psychological factors.
d. culture.
e. contributing factors.
ANS: A, B, C, E
Factors to be assessed in a focused assessment of pain are the history, verbal and nonverbal
indicators, psychological and other contributing factors, medication, and treatments used.
Although culture may limit a person‘s ability to express pain, it is not an important issue on
a focused assessment.

DIF: Cognitive Level: Comprehension REF: p. 598


OBJ: Clinical Practice #1 TOP: Focused Pain Assessment
KEY: Nursing Process Step: Implementation
MSC: NCLEX: Physiological Integrity: Basic Care and Comfort

COMPLETION

1. is considered to be the fifth vital sign.

ANS:
Pain

Pain assessment is now performed along with each assessment of vital signs, and pain is
now considered the fifth vital sign.

DIF: Cognitive Level: Knowledge REF: p. 594 OBJ: Theory #2


TOP: Pain KEY: Nursing Process Step: Assessment
MSC: NCLEX: Physiological Integrity: Basic Care and Comfort

2. A type of pain that is of short duration, lasting from a few hours to a few days, is known as
pain.

ANS:
acute

Acute pain is of short duration, lasting from a few hours to a few days.

DIF: Cognitive Level: Knowledge REF: p. 594


OBJ: Clinical Practice #2 TOP: Pain
KEY: Nursing Process Step: Assessment MSC: NCLEX: N/A

3. The is based on the premise that by stimulating large diameter nerve


fibers pain is diminished or totally controlled.

ANS:
Gate Control Theory

Downloaded by: SUCCEEDGRADES | abbieclin@gmail.com


Distribution
Downloaded by Dallen Mae D.ofKadir
this document is illegal
(kadir.dallenmae.d.bcsi@gmail.com)
lOMoARcPSD|35346190

Stuvia.com - The Marketplace to Buy and Sell your Study Material

The Gate Control Theory is based on the premise that by stimulating large diameter nerve
fibers by massage or vibration the ―gate‖ to pain perception is closed against the pain
impulses from the small diameter nerve fibers.

DIF: Cognitive Level: Knowledge REF: p. 595 OBJ: Theory #3


TOP: Gate Control Theory KEY: Nursing Process Step: N/A
MSC: NCLEX: N/A

4. A pain scale FLACC is used in assessing pain in .

ANS:
preverbal children and noncommunicacive adults

FLACC stands for Face, Legs, Activity, Cry, and Consolability, which are assessed to
estimate the pain in preverbal infants.

DIF: Cognitive Level: Knowledge REF: p. 599


OBJ: Clinical Practice #1 TOP: Pain in Infants
KEY: Nursing Process Step: Assessment
MSC: NCLEX: Physiological Integrity: Basic Care and Comfort

5. Travelers can combat ―jet lag‖ by exposure to for several hours.

ANS:
sunlight

Exposure to sunlight for several hours will help ―reset‖ a traveler‘s internal clock and reduce
the effects of jet lag.

DIF: Cognitive Level: Comprehension REF: p. 610


OBJ: Clinical Practice #6 TOP: Jet Lag
KEY: Nursing Process Step: Assessment MSC: NCLEX: N/A

6. A nurse removed a pain medication patch that has a metal clip before the patient goes to
have a(n) .

ANS:
MRI

All metal must be removed from persons who are going to have an MRI.

DIF: Cognitive Level: Application REF: p. 605 OBJ: Theory #4


TOP: MRI Consideration KEY: Nursing Process Step: Implementation
MSC: NCLEX: Health Promotion and Maintenance: Safety and Infection Control

Downloaded by: SUCCEEDGRADES | abbieclin@gmail.com


Distribution
Downloaded by Dallen Mae D.ofKadir
this document is illegal
(kadir.dallenmae.d.bcsi@gmail.com)
lOMoARcPSD|35346190

Stuvia.com - The Marketplace to Buy and Sell your Study Material

Chapter 32: Complementary and Alternative Therapies


Williams: deWit's Fundamental Concepts and Skills for Nursing, 8th Edition

MULTIPLE CHOICE

1. When asked to give an example of complementary therapies, the nurse gives:


a. eating a macrobiotic diet to treat cancer instead of having surgery.
b. using imagery along with pain medication to increase comfort.
c. practicing naturopathic medicine, a natural means of promoting health.
d. practicing traditional Chinese medicine based on yin and yang.
ANS: B
Complementary therapies are used jointly with conventional methods to treat a problem. All
the other possibilities listed are alternative therapies.

DIF: Cognitive Level: Comprehension REF: p. 614 OBJ: Theory #1


TOP: Complementary Therapies KEY: Nursing Process Step: Implementation
MSC: NCLEX: Physiological Integrity: Basic Care and Comfort

2. When the patient asked the nurse about the use of acupuncture for the control of back pain,
the nurse warned that acupuncture can put the patient at risk for:
a. meningitis.
b. streptococcal infection.
c. hepatitis.
d. Shigella.
ANS: C
The main concern of acupuncture therapy is the use of needles for this treatment. If sterile
needles are not used, there is a high risk of HIV or hepatitis transmission.

DIF: Cognitive Level: Comprehension REF: p. 615 OBJ: Theory #3


TOP: Acupuncture KEY: Nursing Process Step: Assessment
MSC: NCLEX: Health Promotion and Maintenance: Prevention and Early Detection of
Disease

3. A nurse is caring for a patient admitted for electrolyte imbalance who has been practicing
Ayurveda, a traditional Hindu system of medicine. The nurse should inform the physician
that the patient is practicing this traditional form of medicine, because Ayurveda:
a. uses purgatives as part of the treatment.
b. forbids invasive treatment methods.
c. requires that the shaman and physician confer.
d. uses natural electrical currents in meridian areas.
ANS: A
Ayurveda uses special diets and purgatives as part of the practice, which could cause
electrolyte imbalance. It does not forbid invasive procedures such as IVs.

DIF: Cognitive Level: Knowledge REF: p. 615


OBJ: Clinical Practice #2 TOP: Ayurveda

Downloaded by: SUCCEEDGRADES | abbieclin@gmail.com


Distribution
Downloaded by Dallen Mae D.ofKadir
this document is illegal
(kadir.dallenmae.d.bcsi@gmail.com)
lOMoARcPSD|35346190

Stuvia.com - The Marketplace to Buy and Sell your Study Material

KEY: Nursing Process Step: Implementation


MSC: NCLEX: Physiological Integrity: Reduction of Risk

4. A Mexican patient is admitted to the hospital with abdominal pain. The nurse learns that the
patient has been treated for the same condition by a curandero and wants the curandero to
visit and perform rituals. The best response via the interpreter by the nurse is:
a. ―I‘m sorry, but only immediate family can visit.‖
b. ―It is not safe to have burning of herbs in the hospital.‖
c. ―Trust your doctor; he knows what is best for you.‖
d. ―I will inform the doctor so that they can perhaps confer.‖
ANS: D
In Mexico, the use of the curandero is very common. It is best for the doctor to talk to the
curandero to learn about the herbs used to treat the abdominal pain.

DIF: Cognitive Level: Application REF: p. 620 OBJ: Theory #2


TOP: Folk Medicine KEY: Nursing Process Step: Implementation
MSC: NCLEX: Physiological Integrity: Pharmacological Therapies

5. An elderly Hispanic male was admitted with a severe headache and is anxious. The patient
tells the nurse that he was given the mal de ojo. The culturally competent nurse understands
that mal de ojo means that the patient believes that he:
a. has a blockage in the intestine from the influence of Susto.
b. has an opposition in polarities problem because of the effect of empacho.
c. was given the evil eye, which is thought to cause weeping and headache.
d. has had a separation of the soul from the body.
ANS: C
The mal de ojo literally means ―the evil eye‖ cast from someone, which can bring fear,
irritability, headache, and weeping.

DIF: Cognitive Level: Comprehension REF: p. 620


OBJ: Clinical Practice #1 TOP: Folk Medicine
KEY: Nursing Process Step: Assessment
MSC: NCLEX: Psychosocial Integrity: Coping and Adaptation

6. A patient asks about the difference between imagery and meditation. The nurse explains that
imagery uses a visual stimulus that can decrease stress, whereas meditation:
a. is a relaxation technique used to help with acute illness.
b. involves focusing on a single repetitive stimulus, altering consciousness.
c. is a technique that trains the patient to retrieve memories and induce anesthesia.
d. involves a combination of exercise and controlled breathing.
ANS: B
Meditation focuses on a single repetitive stimulus, altering consciousness and inducing a
restful state. It has been helpful for people with chronic conditions such as asthma and
diabetes.

DIF: Cognitive Level: Comprehension REF: p. 616 OBJ: Theory #4


TOP: Meditation KEY: Nursing Process Step: Implementation

Downloaded by: SUCCEEDGRADES | abbieclin@gmail.com


Distribution
Downloaded by Dallen Mae D.ofKadir
this document is illegal
(kadir.dallenmae.d.bcsi@gmail.com)
lOMoARcPSD|35346190

Stuvia.com - The Marketplace to Buy and Sell your Study Material

MSC: NCLEX: Psychosocial Integrity: Coping and Adaptation

7. A patient who is a heavy smoker has been told by the physician to stop smoking, but does
not want to use a smoking cessation patch. The nurse suggests that the most effective
alternative is:
a. meditation.
b. Qi Gong.
c. biofeedback.
d. hypnosis.
ANS: D
Hypnosis is therapy in which the patient is put into a hypnotic state, during which time
suggestions are implanted in the mind that remain after the hypnosis ends.

DIF: Cognitive Level: Comprehension REF: p. 617 OBJ: Theory #4


TOP: Hypnotherapy KEY: Nursing Process Step: Implementation
MSC: NCLEX: Physiological Integrity: Basic Care and Comfort

8. A patient who is in labor is having anxiety and sobs with each contraction. The nurse
decides to use imagery to calm the patient by:
a. having the patient close her eyes and picture herself in her favorite place.
b. having the patient do stretch exercises before each contraction.
c. telling the patient to relax and to calm down.
d. telling the patient strenuously focus on some image in the room.
ANS: A
Imagery involves visual stimuli to produce some physiological change that can decrease
anxiety or decrease pain. It is used often for patients in labor. The patient is asked to
mentally recall some visual image that is pleasant to them.

DIF: Cognitive Level: Comprehension REF: p. 616


OBJ: Clinical Practice #3 TOP: Imagery
KEY: Nursing Process Step: Implementation
MSC: NCLEX: Psychosocial Integrity: Coping and Adaptation

9. A patient‘s blood pressure measures 170/90 mm Hg. There is no history of hypertension, but
the patient is very stressed because of career demands. The best complementary therapy that
the patient might benefit from is:
a. meditation therapy.
b. meditation and medication.
c. dance therapy.
d. prayer and herbal therapy.
ANS: B
Complementary therapy involves both alternative and conventional forms of medical
treatment, such as meditation and antihypertensives.

DIF: Cognitive Level: Analysis REF: p. 614 OBJ: Theory #2


TOP: Complementary Therapies KEY: Nursing Process Step: N/A
MSC: NCLEX: N/A

Downloaded by: SUCCEEDGRADES | abbieclin@gmail.com


Distribution
Downloaded by Dallen Mae D.ofKadir
this document is illegal
(kadir.dallenmae.d.bcsi@gmail.com)
lOMoARcPSD|35346190

Stuvia.com - The Marketplace to Buy and Sell your Study Material

10. A patient is scheduled to have biofeedback therapy for migraine headaches. On arrival to the
clinic, the patient appears anxious and fearful and tells the nurse that she does not want
electric shocks. The most reassuring response by the nurse is:
a. ―Don‘t worry; this will not hurt a bit.‖
b. ―Didn‘t the doctor explain this procedure to you?‖
c. ―There are no electrical shocks involved; the doctor is looking at your body‘s
activity.‖
d. ―You seem too anxious for this procedure. Let‘s reschedule it at another time.‖
ANS: C
Biofeedback involves applying electrodes to the body to look at the internal activity, so that
the patient can gain control of involuntary activity.

DIF: Cognitive Level: Application REF: p. 616


OBJ: Clinical Practice #2 TOP: Biofeedback
KEY: Nursing Process Step: Implementation
MSC: NCLEX: Psychosocial Integrity: Coping and Adaptation

11. A patient who has job-related stress and chronic back pain wants to start exercising but
dislikes physical activity. The nurse suggests that the most beneficial therapy for this patient
is:
a. yoga therapy.
b. dance therapy.
c. Ayurveda.
d. shamanism.
ANS: A
Yoga is a combination of exercise, controlled breathing, posture, and mental focus to bring
positive effects on the body and mind. It is effective for stress reduction and chronic back
problems.

DIF: Cognitive Level: Comprehension REF: p. 615 OBJ: Theory #3


TOP: Yoga KEY: Nursing Process Step: Implementation
MSC: NCLEX: Physiological Integrity: Physiological Adaptation

12. Before integrating humor in the care of an Asian patient, the nurse should:
a. develop jokes about the patient‘s condition.
b. develop jokes about the doctors and nurses.
c. ask the patient whether jokes are understood.
d. ask the patient‘s feelings about hearing a joke or funny story.
ANS: D
Humor and laughter are used at the bedside to distract patients from pain, and they have a
positive effect on the body. However, it is important to check with the patient first to find
out whether it might be considered disrespectful.

DIF: Cognitive Level: Application REF: p. 617 OBJ: Theory #2


TOP: Humor KEY: Nursing Process Step: Assessment
MSC: NCLEX: Psychosocial Integrity: Psychosocial Adaptation

Downloaded by: SUCCEEDGRADES | abbieclin@gmail.com


Distribution
Downloaded by Dallen Mae D.ofKadir
this document is illegal
(kadir.dallenmae.d.bcsi@gmail.com)
lOMoARcPSD|35346190

Stuvia.com - The Marketplace to Buy and Sell your Study Material

13. The nurse caring for a patient without any religious affiliation who was admitted 2 days ago
with a severe gastrointestinal infection encounters the hospital chaplain who wants to pray
with the patient. The most appropriate response by the nurse is:
a. ―That is a wonderful idea. I am sure that the patient will find comfort in that.‖
b. ―Before you do that, we need to get the patient‘s permission.‖
c. ―I need to get an order from the doctor.‖
d. ―I don‘t think the patient wants any religious interventions.‖
ANS: B
Although prayer has been known to promote healing, the patient‘s privacy must be
protected.

DIF: Cognitive Level: Application REF: p. 617


OBJ: Clinical Practice #1 TOP: Prayer
KEY: Nursing Process Step: Implementation
MSC: NCLEX: Psychosocial Integrity: Psychosocial Adaptation

14. A Hispanic female patient visits the clinic with complaints of sleep loss and wants sleeping
pills. In further talking with the patient, the nurse learns that the patient is also drinking
valerian tea. The most appropriate nursing action is to:
a. talk to the patient about her usual sleep habits.
b. instruct the patient on the appropriate dose for sleeping pills.
c. inform the doctor that the patient is drinking valerian tea.
d. give written instructions about the effects and side effects of sleeping pills.
ANS: C
It is vital for health care providers to know whether patients are taking over-the-
counter herbal or homeopathic substances because these may interact with prescription
medications, even leading to organ transplant rejection Valerian is a minor tranquilizer for
sleep disorders. Combining both valerian and hypnotics puts this patient at risk for sedation
overdose, so it is best that the doctor is aware.

DIF: Cognitive Level: Application REF: p. 620 OBJ: Theory #5


TOP: Herbal Remedies KEY: Nursing Process Step: Implementation
MSC: NCLEX: Health Promotion and Maintenance: Prevention and Early Detection of
Disease

15. The home health nurse helps a patient schedule an appointment with a chiropractor because
of chronic back pain, instructing the patient that the chiropractic treatment will involve:
a. manipulation of the spine.
b. deep massage.
c. herbal remedies.
d. energy therapies.
ANS: A
Chiropractic therapy is the manipulation of the spine for relief of pain by correcting the
spine alignment, and is being accepted more widely as an alternative form of health care.

DIF: Cognitive Level: Knowledge REF: p. 616 OBJ: Theory #6

Downloaded by: SUCCEEDGRADES | abbieclin@gmail.com


Distribution
Downloaded by Dallen Mae D.ofKadir
this document is illegal
(kadir.dallenmae.d.bcsi@gmail.com)
lOMoARcPSD|35346190

Stuvia.com - The Marketplace to Buy and Sell your Study Material

TOP: Chiropractic Therapy KEY: Nursing Process Step: Implementation


MSC: NCLEX: Physiological Integrity: Basic Care and Comfort

16. The nurse is assessing a patient and finds that the patient is taking ephedra for weight
control. The finding that is most likely attributable to ephedra is:
a. BP 138/90 mm Hg, pulse rate 95 beats/min.
b. gain of 15 pounds in 4 weeks.
c. excessive diarrhea for the last 3 days.
d. BP 180/98 mm Hg, pulse rate 130 beats/min.
ANS: D
Ephedra, or ma huang, is a cardiac stimulant found in weight loss preparations, and the
patient should be instructed to stop taking it because of its adverse side effects.

DIF: Cognitive Level: Comprehension REF: p. 619


OBJ: Clinical Practice #2 TOP: Herbal Therapy
KEY: Nursing Process Step: Assessment
MSC: NCLEX: Health Promotion and Maintenance: Prevention and Early Detection of
Disease

17. When an asthmatic patient asks the nurse about using aromatherapy oils for decreasing
stress, the nurse expresses concern to the patient about:
a. the cost of the aromatherapy.
b. possible allergic reaction to the oils.
c. the oils causing staining of furniture.
d. the oils not being very effective.
ANS: B
The aromatic properties of certain herbs could cause allergic reactions that can trigger
asthmatic episodes.

DIF: Cognitive Level: Comprehension REF: p. 619


OBJ: Clinical Practice #2 TOP: Aromatherapy
KEY: Nursing Process Step: Implementation
MSC: NCLEX: Health Promotion and Maintenance: Prevention and Early Detection of
Disease

18. A patient is seen by an alternative medicine practitioner for menopausal symptoms. The
most popular and effective herbal remedy that the alternative medicine practitioner would
suggest is:
a. valerian.
b. ma huang.
c. black cohosh.
d. echinacea.
ANS: C
Women are using alternative hormone replacement therapy such as black cohosh to treat hot
flashes and mood swings.

DIF: Cognitive Level: Comprehension REF: p. 619 OBJ: Theory #3

Downloaded by: SUCCEEDGRADES | abbieclin@gmail.com


Distribution
Downloaded by Dallen Mae D.ofKadir
this document is illegal
(kadir.dallenmae.d.bcsi@gmail.com)
lOMoARcPSD|35346190

Stuvia.com - The Marketplace to Buy and Sell your Study Material

TOP: Herbal Remedies KEY: Nursing Process Step: N/A


MSC: NCLEX: N/A

19. A patient asks the nurse about the difference between manipulative and body-based methods
and mind–body therapies. The nurse‘s correct response is that:
a. manipulative and body-based methods involve touching or manipulation, and
mind–body therapies deal with the mind‘s ability to affect the body.
b. mind–body therapies involve touch and manipulation, and body-based methods
involve the mind‘s ability to affect the body.
c. mind–body therapies involve the use of herbal remedies, whereas manipulative and
body-based methods deal with touch.
d. an example of manipulative body-based methods is biofeedback, and an example of
mind–body therapies is massage.
ANS: A
Manipulative and body-based methods involve touching, such as in chiropractic therapy, and
mind–body therapy is the mind working to heal the body, such as with imagery.

DIF: Cognitive Level: Comprehension REF: p. 616 OBJ: Theory #4


TOP: Manipulative and Body-Based Methods Versus Mind Body Therapies
KEY: Nursing Process Step: Implementation
MSC: NCLEX: Physiological Integrity: Basic Care and Comfort

20. An elderly female patient is placed on warfarin sodium (Coumadin) for thrombophlebitis
and is seen in the clinic for a follow-up visit. On assessment, the patient complains that her
forgetfulness is becoming worse and asks if ginkgo biloba would help. The most appropriate
response by the nurse is:
a. ―Ginkgo biloba improves memory, and you should have good results.‖
b. ―Ginkgo biloba should not be part of your regimen, and the doctor does not believe
in it.‖
c. ―You should take St. John‘s wort instead, because it works better.‖
d. ―Coumadin and gingko biloba should not be taken together because the herb
prolongs bleeding time.‖
ANS: D
Coumadin and ginkgo biloba should not be used together, because the herb prolongs the
bleeding time, placing the patient in danger of hemorrhage.

DIF: Cognitive Level: Application REF: p. 618 OBJ: Theory #5


TOP: Complementary Therapy Effects KEY: Nursing Process Step: Implementation
MSC: NCLEX: Physiological Integrity: Pharmacological Therapies

21. A patient with thrombophlebitis is contemplating being seen by a massage therapist for a
deep body massage. The nurse warns that such a therapy may be harmful because:
a. the practitioner will use magnets on the body to relieve pain.
b. the practitioner will use electrical stimulation.
c. there is an increased risk of breaking loose a clot with the massage.
d. massage may cause vasoconstriction.
ANS: C

Downloaded by: SUCCEEDGRADES | abbieclin@gmail.com


Distribution
Downloaded by Dallen Mae D.ofKadir
this document is illegal
(kadir.dallenmae.d.bcsi@gmail.com)
lOMoARcPSD|35346190

Stuvia.com - The Marketplace to Buy and Sell your Study Material

With a history of thrombophlebitis, the massage might cause more injury to the patient by
breaking loose a clot with the stroking and kneading performed to the legs and by the clot
traveling to the lung, heart, or brain.

DIF: Cognitive Level: Analysis REF: p. 616


OBJ: Clinical Practice #2 TOP: Massage
KEY: Nursing Process Step: Implementation
MSC: NCLEX: Health Promotion and Maintenance: Prevention and Early Detection of
Disease

22. When taking the depressed patient‘s history, the nurse finds that the patient is taking Prozac
and the herbal remedy St. John‘s wort. The nurse gives this information to the physician
immediately because:
a. the patient is still depressed and needs another antidepressant added.
b. Prozac and St. John‘s wort should not be taken together, because this can cause
toxicity.
c. Prozac and St. John‘s wort are complementary therapies and the doctor should
know this.
d. the patient should not be taking herbal remedies, because they have proven to be
ineffective.
ANS: B
Prozac and St. John‘s wort together have a negative effect on the body, causing toxicity.

DIF: Cognitive Level: Analysis REF: p. 618 OBJ: Theory #5


TOP: Herbal Remedies KEY: Nursing Process Step: Implementation
MSC: NCLEX: Health Promotion and Maintenance: Prevention and Early Detection of
Disease

MULTIPLE RESPONSE

1. The nurse‘s role when patients practice complementary and alternative therapies include:
(Select all that apply.)
a. asking whether patients are taking over the counter herbal remedies.
b. questioning the patient extensively regarding why he or she is practicing
complementary or alternative therapies.
c. exploring the cost of treatment and how many treatments are required.
d. instructing the patient to inform the health care provider when such therapies are
being used.
ANS: A, D
The nurse‘s responsibility is gathering necessary information about the complementary or
alternative therapies the patient might be practicing to help the physician make the best
decision for the health care of the patient.

DIF: Cognitive Level: Application REF: p. 620 OBJ: Theory #5


TOP: Nurse‘s Role KEY: Nursing Process Step: Implementation
MSC: NCLEX: Physiological Integrity: Basic Care and Comfort

Downloaded by: SUCCEEDGRADES | abbieclin@gmail.com


Distribution
Downloaded by Dallen Mae D.ofKadir
this document is illegal
(kadir.dallenmae.d.bcsi@gmail.com)
lOMoARcPSD|35346190

Stuvia.com - The Marketplace to Buy and Sell your Study Material

2. The culturally competent nurse is aware that the American Indian medicine techniques are
very significant to the patient. This medical intervention involves: (Select all that apply.)
a. healing ceremonies.
b. rituals performed by highly trained medicine people.
c. burning of sweet grass.
d. purging with minerals.
e. drums to align the patient‘s heart with Mother Earth.
ANS: A, B, C, E
The American Indian medicine techniques involve many specific ceremonies and rituals
performed by highly trained medicine people. The burning of herbs, shaking rattles, and
drumming realigns the patient with Mother Earth.

DIF: Cognitive Level: Application REF: p. 620 OBJ: Theory #3


TOP: American Indian Medicine KEY: Nursing Process Step: Planning
MSC: NCLEX: Psychosocial Integrity: Psychosocial Adaptation

3. The practitioner of Reiki acts as a conduit for healing energy by directing healing energy to
the patient‘s receptors located in the: (Select all that apply.)
a. arms.
b. forehead.
c. groin.
d. feet.
e. hands.
ANS: B, C
Reiki directs energy through the practitioner to the receptors located in the patient‘s crown,
forehead, throat, heart, stomach, abdomen, and groin.

DIF: Cognitive Level: Knowledge REF: p. 617 OBJ: Theory #3


TOP: Reiki KEY: Nursing Process Step: N/A MSC: NCLEX: N/A

COMPLETION

1. A branch of traditional Chinese medicine in which very fine needles are used to stimulate
certain points on the body along lines called meridians to increase or disperse the flow of
energy is known as .

ANS:
acupuncture

Acupuncture is a branch of traditional Chinese medicine. Very fine needles are used to
stimulate certain points on the body along lines called meridians to increase or disperse the
flow of energy.

DIF: Cognitive Level: Comprehension REF: p. 615 OBJ: Theory #3


TOP: Alternative Therapy KEY: Nursing Process Step: N/A
MSC: NCLEX: N/A

Downloaded by: SUCCEEDGRADES | abbieclin@gmail.com


Distribution
Downloaded by Dallen Mae D.ofKadir
this document is illegal
(kadir.dallenmae.d.bcsi@gmail.com)
lOMoARcPSD|35346190

Stuvia.com - The Marketplace to Buy and Sell your Study Material

2. An alternative type of medical practice that stimulates the patient‘s natural defenses to
alleviate the problem because illness is considered to be specific to the individual is
.

ANS:
naturopathic medicine

Naturopathic medicine uses a combination of lifestyle management, application of


botanicals, light, fresh air, and exercise to maintain a high level of wellness.

DIF: Cognitive Level: Comprehension REF: p. 619 OBJ: Theory #3


TOP: Naturopathic Medicine KEY: Nursing Process Step: N/A
MSC: NCLEX: N/A

3. The technique in which the practitioner alters body energy fields by passing his or her hands
over the patient to determine where tensions exist is the practice of .

ANS:
therapeutic touch

The concept of therapeutic touch involves the caregiver causing an alteration of the patient‘s
body energy field by passing his or her hands over the patient‘s body.

DIF: Cognitive Level: Comprehension REF: p. 617 OBJ: Theory #3


TOP: Therapeutic Touch KEY: Nursing Process Step: N/A
MSC: NCLEX: N/A

4. A form of alternative therapy that is used by more than 70% of the world‘s population for
treatment of disorders with botanicals is .

ANS:
herbal therapy
phytotherapy

Herbal therapy (phytotherapy) is a system of herbal applications used to treat a wealth of


disorders and is used by 70% of the world‘s population.

DIF: Cognitive Level: Comprehension REF: p. 617 OBJ: Theory #3


TOP: Herbal Therapy KEY: Nursing Process Step: N/A
MSC: NCLEX: N/A

5. The alternative practice in which the practitioner journeys to other planes of existence to
retrieve information for the healing process is .

ANS:
Shamanism

Downloaded by: SUCCEEDGRADES | abbieclin@gmail.com


Distribution
Downloaded by Dallen Mae D.ofKadir
this document is illegal
(kadir.dallenmae.d.bcsi@gmail.com)
lOMoARcPSD|35346190

Stuvia.com - The Marketplace to Buy and Sell your Study Material

The Shaman journeys to different planes of existence to retrieve information for healing.
The Shaman performs rituals with burning plants and herbs to rebalance the patient with
nature.

DIF: Cognitive Level: Comprehension REF: p. 619 OBJ: Theory #2


TOP: Shamanism KEY: Nursing Process Step: N/A MSC: NCLEX: N/A

Downloaded by: SUCCEEDGRADES | abbieclin@gmail.com


Distribution
Downloaded by Dallen Mae D.ofKadir
this document is illegal
(kadir.dallenmae.d.bcsi@gmail.com)
lOMoARcPSD|35346190

Stuvia.com - The Marketplace to Buy and Sell your Study Material

Chapter 33: Pharmacology and Preparation for Drug Administration


Williams: deWit's Fundamental Concepts and Skills for Nursing, 8th Edition

MULTIPLE CHOICE

1. A nurse has an order to administer a schedule II drug to a patient. When working with
medications of this type, the responsibility of the nurse is to:
a. leave the medication in a cup at the bedside.
b. ask another licensed nurse to check the dose.
c. sign out the drug on a narcotic control inventory sheet.
d. tell the patient to drink extra water with the pill.
ANS: C
Schedule II drugs are narcotics, which are controlled substances that are kept in a locked
area on the nursing unit, and each dose must be signed out.

DIF: Cognitive Level: Application REF: p. 622 OBJ: Theory #2


TOP: Legal Control of Drugs KEY: Nursing Process Step: Implementation
MSC: NCLEX: Physiological Integrity: Pharmacological Therapies

2. The nurse is aware that the primary care provider has ordered a pain relief drug to be
delivered in the manner in which postoperative pain would be relieved most rapidly. This
method is:
a. intradermally.
b. orally.
c. intramuscularly.
d. intravenously.
ANS: D
Intravenously administered medications are absorbed more quickly than medications
administered by other routes.

DIF: Cognitive Level: Knowledge REF: p. 625|Table 33-5


OBJ: Theory #3 TOP: Medication Absorption
KEY: Nursing Process Step: Planning
MSC: NCLEX: Physiological Integrity: Pharmacological Therapies

3. A nurse is administering a medication to an older adult patient who is normally highly


protein bound. The nurse would be concerned about increased drug activity and possible
toxicity if the patient‘s laboratory values show:
a. low albumin levels.
b. high glucose levels.
c. low sodium levels.
d. high potassium levels.
ANS: A
Albumin is a type of protein, and this patient is at risk if the albumin level is low, because
this larger amount of drug will circulate in unbound form, increasing risk of adverse and
toxic effects.

Downloaded by: SUCCEEDGRADES | abbieclin@gmail.com


Distribution
Downloaded by Dallen Mae D.ofKadir
this document is illegal
(kadir.dallenmae.d.bcsi@gmail.com)
lOMoARcPSD|35346190

Stuvia.com - The Marketplace to Buy and Sell your Study Material

DIF: Cognitive Level: Analysis REF: p. 632


OBJ: Clinical Practice #1 TOP: Medication Absorption
KEY: Nursing Process Step: Assessment
MSC: NCLEX: Physiological Integrity: Pharmacological Therapies

4. A patient with liver disease is beginning medication therapy with a drug that is metabolized
in the liver. The nurse anticipates the dose of the medication to be:
a. increased above the normal dose.
b. double the normal dose.
c. unchanged from the normal dose.
d. lower than the normal dose.
ANS: D
When there is a decrease in liver function resulting from disease or aging, a smaller dose
may be ordered to prevent excess drug accumulation and development of toxicity.

DIF: Cognitive Level: Analysis REF: p. 632


OBJ: Clinical Practice #1 TOP: Factors Leading to Toxicity
KEY: Nursing Process Step: Planning
MSC: NCLEX: Health Promotion and Maintenance: Prevention and Early Detection of
Disease

5. A patient is receiving antibiotic drug therapy. It is important to ensure the scheduled dose is
not given late to keep the circulating drug level above the:
a. peak concentration level.
b. minimum concentration level.
c. average concentration level.
d. baseline concentration level.
ANS: B
If a drug is given late, the concentration level of the drug in the circulation could drop below
the minimum effective concentration level.

DIF: Cognitive Level: Comprehension REF: p. 627


OBJ: Clinical Practice #1 TOP: Pharmacodynamics
KEY: Nursing Process Step: Planning
MSC: NCLEX: Physiological Integrity: Pharmacological Therapies

6. A patient is receiving an initial dose of penicillin for pneumonia. The nurse should be alert
and monitor for:
a. hives.
b. nausea.
c. fever.
d. dizziness.
ANS: A
A rash or hives can indicate allergic response; nausea and dizziness are examples of adverse
drug effects. With an allergic reaction the patient is cautions to never take the drug again.

Downloaded by: SUCCEEDGRADES | abbieclin@gmail.com


Distribution
Downloaded by Dallen Mae D.ofKadir
this document is illegal
(kadir.dallenmae.d.bcsi@gmail.com)
lOMoARcPSD|35346190

Stuvia.com - The Marketplace to Buy and Sell your Study Material

DIF: Cognitive Level: Application REF: p. 628


OBJ: Clinical Practice #1 TOP: Allergic Response
KEY: Nursing Process Step: Implementation
MSC: NCLEX: Health Promotion and Maintenance: Prevention and Early Detection of
Disease

7. A nurse is reinforcing instructions to a patient who is beginning medication therapy with a


central nervous system (CNS) depressant drug. The nurse cautions the patient not to drink
alcohol, because alcohol and the drug could cause a synergic effect, which means:
a. the alcohol makes the drug have less than the desired effect.
b. the drug undergoes a rapid breakdown and is rapidly excreted.
c. the drug and alcohol increase the effect on the central nervous system.
d. the drug changes the alcohol to a toxic substance.
ANS: C
Alcohol has a synergistic effect when combined with any drug that depresses the CNS,
because it is also a CNS depressant. The combination of the two makes the drug more
powerful.

DIF: Cognitive Level: Knowledge REF: p. 628


OBJ: Clinical Practice #2 TOP: Drug Interactions
KEY: Nursing Process Step: Implementation
MSC: NCLEX: Physiological Integrity: Pharmacological Therapies

8. A patient who is to receive a daily medication by the oral route has had nausea and vomiting
for the last 24 hours. The best action to ensure that the patient receives the scheduled dose is
to:
a. have the patient take the pill with sips of water.
b. have the patient take the pill with crackers.
c. acquire an order to administer by the rectal or parenteral route.
d. withhold the dose for 1 hour and see whether the nausea subsides.
ANS: C
When a patient is experiencing nausea and vomiting, the nurse can consult with the primary
care provider to get an order for the drug to be changed to the rectal or parenteral route, as
long as the drug is also supplied in that form.

DIF: Cognitive Level: Analysis REF: p. 629


OBJ: Clinical Practice #4 TOP: Safe Drug Administration
KEY: Nursing Process Step: Implementation
MSC: NCLEX: Physiological Integrity: Pharmacological Therapies

9. A hospitalized 3-year-old toddler is to receive an oral medication. For the most effective
approach, the nurse should tell the child:
a. firmly that the drug is important to take as soon possible.
b. in a confident manner what the medication is for and how it will be given.
c. that the medication is candy and tastes good.
d. that it will make him feel better right away.
ANS: B

Downloaded by: SUCCEEDGRADES | abbieclin@gmail.com


Distribution
Downloaded by Dallen Mae D.ofKadir
this document is illegal
(kadir.dallenmae.d.bcsi@gmail.com)
lOMoARcPSD|35346190

Stuvia.com - The Marketplace to Buy and Sell your Study Material

The best approach is to confidently explain to the child what the drug is for and how it will
be given using simple language and short sentences.

DIF: Cognitive Level: Application REF: p. 631 OBJ: Theory #4


TOP: Drug Administration for Children KEY: Nursing Process Step: Implementation
MSC: NCLEX: Physiological Integrity: Basic Care and Comfort

10. An older adult patient with arthritis who has been taking anti-inflammatory drugs for the last
5 years should be monitored for:
a. dizziness and fever.
b. abdominal cramps and bloating.
c. restlessness and dyspnea.
d. gastrointestinal (GI) bleeding and anemia.
ANS: D
Older patients who are on long-term anti-inflammatory therapy for arthritis should be
monitored for GI bleeding and anemia.

DIF: Cognitive Level: Application REF: p. 632


OBJ: Clinical Practice #1
TOP: Medication Administration and Safety for an Older Adult
KEY: Nursing Process Step: Implementation
MSC: NCLEX: Health Promotion and Maintenance: Prevention and Early Detection of
Disease

11. An older adult patient who lives in a skilled nursing facility and who likes to walk is taking
a medication that lowers blood pressure by dilating blood vessels. The best nursing action
for this patient is to:
a. suggest total bed rest.
b. monitor intake and output.
c. assist the patient when ambulating in the hall.
d. instruct the resident to rise slowly when getting out of bed or a chair.
ANS: D
Older adult patients are likely to have greater blood pressure fluctuations with position
changes and are more susceptible to falls when taking drugs that cause orthostatic
hypotension. Assistance with ambulation may offer safety, but if the patient has already
fallen when getting out of bed or a chair, assisted ambulation is pointless.

DIF: Cognitive Level: Application REF: p. 632 OBJ: Theory #4


TOP: Medication Administration and Safety for an Older Adult
KEY: Nursing Process Step: Implementation
MSC: NCLEX: Physiological Integrity: Reduction of Risk

12. The nurse explains that a drug may have several names. The trade name is the only name
that can be:
a. used in an order.
b. trademarked.
c. recognized as its chemical makeup.
d. used by retailers to sell the drug.

Downloaded by: SUCCEEDGRADES | abbieclin@gmail.com


Distribution
Downloaded by Dallen Mae D.ofKadir
this document is illegal
(kadir.dallenmae.d.bcsi@gmail.com)
lOMoARcPSD|35346190

Stuvia.com - The Marketplace to Buy and Sell your Study Material

ANS: B
The trade name of a drug is patented or trademarked. Generic names cannot be trademarked
and are frequently cheaper to purchase under that name. The chemical name is one that
identifies the compounds in the drug. Retailers can sell the drug by either name.

DIF: Cognitive Level: Knowledge REF: p. 623 OBJ: Theory #1


TOP: Trademarked Drugs KEY: Nursing Process Step: Planning
MSC: NCLEX: Physiological Integrity: Pharmacological Therapies

13. The nurse is aware that for a drug to be effectively eliminated from the body, the patient
must have a fluid intake of 50 mL/kg/day. The nurse would provide for a patient who
weighs 125 pounds mL of water per day.
a. 1560
b. 899.2
c. 2840.9
d. 3039.1
ANS: C
The patient‘s weight is 125 pounds/2.2 pounds = 56.8 kg 50 mL = 2840.9 mL.

DIF: Cognitive Level: Analysis REF: p. 627 OBJ: Theory #3


TOP: Distribution of Drugs KEY: Nursing Process Step: Planning
MSC: NCLEX: Physiological Integrity: Pharmacological Therapies

14. Before administering a medication to a newly assigned patient, the nurse should determine
why the patient is receiving it by checking the:
a. medication administration record (MAR) or electronic medication administration
record (eMAR).
b. medical history.
c. laboratory test results.
d. intake and output record.
ANS: B
The medical history contains information about the medical problems a patient has, so the
nurse can correlate the reason a drug is being administered.

DIF: Cognitive Level: Application REF: p. 634


OBJ: Clinical Practice #4
TOP: Assessment Before Medication Administration
KEY: Nursing Process Step: Assessment
MSC: NCLEX: Safe, Effective Care Environment: Safety and Infection Control

15. The nurse is going to administer a medication that must be crushed for the patient to take it.
This medication is best given to the patient by:
a. adding it to water.
b. dissolving it in juice.
c. mixing it in applesauce or soft food.
d. sprinkling it on meat or vegetables.
ANS: C

Downloaded by: SUCCEEDGRADES | abbieclin@gmail.com


Distribution
Downloaded by Dallen Mae D.ofKadir
this document is illegal
(kadir.dallenmae.d.bcsi@gmail.com)
lOMoARcPSD|35346190

Stuvia.com - The Marketplace to Buy and Sell your Study Material

A drug that is crushed needs to be mixed in something else, such as applesauce.

DIF: Cognitive Level: Application REF: p. 637


OBJ: Clinical Practice #4 TOP: Medication Preparation
KEY: Nursing Process Step: Implementation
MSC: NCLEX: Physiological Integrity: Basic Care and Comfort

16. When the nurse is administering a medication to a patient, the patient states that the tablet
looks different from the one usually taken. The most prudent action by the nurse would be
to:
a. reassure the patient that the medication is the same as the one ordered.
b. determine why the patient is refusing to take the medication and call the primary
care provider.
c. assess for possible causes of this patient‘s confusion.
d. withhold the dose and verify the drug order.
ANS: D
If a patient questions the dose given, the nurse should stop and verify the order.

DIF: Cognitive Level: Application REF: p. 631


OBJ: Clinical Practice #4 TOP: Safe Drug Administration
KEY: Nursing Process Step: Implementation
MSC: NCLEX: Safe, Effective Care Environment: Safety and Infection Control

17. A patient is due for a 40 mg dose of furosemide (Lasix), at 9:00 AM on May 5, 2013. The
drug label reads ―20 mg per tablet.‖ The tablets in the bottle appear firm and unbroken. The
expiration date on the bottle reads ―April 2, 2013.‖ The best nursing action is to:
a. administer two tablets.
b. administer one half tablet.
c. call the pharmacy to see if 40 mg tablets are available.
d. call the pharmacy for a new bottle of the medication.
ANS: D
The pharmacy should be called, because the medication is past the expiration date and
should not be given to the patient.

DIF: Cognitive Level: Analysis REF: p. 641


OBJ: Clinical Practice #4 TOP: Patients‘ Five Rights
KEY: Nursing Process Step: Implementation
MSC: NCLEX: Safe, Effective Care Environment: Safety and Infection Control

18. A nurse is administering medications to a group of patients. The safest way to identify each
patient is to:
a. call each patient by his given name, ask for his birthday, and compare with the
MAR or eMAR.
b. check the patient‘s name on the wristband and compare it with the MAR or eMAR.
c. check the name and identification number on the wristband and compare them to
the MAR.
d. check the patient‘s identification number on the wristband.

Downloaded by: SUCCEEDGRADES | abbieclin@gmail.com


Distribution
Downloaded by Dallen Mae D.ofKadir
this document is illegal
(kadir.dallenmae.d.bcsi@gmail.com)
lOMoARcPSD|35346190

Stuvia.com - The Marketplace to Buy and Sell your Study Material

ANS: C
The best method is to check both the name and identification number on the wristband and
compare them to the MAR or eMAR.

DIF: Cognitive Level: Application REF: p. 641|Box 33-3


OBJ: Clinical Practice #4 TOP: Patients‘ Five Rights
KEY: Nursing Process Step: Implementation
MSC: NCLEX: Safe, Effective Care Environment: Safety and Infection Control

19. When preparing medications for delivery to an assigned patient, the nurse should check each
medication for accuracy of drug and dose:
a. five times.
b. three times.
c. two times.
d. one time.
ANS: B
Medications should be checked three times to prevent medication errors.

DIF: Cognitive Level: Application REF: p. 641


OBJ: Clinical Practice #4 TOP: Three Medication Checks
KEY: Nursing Process Step: Implementation
MSC: NCLEX: Safe, Effective Care Environment: Safety and Infection Control

20. A nurse is administering a dose of insulin to a patient. To practice nursing safely and
effectively, the nurse should:
a. confirm with the patient the site of the last injection.
b. give the drug before a meal.
c. inject the insulin in the deltoid muscle.
d. have another licensed nurse double check the dose.
ANS: D
The prudent nurse asks another nurse to check the prepared dose of drugs such as insulin,
anticoagulants, and injectable digoxin.

DIF: Cognitive Level: Application REF: p. 641


OBJ: Clinical Practice #4 TOP: Prevention of Medication Errors
KEY: Nursing Process Step: Implementation
MSC: NCLEX: Safe, Effective Care Environment: Safety and Infection Control

21. A patient will be started on furosemide (Lasix). The primary care provider has also ordered
potassium chloride (KCl) 40 mEq. There is a bottle of KCl labeled 45 mEq per 15 mL. How
many milliliters should the patient receive?
a. 10.8 mL
b. 12.6 mL
c. 13.3 mL
d. 14 mL
ANS: C

Downloaded by: SUCCEEDGRADES | abbieclin@gmail.com


Distribution
Downloaded by Dallen Mae D.ofKadir
this document is illegal
(kadir.dallenmae.d.bcsi@gmail.com)
lOMoARcPSD|35346190

Stuvia.com - The Marketplace to Buy and Sell your Study Material

Using the drug problem formula ―desired over on hand‖ (D/H), divide 40 by 45 and
multiply by 15, which equals 13.3 mL (ie, 40/45 = 0.88 15 = 13.33).

DIF: Cognitive Level: Application REF: p. 638


OBJ: Clinical Practice #3 TOP: Drug Calculations
KEY: Nursing Process Step: Implementation
MSC: NCLEX: Physiological Integrity: Basic Care and Comfort

22. When categorizing medications, drug classifications may be defined by the effects of the
drug and:
a. the symptoms the drug relieves.
b. patient tolerance.
c. the nursing implications.
d. the dosage amounts.
ANS: A
Drug classifications may be defined by the effects of the drug, the symptoms the drug
relieves, or the drug‘s desired effect.

DIF: Cognitive Level: Comprehension REF: p. 623 OBJ: Theory #1


TOP: Classification of Medications KEY: Nursing Process Step: N/A
MSC: NCLEX: N/A

23. An arthritic patient will be discharged home with a variety of medications. The best way for
the home health nurse to assist the patient who lives alone in taking his medications is to:
a. ensure the medications are secured with childproof caps.
b. arrange the medication in a user-friendly pill organizer.
c. verbally tell the patient about what to report to the doctor.
d. leave outdated medications in the medicine cabinet for future use.
ANS: B
To ensure that medications are taken on schedule, a pill organizer should be set up for the
patient.

DIF: Cognitive Level: Application REF: p. 632 OBJ: Theory #5


TOP: Medications and Home Care KEY: Nursing Process Step: Implementation
MSC: NCLEX: Health Promotion and Maintenance: Safety and Infection Control

24. An older adult patient visits an outpatient clinic for the third time this month to be seen for
hypertension. The nurse finds that the patient has not taken his blood pressure medication
for 10 days. The most therapeutic response from the nurse would be:
a. ―You really need to take your blood pressure medicine.‖
b. ―Your medicine will keep your blood pressure down in a safe range.‖
c. ―Why haven‘t you taken your blood pressure medicine?‖
d. ―Did you stop adding salt on your food like we asked you to?‖
ANS: B
Being reminded of the drugs‘ benefits and the need to keep a consistent blood level
frequently supports better adherence. ―Why‖ questions and calling for justification are not
therapeutic.

Downloaded by: SUCCEEDGRADES | abbieclin@gmail.com


Distribution
Downloaded by Dallen Mae D.ofKadir
this document is illegal
(kadir.dallenmae.d.bcsi@gmail.com)
lOMoARcPSD|35346190

Stuvia.com - The Marketplace to Buy and Sell your Study Material

DIF: Cognitive Level: Application REF: p. 634 OBJ: Theory #7


TOP: Noncompliance with Medications KEY: Nursing Process Step: N/A
MSC: NCLEX: Physiological Integrity: Basic Care and Comfort

25. The nurse is administering an enteric-coated oral medication to a patient who is unable to
swallow tablets. The best nurse action is to:
a. give the patient extra water to take with the pill.
b. crush the tablet for easier swallowing.
c. discontinue the medication and document why.
d. ask the primary care provider to consider a liquid form.
ANS: D
Enteric-coated tablets should not be crushed for easier swallowing because the coating is
meant to delay absorption. If a liquid form of the drug is available, the nurse should ask the
primary care provider to change the order.

DIF: Cognitive Level: Application REF: p. 640


OBJ: Clinical Practice #1 TOP: Enteric-Coated Tablets
KEY: Nursing Process Step: Implementation
MSC: NCLEX: Physiological Integrity: Pharmacological Therapies

MULTIPLE RESPONSE

1. Identify the concepts considered one of the six rights of medication administration. (Select
all that apply.)
a. The right patient
b. The right time
c. The right route
d. The right dose
e. The right room
f. The right drug
ANS: A, B, C, D, F
Before administering a drug, check the ―six rights‖ of medication administration: the right
drug, the right dose, the right route, the right time, the right patient and the right
documentation.

DIF: Cognitive Level: Comprehension REF: p. 639


OBJ: Clinical Practice #4 TOP: Medication Administration
KEY: Nursing Process Step: N/A MSC: NCLEX: N/A

2. The nurse explains that the common aspirin can be classified as an: (Select all that apply.)
a. antipyretic
b. analgesic
c. anti-inflammatory
d. antibiotic
e. anti-clotting
ANS: A, B, C, E

Downloaded by: SUCCEEDGRADES | abbieclin@gmail.com


Distribution
Downloaded by Dallen Mae D.ofKadir
this document is illegal
(kadir.dallenmae.d.bcsi@gmail.com)
lOMoARcPSD|35346190

Stuvia.com - The Marketplace to Buy and Sell your Study Material

Aspirin is classified as an antipyretic, analgesic, anti-inflammatory, and anti-clotting drug.

DIF: Cognitive Level: Comprehension REF: p. 623 OBJ: Theory #1


TOP: Drug Classification KEY: Nursing Process Step: Implementation
MSC: NCLEX: Physiological Integrity: Pharmacological Therapies

COMPLETION

1. A patient is being prepared for surgery. There is an on call order for meperidine
hydrochloride (Demerol) 75 mg, with 50 mg/mL available. The patient should receive
mL.

ANS:
1.5

Calculate by using the ―desired over on hand‖ (D/H) method for Demerol:
(1) Desired = 75 mg, on hand = 50 mg/mL; (2) 75/50 = 1.5 mL.

DIF: Cognitive Level: Analysis REF: p. 638


OBJ: Clinical Practice #3 TOP: Calculation
KEY: Nursing Process Step: Implementation
MSC: NCLEX: Physiological Integrity: Pharmacological Therapies

2. The nurse must be aware of how drugs enter the body and how they are metabolized and
excreted. The name of this information is .

ANS:
pharmacokinetics

Pharmacokinetics is the study of how drugs enter the body, how they reach their site of
action, and how they are metabolized and excreted.

DIF: Cognitive Level: Knowledge REF: p. 626 OBJ: Theory #3


TOP: Pharmacokinetics KEY: Nursing Process Step: Planning
MSC: NCLEX: Physiological Integrity: Pharmacological Therapies

Downloaded by: SUCCEEDGRADES | abbieclin@gmail.com


Distribution
Downloaded by Dallen Mae D.ofKadir
this document is illegal
(kadir.dallenmae.d.bcsi@gmail.com)
lOMoARcPSD|35346190

Stuvia.com - The Marketplace to Buy and Sell your Study Material

Chapter 34: Administering Oral, Topical, and Inhalant Medications


Williams: deWit's Fundamental Concepts and Skills for Nursing, 8th Edition

MULTIPLE CHOICE

1. Before the nurse administers a liquid medication to an 83-year-old male patient, the nurse
should:
a. assess the swallowing reflex by offering a sip of water.
b. ask the patient if he would prefer to give the medication to himself.
c. mix thoroughly in applesauce or pudding.
d. assess the ability to understand information relative to the medication.
ANS: A
A factor to consider when giving anything orally is the ability of the patient to swallow.

DIF: Cognitive Level: Application REF: p. 647 OBJ: Theory #7


TOP: Routes of Medication Administration
KEY: Nursing Process Step: Implementation
MSC: NCLEX: Physiological Integrity: Reduction of Risk

2. The nurse receives an order to give vitamin D 10 mcg bid. The nurse recognizes that the
abbreviation mcg refers to a measurement in:
a. milligrams.
b. milliequivilants.
c. millileters.
d. micrograms.
ANS: D
The abbreviation mcg refers to a measurement of micrograms.

DIF: Cognitive Level: Knowledge REF: p. 647|Table 24-3


OBJ: Clinical Practice #4 TOP: Abbreviations
KEY: Nursing Process Step: Planning
MSC: NCLEX: Physiological Integrity: Pharmacological Therapies

3. The licensed nurse who is responsible for doing the narcotic count for the shift should count
the medications:
a. alone for accuracy.
b. with any licensed person.
c. with another nurse working on the shift.
d. with a nurse coming on duty for the next shift.
ANS: D
Controlled substances must be counted by one nurse going off duty and one coming on duty
at the change of each shift.

DIF: Cognitive Level: Comprehension REF: p. 652 OBJ: Theory #1


TOP: Counting Controlled Substances KEY: Nursing Process Step: N/A
MSC: NCLEX: N/A

Downloaded by: SUCCEEDGRADES | abbieclin@gmail.com


Distribution
Downloaded by Dallen Mae D.ofKadir
this document is illegal
(kadir.dallenmae.d.bcsi@gmail.com)
lOMoARcPSD|35346190

Stuvia.com - The Marketplace to Buy and Sell your Study Material

4. Before the nurse administers a dose of digoxin (Lanoxin) to a patient, the nurse should
assess:
a. blood pressure.
b. respiratory rate.
c. apical heart rate.
d. level of consciousness.
ANS: C
The apical heart rate is measured before giving a dose of digoxin (Lanoxin) to determine
whether it is safe to give; the apical rate should be greater than 60 beats/min for an adult
patient.

DIF: Cognitive Level: Application REF: p. 657|Skill 34-1


OBJ: Theory #7 TOP: Medication Administration
KEY: Nursing Process Step: Assessment
MSC: NCLEX: Physiological Integrity: Pharmacological Therapies

5. A nurse is administering oral medications to a patient who is having intake and output
(I&O) measured. When giving medications, it is most important to:
a. give the medication with a small piece of cracker or cookie.
b. give the medication with as much fluid as possible.
c. record the fluid taken on the MAR or eMAR.
d. record the fluid taken on the intake record.
ANS: D
When a patient is having I&O measured, the nurse must record all fluid that the patient
drinks while taking medications in the oral intake column of the I&O sheet.

DIF: Cognitive Level: Application REF: p. 653 OBJ: Theory #1


TOP: Medication Administration KEY: Nursing Process Step: Implementation
MSC: NCLEX: Physiological Integrity: Basic Care and Comfort

6. An older adult patient is having difficulty swallowing an enteric-coated tablet for which
there is no liquid form available. To help the patient swallow the dose more easily, the nurse
should:
a. request the patient to tilt the chin down slightly to swallow.
b. crush the pill and administer it in applesauce.
c. use a spoon to place the tablet at the back of the tongue.
d. take only a small sip of water to swallow the tablet.
ANS: A
If older adult patients have difficulty swallowing, they should swallow a sip of water first,
then place the tablet toward the back of the tongue, then take a large sip of water. Finally,
they should place the tongue on the roof of the mouth and swallow with the chin tilted
downward, followed by another sip of water. Enteric-coated medications should not be
crushed.

DIF: Cognitive Level: Application REF: p. 647 OBJ: Theory #7


TOP: Safe Administration of Medications: Older Adults

Downloaded by: SUCCEEDGRADES | abbieclin@gmail.com


Distribution
Downloaded by Dallen Mae D.ofKadir
this document is illegal
(kadir.dallenmae.d.bcsi@gmail.com)
lOMoARcPSD|35346190

Stuvia.com - The Marketplace to Buy and Sell your Study Material

KEY: Nursing Process Step: Implementation


MSC: NCLEX: Pharmacological Integrity: Reduction of Risk

7. A patient complains about the taste of the sublingual nitroglycerin and admits that he
swallows it rather than holding it under his tongue. The nurse explains that sublingual
medications:
a. should not be swallowed because it alters the absorption potential.
b. can be inserted rectally without loss of absorption potential.
c. can be held against the roof of the mouth with the tongue to reduce taste.
d. can be taken between the cheek and tongue to diminish taste.
ANS: A
Sublingual medications are placed under the tongue and they should not be swallowed
because that alters the absorption potential.

DIF: Cognitive Level: Comprehension REF: p. 658 OBJ: Theory #1


TOP: Medication Administration: Sublingual Route
KEY: Nursing Process Step: Implementation
MSC: NCLEX: Physiological Integrity: Pharmacological Therapies

8. To reduce the systemic absorption of eye drops, the nurse should:


a. use finger pressure to close the eyelid tightly.
b. apply slight finger pressure over the lacrimal duct.
c. request the patient tilt the head slightly to the side of the unaffected eye.
d. instruct the patient to widen the eyes in order to increase access to the lacrimal
duct.
ANS: B
Blocking the entrance to the lacrimal duct by placing a finger over it helps reduce systemic
absorption of an eye drop.

DIF: Cognitive Level: Application REF: p. 659|Skill 34-2


OBJ: Clinical Practice #3 TOP: Medication Administration: Optic Route
KEY: Nursing Process Step: Implementation
MSC: NCLEX: Physiological Integrity: Pharmacological Therapies

9. A patient is attempting to put pills in his mouth from a medicine cup and drops one pill on
the bed sheet. The nurse should:
a. retrieve the pill from the linens and allow the patient to take it.
b. scoop up the pill in a soufflé cup and hand the cup to the patient.
c. discard the pill and get another from the dose pack.
d. report the loss of the pill as a medication error.
ANS: C
A pill that is dropped on the dirty linen or the floor must be discarded and replaced from the
dose pack.

DIF: Cognitive Level: Application REF: p. 652 OBJ: Theory #1


TOP: Dropped Pill KEY: Nursing Process Step: Implementation
MSC: NCLEX: Safe, Effective Care Environment: Safety and Infection Control

Downloaded by: SUCCEEDGRADES | abbieclin@gmail.com


Distribution
Downloaded by Dallen Mae D.ofKadir
this document is illegal
(kadir.dallenmae.d.bcsi@gmail.com)
lOMoARcPSD|35346190

Stuvia.com - The Marketplace to Buy and Sell your Study Material

10. For an adult patient who has an order to receive an otic medication, the nurse should plan to
administer it by pulling the pinna:
a. down and forward.
b. up and forward.
c. down and back.
d. up and back.
ANS: D
The pinna of the adult should be pulled up and back, whereas that of a child younger than 3
years of age is pulled slightly down and back.

DIF: Cognitive Level: Application REF: p. 660


OBJ: Clinical Practice #3 TOP: Medication Administration: Otic Route
KEY: Nursing Process Step: Implementation
MSC: NCLEX: Physiological Integrity: Pharmacological Therapies

11. The nurse administering a nasal medication via an atomizer bottle should:
a. leave the other nostril open while giving the medication.
b. have the patient squeeze the bottle while inhaling.
c. have the patient sit up straight.
d. have the patient tilt the head forward.
ANS: B
The proper procedure for using an atomizer bottle is to have the head hyperextended,
holding one nostril closed and squeezing the bottle and inhaling at the same time.

DIF: Cognitive Level: Application REF: p. 661


OBJ: Clinical Practice #2 TOP: Medication Administration: Nasal Route
KEY: Nursing Process Step: Implementation
MSC: NCLEX: Physiological Integrity: Pharmacological Therapies

12. A nurse is providing instructions to a patient about how to use a metered dose inhaler with a
spacer. The nurse should instruct the patient to:
a. lie down while taking the medication.
b. gently roll the canister in the hands to mix the medication.
c. place the mouthpiece of the chamber in the mouth between the teeth and seal the
lips around it. Completely exhale from the nose.
d. try to hold the breath for at least 3 seconds after inhaling the medication.
ANS: C
The patient should sit up or stand to take the medication, shake the canister to mix the
medication with the propellant, place the mouthpiece of the chamber in the mouth between
the teeth and seal the lips around it. Completely exhale from the nose, depress the canister
once to release the medication into the spacer, breathe in slowly and hold the breath for 10
seconds and try not to cough.

DIF: Cognitive Level: Application REF: p. 661 OBJ: Theory #4


TOP: Medication Administration: Inhalant Route
KEY: Nursing Process Step: Implementation

Downloaded by: SUCCEEDGRADES | abbieclin@gmail.com


Distribution
Downloaded by Dallen Mae D.ofKadir
this document is illegal
(kadir.dallenmae.d.bcsi@gmail.com)
lOMoARcPSD|35346190

Stuvia.com - The Marketplace to Buy and Sell your Study Material

MSC: NCLEX: Physiological Integrity: Pharmacological Therapies

13. The nurse explains that the patient with a respiratory disorder can open small airways to
ease respiration effort with the use of:
a. albuterol (Proventil).
b. montelukast (Singulair).
c. ipratropium (Atrovent).
d. beclomethasone (Vanceril).
ANS: A
Albuterol eases respiratory effort by opening the small airways.

DIF: Cognitive Level: Comprehension REF: p. 662|Table 34-6


OBJ: Theory #4 TOP: Use of Inhalants
KEY: Nursing Process Step: Implementation
MSC: NCLEX: Physiological Integrity: Pharmacological Therapies

14. A patient of the Cambodian culture reports that a new medication is not adequate for
treatment because it is:
a. colored red.
b. a smaller size than the older medication.
c. offered before a meal.
d. is in liquid form.
ANS: B
Persons of Cambodian origin believe the size of the medication indicates its curative value.
A small pill does not have as much curative value as a larger one.

DIF: Cognitive Level: Comprehension REF: p. 654 OBJ: Theory #1


TOP: Medication Administration: Cultural Concerns
KEY: Nursing Process Step: Assessment
MSC: NCLEX: Physiological Integrity: Pharmacological Therapies

15. For easier insertion of a rectal suppository, the nurse should position the patient in the
position.
a. knee chest
b. prone
c. left Sims‘
d. dorsal lithotomy
ANS: C
Placing the patient in the left Sims‘ position provides for easier insertion of the suppository
into the rectum.

DIF: Cognitive Level: Comprehension REF: p. 664|Step 34-2


OBJ: Clinical Practice #6
TOP: Medication Administration: Rectal Route
KEY: Nursing Process Step: Planning
MSC: NCLEX: Physiological Integrity: Pharmacological Therapies

Downloaded by: SUCCEEDGRADES | abbieclin@gmail.com


Distribution
Downloaded by Dallen Mae D.ofKadir
this document is illegal
(kadir.dallenmae.d.bcsi@gmail.com)
lOMoARcPSD|35346190

Stuvia.com - The Marketplace to Buy and Sell your Study Material

16. A patient has an order for a nitroglycerin transdermal patch. The best way to ensure proper
administration of this medication is to:
a. apply it behind the ear.
b. rotate sites to avoid skin irritation.
c. place it over a hairy skin area.
d. put the initials on patch when applied.
ANS: B
A nitroglycerin transdermal patch should be applied to an area with good circulation, such
as the chest, shoulders, or upper arm, and should be placed over hairless areas, with the date
and the nurse‘s initials written on the patch. Rotating sites prevents irritation.

DIF: Cognitive Level: Application REF: p. 666|Skill 34-3


OBJ: Clinical Practice #7
TOP: Medication Administration: Transdermal Route
KEY: Nursing Process Step: Implementation
MSC: NCLEX: Physiological Integrity: Pharmacological Therapies

17. The nurse administering nitroglycerin ointment to a patient will:


a. apply with gloves or tongue blade.
b. apply in same area as the old patch.
c. place the paste on the chest and massage it in the skin.
d. inform the patient that the medicinal effect will take about 45 minutes.
ANS: A
The nurse should wear gloves to avoid exposure to nitroglycerin. The area of the old patch
should be cleaned and the new patch placed in another area and not massaged in. The effect
of the patch occurs in about 30 minutes.

DIF: Cognitive Level: Application REF: p. 665|Skill 34-3


OBJ: Clinical Practice #2
TOP: Medication Administration: Topical Route
KEY: Nursing Process Step: Planning
MSC: NCLEX: Physiological Integrity: Pharmacological Therapies

18. When the nurse sees the order for ―Milk of Magnesia 2 tablespoons, qod, hs,‖ the nurse
translates to mean he should give:
a. 1 ounce of Milk of Magnesia every other day at bedtime.
b. 1 1/2 ounces of Milk of Magnesia every day.
c. 2 tablespoons of Milk of Magnesia whenever necessary.
d. 2 ounces of Milk of Magnesia every night.
ANS: A
The order is asking that 3 tablespoons (1 ounce) of Milk of Magnesia be given every other
day at bedtime.

DIF: Cognitive Level: Analysis REF: p. 647|Table 34-3


OBJ: Theory #2 TOP: Abbreviations
KEY: Nursing Process Step: Implementation
MSC: NCLEX: Physiological Integrity: Basic Care and Comfort

Downloaded by: SUCCEEDGRADES | abbieclin@gmail.com


Distribution
Downloaded by Dallen Mae D.ofKadir
this document is illegal
(kadir.dallenmae.d.bcsi@gmail.com)
lOMoARcPSD|35346190

Stuvia.com - The Marketplace to Buy and Sell your Study Material

19. When administering medications to a patient with a feeding tube, the nurse should dissolve
each crushed medication in at least:
a. 30 to 60 mL of water.
b. 20 to 30 mL of water.
c. 15 to 20 mL of water.
d. 5 to 15 mL of water.
ANS: B
Each medication should be dissolved in 20 to 30 mL of water, which does not include the
water used to flush the tube before and after giving medications.

DIF: Cognitive Level: Comprehension REF: p. 667|Skill 34-4


OBJ: Theory #8 TOP: Medication Administration: Feeding Tube
KEY: Nursing Process Step: Planning
MSC: NCLEX: Physiological Integrity: Pharmacological Therapies

20. Data pertaining to a patient‘s medication therapy that the nurse should document in the
nurses‘ notes, in addition to documenting in the medication administration record (MAR or
eMAR), is:
a. medication name and dose.
b. the route of the medication.
c. the time of the medication.
d. medication side effects experienced.
ANS: D
Side effects of medication therapy are documented in the nurses‘ notes.

DIF: Cognitive Level: Comprehension REF: p. 669 OBJ: Theory #3


TOP: Documentation KEY: Nursing Process Step: Assessment
MSC: NCLEX: Physiological Integrity: Pharmacological Therapies

21. A patient on the long-term care unit receives the wrong medication. The charge nurse should
instruct which staff member to complete the incident report?
a. The nurse who administered the wrong medication.
b. The nursing supervisor for the day.
c. The nurse who discovered the error.
d. No one, because the charge nurse should do it.
ANS: C
The nurse who discovers the error reports it and fills out the incident report.

DIF: Cognitive Level: Application REF: p. 669 OBJ: Theory #8


TOP: Medication Errors KEY: Nursing Process Step: N/A
MSC: NCLEX: N/A

22. The primary care provider writes a medication order on the order sheet of the patient. The
order that includes all the necessary information is:
a. 1/5/13 at 0900: warfarin (Coumadin) 1 mg p.o. qd A. Primary Care Provider
b. 1/5/13 Give warfarin 1 tab qd A. Primary Care Provider

Downloaded by: SUCCEEDGRADES | abbieclin@gmail.com


Distribution
Downloaded by Dallen Mae D.ofKadir
this document is illegal
(kadir.dallenmae.d.bcsi@gmail.com)
lOMoARcPSD|35346190

Stuvia.com - The Marketplace to Buy and Sell your Study Material

c. 1/5/13 Coumadin 1 tab p.o. A. Primary Care Provider


d. 0900 Give warfarin (Coumadin) 1 mg p.o. A. Primary Care Provider
ANS: A
A complete medication order includes the full name of the medication, the dose to be given,
the route of administration, how often the medication is to be given, and the date and time
written, as well as the prescriber‘s signature.

DIF: Cognitive Level: Analysis REF: p. 645 OBJ: Theory #1


TOP: Medication Orders KEY: Nursing Process Step: N/A
MSC: NCLEX: N/A

23. The nurse checking the MAR or eMAR finds that an order for an antibiotic is now 8 days
old. The nurse should:
a. check the medications, performing three medication checks.
b. give the ordered medication.
c. contact the primary care provider for a new order.
d. give the medication, then notify the primary care provider.
ANS: C
The nurse contacts the primary care provider for a new order. Antibiotic orders generally
have a 5- to 7-day limit before they need to be renewed.

DIF: Cognitive Level: Application REF: p. 646 OBJ: Theory #1


TOP: Expired Orders KEY: Nursing Process Step: Implementation
MSC: NCLEX: Safe, Effective Care Environment: Safety and Infection Control

24. The nurse is to administer a dissolved medication via feeding tube. After donning gloves
and attaching the irrigation syringe to the tube, the nurse should next:
a. instill the medication into the syringe slowly.
b. draw the medication into the syringe and gently push into the tube.
c. flush the tubing with 15 to 30 mL of tap water and add the medication just as the
water is about to finish.
d. flush the tubing with 15 to 30 mL of sterile water and add the medication just as the
water is about to finish.
ANS: C
The nurse should flush the tubing with tap water and add the medication as the water is
about to finish. Administration of medication into the feeding tube should be done by
gravity instillation, and pressure should be applied gently only if needed to initiate flow.

DIF: Cognitive Level: Application REF: p. 667|Skill 34-4


OBJ: Theory #8 TOP: Medication Administration: Feeding Tube
KEY: Nursing Process Step: Implementation
MSC: NCLEX: Physiological Integrity: Basic Care and Comfort

25. When administering several medications via feeding tube, the nurse should:
a. dilute each medication with at least 40 mL of water.
b. mix each medication individually.
c. mix all medications together to facilitate administration.

Downloaded by: SUCCEEDGRADES | abbieclin@gmail.com


Distribution
Downloaded by Dallen Mae D.ofKadir
this document is illegal
(kadir.dallenmae.d.bcsi@gmail.com)
lOMoARcPSD|35346190

Stuvia.com - The Marketplace to Buy and Sell your Study Material

d. use sterile gloves for the procedure.


ANS: B
Medications should be mixed separately to prevent clumping.

DIF: Cognitive Level: Application REF: p. 667|Skill 34-4


OBJ: Theory #8 TOP: Medication Administration: Feeding Tube
KEY: Nursing Process Step: Implementation
MSC: NCLEX: Physiological Integrity: Pharmacological Therapies

26. In the event of a discrepancy in the count of the narcotics between the day shift and the
evening shift, the day nurse is required to:
a. correct the count to the number of pills counted and sign full name.
b. write a report and give it to the charge nurse with signatures of both nurses.
c. notify the pharmacy of the discrepancy.
d. remain on duty until the miscount is resolved.
ANS: D
The nurse of the ending shift must resolve the discrepancy before leaving the unit.

DIF: Cognitive Level: Application REF: p. 652 OBJ: Theory #1


TOP: Narcotic Discrepancy KEY: Nursing Process Step: N/A
MSC: NCLEX: N/A

MULTIPLE RESPONSE

1. The nurse is aware that medications that should not be crushed and administered through a
feeding tube include: (Select all that apply.)
a. enteric coated.
b. liquid.
c. sublingual.
d. buccal.
e. sustained release.
f. antineoplastics.
ANS: A, C, D, E, F
Many oral medications can be given through a feeding tube. Liquid medications are best,
but if a tablet is crushable or a capsule can be opened and the contents mixed with liquid, or
if the liquid within the capsule can be aspirated with a needle and syringe, then the
medication can be administered through the feeding tube. Medications that should not be
crushed and administered through the tube are sublingual or buccal, enteric coated, or
sustained release preparations or products with a carcinogenic potential (eg, antineoplastics).

DIF: Cognitive Level: Application REF: p. 667|Skill 34-4


OBJ: Theory #6 TOP: Medication Administration
KEY: Nursing Process Step: Planning
MSC: NCLEX: Physiological Integrity: Pharmacological Therapies

2. When applying ophthalmic ointments, the nurse should: (Select all that apply.)
a. fill only the center of the conjunctival sac.

Downloaded by: SUCCEEDGRADES | abbieclin@gmail.com


Distribution
Downloaded by Dallen Mae D.ofKadir
this document is illegal
(kadir.dallenmae.d.bcsi@gmail.com)
lOMoARcPSD|35346190

Stuvia.com - The Marketplace to Buy and Sell your Study Material

b. ask the patient to roll the eye around and from side to side.
c. remove excess ointment from the lid with a cotton ball.
d. ask the patient to close the eyelids tightly to distribute ointment.
e. remove gloves and perform hand hygiene.
ANS: B, C, E
When the nurse administers ophthalmic ointment, the entire conjunctival sac should be
filled, the patient should be asked to close the lids lightly and move eye about to distribute
the medication, the excess ointment should be removed with a cotton ball, and hand hygiene
should be performed.

DIF: Cognitive Level: Application REF: p. 659|Skill 34-2


OBJ: Clinical Practice #2 TOP: Ophthalmic Ointment
KEY: Nursing Process Step: Implementation
MSC: NCLEX: Physiological Integrity: Pharmacological Therapies

COMPLETION

1. There is an order to give a patient 45 mL of Maalox. The nurse should administer


ounces.

ANS:
1.5

There are 30 mL in 1 ounces. Using the D/H formula, the calculation is as follows:
45/30 mL/ounces = 1.5 ounces.

DIF: Cognitive Level: Analysis REF: p. 648|Table 34-4


OBJ: Clinical Practice #2 TOP: Conversions
KEY: Nursing Process Step: N/A MSC: NCLEX: N/A

2. The nurse is aware that a medication error event that causes a patient death or causes serious
injury to a patient is classified as a(n) event.

ANS:
sentinel

The JCAHO views an event that causes the patient death or causes serious injury a sentinel
event and is followed up with a root cause analysis.

DIF: Cognitive Level: Knowledge REF: p. 669 OBJ: Theory #8


TOP: Sentinel Event KEY: Nursing Process Step: N/A
MSC: NCLEX: N/A

Downloaded by: SUCCEEDGRADES | abbieclin@gmail.com


Distribution
Downloaded by Dallen Mae D.ofKadir
this document is illegal
(kadir.dallenmae.d.bcsi@gmail.com)
lOMoARcPSD|35346190

Stuvia.com - The Marketplace to Buy and Sell your Study Material

Chapter 35: Administering Intradermal, Subcutaneous, and Intramuscular Injections


Williams: deWit's Fundamental Concepts and Skills for Nursing, 8th Edition

MULTIPLE CHOICE

1. When the 8-year-old child complains that he does not want to have a ―shot,‖ the nurse
explains that the use of a parenteral route:
a. is the best way to give medicine.
b. will hasten the action of the medication.
c. will take less medicine to make him well.
d. will be painless because the needles are so sharp.
ANS: B
The parenteral route will hasten the action of the medication. Although the equipment is
better, there is still some pain involved in a parenteral application. The parenteral method is
not always the best way to administer a medication.

DIF: Cognitive Level: Application REF: p. 674 OBJ: Theory #2


TOP: Medication Administration: Parenteral
KEY: Nursing Process Step: Implementation
MSC: NCLEX: Physiological Integrity: Pharmacological Therapies

2. To ensure the proper administration of a tuberculin test, the nurse will:


a. use a 3 mL syringe.
b. choose a 21 gauge, 1 inch needle.
c. insert the needle at a 30-degree angle.
d. inject slowly to form a bleb.
ANS: D
An intradermal injection should be done using a 1 mL syringe with a 25, 27, or 29 gauge
needle that is 5/8 inch long. The needle is inserted at a 15 degree angle, and medication is
injected slowly to form a bump or a bleb underneath the skin.

DIF: Cognitive Level: Application REF: p. 675 OBJ: Theory #1


TOP: Medication Administration: Intradermal
KEY: Nursing Process Step: Planning
MSC: NCLEX: Pharmacological Integrity: Pharmacological Therapies

3. The best angle to insert the needle when administering a subcutaneous injection is at an
angle of:
a. 45 to 90 degrees.
b. 30 to 45 degrees.
c. 15 to 30 degrees.
d. 5 to 15 degrees.
ANS: A
The needle is inserted at a 45- or 90-degree angle depending on the needle length and the
size of the patient.

Downloaded by: SUCCEEDGRADES | abbieclin@gmail.com


Distribution
Downloaded by Dallen Mae D.ofKadir
this document is illegal
(kadir.dallenmae.d.bcsi@gmail.com)
lOMoARcPSD|35346190

Stuvia.com - The Marketplace to Buy and Sell your Study Material

DIF: Cognitive Level: Comprehension REF: p. 675 OBJ: Theory #1


TOP: Medication Administration: Subcutaneous
KEY: Nursing Process Step: Planning
MSC: NCLEX: Pharmacological Integrity: Pharmacological Therapies

4. The nurse computes the dose of medication as 2.4 million units of penicillin to be delivered
in 4 mL. The nurse should:
a. give the 4 mL using a 5 mL syringe.
b. inform the charge nurse that the dose is too large to be given IM.
c. divide the dose into two 3 mL syringes and give as a divided dose.
d. ask the primary care provider if another medication can be used.
ANS: C
The maximum number of milliliters that can be injected into the ventrogluteal muscle is 3
mL. If the person has small muscle mass, or if the dose exceeds 3 mL, the dose should be
divided into two doses.

DIF: Cognitive Level: Analysis REF: p. 690 OBJ: Theory #1


TOP: Intramuscular Medication Administration
KEY: Nursing Process Step: Planning
MSC: NCLEX: Pharmacological Integrity: Pharmacological Therapies

5. The nurse understands that the only part of the syringe that can be touched and not
contaminated is the:
a. needle.
b. outside of the barrel.
c. sides of the plunger.
d. tip of the syringe.
ANS: B
The needle, inside of the barrel, sides of the plunger, and tip of the syringe must be kept
sterile.

DIF: Cognitive Level: Knowledge REF: p. 676


OBJ: Clinical Practice #2 TOP: Injection Equipment
KEY: Nursing Process Step: Planning
MSC: NCLEX: Safe, Effective Care Environment: Safety and Infection Control

6. A nurse has just administered a medication to a patient using a syringe that is not a safety
syringe. To dispose of the needle and syringe safely, the nurse should:
a. recap the needle and dispose of it in the trash receptacle.
b. recap the needle and dispose of it in the sharps container.
c. leave the needle uncapped and dispose of it in the trash receptacle.
d. leave the needle uncapped and dispose of it in the sharps container.
ANS: D
Needles are not to be recapped and should be deposited in the sharps container.

DIF: Cognitive Level: Application REF: p. 678


OBJ: Clinical Practice #2 TOP: Preventing Needle Sticks

Downloaded by: SUCCEEDGRADES | abbieclin@gmail.com


Distribution
Downloaded by Dallen Mae D.ofKadir
this document is illegal
(kadir.dallenmae.d.bcsi@gmail.com)
lOMoARcPSD|35346190

Stuvia.com - The Marketplace to Buy and Sell your Study Material

KEY: Nursing Process Step: Implementation


MSC: NCLEX: Safe, Effective Care Environment: Safety and Infection Control

7. When the nurse is preparing to draw medication from an ampule, the proper procedure is to:
a. allow medication to float freely in the body, neck, and stem.
b. wrap the neck with a gauze or alcohol sponge to the open ampule.
c. break the ampule so that it opens toward her.
d. inject air into the ampule to ease the withdrawal of the medication.
ANS: B
The medication should rest in the body of the ampule before being withdrawn, and the neck
should be wrapped to protect the nurse from glass cuts when the ampule is snapped open.

DIF: Cognitive Level: Application REF: p. 679


OBJ: Clinical Practice #3 TOP: Using Ampules
KEY: Nursing Process Step: Implementation
MSC: NCLEX: Safe, Effective Care Environment: Safety and Infection Control

8. When withdrawing medication from an ampule, the best needle to use is a:


a. beveled needle.
b. 1 inch needle.
c. 1 1/2 inch needle.
d. filter needle.
ANS: D
Medication should be withdrawn from an ampule using a filter needle, which prevents small
glass fragments from being drawn into the syringe.

DIF: Cognitive Level: Application REF: p. 679


OBJ: Clinical Practice #3 TOP: Medication via Ampule
KEY: Nursing Process Step: Planning
MSC: NCLEX: Safe, Effective Care Environment: Safety and Infection Control

9. The nurse performs the proper technique when withdrawing medication from the vial by:
a. wiping the rubber stopper with a povidone iodine swab.
b. inserting the needle into the vial at a 90-degree angle.
c. injecting into the vial an amount of air that is equal to the dose.
d. keeping the needle above the level of solution while withdrawing into the syringe.
ANS: C
The vial should be wiped with an alcohol swab before use, the needle should be inserted at a
slight lateral angle to avoid coring the rubber stopper, and an amount of air equal to the dose
should be injected into the vial, while the needle is kept below the level of the solution to
withdraw the dose.

DIF: Cognitive Level: Application REF: p. 680


OBJ: Clinical Practice #3 TOP: Withdrawing Medication from a Vial
KEY: Nursing Process Step: Planning
MSC: NCLEX: Physiological Integrity: Pharmacological Therapies

Downloaded by: SUCCEEDGRADES | abbieclin@gmail.com


Distribution
Downloaded by Dallen Mae D.ofKadir
this document is illegal
(kadir.dallenmae.d.bcsi@gmail.com)
lOMoARcPSD|35346190

Stuvia.com - The Marketplace to Buy and Sell your Study Material

10. A patient has an order to receive two intramuscular injections in the same syringe. The nurse
should initially:
a. determine if the two medications are compatible in the same syringe.
b. obtain a larger syringe that will accommodate both medications.
c. select two syringes to give the medications separately.
d. ask the patient whether he would prefer one or two injections.
ANS: A
The first step is to determine whether the two medications are compatible in the same
syringe.

DIF: Cognitive Level: Analysis REF: p. 681


OBJ: Clinical Practice #5 TOP: Compatibility of Medications
KEY: Nursing Process Step: Planning
MSC: NCLEX: Physiological Integrity: Pharmacological Therapies

11. A patient has an order to receive a mixture of short- and long-acting insulin. The first step to
properly draw them up in the same syringe is to:
a. shake both vials vigorously before use.
b. inject air into the short acting clear insulin.
c. withdraw the short acting clear insulin.
d. inject air into the long acting cloudy insulin.
ANS: D
The vials should be rolled gently to mix the insulin suspension evenly, but they should not
be shaken. Air is injected first into the long acting cloudy insulin vial and then into the short
acting clear insulin vial.

DIF: Cognitive Level: Analysis REF: p. 682


OBJ: Clinical Practice #5
TOP: Combining Short- and Long-Acting Insulin
KEY: Nursing Process Step: Planning
MSC: NCLEX: Physiological Integrity: Pharmacological Therapies

12. A nurse has opened and used part of a new multidose vial. The nurse should:
a. write the current date on the vial.
b. wipe the top of the vial.
c. check the expiration date of the vial.
d. replace the plastic top that covered the vial.
ANS: A
Before replacing a newly opened multidose vial in the medication storage area, the nurse
should write the date the vial was opened, because out of date medication can chemically
change.

DIF: Cognitive Level: Application REF: p. 681


OBJ: Clinical Practice #3 TOP: Multidose Vial Tips
KEY: Nursing Process Step: Planning
MSC: NCLEX: Physiological Integrity: Pharmacological Therapies

Downloaded by: SUCCEEDGRADES | abbieclin@gmail.com


Distribution
Downloaded by Dallen Mae D.ofKadir
this document is illegal
(kadir.dallenmae.d.bcsi@gmail.com)
lOMoARcPSD|35346190

Stuvia.com - The Marketplace to Buy and Sell your Study Material

13. When reinforcing instructions to a patient who will self-administer insulin injections at
home, it is important to remind the patient to:
a. always use a new insulin vial with each dose.
b. rotate injection sites systematically.
c. use a tuberculin syringe to draw up insulin.
d. aspirate before injecting the insulin.
ANS: B
The patient should rotate injection sites systematically to promote absorption and to
decrease tissue irritation.

DIF: Cognitive Level: Application REF: p. 683


OBJ: Clinical Practice #11 TOP: Self-Administering Insulin
KEY: Nursing Process Step: N/A
MSC: NCLEX: Health Promotion and Maintenance

14. The nurse has an order to administer an injection of purified protein derivative (PPD) by the
intradermal route. The maximum amount of medication that can be given using this route is:
a. 0.1 mL.
b. 0.75 mL.
c. 0.5 mL.
d. 0.2 mL.
ANS: A
The maximum dose that can be given via the intradermal route is 0.1 mL.

DIF: Cognitive Level: Knowledge REF: p. 675 OBJ: Theory #7


TOP: Mantoux Test KEY: Nursing Process Step: Planning
MSC: NCLEX: Physiological Integrity: Pharmacological Therapies

15. A nurse has administered a Tuberculin skin test to a patient in the outpatient clinic at 9:00
AM on Monday. The patient should be scheduled to return to the clinic to have the result
read:
a. late Monday afternoon.
b. late Tuesday afternoon.
c. any time on Wednesday.
d. any time on Friday.
ANS: C
The results of the Tuberculin skin test should be read within 48 to 72 hours after injection.

DIF: Cognitive Level: Comprehension REF: p. 684


OBJ: Clinical Practice #8 TOP: Tuberculosis Testing
KEY: Nursing Process Step: Planning
MSC: NCLEX: Physiological Integrity: Pharmacological Therapies

16. A hospitalized patient has an order for subcutaneous heparin. The best location to administer
this medication is the:
a. upper arm.
b. anterior thigh.

Downloaded by: SUCCEEDGRADES | abbieclin@gmail.com


Distribution
Downloaded by Dallen Mae D.ofKadir
this document is illegal
(kadir.dallenmae.d.bcsi@gmail.com)
lOMoARcPSD|35346190

Stuvia.com - The Marketplace to Buy and Sell your Study Material

c. buttock.
d. abdomen.
ANS: D
The optimal site for heparin injection is the abdomen, because this area is not involved in
muscular activity, as are the arms, buttocks, and legs.

DIF: Cognitive Level: Knowledge REF: p. 686


OBJ: Clinical Practice #8 TOP: Heparin Subcutaneous Injection
KEY: Nursing Process Step: Planning
MSC: NCLEX: Physiological Integrity: Pharmacological Therapies

17. When administering an intramuscular injection to an adult patient using the ventrogluteal
site, the nurse should use which landmark to locate the area for injection?
a. The lower end of the trochanter and the knee
b. The upper end of the trochanter and the knee
c. The head of the trochanter and the posterior iliac spine
d. The head of the trochanter and the anterior iliac spine
ANS: D
The head of the trochanter and the anterior iliac spine are the landmarks used to give an
injection in the ventrogluteal site. The ventrogluteal site is the safest in regard to possible
injury to the patient‘s sciatic nerve.

DIF: Cognitive Level: Comprehension REF: p. 691 OBJ: Theory #10


TOP: Landmarks for Injection KEY: Nursing Process Step: Planning
MSC: NCLEX: Physiological Integrity: Reduction of Risk

18. When administering an intramuscular injection for a 4-year-old child, the best site to use is
the:
a. gluteus medius.
b. vastus lateralis.
c. ventrogluteal.
d. dorsogluteal.
ANS: B
The vastus lateralis is the best choice for children younger than 5 years old, because the
gluteal muscle is not well developed.

DIF: Cognitive Level: Comprehension REF: p. 693


OBJ: Clinical Practice #7 TOP: Vastus Lateralis Site
KEY: Nursing Process Step: Planning
MSC: NCLEX: Safe, Effective Care Environment: Safety and Infection Control

19. The most effective nursing actions to decrease discomfort to the patient during a parenteral
injection would be:
a. inserting the needle while the skin is still wet from the alcohol wipe.
b. asking the patient to look at the injection site for learning purposes.
c. using the smallest gauge needle that is appropriate for the site.
d. removing the needle slowly to avoid damaging the tissue.

Downloaded by: SUCCEEDGRADES | abbieclin@gmail.com


Distribution
Downloaded by Dallen Mae D.ofKadir
this document is illegal
(kadir.dallenmae.d.bcsi@gmail.com)
lOMoARcPSD|35346190

Stuvia.com - The Marketplace to Buy and Sell your Study Material

ANS: C
Using the smallest gauge needle that is appropriate for the site is one way to decrease patient
discomfort.

DIF: Cognitive Level: Comprehension REF: p. 694|Box 35-4


OBJ: Clinical Practice #9 TOP: Parenteral Injection
KEY: Nursing Process Step: Planning
MSC: NCLEX: Physiological Integrity: Basic Care and Comfort

20. A patient has a medication order for iron dextran (Imferon) to be given using the Z-track
technique. The rationale for using this method is to:
a. avoid medication irritation.
b. avoid tissue scarring.
c. cause less painful method.
d. protect the sciatic nerve.
ANS: A
Z-track technique should be used with injection of this medication, because it creates a
slanted needle track and avoids seepage of the medication back into subcutaneous or skin
layers.

DIF: Cognitive Level: Comprehension REF: p. 694|Box 35-4


OBJ: Theory #2 TOP: Z-Track Technique
KEY: Nursing Process Step: N/A MSC: NCLEX: N/A

21. A patient asks why the clinic nurse asked him to remain in the clinic for 30 minutes after the
injection of penicillin. The nurse explains that it is part of the standards of care to monitor
for:
a. any pain reaction.
b. bleeding at the site.
c. infection at the site.
d. any allergic reaction.
ANS: D
The nurse should plan to monitor this patient for allergic response for 30 minutes after
giving the first dose of a medication.

DIF: Cognitive Level: Application REF: p. 695


OBJ: Clinical Practice #6 TOP: Allergic Reaction to Parenteral Injection
KEY: Nursing Process Step: Assessment
MSC: NCLEX: Physiological Integrity: Pharmacological Therapies

22. When preparing to reconstitute a medication from a powder form, the nurse should first:
a. use sterile water.
b. vigorously shake the powder prior to reconstituting medication.
c. follow directions on label for diluent to use.
d. discard the vial and the unused medication.
ANS: C
Instructions for the diluent should be followed from the directions on the label.

Downloaded by: SUCCEEDGRADES | abbieclin@gmail.com


Distribution
Downloaded by Dallen Mae D.ofKadir
this document is illegal
(kadir.dallenmae.d.bcsi@gmail.com)
lOMoARcPSD|35346190

Stuvia.com - The Marketplace to Buy and Sell your Study Material

DIF: Cognitive Level: Application REF: p. 681


OBJ: Clinical Practice #4 TOP: Reconstitution
KEY: Nursing Process Step: Implementation
MSC: NCLEX: Pharmacological Integrity: Pharmacological Therapies

23. Which medication order should be documented in the MAR and in the nurses‘ notes after it
is given?
a. Digoxin 0.25 mg PO at 9:00 AM.
b. Demerol 75 mg IM PRN pain.
c. Lasix 40 mg PO twice daily.
d. KCl 20 mEq PO daily.
ANS: B
PRN and STAT orders are recorded in the eMAR, MAR and nurses‘ notes along with the
reason why the medication was given, the result, and the duration of effect of medication.

DIF: Cognitive Level: Application REF: p. 684|Skill 35-3


OBJ: Clinical Practice #11 TOP: Documentation
KEY: Nursing Process Step: N/A MSC: NCLEX: N/A

24. A nurse giving a subcutaneous injection will select a:


a. 3 mL syringe and 22 gauge, 1 1/2 inch needle.
b. 3 mL syringe and 18 gauge, 1 1/2 inch needle.
c. 3 mL syringe and 25 gauge, 5/8 inch needle.
d. 3 mL syringe and 20 gauge, 1 inch needle.
ANS: C
For subcutaneous injection, it is best to use a 25 gauge, 5/8 inch needle.

DIF: Cognitive Level: Application REF: p. 686


OBJ: Clinical Practice #1 TOP: Needle and Syringe Size
KEY: Nursing Process Step: Planning
MSC: NCLEX: Physiological Integrity: Basic Care and Comfort

25. The nurse is educating a patient who weighs 325 pounds on how to administer a
subcutaneous would suggest that the patient would:
a. require a longer needle because of his weight.
b. experience a faster response to the medication.
c. use a 15 degree angle to inject the medication.
d. need extra pressure at the injection site to prevent bleeding.
ANS: A
For the obese patient, the needle length should be longer than the needle length for a thin
person because of excess fatty layers.

DIF: Cognitive Level: Application REF: p. 677


OBJ: Clinical Practice #1 TOP: Subcutaneous Injection
KEY: Nursing Process Step: Implementation
MSC: NCLEX: Physiological Integrity: Basic Care and Comfort

Downloaded by: SUCCEEDGRADES | abbieclin@gmail.com


Distribution
Downloaded by Dallen Mae D.ofKadir
this document is illegal
(kadir.dallenmae.d.bcsi@gmail.com)
lOMoARcPSD|35346190

Stuvia.com - The Marketplace to Buy and Sell your Study Material

26. When administering heparin, the nurse will avoid:


a. using the lower abdomen as an injection site.
b. rotating sites.
c. massaging area for more than 3 seconds.
d. aspirating before injection.
ANS: D
The nurse should not aspirate before the insertion of heparin because evidence does not
support this practice.

DIF: Cognitive Level: Application REF: p. 675 OBJ: Theory #1


TOP: Heparin Injection KEY: Nursing Process Step: Application
MSC: NCLEX: Pharmacological Integrity: Pharmacological Therapies

MULTIPLE RESPONSE

1. When reconstituting a medication from a powder, the nurse will: (Select all that apply.)
a. confirm the type of diluent required.
b. use only a 23-gauge needle to inject a reconstituted medication.
c. thoroughly mix solute with diluent.
d. roll the solute between hands to warm powder.
e. label the medication as to the amount of medication per volume after dilution.
ANS: A, C, E
After confirming the type of diluent required, the solute must be mixed thoroughly with the
recommended diluent, then labeled as to the amount of medication per volume after
dilution.

DIF: Cognitive Level: Application REF: p. 681


OBJ: Clinical Practice #4 TOP: Mixing Medications from Powder
KEY: Nursing Process Step: Implementation
MSC: NCLEX: Physiological Integrity: Pharmacological Therapies

COMPLETION

1. The IM injection site recommended for infants under 12 months of age is the
.

ANS:
vastus lateralis

The vastus lateralis muscle is the site of choice for infants under 12 months for IM
injections.

DIF: Cognitive Level: Knowledge REF: p. 693|Skill 35-3


OBJ: Theory #2 TOP: Medication Administration - Intramuscular
KEY: Nursing Process Step: N/A MSC: NCLEX: N/A

Downloaded by: SUCCEEDGRADES | abbieclin@gmail.com


Distribution
Downloaded by Dallen Mae D.ofKadir
this document is illegal
(kadir.dallenmae.d.bcsi@gmail.com)
lOMoARcPSD|35346190

Stuvia.com - The Marketplace to Buy and Sell your Study Material

ORDERING

1. You are preparing to mix short-acting and long-acting insulin. Prioritize these steps
accordingly. (Separate letters with a comma and space as follows: A, B, C, D, E, F.)
a. Withdraw prescribed amount of short-acting insulin.
b. Inject air into short-acting insulin.
c. Check medications with eMAR and MAR using three medication checks.
d. Choose and prepare site before injection.
e. Withdraw prescribed amount of long-acting insulin.
f. Inject air into long-acting insulin.

ANS:
C, F, B, A, E, D

These are the proper steps to take when mixing short- and long-acting insulin in one syringe.

DIF: Cognitive Level: Analysis REF: p. 682|Skill 35-2


OBJ: Clinical Practice #5
TOP: Mixing Short- and Long-Acting Insulin in One Syringe
KEY: Nursing Process Step: Implementation
MSC: NCLEX: Pharmacological Therapies

Downloaded by: SUCCEEDGRADES | abbieclin@gmail.com


Distribution
Downloaded by Dallen Mae D.ofKadir
this document is illegal
(kadir.dallenmae.d.bcsi@gmail.com)
lOMoARcPSD|35346190

Stuvia.com - The Marketplace to Buy and Sell your Study Material

Chapter 36: Administering Intravenous Solutions and Medications


Williams: deWit's Fundamental Concepts and Skills for Nursing, 8th Edition

MULTIPLE CHOICE

1. The nurse anticipates that the malnourished postoperative 70-year-old patient will receive an
intravenous (IV) infusion of 5% dextrose in 0.45% saline, because it is:
a. isotonic.
b. hypotonic.
c. hypertonic.
d. total parenteral nutrition.
ANS: C
5% Dextrose in 0.45% saline is a hypertonic or high molecular solution and is a frequent
choice for postoperative maintenance fluid.

DIF: Cognitive Level: Knowledge REF: p. 702|Table 36-1


OBJ: Theory #1 TOP: Types of Intravenous Solutions
KEY: Nursing Process Step: Planning
MSC: NCLEX: Physiological Integrity: Physiological Adaptation

2. The nurse is planning for the initiation of a blood transfusion. The type of tubing the nurse
will prepare is a:
a. piggyback set.
b. primary infusion set.
c. controlled volume set.
d. Y administration set.
ANS: D
A Y administration set is used to place the blood on one side and normal saline on the other.
This is necessary so that the blood can be discontinued but the vein can remain open with the
saline in the case of a transfusion reaction or other medically necessary situation.

DIF: Cognitive Level: Application REF: p. 728 OBJ: Clinical Practice #7


TOP: Administration Sets
KEY: Nursing Process Step: Physiological Integrity: Reduction of Risk
MSC: NCLEX: N/A

3. The nurse is aware that as a safety precaution against over hydration, the tubing drip factor set
appropriate for a 6-month-old infant is:
a. 60 gtt/mL.
b. 20 gtt/mL.
c. 15 gtt/mL.
d. 10 gtt/mL.
ANS: A
A microdrip infusion set, which delivers 60 gtt/mL, is used for infants and children.

DIF: Cognitive Level: Comprehension REF: p. 731 OBJ: Theory #3


TOP: Tubing Size KEY: Nursing Process Step: Planning
MSC: NCLEX: Safe, Effective Care Environment: Safety and Infection Control

Downloaded by: SUCCEEDGRADES | abbieclin@gmail.com


Distribution
Downloaded by Dallen Mae D.ofKadir
this document is illegal
(kadir.dallenmae.d.bcsi@gmail.com)
lOMoARcPSD|35346190

Stuvia.com - The Marketplace to Buy and Sell your Study Material

4. The nurse evaluating a piggyback IV setup finds an error in the construction of the fluids.
Which situation would the nurse correct?
a.Secondary bag is hung higher than the primary bag.
b.Primary line clamp is closed.
c.Slide clamp near the insertion site is open.
d.Secondary line clamp is open.
ANS: B
When a medication is given via piggyback setup, the secondary bag is hung slightly higher
than the primary line and, when the secondary infusion finishes, the primary one takes over
again; therefore, all clamps (roller and slide) must be open for the setup to work properly.

DIF: Cognitive Level: Application REF: p. 704 OBJ: Clinical Practice #1B
TOP: Secondary or Piggyback Intravenous Set
KEY: Nursing Process Step: Evaluation
MSC: NCLEX: Physiological Integrity: Physical Care and Comfort

5. The nurse assisting in the initiation of a blood transfusion is aware that the only appropriate
solution to infuse through a parallel infusion set before and after the transfusion is:
a. 5% dextrose in water.
b. 10% dextrose in water.
c. lactated Ringer‘s solution.
d. normal saline.
ANS: D
Normal saline is the only solution used in conjunction with infusion of a blood product.

DIF: Cognitive Level: Comprehension REF: p. 728 OBJ: Clinical Practice #7


TOP: Blood Infusion KEY: Nursing Process Step: Planning
MSC: NCLEX: Physiological Integrity: Reduction of Risk

6. To facilitate the administration of an IV antibiotic every 6 hours to a patient who is


ambulatory, well hydrated, and on a regular diet, the nurse would insert a(n):
a.primary IV line.
b.secondary IV line.
c.intermittent infusion device.
d.central venous line.
ANS: C
Patients who do not require large amounts of fluid but receive intermittent IV medications
benefit from an intermittent infusion device.

DIF: Cognitive Level: Application REF: p. 720 OBJ: Theory #1


TOP: Saline or PRN Lock KEY: Nursing Process Step: Planning
MSC: NCLEX: Physiological Integrity: Pharmacological Therapies

7. A patient is receiving IV fluids through an infusion pump. How often should the nurse check
the functioning of the pump?
a. Every 15 to 30 minutes
b. Every 1 to 2 hours
c. Every 2 to 4 hours

Downloaded by: SUCCEEDGRADES | abbieclin@gmail.com


Distribution
Downloaded by Dallen Mae D.ofKadir
this document is illegal
(kadir.dallenmae.d.bcsi@gmail.com)
lOMoARcPSD|35346190

Stuvia.com - The Marketplace to Buy and Sell your Study Material

d. Once during the shift


ANS: B
An IV infusion pump should be checked every 1 to 2 hours to ensure that it is functioning
properly.

DIF: Cognitive Level: Comprehension REF: p. 707 OBJ: Clinical Practice #1A
TOP: Infusion Pumps KEY: Nursing Process Step: Planning
MSC: NCLEX: Safe, Effective Care Environment: Safety and Infection Control

8. When a patient receiving IV medication exhibits light headedness, tightness in the chest,
flushed face, and irregular pulse, the nurse suspects:
a. speed shock.
b. drug allergy.
c. fluid overload.
d. air embolus.
ANS: A
Light headedness, tightness in the chest, flushed face, and irregular pulse are all signs of speed
shock. Speed shock is when a foreign substance is infused into the body rapidly. The infusion
should be stopped, the primary care provider notified, and the patient monitored.

DIF: Cognitive Level: Analysis REF: p. 710 OBJ: Theory #3


TOP: Intravenous Catheters KEY: Nursing Process Step: Intervention
MSC: NCLEX: Safe, Effective Care Environment: Safety and Infection Control

9. A patient is admitted with a peripherally inserted central catheter (PICC). As part of standard
care for this patient, the nurse should:
a. obtain the patient‘s temperature every 2 hours.
b. prepare to infuse fluids at high volumes.
c. avoid taking blood pressures on the arm with the PICC line.
d. have the catheter withdrawn while the patient is hospitalized.
ANS: C
PICC lines are inserted by physicians or specially trained nurses, and they are used for long-
term therapy; blood pressures are not taken in the arm that has the PICC line to avoid
interfering with the function or the life of the catheter. Many times this catheter is used in
home care.

DIF: Cognitive Level: Application REF: p. 708 OBJ: Theory #3


TOP: PICC KEY: Nursing Process Step: Implementation
MSC: NCLEX: Physiological Integrity: Reduction of Risk

10. A patient has just undergone placement of a central venous catheter through the subclavian
vein. When the placement is complete, the nurse should:
a. hang the prescribed fluid at a rate of 1 mL/min.
b. assess the quality of the breath sounds.
c. note the length of the tubing.
d. wait for the results of the chest radiograph before beginning fluids.

ANS: D

Downloaded by: SUCCEEDGRADES | abbieclin@gmail.com


Distribution
Downloaded by Dallen Mae D.ofKadir
this document is illegal
(kadir.dallenmae.d.bcsi@gmail.com)
lOMoARcPSD|35346190

Stuvia.com - The Marketplace to Buy and Sell your Study Material

Correct placement of subclavian catheters must be verified by radiographic studies before any
fluid is infused through them.

DIF: Cognitive Level: Application REF: p. 709 OBJ: Theory #3


TOP: Central Venous Catheter Placement
KEY: Nursing Process Step: Intervention
MSC: NCLEX: Physiological Integrity: Reduction of Risk

11. The nurse observes that the insertion site of an IV catheter looks pale and puffy and the area
feels cool to the touch. The initial action for the nurse should be to:
a. discontinue the infusion and start a new IV site.
b. apply warm compresses to the site.
c. monitor the patient‘s temperature every 4 hours.
d. call the primary care provider and report these findings.
ANS: A
Infiltration is the most common complication of IV therapy, and it occurs when fluid or
medication leaks out of the vein and into the tissue. The infusion should be discontinued
immediately and a new insertion site initiated. Signs are pale, cool skin that is edematous
(puffy).

DIF: Cognitive Level: Application REF: p. 709 OBJ: Theory #3


TOP: Infiltration of Intravenous Fluids KEY: Nursing Process Step: Implementation
MSC: NCLEX: Physiological Integrity: Basic Care and Comfort

12. A patient rings the call bell and states that the IV insertion site is painful. The site is reddened,
warm, and swollen. The nurse assesses that the patient is most likely experiencing:
a. bloodstream infection.
b. catheter embolus.
c. infiltration of the line.
d. phlebitis.
ANS: D
Phlebitis is caused by irritation of the vessel by the needle, cannula, medications, or additives
to IV solution. Typical signs are erythema, warmth, swelling, and tenderness.

DIF: Cognitive Level: Comprehension REF: p. 709 OBJ: Theory #3


TOP: Phlebitis KEY: Nursing Process Step: Assessment
MSC: NCLEX: Physiological Integrity: Basic Care and Comfort

13. A patient receiving TPN fluid therapy experiences an air embolus in the central line. The
nurse should immediately turn the patient onto the:
a. right side and raise the head of the bed.
b. right side and lower the head of the bed.
c. left side and raise the head of the bed.
d. left side and lower the head of the bed.
ANS: D
To anatomically minimize the risk of the air embolus reaching the lungs, the nurse should turn
the patient onto the left side and lower the head of the bed. The primary care provider is
notified immediately.

Downloaded by: SUCCEEDGRADES | abbieclin@gmail.com


Distribution
Downloaded by Dallen Mae D.ofKadir
this document is illegal
(kadir.dallenmae.d.bcsi@gmail.com)
lOMoARcPSD|35346190

Stuvia.com - The Marketplace to Buy and Sell your Study Material

DIF: Cognitive Level: Application REF: p. 710|Table 36-2


OBJ: Theory #3 TOP: Catheter Bolus
KEY: Nursing Process Step: Implementation
MSC: NCLEX: Physiological Integrity: Reduction of Risk

14. The nurse takes into consideration that according to The Joint Commission, the first IV
antibiotics order for a community acquired pneumonia must be administered within:
a. 2 hours.
b. 4 hours.
c. 6 hours.
d. 24 hours.

ANS: B
The Joint Commission suggests that the first IV antibiotic administered for community
acquired pneumonia be administered in the first 6 to 8 hours after admission.

DIF: Cognitive Level: Comprehension REF: p. 723 OBJ: Theory #4


TOP: IV Antibiotics KEY: Nursing Process Step: Planning
MSC: NCLEX: Physiological Integrity: Pharmacological Therapies

15. A nurse is monitoring the status of an older adult patient who is receiving IV therapy.
Indicator of fluid volume overload is suspected when the nurse assesses:
a. crackles in the lung fields.
b. pulse rate of 64 beats/min, irregular.
c. respirations of 16 breaths/min, regular.
d. slight edema to the feet.
ANS: A
Fluid overload is signaled by crackles in the lung fields, increasing pulse rate, and shortness of
breath.

DIF: Cognitive Level: Analysis REF: p. 701|Table 36-2


OBJ: Theory #5 TOP: Elder Care: IV Therapy
KEY: Nursing Process Step: Assessment
MSC: NCLEX: Health Promotion and Maintenance: Prevention and Early Detection of Disease

16. A patient has an order for an infusion of 5% dextrose in 0.45% sodium chloride at a rate of
100 mL/hr IV. The IV tubing has a drop factor of 15 gtt/mL. At how many drops per minute
should the nurse regulate the infusion?
a. 15
b. 17
c. 25
d. 33

ANS: C
The formula for calculating IV flow rates is as follows:
(Amount of solution in mL # of gtts/min)/Time in minutes
(100 15)/60 = 1500/60 = 150/6 = 25 drops/min.

DIF: Cognitive Level: Analysis REF: p. 712|Box 36-2


OBJ: Clinical Practice #3 TOP: IV Calculations
KEY: Nursing Process Step: N/A MSC: NCLEX: N/A

Downloaded by: SUCCEEDGRADES | abbieclin@gmail.com


Distribution
Downloaded by Dallen Mae D.ofKadir
this document is illegal
(kadir.dallenmae.d.bcsi@gmail.com)
lOMoARcPSD|35346190

Stuvia.com - The Marketplace to Buy and Sell your Study Material

17. A nurse accessing the injection port of the IV tubing will ―scrub the hub‖ for:
a. 5 seconds.
b. 10 seconds.
c. 15 seconds.
d. 30 seconds.

ANS: C
The hub of the injection port on a piggyback setup should be scrubbed for 15 seconds.

DIF: Cognitive Level: Application REF: p. 724 OBJ: Clinical Practice #1E
TOP: IV Guidelines KEY: Nursing Process Step: Implementation
MSC: NCLEX: Safe, Effective Care Environment: Safety and Infection Control

18. A patient is to have an IV insertion site changed. The current line is in the lower right forearm.
Which location is contraindicated for the new site?
a. Right upper forearm
b. Right hand
c. Left upper forearm
d. Left hand

ANS: B
A new IV site should not be placed distal to an old site; the right hand is distal to the right
forearm, so it should not be used.

DIF: Cognitive Level: Analysis REF: p. 713 OBJ: Clinical Practice #4


TOP: Changing IV Site KEY: Nursing Process Step: Planning
MSC: NCLEX: Physiological Integrity: Reduction of Risk

19. The nurse would plan to get another nurse to try to obtain a successful venipuncture if the first
nurse was not successful in:
a. five attempts.
b. three attempts.
c. two attempts.
d. one attempt.
ANS: C
If the nurse cannot initiate a patent IV in two attempts, it is good judgment to ask another
nurse to perform the task.

DIF: Cognitive Level: Application REF: p. 716 OBJ: Clinical Practice #4


TOP: Starting an IV KEY: Nursing Process Step: Planning
MSC: NCLEX: Physiological Integrity: Basic Care and Comfort

20. A nurse is aware that for a patient with a continuous IV infusion running, the IV bag should be
changed when only mL of solution remains in the bag.
a. 10 mL
b. 25 mL
c. 50 mL
d. 100 mL
ANS: C

Downloaded by: SUCCEEDGRADES | abbieclin@gmail.com


Distribution
Downloaded by Dallen Mae D.ofKadir
this document is illegal
(kadir.dallenmae.d.bcsi@gmail.com)
lOMoARcPSD|35346190

Stuvia.com - The Marketplace to Buy and Sell your Study Material

When the container has only 50 mL of solution left, the next ordered solution is added to the
setup and the flow begun to prevent air from entering the line.

DIF: Cognitive Level: Comprehension REF: p. 718 OBJ: Clinical Practice #5


TOP: Maintaining an IV KEY: Nursing Process Step: Planning
MSC: NCLEX: Physiological Integrity: Pharmacological Therapies

21. A patient who requires an immediate transfusion of blood has previously signed a consent
form to receive it. The nurse confirms that the consent was signed within the last:
a. 8 hours.
b. 12 hours.
c. 24 hours.
d. 48 to 72 hours.
ANS: D
A consent to receive blood must be signed by the patient, usually no more than 48 to 72 hours
before receiving the blood product.

DIF: Cognitive Level: Knowledge REF: p. 728 OBJ: Clinical Practice #7


TOP: Blood Transfusion Consent Form KEY: Nursing Process Step: Implementation
MSC: NCLEX: Physiological Integrity: Pharmacological Therapies

22. A patient complains of chills, back pain, and shortness of breath a few minutes after the blood
infusion is started. The first thing the nurse should do is:
a. slow down the blood infusion.
b. stop the blood infusion and start the saline.
c. monitor vital signs and call the primary care provider.
d. start low flow oxygen as per facility protocol.
ANS: B
If a transfusion reaction occurs, such as chills, back pain, and shortness of breath or itching,
the nurse should stop the infusion and start the saline to keep the line open.

DIF: Cognitive Level: Application REF: p. 728 OBJ: Theory #6


TOP: Blood Transfusion Reaction KEY: Nursing Process Step: Implementation
MSC: NCLEX: Safe, Effective Care Environment: Safety and Infection Control

23. The LVN/LPN is told by the RN to discontinue an IV line to the patient. The best nursing
action is to:
a. check the primary care provider‘s order.
b. stop the IV flow by clamping the tubing securely.
c. wash hands and don gloves.
d. quickly withdraw the cannula and apply pressure.
ANS: A
Checking the primary care provider‘s order will prevent inadvertently discontinuing the IV
and having to restart it.

DIF: Cognitive Level: Application REF: p. 727 OBJ: Clinical Practice #6


TOP: Discontinuing an IV KEY: Nursing Process Step: Implementation
MSC: NCLEX: Physiological Integrity: Basic Care and Comfort

Downloaded by: SUCCEEDGRADES | abbieclin@gmail.com


Distribution
Downloaded by Dallen Mae D.ofKadir
this document is illegal
(kadir.dallenmae.d.bcsi@gmail.com)
lOMoARcPSD|35346190

Stuvia.com - The Marketplace to Buy and Sell your Study Material

24. The nurse caring for a patient with an intermittent IV device should:
a. attach continuous fluid infusion to the device.
b. infuse saline or heparin solution to maintain patency.
c. discontinue when the IV medication is finished.
d. reduce patient activity to prevent dislodgement.

ANS: B
The intermittent IV device should be flushed periodically with saline or heparin, depending on
facility policy, to maintain patency, which allows more freedom of movement for the patient.

DIF: Cognitive Level: Application REF: p. 705 OBJ: Clinical Practice #1D
TOP: Medication to Intermittent Intravenous Device
KEY: Nursing Process Step: Implementation
MSC: NCLEX: Physiological Integrity: Pharmacological Therapies

MULTIPLE RESPONSE

1. The primary care provider orders an IV of 5% dextrose in normal saline (0.45% sodium
chloride) to infuse over a 10-hour period. Which of the following actions should the nurse
take? (Select all that apply.)
a. Monitor intake and output (I&O) every shift.
b. Monitor weight daily.
c. Flush with heparin solution intermittently.
d. Monitor lung sounds every 4 hours.
e. Monitor IV site for infiltration.
f. Monitor blood sugar levels.
ANS: A, D, E
To monitor fluid overload, it is important to assess lung sounds and I&O. Monitoring the IV
site for infiltration or phlebitis is also critical.

DIF: Cognitive Level: Application REF: p. 700 OBJ: Theory #3


TOP: IV Nursing Care KEY: Nursing Process Step: Implementation
MSC: NCLEX: Physiological Integrity: Reduction of Risk

2. The nurse is aware that the disadvantages of infusion pumps include: (Select all that apply.)
a. a saline lock is required.
b. infusion pump change out every shift.
c. the initial expense of machines.
d. an alarm that can be deactivated by family.
e. the need for special administration sets.

ANS: C, D, E
Infusion pumps have some disadvantages such as their initial expense, the need for special
administration sets, and the fact that the alarm button sounds when the IV container is empty,
when there is air in the line, and when there is an occlusion.

DIF: Cognitive Level: Comprehension REF: p. 706 OBJ: Theory #3


TOP: Infusion Pumps KEY: Nursing Process Step: Planning
MSC: NCLEX: Physiological Integrity: Reduction of Risk

Downloaded by: SUCCEEDGRADES | abbieclin@gmail.com


Distribution
Downloaded by Dallen Mae D.ofKadir
this document is illegal
(kadir.dallenmae.d.bcsi@gmail.com)
lOMoARcPSD|35346190

Stuvia.com - The Marketplace to Buy and Sell your Study Material

3. Signs that would cause the nurse to discontinue a blood transfusion would include: (Select all
that apply.)
a. hives.
b. facial flushing.
c. nosebleed.
d. back pain.
e. bloody colored urine.
ANS: A, B, D
Symptoms such as hives, facial flushing, back pain, itching, chills, apprehension, and fever
are the most common reactions. Many times the primary care provider will order Benadryl for
the itching or hives and allow the blood to run.

DIF: Cognitive Level: Comprehension REF: p. 713 OBJ: Theory #6


TOP: Blood Reaction KEY: Nursing Process Step: Assessment
MSC: NCLEX: Physiological Integrity: Reduction of Risk

COMPLETION

1. A patient has an IV of 1000 mL 5% dextrose in 1/2 normal saline (0.45% sodium chloride)
infusing via microdrip for 12 hours. The IV is infusing gtt/min.

ANS:
83

1000/12 = 83.

DIF: Cognitive Level: Analysis REF: p. 712 OBJ: Clinical Practice #2


TOP: Intravenous Medication Administration
KEY: Nursing Process Step: Intervention
MSC: NCLEX: Physiological Integrity: Pharmacological Therapies

2. The nurse instills diluted medication in the portion of the controlled volume IV setup, which is
called the .

ANS:
burette

The burette is the tube-like chamber that holds only about 150 mL of fluid with diluted
medication.

DIF: Cognitive Level: Knowledge REF: p. 705 OBJ: Theory #1


TOP: Burette KEY: Nursing Process Step: Implication
MSC: NCLEX: Physiological Integrity: Reduction of Risk

3. After the blood infusion has started, the nurse should let the blood flow at 2 mL/min for the
first minutes.

ANS:
15

Downloaded by: SUCCEEDGRADES | abbieclin@gmail.com


Distribution
Downloaded by Dallen Mae D.ofKadir
this document is illegal
(kadir.dallenmae.d.bcsi@gmail.com)
lOMoARcPSD|35346190

Stuvia.com - The Marketplace to Buy and Sell your Study Material

The initial rate of blood infusion is 2 mL/min for the first 15 minutes. If the patient tolerates
this rate, it can be gradually increased.

DIF: Cognitive Level: Comprehension REF: p. 712 OBJ: Clinical Practice #7


TOP: Monitoring Blood KEY: Nursing Process Step: Implementation
MSC: NCLEX: Physiological Integrity: Reduction of Risk

ORDERING

1. Place the steps in order for the preparation to initiate a blood line: (Separate letters by a
comma and space as follows: A, B, C, D, E.)
a. Compare patient name, ID number on wrist bank with transfusion record.
b. Obtain Y connector setup and saline and prime the filter with saline.
c. Clamp off saline and start blood.
d. Confirm the presence of a permission slip.
e. Obtain baseline vital signs.

ANS:
D, A, B, E, C

All permission slips and then identification must be accomplished prior to starting blood. The
blood information is checked with two licensed persons. The filter is primed and the saline is
started; vital signs are taken and then the blood is started.

DIF: Cognitive Level: Application REF: p. 728 OBJ: Theory #1


TOP: Preparation of Transfusion KEY: Nursing Process Step: Implementation
MSC: NCLEX: Safe, Effective Care Environment: Safety and Infection Control

Downloaded by: SUCCEEDGRADES | abbieclin@gmail.com


Distribution
Downloaded by Dallen Mae D.ofKadir
this document is illegal
(kadir.dallenmae.d.bcsi@gmail.com)
lOMoARcPSD|35346190

Stuvia.com - The Marketplace to Buy and Sell your Study Material

Chapter 37: Care of the Surgical Patient


Williams: deWit's Fundamental Concepts and Skills for Nursing, 8th Edition

MULTIPLE CHOICE

1. The circulating nurse notes that the anesthetized patient has tensed muscles and irregular
respirations. The nurse is aware that the patient has reached stage:
a. I and the patient‘s hearing is amplified.
b. II and the surgical environment should be kept quiet.
c. III and the patient has depressed reflexes.
d. IV and the patient will not depend on the anesthesia machine of oxygenations.
ANS: B
Quiet must be maintained while the patient is in stage II, because noise may cause the patient
to become excited, resulting in instability of vital signs.

DIF: Cognitive Level: Comprehension REF: p. 736 OBJ: Theory #5


TOP: General Anesthesia KEY: Nursing Process Step: Implementation
MSC: NCLEX: Physiological Integrity: Basic Care and Comfort

2. In order for the anesthesiologist to accurately calculate the amount of anesthesia needed for an
82-year-old patient, the nurse should have data available on the medical record, such as:
a. time of last meal.
b. pulse rate and blood pressure.
c. respiratory rate and oxygen saturation.
d. height and weight.
ANS: D
Accurate height and weight are significant for calculation of the anesthetic agents needed for
older adult patients.

DIF: Cognitive Level: Comprehension REF: p. 736 OBJ: Theory #3


TOP: Anesthesia KEY: Nursing Process Step: Assessment
MSC: NCLEX: Physiological Integrity: Pharmacological Therapies

3. The illiterate patient signs the surgical consent form with an ―X.‖ The nurse is aware that this
―X‖ is:
a.not an acceptable signature.
b.required to be accompanied by a picture identification.
c.legal if it is witnessed.
d.acceptable if the surgeon is willing.
ANS: C
An ―X‖ is an acceptable signature if it is witnessed.

DIF: Cognitive Level: Comprehension REF: p. 737 OBJ: Theory #8


TOP: Surgical Consent KEY: Nursing Process Step: Implementation
MSC: NCLEX: Safe, Effective Care Environment: Coordinated Care

https://www.coursehero.com/file/30335441/c37rtf/

Downloaded by: SUCCEEDGRADES | abbieclin@gmail.com


Distribution
Downloaded by Dallen Mae D.ofKadir
this document is illegal
(kadir.dallenmae.d.bcsi@gmail.com)
lOMoARcPSD|35346190

Stuvia.com - The Marketplace to Buy and Sell your Study Material

4. A 16-year-old boy injured in a motorcycle accident is unconscious and cannot sign the
emergency surgical consent form; no family members can be located. The nurse anticipates
that:
a. the hospital social worker can sign the permit.
b. the surgeon will write a detailed note about the need for surgery and a registered
nurse will then sign the consent.
c. the opinion of a second surgeon is sought regarding the necessity of the surgery.
d. the hospital attorney must authorize the surgery and sign the consent form.
ANS: C
If no family can be found to sign the permit for an unconscious person, a second surgeon can
confirm the need for the immediate surgical intervention and the procedure will take place.

DIF: Cognitive Level: Application REF: p. 737 OBJ: Theory #8


TOP: Surgical Consent KEY: Nursing Process Step: Planning
MSC: NCLEX: Safe, Effective Care Environment: Coordinated Care

5. The nurse discovers that the signed operative permit has misspelled the patient‘s name. The
nurse must:
a. request a corrected consent form to be signed.
b. inform the surgeon of the error.
c. have the new form attached to the old incorrect one and document it.
d. allow the patient to be taken to surgery after notifying the circulating nurse.
ANS: A
A new consent form must be made, signed, and witnessed, provided the patient has not been
sedated already.

DIF: Cognitive Level: Application REF: p. 737 OBJ: Theory #8


TOP: Incorrect Surgical Consent Form KEY: Nursing Process Step: Implementation
MSC: NCLEX: Safe, Effective Care Environment: Coordinated Care

6. A patient who has recently arrived on the surgical unit after being transferred from the
postanesthesia care unit asks for a drink. The nurse should first check the:
a. status of the IV fluids.
b. ability to swallow.
c. nursing progress notes.
d. anesthesia record.
ANS: B
Before a postsurgical patient is offered anything to eat or drink, the ability of the patient to
swallow should be assessed.

DIF: Cognitive Level: Application REF: p. 752 OBJ: Clinical Practice #3


TOP: NPO Status KEY: Nursing Process Step: Implementation
MSC: NCLEX: Physiological Integrity: Basic Care and Comfort

7. A patient scheduled for surgery has an order for a preoperative surgical skin preparation. The
nurse may be required to:
a. shave the entire surgical site.
b. spray the surgical area with an antimicrobial solution.
c. scrub the surgical area for 1 minute with antibacterial solution.

https://www.coursehero.com/file/30335441/c37rtf/

Downloaded by: SUCCEEDGRADES | abbieclin@gmail.com


Distribution
Downloaded by Dallen Mae D.ofKadir
this document is illegal
(kadir.dallenmae.d.bcsi@gmail.com)
lOMoARcPSD|35346190

Stuvia.com - The Marketplace to Buy and Sell your Study Material

d. instruct the patient in the use of an antimicrobial soap in the shower.


ANS: D
The nurse may be required to instruct in the use of an antimicrobial soap in the shower.
Although hair may be clipped away from the incision line, this is usually not done by the floor
nurse but accomplished in the surgery suite.

DIF: Cognitive Level: Application REF: p. 741 OBJ: Clinical Practice #2


TOP: Preoperative Surgical Skin Preparation
KEY: Nursing Process Step: Implementation
MSC: NCLEX: Safe, Effective Care Environment: Safety and Infection Control

8. To help prevent errors in the surgical procedure, the National Patient Safety Goal instituted a
directive that prior to presurgical medication, each patient must:
a. verbally state the location of the surgical site and the expected procedure.
b. when still conscious participate in marking the surgical site.
c. have the unit nurse confirm their identity.
d. have a photograph of the surgical site in the medical record.

ANS: B
Prior to presurgical medication, the conscious patient participates in a marking of the body
part and site of the expected surgical procedure.

DIF: Cognitive Level: Comprehension REF: p. 737 OBJ: Theory #6


TOP: Immediate Preoperative Care KEY: Nursing Process Step: Planning
MSC: NCLEX: Safe, Effective Care Environment: Coordinated Care

9. While completing the preoperative checklist, a patient who is almost ready for transport to the
operating room states that he does not want to remove his wedding band. The nurse should:
a. tape it in place on his finger.
b. remind him it must be removed, and lock it in the narcotic cabinet.
c. ask a family member to take care of it.
d. inform him that the hospital cannot be responsible for its loss.
ANS: A
A wedding band may be worn to surgery, but it must be taped to the finger in a manner that
does not restrict circulation.

DIF: Cognitive Level: Application REF: p. 741 OBJ: Clinical Practice #2


TOP: Immediate Preoperative Care KEY: Nursing Process Step: Implementation
MSC: NCLEX: Psychosocial Integrity: Coping and Adaptation

10. The nurse is aware that in both the very young and the older adult surgical patient, the risk is
much higher for:
a.nausea and vomiting.
b.hydration issues.
c.delayed healing.
d.anorexia.
ANS: B
Both the very young and older adults are at greater risk for dehydration or overhydration and
alterations in body temperature control.

https://www.coursehero.com/file/30335441/c37rtf/

Downloaded by: SUCCEEDGRADES | abbieclin@gmail.com


Distribution
Downloaded by Dallen Mae D.ofKadir
this document is illegal
(kadir.dallenmae.d.bcsi@gmail.com)
lOMoARcPSD|35346190

Stuvia.com - The Marketplace to Buy and Sell your Study Material

DIF: Cognitive Level: Comprehension REF: p. 734 OBJ: Theory #2


TOP: Surgical Risks in Young and Older Adult
KEY: Nursing Process Step: Planning
MSC: NCLEX: Physiological Integrity: Reduction of Risk

11. A patient undergoing preadmission testing before same day surgery asks how long he will
remain in the recovery area before being allowed to go home. The nurse‘s most informative
response would be:
a. 30 to 60 minutes.
b. 2 to 6 hours.
c. 5 to 6 hours.
d. 6 to 8 hours.

ANS: B
The usual recovery time in a same day surgery recovery area is 2 to 6 hours.

DIF: Cognitive Level: Knowledge REF: p. 748 OBJ: Theory #7


TOP: Postanesthesia Care KEY: Nursing Process Step: Implementation
MSC: NCLEX: Psychosocial Integrity: Psychosocial Adaptation

12. The circulating nurse is responsible for:


a. preparing the sterile field.
b. assisting with sterile draping of the patient.
c. maintaining an accurate count of sponges.
d. pointing out the observation of contamination immediately to the personnel
involved.
ANS: D
Any break in sterile technique in the operating room should be immediately pointed out and
remedied.

DIF: Cognitive Level: Comprehension REF: p. 748 OBJ: Theory #9


TOP: Circulating Nurse KEY: Nursing Process Step: Planning
MSC: NCLEX: Safe, Effective Care Environment: Safety and Infection Control

13. When the patient is ready to return from the postanesthesia care unit (PACU), a minimal
Aldrete score of is an indicator that the patient is ready to return to the floor.
a. 2 to 4
b. 4 to 8
c. 9 to 10
d. 11 to 12
ANS: C
Using an Aldrete scoring system of activity, respiration, circulation, consciousness, and skin
color being scored from 1 to 3, a score of 9 to 10 is the minimal indicator that the patient is
ready to return to the floor.

DIF: Cognitive Level: Comprehension REF: p. 748 OBJ: Theory #10


TOP: Postanesthesia Care KEY: Nursing Process Step: Assessment
MSC: NCLEX: Physiological Integrity: Physiological Adaptation

https://www.coursehero.com/file/30335441/c37rtf/

Downloaded by: SUCCEEDGRADES | abbieclin@gmail.com


Distribution
Downloaded by Dallen Mae D.ofKadir
this document is illegal
(kadir.dallenmae.d.bcsi@gmail.com)
lOMoARcPSD|35346190

Stuvia.com - The Marketplace to Buy and Sell your Study Material

14. A patient who has returned to the surgical nursing unit from the postanesthesia care unit
(PACU) is drowsy and requires verbal stimulation to remain aroused. The best position to
maintain an airway for this patient is:
a. supine.
b. side lying.
c. head of bed at 30 degrees with head and neck midline.
d. head of bed at 45 degrees with head and neck midline.

ANS: B
The patient should be positioned on the side or with the head turned to the side to prevent
aspiration. Maintaining an open airway is a priority measure.

DIF: Cognitive Level: Application REF: p. 751 OBJ: Theory #10


TOP: Safety: Airway KEY: Nursing Process Step: Implementation
MSC: NCLEX: Health Promotion and Maintenance: Prevention and Early Detection of Disease

15. The nurse is assessing the surgical dressing of a patient who arrived on the unit an hour ago.
The surgical dressing has serosanguineous drainage on the dressing. The nurse should:
a.make a note of the drainage on the worksheet to report it at the end of shift.
b.change the surgical dressing immediately to prevent infection.
c.outline the area of drainage with a pen and mark it with the date and time.
d.reinforce the dressing with clean gauze sponges and tape.
ANS: C
The area should be outlined, dated, and timed for future reference and comparisons.

DIF: Cognitive Level: Application REF: p. 751 OBJ: Theory #2


TOP: Safety: Bleeding KEY: Nursing Process Step: Implementation
MSC: NCLEX: Physiological Integrity: Reduction of Risk

16. A nurse is instructing a patient who had surgical removal of a brain tumor on how to prevent
respiratory complications from surgery. The nurse would teach the patient to:
a. turn, cough, and deep breathe.
b. use humidified oxygen.
c. turn gently from side to side.
d. use deep breathing and an incentive spirometer.
ANS: D
Coughing may be contraindicated for patients who have had hernia repair, eye, ear, or brain
surgery. This is because the act of coughing could create increased pressure in the surgical
area, which is contraindicated. Huffing or the use of an incentive spirometer is effective for
postsurgical respiratory health.

DIF: Cognitive Level: Application REF: p. 751 OBJ: Clinical Practice #4


TOP: Promoting Respiratory Function KEY: Nursing Process Step: N/A
MSC: NCLEX: Physiological Integrity: Reduction of Risk Potential

17. A postoperative surgical patient asks how the sequential pneumatic compression boots applied
in the operating room will help lower the risk of blood clots forming in the legs. The nurse‘s
most appropriate response would be that the boots:
a. measure pressure in the leg blood vessels and sound an alarm if pressure rises.
b. alternately compress and release to help blood flow through vessels.

https://www.coursehero.com/file/30335441/c37rtf/

Downloaded by: SUCCEEDGRADES | abbieclin@gmail.com


Distribution
Downloaded by Dallen Mae D.ofKadir
this document is illegal
(kadir.dallenmae.d.bcsi@gmail.com)
lOMoARcPSD|35346190

Stuvia.com - The Marketplace to Buy and Sell your Study Material

c. provide gentle continuous compression at low pressure.


d. provide firm continuous compression at high pressure.

ANS: B
Pneumatic boots alternately compress and release to squeeze the blood vessels and thus propel
blood through the vessels back to the heart.

DIF: Cognitive Level: Comprehension REF: p. 752 OBJ: Theory #10


TOP: Promoting Circulation KEY: Nursing Process Step: Implementation
MSC: NCLEX: Physiological Integrity: Reduction of Risk

18. A nurse is monitoring the urinary drainage from a patient who returned to the unit a few hours
ago from the postanesthesia care unit (PACU) following a surgical procedure. The urine total
is 54 mL for the last 2 hours. The most appropriate nursing action is to:
a. increase the flow rate of the IV for 10 to 15 minutes.
b. irrigate the indwelling urinary catheter.
c. apply manual pressure to the patient‘s bladder.
d. notify the surgeon of the findings.
ANS: D
If the urinary flow rate is lower than 60 mL for a 2-hour period, the surgeon is notified.

DIF: Cognitive Level: Analysis REF: p. 752 OBJ: Theory #10


TOP: Inadequate Urine Output KEY: Nursing Process Step: Implementation
MSC: NCLEX: Health Promotion and Maintenance: Prevention and Early Detection of Disease

19. A patient who had surgery earlier in the day using general anesthesia asks whether he can
have something to eat. The diet order indicates clear liquids can be taken. Before giving a Jell
O to the patient, the nurse should check for the presence of:
a. clear lung sounds.
b. adequate urinary drainage.
c. bowel sounds in all quadrants.
d. palpable peripheral pulses.

ANS: C
Before allowing a patient to eat or drink after surgery, the nurse must ensure that bowel
sounds are present in all four quadrants. This is because of the risk of paralytic ileus (lack of
return of peristalsis) after surgery.

DIF: Cognitive Level: Analysis REF: p. 754 OBJ: Theory #10


TOP: Maintaining Fluid Balance KEY: Nursing Process Step: Implementation
MSC: NCLEX: Physiological Integrity: Reduction of Risk

20. A patient who had abdominal surgery is complaining of ―gas pains‖ and has distention of the
abdomen and flatus. To promote patient comfort, the nurse should advise:
a. early ambulation.
b. turning to the left side.
c. drinking fluids that are very hot.
d. lying supine with knees flexed.
ANS: A

https://www.coursehero.com/file/30335441/c37rtf/

Downloaded by: SUCCEEDGRADES | abbieclin@gmail.com


Distribution
Downloaded by Dallen Mae D.ofKadir
this document is illegal
(kadir.dallenmae.d.bcsi@gmail.com)
lOMoARcPSD|35346190

Stuvia.com - The Marketplace to Buy and Sell your Study Material

Ambulation is helpful in expelling gas. Taking large amounts of food or liquid at a time, and
drinking fluids that are either very hot or cold, can aggravate the symptoms.

DIF: Cognitive Level: Application REF: p. 755 OBJ: Clinical Practice #5


TOP: Promoting Comfort KEY: Nursing Process Step: Implementation
MSC: NCLEX: Physiological Integrity: Basic Care and Comfort

21. A nurse preparing to get a patient out of bed for the first time since surgery will initially:
a. assist the patient to sit and dangle his or her legs on the side of the bed.
b. allow the patient to sit with the head of bed raised to the high Fowler‘s position.
c. assist the patient from a supine position to a standing position.
d. place a walker at the side of the bed.
ANS: B
The first step is to raise the head of the bed and let the body adjust to the position change.
After a few minutes, the patient can be assisted to sit on the side of the bed with his or her legs
dangling (with feet on floor). Finally, the patient is assisted to a standing position.

DIF: Cognitive Level: Application REF: p. 755 OBJ: Clinical Practice #6


TOP: Rest and Activity KEY: Nursing Process Step: Implementation
MSC: NCLEX: Physiological Integrity: Reduction of Risk

22. A patient is ready for discharge following same day surgery. The education plan for this
patient includes:
a. limiting wine intake for 12 hours.
b. reporting temperatures greater than 99° F.
c. not driving or making important decisions for 24 hours.
d. that vomiting might be an expected symptom.
ANS: C
Patients should not drive or make important decisions for at least 24 hours after surgery, until
all residual effects of anesthesia have worn off.

DIF: Cognitive Level: Application REF: p. 735 OBJ: Clinical Practice #7


TOP: Same Day Surgery Discharge KEY: Nursing Process Step: Planning
MSC: NCLEX: Physiological Integrity: Physiological Adaptation

MULTIPLE RESPONSE

1. The nurse explains that the informed surgical consent form should include information
relative to: (Select all that apply.)
a. the procedure to be performed.
b. the related risks of the procedure.
c. consent to blood transfusion.
d. time and date signed.
e. marital status.
ANS: A, B, D
The consent must include the procedure to be performed and the related risks of the
procedure; it must also be timed, dated, and witnessed. Consent for transfusion is another
permit entirely.

https://www.coursehero.com/file/30335441/c37rtf/

Downloaded by: SUCCEEDGRADES | abbieclin@gmail.com


Distribution
Downloaded by Dallen Mae D.ofKadir
this document is illegal
(kadir.dallenmae.d.bcsi@gmail.com)
lOMoARcPSD|35346190

Stuvia.com - The Marketplace to Buy and Sell your Study Material

DIF: Cognitive Level: Comprehension REF: p. 737 OBJ: Theory #38


TOP: Surgical Consent KEY: Nursing Process Step: Implementation
MSC: NCLEX: Physiological Integrity: Reduction of Risk

2. The nurse preparing the surgical patient‘s room for the patient‘s return from the postanesthesia
unit should: (Select all that apply.)
a. fan fold the sheets on the near side of the bed.
b. lower the bed for easy transfer of the patient.
c. place an IV pole at the head of the bed.
d. gather an emesis basin, tissues, and a small towel.
e. collect extra dressing supplies and place them on the bedside table.

ANS: C, D
The postsurgical room should be prepared with the sheets fan folded on the far side of the
heightened bed. IV poles, emesis basins, oxygen, and suction equipment should be ready to
use; a thermometer, sphygmomanometer, and stethoscope should be available.

DIF: Cognitive Level: Application REF: p. 749 OBJ: Theory #3


TOP: Room Preparation KEY: Nursing Process Step: Implementation
MSC: NCLEX: Physiological Integrity: Basic Care and Comfort

COMPLETION

1. A postoperative patient experiences separation of the layers of his abdominal surgical wound
during coughing. Separation of the layers of a surgical wound is known as .

ANS:
dehiscence

Dehiscence is separation of the layers of the surgical wound that may occur when the patient
is coughing, particularly if the abdominal incision is not properly splinted.

DIF: Cognitive Level: Knowledge REF: p. 755 OBJ: Theory #2


TOP: Postoperative Care KEY: Nursing Process Step: N/A
MSC: NCLEX: N/A

2. The patient informs the admitting nurse that she has been drinking feverfew tea for the herbal
treatment of migraine headaches. The nurse reports this to the surgeon because this herb can
cause .

ANS:
bleeding

The herb feverfew can inhibit platelet aggregation and increase the possibility of bleeding.

DIF: Cognitive Level: Comprehension REF: p. 732 OBJ: Theory #2


TOP: Herbal Substances KEY: Nursing Process Step: Assessment
MSC: NCLEX: Physiological Integrity: Reduction of Risk

https://www.coursehero.com/file/30335441/c37rtf/

Downloaded by: SUCCEEDGRADES | abbieclin@gmail.com


Distribution
Downloaded by Dallen Mae D.ofKadir
this document is illegal
(kadir.dallenmae.d.bcsi@gmail.com)
lOMoARcPSD|35346190

Stuvia.com - The Marketplace to Buy and Sell your Study Material

3. To help prevent surgical site infections in the diabetic patient, the glycemic should be
maintained with a blood glucose of less than mg/dL.

ANS:
200

A diabetic patient can be better protected from surgical site infection by maintaining blood
glucose below 200 mg/dL.

DIF: Cognitive Level: Comprehension REF: p. 735|Box 37-1


OBJ: Theory #3 TOP: Glucose Control KEY: Nursing Process Step: N/A
MSC: NCLEX: N/A

4. The nurse reminds the patient that the member of the surgical team who is responsible for
obtaining the surgical consent is the .

ANS:
surgeon

The surgeon is the person responsible for obtaining an informed surgical consent.

DIF: Cognitive Level: Knowledge REF: p. 737 OBJ: Theory #8


TOP: Surgical Consent Form KEY: Nursing Process Step: Implementation
MSC: NCLEX: Psychosocial Integrity: Coping and Adaptation

https://www.coursehero.com/file/30335441/c37rtf/

Downloaded by: SUCCEEDGRADES | abbieclin@gmail.com


Distribution
Downloaded by Dallen Mae D.ofKadir
this document is illegal
(kadir.dallenmae.d.bcsi@gmail.com)
lOMoARcPSD|35346190

Stuvia.com - The Marketplace to Buy and Sell your Study Material

Chapter 38: Providing Wound Care and Treating Pressure Ulcers


Williams: deWit's Fundamental Concepts and Skills for Nursing, 8th Edition

MULTIPLE CHOICE

1. The nurse clarifies that the first stage of wound healing is:
a. proliferation.
b. maturation.
c. reconstruction.
d. inflammation.

ANS: D
Inflammation is the first stage of wound healing, followed by the proliferation, maturation,
and reconstruction stages.

DIF: Cognitive Level: Knowledge REF: p. 761 OBJ: Theory #1


TOP: Inflammatory Process KEY: Nursing Process Step: Implementation
MSC: NCLEX: Physiological Integrity: Physiological Adaptation

2. The nurse is taking care of a postsurgical patient and notes the incision is clean and dry, with
sutures intact. The nurse further assesses that the wound is healing by:
a. fourth intention.
b. third intention.
c. second intention.
d. first intention.
ANS: D
A wound with minimal tissue loss, such as a surgical incision, heals by closure, which is first,
or primary, intention. Wounds that are not closed heal by either second (secondary) or third
(tertiary) intention.

DIF: Cognitive Level: Comprehension REF: p. 762 OBJ: Theory #1


TOP: Wound Healing Stages KEY: Nursing Process Step: Assessment
MSC: NCLEX: Physiological Integrity: Physiological Adaptation

3. The nurse gives an example of a wound that heals by second (secondary) intention as a:
a. laceration with edges that do not approximate.
b. surgical incision closed with staples.
c. chest wound left open for a closed system.
d. puncture wound sutured with silk suture.
ANS: A
A secondary intention healing occurs when there is a jagged wound whose edges do not
approximate.

DIF: Cognitive Level: Comprehension REF: p. 762 OBJ: Theory #1


TOP: Wound Types KEY: Nursing Process Step: Implementation
MSC: NCLEX: Physiological Integrity: Physiological Adaptation

Downloaded by: SUCCEEDGRADES | abbieclin@gmail.com


Distribution
Downloaded by Dallen Mae D.ofKadir
this document is illegal
(kadir.dallenmae.d.bcsi@gmail.com)
lOMoARcPSD|35346190

Stuvia.com - The Marketplace to Buy and Sell your Study Material

4. When the patient complains that he feels he is getting worse because of the increased swelling
at his wound site on his leg, the nurse‘s most helpful response would be that swelling indicates
that:
a. an infection is in progress at the wound site.
b. vessels have dilated and allowed plasma to leak into the wound site.
c. he has lain in one position for such a long time that swelling has occurred.
d. there is probably a deeper injury than what appears on the surface.

ANS: B
As part of the healing process, histamines and prostaglandins have caused small vessels to
dilate and leak plasma and electrolytes into the wound site causing swelling, which causes the
wound to become reddened and swollen as the phagocytosis cleans up the microorganisms.

DIF: Cognitive Level: Application REF: p. 761 OBJ: Theory #3


TOP: Swelling and Inflammation KEY: Nursing Process Step: Implementation
MSC: NCLEX: Physiological Integrity: Physiological Adaptation

5. The nurse warns the patient that one of the patient‘s habits has caused the reduction of
functional hemoglobin, which limits the hemoglobin‘s oxygen carrying ability. To improve
this situation, the nurse suggests that the patient quit:
a. drinking.
b. using marijuana.
c. smoking cigarettes.
d. eating excessive fats.
ANS: C
Smoking reduces the functional hemoglobin which, in turn, reduces the amount of oxygen
carried to the cells of the body.

DIF: Cognitive Level: Analysis REF: p. 762 OBJ: Theory #2


TOP: Smoking KEY: Nursing Process Step: Assessment
MSC: NCLEX: Physiological Integrity: Physiological Adaptation

6. A nurse is assessing a surgical patient for internal hemorrhage, which would be indicated by:
a. restlessness, rising pulse, and falling blood pressure.
b. restlessness, falling pulse, and rising blood pressure.
c. headache, rising pulse, and falling blood pressure.
d. lethargy, falling pulse, and rising blood pressure.

ANS: A
If hemorrhage occurs, it can lead to hypovolemic shock. Indicators of hemorrhage include
restlessness, rising pulse, and falling blood pressure.

DIF: Cognitive Level: Application REF: p. 765 OBJ: Theory #2


TOP: Hemorrhage KEY: Nursing Process Step: Assessment
MSC: NCLEX: Physiological Integrity: Physiological Adaptation

7. The nurse is alert to the indication of possible dehiscence of an abdominal surgical wound,
which would be evidenced by:
a. increased pallor of the surgical site.
b. complaint of constipation.
c. excessive gas.

Downloaded by: SUCCEEDGRADES | abbieclin@gmail.com


Distribution
Downloaded by Dallen Mae D.ofKadir
this document is illegal
(kadir.dallenmae.d.bcsi@gmail.com)
lOMoARcPSD|35346190

Stuvia.com - The Marketplace to Buy and Sell your Study Material

d. increased serosanguineous drainage from the wound.


ANS: D
Increase in the serosanguineous drainage from the surgical wound is a common sign of
impending dehiscence.

DIF: Cognitive Level: Application REF: p. 766 OBJ: Theory #4


TOP: Dehiscence KEY: Nursing Process Step: Assessment
MSC: NCLEX: Health Promotion and Maintenance: Prevention and Early Detection of Disease

8. A nurse is ambulating a patient in the hall a few days after abdominal surgery and the patient
says, ―I think something just let go.‖ The initial intervention by the nurse should be to:
a. seat the patient in a nearby chair.
b. assist the patient in a supine position.
c. ask someone to quickly get an abdominal binder.
d. instruct the patient to pant to reduce abdominal tension.
ANS: B
The patient is likely experiencing wound dehiscence and should immediately be assisted into
a supine position. This eliminates the force of gravity from putting additional stress on the
suture line and possibly causing evisceration.

DIF: Cognitive Level: Application REF: p. 766 OBJ: Theory #4


TOP: Dehiscence and Evisceration KEY: Nursing Process Step: Implementation
MSC: NCLEX: Physiological Integrity: Reduction of Risk

9. A patient who underwent removal of a breast must be discharged home with a Jackson-Pratt
wound drain in place. As the patient demonstrates the procedure for emptying it, the nurse
should correct her if she:
a. uses one alcohol wipe to clean both the spout and the plug.
b. compresses the device in the hand before closing.
c. refrains from touching the drainage spout with the hand.
d. points the device away from herself while opening it.

ANS: A
Separate alcohol swabs should be used to clean the spout and the plug.

DIF: Cognitive Level: Application REF: p. 768|Step 38-1


OBJ: Clinical Practice #1 TOP: Drainage Devices
KEY: Nursing Process Step: Evaluation
MSC: NCLEX: Safe, Effective Care Environment: Safety and Infection Control

10. The nurse chooses a nonadherent dressing to apply to a wound because the nonadherent
dressing:
a. is smaller and less bulky and will absorb more drainage.
b. retains sterility longer than plain gauze.
c. allows drainage to seep through the barrier and be absorbed on the other side.
d. does not require the use of tape to make it adhere to the skin.
ANS: C

Downloaded by: SUCCEEDGRADES | abbieclin@gmail.com


Distribution
Downloaded by Dallen Mae D.ofKadir
this document is illegal
(kadir.dallenmae.d.bcsi@gmail.com)
lOMoARcPSD|35346190

Stuvia.com - The Marketplace to Buy and Sell your Study Material

Telfa dressings have a shiny, nonadherent surface; the shiny side is applied to the wound to
prevent the dressing from sticking to the skin. The drainage seeps through the barrier and is
absorbed on the other side. It does require some sort of adhesive or binder to keep the pad in
place.

DIF: Cognitive Level: Comprehension REF: p. 769 OBJ: Clinical Practice #1


TOP: Dressings KEY: Nursing Process Step: Implementation
MSC: NCLEX: Physiological Integrity: Basic Care and Comfort

11. Because the patient with an abdominal dressing requires frequent dressing changes, the
abdomen is beginning to show skin irritation from repeated tape removal. The nurse would
change the dressing procedure in order to use:
a. paper tape.
b. Montgomery straps.
c. Karaya paste.
d. elastic adhesive tape.
ANS: B
Montgomery straps allow the dressing to be changed without constantly applying and
removing tape.

DIF: Cognitive Level: Analysis REF: p. 770 OBJ: Clinical Practice #1


TOP: Securing Dressings KEY: Nursing Process Step: Implementation
MSC: NCLEX: Physiological Integrity: Basic Care and Comfort

12. A nurse caring for a patient with a Stage I pressure ulcer would most appropriately select:
a. nonocclusive dressing.
b. exudate absorbing dressing.
c. hydrocolloid dressing.
d. thin film dressing.
ANS: D
Thin film dressings are used on Stage I ulcers to protect them from shearing forces and to
keep them moist.

DIF: Cognitive Level: Application REF: p. 769 OBJ: Clinical Practice #2


TOP: Treatment of Ulcers KEY: Nursing Process Step: Implementation
MSC: NCLEX: Physiological Integrity: Reduction of Risk

13. A patient has a pooling of blood under unbroken skin of the hip after a fall. The nurse should
document that this patient has a(n):
a. abrasion.
b. laceration.
c. hematoma.
d. avulsion.

ANS: C
A hematoma is a pooling of blood under unbroken skin. An abrasion is a scraping away of
skin tissue. A laceration is a torn, ragged, or mangled wound, and a contusion is a bruise.

DIF: Cognitive Level: Comprehension REF: p. 765 OBJ: Theory #1


TOP: Documentation KEY: Nursing Process Step: Assessment

Downloaded by: SUCCEEDGRADES | abbieclin@gmail.com


Distribution
Downloaded by Dallen Mae D.ofKadir
this document is illegal
(kadir.dallenmae.d.bcsi@gmail.com)
lOMoARcPSD|35346190

Stuvia.com - The Marketplace to Buy and Sell your Study Material

MSC: NCLEX: Physiological Integrity: Physiological Adaptation

14. The nurse is performing a dry sterile dressing change for an abdominal wound. The nurse
should use a swab to clean:
a. from the outer abdomen toward the wound.
b. in a circular motion around the wound circling to the outside.
c. from the left to the right across the wound.
d. directly over the wound.
ANS: B
A circular motion around the wound toward the outside keeps the wound area cleanest.

DIF: Cognitive Level: Application REF: p. 776|Skill 38-1


OBJ: Clinical Practice #1 TOP: Wound Cleaning
KEY: Nursing Process Step: Implementation
MSC: NCLEX: Safe, Effective Care Environment: Safety and Infection Control

15. A patient is due for a wound dressing change for a horizontal lower abdominal incision. In
which direction should the nurse pull to remove the tape from the old dressing?
a.From left to right across the abdomen
b.From right to left across the abdomen
c.From the top of the wound to the bottom
d.From each of the four sides toward the wound
ANS: D
The tape should be removed by pulling it off toward the wound. This helps prevent alteration
of the wound.

DIF: Cognitive Level: Application REF: p. 770 OBJ: Clinical Practice #1


TOP: Wound Care KEY: Nursing Process Step: Implementation
MSC: NCLEX: Physiological Integrity: Reduction of Risk

16. A nurse explains that the major purpose of the use of a hydrocolloid dressing is to:
a. keep the wound dry.
b. help destroy microorganisms in an infected wound.
c. occlude air and promote breakdown of necrotic tissue.
d. leave the dressing in place for 10 days.
ANS: C
Hydrocolloid dressings are air occlusive dressings used on noninfected wounds that provide a
moist environment for wound healing. They can be left in place for up to 7 days.

DIF: Cognitive Level: Comprehension REF: p. 769 OBJ: Clinical Practice #1


TOP: Hydrocolloid Dressing KEY: Nursing Process Step: Implementation
MSC: NCLEX: Safe, Effective Care Environment: Safety and Infection Control

17. The nurse changing a wet to dry normal saline dressing for a patient with an ulcer on the heel
finds that the old dressing is stuck to the wound bed. The nurse‘s most beneficial intervention
would be to:
a. add normal saline to loosen it.
b. pull it off using slow, steady pressure.
c. leave it in place and cover it with new, wet dressings.

Downloaded by: SUCCEEDGRADES | abbieclin@gmail.com


Distribution
Downloaded by Dallen Mae D.ofKadir
this document is illegal
(kadir.dallenmae.d.bcsi@gmail.com)
lOMoARcPSD|35346190

Stuvia.com - The Marketplace to Buy and Sell your Study Material

d. moisten it with povidone iodine.


ANS: A
If the dressing sticks to the wound, normal saline should be added to loosen it. Pulling loose a
stuck dressing damages new tissue. Leaving it in place does not promote a clean wound.
Povidone iodine must be ordered.

DIF: Cognitive Level: Analysis REF: p. 775|Skill 38-1


OBJ: Clinical Practice #3 TOP: Wet to Dry Dressings
KEY: Nursing Process Step: Implementation
MSC: NCLEX: Physiological Integrity: Reduction of Risk

18. A nurse performing a right eye irrigation will position the patient:
a. upright with the head hyperextended.
b. upright with the head tilted toward the left eye.
c. supine with the head hyperextended.
d. supine with the head tilted toward the right eye.

ANS: D
The patient should be positioned supine with the head tilted toward the affected eye. This
position allows the irrigation solution to drain away from the eye and not contaminate the
other eye.

DIF: Cognitive Level: Application REF: p. 783|Step 38-4


OBJ: Clinical Practice #3 TOP: Eye Irrigations
KEY: Nursing Process Step: Implementation
MSC: NCLEX: Safe, Effective Care Environment: Safety and Infection Control

19. A nurse removing wound staples would engage the staple puller and squeeze the handles
completely and:
a.pull to the right.
b.pull outward.
c.pull to the left.
d.rotate.
ANS: B
The handles should be squeezed together all the way. This depresses the center of the staple
and allows it to be lifted outward from the skin.

DIF: Cognitive Level: Application REF: p. 782|Step 38-3


OBJ: Clinical Practice #4 TOP: Staple Removal
KEY: Nursing Process Step: Implementation
MSC: NCLEX: Physiological Integrity: Basic Care and Comfort

20. The nurse clarifies that a vacuum-assisted closure supports healing of a wound by:
a. drawing the wound edges together by negative pressure.
b. interrupting the proliferation of bacteria in the wound.
c. strengthening the wall of the wound.
d. making an air occlusive cover for the wound.

ANS: A

Downloaded by: SUCCEEDGRADES | abbieclin@gmail.com


Distribution
Downloaded by Dallen Mae D.ofKadir
this document is illegal
(kadir.dallenmae.d.bcsi@gmail.com)
lOMoARcPSD|35346190

Stuvia.com - The Marketplace to Buy and Sell your Study Material

A vacuum-assisted dressing that is accomplished by a special dressing and vacuum device


applies negative pressure to the wound, which increases blood flow, increases oxygenation,
and improves the delivery of nutrients to the wound.

DIF: Cognitive Level: Knowledge REF: p. 771 OBJ: Theory #6


TOP: Vacuum-Assisted Dressing KEY: Nursing Process Step: Implementation
MSC: NCLEX: Physiological Integrity: Physiological Adaptation

21. The nurse is aware that the only necrotic wound for which debridement is not recommended is
a pressure ulcer located on the:
a. scapula.
b. sacrum.
c. heel.
d. femoral head.
ANS: C
Debridement is not recommended for treatment of a pressure ulcer on the heel because of the
small amount of tissue available at that site.

DIF: Cognitive Level: Knowledge REF: p. 769 OBJ: Clinical Practice #2


TOP: Debridement KEY: Nursing Process Step: Planning
MSC: NCLEX: Physiological Integrity: Reduction of Risk

MULTIPLE RESPONSE

1. The nurse is concerned about an HIV immunocompromised patient‘s ability to heal because of
the lack of: (Select all that apply.)
a. hemoglobin.
b. adequate fibroblast function.
c. synthesis of collagen.
d. intrinsic factor.
e. adequate phagocytosis.
ANS: B, C, E
Persons who are immunosuppressed have inadequate fibroblast function, phagocytosis, and
synthesis of collagen.

DIF: Cognitive Level: Comprehension REF: p. 765 OBJ: Theory #2


TOP: Immunocompromise KEY: Nursing Process Step: Planning
MSC: NCLEX: Physiological Integrity: Physiological Adaptation

2. The nurse recognizes that of the drugs a patient is currently taking, several contribute to
delayed healing, such as: (Select all that apply.)
a. vitamin C.
b. antineoplastic drugs.
c. pyrixidine.
d. heparin.
e. steroids.
ANS: B, D, E

Downloaded by: SUCCEEDGRADES | abbieclin@gmail.com


Distribution
Downloaded by Dallen Mae D.ofKadir
this document is illegal
(kadir.dallenmae.d.bcsi@gmail.com)
lOMoARcPSD|35346190

Stuvia.com - The Marketplace to Buy and Sell your Study Material

Drugs such as antineoplastic agents, anticoagulants, steroids, and immunosuppressants all


delay healing.

DIF: Cognitive Level: Comprehension REF: p. 765 OBJ: Theory #2


TOP: Delayed Healing KEY: Nursing Process Step: Assessment
MSC: NCLEX: Physiological Integrity: Pharmacological Therapies

3. The nurse reminds the 85-year-old patient that his healing will be slower because of age-
related changes such as: (Select all that apply.)
a. excessive production of blood factors.
b. atherosclerosis.
c. diminished lung function.
d. slow metabolism.
e. increased immunity.
ANS: B, C, D
Age slows metabolism and production of blood factors. Decreased lung function reduces the
body‘s supply of oxygen and slows healing, Atherosclerosis impairs blood flow.

DIF: Cognitive Level: Comprehension REF: p. 762 OBJ: Theory #2


TOP: Factor That Affect Wound Health KEY: Nursing Process Step: Implementation
MSC: NCLEX: Physiological Integrity: Physiological Adaptation

COMPLETION

1. The nurse places Dakin solution in a wound to accomplish chemical .

ANS:
debridement

Dakin solution is placed in a wound to destroy the necrotic tissue so that granulation tissue
can form to heal the wound (debridement).

DIF: Cognitive Level: Knowledge REF: p. 768 OBJ: Clinical Practice #2


TOP: Chemical Debridement KEY: Nursing Process Step: Implementation
MSC: NCLEX: Physiological Integrity: Pharmacological Therapies

2. The nurse assesses the large raised scar on the African American patient. The nurse documents
the lesion as a .

ANS:
keloid

Keloids are large raised permanent scars resulting from colloid overgrowth that are seen most
frequently on darkly pigmented skin.

DIF: Cognitive Level: Comprehension REF: p. 762 OBJ: Theory #1


TOP: Keloid KEY: Nursing Process Step: Assessment
MSC: NCLEX: Physiological Integrity: Physiological Adaptation

Downloaded by: SUCCEEDGRADES | abbieclin@gmail.com


Distribution
Downloaded by Dallen Mae D.ofKadir
this document is illegal
(kadir.dallenmae.d.bcsi@gmail.com)
lOMoARcPSD|35346190

Stuvia.com - The Marketplace to Buy and Sell your Study Material

3. The nurse explains to the patient that the foot will be submerged in warm water for a
maximum of minutes.

ANS:
20

Warm soaks that involve submerging the limb should only last for 15 to 20 minutes.

DIF: Cognitive Level: Knowledge REF: p. 786 OBJ: Clinical Practice #5


TOP: Foot Soak KEY: Nursing Process Step: Implementation
MSC: NCLEX: Physiological Integrity: Reduction of Risk

ORDERING

1. The nurse irrigating an infected wound of the hand would: (Prioritize the steps. Separate the
letters by a comma and a space as follows: A, B, C, D, E, F, G.)
a. Open sterile irrigation basin and solution.
b. Don sterile gloves to apply dressing.
c. Pour irrigating solution in basin.
d. Irrigate keeping the syringe tip 1 inch from the wound surface.
e. Document procedure.
f. Pat wound dry and redress.
g. Place pad under the infected hand.

ANS:
A, C, G, B, D, F, E

Prior to donning gloves, the basin and solution should be opened, the basin filled with the
solution, and the pad placed under the wound. The gloves are donned, the irrigation
completed, the wound dried and redressed, and the intervention documented.

DIF: Cognitive Level: Application REF: p. 778|Step 38-2


OBJ: Clinical Practice #3 TOP: Wound Dressing Change
KEY: Nursing Process Step: Implementation
MSC: NCLEX: Safe, Effective Care Environment: Safety and Infection Control

2. The nurse changing a patient‘s surgical dressing will: (Prioritize the steps. Separate the letters
by a comma and a space as follows: A, B, C, D, E, F, G.)
a. Apply new dressing.
b. Remove old dressing.
c. Gather supplies.
d. Wash hands.
e. Don clean gloves.
f. Document findings.
g. Apply sterile gloves.

ANS:
C, D, E, B, G, A, F

Downloaded by: SUCCEEDGRADES | abbieclin@gmail.com


Distribution
Downloaded by Dallen Mae D.ofKadir
this document is illegal
(kadir.dallenmae.d.bcsi@gmail.com)
lOMoARcPSD|35346190

Stuvia.com - The Marketplace to Buy and Sell your Study Material

The nurse gathers needed equipment for time management, washes hands, dons clean gloves
to remove old dressing, dons sterile gloves to apply new dressing, and documents the
outcome.

DIF: Cognitive Level: Application REF: p. 775|Skill 38-1


OBJ: Clinical Practice #1 TOP: Dressing Change
KEY: Nursing Process Step: Implementation
MSC: NCLEX: Safe, Effective Care Environment: Safety and Infection Control

Downloaded by: SUCCEEDGRADES | abbieclin@gmail.com


Distribution
Downloaded by Dallen Mae D.ofKadir
this document is illegal
(kadir.dallenmae.d.bcsi@gmail.com)
lOMoARcPSD|35346190

Stuvia.com - The Marketplace to Buy and Sell your Study Material

Chapter 39: Promoting Musculoskeletal Function


Williams: deWit's Fundamental Concepts and Skills for Nursing, 8th Edition

MULTIPLE CHOICE

1. The nurse directs the immobilized patient in frequent deep breathing exercises during the day
in order to combat:
a. low oxygen saturation.
b. atelectasis.
c. hypostatic pneumonia.
d. respiratory alkalosis.
ANS: C
Hypostatic pneumonia is a result of decreased physical mobility and is the most common
hospital acquired disorder in immobilized patients.

DIF: Cognitive Level: Comprehension REF: p. 790 OBJ: Theory #1


TOP: Hypostatic Pneumonia KEY: Nursing Process Step: Implementation
MSC: NCLEX: Health Promotion and Maintenance: Prevention and Early Detection of Disease

2. The nurse explains that range of motion exercises are necessary so that movement improves
venous circulation by:
a. vasodilation.
b. compression of muscles on venous walls.
c. increased metabolism.
d. maintaining strength in muscles.
ANS: B
The range of motion exercises mimic normal muscle movement, which compresses the
venous walls as a support to venous circulation.

DIF: Cognitive Level: Comprehension REF: p. 790 OBJ: Theory #5


TOP: Psychosocial Effects of Immobilization
KEY: Nursing Process Step: Implementation
MSC: NCLEX: Psychosocial Integrity: Reduction of Risk

3. A nurse enters the room of a patient who is in Buck‘s traction (skin traction). An error in the
traction setup observed would be:
a. feet resting against the foot of the bed.
b. weights hanging free in the air.
c. knee gatch raised.
d. head of bed elevated 20 degrees.
ANS: A
Feet should not rest against the foot of the bed because this interrupts the counter traction.

DIF: Cognitive Level: Analysis REF: p. 793 OBJ: Clinical Practice #1


TOP: Traction KEY: Nursing Process Step: Assessment
MSC: NCLEX: Physiological Integrity: Basic Care and Comfort

https://www.coursehero.com/file/30335434/c39rtf/

Downloaded by: SUCCEEDGRADES | abbieclin@gmail.com


Distribution
Downloaded by Dallen Mae D.ofKadir
this document is illegal
(kadir.dallenmae.d.bcsi@gmail.com)
lOMoARcPSD|35346190

Stuvia.com - The Marketplace to Buy and Sell your Study Material

4. The daughter of an older woman with a diagnosis of a fractured tibia asks why her mother is
in Buck‘s traction. The nurse‘s most informative response would be that Buck‘s traction:
a. helps the bone heal more quickly.
b. allows for large traction weights to reduce the fracture.
c. does not cause skin disruptions.
d. reduces muscle spasm that accompanies fractures.
ANS: D
Skin traction such as Buck‘s traction reduces muscle spasm that accompanies fractures.

DIF: Cognitive Level: Comprehension REF: p. 793 OBJ: Clinical Practice #1


TOP: Skin Traction KEY: Nursing Process Step: Implementation
MSC: NCLEX: Health Promotion and Maintenance: Prevention and Early Detection of Disease

5. An anxious patient in skeletal traction is distressed by the clear fluid drainage that is oozing
from the pin sites. The nurse‘s best intervention would be to:
a. notify the charge nurse of possible infection.
b. wipe off drainage with a damp wash cloth.
c. assure the patient that such drainage is expected.
d. cover the pin with several gauze pads and tape securely.
ANS: C
Clear fluid drainage from pin sites is expected. The fluid can be removed with a sterile swab.

DIF: Cognitive Level: Application REF: p. 794 OBJ: Clinical Practice #1


TOP: Anxiety Relative to Skeletal Traction
KEY: Nursing Process Step: Assessment
MSC: NCLEX: Psychosocial Integrity: Coping and Adaptation

6. A patient who just underwent a left arm cast change to a synthetic fiberglass cast asks when
the cast should be dry. The best response is that it should be hardened enough to be durable
within:
a. 30 minutes.
b. 60 minutes.
c. 4 hours.
d. 24 hours.
ANS: A
A synthetic cast made of material such as fiberglass is hardened enough to be durable within
30 minutes. A plaster cast can take 24 hours or longer to dry.

DIF: Cognitive Level: Knowledge REF: p. 794 OBJ: Theory #2


TOP: Casts KEY: Nursing Process Step: Implementation
MSC: NCLEX: Physiological Integrity: Basic Care and Comfort

7. When handling a freshly applied plaster cast while assisting a patient from stretcher to bed,
the nurse should handle the cast using:
a. fingertips and palms.
b. palms only.
c. fingertips and flat parts of fingers.
d. palms and flat parts of fingers.

https://www.coursehero.com/file/30335434/c39rtf/

Downloaded by: SUCCEEDGRADES | abbieclin@gmail.com


Distribution
Downloaded by Dallen Mae D.ofKadir
this document is illegal
(kadir.dallenmae.d.bcsi@gmail.com)
lOMoARcPSD|35346190

Stuvia.com - The Marketplace to Buy and Sell your Study Material

ANS: D
To prevent indentations in the cast that could lead to complications, the cast should be touched
only with the palms and the flat parts of the fingers.

DIF: Cognitive Level: Comprehension REF: p. 794 OBJ: Theory #2


TOP: Casts KEY: Nursing Process Step: Implementation
MSC: NCLEX: Health Promotion and Maintenance: Prevention and Early Detection of Disease

8. A patient who fractured a leg several weeks ago is scheduled for cast removal after he returns
home. The nurse should explain to the patient to expect the skin underneath the cast to appear:
a. moist and pink.
b. dry and dirty.
c. moist and white.
d. dry and greenish.
ANS: B
The skin that was underneath the cast will appear dry and dirty because of accumulation of
dead skin cells during the weeks that the cast was in place. Warm soapy water to wash and
lotions to moisturize will help.

DIF: Cognitive Level: Comprehension REF: p. 795 OBJ: Clinical Practice #4


TOP: Home Care for Cast Removal KEY: Nursing Process Step: Implementation
MSC: NCLEX: Physiological Integrity: Basic Care and Comfort

9. The nurse explains that an air-fluidized mattress would not be advocated for the patient with:
a. a spinal cord injury.
b. recurrent pressure ulcers.
c. burns that have been newly grafted.
d. severe arthritis.
ANS: A
Air-fluidized therapy is not recommended for patients with unstable spines or for those who
are ambulatory.

DIF: Cognitive Level: Analysis REF: p. 796 OBJ: Theory #3


TOP: Air-Fluidized Mattress KEY: Nursing Process Step: Implementation
MSC: NCLEX: Physiological Integrity: Reduction of Risk

10. The nurse points out that the major advantage of the low air loss mattress is that it reduces the
incidence of:
a. urinary stasis.
b. friction.
c. constipation.
d. contractures.
ANS: B
The low air loss mattress reduces the incidence of skin injury from friction and sheer.

DIF: Cognitive Level: Comprehension REF: p. 796 OBJ: Theory #3


TOP: Low Air Loss Mattress KEY: Nursing Process Step: Implementation
MSC: NCLEX: Physiological Integrity: Reduction of Risk

https://www.coursehero.com/file/30335434/c39rtf/

Downloaded by: SUCCEEDGRADES | abbieclin@gmail.com


Distribution
Downloaded by Dallen Mae D.ofKadir
this document is illegal
(kadir.dallenmae.d.bcsi@gmail.com)
lOMoARcPSD|35346190

Stuvia.com - The Marketplace to Buy and Sell your Study Material

11. The nurse placing a patient following knee replacement surgery into a continuous passive
motion (CPM) machine has the responsibility to:
a.raise the head of the bed to 30 to 45 degrees.
b.set the proper flexion and extension limits.
c.elevate the machine at the foot of the bed on one pillow.
d.keep the operated knee warm.
ANS: B
The nurse is responsible for securing the limb in the machine and setting the proper flexion
and extension limits ordered by the primary care provider.

DIF: Cognitive Level: Application REF: p. 797 OBJ: Theory #4


TOP: Continuous Passive Motion Machine
KEY: Nursing Process Step: Implementation
MSC: NCLEX: Physiological Integrity: Reduction of Risk

12. A nurse performing a head to toe neurovascular check on a patient in a long leg cast notes an
indication of altered perfusion as evidenced by:
a. capillary refill of 3 seconds.
b. palpable peripheral pulses.
c. warm feet and hands.
d. numbness of distal limb.
ANS: D
Pallor, numbness, or cyanosis indicates reduced circulation. Other adverse changes are
coolness, diminished or absent pulses, and possibly pain.

DIF: Cognitive Level: Analysis REF: p. 799|Box 39-3


OBJ: Theory #5 TOP: Neurovascular Impairment
KEY: Nursing Process Step: Assessment
MSC: NCLEX: Physiological Integrity: Reduction of Risk

13. The nurse assesses a patient as having delayed capillary refill if the blanching lasts longer
than:
a. 1 second.
b. 2 seconds.
c. 3 seconds.
d. 5 seconds.
ANS: D
Normal capillary refill should be within 3 seconds for the general population and within 5
seconds for an older adult.

DIF: Cognitive Level: Comprehension REF: p. 799|Box 39-3


OBJ: Theory #5 TOP: Neurovascular Assessment
KEY: Nursing Process Step: Assessment
MSC: NCLEX: Health Promotion and Maintenance: Prevention and Early Detection of Disease

14. A patient has just had a leg cast applied with plaster of Paris. The nurse can best reduce the
incidence of edema by:
a. elevating the leg on one to two pillows.
b. petaling the edges of the cast.

https://www.coursehero.com/file/30335434/c39rtf/

Downloaded by: SUCCEEDGRADES | abbieclin@gmail.com


Distribution
Downloaded by Dallen Mae D.ofKadir
this document is illegal
(kadir.dallenmae.d.bcsi@gmail.com)
lOMoARcPSD|35346190

Stuvia.com - The Marketplace to Buy and Sell your Study Material

c. placing the patient in high Fowler‘s position.


d. speeding the drying with a hair dryer.

ANS: A
Elevating the leg to heart level (one or two pillows) helps reduce edema formation.

DIF: Cognitive Level: Application REF: p. 792 OBJ: Theory #2


TOP: Casts KEY: Nursing Process Step: Implementation
MSC: NCLEX: Physiological Integrity: Reduction of Risk

15. A nurse is applying an elasticized bandage to the leg of a patient. To perform this procedure
correctly, the nurse should:
a. face the patient and wrap from proximal to distal.
b. adjust the pressure or tension as needed.
c. use metal clips to secure the wrap.
d. overlap turns of the bandage equally.
ANS: D
Turns of the bandage should be overlapped evenly. The wrap should be applied from distal to
proximal, maintaining even pressure or tension. Metal clips could fall off in the bed and injure
the patient; tape or pins should be used if the bandage does not have an adherent strip.

DIF: Cognitive Level: Application REF: p. 805|Box 39-4


OBJ: Theory #6 TOP: Bandages KEY: Nursing Process Step: Implementation
MSC: NCLEX: Physiological Integrity: Basic Care and Comfort

16. A nurse applying a pressure bandage for a patient should terminate the wrap by a:
a. spiral reverse turn.
b. circular turn.
c. spiral turn.
d. figure of eight turn.
ANS: B
A circular turn should be used to either anchor a bandage or terminate it.

DIF: Cognitive Level: Application REF: p. 805|Step 39-2


OBJ: Theory #6 TOP: Bandages KEY: Nursing Process Step: Implementation
MSC: NCLEX: Physiological Integrity: Basic Care and Comfort

17. A patient needs to have a triangular bandage applied. The nurse should position the sling so
that the hand is inch(es) below the elbow.
a. 1
b. 2
c. 4
d. 6
ANS: C
The hand should be approximately 4 inches higher than the elbow.

DIF: Cognitive Level: Application REF: p. 809|Step 39-3


OBJ: Theory #6 TOP: Bandages KEY: Nursing Process Step: Implementation
MSC: NCLEX: Physiological Integrity: Basic Care and Comfort

https://www.coursehero.com/file/30335434/c39rtf/

Downloaded by: SUCCEEDGRADES | abbieclin@gmail.com


Distribution
Downloaded by Dallen Mae D.ofKadir
this document is illegal
(kadir.dallenmae.d.bcsi@gmail.com)
lOMoARcPSD|35346190

Stuvia.com - The Marketplace to Buy and Sell your Study Material

18. When transferring a patient from bed to chair using a mechanical lift, the nurse should:
a. put the bed in the lowest position.
b. position the sling under the patient from the top of the head to the buttocks.
c. lower the far bed rail.
d. widen the stance of the lift‘s base and lock it.

ANS: D
The bed should be adjusted to working height with the far side rail raised. The sling is
positioned so that it lies from the back of the head or shoulders to the mid-thigh. The base of
the lift is widened to provide a good base of support and is locked for safety.

DIF: Cognitive Level: Application REF: p. 810|Skill 39-3


OBJ: Clinical Practice #6 TOP: Mechanical Lift
KEY: Nursing Process Step: Implementation
MSC: NCLEX: Safe, Effective Care Environment: Safety and Infection Control

19. A nurse is observing a patient in a skilled nursing facility using a walker. The nurse concludes
that the walker is at proper height if the patient‘s elbows are bent to which angle while the
patient is upright and grasping the handgrips?
a. 5 to 15 degrees
b. 15 to 30 degrees
c. 30 to 45 degrees
d. 45 to 60 degrees
ANS: B
The height is correct if the patient‘s elbow is bent at a 15- to 30-degree angle while standing
upright and grasping the handgrips.

DIF: Cognitive Level: Comprehension REF: p. 806 OBJ: Clinical Practice #7


TOP: Use of a Walker KEY: Nursing Process Step: Assessment
MSC: NCLEX: Physiological Integrity: Basic Care and Comfort

20. A nurse giving instructions to a patient who will be using stairs while ambulating with
crutches will instruct the patient:
a. ―Take off the rubber tips of the crutches while using stairs.‖
b. ―Rest the axillae on the axillary bar of the crutch.‖
c. ―Bring the good leg up first when going up stairs.‖
d. ―Move the good leg and the crutches together.‖

ANS: C
The patient should bring the good leg up first when going up stairs on crutches. The affected
leg and the crutches then follow. The rubber tips are left on at all times for safety. The axillae
should never rest on the axillary bar of the crutch, because this could result in nerve injury.

DIF: Cognitive Level: Application REF: p. 807|Boxes 39-5 and 39-29


OBJ: Clinical Practice #7 TOP: Crutch Walking
KEY: Nursing Process Step: Implementation
MSC: NCLEX: Safe, Effective Care Environment: Safety and Infection Control

21. A certified nursing assistant (CNA) is assisting a patient into a wheelchair. The nurse
intervenes if the CNA has:
a. left the brakes of the wheelchair unlocked.

https://www.coursehero.com/file/30335434/c39rtf/

Downloaded by: SUCCEEDGRADES | abbieclin@gmail.com


Distribution
Downloaded by Dallen Mae D.ofKadir
this document is illegal
(kadir.dallenmae.d.bcsi@gmail.com)
lOMoARcPSD|35346190

Stuvia.com - The Marketplace to Buy and Sell your Study Material

b. placed the patient‘s feet centered on the footrests.


c. placed slippers on the patient.
d. left the patient with the lap robe tucked underneath.

ANS: A
The brakes should always be locked when a wheelchair is not in motion, and especially when
someone is being assisted in and out of a wheelchair.

DIF: Cognitive Level: Analysis REF: p. 809|Step 39-3


OBJ: Clinical Practice #7 TOP: Assisting a Patient in a Wheelchair
KEY: Nursing Process Step: Implementation
MSC: NCLEX: Safe, Effective Care Environment: Safety and Infection Control

MULTIPLE RESPONSE

1. The nurse plans for an immobilized patient who suffered a cerebrovascular accident to be
protected from skin disruption by the use of: (Select all that apply.)
a. rubber sheets.
b. alcohol rubs to the heels.
c. sheepskin pads.
d. water mattresses.
e. pulsating air pads.
ANS: C, D, E
Sheepskin pads, water mattresses, and pulsating air pads help protect the immobilized patient
from skin disruption.

DIF: Cognitive Level: Application REF: p. 796 OBJ: Theory #4


TOP: Pressure Relief Devices KEY: Nursing Process Step: Planning
MSC: NCLEX: Physiological Integrity: Reduction of Risk

2. A patient has a hip spica cast and will be discharged home to family. The nurse would include
in the home education plan information relative to: (Select all that apply.)
a. protecting the cast from soiling.
b. easing itching under the cast by scratching with a bent coat hanger.
c. grasping the cast over the leg to help in turning.
d. using the spreader bar to turn the patient.
e. turning frequently to the prone position.
ANS: A, C, E
A patient with a hip spica cast will not be able to walk and will require frequent turning by
grasping the cast over the leg to help in the turning process. A challenge is protecting the cast
from stool or urine soiling. The patient should be turned frequently, especially to the prone
position.

DIF: Cognitive Level: Application REF: p. 794 OBJ: Clinical Practice #4


TOP: Hip Spica Cast KEY: Nursing Process Step: Planning
MSC: NCLEX: Health Promotion and Maintenance: Prevention and Early Detection of Disease

3. The nurse designs care for the immobilized patient to help combat the major dangers of
immobilization, which include: (Select all that apply.)

https://www.coursehero.com/file/30335434/c39rtf/

Downloaded by: SUCCEEDGRADES | abbieclin@gmail.com


Distribution
Downloaded by Dallen Mae D.ofKadir
this document is illegal
(kadir.dallenmae.d.bcsi@gmail.com)
lOMoARcPSD|35346190

Stuvia.com - The Marketplace to Buy and Sell your Study Material

a. pressure injuries.
b. loss of bone mass.
c. urinary infection.
d. pneumonia.
e. permanent loss of function.
ANS: A, B, D, E
Immobilization can cause pressure injuries, loss of bone mass, pneumonia, and possible
permanent loss of function of the immobilized part. Although urinary infections are common,
immobility is not their cause.

DIF: Cognitive Level: Comprehension REF: p. 790 OBJ: Theory #1


TOP: Effects of Immobility KEY: Nursing Process Step: Planning
MSC: NCLEX: Health Promotion and Maintenance: Prevention and Early Detection of Disease

4. When cleaning the pins on a patient in skeletal traction, the nurse should: (Select all that
apply.)
a. spray pin insertion with antimicrobial solution.
b. clean closest to the skin puncture site in a circular motion.
c. slightly rotate each pin to prevent adhesion to the bone.
d. secure ends of wire with cork or adhesive tape.
e. accomplish care with clean technique.
ANS: B, D
Pins should be cleansed with a sterile swab starting at the skin puncture; the ends of the wire
or pin can be secured with a piece of cork or adhesive tape. The process is performed with
sterile technique.

DIF: Cognitive Level: Application REF: p. 804|Box 39-2


OBJ: Clinical Practice #4 TOP: Pin Care
KEY: Nursing Process Step: Implementation
MSC: NCLEX: Health Promotion and Maintenance: Prevention and Early Detection of Disease

COMPLETION

1. The nurse is aware that the maximum weight that can be applied with a skin traction is
pounds.

ANS:
15

The maximum weight that can be applied to a skin traction is 15 pounds.

DIF: Cognitive Level: Knowledge REF: p. 793 OBJ: Clinical Practice #2


TOP: Skin Traction KEY: Nursing Process Step: Planning
MSC: NCLEX: Health Promotion and Maintenance: Prevention and Early Detection of Disease

2. The nurse demonstrates a crutch walking technique by advancing the left crutch and the right
foot and then the right crutch and the left foot. This is the gait.

ANS:

https://www.coursehero.com/file/30335434/c39rtf/

Downloaded by: SUCCEEDGRADES | abbieclin@gmail.com


Distribution
Downloaded by Dallen Mae D.ofKadir
this document is illegal
(kadir.dallenmae.d.bcsi@gmail.com)
lOMoARcPSD|35346190

Stuvia.com - The Marketplace to Buy and Sell your Study Material

two-point
two point

The two-point gait advances the opposite foot to the crutch.

DIF: Cognitive Level: Comprehension REF: p. 807|Boxes 39-5 and 39-29


OBJ: Clinical Practice #7 TOP: 2-Point Gait
KEY: Nursing Process Step: Implementation
MSC: NCLEX: Physiological Integrity: Basic Care and Comfort

3. The nurse, in order to prevent the cast from chafing, will instruct the patient for home care to
the rough edges with adhesive tape.

ANS:
petal

Adhesive tape may be used to ―petal‖ around the rough, crumbling edges of a cast to prevent
chafing.

DIF: Cognitive Level: Application REF: p. 39 21|Skill 39-1


OBJ: Clinical Practice #4 TOP: Petaling
KEY: Nursing Process Step: Implementation
MSC: NCLEX: Physiological Integrity: Basic Care and Comfort

https://www.coursehero.com/file/30335434/c39rtf/

Downloaded by: SUCCEEDGRADES | abbieclin@gmail.com


Distribution
Downloaded by Dallen Mae D.ofKadir
this document is illegal
(kadir.dallenmae.d.bcsi@gmail.com)
lOMoARcPSD|35346190

Stuvia.com - The Marketplace to Buy and Sell your Study Material

Chapter 40: Common Physical Care Problems of the Older Adult


Williams: deWit's Fundamental Concepts and Skills for Nursing, 8th Edition

MULTIPLE CHOICE

1. The nurse takes into consideration that of all the physical changes that the older adult
experiences, the most common cause of most problems is that of:
a. visual disturbance.
b. hearing deficit.
c. loss of muscle mass.
d. impaired mobility.
ANS: D
Constipation, urinary incontinence, and alteration in nutrition and depression are all problems
that are complicated or caused by impaired mobility.

DIF: Cognitive Level: Knowledge REF: p. 817|Table 40-2


OBJ: Theory #1 TOP: Common Physical Care Problems with the Older Adult
KEY: Nursing Process Step: Assessment
MSC: NCLEX: Physiological Integrity: Physiological Adaptation

2. While discussing ways to increase exercise with an older adult patient with no
musculoskeletal disorders, the nurse should encourage the patient to consider walking at a
frequency of:
a. 10 to 20 minutes once or twice a week.
b. 10 to 20 minutes four times a week.
c. 20 to 30 minutes once or twice a week.
d. 20 to 30 minutes three times a week.
ANS: D
It has been proven that walking for 20 to 30 minutes three times per week is very beneficial.

DIF: Cognitive Level: Comprehension REF: p. 816 OBJ: Theory #3


TOP: Mobility KEY: Nursing Process Step: Implementation
MSC: NCLEX: Health Promotion and Maintenance: Prevention and Early Detection of Disease

3. The home health nurse assesses all of the following relative to a resident in her own home:
glasses with a missing eye piece, soft soled floppy house shoes, walker with wheels, a floor
devoid of rugs. The item that is most likely to cause a fall would be the:
a. broken glasses.
b. floppy house shoes.
c. rolling walker.
d. no rug on floor.

ANS: B
Safe ambulation requires that the patient have an assistive walker and sturdy shoes. A clear
floor is a positive step in the direction of fall prevention. The glasses, although they may
distort the resident‘s perception, are not as dangerous as the nonsupportive shoes.

DIF: Cognitive Level: Analysis REF: p. 819 OBJ: Theory #4


TOP: Fall Prevention KEY: Nursing Process Step: Assessment

https://www.coursehero.com/file/30335537/c40rtf/

Downloaded by: SUCCEEDGRADES | abbieclin@gmail.com


Distribution
Downloaded by Dallen Mae D.ofKadir
this document is illegal
(kadir.dallenmae.d.bcsi@gmail.com)
lOMoARcPSD|35346190

Stuvia.com - The Marketplace to Buy and Sell your Study Material

MSC: NCLEX: Physiological Integrity: Reduction of Risk

4. An older adult patient is too weak to walk independently after surgery. Based on the services
available on the rehabilitation unit, the nurse should work collaboratively with:
a. an exercise physiologist.
b. a nutritionist.
c. a physical therapist.
d. an occupational therapist.
ANS: C
Physical therapists can assist patients with mobility and teach them to use assistive devices as
needed, such as walkers and canes.

DIF: Cognitive Level: Application REF: p. 816 OBJ: Theory #3


TOP: Mobility KEY: Nursing Process Step: Implementation
MSC: NCLEX: Safe, Effective Care Environment: Safety and Infection Control

5. The nurse adds to the nursing care plan for a resident with presbycusis. To better communicate
with the patient, the staff should use:
a. written notes.
b. a slower speed of speech.
c. a lower, deeper voice.
d. hand signals.
ANS: C
Speaking in a lower, deeper voice will allow the person with presbycusis to hear better since
these persons have difficulty picking up higher pitched sounds and spoken words.

DIF: Cognitive Level: Application REF: p. 823 OBJ: Theory #2


TOP: Presbycusis KEY: Nursing Process Step: Planning
MSC: NCLEX: Physiological Integrity: Physiological Adaptation

6. The nursing strategy that may be most helpful in preventing falls in older adult patients on a
skilled nursing unit would be to:
a. answer call bells promptly.
b. use vest restraints as needed.
c. keep lights dim for eye protection.
d. always keep bed rails up.
ANS: A
Nurses should answer call bells promptly to avoid patients‘ unsafe attempts to get out of bed.

DIF: Cognitive Level: Analysis REF: p. 818 OBJ: Theory #3


TOP: Fall Prevention KEY: Nursing Process Step: Implementation
MSC: NCLEX: Safe, Effective Care Environment: Safety and Infection Control

7. The home health nurse assesses a hazard for a patient in the home setting. Which of the
following assessments is considered a safety hazard?
a. Scatter rugs present in all rooms.
b. Stairways with handrails.
c. Grab bars in the bathroom.
d. Nonskid tape in the bathtub.

https://www.coursehero.com/file/30335537/c40rtf/

Downloaded by: SUCCEEDGRADES | abbieclin@gmail.com


Distribution
Downloaded by Dallen Mae D.ofKadir
this document is illegal
(kadir.dallenmae.d.bcsi@gmail.com)
lOMoARcPSD|35346190

Stuvia.com - The Marketplace to Buy and Sell your Study Material

ANS: A
An older adult patient should avoid the use of scatter rugs and should use nonskid mats
underneath other rugs.

DIF: Cognitive Level: Analysis REF: p. 819 OBJ: Theory #3


TOP: Fall Prevention at Home KEY: Nursing Process Step: Assessment
MSC: NCLEX: Safe, Effective Care Environment: Safety and Infection Control

8. A nurse is assisting an older adult neighbor to rearrange her kitchen to reduce fall risk. The
nurse should encourage her to avoid unnecessary reaching by placing all objects that are
needed below the level of the:
a. knees.
b. waist.
c. head.
d. chest.

ANS: C
Older adult patients should be instructed not to reach for objects that are above head level,
causing them to tip their heads backward. Step stools with a wide base of support may also be
helpful if objects cannot be stored in this manner.

DIF: Cognitive Level: Comprehension REF: p. 819 OBJ: Theory #3


TOP: Fall Prevention KEY: Nursing Process Step: Implementation
MSC: NCLEX: Safe, Effective Care Environment: Safety and Infection Control

9. An older adult patient in a skilled nursing facility tells the nurse that he has controlled his
incontinence with the herbal remedies of:
a. black cohosh.
b. pumpkin seeds.
c. feverfew.
d. St. John‘s wort.
ANS: B
Pumpkin seeds have been found beneficial in controlling incontinence in men by the reduction
of prostate swelling.

DIF: Cognitive Level: Comprehension REF: p. 820 OBJ: Clinical Practice #2


TOP: Urinary Incontinence KEY: Nursing Process Step: N/A
MSC: NCLEX: Physiological Integrity: Physiological Adaptation

10. The nurse uses the behavioral technique of habit voiding with a confused older adult patient to
reduce the frequency of urinary incontinence. This means the:
a. patient is assisted to the bathroom to use the toilet at regular intervals.
b. patient is being taught to request assistance from nursing staff.
c. staff are trying to lengthen the time between voiding for the patient.
d. fluid intake of the patient is being reduced so that voidings are less frequent.
ANS: A
Habit voiding, also called timed voiding, involves taking a confused or cognitively impaired
patient to the toilet at regular intervals.

DIF: Cognitive Level: Comprehension REF: p. 820|Box 40-1

https://www.coursehero.com/file/30335537/c40rtf/

Downloaded by: SUCCEEDGRADES | abbieclin@gmail.com


Distribution
Downloaded by Dallen Mae D.ofKadir
this document is illegal
(kadir.dallenmae.d.bcsi@gmail.com)
lOMoARcPSD|35346190

Stuvia.com - The Marketplace to Buy and Sell your Study Material

OBJ: Clinical Practice #2 TOP: Urinary Incontinence


KEY: Nursing Process Step: Implementation
MSC: NCLEX: Physiological Integrity: Physiological Adaptation

11. The nurse reminds the staff that the most effective method in preventing skin breakdown from
urinary incontinence is:
a. reducing fluid intake.
b. turning frequently.
c. ambulating frequently.
d. using protective pads.
ANS: D
Use of protective pads is an effective method of preventing skin breakdown. Discouraging
fluid intake will cause dehydration and more concentrated urine, turning frequently will not
take care of the problem.

DIF: Cognitive Level: Comprehension REF: p. 820 OBJ: Theory #2


TOP: Methods to Prevent Skin Impairment
KEY: Nursing Process Step: Implementation
MSC: NCLEX: Physiological Integrity: Basic Care and Comfort

12. An older adult patient on bed rest has been eating poorly. The patient is exhibiting abdominal
distention and cramping and is passing small amounts of liquid stool. The nurse assesses these
signs as an indication of:
a. constipation.
b. fecal impaction.
c. diarrhea.
d. GI tract infection.
ANS: B
Abdominal distention, cramping, and passage of small amounts of liquid stool are signs and
symptoms of fecal impaction. The risk factors that contribute to this are bed rest, not eating a
normal diet, and the use of pain medication.

DIF: Cognitive Level: Analysis REF: p. 820 OBJ: Theory #5


TOP: Fecal Impaction KEY: Nursing Process Step: Assessment
MSC: NCLEX: Physiological Integrity: Physiological Adaptation

13. When performing a digital rectal examination to determine the presence of fecal impaction,
the nurse must be alert for:
a. increasing blood pressure.
b. increasing respiratory rate.
c. reflexing incontinence.
d. decreasing heart rate.
ANS: D
The stimulation of the rectum by digital examination may stimulate the vagus nerve, which
then slows the heart rate. This is potentially hazardous, so it is done cautiously and only when
allowed by agency policy.

DIF: Cognitive Level: Application REF: p. 820 OBJ: Theory #2


TOP: Fecal Impaction KEY: Nursing Process Step: Implementation

https://www.coursehero.com/file/30335537/c40rtf/

Downloaded by: SUCCEEDGRADES | abbieclin@gmail.com


Distribution
Downloaded by Dallen Mae D.ofKadir
this document is illegal
(kadir.dallenmae.d.bcsi@gmail.com)
lOMoARcPSD|35346190

Stuvia.com - The Marketplace to Buy and Sell your Study Material

MSC: NCLEX: Physiological Integrity: Reduction of Risk

14. The nurse, in reviewing with an older adult patient the nutritional changes that would be most
beneficial, would suggest:
a. reducing sugar intake.
b. increasing fat intake.
c. increasing intake of oils.
d. decreasing intake of roughage.
ANS: A
Dietary recommendations for the older adult include decreasing sugar and fat intake.
Roughage should be increased to maintain proper bowel elimination.

DIF: Cognitive Level: Comprehension REF: p. 820 OBJ: Theory #6


TOP: Nutrition KEY: Nursing Process Step: Implementation
MSC: NCLEX: Physiological Integrity: Physiological Adaptation

15. The nurse stresses taking vitamins and minerals to older adult postmenopausal patients. To
reduce the risk of osteoporosis, women should increase their intake of:
a. iron.
b. magnesium.
c. calcium.
d. selenium.
ANS: C
Osteoporosis is the loss of calcium from bone. Calcium intake for postmenopausal and
perimenopausal women should be increased to 1000 to 15,000 mg/day, up from 800 mg/day
for the general population.

DIF: Cognitive Level: Comprehension REF: p. 821|Table 40-3


OBJ: Theory #5 TOP: Nutrition KEY: Nursing Process Step: Implementation
MSC: NCLEX: Health Promotion and Maintenance: Prevention and Early Detection of Disease

16. The nurse recognizes that of all the interventions to assist a dysphagic patient to eat safely, the
most significant to preventing aspiration is to:
a.sit the patient upright and remind the patient to tuck in the chin when swallowing.
b.feed small bites of 1/2 inch square.
c.thicken liquids.
d.offer frequent sips of fluid.
ANS: A
Upright positioning and reminders to tuck in the chin when swallowing are the most effective
prevention for aspiration.

DIF: Cognitive Level: Comprehension REF: p. 821 OBJ: Theory #6


TOP: Nutritional Support KEY: Nursing Process Step: Implementation
MSC: NCLEX: Safe, Effective Care Environment: Safety and Infection Control

17. The nurse is aware that the newly admitted resident has age-related macular degeneration
(AMD). The nurse will modify the care plan to accommodate the patient‘s:
a. loss of central vision.
b. lack of ability to focus on near objects.

https://www.coursehero.com/file/30335537/c40rtf/

Downloaded by: SUCCEEDGRADES | abbieclin@gmail.com


Distribution
Downloaded by Dallen Mae D.ofKadir
this document is illegal
(kadir.dallenmae.d.bcsi@gmail.com)
lOMoARcPSD|35346190

Stuvia.com - The Marketplace to Buy and Sell your Study Material

c. inability to adjust from light to dark environments.


d. increasing pressure in the eye with progressive blindness.

ANS: A
AMD causes loss of central vision as well as color perception.

DIF: Cognitive Level: Comprehension REF: p. 822 OBJ: Theory #1


TOP: Vision and the Older Adult KEY: Nursing Process Step: Assessment
MSC: NCLEX: Physiological Integrity: Basic Care and Comfort

18. For a patient with visual impairment who wishes to continue to eat independently, the nurse‘s
most helpful intervention would be to:
a.describe positions of foods on the plate by clock position.
b.tell the patient to eat all foods that are firmest first.
c.raise the over the bed table so that all food is within three (3) inches of the eyes.
d.have the patient use a spoon instead of a fork.
ANS: A
It is helpful to describe the position of the foods on the plate. The texture of the food has
nothing to do with visual impairment. It is not helpful or realistic to raise the over the bed
table so that the plate is three (3) inches away from the eyes. Using a spoon will not help the
patient identify foods on the plate.

DIF: Cognitive Level: Analysis REF: p. 822 OBJ: Theory #7


TOP: Vision KEY: Nursing Process Step: Implementation
MSC: NCLEX: Physiological Integrity: Basic Care and Comfort

19. A nurse who is assisting a blind patient to ambulate should:


a. hold the patient‘s arm firmly to gently push him in the proper direction.
b. hold the patient by a strap around the patient‘s waist to prevent his falling.
c. offer the patient an arm for guidance.
d. acquire a cane for the patient.

ANS: C
Offering the patient an arm or walking in front of the patient with the patient‘s hand on the
nurse‘s shoulder gently guides the visually impaired.

DIF: Cognitive Level: Application REF: p. 822 OBJ: Theory #6


TOP: Hearing and the Older Adult KEY: Nursing Process Step: Implementation
MSC: NCLEX: Health Promotion and Maintenance: Reduction of Risk

20. An older adult patient with arthritis is having difficulty using a weekly pillbox as a reminder
to take daily medications. The nurse would suggest as the best alternative:
a. a paper and pencil check off system.
b. a colorful calendar.
c. a homemade egg carton container.
d. symbol and color coded medication bottles.
ANS: C
The egg container will ease the difficulty with the arthritic patient best.

DIF: Cognitive Level: Application REF: p. 825 OBJ: Clinical Practice #4


TOP: Medication System for the Older Adult

https://www.coursehero.com/file/30335537/c40rtf/

Downloaded by: SUCCEEDGRADES | abbieclin@gmail.com


Distribution
Downloaded by Dallen Mae D.ofKadir
this document is illegal
(kadir.dallenmae.d.bcsi@gmail.com)
lOMoARcPSD|35346190

Stuvia.com - The Marketplace to Buy and Sell your Study Material

KEY: Nursing Process Step: Implementation


MSC: NCLEX: Physiological Integrity: Basic Care and Comfort

21. A nurse is caring for an 86-year-old patient who still takes pride in the fact that he drives. The
nurse suggests that his driving be limited to:
a. back roads.
b. large shopping centers.
c. going to church and the grocery store.
d. daytime driving.
ANS: D
Daytime driving is the safest in areas that are familiar. Back roads may be hard to navigate
and help may not always be available.

DIF: Cognitive Level: Application REF: p. 823 OBJ: Theory #1


TOP: Vision KEY: Nursing Process Step: Implementation
MSC: NCLEX: Psychosocial Integrity: Behavioral management

22. An older adult Hispanic patient is brought to a clinic. She brings a bag full of medications
with her. When the nurse is talking to the patient, a significant question to ask to get a full
picture of the patient would be:
a. ―Do you live with your family?‖
b. ―Tell me about your diet.‖
c. ―How many doctors prescribe drugs for you?‖
d. ―Are you drinking herbal supplements?‖

ANS: D
In Hispanic cultures, the use of herbal remedies is very common. This question is appropriate
to ask to have a full picture of this patient‘s medical history.

DIF: Cognitive Level: Analysis REF: p. 825 OBJ: Theory #9


TOP: Polypharmacy KEY: Nursing Process Step: Assessment
MSC: NCLEX: Physiological Integrity: Pharmacological Therapies

MULTIPLE RESPONSE

1. The nurse lists the most common causes of polypharmacy as: (Select all that apply.)
a. use of mail order sources.
b. being prescribed to by several health care providers.
c. sharing drugs with others.
d. many drugs being prescribed under different names.
e. availability of OTC medications.
ANS: A, B, D, E
Treatment by several health care providers, many drugs prescribed under different names,
ordering drugs from mail order sources, and the extensive availability of OTC medications
increase the risk for polypharmacy.

DIF: Cognitive Level: Comprehension REF: p. 824 OBJ: Theory #9


TOP: Polypharmacy KEY: Nursing Process Step: Implementation
MSC: NCLEX: Safe, Effective Care Environment: Safety and Infection Control

https://www.coursehero.com/file/30335537/c40rtf/

Downloaded by: SUCCEEDGRADES | abbieclin@gmail.com


Distribution
Downloaded by Dallen Mae D.ofKadir
this document is illegal
(kadir.dallenmae.d.bcsi@gmail.com)
lOMoARcPSD|35346190

Stuvia.com - The Marketplace to Buy and Sell your Study Material

2. The older adults, especially women, are at high risk for decreased mobility. Which nutrient(s)
are critical for women to take to decrease this risk? (Select all that apply.)
a. Protein
b. Fat
c. Carbohydrates
d. Calcium
e. Vitamin D
ANS: D, E
Proper treatment for arthritis and osteoporosis will help prevent immobility problems.
Estrogen replacement therapy (ERT) and adequate dietary or supplemental calcium and
vitamin D in combination with weight bearing exercise are protective against osteoporosis in
women.

DIF: Cognitive Level: Comprehension REF: p. 821|Table 40-3


OBJ: Theory #5 TOP: Mobility and Nutrients KEY: Nursing Process Step: N/A
MSC: NCLEX: N/A

3. A nurse reviewing the medication list for an older adult patient notices several drugs that
would increase the risk of falls because of orthostatic hypotension, which are: (Select all that
apply.)
a. anticoagulants.
b. diuretics.
c. stool softeners.
d. antihypertensives.
e. antihistamines.
ANS: B, D, E
Diuretics, hypertensives, and antihistamines contribute to orthostatic hypotension, a common
cause of falls.

DIF: Cognitive Level: Comprehension REF: p. 818 OBJ: Theory #2


TOP: Multi pharmacy KEY: Nursing Process Step: Assessment
MSC: NCLEX: Physiological Integrity: Pharmacological Therapies

COMPLETION

1. The nurse takes into consideration that the resident in a nursing home has a hearing deficit
related to a continuous ringing in his ears, which is a condition called .

ANS:
tinnitus

Tinnitus is a continuous ringing or buzzing in the ear, which causes further hearing loss.

DIF: Cognitive Level: Knowledge REF: p. 823 OBJ: Theory #2


TOP: Tinnitus KEY: Nursing Process Step: Assessment
MSC: NCLEX: Physiological Integrity: Physiological Adaptation

2. The nurse documents the report of painful intercourse as .

https://www.coursehero.com/file/30335537/c40rtf/

Downloaded by: SUCCEEDGRADES | abbieclin@gmail.com


Distribution
Downloaded by Dallen Mae D.ofKadir
this document is illegal
(kadir.dallenmae.d.bcsi@gmail.com)
lOMoARcPSD|35346190

Stuvia.com - The Marketplace to Buy and Sell your Study Material

ANS:
dyspareunia

Dyspareunia is the occasion of painful intercourse.

DIF: Cognitive Level: Knowledge REF: p. 824 OBJ: Theory #8


TOP: Dyspareunia KEY: Nursing Process Step: Assessment
MSC: NCLEX: Physiological Integrity: Physiological Adaptation

https://www.coursehero.com/file/30335537/c40rtf/

Downloaded by: SUCCEEDGRADES | abbieclin@gmail.com


Distribution
Downloaded by Dallen Mae D.ofKadir
this document is illegal
(kadir.dallenmae.d.bcsi@gmail.com)
lOMoARcPSD|35346190

Stuvia.com - The Marketplace to Buy and Sell your Study Material

Chapter 41: Common Psychosocial Care Problems of Older Adults


Williams: deWit's Fundamental Concepts and Skills for Nursing, 8th Edition

MULTIPLE CHOICE

1. The nurse is planning an instruction for an 84-year-old man relative to a significant change
in his diet for diabetes. The nurse will plan her education around the idea that older adults:
a. need to have their family to hear the instruction.
b. cannot learn complex information or skills.
c. need more time to learn because of slower processing skills.
d. are fixed in their ideas and reject information that does not agree with them.
ANS: C
The older adult patient needs more time to learn because of slower processing skills.

DIF: Cognitive Level: Comprehension REF: p. 831 OBJ: Theory #2


TOP: Changes in Cognitive Functioning KEY: Nursing Process Step: Planning
MSC: NCLEX: Health Promotion and Maintenance: Prevention and Early Detection of
Disease

2. The nurse clarifies to a family of a resident with Alzheimer disease that dementia differs
from confusion and delirium in that dementia is:
a. usually rapid in onset.
b. permanent.
c. caused by depression.
d. effectively treatable.
ANS: B
Dementia is generally a permanent condition characterized by cognitive deficits with a slow
onset. It is primarily seen in Alzheimer disease patients but also occurs in persons with brain
tumors.

DIF: Cognitive Level: Comprehension REF: p. 832 OBJ: Theory #3


TOP: Confusion KEY: Nursing Process Step: Implementation
MSC: NCLEX: Psychosocial Integrity: Psychosocial Adaptation

3. The nurse in a long-term care facility emphasizes to the family of a resident recently
admitted that one of the purposes of the creative behavioral therapies is to:
a. entertain the residents who have become bored.
b. stimulate an avid interest in music or art.
c. keep the residents out of their rooms.
d. slow the rate of deterioration.
ANS: D
The creative behavioral therapies of art, music, dancing, and humor are designed to delay
the deterioration of the resident.

DIF: Cognitive Level: Knowledge REF: p. 833 OBJ: Theory #1


TOP: Creative Therapies KEY: Nursing Process Step: Implementation

https://www.coursehero.com/file/30335509/c41rtf/

Downloaded by: SUCCEEDGRADES | abbieclin@gmail.com


Distribution
Downloaded by Dallen Mae D.ofKadir
this document is illegal
(kadir.dallenmae.d.bcsi@gmail.com)
lOMoARcPSD|35346190

Stuvia.com - The Marketplace to Buy and Sell your Study Material

MSC: NCLEX: Psychosocial Integrity: Psychosocial Adaptation

4. The family of a retired army veteran diagnosed with Alzheimer disease is concerned about
obtaining care for the patient while away on vacation. The home health nurse informs the
family that the Department of Veterans Affairs offers in facility care for patients with
dementia for up to:
a. 10 days a year.
b. 15 days a year.
c. 20 days a year.
d. 30 days a year.
ANS: D
The Department of Veterans Affairs offers in facility care for demented patients who are
veterans for up to 30 days a year.

DIF: Cognitive Level: Knowledge REF: p. 834


OBJ: Clinical Practice #1 TOP: Respite Care
KEY: Nursing Process Step: Implementation
MSC: NCLEX: Psychosocial Integrity: Coping and Adaptation

5. When the nurse plans to use reminiscence as a psychosocial approach to managing


confusion with cognitively impaired patients, the nurse should:
a. use plants, pictures, and animals to encourage interactions in the group.
b. use memory aids such as television, radio, clock, and calendar.
c. encourage individual and group sharing of information about previous life
experiences.
d. increase socialization roles in the group, such as serving each other refreshments.
ANS: C
Reminiscence involves individual and group sharing about previous life experiences.

DIF: Cognitive Level: Application REF: p. 833|Table 41-2


OBJ: Clinical Practice #1 TOP: Psychosocial Approaches
KEY: Nursing Process Step: Planning
MSC: NCLEX: Psychosocial Integrity: Psychosocial Adaptation

6. When a patient with dementia exhibits increasing agitation, hostility, and paranoia, the nurse
anticipates the health care provider will prescribe a(n):
a. anticonvulsant.
b. antidepressant.
c. minor tranquilizer.
d. major tranquilizer.
ANS: D
Major tranquilizers are often prescribed to manage the anxiety, agitation, hostility, and
paranoia associated with dementia. Minor tranquilizers may be used to treat symptoms that
are less severe than the ones experienced by this patient. There are, however, many
undesirable side effects.

DIF: Cognitive Level: Analysis REF: p. 833

https://www.coursehero.com/file/30335509/c41rtf/

Downloaded by: SUCCEEDGRADES | abbieclin@gmail.com


Distribution
Downloaded by Dallen Mae D.ofKadir
this document is illegal
(kadir.dallenmae.d.bcsi@gmail.com)
lOMoARcPSD|35346190

Stuvia.com - The Marketplace to Buy and Sell your Study Material

OBJ: Clinical Practice #1 TOP: Pharmacology


KEY: Nursing Process Step: Planning
MSC: NCLEX: Physiological Integrity: Pharmacological Therapies

7. The family of a patient with Alzheimer disease indicates that they want to keep the patient at
home but are not sure how much longer they can care for the patient because of stress on
family members. A helpful suggestion by the home health nurse would be to:
a. consider use of respite services.
b. face the reality of need for long-term care.
c. encourage the hiring of a full time caregiver.
d. encourage family counseling.
ANS: A
Respite care or adult day services can provide for much needed psychological and physical
rest for caregivers.

DIF: Cognitive Level: Application REF: p. 834 OBJ: Theory #2


TOP: Respite Care KEY: Nursing Process Step: Implementation
MSC: NCLEX: Psychosocial Integrity: Coping and Adaptation

8. A home health nurse working with an older adult patient assesses an early indication that
this patient is developing Alzheimer disease. This early indication would be:
a. wandering behavior.
b. agitation.
c. difficulty learning new things.
d. deteriorating speech.
ANS: C
Early signs of Alzheimer disease are mild short-term memory loss, difficulty learning new
things, and mild depression.

DIF: Cognitive Level: Comprehension REF: p. 834|Box 41-4


OBJ: Theory #3 TOP: Alzheimer Disease
KEY: Nursing Process Step: Assessment
MSC: NCLEX: Psychosocial Integrity: Psychosocial Adaptation

9. An older adult patient with early Alzheimer disease is receiving the drug donepezil
(Aricept). The nurse assesses the patient laboratory reports carefully for drug side effects
because of the drug‘s potential toxicity to the:
a. kidneys.
b. liver.
c. spleen.
d. heart.
ANS: B
This drug has toxic effects on the liver and requires that the patient be monitored carefully.

DIF: Cognitive Level: Knowledge REF: p. 833 OBJ: Theory #5


TOP: Drug Therapy for Alzheimer Disease
KEY: Nursing Process Step: Assessment

https://www.coursehero.com/file/30335509/c41rtf/

Downloaded by: SUCCEEDGRADES | abbieclin@gmail.com


Distribution
Downloaded by Dallen Mae D.ofKadir
this document is illegal
(kadir.dallenmae.d.bcsi@gmail.com)
lOMoARcPSD|35346190

Stuvia.com - The Marketplace to Buy and Sell your Study Material

MSC: NCLEX: Physiological Integrity: Pharmacological Therapies

10. When a patient becomes violent and hits a table with his cane, the initial appropriate nursing
approach is to:
a. medicate the patient to help control his anxiety.
b. call for assistance to apply restraints.
c. attempt to distract the patient.
d. direct the patient in a loud authoritarian voice to sit down.
ANS: C
A behavioral approach such as distraction might diffuse the situation until the cause can be
determined. Chemical restraint (medication) or a restrictive restraint should not be the first
intervention. Loud voices frequently increase the violent behavior.

DIF: Cognitive Level: Analysis REF: p. 836 OBJ: Theory #5


TOP: Hostility and Agitation KEY: Nursing Process Step: Implementation
MSC: NCLEX: Psychosocial Integrity: Coping and Adaptation

11. The nurse adds to the plan of care for a cognitively impaired resident who has begun to
wander night and day throughout the long-term care facility. An appropriate intervention to
add to the plan of care for the resident with wandering behavior would be to:
a. place the resident on a locked unit to prevent long-range wandering.
b. obtain an order for wrist restraints or a vest restraint.
c. apply a bracelet that alarms as the resident approaches an outside door.
d. discuss with the health care provider the need for stronger medication.
ANS: C
Wanderers may be finding a means to combat boredom. Placing a door alarm bracelet will
alert the staff if the resident is near an outside door. Frequent checks on the resident‘s
location can also help keep such a resident safe.

DIF: Cognitive Level: Application REF: p. 835 OBJ: Theory #5


TOP: Interventions for Wandering KEY: Nursing Process Step: Implementation
MSC: NCLEX: Safe, Effective Care Environment: Safety and Infection Control

12. The 64-year-old resident newly admitted to a long-term care facility refuses to sit down and
eat, preferring to wander aimlessly through the facility. The initial intervention by the
nursing staff should be to:
a. apply an alarm bracelet to monitor wandering.
b. offer high protein malts to drink on the go.
c. feed the resident in his room away from other residents.
d. feed the patient rapidly before he begins to wander.
ANS: B
The offering of high protein drinks or nutritious snacks to eat on the go may be an initial
approach to the problem.

DIF: Cognitive Level: Analysis REF: p. 837 OBJ: Theory #5


TOP: Sundown Syndrome KEY: Nursing Process Step: Implementation
MSC: NCLEX: Physiological Integrity: Basic Care and Comfort

https://www.coursehero.com/file/30335509/c41rtf/

Downloaded by: SUCCEEDGRADES | abbieclin@gmail.com


Distribution
Downloaded by Dallen Mae D.ofKadir
this document is illegal
(kadir.dallenmae.d.bcsi@gmail.com)
lOMoARcPSD|35346190

Stuvia.com - The Marketplace to Buy and Sell your Study Material

13. In order to minimize the risk of aspiration in a resident with advanced Alzheimer disease,
the person feeding the patient should:
a. keep a suction machine available.
b. have the patient consume only liquids.
c. remind the patient to chew and swallow.
d. offer large amounts of water after each bite.
ANS: C
Reminding the demented patient to chew and swallow will help prevent the resident from
holding food in his mouth.

DIF: Cognitive Level: Application REF: p. 837 OBJ: Theory #5


TOP: Strategies to Prevent Aspiration KEY: Nursing Process Step: Implementation
MSC: NCLEX: Psychosocial Integrity: Reduction of Risk

14. A depressed older adult patient was started on antidepressant drug therapy 3 weeks ago. The
highest nursing priority when working with this patient at this time would be:
a. stimulating appetite.
b. providing reality orientation.
c. encouraging socialization.
d. protecting the patient from self-injury.
ANS: D
The primary nursing responsibility for a depressed patient is to protect him from self-injury,
especially after the patient has been started on antidepressant therapy. Before that time, the
patient may not have had the energy to commit self-injury.

DIF: Cognitive Level: Analysis REF: p. 838


OBJ: Clinical Practice #2 TOP: Depression
KEY: Nursing Process Step: Implementation
MSC: NCLEX: Psychosocial Integrity: Coping and Adaptation

15. The behavior in a depressed older adult patient that would indicate that this patient is
contemplating suicide is:
a. giving away personal belongings.
b. watching television in the activity room.
c. talking with other patients.
d. spending time sitting near the nurses‘ station.
ANS: A
Signs of potential suicide include giving away treasured possessions, meticulous planning of
personal affairs, sudden euphoria, and statements of death wishes. The other options do not
indicate behaviors that warn of a possible upcoming suicide attempt.

DIF: Cognitive Level: Analysis REF: p. 838


OBJ: Clinical Practice #2 TOP: Suicide
KEY: Nursing Process Step: Assessment
MSC: NCLEX: Psychosocial Integrity: Coping and Adaptation

https://www.coursehero.com/file/30335509/c41rtf/

Downloaded by: SUCCEEDGRADES | abbieclin@gmail.com


Distribution
Downloaded by Dallen Mae D.ofKadir
this document is illegal
(kadir.dallenmae.d.bcsi@gmail.com)
lOMoARcPSD|35346190

Stuvia.com - The Marketplace to Buy and Sell your Study Material

16. The nurse would question a new order for a tricyclic antidepressant for a patient who has
had a recent:
a. peptic ulcer.
b. myocardial infarct.
c. abdominal surgery.
d. diagnosis of diabetes.
ANS: B
Tricyclics are contraindicated in patients with recent myocardial infarctions because these
drugs may cause cardiac arrhythmias.

DIF: Cognitive Level: Analysis REF: p. 838|Table 41-3


OBJ: Clinical Practice #2 TOP: Depression
KEY: Nursing Process Step: Assessment MSC: NCLEX: Psychosocial Integrity

17. A family member tells a hospitalized older adult patient to cooperate better with the
treatment plan or placement in a long-term care facility will result. The nurse recognizes this
statement is consistent with elder abuse.
a. physical
b. material
c. psychological
d. neglect
ANS: C
Elder abuse can be inflicted physically, verbally, or emotionally.

DIF: Cognitive Level: Comprehension REF: p. 839|Box 41-8


OBJ: Theory #7 TOP: Elder Abuse KEY: Nursing Process Step: Assessment
MSC: NCLEX: Psychosocial Integrity: Coping and Adaptation

18. When the nurse determines that an older adult patient has a reasonable risk of being
physically abused by family members, it is the nurse‘s legal obligation to:
a. report the suspected abuse to the proper authority.
b. refer the family for counseling.
c. advise the patient to leave the family home.
d. tell the family to stop or face legal consequences.
ANS: A
It is a legal obligation under state mandated reporting laws for suspected abuse for nurses to
report instances in which there is a reasonable belief that an individual has been or is likely
to be abused, neglected, or exploited.

DIF: Cognitive Level: Application REF: p. 839|Box 41-8


OBJ: Theory #7 TOP: Elder Abuse KEY: Nursing Process Step: Implementation
MSC: NCLEX: Safe, Effective Care Environment: Safety and Infection Control

19. An older female adult is most at risk for becoming a victim of a crime by:
a. having a peephole on the front door.
b. keeping doors locked with dead bolts.
c. having locks changed if keys are lost.

https://www.coursehero.com/file/30335509/c41rtf/

Downloaded by: SUCCEEDGRADES | abbieclin@gmail.com


Distribution
Downloaded by Dallen Mae D.ofKadir
this document is illegal
(kadir.dallenmae.d.bcsi@gmail.com)
lOMoARcPSD|35346190

Stuvia.com - The Marketplace to Buy and Sell your Study Material

d. telling a stranger on the phone that she is alone at home.


ANS: D
The riskiest behavior is telling strangers on the phone that she is alone.

DIF: Cognitive Level: Analysis REF: p. 840


OBJ: Clinical Practice #3 TOP: Crime
KEY: Nursing Process Step: N/A MSC: NCLEX: N/A

20. The older adults are vulnerable not only to crime, but also to scams. The best advice to give
an older adult in avoiding scams is to:
a. travel with a group.
b. hang up when a timesharing agent calls.
c. lock windows at night.
d. consider getting a pet for protection.
ANS: B
Scams are white collar crimes such as timesharing, telemarketing, or funeral planning.
Traveling in groups, locking windows, and acquiring a dog for protection are safety
precautions. The best thing to do is to not talk to the caller.

DIF: Cognitive Level: Comprehension REF: p. 840


OBJ: Clinical Practice #3 TOP: Crime
KEY: Nursing Process Step: N/A MSC: NCLEX: N/A

MULTIPLE RESPONSE

1. The nurse is aware that the older adults of today face some functional psychosocial issues,
which include: (Select all that apply.)
a. altered mobility.
b. becoming crime victims.
c. housing.
d. making provision for physical care.
e. cognitive impairments.
ANS: A, D, E
Altered mobility, making provisions for physical care, and dealing with cognitive
impairments are functional issues the older adult must resolve. Housing and crime are
external issues.

DIF: Cognitive Level: Comprehension REF: p. 830 OBJ: Theory #9


TOP: Psychosocial Crises of the Older Adult
KEY: Nursing Process Step: Planning
MSC: NCLEX: Psychosocial Integrity: Psychosocial Adaptation

2. The home health nurse counsels the family of a cognitively impaired man that to best
provide for his welfare, the family should: (Select all that apply.)
a. rearrange furniture and art to stimulate him.
b. use concise and direct communication.
c. enroll him in an Older Adult Activity Program.

https://www.coursehero.com/file/30335509/c41rtf/

Downloaded by: SUCCEEDGRADES | abbieclin@gmail.com


Distribution
Downloaded by Dallen Mae D.ofKadir
this document is illegal
(kadir.dallenmae.d.bcsi@gmail.com)
lOMoARcPSD|35346190

Stuvia.com - The Marketplace to Buy and Sell your Study Material

d. monitor nutrition for adequacy.


e. install a door alarm that sounds when it is opened.
ANS: B, C, D, E
Using direct communication enhances the impaired person‘s perception. Making the
environment safe by installing an alarm and providing for socialization and adequate
nutrition are also part of the principles of the care of the cognitively impaired.

DIF: Cognitive Level: Application REF: p. 833|Box 41-3


OBJ: Theory #1 TOP: Care of the Cognitively Impaired
KEY: Nursing Process Step: Implementation
MSC: NCLEX: Psychosocial Integrity: Psychosocial Adaptation

3. The nurse instructs a family of an 87-year-old resident in a long-term care facility that his
nocturnal delirium is most likely caused by: (Select all that apply.)
a. his schedule changing as a result of recent admission to the facility.
b. lack of adequate medication for anxiety.
c. increased shadows of the evening hours.
d. dehydration.
e. food allergy.
ANS: A, C, D
Nocturnal delirium is a disorder about which little is known. It is believed that among its
many possible causes, the most prominent are fatigue, low lighting, increased shadows of
the evening hours, disruption in the body clock, and dehydration.

DIF: Cognitive Level: Comprehension REF: p. 831 OBJ: Theory #3


TOP: Nocturnal Delirium KEY: Nursing Process Step: Implementation
MSC: NCLEX: Psychosocial Integrity: Psychosocial Adaptation

COMPLETION

1. The nurse clarifies that the diagnosis of nocturnal delirium refers to a syndrome also called
.

ANS:
sundowning

Sundowning is a syndrome in which confusion and agitation increases with the evening
hours.

DIF: Cognitive Level: Knowledge REF: p. 831 OBJ: Theory #3


TOP: Sundowning KEY: Nursing Process Step: Implementation
MSC: NCLEX: Psychosocial Integrity: Psychosocial Adaptation

2. When the nurse observes the warning on the label on a bottle of


antipsychotics, the nurse is aware that it is a very strong advisory from the Food and Drug
Administration (FDA) prior to pulling the drug off the market.

https://www.coursehero.com/file/30335509/c41rtf/

Downloaded by: SUCCEEDGRADES | abbieclin@gmail.com


Distribution
Downloaded by Dallen Mae D.ofKadir
this document is illegal
(kadir.dallenmae.d.bcsi@gmail.com)
lOMoARcPSD|35346190

Stuvia.com - The Marketplace to Buy and Sell your Study Material

ANS:
Black Box

The FDA may place the ―Black Box‖ warning on a label of a drug connoting that the drug
has been associated with an increased risk of death. It is a warning that may precede pulling
the drug from the market. This practice was initiated in 2008 by the FDA.

DIF: Cognitive Level: Knowledge REF: p. 833 OBJ: Theory #4


TOP: Black Box KEY: Nursing Process Step: Assessment
MSC: NCLEX: Physiological Integrity: Pharmacological Therapies

3. The nurse frequently refers to ―The ,‖ a medication list that names drugs
that are potentially harmful for use in elderly patients.

ANS:
STOPP Criteria List

The STOPP Criteria lists drugs that are potentially harmful to the older adult.

DIF: Cognitive Level: Knowledge REF: p. 834 OBJ: Theory #4


TOP: The STOPP Criteria List KEY: Nursing Process Step: Planning
MSC: NCLEX: Physiological Integrity: Pharmacological Therapies

ORDERING

1. A recently widowed 80-year-old man has been admitted to a long-term care facility for
complaints of anorexia, loss of energy, and loss of sleep. The nurse learns that the patient is
recently widowed. Prioritize the nursing actions that might be needed for this patient.
(Separate letters by a comma and a space as follows: A, B, C, D.)
a. Provide a quiet environment.
b. Involve him in some activity.
c. Be alert to suicidal tendencies.
d. Spend time with the patient at mealtime.

ANS:
C, B, D, A

The nurse must be alert to suicidal ideation or tendencies and assess for their effect on the
resident. Involving the resident in some activity can generate socialization and reduce
depressive thoughts. Spending time with the patient allows for assessment of his ideation as
well as giving evidence of his worth. Provision of a quiet environment is restful for all
residents, but is of low priority for a suicidal person.

DIF: Cognitive Level: Analysis REF: p. 838


OBJ: Clinical Practice #6 TOP: Suicide
KEY: Nursing Process Step: Implementation
MSC: NCLEX: Psychosocial Integrity: Psychosocial Adaptation

https://www.coursehero.com/file/30335509/c41rtf/

Downloaded by: SUCCEEDGRADES | abbieclin@gmail.com


Distribution
Downloaded by Dallen Mae D.ofKadir
this document is illegal
(kadir.dallenmae.d.bcsi@gmail.com)

You might also like